Vous êtes sur la page 1sur 1838

1

2013
H2 BIOLOGY
1 ANGLO - CHINESE 2013
2 ANDERSON 2013
3 CATHOLIC JC 2013
4 DUNMAN 2013
5 ST. ANDREW'S 2013
6 HWA CHONG INSTITUTION 2013
7 INNOVA JC 2013
8 JURONG JC 2013
9 MILLENNIA INSTITUTE 2013
10 MERIDIAN 2013
11 NATIONAL JC 2013
12 NANYANG 2013
13 PIONEER 2013
14 RAFFLES INSTITUTION 2013
15 RIVER VALLEY 2013
16 SERANGOON 2013
17 TAMPINES 2013
18 YISHUN 2013
2
3

ANGLO-CHINESE JUNIOR COLLEGE


Preliminary Examination 2013

BIOLOGY 9648/01

HIGHER 2 20 September 2013

Paper 1 Multiple Choice 1 hour 15 minutes

Additional Material: Multiple Choice Answer Sheet

READ THESE INSTRUCTIONS FIRST

Write in soft pencil.


Do not use staples, pencil clips, highlighters, glue or correction fluid.
Write your name, Centre number and index number on the Answer Sheet provided.

There are forty questions in this paper. Answer all questions. For each question there are four
possible answers, A, B, C and D.
Choose the one you consider correct and record your choice in soft pencil on the separate
answer sheet.

Read the instructions on the Answer Sheet very carefully.

Each correct answer will score one mark. A mark will not be deducted for a wrong answer.
Any rough working should be done in this booklet.

Calculators may be used.

This question paper consists of 23 printed pages.

ACJC 9648/01/Prelim 2013 [Turn Over


4
2

1 If the cell shown below has been cultured in an environment where the source of nitrogen
provided is labelled radioactively, in which of the following labelled regions would we
expect to find high radioactivity?
S

A S and T only
B T and U only
C S, T and U only
D S, T, U and V

2 Which row of molecules found in the cell surface membrane have their corresponding roles
listed?

Act as receptor Form hydrogen Allow passage Cell to cell


sites for bonds with water of ions adhesion and
hormones recognition
Glycoproteins and Phospholipids Glycoproteins Glycoproteins
A
glycolipids and glycoproteins and glycolipids and glycolipids
Glycoproteins Phospholipids Glycoproteins Glycoproteins
B
and proteins and glycoproteins and proteins and glycolipids
Glycoproteins Glycoproteins Glycoproteins Glycoproteins
C
and phospholipids and glycolipids and glycolipids and cholesterol
Glycoproteins Glycoproteins Glycoproteins Glycoproteins
D and glycolipids and cholesterol and and cholesterol
phospholipids

ACJC 9648/01/Prelim 2013 [Turn Over


5
3

3 During the production of fruit juice, enzymes are used to break down the components of
cell walls. Which carbohydrate will be produced by this hydrolysis?

A Sucrose
B Maltose
C - glucose
D - glucose

4 Two enzyme experiments were carried out. The first, experiment X, was carried out at a
constant temperature of 37oC. During the second experiment, Y, the temperature was
increased from 37oC to 80oC.

Which graph shows the results?

ACJC 9648/01/Prelim 2013 [Turn Over


6
4

5 A cell in the G1 phase has two homologous pairs of chromosomes. It then undergoes a
mitotic division, followed by meiosis. At the end of meiosis II, what is the total number of
chromosomes and gene loci found in all the daughter cells formed?

A 8 chromosomes and 4 times as many gene loci as the original parent cell
B 8 chromosomes and 8 times as many gene loci as the original parent cell
C 16 chromosomes and 4 times as many gene loci as the original parent cell
D 16 chromosomes and 8 times as many gene loci as the original parent cell

6 The graph represents the changes in the DNA content within a cell at different stages in the
cell cycle.

DNA
content in
a cell/
arbitrary
unit

Name the events occurring at P, Q and R, and identify the stage where meiosis is
occurring.

P Q R Meiosis occurring
at
A S phase Fertilisation Cytokinesis Y

B Fertilisation Interphase Cytokinesis Z

C S phase Prophase Telophase Y

D Fertilisation Metaphase Telophase Z

ACJC 9648/01/Prelim 2013 [Turn Over


7
5

7 The RNA triplet UAG acts as a stop codon terminating the synthesis of a polypeptide. The
diagram shows a template strand of DNA which codes for four amino acids.

Where would a mutation, introducing a thymine nucleotide, result in the termination of


translation?

3 T C C A C A C G A T G C 5

A B C D

8 Part of the amino acid sequences in normal and sickle cell haemoglobin are shown.

Normal haemoglobin sickle cell haemoglobin


-thr-pro- glu-glu -thr-pro-val-glu

Possible mRNA codons for these amino acids are

glutamine (glu) GAA GAG proline (pro) CCU CCC


threonine (thr) ACU ACC valine (val) GUA GUG

Which tRNA molecule is not involved in the formation of this part of the sickle cell
haemoglobin?

9 Haemoglobin is a globular protein consisting of four polypeptide chains two alpha chains
and two beta chains. In normal individuals, the gene for beta chain codes for glutamic acid
at the sixth triplet.

In individuals with sickle cell anaemia, this base triplet is changed and codes for valine.

Which aspect of the haemoglobin molecule does this mutation directly affect?

A the iron content


B the primary structure
C the quaternary structure
D the secondary structure

ACJC 9648/01/Prelim 2013 [Turn Over


8
6

10 Which of the following statements about the HIV is not true?

A The viral nucleotides contain ribose.


B HIV contains uracil, not thymine.
C gp 120 and gp 41 enable HIV to enter host by fusion.
D HIV makes the host cell produce reverse transcriptase.

11 How do viruses cause disease in animals?

I They inhibit normal host cell DNA, RNA or protein synthesis.

II They disrupt and inactivate the tumor suppressor genes of the host cell causing
uncontrolled cell division.

III They disrupt and inactivate the oncogenes of the host cell causing uncontrolled cell
division.

IV Their viral proteins and glycoproteins on the surface membrane of host cells cause
them to be recognized and destroyed by the bodys immune system.

V They deplete the host cell of cellular materials essential for metabolic functions.

A I, II and V
B I, II, III and V
C I, II, IV and V
D All of the above

12 A bacterial cell can transfer DNA during conjugation when it

A contains an F factor.
B has been infected by a bacteriophage.
C is virulent.
D integrates into host chromosome.

ACJC 9648/01/Prelim 2013 [Turn Over


9
7

13 When the lac operon for lactose metabolism is switched off, which of the following genes
would still be expressed?

I -galactosidase gene

II RNA polymerase gene

III CAP gene

IV Repressor gene

A I and II
B I and III
C II, III and IV
D All of the above

14 Which of the following statements correctly describes the control of transcription for genes
in a lac operon?

A The genes are transcribed as a single transcription unit, with each gene having its own
promoter.
B Methylation of DNA causes the DNA to be tightly packed, hence RNA polymerase will
not have access to the promoter region.
C The genes have control elements such as enhancers that enable activators to bind,
hence affecting the rate of transcription.
D The genes are found next to one another on the same chromosome, and repressor
proteins act as a form of transcriptional control.

15 Housekeeping genes are expressed all the time in all cells, whereas other non-essential
genes are expressed only when needed. Under certain conditions, gene expression of
selected genes can be increased.

Which of the following will increase gene expression in a eukaryotic cell?

A A chemical is added which will increase the activity of enzyme poly-A polymerase.
B Alternative splicing links different combinations of exons together in mRNA.
C Histone methylation results in the formation of heterochromatin.
D Activator proteins will bind to silencers to speed up the rate of transcription.

ACJC 9648/01/Prelim 2013 [Turn Over


10
8

16 Gene expression in eukaryotes can be regulated at the translational level.

Which combination of statements correctly describes eukaryotic translational control?

Condition Effect
Lack of translation initiation factor Inhibition of translation of selected
A
proteins mRNA
Presence of repressor proteins binding Prevents small ribosomal subunit
B
to 5-UTR of selected mRNA from binding
Presence of repressor proteins to distal
C mRNA is translationally-repressed
control elements
mRNA is degraded slowly by
D mRNA with a long poly-A tail
restriction endonuclease

17 A human papillomavirus (HPV) infection may cause cervical cancer in women. Which of the
following is not a reasonable explanation for cancer development caused by HPV infection?

A HPV integrates its viral genes into host cells genome.

B HPV may carry oncogenes that promote cell division.


C HPV may induce apoptosis of host cells.
D HPV promotes ubiquitination of p53 protein.

ACJC 9648/01/Prelim 2013 [Turn Over


11
9

18 After explaining about cell structure, DNA, chromosomes, telomeres and centromeres in
great detail during lectures and tutorials, a teacher decided to conduct a simple test to
determine the level of understanding in students. The following statements were observed in
the students scripts.

I Telomeres shorten after every cell division as a result of the end-replication


problem in eukaryotic cells.

II Telomeres are only found in eukaryotic chromosomes, since the prokaryotic


chromosome is circular and not linear.

III Centromeres are the protein molecules found at right angles to each other
in centrosomes, and they are absent in plant cells.

IV Telomerase is the enzyme that degrades telomeres, hence telomerase


gene is switched off in adult somatic cells.

Which of the students statements are correct?

A I and II only
B III and IV only
C I, II and IV only
D All of the above

ACJC 9648/01/Prelim 2013 [Turn Over


12
10

19 A genetic disorder causes degeneration of muscle tissue in humans. The diagram shows
the inheritance of such a genetic disorder in one family.

affected male

affected female

Which of the following statements can be concluded based on the diagram above?

A Individual 5 was affected with the genetic disorder as a result of genetic mutation
during his lifetime.
B Individual 10 should be an affected male too, since his brother (individual 11) was
affected with the genetic disorder.
C This genetic disorder is a sex-linked recessive trait.
D This genetic disorder is due to an autosomal dominant allele.

ACJC 9648/01/Prelim 2013 [Turn Over


13
11

20 A plant researcher tried to investigate a cross between two heterozygous Snapdragon


plants that produced red flowers. She predicted three possible phenotypic outcomes,
namely plants with white flowers, pink flowers and red flowers, with a phenotypic ratio of
4:3:9 respectively. When the cross was performed, she found 50 plants with white flowers
only, 41 plants with pink flowers, and 85 plants with red flowers. A chi-squared test was
performed, and the chi-squared value was calculated to be 4.74.

Probability, P
Degree of
freedom
0.10 0.05 0.02 0.01 0.001
1 2.71 3.84 5.41 6.64 10.83
2 4.61 5.99 7.82 9.21 13.82
3 6.25 7.82 9.84 11.35 16.27
4 7.78 9.49 11.67 13.28 18.47

Which of the following statements is correct?

A The degree of freedom is 3.


B The calculated chi-squared value is greater than the critical chi-squared value.
C There is a high probability that the difference between the observed and expected
values is due to chance.
D The probability that the difference between observed and expected values is due to
chance is less than 5%.

21 A PhD student performed a cross between a pure-breeding plant that bore white flowers,
and another pure-breeding plant that also bore white flowers. All the F1 plants obtained
bore purple flowers. Upon selfing the F1 plants, a F2 phenotypic ratio of 9 purple flowers: 7
white flowers was obtained. Gene A codes for the production of enzyme A, and gene B
codes for enzyme B. The diagram below shows the metabolic pathway involved.

Enzyme A Enzyme B
White pigment White pigment Purple pigment

Which of the following statements cannot be deduced from the above information?

A The parental pure-breeding plants must have the genotype AAbb and aaBB.
B Gene A is epistatic to gene B.
C Genes A and B are found on the same chromosome.
D All the F1 plants have the genotype AaBb.

ACJC 9648/01/Prelim 2013 [Turn Over


14
12

22 A genetic cross was performed between two heterozygous Drosophila fruit flies with normal
wings and red eyes. The allele for normal wings is dominant to the allele for vestigial wings,
and the allele for red eyes is dominant to the allele for white eyes.

What is the probability of obtaining offspring that are pure-breeding?

A 1 out of 16
B 3 out of 16
C 4 out of 16
D 9 out of 16

23 Which of the following statement(s) is/are correct?

I Polygenic inheritance is when a single phenotype is controlled by two or more sets


of alleles.

II The effect of environment has minimal influence in the expression of phenotype in


polygenic inheritance.

III Polygenic inheritance results in distribution of phenotypes that follow a normal


distribution curve.

A I only
B I and II only
C I and III only
D All of the above

24 Which of the following statements correctly compares oxidative phosphorylation and non-
cyclic photophosphorylation?

A Both types of phosphorylation produce ATP and oxygen as end products.


B Both types of phosphorylation produce ATP and the reduced form of a redox reagent.
C Oxidative phosphorylation is involved in the conversion of one form of chemical energy
to another while non-cyclic photophosphorylation is involved in converting light energy
to chemical energy.
D Water is an electron donor in non-cyclic photophosphorylation while it is an electron
acceptor in oxidative phosphorylation.

ACJC 9648/01/Prelim 2013 [Turn Over


15
13

25 14
C-labelled carbon dioxide was supplied to photosynthesising algae. The relative amounts
of three organic compounds were measured. The diagram shows the results.

Relative amount
of substance /
arbitrary units

Which of the following are correct explanations for the graph above?

I GP level falls as shown in graph 2 due to the absence of reduced NADP when light is
switched off.
II GP level rises as shown in graph 1 due to the absence of ATP when light is switched
off.
III Levels of RuBP and GP are constant during periods of light as they serve as
intermediates in the Calvin cycle.
IV RuBP level falls as carboxylation of RuBP is independent of light as shown in graph 3.
V Sucrose level falls as shown in graph 3 due to the absence of ATP and reduced
NADP.

A I, II and V only
B I, II and III only
C II, III and IV only
D III, IV and V only

ACJC 9648/01/Prelim 2013 [Turn Over


16
14

26 The diagram below represents a cell surface membrane.

Which biomolecules shown above are amphipathic?

A 1, 2 and 4 only
B 1, 2 and 3 only
C 2 and 3 only
D 3 and 4 only

27 The diagram below shows the cell signalling pathway involving a growth factor receptor.

From the given diagram, which step is involved in the role of signal amplification?

A Binding of L1 to Re1 and Re2


B Cross phosphorylation of Re1 and Re2
C Phosphorylation of ERK by MEK
D Dephosphorylation by protein phosphatase

ACJC 9648/01/Prelim 2013 [Turn Over


17
15

28 The diagram below shows the events of an action potential.

Which of the following statements are correct?

I. Events 1 to 5 occur in the synaptic knob during the transmission of an action potential
across the synapse.
II. Event 3, 4 and 5 are critical to ensure that the action potential travels unidirectionally
across a synapse.
III. Event 5 prevents a weak signal from initiating an action potential.
IV. Event 1 is necessary for an impulse to be transmited along an axon.

A I and II only
B I and III only
C II and IV only
D III and IV only

29 Which of the following statements does not correctly compare the neutral theory of
molecular evolution and natural selection?

A Neutral theory of molecular evolution accounts for most of the differences that we
observe at the phenotypic level as compared to natural selection.
B Neutral theory of molecular evolution accounts for most of the differences that we
observe at the genotypic level as compared to natural selection.
C The rate of change in the nucleotide sequence brought about by neutral theory of
molecular evolution occurs at a constant rate due to random chance events while the
rate of change brought about by natural selection can be fast or slow depending on the
strength of the selection pressure.
D Neutral theory of molecular evolution is largely responsible for the RFLP that we
observe between species as compared to natural selection.

ACJC 9648/01/Prelim 2013 [Turn Over


18
16

30 The figure below shows the alignment of amino acid residues of the alpha haemoglobin
chain of the cow and the sheep. There are a total of 142 amino acid residues in both the
alpha chain of the cow and sheep. Regions of polypeptide that form an alpha helix are
underlined while positions where amino acid residues differ between the cow and the
sheep alpha haemoglobin chain are marked with an arrow.

Amino Amino
acid acid
position position
Cow
Sheep

Cow
Sheep

Cow
Sheep

Which of the following statements can be concluded from the diagram shown above?

I. Differences between the amino acid residues between the 2 polypeptides are due to
neutral mutation brought about by the degeneracy of the genetic code.
II. Differences between the amino acid residues between the 2 polypeptides are due to
neutral mutation brought about by mutations in the introns.
III. Natural selection is one of the main driving forces that bring about the differences
between the 2 polypeptides.
IV. Differences between the amino acid sequences did not change the secondary
structure of protein.

A I and II only
B I and III only
C II and IV only
D III and IV only

ACJC 9648/01/Prelim 2013 [Turn Over


19
17

31 The diagram below shows the blood glucose concentration of two persons after eating a
meal.

How can the levels of blood glucose concentrations in Jack and Jill be best explained?

I. Jack and Jill both had insulin released into their bloodstream by their pancreas after
the meal.
II. The number of glucose transporter in the cell surface membrane of the muscle cells
increased in Jill but not in Jack.
III. Lowering of blood glucose level in Jack is brought about by utilisation of glucose in
respiration and excretion through urine.
IV. Jack must have type II diabetes while Jill is a healthy individual.

A I and II only
B II and III only
C I and IV only
D II, III and IV only

ACJC 9648/01/Prelim 2013 [Turn Over


20
18

32 The figure below shows the phylogeny of selected primates and their lice which live on
them. The dotted lines indicate the host of the respective lice.

Which of the following statement(s) can be concluded from diagram above?

I. The common ancestor of Pediculus humanus and Pediculus schaeffi underwent


allopatric speciation after the divergence of chimpanzee and humans.
II. Pithrus pubis and Pediculus humanus underwent sympatric speciation on the human
host.
III. Pithrus pubis and Pediculus humanus are reproductively isolated from each other due
to their ecological niches.
IV. Gorillas and humans once stayed in close physical proximity, allowing pubic lice to
change host.

A I only
B I and II only
C II and III only
D I, III and IV only

ACJC 9648/01/Prelim 2013 [Turn Over


21
19

33 The figure below shows how a metalmark moth mimics a jumping spider. It does so by
having hairs on both its hind wings and forewings.

Which of the following statements best explains the evolution of spider mimicry in
metalmark moth?

A Moths mutate their genes to grow hairs on their wings in order to escape predation.
B Moths develop hairs on their wings to mimic spiders.
C Moths with hairs on their wings leave behind a higher proportion of offspring than
moths without hairy wings.
D Moths with hairy wings experience a higher selection pressure than moth without hairy
wings.

34 During the process of polymerase chain reaction (PCR), the amount of DNA synthesised
can be traced using fluorescent probes and the measurements are shown in the following
plot. The process initially goes through an exponential phase, followed by a plateau phase
eventually.

Amount
of DNA/
arbitrary
units

Number of cycles
Which of the following statements is true?

A During the exponential phase, the number of DNA molecules synthesized after 15
cycles is 152.
B During the exponential phase, the temperature is always maintained at the optimum
temperature of 72oC hence there is rapid amplification.
C During the plateau phase, the reaction mixture is being depleted of ribonucleotides.
D During the plateau phase, Taq polymerase may be denatured.

ACJC 9648/01/Prelim 2013 [Turn Over


22
20

35 The insulin hormone is made up of two different polypeptide chains, A-chain and B-chain,
joined together by disulphide bonds. The human insulin gene codes for both polypeptide
chains. To produce the insulin hormone using E. coli, cDNA coding for each polypeptide
chain are inserted into separate plasmids and used to transform separate batches of
bacteria. The synthesised polypeptide chains are then mixed together to form the hormone.

Disulphide bond

Which one of the following reasons explains why the two chains have to be produced
separately?

A Bacterial RNA polymerases cannot recognise eukaryotic promoters.


B Bacterial cells are not able to carry out post-translational modification of insulin.
C Bacterial cells are unable to excise introns from eukaryotic RNA.
D Bacterial ribosomes can only translate polycistronic mRNA.

ACJC 9648/01/Prelim 2013 [Turn Over


23
21

36 A family with a history of a genetic disease is studied using restriction fragment length
polymorphism (RFLP) analysis. The mutation responsible for the disease allele produces a
RFLP which can be detected by Southern blotting using an appropriate probe. The
pedigree chart of the family is aligned with the autoradiogram obtained from Southern
blotting. (Shaded symbols in the pedigree chart indicate individuals affected by disease.)

Pedigree
chart

Autoradiogram

Based on the information given, which of the following can be deduced?

A The disease allele is dominant to the normal allele.


B The mutation creates a new restriction site in the affected gene.
C One of the parents in generation I is a carrier.
D The offspring in generation II is a carrier.

37 The Human Genome Project (HGP) has brought about great advancements in health and
medicine. Which of the following applications is not a result of the knowledge gained from
the HGP?

A Designing of new antibody-based medicines which target proteins coded for by


oncogenes.
B Determining whether an individual is a suitable candidate for working in a nuclear
power plant due to a genetic predisposition to cancer.
C Predicting whether an individual may suffer adverse side effects from taking a
particular medicine due to the individuals inability to break down the medicine.
D Testing the gender of a fetus by detecting the presence or absence of the fetal Y
chromosome in maternal blood.

ACJC 9648/01/Prelim 2013 [Turn Over


24
22

38 The following diagram shows how a stem cell can differentiate into different specialized cell
types.

Which of these statements is false with regards to the stem cells shown?

A The stem cells are multipotent.


B The stem cells can be found in both a fetus and an adult body.
C The stem cells can differentiate into the three germ layers in the adult body.
D The stem cells may be used in a bone marrow transplant to treat a patient with
leukemia, a form of blood cancer.

39 Cholera is a disease caused by the bacteria Vibrio cholerae and is characterised by


incessant diarrhoea and vomiting. Scientists have discovered a way to genetically modify
rice plants such that these plants can produce part of the cholera toxin, which acts as a
vaccine when consumed. Which statement is not a valid social or ethical implication of
such edible vaccines?

A The mass production of edible vaccines by rice plants lowers production costs
compared to traditional pharmaceutical vaccines.
B The genes coding for the cholera toxin may spread to wild plant species through cross-
pollination, causing the development of superweeds.
C Antibiotic resistance marker genes used in the process of cloning the cholera toxin
gene may lead to the development of antibiotic resistance in gut bacteria.
D The insertion of a foreign gene into the rice plant genome may lead to the production
of unknown toxic components.

ACJC 9648/01/Prelim 2013 [Turn Over


25
23

40 The following diagram shows one method where gene therapy can be used to treat cystic
fibrosis. Inactivated adeno-associated virus (AAV) is used to introduce the normal gene into
lung epithelial cells.

Which statement correctly describes the procedure shown?

A An ex vivo mode of therapy is used.


B Effects of the treatment by the inserted gene will not be permanent.
C Somatic stem cells are specifically targeted by the virus.
D Introduction of the normal gene into the host cell is less efficient using this method
compared to using a liposomal vector.

End of Paper

ACJC 9648/01/Prelim 2013 [Turn Over


26

ACJC Prelim 2013


H2 Biology Paper 1 (9648/01) Answers

Question Answer Question Answer

1 B 21 C

2 B 22 C

3 D 23 C

4 A 24 C

5 C 25 C

6 B 26 C

7 B 27 C

8 D 28 D

9 B 29 A

10 D 30 D

11 C 31 B

12 A 32 D

13 C 33 C

14 D 34 D

15 A 35 B

16 B 36 A

17 C 37 D

18 A 38 C

19 C 39 B

20 C 40 B
27

Subject
Name Class Class Index Number
2BI

ANGLO-CHINESE JUNIOR COLLEGE


Preliminary Examination 2013

BIOLOGY 9648/03

Applications Paper and Planning Question


3 Sept 2013
PAPER 3 2 hours

Additional Materials: Writing Paper

READ THESE INSTRUCTIONS FIRST

Write your name, subject class, form class and index number on all the work you hand in.
Write in dark blue or black pen on both sides of the paper.
You may use a soft pencil for any diagrams, graphs or rough working.
Do not use staples, paper clips, highlighters, glue or correction fluid.

Answer all questions.

At the end of the examination, fasten your work securely together.


The number of marks is given in brackets [ ] at the end of each question or part question.

FOR EXAMINERS USE

TOTAL
72

This document consists of 16 printed pages.

ACJC 9648/03 Prelim 2013


28
2 For
Examiners
Use

1. Embryonic stem (ES) cells are highly regarded in research because their pluripotency offers the
potential for a wide range of therapeutic and research applications. However, ethical concerns
have been raised with regards to the source of ES cells.
Scientists have come up with an alternative method of generating pluripotent cells, which is to
genetically reprogramme adult cells to an embryonic stem cell-like state. Genetic reprogramming
is carried out by using deactivated retroviruses to introduce the genes of four transcription factors
into adult cells from a patient (Fig. 1.1).The reprogrammed cells, called induced pluripotent stem
(iPS) cells, are specific to the patient from which the adult cells were taken.

Fig. 1.1

(a) (i) Describe what is meant by the term pluripotent.

It describes the potential of a cell to become/differentiate into any cell type in the

adult body but not those of the extra-embryonic membranes; @ 1m [1]

(ii) Suggest one reason why the use of iPS cells evades the ethical concerns regarding the
source of ES cells.

Ref. to iPS uses adult cells, whereas obtaining ES cells involves the destruction

of embryos; @ 1m [1]

ACJC 9648/03 Prelim 2013 [Turn over


29
3 For
Examiners
Use

(iii) Give three reasons why deactivated retroviruses are used for genetic reprogramming of
the adult cells rather than active retroviruses or liposomes.

*1. deactivated because they will not cause disease/will not become virulent;
2. retoviruses allow for integration of genes into host genome while liposomes
do not;
3. retroviruses are more efficient at delivering genes into the host cell than
liposomes because they target specific cells;

*required for full marks


[3]

(iv) Suggest how the expression of such a small number of transcription factors in adult cells
could genetically reprogramme these adult cells to an embryonic stem cell-like state.

1. Each transcription factor can activate the transcription of multiple genes;


2. Each gene in turn could code for a transcription factor which activates other
genes;

3. The genes switched on are those expressed in pluripotent/ES cells;


4. e.g. telomerase gene/ genes which promote cell division/ genes which cause
the cell to revert to undifferentiated state;
5. resulting in the synthesis of proteins which are found in ES cells;
6. The transcription factor could also inhibit gene expression;
7. e.g. genes which result in specialisation (are repressed);
@ 1m, max 3 [3]

ACJC 9648/03 Prelim 2013 [Turn over


30
4 For
Examiners
Use

Patient-specific iPS cells can be induced to differentiate into the affected diseased tissue as this
will allow the patient's disease to be modelled in vitro. Using this diseased tissue, potential drugs
to treat the disease can be screened for, aiding in the discovery of new drugs.

Scientists have managed to obtain iPS cells from cystic fibrosis (CF) patients with a mutation at
amino acid position 508 (F508). They have caused these iPS cells to differentiate into diseased
human lung tissue, which is now being used to screen for potential drugs to treat cystic fibrosis
caused by the F508 mutation (Fig. 1.2).

Fig. 1.2

Previously, a cell line comprising rat cells expressing recombinant human mutant CFTR protein
was used for screening purposes. This cell line was successfully used to identify a drug to treat
cystic fibrosis.

(b) (i) Explain the effect of the F508 mutation on the function of the CFTR protein.

1. Mutation results in the deletion of phenylalanine from CFTR protein;

2. resulting in change in tertiary/overall 3D shape change in structure;


3. The CFTR protein is unable to transport chloride ions across the
membrane/out of the cell;

[3]

ACJC 9648/03 Prelim 2013 [Turn over


31
5 For
Examiners
Use

(ii) Suggest and explain one advantage of using a drug to treat CF rather than to use
liposomes to deliver a functional copy of a CFTR gene.
1. easier to regulate dosage of drug/avoid using high concentrations of
liposomes which could be toxic;
2. using the drug avoids the problem that the CFTR gene may not be properly
expressed (to form the CFTR protein); [1]

(iii) Suggest and explain one advantage of using diseased lung tissue derived from human
iPS cells to screen for CF drugs rather than the use of rat cells expressing recombinant
human CFTR.
1. ref. to iPS cells are (human cells and are) more representative of the actual
diseased cell than rat cells e.g. rat cells may have different biochemical
2. ref. to iPS cells are patient-specific and could be used to screen for drugs
specific to that patient; [1]

[Total: 13]

ACJC 9648/03 Prelim 2013 [Turn over


32
6 For
Examiners
Use

2. Cysteine proteinase (CP) is an enzyme that is coded by the mir-1 gene, which is found in plants
that are naturally resistant to attacks by Lepidopteran larvae. Lepidopteran larvae feeding on the
Black Mexican Sweet (BMS) maize leaf tissue cause crop losses. In an attempt to increase the
yield of BMS maize, plant biologists have used genetic technology to transform the wild-type BMS
maize callus with mir-1 gene. The growth of larvae feeding on transformed and non-transformed
BMS maize calli was studied for seven days, and the experiment was repeated. The results are
shown in Fig. 2.1.

Fig. 2.1

(a) (i) Suggest one delivery method that could be used to introduce the mir-1 gene into BMS
maize callus.

[1]

(ii) With reference to Fig. 2.1, describe and explain the effectiveness of genetically engineered
BMS maize with mir-1 gene on larvae growth.

[3]

ACJC 9648/03 Prelim 2013 [Turn over


33
7 For
Examiners
Use

(b) An in vitro experimental study was performed to investigate the effects of transformed BMS
plant cells expressing both cysteine proteinase (CP) and Bacillus thuringiensis (Bt) toxin on
Lepidopteran larvae. Table 2.1 shows the relative growth rate and percentage mortality of
Lepidopteran larvae fed with maize cell lines containing CP, Bt-toxin and a combination of CP
and Bt-toxin.

Table 2.1

CP Bt-toxin CP+ Bt-toxin


Relative Growth Rate/ Arbitrary Units 0.333 0.568 0.220
Percentage Mortality/% 38.0 4.0 50.0

With reference to Table 2.1, describe the effects of maize plant cells expressing CP together
with Bt-toxin on the percentage mortality of Lepidopteran larvae.

[2]

(c) Both CP and Bt-toxin target the mid-gut lumen. Fig. 2.2 shows a cross section of mid-gut. CP is
known to damage the peritrophic protein matrix of mid-gut lumen in the larvae, a structure that
protects the mid-gut from microbial infection.

The mechanism of damage by Bt-toxin is different compared to that of CP. Bt-toxin is


converted to its active form in the alkaline mid-gut environment, which then binds to the
specific mid-gut receptors on the brush border membrane of mid-gut cells with very high
affinity, forming lytic pores that result in cell death.

Fig. 2.2

ACJC 9648/03 Prelim 2013 [Turn over


34
8 For
Examiners
Use

(i) With reference to Table 2.1, Fig. 2.2 and the information given, suggest how CP may
enhance the effect of Bt-toxin on Lepidopteran larvae.

[3]

(ii) Over many generations, laboratory populations of Lepidopteran larvae have evolved
resistance against Bt-toxins. Suggest and explain how this resistance is developed.

[2]

[Total: 11]

ACJC 9648/03 Prelim 2013 [Turn over


35
9 For
Examiners
Use

3. Fig. 3.1 below shows a schematic diagram of targeted gene replacement in a mammalian cell
involving the gene Ig M. Targeted Ig M gene replacement is a method of replacing Ig M mutated
allele (Fig. 3.1a) in the genome in a cell with wildtype allele (Fig. 3.1b) via crossing over, giving
rise to the recombinant DNA (Fig. 3.1c). The wildtype allele is carried by a linearised plasmid (Fig.
3.1b) and introduced into the cell.

As shown in Fig. 3.1b, the linearised plasmid contains an antibiotic resistance gene (Neo gene),
wildtype alllele with the exons represented by the black boxes, as well as several restrictions sites
in between the exons. In this technique, the enhancer of the Neo gene on the linearised plasmid
was removed prior to the therapy.

Ig M mutated alleleNhe1
Ava1
Sca1

Afl1

Enhancer Genomic Fig. 3.1 a


fragment
F primer
Region of crossing over
Hpa1
Ecor1

Aat1
Dra1
Kpn1

Neo gene
Linearized Fig. 3.1 b
Ig M wildtype allele plasmid
R primer
Hpa1
Aat1
Ava1

Dra1
Sca1

Enhancer Neo gene


Recombinant
Fig. 3.1 c
DNA

Fig. 3.1

Legend
Exon
Restriction sites on mutated allele Ava1, Sca1, Afl1, Nhe1
Restriction sites on wildtype allele Kpn1, Ecor1, Dra1, Aat1
Naturally occurring restriction site on Hpa1
linearised plasmid

(a) (i) Explain the significance of restriction enzyme in bacteria.

[1]

(ii) Although the mutated and wildtype alleles are homologous regions, different restriction
sites are present in them. Explain the presence of these different restriction sites.

[2]
ACJC 9648/03 Prelim 2013 [Turn over
36
10 For
Examiners
Use

(iii) With reference to Fig. 3.1, explain how the Neo gene allows the transformed cells with
recombinant DNA to be distinguished from the transformed cells which did not undergo
recombination in their DNA.

[4]

(iv) With reference to Fig. 3.1a and Fig. 3.1b, briefly describe another method how Hpa1 can
be used to verify the identity of cells with recombinant DNA.

[4]

(v) The wildtype IgM allele was isolated from a genomic DNA library and not a cDNA library
before being introduced into the linearised plasmid. With reference to Fig. 3.1, state one
evidence which suggests this.

[1]

ACJC 9648/03 Prelim 2013 [Turn over


37
11 For
Examiners
Use

(b) In the gene replacement therapy shown in Fig. 3.1, the site of DNA breakage occurred
between restriction sites Sca1 and Afl1 as shown in Fig. 3.1a. In another case of therapy, the
site of DNA breakage along the mutated allele was unknown. To determine the exact site of
crossing over between the genomic DNA and the linearised plasmid, RFLP analysis was
carried out on the recombinant DNA. The recombinant DNA was isolated from the
transformed cell and amplified using PCR via the forward and reverse primers as shown in Fig.
3.1a and Fig. 3.1b. The amplified DNA products were separated into eight tubes and digested
using eight restriction enzymes respectively: Ava1, Sca1, Afl1, Nhe1, Kpn1, Ecor1, Dra1, Aat1.
The products of digestions were separated using gel electrophoresis and the results are
shown in Fig. 3.2.

Ava1 Sca1 Afl1 Nhe1 Kpn1 Ecor1 Dra1 Aat1

Fig. 3.2

(i) With reference to Fig. 3.1 and 3.2, describe and explain the band pattern of the restriction
digest for the recombinant cell.

[3]

(ii) Hence identify the site of DNA breakage where crossing over took place in the mutated
allele.

[1]

[Total: 16]

ACJC 9648/03 Prelim 2013 [Turn over


38
12 For
Examiners
Use

Planning Question
4. Electroporation is a method which is routinely used to introduce foreign DNA into E. coli cells. This
involves subjecting a mixture of bacteria and plasmid to a brief electric pulse. You are required to
plan, but not carry out, an investigation into the effect of voltage on the efficiency of transformation
by electroporation.
In this investigation, the recombinant plasmid used to transform the E. coli cells is made up of two
components: the plasmid vector and the gene of interest. pGFP is a plasmid with two marker
genes, GFP and AmpR, and a number of restriction sites throughout the plasmid as shown in Fig.
4.1 below. GFP is a marker gene coding for the green fluorescent protein. Expression of GFP in
bacteria will cause bacteria to fluoresce under UV light. The gene of interest is the ras proto-
oncogene, which has been inserted into the plasmid using the restriction enzyme BmtI.

Fig. 4.1
An electroporator is a machine used to carry out electroporation. It is possible to vary the voltage
and length of the electrical pulse with this machine. The range of voltage used is typically 1400
2000 V and the length of electrical impulse between 4-6 ms. A voltage which is too high will cause
irreversible changes in membrane permeability. The mixture of bacteria and recombinant plasmid,
which must be kept on ice prior to electroporation, is put into a cuvette (Fig. 4.2) which is then
placed into a chamber in the electroporator (Fig. 4.3).
After electroporation, the mixture of bacteria must be quickly resuspended in SOC broth (a type of
growth medium) and incubated at 37C for 1 hour for the surviving cells to recover. This is then
followed by plating the bacteria on agar and incubation at 37C for 16-24 hours. The plates can
then be examined to calculate transformation efficiency.

Fig. 4.2
ACJC 9648/03 Prelim 2013 [Turn over
39
13 For
Examiners
Use

Fig. 4.3

Transformation efficiency can be calculated using the following equation: -

Transformation efficiency / cfu g1 = Total number of colonies on agar plate

[cfu: colony forming unit] Amount of DNA plated (in g)

Your planning must be based on the assumption that you have been provided with the following
equipment and materials which you must use:
Microcentrifuge tubes of 100 l E. coli cells in electroporation buffer
Microcentrifuge tubes of 1 l recombinant DNA (concentration of 0.1 g / l)
Microcentrifuge tubes of 1 l sterile water
Ice bucket with ice
Electroporator
Sterile cuvettes for electroporation (max. volume of 150l)
Agar plates with ampicillin
Sterile spreader for spreading bacterial culture on agar plate
SOC broth
Incubator
UV transilluminator (a device which emits UV light)
UV shields which provide protection from UV light
A variety of micropipettes, sterile micropipette tips and sterile 2 ml microcentrifuge tubes

Your plan should have a clear and helpful structure to include


an explanation of the theory to support your practical procedure
a description of the method used, including the scientific reasoning behind the method, a
control experiment and any recommended safety measures
an explanation of the dependent and independent variables involved
relevant, clearly labelled diagram/s showing selection of the recombinant bacteria on plates
how you will record your results and ensure that they are as accurate and reliable as possible
proposed layout of results tables and graphs with clear headings and labels
the correct use of technical and scientific terms

[Total: 12]

ACJC 9648/03 Prelim 2013 [Turn over


40
14 For
Examiners
Use

ACJC 9648/03 Prelim 2013 [Turn over


41
15 For
Examiners
Use

ACJC 9648/03 Prelim 2013 [Turn over


42
16 For
Examiners
Use

Free-response question

Write your answers to this question on the separate paper provided.

Your answer:

should be illustrated by large, clearly labelled diagrams, where appropriate.


must be in continuous prose, where appropriate.
must be set out in sections (a), (b) etc., as indicated in the question.

5. (a) Describe and explain how bacteria can be used to mass produce human insulin without the
use of any DNA libraries. [7]

(b) Discuss the advantages and the limitations of the Polymerase Chain Reaction. [5]

(c) Discuss the implications of the Human Genome Project, including the benefits and difficult
ethical concerns for humans. [8]

[Total: 20]

ACJC 9648/03 Prelim 2013 [Turn over


43

1 (a) Fig 1.1 shows the protein structure of the enzyme cellulase, which is found in animals
feeding on grass such that they can digest the cellulose found in grass.

Fig. 1.1

(i) Name structure A and describe how it is formed.


1. -pleated sheet;

2. is made up of different sections of the polypeptide chain that run alongside


each other/ are antiparallel;
3. Adjacent strands are held together by hydrogen bonds formed between CO
and NH groups (of the polypeptide backbone) to form the sheet; @1m [3]

(ii) State two differences between the structures of cellulase and its substrate.
1. Made up of amino acids vs -glucose;

2. Peptide bond vs (1-4) glycosidic bond between monomers;

3. Ionic bonds, hydrophobic interactions and disulphide bonds(any 2 of 3) found


in the structure of cellulase but not its substrate;

4. No rotation of amino acids vs alternate -glucose rotated 180;

5. No crosslinks vs crosslinks between cellulose;

6. Globular vs fibrous; @ 1m, max 2


[2]

ACJC 9648/02/Prelim 2012 (Mark Scheme Post-Standardization v1) [Turn over


44
2 For
examiners
(iii) Briefly describe the mode of action of cellulase. use

1. Enzyme lowers activation energy;

2. Ref to acting as orientation surfaces/ ref to strain effect/ proximity effect/


microenvironment effect/ acid-base catalysis;

3. The (3D conformation of) substrate (e.g., cellulose) is complementary to the


active site of the enzyme, thus forming enzyme-substrate complexes;

E
4. With ref. to change in 3D conformation of enzyme for a better fit/ tighter
binding between substrate and enzyme;

5. Resulting in cleaving of glycosidic bond between -glucose;


[4]
@ 1m, max 4

(b) Collagen is a protein found extensively in connective tissue, bones, cartilage, dermis (skin),
blood vessels and teeth. It is a fibrous protein, and has a very distinct structure and function
from cellulase.

Describe two differences between the primary structures of collagen and cellulase.
1. Amino acid sequence in collagen is highly regular with repeating tripeptide
sequence while amino acid sequence in cellulase rarely shows regularity;
2. Amino acid sequence in collagen may vary between two samples of collagen;
while amino acid sequence in cellulase is highly specific;
3. Length of polypeptide chain varies in collagen while length of polypeptide chain For
seldom varies in cellulase; Examiner's
Use
4. More amino acids make up collagen while less amino acids make up cellulase; For
5. Modified amino acids such as hydroxylysine and hydroxyproline are found in Examiner's
Use
collagen but not cellulase;

@ 1 m per comparison, max 2 [2]

[Total: 11]

2 Matrix metalloproteinase-13 (MMP-13) gene is found on chromosome 11 in humans. It codes for


MMP-13, an enzyme that is involved in the breakdown of extracellular matrix proteins such as
collagen, in normal physiological processes. During inflammation, a protein, AP1, is activated
and binds to a conserved DNA sequence about 20kb (kilobases) upstream of the MMP-13 gene
(Fig. 2.1) to increase (up-regulate) its expression. Histone modification is found to occur at the
stretch of DNA from the conserved DNA region to the transcribed region of the MMP-13 gene.
The AP1-bound conserved DNA region is also found to come into close proximity to the MMP-13
transcription start site.

Conserved DNA MMP-13 Gene

AP1 binding site

Fig. 2.1

ACJC 9648/02/Prelim 2012 (Mark Scheme Post-Standardization v1) [Turn over


45
3 For
examiners
(a) A gene is a region along a DNA molecule that codes for a specific sequence of amino use
acids in a polypeptide chain. Describe the structure of DNA.
1. Polymer of deoxyribonucleotides/composed of 4 types of deoxyribonucleotide
containing base thymine, cytosine, adenine and guanine;
2. two antiparallel polynucleotide strands/double helix;
3. ref to complementary base-pairing by hydrogen bonds / 3 H bonds bet G-C and
2 H bonds bet A-T/purine with pyrimidine/ref to 1:1 ratio;
4. width betw sugar-phosphate backbone constant and equals to the width of a

E
purine and a pyrimidine/2 nm wide;
5. One complete turn has 10 base pairs with length of 3.4 nm;
6. Phosphodiester bonds between adjacent deoxyribonucleotides; [3]
Any 3 @ 1m each

(b) Describe the role of the AP-1 protein in increasing the expression of the MMP-13 gene.
1. AP1 act as the activator (R! repressor) and binds to the conserved DNA region,
the enhancer (R! silencer);
2. DNA bends to bring the AP1 protein and conserved DNA region to close
proximity to the promoter/transcription initiation complex/MMP-13 gene;
3. Ref. to mediator proteins and transcription factors binding at promoter/AP-1
protein;
4. Increases the affinity of RNA polymerase to the promoter of the MMP13
gene/ enhances formation of the transcription initiation complex; @1m [3]

(c) Describe one type of histone modification that can occur in the presence of the activated
AP1 protein.
For
Examiner's
1. *Acetylation of histones; Use
2. neutralises the positively-charged R groups on histones / decreases the affinity For
Examiner's
of the histone tails for DNA; Use
OR
4. *De-methylation of histones;
5. proteins that bind to methylated histones removed;
@1m [2]

(d) MMP-13 is secreted as an inactive form. It is activated once the pro-domain is cleaved,
leaving an active enzyme that participates in collagen degradation. State this level at
which the expression of MMP-13 gene is controlled.

1. post-translational; [1]

(e) MMP-13 is involved in the degradation of the basement membrane where cells are
anchored. Tumour cells have elevated MMP-13 production and secretion. Suggest how
the elevated secretion of MMP-13 contributes to the malignant properties of the tumour
cells.

1. Metastasis/metastasizing;

2. Tumour cells freed/no longer bound/not anchored;

3. Released into bloodstream;

4. Resulting in formation of other/secondary tumours; @ 1 m, max 3 [3]


[Total: 12]

ACJC 9648/02/Prelim 2012 (Mark Scheme Post-Standardization v1) [Turn over


46
4 For
examiners
3. Fig. 3.1 below is a diagram of a dividing animal cell. use

E
Fig 3.1
(a) Name the structures labelled A and B.

A: Centrioles; R: Asters
For
B: Nucleolus; @1/2 m [1] Examiner's
Use
For
Examiner's
(b) State one visible feature in Fig 3.1 that enables you to identify that the cell is at Use

(i) Prophase of (ii) Meiosis II.

(i) Presence of nuclear envelope/separating centrioles/absence of


spindle/formation of spindle fibres/

Condensed chromosomes but not aligned at equatorial plane; any 1

(ii) Absence of homologous pairs of chromosomes/bivalents/1 of each type of [2]


chromosome/presence of odd number of chromosomes; @1m

(c) Discuss the significance of meiosis II.

1. Separation of sister chromatids of each chromosome (into 2 daughter nuclei);

2. Results in haploid cells/each with only one of each type of


chromosomes/halving the number of chromosomes;

3. Diploid number of organism will be restored/prevents doubling of


chromosomes upon fertilisation;
4. Daughter cells are genetically different (due to crossing over and
independent Segregation); @1 m [3]

ACJC 9648/02/Prelim 2012 (Mark Scheme Post-Standardization v1) [Turn over


47
5 For
examiners
(d) Describe the event(s) that could possibly take place following prophase of Meiosis II, use
resulting in the formation of a zygotic cell with diploid number of 7 upon fertilisation.

1. * Non-disjunction;

2. One pair of sister chromatids fail to separate at anaphase II;

3. Due to failure of attachment to spindle fiber/ of centromere dividing;

E
4. One daughter cell with (n+1/4) and the other (n-1/2) chromosomes/OWTTE;

5. Fusion of (n+1/4) gamete with normal gamete (3) to form 2n = 7 zygote; [3]
@ 1 m, * point essential to get full mark

[Total: 9]

4 Fig. 4.1 shows the components of the lac operon and its regulatory gene which is some
distance away.
lacO lacZ lacY IacA
lacI
Promoter (Operator)

mRNA mRNA

For
Examiner's
repressor -galactosidase lactose transacetylase Use
protein For
permease Examiner's
Use
Fig. 4.1
(a) (i) Describe what will happen to the repressor protein if lactose is present.

1. Allolactose/lactose binds to ( allosteric site of) repressor protein and


2. Alters configuration/3D conformation/tertiary structure of repressor

3. repressor detaches / cannot bind to operator @1m [2]

lacI+ lacO- lacZ+ lacY- lacA+


- - + - +
(ii) Explain how a bacterial cell with a diploid genotype lacI lacO lacZ lacY lacA
will respond in the presence of lactose (+: wild type allele; - : loss of function allele)

1. (lacI+ is dominant over lacI-) hence repressor protein is produced;

2. both copies of operator are defective;

3. (Normal) repressor cannot bind to operator

4. structural genes will be constitutively/continuously expressed

5. LacY gene for lactose permase not expressed/ lack of lactose


permease/non- functional lactose permease

6. Functional -galactosidase and transcetylase formed;


7. lactose not transported into cell (and metabolised); @1m [5]

ACJC 9648/02/Prelim 2012 (Mark Scheme Post-Standardization v1) [Turn over


48
6 For
examiners
use
(b) 5-capping and 3-polyadenylation are two kinds of post-transcriptional modifications that are
present in eukaryotes but absent in prokaryotes. Describe another post-transcriptional
modification necessary to produce mature mRNA in eukaryotes.

1. Spliceosome recognises/binds splice site;


2. Bring about the removal of introns/ spliced exons together to form
mature mRNA @1m [2]

(c) Human DNA has been analysed and details of certain genes are shown in Table 4.1.

Gene
Insulin
Albumin
Phenyalanine
Gene size / kb
1.7
25.0
90.0
Table 4.1

mRNA size / kb
0.4
2.1
2.4
Number of introns
2
14
12
E
hydroxylase
Dystrophin 2000.0 17.0 50

(i) Calculate the average size of the introns for the albumin gene (show your workings.)

25.0 2.1
= 1.6 kb;
14 For
Examiner's
Use
[1] For
Examiner's
Use
(ii) With reference to Table 4.1, describe the relationship(s) between the gene size and the
number of introns.

1. Generally bigger the gene size, the larger number of introns

2. Correct ref to data comparison of gene size and number of introns;

3. Ref to exceptions in trend as shown by phenylalanine hydroxylase / [3]


albumin gene @1m

[Total: 13]

ACJC 9648/02/Prelim 2012 (Mark Scheme Post-Standardization v1) [Turn over


49
7 For
examiners
5 CoffinLowry Syndrome (CLS) is a sex-linked dominant genetic disease that causes severe use
mental disability in humans. This disorder is caused by mutations in gene R. The normal
protein coded for by this gene is required for the formation of long-term memory, and the
survival of nerve cells. Lack of this protein hence affects mental development. Patients with
CLS experience brief episodes of collapse when they are excited or startled by a loud noise.

(a) A phenotypically normal woman married a CLS sufferer. Using a genetic diagram,
illustrate this cross and determine the probability of the couple having a male child

E
suffering from CLS. [3]

Penalise once for symbol other than XR and Xr used;

Parental phenotype: Normal female x CLS male 1. 1m for both genotype


and gametes correct
Parental genotype: XrXr x XRY

Gametes Xr XR Y
2. 1m for both F1
genotype and F1
phenotypic ratio correct
F1 genotype: XRXr XrY
F1 phenotypic ratio: 1 CLS female : 1 normal male
0 3. 1m for correct answer
Probability of a male child suffering from CLS: ___________________

For
The Canine Genome Sequencing Project was successfully completed in year 2005. Examiner's
During the project, researchers needed to investigate two particular genes in dogs. The Use
For
genes were the amino-methyl-transferase (AMT) gene (which codes for AMT enzyme to Examiner's
degrade excess glycine), and the T-cell leukemia translocation-associated (TCTA) gene Use
(which codes for TCTA protein involved in osteoclastogenesis, the formation of osteoclasts
for the breakdown of bone tissue).

(b) To increase sample size for statistical reliability, scientists selected several
heterozygous female poodles for normal AMT phenotype and normal TCTA phenotype,
and conducted test crosses on them. The following results were observed in the
offspring.

Normal AMT, normal TCTA dogs 96


Normal AMT, mutant TCTA dogs 18
Mutant AMT, normal TCTA dogs 20
Mutant AMT, mutant TCTA dogs 94

ACJC 9648/02/Prelim 2012 (Mark Scheme Post-Standardization v1) [Turn over


50
8 For
examiners
(i) Using A and a to represent the alleles for the AMT gene, and T and t to represent use
the alleles for the TCTA gene, illustrate the above cross with the use of a genetic
diagram. [4]

R: Linkage not indicated by brackets

Parental phenotype: normal AMT, normal TCTA x mutant AMT, mutant TCTA

E
1. 1m for P genotype, with
Parental genotype: (AT) (at) x (at) (at)
brackets indicating linked
genes;

Gametes (AT) (at) (At) (aT) (at)


2. 1m for circled
gametes;

F1 genotype:

(AT) (at) (at) (at) (At) (at) (aT) (at)

F1 phenotype: 3. 1m for F1 genotype;

Normal AMT, Mutant AMT, Normal AMT, Mutant AMT,


normal TCTA mutant TCTA mutant TCTA normal TCTA
4. 1m for corresponding
phenotype; For
Examiner's
Use
For
Examiner's
Use
(ii) Calculate the recombinant frequency between the AMT gene and TCTA gene. [1]

C.O.V. = [20+18] /228 x 100 = 16.67%

R: fraction

@1 mark

(iii) The recombinant frequency between the AMT gene and the canine DAG1 gene
was found to be 8.4%, and the recombinant frequency between the TCTA gene
and DAG1 gene was found to be 8.3%. On the diagram of a chromosome below,
indicate the relative positions of the AMT, TCTA and DAG1 genes. [1]

Ans: AMT gene, DAG1 gene, TCTA gene

OR TCTA gene, DAG1 gene, AMT gene

[Total: 9 marks]

ACJC 9648/02/Prelim 2012 (Mark Scheme Post-Standardization v1) [Turn over


51
9 For
examiners
6 Other than activating adenylyl cyclase, activated G-protein-coupled-receptors (GPCRs) can use
activate another signalling pathway. Fig. 6.1 below shows how the activation of GPCR
eventually leads to the activation of protein kinase C.

E
Fig. 6.1

(a) Describe how G protein is activated upon binding of the signal molecule to GPCR.

1. GPCR changes conformation/3D shape; For


Examiner's
Use
2. Binds to G protein, and causes the displacement of GDP with GTP; For
Examiner's
Use
@ 1m [2]

(b) With reference to Fig. 6.1, describe how the activated phospholipase C- results in the
activation of protein kinase C.

1. Activated phospholipase C- cleaves PIP2 into IP3 and DAG;


2. IP3 binds causes the IP3-gated Ca2+ channel to open, allowing efflux of Ca2+ from
SER (into cytoplasm)/ increasing [Ca2+] in the cytoplasm;

3. Ca2+ and DAG bind to protein kinase C (and activate it);

@ 1m

[3]

(c) Explain what is meant by the term phosphorylation cascade.


1. Phosphorylation cascade refers to a series of protein kinases which
phosphorylate other proteins/ in sequence/series;

2. Ref. to signal amplification; @ 1m [2]

ACJC 9648/02/Prelim 2012 (Mark Scheme Post-Standardization v1) [Turn over


52
10 For
examiners
(d) Other than through the phosphorylation cascade, explain how the signal is amplified in the use
pathway shown in Fig. 6.1.
1. amplification occurs when (one) signal molecule/activated GPCR activates many
G proteins;
2. when (one) activated phospholipase C-converts many molecules of PIP2 into IP3
and DAG;
3. and when (one) IP3 molecule results in many Ca2+ ions entering the cytoplasm/
activation of many protein kinase C; @ 1m [2]

E
(e) Besides storing Ca2+, describe two other functions of the smooth endoplasmic reticulum.

1. synthesis of lipids/steroids/triglycerides/phospholipids;

2. detoxification of drugs/poisons (in liver cells);

3. carbohydrate metabolism/breakdown of glycogen (in liver cells); any 2 @ 1m [2]

Some types of protein kinase C do not require presence of Ca2+ for activation. Protein kinase C
is known to be involved in cell signalling pathways which promote cell division.

(f) Phorbol esters are plant compounds which mimic the action of DAG. Phorbol esters are
known to promote the formation of tumours. With reference to Fig. 6.1, suggest and explain
how phorbol esters can promote tumour formation.

1. Phorbol esters bind to/activate protein kinase C; For


Examiner's
2. (activated) protein kinase C activates proteins/genes/cell signal pathways Use
which promote cell division / prevent apoptosis; @ 1m [2] For
Examiner's
Use
[Total: 13]

7 The mechanisms of speciation in ferns have been studied in temperate and tropical habitats.
One group of three species from the genus Polypodium lives in rocky areas in temperate
forests in North America. Members of this group have similar morphology (form and
structure). Another group of four species from the genus Pleopeltis live at different altitudes
in tropical mountains in Mexico and Central America. Members of this group are
morphologically distinct.
Data from the different species within each group was compared in order to study the
mechanisms of speciation.
Genetic identity was determined by comparing the similarities of certain proteins and genes
in each species. Values between 0 and 1 were assigned to pairs of species to indicate the
degree of similarity in genetic identity. A value of 1 would mean that all the genetic factors
studied were identical between the species being compared.

Fig. 7.1 shows the approximate distribution in North America of the first group comprising
three species of Polypodium (Po.) and a summary of genetic identity.

ACJC 9648/02/Prelim 2012 (Mark Scheme Post-Standardization v1) [Turn over


53
11 For
examiners
use

IRS1

p110
E
Fig. 7.1

Fig. 7.2 shows the approximate distribution in Central America and Mexico of the second group
comprising four species of Pleopeltis (Pl.) and a summary of genetic identity.
For
Examiner's
Use
For
Examiner's
Use

Fig. 7.2

(a) State, with reasons which group, Polypodium or Pleopeltis, has most probably been
genetically isolated for a longer period of time.
1. Polypodium;

2. it has lower genetic identity values of 0.338 0.608/the highest genetic value of

0.608 in Polypodium is still lower than those of Pleopeltis/ the lowest genetic

value of Pleopeltis is still higher than those of Polypodium;

ACJC 9648/02/Prelim 2012 (Mark Scheme Post-Standardization v1) [Turn over


54
12 For
examiners
use
3. takes time to accumulate mutations / genetic changes;

4. Isolation due to geographical barrier limiting gene flow;

@ 1m [4]

E
(b) Describe the distributions of the two groups of ferns within their respective geographical
locations.
1. The Polypodium species are (completely) found in different parts of the

continent/geographically separated;

2. The Pleopeltis species are much closer together / physically overlapping / share

same habitats; @ 1m
[2]

(c) Suggest how the process of speciation could have occurred in Polypodium.
1. Allopatric speciation;

2. The three species of Polypodium are geographically isolated (on different parts of
the continent);
For
3. Inherited variation is present in the (original) population of Polypodium; Examiner's
Use
4. Different habitats exert different selection pressures; For
Examiner's
Use
5. Only those with favourable traits are selected for and survive to pass on their
alleles to their offspring;

6. No interbreeding/gene flow between the different populations;


7. Separation of gene pools of isolated populations/change in allele frequencies
over time; @ 1m, max 5 [5]

(d) According to neutralists, when one compares the genomes of existing species, the vast
majority of molecular differences, which arises from gene mutations, are selectively neutral.
Describe the effect of changing the selection pressure on populations of species which have
these neutral mutations.
1. These mutations do not affect fitness/reproductive success;

2. These organisms do not come under selection pressure/changing selection


pressure has no effect;
3. Neutral mutations would either become fixed in all of its members, or be lost
entirely due to genetic drift. @ 1m [2]

[Total: 13]

ACJC 9648/02/Prelim 2012 (Mark Scheme Post-Standardization v1) [Turn over


55
13 For
examiners
Section B use
8 (a) Describe the roles of organelles involved in the synthesis and transport of proteins destined
to be embedded on the cell surface membrane. [8]

1. Ref to gene coding for membrane protein transcribed in nucleus;


2. Ref. to export of mRNA to cytoplasm via nuclear pores;
3. mRNA attaches to ribosome at RER surface for translation;
4. Ref to formation of translation initiation complex involving methionyl-tRNA,

E
ribosomes at the 5-end of mRNA;
5. Ref to complementary base pairing between anti-codon of tRNA with codons of
mRNA;
6. Ref to (elongation via) peptidyl transferase catalyzing the formation of peptide bond
between amino acids;
7. As ribosome moves along mRNA in 5 to 3 direction till stop codon;
8. polyribosomes may form to increase the rate of protein synthesis;
9. Completed polypeptide released into and embedded into membrane of RER;
10. Transport vesicles with polypeptide (embedded) moved towards and fused with (cis
face of) Golgi apparatus;
11. Proteins undergo post-translational modifications in RER/GA;
12. Membrane (with embedded proteins) pinches off as secretory vesicles;
13. Vesicles travel along microtubules;
14. And fuse with plasma membrane/ cell surface membrane;
@ 1m , max 8

(b) Compare the process in which ATP molecules are synthesised during respiration with that in
photosynthesis. [6]
Similarities: For
1. Both processes involved chemiosmotic synthesis/proton gradient across Examiner's
membrane surfaces; Use
For
2. Both processes involved hydrogen carriers, ETC and ATP synthase (any 2 of 3); Examiner's
Use
Differences:
Respiration Photosynthesis
3. ATP synthesized via both substrate- ATP synthesized via
level and oxidative phosphorylation; photophosphorylation only;

4. ATP produced in cytoplasm as well as ATP produced only in chloroplasts;


inside mitochondria;

5. Energy/(H+ and) electrons used for Energy/H+ and electrons used for ATP
ATP synthesis from chemical synthesis from light absorbed by
reactions/oxidation involving organic photosynthetic pigments/photolysis of
compounds; water;

6. Proton pool maintained in Proton pool maintained in thylakoid


intermembrane space in mitochondria / space in chloroplasts;
Ref to direction of proton
movement/location of ATP
synthase/ETC;
7. ATP formation during aerobic ATP formation during photosynthesis
respiration leads to the formation of leads to the formation of reduced NADP
water using O2 as final electron and release of O2 during non-cyclic
acceptor; photophosphorylation;
8. while lactic acid (in mammalian cells)
and ethanol + CO2 (in plant and
yeasts) formed during anaerobic
respiration;

ACJC 9648/02/Prelim 2012 (Mark Scheme Post-Standardization v1) [Turn over


56
14 For
examiners
9. ATP produced continuously/both in ATP production only in the presence of use
presence or absence of light/in all light/cells with chloroplasts;
cells;
10. Hydrogen carriers involved are NAD+ Hydrogen carriers involved are NADP+;
and FAD;
11. Process results in oxidation of Process results in reduction of NADP+;
reduced NAD+ and FAD;

E
@ 1 m max 5

(c) Describe how a phylogenetic tree is constructed. [6]

1. Involves a comparison of nucleotide sequences / amino acid sequences of


different groups of organisms;
2. Ref to the method of DNA hybridization to compare nucleotide differences;
3. Genes / proteins being compared must be homologous;
4. Grouped according to degree of similarities / with highest similarity index/closely
related species are grouped/ clustered together;
5. Branch length represents the number / degree of changes that have occurred;
6. Common ancestor between species is represented by the nodes in the
phylogenetic tree;
7. A common ancestor together with all its descendants are organized into a clade
8. Ref to use of molecular clock to elucidate time of divergence;
9. Ref to use of radiometric dating/stratigraphy/index fossils to find age of fossils;
10. Ref. to use of transition fossils to verify/support phylogeny;
@1m, max 6
For
Examiner's
Use
9 (a) With reference to the control of blood glucose concentration, explain the principles of For
homeostasis in terms of receptors, effectors, and negative feedback. [8] Examiner's
Use

1 Homeostasis is the maintenance of a constant internal environment;


2 Blood glucose concentration is maintained at 80-90 mg/100 ml blood;

Receptor
3 / cells of Islets of Langerhans are the receptors (which detect the
increase/decrease/deviation in blood glucose concentration);
4 Resulting in the (increased) secretion of insulin/glucagon;

Effector
5 The effectors are liver, fat and muscle (any 2 of 3 for insulin) / liver (for glucagon);
6 Any e.g. of corrective mechanism by effector i.e. insulin causes increase in
glycogenesis in liver/muscle cells / increase in glucose carriers in the plasma
membrane / increase in glucose uptake in fat/muscle cells / increase in rate of
respiration/ATP synthesis/ DNA/RNA synthesis OR glucagon causes increase in
glycogenolysis/gluceoneogenesis / decrease in glycogenolysis in liver cells ;
7 Insulin/glucagon brings about a decrease/increase in blood glucose levels (back to
norm);

Negative feedback
8 Ref. to negative feedback e.g. a disturbance in the system sets in motion a sequence
of events which counteracts the disturbance and brings the system to its original
state/ Negative feedback brings about change in the opposite direction restoring
stability / Negative feedback is seen when the initial increase in blood glucose
concentration brings about the subsequent decrease in blood glucose concentration
/ OWTTE;

ACJC 9648/02/Prelim 2012 (Mark Scheme Post-Standardization v1) [Turn over


57
15 For
examiners
9 (Negative feedback mechanism is also involved in inhibiting) further secretion of use
insulin/glucagon as receptor/ / cells of Islets of Langerhans detects that blood
glucose concentration is back to norm;

10 Ref. made to the opposite scenario when blood glucose concentration


decreases/increases below/above set point, resulting in the release of the other
hormone (glucagon / insulin;
11 Ref. to self-regulating nature of homeostasis;

E
Note: students need to have at 1 point from 3-5 (receptor section) AND 5-6 (effector
section) AND 8-9 (negative feedback section) to score full marks.

@1m, max 8

(b) Describe how the uni-directional transmission of nerve impulses is ensured along a
neuron and across a synapse. [6]

Along a neuron (max 3)


1. Ref to AP/repolarisation phase, prolonged efflux of K+ results in hyperpolarisaton;
2. Leading to refractory period;
3. where the membrane cannot respond to another normal stimulus/be depolarised;
4. Absolute refractory period (when no stimulus, however strong, can initiate another
AP) and relative refractory period (when another AP can only be initiated if the
stimulus is stronger than normally required);
5. Ensures that the AP is propagated towards the synaptic terminal; For
Examiner's
Use
Across a synapse (max 3) For
6. Synaptic vesicles/neurotransmitter present only in pre-synaptic terminals; Examiner's
Use

7. Neurotransmitter receptors / ligand-gated ion channels are present only on the


post-synaptic membrane / dendrites / cell body of the post-synaptic neuron;
8. Acetylcholinesterase present in cleft/at post-synaptic membrane to hydrolyse
neurotransmitter to maintain diffusion gradient;
9. Hence impulse/neurotransmitter is transmitted only from pre-synaptic terminal to
post-synaptic neuron;

@ 1m

(c) With reference to sickle-cell anemia, explain how gene mutation may affect the
phenotype of a person. [6]

1. The mutation in sickle-cell anemia involves the substitution of one base, from
thymine to adenine in haemoglobin;
2. (Alteration in nucleotide sequence) changes the codon of mRNA(from GAA to GUA
and thus the amino acid sequence in a polypeptide);
3. Mutated beta-globin/haemoglobin has hydrophilic amino acid glutamic acid
replaced by hydrophobic valine;
4. Affects the conformation/tertiary structure of protein (may result in loss of protein
activity/change in function/ non-functional protein, leading to a change in
phenotype of a person);
5. At low oxygen concentrations, (the hydrophobic portion of HbS molecules stick
together, causing) the haemoglobin (HbS) molecules (to) polymerize into fibres.

ACJC 9648/02/Prelim 2012 (Mark Scheme Post-Standardization v1) [Turn over


58
16 For
examiners
(This in turn causes) the red blood cells (to) change (from a circular biconcave use
shape) to a sickle-shape;
6. Sickle-shaped red blood cells may obstruct blood capillaries, hence, (multiple)
organs deprived of oxygen/resulting in their damage.
7. These cells (more rigid and fragile than normal red blood cells) haemolyse readily
resulting in anaemia.
8. They also accumulate in spleen for destruction leading to an enlargement of the

E
spleen.
Points 5 to 8 (phenotype) max 3 m

@1m, max 6

For
Examiner's
Use
For
Examiner's
Use

ACJC 9648/02/Prelim 2012 (Mark Scheme Post-Standardization v1) [Turn over


59

Subject
Name Class Class Index Number
2BI

ANGLO-CHINESE JUNIOR COLLEGE


Preliminary Examination 2013

BIOLOGY 9648/03

Applications Paper and Planning Question


3 Sept 2013
PAPER 3 2 hours

Additional Materials: Writing Paper

READ THESE INSTRUCTIONS FIRST

Write your name, subject class, form class and index number on all the work you hand in.
Write in dark blue or black pen on both sides of the paper.
You may use a soft pencil for any diagrams, graphs or rough working.
Do not use staples, paper clips, highlighters, glue or correction fluid.

Answer all questions.

At the end of the examination, fasten your work securely together.


The number of marks is given in brackets [ ] at the end of each question or part question.

FOR EXAMINERS USE

TOTAL
72

This document consists of 16 printed pages.

ACJC 9648/03 Prelim 2013


60
2 For
Examiners
Use

1. Embryonic stem (ES) cells are highly regarded in research because their pluripotency offers the
potential for a wide range of therapeutic and research applications. However, ethical concerns
have been raised with regards to the source of ES cells.
Scientists have come up with an alternative method of generating pluripotent cells, which is to
genetically reprogramme adult cells to an embryonic stem cell-like state. Genetic reprogramming
is carried out by using deactivated retroviruses to introduce the genes of four transcription factors
into adult cells from a patient (Fig. 1.1).The reprogrammed cells, called induced pluripotent stem
(iPS) cells, are specific to the patient from which the adult cells were taken.

Fig. 1.1

(a) (i) Describe what is meant by the term pluripotent.

It describes the potential of a cell to become/differentiate into any cell type in the

adult body but not those of the extra-embryonic membranes; @ 1m [1]

(ii) Suggest one reason why the use of iPS cells evades the ethical concerns regarding the
source of ES cells.

Ref. to iPS uses adult cells, whereas obtaining ES cells involves the destruction

of embryos; @ 1m [1]

ACJC 9648/03 Prelim 2013 [Turn over


61
3 For
Examiners
Use

(iii) Give three reasons why deactivated retroviruses are used for genetic reprogramming of
the adult cells rather than active retroviruses or liposomes.

*1. deactivated because they will not cause disease/will not become virulent;
2. retoviruses allow for integration of genes into host genome while liposomes
do not;
3. retroviruses are more efficient at delivering genes into the host cell than
liposomes because they target specific cells;

*required for full marks


[3]

(iv) Suggest how the expression of such a small number of transcription factors in adult cells
could genetically reprogramme these adult cells to an embryonic stem cell-like state.

1. Each transcription factor can activate the transcription of multiple genes;


2. Each gene in turn could code for a transcription factor which activates other
genes;

3. The genes switched on are those expressed in pluripotent/ES cells;


4. e.g. telomerase gene/ genes which promote cell division/ genes which cause
the cell to revert to undifferentiated state;
5. resulting in the synthesis of proteins which are found in ES cells;
6. The transcription factor could also inhibit gene expression;
7. e.g. genes which result in specialisation (are repressed);
@ 1m, max 3 [3]

ACJC 9648/03 Prelim 2013 [Turn over


62
4 For
Examiners
Use

Patient-specific iPS cells can be induced to differentiate into the affected diseased tissue as this
will allow the patient's disease to be modelled in vitro. Using this diseased tissue, potential drugs
to treat the disease can be screened for, aiding in the discovery of new drugs.

Scientists have managed to obtain iPS cells from cystic fibrosis (CF) patients with a mutation at
amino acid position 508 (F508). They have caused these iPS cells to differentiate into diseased
human lung tissue, which is now being used to screen for potential drugs to treat cystic fibrosis
caused by the F508 mutation (Fig. 1.2).

Fig. 1.2

Previously, a cell line comprising rat cells expressing recombinant human mutant CFTR protein
was used for screening purposes. This cell line was successfully used to identify a drug to treat
cystic fibrosis.

(b) (i) Explain the effect of the F508 mutation on the function of the CFTR protein.

1. Mutation results in the deletion of phenylalanine from CFTR protein;

2. resulting in change in tertiary/overall 3D shape change in structure;


3. The CFTR protein is unable to transport chloride ions across the
membrane/out of the cell;

[3]

ACJC 9648/03 Prelim 2013 [Turn over


63
5 For
Examiners
Use

(ii) Suggest and explain one advantage of using a drug to treat CF rather than to use
liposomes to deliver a functional copy of a CFTR gene.
1. easier to regulate dosage of drug/avoid using high concentrations of
liposomes which could be toxic;
2. using the drug avoids the problem that the CFTR gene may not be properly
expressed (to form the CFTR protein); [1]

(iii) Suggest and explain one advantage of using diseased lung tissue derived from human
iPS cells to screen for CF drugs rather than the use of rat cells expressing recombinant
human CFTR.
1. ref. to iPS cells are (human cells and are) more representative of the actual
diseased cell than rat cells e.g. rat cells may have different biochemical
2. ref. to iPS cells are patient-specific and could be used to screen for drugs
specific to that patient; [1]

[Total: 13]

ACJC 9648/03 Prelim 2013 [Turn over


64
6 For
Examiners
Use

2. Cysteine proteinase (CP) is an enzyme that is coded by the mir-1 gene, which is found in plants
that are naturally resistant to attacks by Lepidopteran larvae. Lepidopteran larvae feeding on the
Black Mexican Sweet (BMS) maize leaf tissue cause crop losses. In an attempt to increase the
yield of BMS maize, plant biologists have used genetic technology to transform the wild-type BMS
maize callus with mir-1 gene. The growth of larvae feeding on transformed and non-transformed
BMS maize calli was studied for seven days, and the experiment was repeated. The results are
shown in Fig. 2.1.

Fig. 2.1

(a) (i) Suggest one delivery method that could be used to introduce the mir-1 gene into BMS
maize callus.

[1]

(ii) With reference to Fig. 2.1, describe and explain the effectiveness of genetically engineered
BMS maize with mir-1 gene on larvae growth.

[3]

ACJC 9648/03 Prelim 2013 [Turn over


65
7 For
Examiners
Use

(b) An in vitro experimental study was performed to investigate the effects of transformed BMS
plant cells expressing both cysteine proteinase (CP) and Bacillus thuringiensis (Bt) toxin on
Lepidopteran larvae. Table 2.1 shows the relative growth rate and percentage mortality of
Lepidopteran larvae fed with maize cell lines containing CP, Bt-toxin and a combination of CP
and Bt-toxin.

Table 2.1

CP Bt-toxin CP+ Bt-toxin


Relative Growth Rate/ Arbitrary Units 0.333 0.568 0.220
Percentage Mortality/% 38.0 4.0 50.0

With reference to Table 2.1, describe the effects of maize plant cells expressing CP together
with Bt-toxin on the percentage mortality of Lepidopteran larvae.

[2]

(c) Both CP and Bt-toxin target the mid-gut lumen. Fig. 2.2 shows a cross section of mid-gut. CP is
known to damage the peritrophic protein matrix of mid-gut lumen in the larvae, a structure that
protects the mid-gut from microbial infection.

The mechanism of damage by Bt-toxin is different compared to that of CP. Bt-toxin is


converted to its active form in the alkaline mid-gut environment, which then binds to the
specific mid-gut receptors on the brush border membrane of mid-gut cells with very high
affinity, forming lytic pores that result in cell death.

Fig. 2.2

ACJC 9648/03 Prelim 2013 [Turn over


66
8 For
Examiners
Use

(i) With reference to Table 2.1, Fig. 2.2 and the information given, suggest how CP may
enhance the effect of Bt-toxin on Lepidopteran larvae.

[3]

(ii) Over many generations, laboratory populations of Lepidopteran larvae have evolved
resistance against Bt-toxins. Suggest and explain how this resistance is developed.

[2]

[Total: 11]

ACJC 9648/03 Prelim 2013 [Turn over


67
9 For
Examiners
Use

3. Fig. 3.1 below shows a schematic diagram of targeted gene replacement in a mammalian cell
involving the gene Ig M. Targeted Ig M gene replacement is a method of replacing Ig M mutated
allele (Fig. 3.1a) in the genome in a cell with wildtype allele (Fig. 3.1b) via crossing over, giving
rise to the recombinant DNA (Fig. 3.1c). The wildtype allele is carried by a linearised plasmid (Fig.
3.1b) and introduced into the cell.

As shown in Fig. 3.1b, the linearised plasmid contains an antibiotic resistance gene (Neo gene),
wildtype alllele with the exons represented by the black boxes, as well as several restrictions sites
in between the exons. In this technique, the enhancer of the Neo gene on the linearised plasmid
was removed prior to the therapy.

Ig M mutated alleleNhe1
Ava1
Sca1

Afl1

Enhancer Genomic Fig. 3.1 a


fragment
F primer
Region of crossing over
Hpa1
Ecor1

Aat1
Dra1
Kpn1

Neo gene
Linearized Fig. 3.1 b
Ig M wildtype allele plasmid
R primer
Hpa1
Aat1
Ava1

Dra1
Sca1

Enhancer Neo gene


Recombinant
Fig. 3.1 c
DNA

Fig. 3.1

Legend
Exon
Restriction sites on mutated allele Ava1, Sca1, Afl1, Nhe1
Restriction sites on wildtype allele Kpn1, Ecor1, Dra1, Aat1
Naturally occurring restriction site on Hpa1
linearised plasmid

(a) (i) Explain the significance of restriction enzyme in bacteria.

[1]

(ii) Although the mutated and wildtype alleles are homologous regions, different restriction
sites are present in them. Explain the presence of these different restriction sites.

[2]
ACJC 9648/03 Prelim 2013 [Turn over
68
10 For
Examiners
Use

(iii) With reference to Fig. 3.1, explain how the Neo gene allows the transformed cells with
recombinant DNA to be distinguished from the transformed cells which did not undergo
recombination in their DNA.

[4]

(iv) With reference to Fig. 3.1a and Fig. 3.1b, briefly describe another method how Hpa1 can
be used to verify the identity of cells with recombinant DNA.

[4]

(v) The wildtype IgM allele was isolated from a genomic DNA library and not a cDNA library
before being introduced into the linearised plasmid. With reference to Fig. 3.1, state one
evidence which suggests this.

[1]

ACJC 9648/03 Prelim 2013 [Turn over


69
11 For
Examiners
Use

(b) In the gene replacement therapy shown in Fig. 3.1, the site of DNA breakage occurred
between restriction sites Sca1 and Afl1 as shown in Fig. 3.1a. In another case of therapy, the
site of DNA breakage along the mutated allele was unknown. To determine the exact site of
crossing over between the genomic DNA and the linearised plasmid, RFLP analysis was
carried out on the recombinant DNA. The recombinant DNA was isolated from the
transformed cell and amplified using PCR via the forward and reverse primers as shown in Fig.
3.1a and Fig. 3.1b. The amplified DNA products were separated into eight tubes and digested
using eight restriction enzymes respectively: Ava1, Sca1, Afl1, Nhe1, Kpn1, Ecor1, Dra1, Aat1.
The products of digestions were separated using gel electrophoresis and the results are
shown in Fig. 3.2.

Ava1 Sca1 Afl1 Nhe1 Kpn1 Ecor1 Dra1 Aat1

Fig. 3.2

(i) With reference to Fig. 3.1 and 3.2, describe and explain the band pattern of the restriction
digest for the recombinant cell.

[3]

(ii) Hence identify the site of DNA breakage where crossing over took place in the mutated
allele.

[1]

[Total: 16]

ACJC 9648/03 Prelim 2013 [Turn over


70
12 For
Examiners
Use

Planning Question
4. Electroporation is a method which is routinely used to introduce foreign DNA into E. coli cells. This
involves subjecting a mixture of bacteria and plasmid to a brief electric pulse. You are required to
plan, but not carry out, an investigation into the effect of voltage on the efficiency of transformation
by electroporation.
In this investigation, the recombinant plasmid used to transform the E. coli cells is made up of two
components: the plasmid vector and the gene of interest. pGFP is a plasmid with two marker
genes, GFP and AmpR, and a number of restriction sites throughout the plasmid as shown in Fig.
4.1 below. GFP is a marker gene coding for the green fluorescent protein. Expression of GFP in
bacteria will cause bacteria to fluoresce under UV light. The gene of interest is the ras proto-
oncogene, which has been inserted into the plasmid using the restriction enzyme BmtI.

Fig. 4.1
An electroporator is a machine used to carry out electroporation. It is possible to vary the voltage
and length of the electrical pulse with this machine. The range of voltage used is typically 1400
2000 V and the length of electrical impulse between 4-6 ms. A voltage which is too high will cause
irreversible changes in membrane permeability. The mixture of bacteria and recombinant plasmid,
which must be kept on ice prior to electroporation, is put into a cuvette (Fig. 4.2) which is then
placed into a chamber in the electroporator (Fig. 4.3).
After electroporation, the mixture of bacteria must be quickly resuspended in SOC broth (a type of
growth medium) and incubated at 37C for 1 hour for the surviving cells to recover. This is then
followed by plating the bacteria on agar and incubation at 37C for 16-24 hours. The plates can
then be examined to calculate transformation efficiency.

Fig. 4.2
ACJC 9648/03 Prelim 2013 [Turn over
71
13 For
Examiners
Use

Fig. 4.3

Transformation efficiency can be calculated using the following equation: -

Transformation efficiency / cfu g1 = Total number of colonies on agar plate

[cfu: colony forming unit] Amount of DNA plated (in g)

Your planning must be based on the assumption that you have been provided with the following
equipment and materials which you must use:
Microcentrifuge tubes of 100 l E. coli cells in electroporation buffer
Microcentrifuge tubes of 1 l recombinant DNA (concentration of 0.1 g / l)
Microcentrifuge tubes of 1 l sterile water
Ice bucket with ice
Electroporator
Sterile cuvettes for electroporation (max. volume of 150l)
Agar plates with ampicillin
Sterile spreader for spreading bacterial culture on agar plate
SOC broth
Incubator
UV transilluminator (a device which emits UV light)
UV shields which provide protection from UV light
A variety of micropipettes, sterile micropipette tips and sterile 2 ml microcentrifuge tubes

Your plan should have a clear and helpful structure to include


an explanation of the theory to support your practical procedure
a description of the method used, including the scientific reasoning behind the method, a
control experiment and any recommended safety measures
an explanation of the dependent and independent variables involved
relevant, clearly labelled diagram/s showing selection of the recombinant bacteria on plates
how you will record your results and ensure that they are as accurate and reliable as possible
proposed layout of results tables and graphs with clear headings and labels
the correct use of technical and scientific terms

[Total: 12]

ACJC 9648/03 Prelim 2013 [Turn over


72
14 For
Examiners
Use

ACJC 9648/03 Prelim 2013 [Turn over


73
15 For
Examiners
Use

ACJC 9648/03 Prelim 2013 [Turn over


74
16 For
Examiners
Use

Free-response question

Write your answers to this question on the separate paper provided.

Your answer:

should be illustrated by large, clearly labelled diagrams, where appropriate.


must be in continuous prose, where appropriate.
must be set out in sections (a), (b) etc., as indicated in the question.

5. (a) Describe and explain how bacteria can be used to mass produce human insulin without the
use of any DNA libraries. [7]

(b) Discuss the advantages and the limitations of the Polymerase Chain Reaction. [5]

(c) Discuss the implications of the Human Genome Project, including the benefits and difficult
ethical concerns for humans. [8]

[Total: 20]

ACJC 9648/03 Prelim 2013 [Turn over


75
ACJC H2 Bio P3 Mark Scheme 2013

1 Embryonic stem (ES) cells are highly regarded in research because their pluripotency offers the
potential for a wide range of therapeutic and research applications. However, ethical concerns
have been raised with regards to the source of ES cells.
Scientists have come up with an alternative method of generating pluripotent cells, which is to
genetically reprogramme adult cells to an embryonic stem cell-like state. Genetic reprogramming
is carried out by using deactivated retroviruses to introduce the genes of four transcription factors
into adult cells from a patient (Fig. 1.1).The reprogrammed cells, called induced pluripotent stem
(iPS) cells, are specific to the patient from which the adult cells were taken.

Fig. 1.1

(i) Describe what is meant by the term pluripotent.

It describes the potential of a cell to become/differentiate into any cell type in the

adult body but not those of the extra-embryonic membranes; @ 1m [1]

(ii) Suggest one reason why the use of iPS cells evades the ethical concerns regarding the
source of ES cells.

Ref. to iPS uses adult cells, whereas obtaining ES cells involves the destruction /
endangering / risk of embryos;
A! pts
@ 1m
[1]

ACJC 9648/03 Prelim 2013


76
2 For
Examiners
Use

(iii) Give three reasons why deactivated retroviruses are used for genetic reprogramming of
the adult cells rather than active retroviruses or liposomes.

*1. deactivated because they will not cause disease/infect other cells /will not
become virulent;
2. retoviruses allow for integration of genes into host genome while liposomes
do not;
3. retroviruses are more efficient at delivering genes into the host cell than
liposomes because they target specific cells while liposome does not fuse with
target cell;

R! ref. to toxicity of liposomes as liposomes not administered in vivo.


Both side of ans must be given to award 1 mark except for pt 1.
R! Lysozyme;
*required for full marks
[3]

(iv) Suggest how the expression of such a small number of transcription factors in adult cells
could genetically reprogramme these adult cells to an embryonic stem cell-like state.

1. Each transcription factor can activate the transcription of multiple genes;


2. Each gene in turn could code for a transcription factor which
activates/inactivate other genes;
3. transcription factors switched on are those expressed in (pluripotent/ES cells)
/ transcription factor could also inhibit gene expression;
4. e.g. telomerase gene/ genes which promote cell division/ genes which cause
the cell to revert to undifferentiated state;
5. resulting in the synthesis of proteins which are found in ES cell;

6. e.g. genes which result in specialisation (are repressed);


@ 1m, max 3 [3]

ACJC 9648/03 Prelim 2013 [Turn over


77
3 For
Examiners
Use

Patient-specific iPS cells can be induced to differentiate into the affected diseased tissue as
this will allow the patient's disease to be modelled in vitro. Using this diseased tissue, potential
drugs to treat the disease can be screened, aiding in the discovery of new drugs.

Scientists have managed to obtain iPS cells from cystic fibrosis (CF) patients with a mutation
at amino acid position 508 (F508). They have caused these iPS cells to differentiate into
diseased human lung tissue, which is now being used to screen for potential drugs to treat
cystic fibrosis caused by the F508 mutation (Fig. 1.2)

Fig. 1.2

Previously, a cell line comprising of rat cells expressing recombinant human mutant CFTR
was used for screening purposes. This cell line was successfully used to identify a drug to
treat cystic fibrosis.

(v) Explain the effect of the F508 mutation on the function of the CFTR protein.

1. Mutation results in the deletion of 3 bases / phenylalanine from CFTR protein;

2. resulting in change in tertiary/overall 3D shape change in structure;


3. The CFTR protein is unable to transport chloride ions across the
membrane/out of the cell;

[3]

ACJC 9648/03 Prelim 2013 [Turn over


78
4 For
Examiners
Use

(vi) Suggest and explain one advantage of using a drug to treat CF rather than to use
liposomes to deliver a functional copy of a CFTR gene.
1. easier to regulate dosage of drug/avoid using high concentrations of
liposomes which could be toxic;
2. using the drug avoids the problem that the CFTR gene may not be properly
expressed (to form the CFTR protein); [1]
3. Drug is able to travel to many/most parts of body and treat all affected parts
whereas liposomes are able to treat only localised areas; (note: liposomes can
be injected into the blood stream but the e.g. in the notes relates to the
treatment of a localised area)
4. Drug with (specific 3D config) can bind to specific cells while liposome are
not specific in targeting cell

Award when either side of point given.


R! Ref. to easier to administer drug than liposomes to patient e.g. drug can be
taken orally/by inhaler because liposomes can be administered by inhaler too.
R! Drugs are cheaper than using liposomes to deliver CFTR gene

(vii) Suggest and explain one advantage of using diseased lung tissue derived from human
iPS cells to screen for CF drugs rather than the use of rat cells expressing recombinant
human CFTR.
1. ref. to iPS cells are (human cells and are) more representative of the actual
diseased cell than rat cells e.g. rat cells may have different biochemical
pathways (which affect CFTR protein or drug)/ OWTTE;
2. ref. to iPS cells are patient-specific and could be used to screen for drugs
specific to that patient (i.e. personalised treatment); [1]

[Total: 13]

ACJC 9648/03 Prelim 2013 [Turn over


79
5 For
Examiners
Use

2 (a) Cysteine proteinase (CP) is an enzyme that is coded by mir-1 gene, which is found in plants
that are naturally resistant to attacks by Lepidopteran larvae. Lepidopteran larvae feeding on
the Black Mexican Sweet (BMS) maize leaf tissue causes crop losses. In an attempt to
increase the yield of BMS maize, plant biologists have used genetic technology to transform
the wild-type BMS maize callus with mir-1 gene. The growth of larvae feeding on transformed
and non-transformed BMS maize calli was studied for seven days, and the experiments were
repeated twice. The results are shown in Fig. 2.1.

Fig. 2.1

(i) Suggest one delivery method that could be used to introduce the mir-1 gene into BMS maize
callus.
1. Ref. to the use of gene gun/soil bacterium/ Agrobacterium/Electroporation; @ 1m

R! liposome (due to presence of plant cell wall) [1]

(ii) With reference to Fig. 2.1, describe and explain the effectiveness of genetically engineered
BMS maize with mir-1 gene on larvae growth.
1. Larvae feeding on transformed BMS maize have a decreased larvae growth /
lower weight (as compared to that of the untransformed BMS maize);
2. Ref. to 140 mg as shown in exp. 1 as compared to 25 mg for larvae feeding on
transformed maize callus
OR
(85 - 86) mg as shown in exp. 2 for larvae feeding on untransformed maize callus
compared to (28-30) mg as shown for larvae feeding on transformed maize
callus; @ 1m
3. CP/mir-1 protein was expressed in transformed BMS maize callus and
ingested/eaten/consumed by larvae (which slows down/reduces/stunts their
growth);

[3]

ACJC 9648/03 Prelim 2013 [Turn over


80
6 For
Examiners
Use

(b) An in vitro experimental study was performed to investigate the effects of transformed BMS
plant cells expressing both cysteine proteinase (CP) and Bacillus thuringiensis (Bt) toxin on
Lepidopteran larvae. Table 2.1 shows the relative growth rate and percentage mortality of
Lepidopteran larvae fed with maize cell lines containing CP, Bt-toxin and a combination of CP
and Bt-toxin.

Table 2.1

CP Bt-toxin CP+ Bt-toxin


Relative Growth Rate/ Arbitrary Units 0.333 0.568 0.220
Percentage Mortality/% 38.0 4.0 50.0

With reference to Table 2.1, describe the effects of maize plant cells expressing CP together
with Bt-toxin on the percentage mortality of Lepidopteran larvae.

1. Highest/higher/significant percentage mortality as compared to that of larvae


feeding on CP or Bt-toxin maize cells;

2. Ref. to figs 50% vs 4 % vs 38 % in table with appropriate units; @ 1 m

[2]

(c) Both CP and Bt-toxin target the mid-gut lumen. Fig 2.2 shows a section of mid-gut. CP is
known to damage the peritrophic protein matrix of mid-gut lumen in the larvae, a structure that
protects the mid-gut from microbial infection.

The mechanism of damage by Bt-toxin is different compared to that of CP. Bt-toxin is


converted to its active form in the alkaline mid-gut environment, which then binds to the
specific mid-gut receptors on the brush border membrane of midgut cells with very high affinity,
forming lytic pores that result in cell death.

Fig. 2.2

ACJC 9648/03 Prelim 2013 [Turn over


81
7 For
Examiners
Use

(i) With reference to Table 2.1 and information given above, suggest how CP may enhance
the effect of Bt-toxin on Lepidopteran larvae.

1. ref. to 4% mortality (relative growth rate 0.568 A.U.) due to Bt toxin alone;

2. CP digests/breakdown proteins found in peritrophic protein matrix, (therefore


more porous);
3. Ref. to facilitate the movement of Bt-toxin through the membrane into the
brush border membrane of mid-gut lumen /higher accessibility for Bt toxin; [3]

(ii) Over many generations, laboratory populations of Lepidopteran larvae have evolved
resistance against Bt-toxins. Suggest and explain how the resistance is developed.

*1. Ref. to mutation in genes


2. The alteration of Bt-toxin binding site on the mid-gut receptors e.g. decreased
affinity or reduction in the number of binding sites;
3. Changes in mid-gut environment such that Bt-toxin is not converted to its
active form;
4. Rapid regeneration of the damaged mid-gut epithelium; @ 1m
Either pt 1 and 2/3/4

5. (Genotypic/ phenotypic) variation in resistance and selection pressure due


to Bt toxin ; [2]
6. Differential reproductive success/fitness;
7. Change in allele frequency (over time);

[Total: 11]

ACJC 9648/03 Prelim 2013 [Turn over


82
8 For
Examiners
Use

3. Fig. 3.1 below shows a schematic diagram of targeted gene replacement in a mammalian cell
involving the gene Ig M. Targeted Ig M gene replacement is a method of replacing Ig M mutated
allele (Fig. 3.1a) in the genome in a cell with wildtype allele (Fig. 3.1b) via crossing over, giving
rise to the recombinant DNA (Fig. 3.1c). The wildtype allele is carried by a linearised plasmid (Fig.
3.1b) and introduced into the cell.

As shown in Fig. 3.1b, the linearised plasmid contains an antibiotic resistance gene (Neo gene),
wildtype alllele with the exons represented by the black boxes, as well as several restrictions sites
in between the exons. In this technique, the enhancer of the Neo gene on the linearised plasmid
was removed prior to the therapy.

Ig M mutated alleleNhe1
Ava1
Sca1

Afl1

Enhancer Genomic Fig. 3.1 a


fragment
F primer
Region of crossing over
Hpa1
Ecor1

Aat1
Dra1
Kpn1

Neo gene
Linearized Fig. 3.1 b
Ig M wildtype allele plasmid
R primer
Hpa1
Aat1
Ava1

Dra1
Sca1

Enhancer Neo gene


Recombinant
Fig. 3.1 c
DNA

Fig. 3.1

Legend
Exon
Restriction sites on mutated allele Ava1, Sca1, Afl1, Nhe1
Restriction sites on wildtype allele Kpn1, Ecor1, Dra1, Aat1
Naturally occurring restriction site on Hpa1
linearised plasmid

i. Explain the significance of the restriction enzyme in bacteria.

1.To (protect) against foreign DNA by digesting/cutting it ; @1m

A! destroy, degrade [1]

ii. Although the mutated and wildtype alleles are homologous regions, different restriction
sites are present on them. Explain the presence of these different restriction sites.
1. (Single) DNA polymorphism / changes in DNA sequences between individual /
mutation;

2. Creates / destroy / changes restriction sites @1m

R! RFLP [2]
ACJC 9648/03 Prelim 2013 [Turn over
83
9 For
Examiners
Use

iii. With reference to Fig 3.1, explain how the Neo gene allows the transformed cells with
recombinant DNA to be distinguished from transformed cells which did not undergo
recombination in their DNA.
1. Cells that were transformed with subsequent recombination in the
chromosome/DNA would survive in the presence of neomycin;
A! Neo antibiotic or just a/the antibiotic. R! antibiotics.
2. Transformed cell without recombination event would not survive
3. Neo gene from vector would come under the control of an active enhancer
4. High expression of neo gene confers resistance against neomycin. A!
comparison.
5. No /low expression of neo gene due to absence of enhancer in linearized
plasmid / absence of Neo gene on genomic fragment;

[4]

iv. With reference to Fig 3.1a and Fig.3.1 b, briefly describe another method how Hpa1 can be
used to verify the identity of cells with recombinant DNA.

(Replica plating of cells with recombinant DNA)


*1. (lyse cell) followed by cutting/digesting it with restriction enzyme Hpa1
R! restriction enzyme added to cells, DNA treated with restriction enzyme
2. separate DNA fragment via gel electrophoresis;
3. stain / Intercalate with ethidium bromide and visualised under UV light;
4. recombinant DNA will show 2 bands while non-recombinant DNA will show
band; A! more bands
OR
[4]
*1. (lyse cell) followed by cutting it with restriction enzyme hpa1
2. separate DNA fragment via gel electrophoresis;
5.Ref to southern blotting / transfer to nitrocellulose membrane;
6.Radioactive probe binds to enhancer
A! any other reasonable specified area or sequence;
7.1 (probe targeting enhancer) short band (recombinant DNA) vs 1 long band
(genomic DNA) vs no band (linearised plasmid)
A! other logical patterns depending on the specified probe
Compulsory pt 1

v. The wildtype IgM allele was isolated from a genomic DNA library and not a cDNA library
before being introduced into the linearized plasmid. With reference to Fig. 3.1 state one
evidence which suggest so.

1. allele shown contain introns / non-coding DNA;


R! enhancer
[1]

ACJC 9648/03 Prelim 2013 [Turn over


84
10 For
Examiners
Use

b) In the gene replacement therapy shown in Fig. 3.1, the site of DNA breakage occurred
between restriction sites Sca1 and Afl1 as shown in Fig. 3.1a. In another case of therapy, the
site of DNA breakage along the mutated allele was unknown. To determine the exact site of
crossing over between the genomic DNA and the linearised plasmid, RFLP analysis was
carried out on the recombinant DNA. The recombinant DNA was isolated from the
transformed cell and amplified using PCR via the forward and reverse primers as shown in Fig.
3.1a and Fig. 3.1b. The amplified DNA products were separated into eight tubes and digested
using eight restriction enzymes respectively: Ava1, Sca1, Afl1, Nhe1, Kpn1, Ecor1, Dra1, Aat1.
The products of digestions were separated using gel electrophoresis and the results are
shown in Fig. 3.2.

Ava1 Sca1 Afl1 Nhe1 Kpn1 Ecor1 Dra1 Aat1

Fig .3.2

(i) With reference to Fig.3.1 and 3.2, describe and explain the band pattern of the restriction
digest for the recombinant cell.
1. presence of 2 restriction fragments each using Ava1, Ecor1, Dra1 and Aat1
digest;
2. Indicates presence of restriction sites on recombinant DNA.
3. digest with Sca1, Afl1, Nhe1 and Kpn1 yields no bands,
4. indicating absence of restriction sites;
5. Ref. to explanation linking action of named restriction enzyme/position of a
named restriction site (Ava1, Ecor1, Dra1 and Aat1) on the size of the fragments
produced;

[3]

(ii) Hence identify the site of DNA breakage where crossing over took place in the mutated
allele.

Between Ava1 and Sca1;

[1]

[Total: 16]

ACJC 9648/03 Prelim 2013 [Turn over


85
11 For
Examiners
Use

Planning Question

4 Electroporation is a method which is routinely used to introduce foreign DNA into E. coli cells. This
involves subjecting a mixture of bacteria and plasmid to a brief electric pulse. You are required to
plan, but not carry out, an investigation into the effect of voltage on the efficiency of transformation
by electroporation.

In this investigation, the recombinant plasmid used to transform the E. coli cells is made up of two
components: the plasmid vector and the gene of interest. pGFP is a plasmid with two marker
genes, GFP and AmpR, and a number of restriction sites throughout the plasmid as shown in Fig.
4.1 below. GFP is a marker gene coding for the green fluorescent protein. Expression of GFP in
bacteria will cause bacteria to fluoresce under UV light. The gene of interest is the ras proto-
oncogene, which has been inserted into the plasmid using the restriction enzyme BmtI.

Fig. 4.1

An electroporator is a machine used to carry out electroporation. It is possible to vary the voltage
and length of the electrical pulse with this machine. The range of voltage used is typically 1400
2000 V and the length of electrical impulse between 4-6 ms. A voltage which is too high will cause
irreversible changes in membrane permeability. The mixture of bacteria and recombinant plasmid,
which must be kept on ice prior to electroporation, is put into a cuvette (Fig. 4.2) which is then
placed into a chamber in the electroporator (Fig. 4.3).

After electroporation, the mixture of bacteria must be quickly resuspended in SOC broth (a type of
growth medium) and incubated at 37C for 1 hour for the surviving cells to recover. This is then
followed by plating the bacteria on agar and incubation at 37C for 16-24 hours. The plates can
then be examined to calculate transformation efficiency.

ACJC 9648/03 Prelim 2013 [Turn over


86
12 For
Examiners
Use

Fig. 4.2

Fig. 4.3
Transformation efficiency can be calculated using the following equation: -

Transformation efficiency / cfu g1 = Total number of colonies on agar plate

[cfu: colony forming unit] Amount of DNA plated (in g)

Your planning must be based on the assumption that you have been provided with the following
equipment and materials which you must use:
Microcentrifuge tubes of 100 l E. coli cells in electroporation buffer
Microcentrifuge tubes of 1 l recombinant DNA (concentration of 0.1 g / l)
Microcentrifuge tubes of 1 l sterile water
Ice bucket with ice
Electroporator
Sterile cuvettes for electroporation (max. volume of 150l)
Agar plates with ampicillin
Sterile spreader for spreading bacterial culture on agar plate
SOC broth
Incubator
UV transilluminator (a device which emits UV light)
UV shields which provide protection from UV light
A variety of micropipettes, sterile micropipette tips and sterile 2 ml microcentrifuge tubes

Your plan should have a clear and helpful structure to include


an explanation of the theory to support your practical procedure
a description of the method used, including the scientific reasoning behind the method, a
control experiment and any recommended safety measures
an explanation of the dependent and independent variables involved
relevant, clearly labelled diagram/s showing selection of the recombinant bacteria on plates
how you will record your results and ensure that they are as accurate and reliable as possible
proposed layout of results tables and graphs with clear headings and labels

ACJC 9648/03 Prelim 2013 [Turn over


87
13 For
Examiners
Use

the correct use of technical and scientific terms


[Total:12]
Mark Scheme
1 Theoretical consideration or rationale of the plan to justify the practical procedure
(a) As voltage increases, transformation efficiency increases up to a point where membrane is
irreversibly damaged, leading a drop in transformation efficiency as voltage further
increases /
At voltages which are too high or too low, the bacteria's ability to take up DNA by
transformation becomes reduced, and at high voltages, many/most bacteria will die.
(b) Only transformed bacteria will be able to grow/form colonies on the agar plates with
ampicillin. These colonies should also fluoresce under UV light.

2 At least 5 different voltages (between 1400 to 2000 V), regular spacing between intervals
3 (a) Constant volume of E. coli cells (eg. 100l) and
(b) constant volume of recombinant DNA (eg. 1l)
4 Specify time for incubation of bacteria and recombinant plasmid in ice (1 - 20 minutes). A!
incubate together or separately.
5 Specify fixed time for electroporation (4 6 ms)
6 Specify fixed vol of SOC (>2ml ) and incubate at 37C for 1 hr
7 (a) Incubate the plates at 37C
(b) for fixed no. of hours between 16-24hr R! overnight
8 Control experiment (a) E. coli cells + sterile water subjected to electroporation (no
colonies observed)
(b) subjected to same conditions as mixtures with plasmid / repeat exp/ OWTTE
9 Count number of colonies which fluoresce under UV light
10 (a) Replicates
(b) At least two repeats
11 Tabulation:
(a) Ref to colony count
(c) Average transformation efficiency/cfu g1 under different voltages/V tabulated with correct
headings and units
12 Plot graph (with trend line) of ave. transformation efficiency/ cfu g1 against voltage/ V A!
ECF for colonies and units.
13 Drawing of Petri dish with fluorescent colonies labelled
14 Two safety precautions / risk hazards identified
(a) Wear gloves when handling E. coli to avoid contact with microorganisms
(b) Wipe table with antiseptic solution (such as ethanol/chlorox) to prevent growth of harmful
microorganisms
(c) Use the UV shields to cut down on exposure to UV light
(d) Proper disposal/treatment of contaminated materials or equipment using sterilizer/
autoclave to prevent growth of harmful microorganisms
(e) Handle (plug of) electroporator with dry hands to prevent electrocution.
Max 10 for 2 14

15 The correct use of technical and scientific terms


(a) transient pores
(b) ref. to transformed cells being resistant to ampicillin/being able to grow on ampicillin

ACJC 9648/03 Prelim 2013 [Turn over


88
14 For
Examiners
Use

Free-response question

5 (a) Describe and explain how a bacteria can be used to mass produce human insulin without the
use of any DNA libraries. [7]

1. using isolated mRNA for insulin ( from human pancreatic cells as template) forming
cDNA using reverse transcriptase/reverse transcribe / OWTTE
2. DNA linkers (containing the restriction site) added to cDNA using (DNA ligase)
3. digest plasmids (containing lacZ and ampR or any antibiotic marker genes) and the
cDNA with the a/same RE to produce sticky ends. (A! reference to plasmids cut within
lacZ by RE)
4. mix the cDNA and the plasmids together and let the sticky ends anneal by H bonds by
complementary base pairing
5. Add DNA ligase to covalently join the fragments with phosphodiester bonds (covalent
bond to form recombinant plasmids)
6. transform bacteria / electroporation / heat shock
7. plate the transformed bacterial cells onto nutrient agar plate with ampicillin and X-gal
added into the medium
8. select colonies which are white in colour as these are the ones with recombinant
plasmid (R! bacteria appears white)
9. due to inactivation of Lac z gene
*10. selected bacteria grown (in large quantities) in a suitable medium and bacteria
secrete insulin which is extracted, (purified and packaged for use)

@1m (pt 1 9 max 6; pt 10 needed for full marks)

(b) Discuss the advantages and the limitations of the Polymerase Chain Reaction. [5]

Advantages: (@1m, max 4)

1. Produces large numbers of copies in a relatively short span of time;


2. Cell-free method of DNA replication/requires no cleanup of unwanted cellular
debris or vector DNA;
3. Shown to (efficiently) amplify targets up to 35kb in length;
4. A specific process that targets only the desired DNA to be copied, (hence
starting material does not have to be purified DNA)
5. Only requires a trace amount of DNA;

Limitations: (@1m)

6. Need to first know the nucleotide sequence of at least one short DNA sequence
on each side of the region of interest in order to synthesise the PCR primers;
7. Some errors can be introduced into the amplified DNA copies at low but
significant frequencies / Due to lack of a built-in proofreading activity in Taq
polymerase, PCR produces a higher than normal frequency of replication errors;

ACJC 9648/03 Prelim 2013 [Turn over


89
15 For
Examiners
Use

(c) Discuss the implications of the Human Genome Project, including the benefits and difficult
ethical concerns for humans. [ 8]

Benefits max 4

Genetic testing (max 1)


1 The HGP has allowed for discovery of genes/alleles/polymorphisms associated with
human diseases/ understanding genetics basis of disease;
2 This allows for improved diagnosis of disease / genetic testing;
3 Earlier detection of genetic predispositions to disease (eg. breast cancer, cystic fibrosis,
Alzheimers disease);
4 It also allows scientists to design drugs to target a specific gene/protein (ie. rational
drug design) ;

Pharmacogenetics (max 1)
5 The HGP allows scientists/doctors to know which genes/alleles/polymorphisms affect a
persons response to a drug / genetic differences affect the way we react to a drug;
6 It is (theoretically) possible to tailor drugs/treatments to fit patients genome for greater
efficacy / avoiding dangerous side effects / personalized medicine based on personal
genotype;

Gene therapy
7 (Since gene sequences are now readily available and we better understand which
genes/alleles/polymorphisms are associated with which diseases,) it is possible to use
gene therapy to treat certain diseases;

Risk assessment of individuals upon exposure to toxic agents


8 (The HGP has allowed for discovery of genes/alleles/polymorphisms associated with
resistance/susceptibility to radiation/carcinogens, and hence) an individuals
genome/polymorphisms can be used as a means of risk assessment to evaluate health
risks of individuals who may be exposed to radiation or carcinogens;

Understanding human evolution (max 1)


9 (The HGP allows for the discovery of alleles/polymorphisms that can be) used to study
evolution through germline mutations in lineages ;
10 Migration/lineage of different population groups can be studied based on female genetic
inheritance (using alleles/polymorphisms in mitochondrial DNA) or male genetic
inheritance (using alleles/polymorphisms on Y chromosome);
11 (The HGP allows for the discovery of alleles/polymorphisms that can be) used to
compare breakpoints in the evolution of mutations with ages of populations and
historical events;

DNA forensics (max 1)


(The HGP allows for the discovery of alleles/polymorphisms that can be used to:)
12 Identify potential suspects whose DNA may match evidence left at crime scenes /
Exonerate persons wrongly accused of crimes ;
13 Identify crime / catastrophe victims ;
14 Establish paternity and other family relationships ;
15 Identify endangered /protected species (as an aid to wildlife officials) (could be used for
prosecuting poachers) ;
16 Match organ donors with recipients in transplant programs ;
17 Detect bacteria/other organisms that may pollute air/water/ soil/ food ;
18 Determine pedigree for seed/livestock breeds ;
19 Authenticate consumables such as caviar/wine/fish ;

ACJC 9648/03 Prelim 2013 [Turn over


90
16 For
Examiners
Use

Concerns max 4
Ethical and social concerns
i) Privacy and confidentiality of genetic information.
20 The issue of who owns (and controls) genetic information individual or
company/researcher/government;

ii) Fairness in the use of genetic information by insurers, employers, courts, schools,
adoption agencies, and the military, among others.
21 The issue of how insurers/employers/courts/schools/adoption agencies/military may
request for/use DNA testing/have access to personal genetic information to
discriminate people based on their genomes;

iii) Psychological impact and stigmatization due to an individual's genetic differences.


22 It is unclear how personal genetic information affects an individual/society's
perceptions of that individual / how genomic information affect members of minority
communities/ref to Psychological impact and stigmatization due to an individual's
genetic differences;

iv) Reproductive issues (max 1)


23 There is an issue of whether healthcare personnel are properly counseling parents
about the risks and limitations of genetic technology
24 The reliability/usefulness of fetal genetic testing has not been verified (in many cases);
25 To-be parents may have to make difficult decisions of whether to terminate pregnancy
due to presence of genetic disorder (especially one for which there is currently no cure
or treatment for) ;

v) Uncertainties associated with gene tests (max 1)


26 The issue of whether testing should be performed when no treatment is
available/treatment is extremely expensive and the patient cannot afford it;
27 The issue of whether parents have the right to have their children tested for adult-onset
diseases, (as there is potential for conflict between a parent's choice and a child's
welfare);
28 There is also the related issue of who has the right to determine whether
newborns/others who are incapable of valid consent (eg. mentally incapacitated) should
undergo genetic screening;
29 The genetic tests may only indicate a probability/chance/not a certainty of a particular
polymorphism/allele being associated with a disease or condition/disease developing;

vi) Conceptual and philosophical implications regarding human responsibility, free will vs
genetic determinism, and concepts of health and disease. (Max 1)
30 The issue of to what extent is a persons behaviour influenced by his genome (instead
of the environment/choices)
31 The issue of defining what is considered a disorder and what is within the limits of
normality.

vii) Commercialization of products including property rights (patents, copyrights, and


trade secrets) and accessibility of data and materials. (Max 1)
32 The issue of whether genes/DNA sequences can be patented by companies/individuals;
33 Patents could impede the development of diagnostics (tests)/therapeutics (treatments)
by third parties because of the costs associated with using patented research data;
34 Designer drugs developed through pharmocogenomics / genetic tests / treatments
discriminates against the poor as they may be expensive/unaffordable;
@1 mk, max 4

pt24/ 29 mark once


[Total: 20]
ACJC 9648/03 Prelim 2013 [Turn over
91
1

H2 ANDERSON JUNIOR COLLEGE


HIGHER 2

CANDIDATE
NAME

PDG
PDG INDEX NUMBER

BIOLOGY 9648/01
Paper 1 Multiple Choice 4 September 2013

1 hour 15 mins

Additional Materials: Multiple Choice Answer Paper 40 marks

READ THESE INSTRUCTIONS FIRST


Write in soft pencil.
Do not use staples, paper clips, highlighters, glue or correction fluid.
Write your name, PDG and identification number on the Answer Sheet.

There are forty questions on this paper. Answer all questions. For each question there are four
possible answers A, B, C and D.
Choose the one you consider correct and record your choice in soft pencil on the separate Answer
Sheet.

Each correct answer will score one mark. A mark will not be deducted for a wrong answer.
Any rough working should be done in this booklet.

Calculators may be used.

This document consists of 26 printed pages


92
2

1 Which row is correct for each of the components found on the cell surface membrane?

ion pump glycoprotein phospholipid cholesterol


A restrains the
a carrier protein has sugar chain contains 2 fatty acid
phospholipids
involves in active attached to the chains and a
movements at high
transport protein phosphate head only
temperatures
B contributes to the
pumps ions against is a receptor site for contains 4
fluidity of the
concentration gradient the signal ligand interconnecting rings
membrane
C cannot move freely may be saturated or
use ATP with the unsaturated fatty acid is a steroid
phospholipids chains
D unsaturated
has a hydrophilic hydrocarbon tails have is a lipid that does
for cell-to-cell
channel to allow ions kinks that allow the not contain fatty
recognition
to diffuse through molecules to pack acids
closely together
93
3
2 The electron micrograph shows two adjacent plant mesophyll cells with structures W to Z labeled.

Which of the following structure to function relationships is/ are correct?

Structure Function
I W Manufactures carbohydrates
II X Manufactures rRNA
III Y Regulates the movement of substances
IV Z Separates the cell from the external environment

A II only
B III only
C I and IV only
D II, III and IV only
94
4

3 Which molecule is found in glycogen?

4 The diagram represents molecules involved in protein synthesis.

Molecules 1,2 and 3 are formed as a result of

A dehydrogenation
B condensation
C enzymatic hydrolysis
D oxidation
95
5

5 An enzyme is completely denatured at 50 C. A fixed concentration of this enzyme is added to a fixed


concentration of its substrate. The time taken for completion of the reaction is measured at different
temperatures.

Which graph shows the results?


96
6
6 In an effort to identify allosteric inhibitors of caspases, close to 8,000 compounds were screened for
their ability to bind to a possible allosteric binding site in caspase 1 and inhibit the enzymes activity.
Each compound was designed to form a disulfide bond with a cysteine near the site in order to
stabilize the low-affinity interaction that is expected of an allosteric inhibitor.

Fig. 6.1.

X-ray diffraction analysis was used to determine the structure of caspase 1 when bound to one of the
inhibitors and to compare it with the active and inactive structures. The enzymes shape when one
such inhibitor was bound resembled the inactive caspase 1 more than the active form as shown in
Fig. 6.1.

Which of the following explains how the allosteric inhibitory site on caspase 1 controls enzymatic
activity?

A The inhibitor locks the enzyme in its inactive form.


B The inhibitor binds reversibly at a site other than the active site.
C The inhibitor binds to the active form and makes it inactive.
D The inhibitor allows caspase 1 to oscillate between catalytically active and inactive forms.
97
7
7 A cell (2n = 2) has some stages of nuclear divisions drawn which are randomly labelled I to VI.

The graph below represents one type of nuclear division.


Amount of
DNA

A C

Stages

Which stage corresponds to the different part of the graph correctly?

A Part A of graph stage III


B Part B of graph stage VI
C Part C of graph stage V
D Part D of graph stage IV

8 What is the basis for the difference in the synthesis of the leading and lagging strands of DNA
molecules?

1 The anti-parallel arrangement of the DNA strands.


2 The RNA primers are required to initiate DNA elongation.
3 DNA polymerase joins new nucleotides to the 3 end of the growing strand.
4 Helicase and single-stranded binding proteins work at the 5end of the DNA strand.

A 2 and 4
B 1 and 3
C 1 and 4
D 2 and 3
98
8
9 Three experiments were carried out to investigate the mode of DNA replication in bacteria.

Experiment 1: Bacteria were grown for many generations with only the light isotope of nitrogen,
14
N, and then allowed to replicate once with the light isotope, 15N.
Experiment 2: Bacteria were grown for many generations with only the heavy isotope of nitrogen,
15
N, and then allowed to replicate once with the light isotope, 14N.
Experiment 3: Bacteria were grown for many generations with only the heavy isotope of nitrogen,
15
N, and then allowed to replicate twice with the light isotope, 14N.

The figure shows possible DNA molecules U to Z and indicates the varying proportion of nitrogen
isotopes present in each DNA molecule.

Which of the following products shows the semi-conservative mode of DNA replication?

Experiment 1 Experiment 2 Experiment 3


A W W W and Y
B U W W and Y
C W W X and Z
D U W V and Z

10 An antibiotic, edeine, was isolated. It inhibits protein synthesis but has no effect on either DNA
synthesis or RNA synthesis. When added to a translation mixture containing fully intact organelles,
edeine stops haemoglobin translation after 10 seconds.
Analysis of the edeine-inhibited mixture by centrifugation showed that no polyribosomes remained by
the time protein synthesis had stopped. Instead, all the mRNA accumulated, together with small
ribosomal subunit and initiator tRNA.

What step in protein synthesis does edeine inhibit?

A It blocks translocation of the ribosome along the mRNA.


B It interferes with chain termination and release of peptide.
C It prevents formation of the translation initiation complex.
D It inhibits binding of amino-acyl-tRNAs to the A site of the ribosome.
99
9
11 The diagram below shows the process of translation.

Which of the following are correct?

1 The ribosome is translocating from right to left.


2 The diagram shows degeneracy of the genetic code.
3 The polypeptide does not dissociate immediately from the tRNA after histidine was
added.
4 The number of hydrogen bonds formed between the respective codons and
anticodons of the two tRNAs shown are equal.

A 1 and 4
B 1 and 2
C 2 and 3
D 2 and 4
100
10
12 The diagram below shows an electron micrograph of viruses entering into a T helper cell.

Which of following statements are true?

1 Most of the components found on the plasma membrane of X can be found on the envelope of
Y.
2 X and Y have the same type of nucleic acid.
3 Y enters by endocytosis.
4 Transduction occurs after Y successfully integrates its nucleic acid inside X.

A 1 only
B 1 and 3 only
C 2 and 4 only
D 3 and 4 only

13 Patients infected with the human immunodeficiency virus (HIV) tend to develop a cancer called
Kaposi Sarcoma. Which is the correct order of steps to explain how HIV causes this cancer?

1 Entry of Kaposi sarcoma herpes virus.


2 Double-stranded provirus integrated into host cell DNA.
3 Budding of HIV kills T helper cell.
4 Development of Acquired Immunodeficiency Disease Syndrome (AIDS).

A 1423
B 2314
C 1234
D 1243
101
11
14 The photomicrographs below show two different processes occurring in two different species of
bacteria.

Process 1 Process 2
Which of the following statements is/are true of both processes?
1 For both processes, only bacteria with genes that code for cytoplasmic bridge are involved.
2 Process 1 requires direct contact between 2 different bacteria whereas process 2 can occur
with 1 bacterium.
3 Process 1 will result in an increase in the number of identical bacteria whereas process 2 will
result in an increase in the number of different bacteria.
4 Both processes involve DNA replication.

A 1 and 3 only
B 2 and 4 only
C 1, 2 and 4 only
D All of the above

15 If glucose and lactose are both absent from the growth medium, and the lac structural genes are
expressed efficiently, what would you suspect about the condition of the E.coli cell?

1 The cell has a mutation in the operator of the lac operon.


2 The cell has a mutation in the lacI gene.
3 The cell has a mutation in the promoter of the lac structural gene.
4 The cell has a mutation in the Catabolite Activator Protein (CAP) binding site.

A 1 and 2 only
B 2 and 3 only
C 3 and 4 only
D 1, 2 and 4 only
102
12
16 The figure below shows the relative distributions of four types of mRNA molecules along the head-to-
tail axis in the cytoplasm of a Drosophila fertilized egg.

bicoid protein binds to the 3 untranslated region of caudal mRNA to interfere with the
interaction between 3 poly-A tail and 5 cap
bicoid protein is also a specific transcription factor that binds to the enhancer of the
hunchback gene
nanos protein inhibits translation of hunchback mRNA

Using the information given above, determine which one of the following graphs shows the relative
protein distribution along the head-to-tail axis of this fertilized egg.

B
103
13

17 In a laboratory, the 5 7-methylguanosine cap and the length of the polyA tail of an mRNA
transcript are altered in order to examine their effects on the number of functional proteins
translated.
mRNA Alteration
1 5 cap removed with 1000-adenine tail
2 5 cap removed with 500-adenine tail
3 5cap present with 500-adenine tail
4 5cap present with 300-adenine tail

Using the information given in the table above, predict the relative amount of protein
produced in the descending order (greatest least).

A 1,3,2,4
B 1,2,3,4
C 3,1,4,2
D 3,4,1,2
104
14
18 The diagram below illustrates the development of colorectal cancers.

Which of these statements can be inferred from this multistep model of carcinogenesis?

I Cells whose APC and -catenin genes are inactivated have lost density dependent inhibition.
II APC and -catenin genes are tumour suppressor genes.
III High levels of Ras protein are produced only when both copies of Ras gene are mutated.
IV Two copies of normal p53 alleles must be present to inhibit cell division.
V Gain-of-function mutation in COX-2 gene is a pre-requisite for the formation of carcinoma.

A I, II and III
B II, III and IV
C I, II and V
D II, III and V
105
15
19 Questions 19 and 20 refer to the following pedigree chart.

Blue eyes

Green eyes

Yellow eyes

Which of the following statements is true?

A Blue eyes are dominant.


B Individual 1 is homozygous.
C Individual 4 is homozygous.
D Yellow eyes are recessive to blue eyes.

20 Which of the following statements has a 50% chance of being true?

I One of the parents of individual 2 had the same phenotype as individual 2.


II If individual 10 married someone with green eyes, the first child would have green eyes.
III If individual 6 married a woman with green eyes, half of the offspring would have blue eyes.

A II only
B I and III only
C II and III only
D All of the above

21 In Shorthorn cattle, the allele for hornless is dominant to the allele for the presence of horns. Coat
colour can be red (genotype CRCR), roan (genotype CRCW) or white (genotype CWCW). A roan bull,
106
16
heterozygous for the hornless trait, is crossed with a cow of the same genotype.

What is the probability that a calf from this cross would have the same genotype as its parents?

A 1/4
B 3/8
C 1/2
D 3/4

22 A yellow flower, green-stemmed plant with the genotype YYrr was crossed with a white flower, red-
stemmed plant with the genotype yyRR. The F1 plants were allowed to self fertilise, A 2 test was
carried out on the results obtained for the F2 generation. Part of the table of values for 2 is shown.

The value of 2 in this investigation was 10.6.

What is the probability of this value of 2 and do the results fit the expected ratio?

results fit
probability
expected ratio
A between 0.01 and 0.05 No
B between 0.01 and 0.05 Yes
C between 0.05 and 0.1 Yes
D between 0.1 and 0.5 No
107
17

23 Fig. 23A and Fig. 23B show the results of two separate experiments on carbon dioxide fixation in
photosynthesis using a unicellular green alga and the radioactive isotope 14C to label carbon
dioxide. The algae were actively photosynthesizing before the start of both experiments.

Light Dark
R
Relative
quantity of
each
radioactive
labelled
substance
(Fig. 23A)
S

addition of time / s
14
CO2 14
CO2
Concentration
reduced
1% 14CO2 0.003% 14CO2

Relative
quantity of T
each
radioactive
labelled
substance
U
(Fig. 23B)

time / s

With reference to Fig. 23A and 23B, identify the graphs R, S, T, and U.

R S T U

A RuBP GP/PGA GP/PGA RuBP


B GP/PGA RuBP GP/PGA RuBP
C GP/PGA RuBP RuBP GP/PGA
D RuBP GP/PGA RuBP GP/PGA
108
18
24 Which of the following statements is true regarding cyclic and non-cyclic photophosphorylation?
A Cyclic photophosphorylation only requires light at 680nm to function optimally while non-
cyclic photophosphorylation requires light at both 680nm and 700nm to function optimally.
B Cyclic photophosphorylation is dependent on products generated through non-cyclic
photophosphorylation.
C The electrochemical gradient generated to drive ATP production by the non-cyclic is
different from that of the cyclic photophosphorylation.
D Only cyclic photophosphorylation can function in the absence of photosystem II.

25 When cyanide is bound to cytochrome oxidase, the cell can no longer produce ATP aerobically.
Which statement below best explains this?

A It prevents reduced NADP from being oxidised to NADP, hence preventing electron flow
down the electron transport chain.
B It prevents oxygen from accepting electrons and protons to form water, hence preventing
electron flow down the electron transport chain.
C It prevents photolysis from occurring to produce oxygen.
D It prevents pyruvate from being synthesised in glycolysis, hence stopping the Krebs cycle.

26 Which of the following shows correctly the differences in the products of glycolysis and Krebs cycle
per glucose molecule?

Glycolysis Krebs cycle

A 4ATP, 1 reduced NAD 4 reduced NAD, 1 reduced FAD, 1 ATP

B 4ATP, 2 reduced NAD 8 reduced NAD, 2reduced FAD, 2 ATP

C 2 net ATP, 1 reduced NAD 3 reduced NAD, 1 reduced FAD, 1 ATP

D 2 net ATP, 2 reduced NAD 6 reduced NAD, 2 reduced FAD, 2 ATP

27 Certain drugs act at synapses and affect the action of neurotransmitter substances. The table
shows the effects of four different drugs.

Drug Effect
1 Inhibits the enzyme cholinesterase
2 Prevents the release of acetylcholine
3 Opening of Ca2+ gates on the pre-synaptic membrane.
4 Binds to the post-synaptic membrane receptors.

Which of the two drugs would most likely to encourage the response from a skeletal response to an
electrical impulse arriving at the neuro-muscular junction?
A 1 and 3
B 1 and 4
C 2 and 3
D 2 and 4
109
19
28 What is the role of active transport in the transmission of nerve impulses by neurones?
A Propagates an action potential by pumping sodium ions across the membrane out of the
neurone.
B Propagates an action potential by pumping sodium ions across the membrane into the
neurone.
C Initiates the action potential needed for the transmission of an impulse by pumping calcium
ions out of the endoplasmic reticulum.
D Establishes the resting potential needed for the transmission of an impulse by pumping
sodium and potassium ions across the membrane.

29 Three proteins isolated from a human cell were investigated for their involvement in cell signalling
pathways.

Protein A Protein B Protein C


ATPase activity + - -
GTPase activity - - +
Kinase activity - + -
Trans-membrane Domain + + +

Key: (+) = present, (-) = absent

Which of the following correctly shows the identity of these three proteins?

Protein A Protein B Protein C


A Na+ - K+ Pumps Insulin receptor Glucagon Receptor
B Glucagon Receptor Insulin receptor Na+ - K+ Pump
C Na+ - K+ Pump Glucagon Receptor Insulin receptor
D Insulin receptor Glucagon Receptor Na+ - K+ Pump
110
20
30 The figure shows the relationship between glucose concentration in the blood and the
concentrations in the blood of the two hormones, Q and R.

Which hormones (Q and R) promote the processes shown?

conversion of glycogen to respiration of glucose in uptake of glucose by


glucose in liver cells liver cells muscle cells
A Q R Q
B Q R R
C R Q Q
D R Q R
111
21
31 Three pairs of enzymes and non-enzymatic proteins with similar primary structures are shown.

Enzyme Non-Enzymatic Protein

Which of the following correctly explains the type of structures for each pair?

A Analogous structures as they perform similar functions.


B Analogous structures as their tertiary structures are similar.
C Homologous structures as they perform different functions.
D Homologous structures as they are all polypeptides folded into a 3D configuration.
112
22
32 The volcanic islands that were formed millions of years ago, range from Kauai (the oldest) to
Hawaii (the youngest). Cytb gene from honey creepers and Yp1 gene from Drosophila were
analysed for divergence.

Which of the following statement is wrong?

A Geographical isolation prevented colonization of newly formed islands.


B There is a positive linear correlation between genetic distance and island age.
C Cytb gene and Yp1 gene are chosen because they are essential genes.
D Genetic drift is a factor that contributes to the increase in the mean genetic distance.

33 Which of the following describes the concept of gene flow correctly?

Plays a role in speciation Explains why population is Affects the strength of


the smallest unit to evolve selection pressure
A No No Yes
B No No No
C Yes Yes Yes
D Yes Yes No
113
23
34 The calls of six different species of frogs belonging to the Hyla genus are recorded and shown.

Which of the following can be inferred from the chart?

1 Frogs with more similar call patterns are more closely related.
2 A frog species can be identified by looking at the duration, intensity, and frequency of the
call.
3 The call of each species of frog acts as a selection pressure.
4 These calls are a form of isolation mechanism.

A 1 and 2 only
B 1, 2 and 4 only
C 1 and 3 only
D All of the above

35 Which primers should be paired to amplify, by PCR, the DNA flanked by the two indicated
sequences?
5 GGAATTCGT -- // -- AATGCTACC 3

A 5 ACGAATTCC 3 & 5 AATCGTACC 3


B 5 AATGCTACC 3 & 5 GGAATTCGT 3
C 5 GGTAGCATT 3 & 5 GGAATTCGT 3
D 5 ACGAATTCC 3 & 5 GGTAGCAAT 3
114
24
36 In people with a gene mutation where a base insertion has occurred, the protein formed has a very
different primary structure. To identify the presence of this mutant allele, DNA nucleotide sequences
were compared using gel electrophoresis. Read from the top to the bottom for the DNA sequence.
Heres an example:

Here is the DNA


sequence -
ATGCT

The electrophoresis result from the DNA of a normal allele of the gene is shown below.

Which diagram represents the DNA sequence for the mutant allele of this gene?
115
25
37 Genetic profiling can be used to determine the paternity of a child. DNA from the mother and the child
is cut into fragments, separated by electrophoresis and made visual using a stain. The diagram
shows the genetic profiles of a mother and child, and four possible fathers.

Who is the father?

38 Which of the following statements are TRUE about all stem cells?

1 Stem cells can be induced to differentiate by environmental signals.


2 Stem cells are easily isolated and propagated.
3 Stem cells are able to develop into whole organisms if implanted into the
womb.
4 Stem cells make more stem cells under appropriate conditions.

A 1 and 4
B 2 and 3
C 1, 3 and 4
D 1, 2, 3 and 4

39 Somaclonal variation may arise during in vitro culturing of plant tissues because

A crossing over occurs between homologous chromosomes during meiosis in the cultured
plant cells, leading to genetic recombination.

B some cells are committed to differentiation into specialized plant tissues, therefore may
silence the expression of certain genes.

C of increased rate of mutation due to high growth rate of cells which is induced by the large
amount of growth regulators used.

D pollinating insects inserts DNA sequences containing genes from other plant cells into the
cultured cells genome, leading to genetic variation.
116
26
40 Corn is a major crop grown in Europe. In the past, it was either ruined by attack from the corn borer
larvae or intensively sprayed with pesticides each year. The biotech company Novartis gained
approval to insert a Bt gene into corn. This gene codes for a protein that kills the larvae feeding on
the corn. The genetically engineered Bt corn initially thrived without the addition of any pesticides, but
later suffered damage from pests again.
Which of the following are possible reasons why the Bt corn is again under attack from pests?

I A strain of resistant larvae has emerged as a chance mutation. The frequency of the gene
conferring resistance as well as the number of larvae with resistance has increased. The Bt
gene is now ineffective.
II Corn is being attacked by other pests which are not killed by the Bt gene protein.
III The Bt allele has undergone insertional mutagenesis. This mutagenesis has had a selective
advantage in seed fertility.

A I and II only
B I and III only
C II and III only
D All of the above
117
1

H2 ANDERSON JUNIOR COLLEGE


HIGHER 2

CANDIDATE
NAME

PDG
PDG INDEX NUMBER

BIOLOGY 9648/01
Paper 1 Multiple Choice 4 September 2013

1 hour 15 mins

Additional Materials: Multiple Choice Answer Paper 40 marks

READ THESE INSTRUCTIONS FIRST


Write in soft pencil.
Do not use staples, paper clips, highlighters, glue or correction fluid.
Write your name, PDG and identification number on the Answer Sheet.

There are forty questions on this paper. Answer all questions. For each question there are four
possible answers A, B, C and D.
Choose the one you consider correct and record your choice in soft pencil on the separate Answer
Sheet.

Each correct answer will score one mark. A mark will not be deducted for a wrong answer.
Any rough working should be done in this booklet.

Calculators may be used.

This document consists of 26 printed pages


118
2

1 Which row is correct for each of the components found on the cell surface membrane?

ion pump glycoprotein phospholipid cholesterol


A restrains the
a carrier protein has sugar chain contains 2 fatty acid
phospholipids
involves in active attached to the chains and a
movements at high
transport protein phosphate head only
temperatures
B contributes to the
pumps ions against is a receptor site for contains 4
fluidity of the
concentration gradient the signal ligand interconnecting rings
membrane
C cannot move freely may be saturated or
use ATP with the unsaturated fatty acid Is a steroid
phospholipids chains
D unsaturated
has a hydrophilic hydrocarbon tails have is a lipid that does
for cell-to-cell
channel to allow ions kinks that allow the not contain fatty
recognition
to diffuse through molecules to pack acids
closely together
119
3

2 The electron micrograph shows two adjacent plant mesophyll cells with structures W to Z labeled.

Which of the following structure to function relationships is/ are correct?


Structure Function
I W Manufactures carbohydrates
II X Manufactures rRNA
III Y Regulates the movement of substances
IV Z Separates the cell from the external environment

A II only
B III only
C I and IV only
D II, III and IV only
120
4

3 Which molecule is found in glycogen?

4 The diagram represents molecules involved in protein synthesis.

Molecules 1,2 and 3 are formed as a result of


A dehydrogenation
B condensation
C enzymatic hydrolysis
D oxidation
121
5
5 An enzyme is completely denatured at 50 C. A fixed concentration of this enzyme is added to a fixed
concentration of its substrate. The time taken for completion of the reaction is measured at different
temperatures.

Which graph shows the results?


122
6
6 In an effort to identify allosteric inhibitors of caspases, close to 8,000 compounds were screened for
their ability to bind to a possible allosteric binding site in caspase 1 and inhibit the enzymes activity.
Each compound was designed to form a disulfide bond with a cysteine near the site in order to
stabilize the low-affinity interaction that is expected of an allosteric inhibitor.

Fig. 6.1.

X-ray diffraction analysis was used to determine the structure of caspase 1 when bound to one of the
inhibitors and to compare it with the active and inactive structures. The enzymes shape when one
such inhibitor was bound resembled the inactive caspase 1 more than the active form as shown in
Fig. 6.1.

Which of the following explains how the allosteric inhibitory site on caspase 1 controls enzymatic
activity?

A The inhibitor locks the enzyme in its inactive form.


B The inhibitor binds reversibly at a site other than the active site.
C The inhibitor binds to the active form and makes it inactive.
D The inhibitor allows caspase 1 to oscillate between catalytically active and inactive forms.
123
7
7 A cell (2n = 2) has some stages of nuclear divisions drawn which are randomly labelled I to VI.

The graph below represents one type of nuclear division.


Amount of
DNA

A C

Stages

Which stage corresponds to the different part of the graph correctly?

A Part A of graph stage III


B Part B of graph stage VI
C Part C of graph stage V
D Part D of graph stage IV

8 What is the basis for the difference in the synthesis of the leading and lagging strands of DNA
molecules?

1 The anti-parallel arrangement of the DNA strands.


2 The RNA primers are required to initiate DNA elongation.
3 DNA polymerase joins new nucleotides to the 3 end of the growing strand.
4 Helicase and single-stranded binding proteins work at the 5end of the DNA strand.

A 2 and 4
B 1 and 3
C 1 and 4
D 2 and 3
124
8
9 Three experiments were carried out to investigate the mode of DNA replication in bacteria.

Experiment 1: Bacteria were grown for many generations with only the light isotope of nitrogen,
14
N, and then allowed to replicate once with the heavy isotope, 15N.
Experiment 2: Bacteria were grown for many generations with only the heavy isotope of nitrogen,
15
N, and then allowed to replicate once with the light isotope, 14N.
Experiment 3: Bacteria were grown for many generations with only the heavy isotope of nitrogen,
15
N, and then allowed to replicate twice with the light isotope, 14N.

The figure shows possible DNA molecules U to Z and indicates the varying proportion of nitrogen
isotopes present in each DNA molecule.

Which of the following products shows the semi-conservative mode of DNA replication?

Experiment 1 Experiment 2 Experiment 3


A W W W and Y
B U W W and Y
C W W X and Z
D U W V and Z

10 An antibiotic, edeine, was isolated. It inhibits protein synthesis but has no effect on either DNA
synthesis or RNA synthesis. When added to a translation mixture containing fully intact organelles,
edeine stops haemoglobin translation after 10 seconds.
Analysis of the edeine-inhibited mixture by centrifugation showed that no polyribosomes remained by
the time protein synthesis had stopped. Instead, all the mRNA accumulated, together with small
ribosomal subunit and initiator tRNA.

What step in protein synthesis does edeine inhibit?

A It blocks translocation of the ribosome along the mRNA.


B It interferes with chain termination and release of peptide.
C It prevents formation of the translation initiation complex.
D It inhibits binding of amino-acyl-tRNAs to the A site of the ribosome.
125
9

11 The diagram below shows the process of translation.

Which of the following are correct?

1 The ribosome is translocating from right to left.


2 The diagram shows degeneracy of the genetic code.
3 The polypeptide does not dissociate immediately from the tRNA after histidine was
added.
4 The number of hydrogen bonds formed between the respective codons and
anticodons of the two tRNAs shown are equal.

A 1 and 4
B 1 and 2
C 2 and 3
D 2 and 4
126
10

12 The diagram below shows an electron micrograph of viruses entering into a T helper cell.

Which of following statements are true?

1 Most of the components found on the plasma membrane of X can be found on the envelope of
Y.
2 X and Y have the same type of nucleic acid.
3 Y enters by endocytosis.
4 Transduction occurs after Y successfully integrates its nucleic acid inside X.

A 1 only
B 1 and 3 only
C 2 and 4 only
D 3 and 4 only

13 Patients infected with the human immunodeficiency virus (HIV) tend to develop a cancer called
Kaposi Sarcoma. Which is the correct order of steps to explain how HIV causes this cancer?

1 Entry of Kaposi sarcoma herpes virus.


2 Double-stranded provirus integrated into host cell DNA.
3 Budding of HIV kills T helper cell.
4 Development of Acquired Immunodeficiency Disease Syndrome (AIDS).

A 1423
B 2314
C 1234
D 1243
127
11

14 The photomicrographs below show two different processes occurring in two different species of
bacteria.

Process 1 Process 2
Which of the following statements is/are true of both processes?
1 For both processes, only bacteria with genes that code for cytoplasmic bridge are involved.
2 Process 1 requires direct contact between 2 different bacteria whereas process 2 can occur
with 1 bacterium.
3 Process 1 will result in an increase in the number of identical bacteria whereas process 2 will
result in an increase in the number of different bacteria.
4 Both processes involve DNA replication.

A 1 and 3 only
B 2 and 4 only
C 1, 2 and 4 only
D All of the above

15 If glucose and lactose are both absent from the growth medium, and the lac structural genes are
expressed efficiently, what would you suspect about the condition of the E.coli cell?

1 The cell has a mutation in the operator of the lac operon.


2 The cell has a mutation in the lacI gene.
3 The cell has a mutation in the promoter of the lac structural gene.
4 The cell has a mutation in the Catabolite Activator Protein (CAP) binding site.

A 1 and 2 only
B 2 and 3 only
C 3 and 4 only
D 1, 2 and 4
128
12

16 The figure below shows the relative distributions of four types of mRNA molecules along the head-to-
tail axis in the cytoplasm of a Drosophila fertilized egg.

bicoid protein binds to the 3 untranslated region of caudal mRNA to interfere with the
interaction between 3 poly-A tail and 5 cap
bicoid protein is also a specific transcription factor that binds to the enhancer of the
hunchback gene
nanos protein inhibits translation of hunchback mRNA

Using the information given above, determine which one of the following graphs shows the relative
protein distribution along the head-to-tail axis of this fertilized egg.

B
129
13

17 In a laboratory, the 5 7-methylguanosine cap and the length of the polyA tail of an mRNA
transcript are altered in order to examine their effects on the number of functional proteins
translated.
mRNA Alteration
1 5 cap removed with 1000-adenine tail
2 5 cap removed with 500-adenine tail
3 5cap present with 500-adenine tail
4 5cap present with 300-adenine tail

Using the information given in the table above, predict the relative amount of protein
produced in the descending order (greatest least).

A 1,3,2,4
B 1,2,3,4
C 3,1,4,2
D 3,4,1,2
130
14
18 The diagram below illustrates the development of colorectal cancers.

Which of these statements can be inferred from this multistep model of carcinogenesis?

I Cells whose APC and -catenin genes are inactivated have lost density dependent inhibition.
II APC and -catenin genes are tumour suppressor genes.
III High levels of Ras protein are produced only when both copies of Ras gene are mutated.
IV Two copies of normal p53 alleles must be present to inhibit cell division.
V Gain-of-function mutation in COX-2 gene is a pre-requisite for the formation of carcinoma.

A I, II and III
B II, III and IV
C I, II and V
D II, III and V
131
15
19 Questions 19 and 20 refer to the following pedigree chart.

Blue eyes

Green eyes

Yellow eyes

Which of the following statements is true?

A Blue eyes are dominant.


B Individual 1 is homozygous.
C Individual 4 is homozygous.
D Yellow eyes are recessive to blue eyes.

20 Which of the following statements has a 50% chance of being true?

I One of the parents of individual 2 had the same phenotype as individual 2.

II If individual 10 married someone with green eyes, the first child would have green eyes.

III If individual 6 married a woman with green eyes, half of the offspring would have blue eyes.

A II only
B I and III only
C II and III only
D All of the above
132
16
21 In Shorthorn cattle, the allele for hornless is dominant to the allele for the presence of horns. Coat
colour can be red (genotype CRCR), roan (genotype CRCW) or white (genotype CWCW). A roan bull,
heterozygous for the hornless trait, is crossed with a cow of the same genotype.

What is the probability that a calf from this cross would have the same genotype as its parents?

A 1/4
B 3/8
C 1/2
D 3/4

22 A yellow flower, green-stemmed plant with the genotype YYrr was crossed with a white flower,
red-stemmed plant with the genotype yyRR. The F1 plants were allowed to self fertilise, A 2
test was carried out on the results obtained for the F2 generation. Part of the table of values for
2 is shown.

The value of 2 in this investigation was 10.6.


What is the probability of this value of 2 and do the results fit the expected ratio?

results fit
probability
expected ratio
A between 0.01 and 0.05 No
B between 0.01 and 0.05 Yes
C between 0.05 and 0.1 Yes
D between 0.1 and 0.5 No
133
17

23 Fig. 23A and Fig. 23B show the results of two separate experiments on carbon dioxide fixation in
photosynthesis using a unicellular green alga and the radioactive isotope 14C to label carbon
dioxide. The algae were actively photosynthesizing before the start of both experiments.

Light Dark
R
Relative
quantity of
each
radioactive
labelled
substance
(Fig. 23A)
S

addition of time / s
14
CO2 14
CO2
Concentration
reduced
1% 14CO2 0.003% 14CO2

Relative
quantity of T
each
radioactive
labelled
substance
U
(Fig. 23B)

time / s

With reference to Fig. 23A and 23B.identify the graphs R, S, T, and U.

R S T U

A RuBP GP/PGA GP/PGA RuBP


B GP/PGA RuBP GP/PGA RuBP
C GP/PGA RuBP RuBP GP/PGA
D RuBP GP/PGA RuBP GP/PGA
134
18
24 Which of the following statements is true regarding cyclic and non-cyclic photophosphorylation?
A Cyclic photophosphorylation only requires light at 680nm to function optimally while non-
cyclic photophosphorylation requires light at both 680nm and 700nm to function optimally.
B Cyclic photophosphorylation is dependent on products generated through non-cyclic
photophosphorylation.
C The electrochemical gradient generated to drive ATP production by the non-cyclic is
different from that of the cyclic photophosphorylation.
D Only cyclic photophosphorylation can function in the absence of photosystem II.

25 When cyanide is bound to cytochrome oxidase, the cell can no longer produce ATP aerobically.
Which statement below best explains this?

A It prevents reduced NADP from being oxidised to NADP, hence preventing electron flow
down the electron transport chain.
B It prevents oxygen from accepting electrons and protons to form water, hence preventing
electron flow down the electron transport chain.
C It prevents photolysis from occurring to produce oxygen.
D It prevents pyruvate from being synthesised in glycolysis, hence stopping the Krebs cycle.

26 Which of the following shows correctly the differences in the products of glycolysis and Krebs cycle
per glucose molecule?

Glycolysis Krebs cycle

A 4ATP, 1 reduced NAD 4 reduced NAD, 1 reduced FAD, 1 ATP

B 4ATP, 2 reduced NAD 8 reduced NAD, 2reduced FAD, 2 ATP

C 2 net ATP, 1 reduced NAD 3 reduced NAD, 1 reduced FAD, 1 ATP

D 2 net ATP, 2 reduced NAD 6 reduced NAD, 2 reduced FAD, 2 ATP

27 Certain drugs act at synapses and affect the action of neurotransmitter substances. The table
shows the effects of four different drugs.

Drug Effect
1 Inhibits the enzyme cholinesterase
2 Prevents the release of acetylcholine
3 Opening of Ca2+ gates on the pre-synaptic membrane.
4 Binds to the post-synaptic membrane receptors.

Which of the two drugs would most likely to encourage the response from a skeletal response to an
electrical impulse arriving at the neuro-muscular junction?
A 1 and 3
B 1 and 4
C 2 and 3
D 2 and 4
135
19
28 What is the role of active transport in the transmission of nerve impulses by neurones?
A Propagates an action potential by pumping sodium ions across the membrane out of the
neurone.
B Propagates an action potential by pumping sodium ions across the membrane into the
neurone.
C Initiates the action potential needed for the transmission of an impulse by pumping calcium
ions out of the endoplasmic reticulum.
D Establishes the resting potential needed for the transmission of an impulse by pumping
sodium and potassium ions across the membrane.

29 Three proteins isolated from a human cell were investigated for their involvement in cell signalling
pathways.

Protein A Protein B Protein C


ATPase activity + - -
GTPase activity - - +
Kinase activity - + -
Trans-membrane Domain + + +

Key: (+) = present, (-) = absent

Which of the following correctly shows the identity of these three proteins?

Protein A Protein B Protein C


A Na+ - K+ Pumps Insulin receptor Glucagon Receptor
B Glucagon Receptor Insulin receptor Na+ - K+ Pump
C Na+ - K+ Pump Glucagon Receptor Insulin receptor
D Insulin receptor Glucagon Receptor Na+ - K+ Pump
136
20
30 The figure shows the relationship between glucose concentration in the blood and the
concentrations in the blood of the two hormones, Q and R.

Which hormones (Q and R) promote the processes shown?

conversion of glycogen to respiration of glucose in uptake of glucose by


glucose in liver cells liver cells muscle cells
A Q R Q
B Q R R
C R Q Q
D R Q R
137
21
31 Three pairs of enzymes and non-enzymatic proteins with similar primary structures are shown.

Enzyme Non-Enzymatic Protein

Which of the following correctly explains the type of structures for each pair?

A Analogous structures as they perform similar functions.


B Analogous structures as their tertiary structures are similar.
C Homologous structures as they perform different functions.
D Homologous structures as they are all polypeptides folded into a 3D configuration.
138
22
32 The volcanic islands that were formed millions of years ago, range from Kauai (the oldest) to
Hawaii (the youngest). Cytb gene from honey creepers and Yp1 gene from Drosophila were
analysed for divergence.

Which of the following statement is wrong?

A Geographical isolation prevented colonization of newly formed islands.


B There is a positive linear correlation between genetic distance and island age.
C Cytb gene and Yp1 gene are chosen because they are essential genes.
D Genetic drift is a factor that contributes to the increase in the mean genetic distance.

33 Which of the following describes the concept of gene flow correctly?

Plays a role in speciation Explains why population is Affects the strength of


the smallest unit to evolve selection pressure
A No No Yes
B No No No
C Yes Yes Yes
D Yes Yes No
139
23
34 The calls of six different species of frogs belonging to the Hyla genus are recorded and shown.

Which of the following can be inferred from the chart?

1 Frogs with more similar call patterns are more closely related.
2 A frog species can be identified by looking at the duration, intensity, and frequency of the
call.
3 The call of each species of frog acts as a selection pressure.
4 These calls are a form of isolation mechanism.

A 1 and 2 only
B 1, 2 and 4 only
C 1 and 3 only
D All of the above

35 Which primers should be paired to amplify, by PCR, the DNA flanked by the two indicated
sequences?
5 GGAATTCGT -- // -- AATGCTACC 3

A 5 ACGAATTCC 3 & 5 AATCGTACC 3


B 5 AATGCTACC 3 & 5 GGAATTCGT 3
C 5 GGTAGCATT 3 & 5 GGAATTCGT 3
D 5 ACGAATTCC 3 & 5 GGTAGCAAT 3
140
24
36 In people with a gene mutation where a base insertion has occurred, the protein formed has a very
different primary structure. To identify the presence of this mutant allele, DNA nucleotide sequences
were compared using gel electrophoresis. Read from the top to the bottom for the DNA sequence.
Heres an example:

Here is the DNA


sequence -
ATGCT

The electrophoresis result from the DNA of a normal allele of the gene is shown below.

Which diagram represents the DNA sequence for the mutant allele of this gene?
141
25
37 Genetic profiling can be used to determine the paternity of a child.

DNA from the mother and the child is cut into fragments, separated by electrophoresis and made
visual using a stain.

The diagram shows the genetic profiles of a mother and child, and four possible fathers.

Who is the father? B

38 Which of the following statements are TRUE about all stem cells?

1 Stem cells can be induced to differentiate by environmental signals.


2 Stem cells are easily isolated and propagated.
3 Stem cells are able to develop into whole organisms if implanted into the
womb.
4 Stem cells make more stem cells under appropriate conditions.

A 1 and 4
B 2 and 3
C 1, 3 and 4
D 1, 2, 3 and 4

39 Somaclonal variation may arise during in vitro culturing of plant tissues because

A crossing over occurs between homologous chromosomes during meiosis in the cultured
plant cells, leading to genetic recombination.

B some cells are committed to differentiation into specialized plant tissues, therefore may
silence the expression of certain genes.

C of increased rate of mutation due to high growth rate of cells which is induced by the large
amount of growth regulators used.

D pollinating insects inserts DNA sequences containing genes from other plant cells into the
cultured cells genome, leading to genetic variation.
142
26

40 Corn is a major crop grown in Europe. In the past, it was either ruined by attack from the corn borer
larvae or intensively sprayed with pesticides each year. The biotech company Novartis gained
approval to insert a Bt gene into corn. This gene codes for a protein that kills the larvae feeding on
the corn. The genetically engineered Bt corn initially thrived without the addition of any pesticides, but
later suffered damage from pests again.
Which of the following are possible reasons why the Bt corn is again under attack from pests?

I A strain of resistant larvae has emerged as a chance mutation. The frequency of the gene
conferring resistance as well as the number of larvae with resistance has increased. The Bt
gene is now ineffective.
II Corn is being attacked by other pests which are not killed by the Bt gene protein.
III The Bt allele has undergone insertional mutagenesis. This mutagenesis has had a selective
advantage in seed fertility.

A I and II only
B I and III only
C II and III only
D All of the above
143
27
Paper 1 Answer scheme

Qns Ans Remarks Qns Ans Remarks


1 B 21 A
2 A 22 A
3 B 23 D
4 B 24 D
5 C 25 B
6 A 26 D
7 C 27 C
8 B 28 D
9 A 29 A
10 C 30 B
11 C 31 C
12 A 32 A
13 B 33 D
14 B 34 B
15 A 35 C
16 D 36 B
17 D 37 B
18 C 38 A
19 C 39 C
20 A 40 D
144

H2 ANDERSON JUNIOR COLLEGE


HIGHER 2

CANDIDATE
NAME

PDG
PDG INDEX NUMBER

BIOLOGY
9648/02
Paper 2 Core Paper 17 September 2013

2 hours

Additional Materials: Answer Paper

READ THESE INSTRUCTIONS FIRST


Write your name and PD group on all the work you hand in.
Write in dark blue or black pen. For Examiners Use
You may use a soft pencil for any diagrams, graph or rough working.
Do not use paper clips, highlighters, glue or correction fluid. Section A 80

1
Section A
Answer all questions. 2

Section B 3
Answer any one question.
4
All working for numerical answers must be shown. 5
At the end of the examination, fasten all your work securely together.
The number of marks is given in brackets [ ] at the end of each question or 6
part question.
7
Calculators may be used
Section B 20

8/9

Total 100

This document consists of 19 printed pages.


145

Section A
Answer all the questions in this section.

1(a) In a DNA structure, what do 5 and 3 represent. [2]

(b) The nitrogenous bases of the double helix are paired in specific combinations:

adenine (A) with thymine (T)


guanine (G) with cytosine (C)

It was mainly by trial and error that Watson and Crick arrived at this key feature of DNA. At first,
Watson imagined that the bases paired like with likefor example, A with A. But this model was
inconsistent with the uniform width of the double helix.

Explain why the earlier DNA model would have inconsistent width. [3]

(c) DNA replicates semi-conservatively by the two strands separating and forming templates for the
synthesis of two new strands.

In addition to the template provided by the DNA, describe the role of two other types of factors that
are required for DNA replication. [2]

.
146

3
(d) Just as a DNA strand provides a template for making a new complementary strand during DNA
replication, it also can serve as a template for transcription.

Fig. 1.1 is a diagrammatic representation of the process of transcription.

Fig. 1.1

Identify molecule E, F and H. [1]

(i) Describe how the structure of molecule E is adapted to its role in transcription. [3]

.
147

(ii) Describe one way by which the structure of molecule(s) F, which is / are adapted for [1]
transcription, differs from that of amino acids, which are adapted for translation.

[Total : 12]
148

2 Influenza A viruses are divided into subtypes based on two proteins on the surface of the
virus: haemagglutinin (HA) and neuraminidase (NA). The influenza A (H7N9) virus designation of
H7N9 identifies it as having HA of the H7 subtype and NA of the N9 subtype.

(a) Describe the roles of haemagglutinin (HA) and neuraminidase (NA) in the reproductive cycle of the
influenza (H7N9) virus. [2]

(b) (i) The influenza genome comprises genes for RNA polymerase. How is the virus RNA
polymerase different from the host RNA polymerase? [1]

(ii) Explain the role of the viral RNA polymerase in the reproductive cycle. [3]

(c) H7N9 is an avian influenza virus but it is also able to infect mammals.

Suggest changes that may have occurred to give H7N9 greater ability to infect mammals. [2]

.
149

6
(d) While the influenza genome comprises 8 distinct linear segments of negative sense ssRNA,
bacterial genome consists of chromosome and plasmid. Operons are needed for the regulation of
gene expression.

Suggest why operons give bacteria a selective advantage. [3]

[Total : 11]
150

3 In normal person, the fragile X mental retardation protein (FMRP), regulates the synthesis of neurone
proteins by stopping ribosomal translocation on target mRNAs. Fig 3.1 shows how patients with fragile
X syndrome (FXS) have non-functional FMRP, resulting in accelerated synaptic protein synthesis that
leads to abnormal synaptic function and intellectual disability.

NonfunctionalFMRP(FXS)

Fig. 3.1

(a) Describe how polysomes are formed. [2]

..

..

..

(b) (i) State the level of control by FMRP on synaptic protein expression. [1]


151

(ii) Describe one other control mechanism of a similar level as (b)(i). [2]

(iii) Explain how FMRP stops the ribosome from translocation. [2]

(c) Initial gene therapy trial uses a short stretch of anti-sense RNA. Explain how this will help to
reduce the symptom of FXS. [2]

..

..

..

..

(d) Synaptic proteins that are folded wrongly are usually degraded.

Suggest the advantages of degrading proteins that are wrongly folded. [2]

..

..

..

[Total : 11]
152

4 Severe combined immunodeficiency (SCID) results from defects in signaling downstream of


cytokine receptors that contain the common cytokine-receptor -chain (c). c is a component of
the receptors (R) for many interleukin such as interleukin-2 (IL-2) and IL-7.

Fig. 4.1 shows the events in the JAK-STAT cell signaling pathway when cytokine binds and this in
turn leads to an activated receptor. Fig. 4.2 shows two causes that could lead to a non-functional
receptor.

Fig. 4.1 Fig. 4.2

(a) Explain why transmembrane proteins such as the cytokine receptor are especially suited for a role
in cell signalling. [2]

(b) With reference to Fig. 4.1, suggest when happens after cytokine binds to the receptor. [2]

.
153

10

(c) With reference to Fig. 4.2, explain how a change in the sequence of the DNA nucleotide may
affect the signaling downstream of cytokine receptors. [5]

(d) Proteins synthesized by ribosomes attached to the endoplasmic reticulum may be embedded in
the cell surface membrane. Outline the route taken by such a protein. [3]

[Total: 12]
154

11

5 Chickens have a structure called a comb on their heads. The drawings show two types of comb.

Fig. 5.1

The shape of the comb is controlled by two alleles of one gene. The allele for pea comb, A, is
dominant to the allele for single comb, a. The colour of chicken eggs is controlled by two alleles of
a different gene. The allele for blue eggs, B, is dominant to the allele for white eggs, b.

The genes for comb shape and egg colour are situated on the same chromosome.

A farmer crossed a chicken that had pea comb and produced blue eggs with a chicken that had a
single comb and produced white eggs. The chicken that had a pea comb and produced blue eggs
are the offspring of the cross between pure bred pea comb, blue eggs and pure bred single comb,
white eggs.

(a) Construct a genetic diagram to illustrate the above cross. [3]


155

12

(b) The farmer could identify which of the female offspring from this cross would eventually produce
blue eggs. Explain how. [2]

(c) However, in certain crosses, the farmer was not able to identify the female offspring which will
produce blue eggs. Explain why. [2]

.
156

13

(e) In chickens the males are the homogametic sex while the females are the heterogametic sex.

A gene on the X chromosome controls the rate of feather production. The allele for slow feather
production, F, is dominant to the allele for rapid feather production, f.

A farmer made a cross between two chickens with known genotypes. He chose these chickens so
that he could tell the sex of the offspring soon after they hatched by looking at their feathers.

Which of the crosses shown in the table did he make? Explain your answer.

Fig. 5.3 [2]

(f) Female chickens are more likely than male chickens to show recessive sex-linked characteristics.
Explain why. [2]

[Total: 11]
157

14

6 Table 6.1 shows the approximate percentage of protein in different membranes.

Table 6.1

Organelle Membrane Protein as a percentage of dry weight


Chloroplast Outer membrane 54
Chloroplast Inner membrane 70
Mitochondrion Outer membrane 55
Mitochondrion Inner membrane 80

(a) Describe one way in which the distribution of protein in the inner and outer membranes of these
two organelles is similar.

Suggest a reason for this similarity. [2]

..

..

..

..

..

(b) (i) Atrazine is a widely used herbicide (weedkiller). It binds to a chloroplast protein involved in
electron transfer between photosystems.

Explain how atrazine works as an effective herbicide. [4]


158

15

(ii) Maize plants are insensitive to atrazine.

The cytoplasm of maize plant cells contains a tripeptide which binds to atrazine. The product
is then transported to the vacuole.

Suggest why maize plants are insensitive to atrazine. [2]

(c) At the end of photosynthesis, glyceraldehyde 3-phosphate (3-carbon sugar) is produced by the [3]
Calvin cycle, which is subsequently converted into starch for storage.

Discuss the suitability of starch as a storage compound in plants.

..

..

..

..

..

..

[Total : 11]
159

16

7 The marine threespine sticklebacks, Gasterosteus aculeatus is a freshwater fish living in the
lakes of British Columbia, Canada as shown in Fig. 7.1.

Fig. 7.1

In order to investigate the process of speciation in these populations, three small lakes were
studied. Each lake contained two varieties of stickleback: a large, bottom-dwelling variety that
fed on invertebrates near the shore and a small, plankton-eating variety that lived in the open
water. The probability of breeding between pairs of individuals was measured under laboratory
conditions in the following breeding combinations:

I different varieties from the same lake


II different varieties from different lakes
III same variety from different lakes
IV same variety from the same lake

The data are summarized in Fig. 7.1 below.

Fig. 7.1
160

17

(a) (i) Identify the highest and lowest probabilities of breeding for individuals of the same variety
from different lakes. [1]

..

...

(ii) With reference to Fig. 7.1, describe the differences in probability of breeding between
individuals from the same lake. [2]

..

..

..

..

(b) Scientists concluded that speciation is taking place in these populations. Explain the type of
speciation as shown in Fig. 7.1. [4]

..

..

..

..

..

..

(c) With reference to Fig. 7.1, explain why all the individuals are still considered the same species. [2]

..

..

..
161

18

(d) The freshwater lakes also contain many different types of parasites that infect the marine
threespine sticklebacks. Suggest why these parasites help to speed up speciation of the marine
threespine sticklebacks. [3]

..

..

..

..

[Total : 12]
162

19

Section B
Answer one question.
Write your answers on the separate answer paper provided.
Your answer should be illustrated by large, clearly labeled diagrams, where appropriate.
Your answers must be in continuous prose, where appropriate.
Your answers must be set out in section (a), (b) etc., as indicated in the question.

8 (a) Explain how the positive and negative control of the lac operon affect bacteria growth. [10]

(b) Describe how regulation of blood glucose level in humans involves cAMP. [5]

(c) Explain the significance of the steps in glycolysis. [5]

9 (a) Describe the differences between cellulose synthase and cellulose. [5]

(b) With reference to photosynthesis and cellular respiration, explain the importance of
compartmentalization within a plant cell. [10]

(c) Explain the role of meiosis in natural selection. [5]


163

H2 ANDERSON JUNIOR COLLEGE


HIGHER 2

CANDIDATE ANSWERS
NAME

PDG
PDG INDEX NUMBER

BIOLOGY
9648/02
Paper 2 Core Paper 17 September 2013

2 hours

Additional Materials: Answer Paper

READ THESE INSTRUCTIONS FIRST


Write your name and PD group on all the work you hand in.
Write in dark blue or black pen. For Examiners Use
You may use a soft pencil for any diagrams, graph or rough working.
Do not use paper clips, highlighters, glue or correction fluid. Section A 80

1
Section A
Answer all questions. 2

Section B 3
Answer any one question.
4
All working for numerical answers must be shown. 5
At the end of the examination, fasten all your work securely together.
The number of marks is given in brackets [ ] at the end of each question or 6
part question.
7
Calculators may be used
Section B 20

8/9
Total 100

This document consists of 19 printed pages.


164
2

Section A
Answer all the questions in this section.

1(a) Explain why replication only occurs in a 5 to 3 direction. [2]


Shape of active site of DNA polymerase;
Needs 3OH end to initiate;
Primers only add at 3;

In a DNA structure, what do 5 and 3 represent. [2]


5 represents end (of strand of nucleotide) with free carbon 5 (on deoxyribose) having
phosphate group
3 represents end (of strand of nucleotide) with free carbon 3 (on deoxyribose) having
hydroxyl group
Antiparallel
(b) The nitrogenous bases of the double helix are paired in specific combinations: [3]
adenine (A) with thymine (T)
guanine (G) with cytosine (C)

It was mainly by trial and error that Watson and Crick arrived at this key feature of DNA. At first,
Watson imagined that the bases paired like with likefor example, A with A. But this model was
inconsistent with the uniform width of the double helix.

Explain why the earlier DNA model would have inconsistent width.
purines (A and G) are twice as wide as pyrimidines (C and T)
purines (A and G) are nitrogenous bases with two organic rings
pyrimidines (C and T) are single ring nitrogenous bases.
A purine-purine pair is too wide and a pyrimidine-pyrimidine pair too narrow to account for a
uniform (2-nm) diameter of the double helix.
Always pairing a purine with a pyrimidine results in a uniform width.
[Any 3]

(c) DNA replicates semi-conservatively by the two strands separating and forming templates for the
synthesis of two new strands.
[2]
In addition to the template provided by the DNA, describe the role of two other types of factors that
are required for DNA replication.
Presence of enzymes to catalyse the processes: (Any one enzyme)
1. Helicase
Unzipping of DNA double helix so that DNA molecule can be separated in order to create
replicating bubble/fork.
2. Topoisomerase
breaking, swiveling and rejoining DNA strands to relieve DNA strains during unwinding
3. DNA Polymerase
Addition of free deoxyribonucleotides for elongation of the new DNA strand.
4. DNA ligase
form phosphodiester bonds between adjacent deoxyribonucleotides to join the Okazaki
fragments.
5. Primase (Reject RNA primase N2010 comments)
to synthesise the RNA primers to provide free 3OH for DNA Polymerase to elongate the
new DNA strand
165
3
Free deoxyribonucleotides / deoxyribonucleoside triphosphate
6. to provide building blocks for the synthesis of new DNA strand
7. to provide free 5 Phosphate group for formation of phosphodiester bond with existing DNA
strand.

RNA primers
provide free 3OH for DNA Polymerase to elongate the new DNA strand

Single stranded binding proteins


to prevent the DNA from reannealing so that DNA Polymerase can bind / allow complementary
base-pairing

ATP
When a nucleoside triphosphate links to the sugar phosphate molecule backbone of a growing
DNA strand, it losses two of its phosphates as a pyrophosphate molecules. Hydrolysis of the bond
between the phosphate groups provides energy for the reaction.

NOTE: Enzymes are consider as ONE group of factors thus only a maximum of 1 mark will be
awarded if students mention more than 1 type of enzymes with explanation.

(d) Just as a DNA strand provides a template for making a new complementary strand during DNA
replication, it also can serve as a template for transcription.

Fig. 1.1 is a diagrammatic representation of the process of transcription.

Fig. 1.1

Identify molecule E, F and H. [1]


166
4
E RNA polymerase
F RNA nucleotides/ Ribonucleoside triphosphate/nucleoside triphosphate
H mRNA

1m for all 3 correct answers;

(i) Describe how the structure of molecule E is adapted to its role in transcription. [3]

1. molecule E has a specific active site which is complementary to substrate such as DNA
template, ribonucleotides;
2. R groups of (contact) amino acid residues forms weak (ionic) bonds with substrate;
3. R groups of (catalytic) amino acid residues; that catalyses phosphoester bond formation
between growing RNA strand and incoming free nucleotide;
4. funnel allows entry of nucleoside triphosphates;
5. rudder separates the RNADNA hybrid;

(ii) Describe one way by which the structure of molecule(s) F, which is / are adapted for [1]
transcription, differs from that of amino acids, which are adapted for translation.

1. a molecule of F (NTP) is made of a ribose sugar, phosphate group and nitrogenous base (also
accept ribulose triphosphate nucleoside (ribose sugar, 3 phosphates, nitrogeneous base); but
an amino acid is made of a central carbon attached to H group, R group, amine group,
carboxyl group;

2. identity of molecule F is determined by identity of nitrogenous base; but identity of amino acid
is determined by identity of Rgroup;

3. Molecule F has 4 different types (A, U, G, C) while amino acid has 20 different types (distinct R
group).
[Any 1 pt]

[Total : 12]
167
5

2 Influenza A viruses are divided into subtypes based on two proteins on the surface of the
virus: haemagglutinin (HA) and neuraminidase (NA). The influenza A (H7N9) virus designation of
H7N9 identifies it as having HA of the H7 subtype and NA of the N9 subtype.

(a) Describe the roles of haemagglutinin (HA) and neuraminidase (NA) in the reproductive cycle of the
influenza (H7N9) virus. [2]
Haemagglutinin facilitates attachment of virus to host cell membrane by binding to specific
sialic acid receptors on epithelial cells.

Neuraminidase facilitates the release of newly formed virus from the infected host cell by
cleaving the sialic acid receptors.

(b) (i) The influenza genome comprises genes for RNA polymerase. How is the virus RNA
polymerase different from the host RNA polymerase? [1]
host RNA polymerase is a DNA-dependent RNA polymerase, while the virus RNA
polymerase is a RNA-dependent RNA polymerase

(ii) Explain the role of the viral RNA polymerase in the reproductive cycle.
[3]
The viral RNA polymerase synthesizes (accept transcribes) complementary ssRNA (+)
using ssRNA (-) as a template.
These complementary ssRNA(+) strands serve as:
mRNAs for eventual translation into viral proteins in the cytoplasm using the host cell
ribosomes; and
templates for making new copies of the ssRNA (-) genome in the nucleus.

(c) H7N9 is an avian influenza virus but it is also able to infect mammals.

Suggest changes that may have occurred to give H7N9 greater ability to infect mammals.
[2]
Caused by mutations to haemagglutinin gene during the replication of viral RNA
Antigenic drift occurs when accumulation of mutations result in a gradual change to
haemagglutinin (HA)
change shape / conformation/ configuration of the haemagglutinin glycoprotein in H7N9
Shape of haemagglutinin have better fit / more complementary to shape of specific receptors
on human host cells

Haemagglutinin of H7N9 was previously only able to bind to sialic acid on epithelial cells of birds.
However, H7N9 has now acquired the ability to bind to sialic acid on epithelial cells of mammals.
Given that both avian and human influenza strains can infect pigs at the same time, suggest how
H7N9 acquired the ability to infect mammals.

Single cell infected with different types of viruses at the same time;
During viral assembly there could be random assembly of RNA segments from different influenza
viruses;
Leading to novel gene combinations/genetic shift;
168
6

(d) While the influenza genome comprises 8 distinct linear segments of negative sense ssRNA,
bacterial genome consists of chromosome and plasmid. Operons are needed for the regulation of
gene expression.

Suggest why operons give bacteria a selective advantage. [3]


Genes of the same metabolic pathways grouped together for control of gene repression/gene
switching
So that bacterium only produces enzymes when required.
Preventing waste of energy/ resources.
bacteria able to use a variety of sugars/ substrates as they contain different operons.
bacteria can respond to environmental change quickly

R description of selective advantage eg. increase in frequency, pass on alleles to next generation.

[Total : 11]
169
7

3 In normal person, the fragile X mental retardation protein (FMRP), regulates the synthesis of neurone
proteins by stopping ribosomal translocation on target mRNAs. Fig 3.1 shows how patients with fragile
X syndrome (FXS) have non-functional FMRP, resulting in accelerated synaptic protein synthesis that
leads to abnormal synaptic function and intellectual disability.

Non-functional FMRP (FXS)

Fig. 3.1

(a) Describe how polysomes are formed.

Ribosome attaches to AUG start codon;


Another ribosome then attaches to AUG as the preceding ribosome translocates from 5 to 3
direction of mRNA;

Comments:
The question asked how are polysomes formed and not what are polysomes or why
do polysomes form. Hence, answers related to beads on the string structure,
polysomes are multiple ribosomes on an mRNA, and they undergo translation
simultaneously to speed up translation are not accepted. [2]

(b) (i) State the level of control by FMRP on synaptic protein expression.
Translational control;

Comments:
This was generally well answered by the majority. Those who got it wrong either identified
the wrong stage of control or answered positive and negative control. [1]
170
8
(ii) Describe one other control mechanism of a similar level as (b)(i).

Presence of translation initiation factors;


Facilitates the initiation of translation;

OR
Binding of translational repressors/ regulatory proteins to 5UTR/ covers AUG;
Prevents the initiation of translation/ assembly of ribosome;

OR
Polyadenylation/ Addition of poly(A) tail;
A longer polyA tail increases the half life of mRNA so more translation;
[2]
(iii) Explain how FMRP stops the ribosome from translocation.

FMRP has a specific binding site complementary in shape to a specific mRNA base
sequence/ codon;
Physically blocks / steric hindrance to ribosome;
Accept: FMRP has amino acid R groups that attach to mRNA by weak hydrogen/ ionic
bonds;

(c) Initial gene therapy trial uses a short stretch of anti-sense RNA. Explain how this will help to
reduce the symptom of FXS.
[2]
Anti-sense RNA binds by complementary base pairing;
Ref to binding to same stretch of mRNA bases that FMRP binds (to cause the same effect of
stopping ribosomal translocation);
(d) Synaptic proteins that are folded wrongly are usually degraded.

Suggest the advantages of degrading proteins that are wrongly folded.


Can recycle amino acids for other protein synthesis;
Regulate cellular processes by removing enzymes and regulatory proteins that are no
longer needed;
Prevent accumulation of unwanted protein in the cytoplasm;
(Any 2)

Comments:
Some candidates suggested preventing the wrongly folded protein from causing a phenotypic
effect. This is not accepted because usually wrongly folded protein is non-functional and would
not cause any phenotypic effect. [2]
[Total : 11]
171
9

4 Severe combined immunodeficiency (SCID) results from defects in signaling downstream of


cytokine receptors that contain the common cytokine-receptor -chain (c). c is a component of
the receptors (R) for many interleukin such as interleukin-2 (IL-2) and IL-7.

Fig. 4.1 shows the events in the JAK-STAT cell signaling pathway when cytokine binds and this in
turn leads to an activated receptor. Fig. 4.2 shows two causes that could lead to a non-functional
receptor.

Fig. 4.1 Fig. 4.2

(a) Explain why transmembrane proteins such as the cytokine receptor are especially suited for a role [2]
in cell signalling.
Spans the entire membrane;
with one part interacting with extracellular ligand/ signal molecule;
and another interacting with intracellular proteins;
ligand binds via complementary shape;
Leading to conformational change of the transmembrane protein/ dimerisation of receptor;
Cause activation of intracellular enzymes / intracellular proteins inside the cell;
Allows cellular responses without the external signal having to move into the cell/ able to
transduce external signal into internal signal;
(1/2 mark each)

(b) With reference to Fig. 4.1, suggest when happens after cytokine binds to the receptor. [2]
Dimerization of and chains;
Activates JAKs;
(Activated JAK) phosphorylates the intracellular tails of the receptors;
Which activate/ phosphorylate STATs
(1/2 mark each)

(c) With reference to Fig. 4.2, explain how a change in the sequence of the DNA nucleotide may
affect the signaling downstream of cytokine receptors. [5]
Compulsory points:
Leads to change in mRNA codon leading to a change in amino acid sequence;
2 examples (2m):
change in amino acid R-group properties, lead to polypeptide chain folded wrongly, resulting in
172
10
a defective JAK
change in amino acid R-group properties, lead to polypeptide chain cannot embed in the
membrane, resulting in a absent c or absent IL-7R chain
ref to stop codon in mRNA resulting in shorter polypeptide or no polypeptide chain made,
resulting in absent c or absent IL-7R chain and;
relevant consequences (2m):
absent chain in receptor, cytokine cannot bind;
defective JAK, cannot be activated, subsequent events cannot occur;

(d) Proteins synthesized by ribosomes attached to the endoplasmic reticulum may be embedded in
the cell surface membrane. Outline the route taken by such a protein. [3]
Protein is inserted into the membrane of the rough endoplasmic reticulum;
Vesicles with protein embedded in membrane bud off from membrane of RER;
move to and fuse with cis face;
of Golgi apparatus;
modified and sorted;
Vesicles bud off from the trans face of Golgi apparatus;
fuse with the plasma/cell membrane and receptor is presented to the exterior of the cell;
movement across cell is aided by microtubules of cytoskeleton;
(1/2 mark each)

[Total: 12]
173
11

5 Chickens have a structure called a comb on their heads. The drawings show two types of comb.

Fig. 5.1

The shape of the comb is controlled by two alleles of one gene. The allele for pea comb, A, is
dominant to the allele for single comb, a. The colour of chicken eggs is controlled by two alleles of
a different gene. The allele for blue eggs, B, is dominant to the allele for white eggs, b.

The genes for comb shape and egg colour are situated on the same chromosome.

A farmer crossed a chicken that had pea comb and produced blue eggs with a chicken that had a
single comb and produced white eggs. The chicken that had a pea comb and produced blue eggs
are the offspring of the cross between pure bred pea comb, blue eggs and pure bred single comb,
white eggs.

(a) Construct a genetic diagram to illustrate the above cross. [3]


Gametes 1M;
Punnett Square 1M;
F2 phenotype and corresponding genotype ratios 1M;

Parental phenotype Pea comb, blue eggs X Single comb, white eggs
A B a b
Parental genotype
a b a b

A B A b
a b
Gametes
a B a b

A B A b a B a b

A B A b a B a b
a b
a b a b a b a b

F2 genotype A B A b a B a b
ratio
174
12
a b a b a b a b
1 single 1 single
F2 phenotype 1 Pea comb 1 Pea comb,
comb, Blue comb, white
ratio blue eggs white eggs
eggs eggs

(b) The farmer could identify which of the female offspring from this cross would eventually produce
blue eggs. Explain how. [2]
Pea comb offspring will produce blue eggs;
Alleles A and B are inherited together / are on the same chromosome;

(c) However, in certain crosses, the farmer was not able to identify the female offspring which will
produce blue eggs. Explain why. [2]
Crossing over will separate alleles;
If crossing over occurred some gametes will contain alleles A and b;

In chickens the males are the homogametic sex while the females are the heterogametic sex.

(e) A gene on the X chromosome controls the rate of feather production. The allele for slow feather
production, F, is dominant to the allele for rapid feather production, f.

A farmer made a cross between two chickens with known genotypes. He chose these chickens so
that he could tell the sex of the offspring soon after they hatched by looking at their feathers.

Which of the crosses shown in the table did he make? Explain your answer.

Fig. 5.3 [2]


Cross C / Xf Xf and XFY;
(Only) cross where all males are one phenotype and all females are a different phenotype;
Cross showing all males offsprings are slow feather production, all females offsprings fast
feather production;

(f) Female chickens are more likely than male chickens to show recessive sex-linked characteristics.
Explain why. [2]
Two alleles for each gene present in male / chromosomes are homologous in male;
OR female has one allele for each gene (for the non-homologous section of chromosome);
Recessive alleles always expressed in female;
Males need two recessive alleles for allele to be expressed / in males recessive alleles can
be masked by dominant allele

[Total: 11]
175
13

6 Table 6.1 shows the approximate percentage of protein in different membranes.

Table 6.1

Organelle Membrane Protein as a percentage of dry weight


Chloroplast Outer membrane 54
Chloroplast Inner membrane 70
Mitochondrion Outer membrane 55
Mitochondrion Inner membrane 80

(a) Describe one way in which the distribution of protein in the inner and outer membranes of these
two organelles is similar.

Suggest a reason for this similarity. [2]


Similarity:
Both the chloroplast and mitochondrion have more protein on their inner membrane as
compared to outer membrane;

Reason:
Inner membrane of mitochondrion and chloroplast has various enzymes and proteins such as
ATP synthase, proton pumps, electron transport proteins/ electron carriers in ETC required
to carry out chemiosmosis in respiration or photosynthesis;

Note: Chloroplast has a double outer membrane and the inner membrane refers to the thylakoid
membrane.

(b) (i) Atrazine is a widely used herbicide (weedkiller). It binds to a chloroplast protein involved in
electron transfer between photosystems.

Explain how atrazine works as an effective herbicide. [4]


Atrazine binds to an electron carrier or name a possible electron carrier;
shape of electron carrier change;
ETC stop/ photophosphorylation stop;
no ATP or reduced NADP will be produced and therefore the Calvin Cycle will cease to
function;
no G3P so no source of energy/sugar/food produced

(ii) Maize plants are insensitive to atrazine.

The cytoplasm of maize plant cells contains a tripeptide which binds to atrazine. The product
is then transported to the vacuole.

Suggest why maize plants are insensitive to atrazine. [2]


Atrazine will not enter the chloroplast.
when in the vacuole the tonoplast will act as an effective barrier preventing the atrazine
affecting the metabolism of the cell
Binding to tripeptide changes configuration of atrazine such that it is not able to enter the
chloroplast.

(c) At the end of photosynthesis, glyceraldehyde 3-phosphate (3-carbon sugar) is produced by the [3]
Calvin cycle, which is subsequently converted into starch for storage.
176
14

Discuss the suitability of starch as a storage compound in plants.


Large molecule prevented from diffusing out
Large molecule made up of a long chain of -glucoses linked by many glycosidic bonds able
to store large amount of energy
In a compact space because it can be coiled into a helix.
Helical formation was made possible with the OH groups pointing into the helix - insoluble
Insoluble thus does not affect the osmotic potential of the cell
No cross linked/glycosidic bonds on the exterior of the helices easily hydrolyzed

[Total : 11]
177
15

6 Fig 6.1 shows T.W. Engelmanns experiment to measure the action spectrum of the filamentous
green alga. He placed the filamentous green alga into a test-tube along with a suspension of
oxygen-seeking bacteria. He allowed the bacteria to use up the available oxygen and then
illuminated the alga with light that had been passed through a prism to form a spectrum. After a
short time he observed the results shown in Fig. 6.1.
(Fig on pg 333)

(a) (i) Sketch the action spectrum for this alga. [1]
2 peaks at correct range: one between 400 to 500 range the other between 600 to 700

(ii) Explain his observation shown in Fig. 6.1. [2]


More light energy absorbed at the two range;
By the light harvesting complexes;
Increase rate of photolysis of water;
To fill the electron hole of chlorophyll a at reaction centre PS II or PS700;
Produce more oxygen at the two region (to attract oxygen-seeking bacteria)

(b) (i) At the end of photosynthesis, glyceraldehyde 3-phosphate (3-carbon sugar) is produced by
the Calvin cycle. Outline how this glyceraldehyde 3-phosphate is made in the Calvin cycle
and converted into starch. [5]
(Carbon fixation phase) Carbon dioxide (1C) is accepted by RuBP (5C compound), [
catalyzed by ribulose bisphosphate carboxylase (Rubisco); 5
6C compound then cleaves to form 2 molecules of 3C glycerate-3-phosphate (GP); ]
(reduction phase) in the presence of ATP and NADPH, GP forms 3C triose phsophate
(G3P);
For every 6 G3P formed, 1 G3P exits the cycle
G3P can form glucose via condensation
Many -glucose monomers are linked to form long chain via glycosidic bonds
Long chain then coils to form compact helix

(ii) Discuss the suitability of starch as a storage compound in plants. [3]


Large molecule prevented from diffusing out
Large molecule made up of a long chain of -glucoses linked by many glycosidic bonds
able to store large amount of energy
In a compact space because it can be coiled into a helix.
Helical formation was made possible with the OH groups pointing into the helix -
insoluble
Insoluble thus does not affect the osmotic potential of the cell
No cross linked/glycosidic bonds on the exterior of the helices easily hydrolyzed

[Total : 11]
178
16

7 The marine threespine sticklebacks, Gasterosteus aculeatus is a freshwater fish living in the
lakes of British Columbia, Canada as shown in Fig. 7.1.

Fig. 7.1

In order to investigate the process of speciation in these populations, three small lakes were
studied. Each lake contained two varieties of stickleback: a large, bottom-dwelling variety that
fed on invertebrates near the shore and a small, plankton-eating variety that lived in the open
water. The probability of breeding between pairs of individuals was measured under laboratory
conditions in the following breeding combinations:

I different varieties from the same lake


II different varieties from different lakes
III same variety from different lakes
IV same variety from the same lake

The data are summarized in Fig. 7.1 below.

Fig. 7.1

(a) (i) Identify the highest and lowest probabilities of breeding for individuals of the same variety
from different lakes.

Highest probability: 0.58 (Allow answers from 0.570.59) [1]


179
17
Lowest probability: 0.25 (Allow answers from 0.240.26)
Both required for the mark.

(ii) With reference to Fig. 7.1, describe the differences in probability of breeding between
individuals from the same lake.
[2]
IV combination (of the same variety): Individuals have higher breeding probability if they
are the same variety and
quote values 0.58;
OR
Accept: Individuals of different varieties have a lower probability of breeding and
quote values;
The probability of breeding between individuals of the same variety shows a larger
range of values / narrower range if of different variety;
[Any 2]

(b) Scientists concluded that speciation is taking place in these populations. Explain the type of
speciation as shown in Fig. 7.1.

Sympatric speciation
is taking place because different varieties from the same lake have a low probability of
breeding
OR
No evidence of allopatric speciation
as same varieties from different lakes do not show strong reproductive isolation/ Comparing
exp III and IV, they show similar range of breeding probability.

Different feeding habits or habitat (shore versus open water) contribute to low breeding
probability/ reproductive isolation;
Reducing interbreeding and gene flow between the varieties in each lake;
[4]

(c) With reference to Fig. 7.1, explain why all the individuals are still considered the same species.

Probability of breeding is not zero/ Not completely reproductively isolated;


(According to Biological species concept), they can still interbreed to produce fertile, viable
offspring; [2]

(d) The freshwater lakes also contain many different types of parasites that infect the marine
threespine sticklebacks. Suggest why these parasites help to speed up speciation of the marine
threespine sticklebacks.

Parasites are host specific/ infect either bottom-dwelling variety or plankton-eating variety;
Parasites constitute a selection pressure;
Cite an example of a characteristic that confer a selective advantage, e.g. resistance against
parasitic infection
Those selected for survive, reproduce and pass on their alleles coding for the beneficial
characteristic to their offsprings. [3]

[Total : 12]
180
18

Section B
Answer one question.
Write your answers on the separate answer paper provided.
Your answer should be illustrated by large, clearly labeled diagrams, where appropriate.
Your answers must be in continuous prose, where appropriate.
Your answers must be set out in section (a), (b) etc., as indicated in the question.

8 (a) Explain how the positive and negative control of the lac operon affect bacteria growth. [10]
[Compulsory 1 point]
Metabolism is biased toward the utilization of glucose - glucose is the simplest form of
sugar /enzymes required for the breakdown of glucose via glycolysis are continuously
present.

[Max 3m]
When [glucose] high and [lactose] is absent, glucose is preferred over lactose/used as
a main respiratory substrate/ main energy source.
Repressor binds to operator;
[cAMP] decreases, CAP is not activated/cAMP does not bind to CAP and CAP unable
to bind to CAP binding site;
RNA polymerase does not bind to promoter/ lac operon is switched off;
bacteria growth increases exponentially because energy from glucose breakdown is
utilized for bacterial growth.

[Compulsory 2 points]
When glucose is fully utilized, bacterial growth will remain constant
[lactose] also remains constant initially as time is needed for lac operon to turn on
genes to produce -galactosidase to hydrolyse lactose;

[Max 3m]
Absence of [glucose] and high [lactose];
Allolactose acts an inducer, binds to repressor to inactivate it;
high levels of cAMP binds and activates CAP and
increases affinity of RNA polymerase to bind to promoter and to turn on expression of
lac genes;
high [lactose] - Bacteria growth increases exponentially due to production of -
galactosidase that hydrolyses lactose to form glucose (main respiratory substrate) and
galactose;
as [lactose] concentration decreases; less allolactose binds to the repressor; and as a
result, the activated repressor can bind to the operator;
this blocks binding of RNA Polymerase to the promoter and inhibits transcription of lac
operon;
bacteria growth plateaus/drops as lactose is used up

Description for bacteria growth [Compulsory 3m]


bacteria growth increases exponentially because energy from glucose breakdown is
utilized for bacterial growth.
When glucose is fully utilized, bacterial growth will remain constant
high [lactose] - Bacteria growth increases exponentially due to production of -
galactosidase that hydrolyses lactose to form glucose (main respiratory substrate) and
galactose;
bacteria growth plateaus/drops as lactose is used up
181
19
(b) Describe how regulation of blood glucose level in humans involves cAMP. [5]
-cells detects low blood glucose level,
secrete glucagon into bloodstream;
glucagon binds to GPCR at the cell surface membrane, activates GPCR;
Activated GPCR binds to G-protein;
Cause GDP to be displaced by GTP, activates G-protein;
Activated G-protein activated adenylyl cyclase;
Converts ATP to cAMP;
Which acts as 2nd messenger
Activates phosphorylation cascade
Activate glycogen phosphorylase
Break down glycogen to glucose (to increase blood glucose level)
(1/2 m each)

(c) Explain the significance of the steps in glycolysis. [5]


Glycolysis is a common step in both anaerobic and aerobic respiration;
Phosphorylation of glucose (by 2 ATP) is to activate it;
Phosphorylation of glucose (by 2 ATP) / glucose-6-phosphate results in glucose being
trapped in the cytosol/ unable to leave the cell through the same glucose carrier
protein/ committed to the end of glycolysis;
PFK which catalyse the phosphorylation of fructose phosphate also control rate of
glycolysis/ high rate of ATP act as allosteric inhibitor to PFK;
ATP synthesis by substrate level phosphorylation;
Forms two glyceraldehyde-3-phosphate/ triose phosphate from one glucose;
Forms two reduced NAD (NADH) (by dehydrogenation);
Which later give 6 ATP by oxidative phosphorylation;
Pyruvate can enter into link reaction/ mitochondria or be converted to lactate (in
mammals) or ethanol and carbon dioxide (in yeast);
Pyruvate is small enough to enter mitochondrion for aerobic respiration;

9 (a) Describe the differences between cellulose synthase and cellulose. [5]

Any five below:

Feature Cellulose synthase Cellulose


Nature Globular protein Polysaccharide
Monomer Amino acid glucose
Bond between Peptide bond (1,4) glycosidic bond
monomer
Shape Spherical Linear (chains)
Bonds Ionic bond, hydrogen bond, Hydrogen bonds
contributing to hydrophobic interaction and disulfide
structure bonds
Groups between R groups between OH groups
contributing to
bond formation
Location/ On cell surface membrane/ enzyme As part of plant cell wall
Function surrounding plant cell/
gives cell a regular shape
182
20
(b) With reference to photosynthesis and cellular respiration, explain the importance of
compartmentalization within a plant cell. [10]
The inner membrane of the chloroplast encloses the stroma where enzymes for Calvin
cycle are
Thylakoid membrane houses light harvesting complexes / photosystems for
harnessing light energy to excite electrons in the reaction centre down the ETC
with NADP reductase as final electron carrier which is reduced to NADPH for Calvin
cycle
Flow of electrons down ETC allows pumping of H+ from the stroma to the thylakoid
space, setting up a proton gradient
Diffusion of the H+ from the thylakoid space to the stroma via ATP synthase / stalked
particles in thylakoid membrane allows phosphorylation of ADP to form ATP required
in Calvin cycle
The inner mitochondrial membrane encloses the matrix where enzymes of the Krebs
cycle are localised
Enzymes localised in the cytoplasm allow glycolysis to occur
The inner mitochondrial membrane is the site of oxidative phosphorylation
and is highly folded forming cristae to increase surface area for more ETCs / stalked
particles
Flow of electrons down ETC allows pumping of H+ from the matrix to the
intermembrane space, setting up a proton gradient since the inner mitochondrial
membrane is impermeable to H+
Diffusion of the H+ from the inner mitochondrial space to the matrix via ATP synthase /
stalked particles in the inner mitochondrial membrane allows phosphorylation of ADP
to form ATP

(c) Explain the role of meiosis in natural selection. [5]

(Any 3)
Crossing over at prophase I (exchanges alleles);
Independent assortment of homologous chromosomes at metaphase I;
Introduces genetic variation in a population;
Random fusion of gametes after meiosis (further increases genetic variation in
individuals);
(Compulsory 2 marks)
Selection pressure results in some organisms having selective advantage;
Those selected for survive and reproduce and pass on their alleles to their offspring,
increasing frequency of alleles over time;
183

H2 ANDERSON JUNIOR COLLEGE


HIGHER 2

CANDIDATE
NAME

PDG PDG I
INDEX NUMBER

BIOLOGY 9648/03
Applications Paper and Planning Question 20 September 2013
Paper 3

2 hours
Additional Materials: Answer Paper

READ THESE INSTRUCTIONS FIRST


Write your name and PD group on all the work you hand in.
Write in dark blue or black pen.
You may use a soft pencil for any diagrams, graph or rough working.
Do not use paper clips, highlighters, glue or correction fluid.

Answer all questions.

All working for numerical answers must be shown.


At the end of the examination, fasten all your work securely For Examiners Use
together.
The number of marks is given in brackets [ ] at the end of each 1
question or part question.
2
Calculators may be used
3
4
5
Total 72

This document consists of 14 printed pages.


184
2
Answer all the questions.

1 In a maternity ward at a local hospital, a mix-up involving three couples and three babies caused
a lot of confusion. Based on phenotypic characteristics, the nurses were unable to correctly
identify the parents of the babies. In order to solve the case, a scientist was called in to carry out a
DNA test to identify the parents of the babies. The test was based on the principle that different
individuals have a different number of repeating units at a particular locus in a chromosome.

(a) State the name of the test that was carried out. [1]

......

Chromosome 13 was isolated from the DNA samples that were obtained from the three couples
and three babies and used for further analysis. The sequence below shows a segment of
chromosome 13, which was used in the analysis where (TTAGGAT) is the repeating unit and n
is the number of repeats.

The gel diagram in Fig. 1.1 shows the results of the DNA test obtained from each individual, Dad
and Mom #1, #2 and #3, and baby A, B, and C.

Fig. 1.1

(b) (i) Explain the purpose of the DNA ladder in Fig. 1.1. [1]

.....

......

(ii) DNA is colourless.


State how the DNA bands in the gel can be made visible. [1]

.....

.....
185
3
(c) With reference to Fig. 1.1, explain why

(i) some bands in a lane are higher than others; [2]

.....

.....

.....

(ii) the lane for Dad#3 has only one band; [2]

.....

.....

.....

(iii) some lanes have two bands. [2]

.....

.....

.....

(d) (i) With reference to Fig. 1.1, state and explain which couple is the parents of Baby A. [2]

.....

.....

.....

(ii) Explain why results of paternity testing are not absolutely certain. [2]

.....

.....

.....

[Total: 13]
186
4

22 One approach of gene therapy to treat cystic fibrosis uses viruses to deliver normal alleles of the
CFTR gene into epithelial cells of the airways.

A team of researchers in the USA developed a new strain of non-pathogenic adeno-associated


virus (AAV), AAV2.5T. Genes for the CFTR protein and the enzyme luciferase were inserted into
the DNA of the viruses. Luciferase catalyses the production of a green fluorescent protein when
luciferin is added.

The normal AAV strain and the AAV2.5T strain were added to cultures of epithelial cells from the
airways. After adding luciferin, the number of cells that had taken up the viral genes was
estimated using the intensity of the green fluorescence which developed. The results are shown
in Fig. 2.1.

Intensity of
green
fluorescence
/ arbitrary
units

AAV

Time / days
Fig. 2.1

(a) Explain why the researchers inserted a gene for luciferase into the viral DNA. [1]

.....

.....

.....

(b) With reference to Fig. 2.1, compare the ability of the two viral strains, AAV and AAV2.5T, to
infect epithelial cells of the airways. [2]

.....

.....

.....


187
5

(c) Suggest why a decrease in intensity of green fluorescence was detected in cells infected
with AAV2.5T during the last 10 days. [2]

.....

.....

.....

(d) Describe how delivering normal alleles of the CFTR gene into epithelial cells in the airways
could relieve the symptoms of cystic fibrosis. [4]

.....

.....

.....

.....

.....

.....

(e) Researchers have started to use cDNA of the CFTR gene for more effective therapeutic
results. Explain why cDNA of the normal CFTR gene is used instead of the normal CFTR
gene. [2]

.....

.....

.....
188
6

(f) AAV does not cause human disease, but because we are routinely exposed to this virus,
30 to 60 percent of people develop antibodies that neutralize AAV if it enters the
circulation.

To extend the potential benefits of gene therapy to a broader population, researchers have
produced a bioengineered decoy, which are empty AAV capsids disabled of their ability to
enter target cells.

Suggest how using both the decoy and vector at the same time would improve the success
rate of the gene therapy. [3]

.....

.....

.....

.....

.....

[Total: 14]
189
7

3 Plants have developed defense mechanisms against pathogens such as bacteria, fungi, and
viruses. Chemicals released by these pathogens can trigger a defense response in infected plant
cells. For example, the production of hydrogen peroxide (H2O2) which reacts with pathogen
membranes and cellular chemicals, eventually kills both the cell and the pathogen.

The OSRac1 gene was isolated and introduced into a number of rice plant (Oryza spp.) lines to
study its role in disease resistance of plants to Blast fungus. Experiments were carried out to see
if the OSRac1 gene was part of the signalling pathway for hydrogen peroxide production.

A control and four other genetically modified rice plant lines were exposed to chemicals known to
initiate a defense response and the production of hydrogen peroxide. The results are shown in
Fig. 3.1 below.

Fig. 3.1

(a) With reference to Fig. 3.1, compare the changes in H2O2 production in the control and [4]
genetically modified plants after the chemical was applied and conclude if OSRac1 gene is
involved in disease resistance.

.....

.....

.....

.....

.....

.....


190
8
(b) Suggest two possible concerns about the use of transgenic plants with the disease
resistance gene. [2]

.....

.....

.....

(c) Edible vaccine against cholera using rice was developed by a team of Japanese scientists
by cloning a gene from cholera bacteria, Vibrio cholerae into the genome of the Kitaake rice
plant. The gene expressed a subunit of the disease-causing cholera toxin B. The steps are
shown in Fig. 3.2.

Coded by

Fig. 3.2

(i) Describe how the antigen gene inserted into rice DNA genome would result in successful
vaccination once the rice is eaten. [2]

.....

.....

.....
191
9

(ii) Explain why it is necessary to form a callus in order to grow genetically modified rice. [3]

.....

.....

.....

(d) Edible vaccines have been made using other plants like tomato and banana. Suggest the
advantages of using rice over other plants. [2]

.....

.....

.....

[Total: 13]
192
10

4 Planning question

DNA is the genetic material in all living organisms. DNA from an external source can be taken up
by bacteria by a process known as transformation.

Plan an experiment to investigate the effect of varying plasmid concentration on the efficiency of
E. coli transformation.

You may, or otherwise, also use the following equipment and materials:
Solution of plasmid containing ampicillin resistance gene of concentration 100
Unit/ml
Suspension of competent E. coli cells
Distilled water
LB broth (nutrient solution)
Petri dishes containing LB agar medium and ampicillin (LB/amp plates)
Sterilised microfuge tubes
Micropipettes
Bleach solution
alcohol

Your plan should have a clear and helpful structure to include:


an explanation of theory to support your practical procedure
a description of the method used including scientific reasoning behind the method
an explanation of the dependent and independent variables involved
relevant, clearly labelled diagrams
how you will record your results and ensure they are as accurate and reliable as possible
proposed layout of results tables and graphs with clear headings and labels
correct use of scientific and technical terms
[Total:12]
193
11

.....

.....

.....

.....

.....

.....

.....

.....

.....

.....

.....

.....

.....

.....

.....

.....

.....

.....

.....

.....

.....

.....

.....

.....

.
194
12

.....

.....

.....

.....

.....

.....

.....

.....

.....

.....

.....

.....

.....

.....

.....

.....

.....

.....

.
195
13

.....

.....

.....

.....

.....

.....

.....

.....

.....

.....

.....

.....

.....

.....

.....

.....

.....

.....

.....

.....

.....

.....

.....

.....

.....

.....

.....

.....
196
14
Free-response question

Write your answers to this question on the separate answer paper provided.

Your answer:

should be illustrated by large, clearly labelled diagrams, where appropriate;


must be in continuous prose, where appropriate;
must be set out in sections (a), (b) etc., as indicated in the question.

5 (a) Rubisco is an important enzyme in the Calvin cycle. [6]

Outline how it might be possible to produce genetically modified plants with increased
rubisco concentrations, or rubisco with enhanced efficiency.

(b) Explain the basis of genetic engineering and how genetic engineering can improve the yield [8]
of one named crop plant and one named animal in solving the demand for food in the
world.

(c) Discuss how the human genome project can help parents customise their babies and the [6]
possible implications of doing so.

[Total: 20]
197

H2 ANDERSON JUNIOR COLLEGE


HIGHER 2

CANDIDATE
NAME

PDG PDG I
INDEX NUMBER

BIOLOGY 9648/03
Applications Paper and Planning Question 20 September 2013
Paper 3

2 hours
Additional Materials: Answer Paper

READ THESE INSTRUCTIONS FIRST


Write your name and PD group on all the work you hand in.
Write in dark blue or black pen.
You may use a soft pencil for any diagrams, graph or rough working.
Do not use paper clips, highlighters, glue or correction fluid.

Answer all questions.

All working for numerical answers must be shown.


At the end of the examination, fasten all your work securely For Examiners Use
together.
The number of marks is given in brackets [ ] at the end of each 1
question or part question.
2
Calculators may be used
3
4
5
Total 72

This document consists of 15 printed pages and 1 blank page.


198
2
Answer all the questions.

1 In a maternity ward at a local hospital, a mix-up involving three couples and three babies caused
a lot of confusion. Based on phenotypic characteristics, the nurses were unable to correctly
identify the parents of the babies. In order to solve the case, a scientist was called in to carry out a
DNA test to identify the parents of the babies. The test was based on the principle that different
individuals have a different number of repeating units at a particular locus in a chromosome.

(a) State the name of the test that was carried out. [1]
Paternity Test/ DNA Fingerprinting/Genetic Fingerprinting

Chromosome 13 was isolated from the DNA samples that were obtained from the three couples
and three babies and used for further analysis. The sequence below shows a segment of
chromosome 13, which was used in the analysis where (TTAGGAT) is the repeating unit and n
is the number of repeats.

The gel diagram in Fig. 1.1 shows the results of the DNA test obtained from each individual, Dad
and Mom #1, #2 and #3, and baby A, B, and C.

Fig. 1.1

(b) (i) Explain the purpose of the DNA ladder in Fig. 1.1. [1]
Each band serves as a comparison / reference for the size / no. of repeats present / found in
the individuals tested.

(ii) DNA is colourless.


State how the DNA bands in the gel can be made visible. [1]
Ethidium bromide under UV light
staining with methylene blue
autoradiography with a labeled probe

(c) With reference to Fig. 1.1, explain why


(i) some bands in a lane are higher than others; [2]
Higher bands are longer as the DNA fragments havea larger number of repeated units
199
3
The longer a DNA fragment, the harder it is for that fragment to travel through the pores
in the gel, and so it migrates slower than the smaller fragments with fewer repeats.

(ii) the lane for Dad#3 has only one band; [2]
They are homozygous at this locus.
they have the same number of repeats on both homologous chromosome13
For example: Dad #3 has 25 repeats on both versions of his chromosome 13.

(iii) some lanes have two bands. [2]


They are heterozygous at this locus.
they have a different number of repeats on each homologue of chromosome 13.

(d) (i) With reference to Fig. 1.1, state and explain which couple is the parents of Baby A. [2]
couple #3
Baby A has a band in n=25 and Dad #3 is the only parent with n=25 @ chromosome.

(ii) Explain why results of paternity testing are not absolutely certain. [2]
Only a limited number of loci were considered.
It is possible that a male may be a match at all the loci considered, but not be a match at
another loci that was not considered in the test.

[Total: 13]
200
4

22 One approach of gene therapy to treat cystic fibrosis uses viruses to deliver normal alleles of the
CFTR gene into epithelial cells of the airways.

A team of researchers in the USA developed a new strain of non-pathogenic adeno-associated


virus (AAV), AAV2.5T. Genes for the CFTR protein and the enzyme luciferase were inserted into
the DNA of the viruses. Luciferase catalyses the production of a green fluorescent protein when
luciferin is added.

The normal AAV strain and the AAV2.5T strain were added to cultures of epithelial cells from the
airways. After adding luciferin, the number of cells that had taken up the viral genes was
estimated using the intensity of the green fluorescence which developed. The results are shown
in Fig. 2.1.

Intensity of
green
fluorescence
/ arbitrary
units

AAV

Time / days
Fig. 2.1

(a) Explain why the researchers inserted a gene for luciferase into the viral DNA. [1]
infected cells are able to produce luciferase;
able to identify infected cells/cells that have taken up viral DNA (which will
fluoresce when luciferin is added)/ ref to gene for luciferase functions as a
reporter gene;

(b) With reference to Fig. 2.1, compare the ability of the two viral strains, AAV and AAV2.5T, to
infect epithelial cells of the airways. [2]
1. Both AAV and AAV2.5T can infect epithelial cells;
[Both pt 2 & 3 - 1m]
2. AAV2.5T infects more cells than AAV / infects cells more readily;
3. Quote relevant values (both intensity and day indicated) from graph;
[Both pt 4 & 5 - 1m]
4. Intensity of green fluorescence increases more in AAV2.5T compared to AAV
from 0-20 days;
5. from 0 to 15 au in AAV2.5T compared to only 2 au in AAV; A! any values correctly
comparing rate of increase anytime during the first 20 days period
[Both pt 6& 7 - 1m]
6. From the 20th to 30th day, ability of AAV2.5T to infect cells fall but AAVs ability
remains constant;
7. Intensity of green fluorescence decreases to 12.5 au in AAV2.5T compared to
remaining at 2 au in AAV; @ m
201
5

(c) Suggest why a decrease in intensity of green fluorescence was detected in cells infected
with AAV2.5T during the last 10 days. [2]
Green fluorescent protein was broken down;
Luciferin was used up;
(Infected) cells die (as virus regain virulence);
R! cells constantly being shed/killed by immune system as gene therapy was done in
cultures of epithelial cells.
(d) Describe how delivering normal alleles of the CFTR gene into epithelial cells in the airways
could relieve the symptoms of cystic fibrosis. [4]
1m, max 3
normal allele of CFTR gene is transcribed and translated / expressed to produce
functional CFTR protein;
CFTR protein embedded in cell membrane allows chloride ions to leave cell /
serves as functional chloride channel;
water also leaves cell down the water potential gradient;
normal / less viscous mucus formed ;
(essential point)
no blockage of airways / less chance of bacterial lung infections/any other
related possible symptoms;

(e) Researchers have started to use cDNA of the CFTR gene for more effective therapeutic
results. Explain why cDNA of the normal CFTR gene is used instead of the normal CFTR
gene. [2]
cDNA does not contain non-coding regions such as introns;
cDNA can be transcribed and translated directly without the need for post-
transcriptional modifications such as splicing;
Hence the CFTR proteins can be synthesized efficiently;

(f) AAV does not cause human disease, but because we are routinely exposed to this virus,
30 to 60 percent of people develop antibodies that neutralize AAV if it enters the
circulation.

To extend the potential benefits of gene therapy to a broader population, researchers have
produced a bioengineered decoy, which are empty AAV capsids disabled of their ability to
enter target cells.

Suggest how using both the decoy and vector at the same time would improve the success
rate of the gene therapy. [3]
Higher ratio of decoy to vector;
Decoy cannot enter target cells, thus they stay in circulation for longer period of
time;
Ref to antibodies bound to the capsid decoys;
Allowing the DNA-carrying vectors to evade the antibodies and enter the targeted
cells in the liver;

[Total: 14]
202
6

3 Plants have developed defense mechanisms against pathogens such as bacteria, fungi,
and viruses. Chemicals released by these pathogens can trigger a defense response in
infected plant cells. For example, the production of hydrogen peroxide (H2O2) which reacts
with pathogen membranes and cellular chemicals, eventually kills both the cell and the
pathogen.

The OSRac1 gene was isolated and introduced into a number of rice plant (Oryza spp.)
lines to study its role in disease resistance of plants to Blast fungus. Experiments were
carried out to see if the OSRac1 gene was part of the signalling pathway for hydrogen
peroxide production.

A control and four other genetically modified rice plant lines were exposed to chemicals
known to initiate a defense response and the production of hydrogen peroxide. The results
are shown in Fig. 3.1 below.

Fig. 3.1
(a) With reference to Fig. 3.1, compare the changes in H2O2 production in the control and [4]
genetically modified plants after the chemical was applied and conclude if OSRac1 gene is
involved in disease resistance.

Production of H2O2 increased less in C (control) plants than A1/A5 plants supported
with relevant data;
Production of H2O2 in D41/D42 plants increased less than in C (control) plants
supported with relevant data;
Production of H2O2 increased the most in A1/least in D41 plants supported with
relevant data;
[Compulsory conclusion point]
Transgenic plants with functioning gene showed more H2O2 production so
hypothesis; OR
Transgenic plants with gene suppressed showed less H2O2 production so
hypothesis;
203
7
(b) Suggest two possible concerns about the use of transgenic plants with the disease
resistance gene.

Disease resistance could spread to species related to rice, e.g. weeds;


Reluctance of consumers to eat genetically modified food products;
Outcompete natural species and change gene pool; [2]
May increase allergenic effects in human/cattle;
Any two above

(c) Edible vaccine against cholera using rice was developed by a team of Japanese scientists
by cloning a gene from cholera bacteria, Vibrio cholerae into the genome of the Kitaake rice
plant. The gene expressed a subunit of the disease-causing cholera toxin B. The steps are
shown in Fig. 3.2.

Coded by

Fig. 3.2

(i) Describe how the antigen gene inserted into rice DNA genome would result in successful [2]
vaccination once the rice is eaten.
[Essential compulsory point] Antigen protein would be expressed in rice cells;
Any 1 below:
Antigen would trigger immune system to produce the antibody;
Antibody have a specific configuration that is complementary to the glycoprotein
(antigen);
Antibody will be synthesized to mount an immune response against a subsequent
vaccination against cholera bacteria;
204
8
Extend Q N06/P2/Q8c - Outline the roles of glycoproteins in the cell surface membrane.
[8]

Aid cell recognition


Act as cell surface receptors (rarely seen)
Cell Attachment
Carbohydrates also form the glycocalyx coat, the layer outside of the plasma
membrane.
Functions of the glycocalyx:
Protection: Cushions the plasma membrane and protects it from chemical injury
Immunity to infection: Enables the immune system to recognize and selectively attack
foreign organisms
Transplant compatibility: Forms the basis for compatibility of blood transfusions,
tissue grafts, and organ transplants
Cell adhesion: Binds cells together so that tissues do not fall apart
Fertilization: Enables sperm to recognize and bind to eggs
Embryonic development: Guides embryonic cells to their destinations in the body

(ii) Explain why it is necessary to form a callus in order to grow genetically modified rice.

Callus divide by mitosis to ensure all the genetically identical daughter cells would
have the gene;
Callus can be genetically modified by A. tumefaciens;
Callus can be sub-cultured to increase the number of callus;
Callus can be manipulated by auxin and cytokinin to induce root and shoot
formation to form a plantlet; [3]
(Any three)

(d) Edible vaccines have been made using other plants like tomato and banana. Suggest the [2]
advantages of using rice over other plants.
Rice can be stored for a long time;
Rice forms a staple diet among many Asians
Rice is versatile enough to be eaten with other dishes;
[Total: 13]
205
9

4 Planning question

DNA is the genetic material in all living organisms. DNA from an external source can be taken up
by bacteria by a process known as transformation.

Plan an experiment to investigate the effect of varying plasmid concentration on the efficiency of
E. coli transformation.

You may, or otherwise, also use the following equipment and materials:
Solution of plasmid containing ampicillin resistance gene of concentration 100
Unit/ml
Suspension of competent E. coli cells
Distilled water
LB broth (nutrient solution)
Petri dishes containing LB agar medium and ampicillin (LB/amp plates)
Sterilised microfuge tubes
Micropipettes
Bleach solution
alcohol

Your plan should have a clear and helpful structure to include:


an explanation of theory to support your practical procedure
a description of the method used including scientific reasoning behind the method
an explanation of the dependent and independent variables involved
relevant, clearly labelled diagrams
how you will record your results and ensure they are as accurate and reliable as possible
proposed layout of results tables and graphs with clear headings and labels
correct use of scientific and technical terms
[Total:12]

Mark Scheme
1. Theoretical consideration / rationale of plan to justify the practical procedure [1]
2. Correct use of technical and scientific terms any 2 of the following [1]
Relate theory to prac
Examples:
DNA is the genetic material in all living organisms. Transformation is the process by
which DNA from an external source is taken up by bacteria.
The plasmid contains an ampicillin-resistance gene that confers resistance to the
antibiotic.
Bacteria that have taken up this plasmid are able to grow on LB/amp medium.
Amp resistance gene serves as genetic marker to identify transformed cell
When made competent by heat-shock, pores appear transiently in the membrane of
bacteria. External DNA/plasmids may then enter the cell via these pores
Consequently, as plasmid concentration increases, the efficiency of E.coli transformation
will also increase proportionally.

3. An explanation of the dependent and independent variables involved [1]


Independent variable: Concentration of plasmid (unit/ml)
Dependent variable: Efficiency of E. coli transformation is represented by Number of
colonies on LB/Amp plates

[A description of method used including the scientific reasoning behind the method]
4. Description of how the plasmid solution is prepared [1]
206
10
i. Using plasmid solution of concentration 100 Unit/ml, make dilutions to obtain 5 different
concentrations.
Concentration of Volume of plasmid solution of Volume of water
plasmid (unit/ml)* conc 100 Units/ml (ml)* (ml)*
100 1.0 0.0
80 0.8 0.2
60 0.6 0.4
40 0.4 0.6
20 0.2 0.8
0 0 1.0
* or any other reasonable range / volume
OR
serial dilutions:
Concentration of plasmid Volume of stock (ml) Volume of water (ml)
(unit/ml)
Undiluted 1.0 0.0
1 to 10 or 10-1 0.1 of non dilute 0.9
-2 -1
1 to 100 or 10 0.1 of 10 0.9
1 to 1000 or 10-3 0.1 of 10-2 0.9
1 to 10000 or 10-4 0.1 of 10-3 0.9

5. Controlling factors that affect experiment (eg. Duration, temp, volumes,etc) [1]

6. Description of expt [1]


ii. Mix a fixed volume of the E. coli culture with a fixed volume (eg 10l) of the each
plasmid solution
iii. Heat-shock the bacteria-plasmid mixture
iv. Plate each sample on LB/amp plate.
v. Seal the plates with tape and incubate for 24 hours in a 370C (range 28-37) incubator
oven.
vi. After an overnight incubation, count the number of colonies in each plate.

[How you will record your results and ensure they are as accurate and reliable as
possible]

7. Control [1]
vii. For the control tube, replace plasmid solution with same volume of 10l of distilled
water/buffer solution.

Rationale:
To show that the presence of bacterial colonies on agar plates containing
ampicillin is due to transformed bacteria which have taken up plasmid with amp
resistance gene OR
in the absence of plasmid which carries the ampicillin resistant genes, there are no
ampicillin resistant bacterial cells.

8. Accuracy and reliability [1]


For each plasmid concentration, perform two replicates / repeat steps __to __ twice
Calculate average
Check for at least 5 different concentrations of plasmid solutions

[Propose layout of results table and graph with clear headings and labels]

9. Table with appropriate headings [1]


Concentration of plasmid Number of colonies in each plate
solution (unit/ml) Reading 1 Reading 2 Reading 3 Average
207
11

10. Graph [1]


Clear label of axes with units and consistent headings of table
propose expected trend
Graph of average number of colonies against
plasmid concentration

Average number
of colonies
colonies

-1
Concentration of plasmid /Unit ml

11. Risk Assessment [1] any 2


E coli is a bacteria and can cause disease.
Safety Precautions:
Avoid contact - wear gloves and safety goggles.
If in contact, wash off with water and antibacterial soap immediately.
If skin is broken, obtain medical attention.
Dispose of used microfuge tubes / pipette tips / inoculation loops in bleach solution
Sterilize bench with alcohol after expt

12. Suitable diagram illustrating plates with bacteria / process of heat shock [1]
208
12

Free-response question

Write your answers to this question on the separate answer paper provided.

Your answer:

should be illustrated by large, clearly labelled diagrams, where appropriate;


must be in continuous prose, where appropriate;
must be set out in sections (a), (b) etc., as indicated in the question.

5 (a) Rubisco is an important enzyme in the Calvin cycle. [6]

Outline how it might be possible to produce genetically modified plants with increased
rubisco concentrations, or rubisco with enhanced efficiency.

Obtain an explant (from e.g. shoot tip);


Culture in a medium with (macro) and (micro) nutrients, sucrose as an energy
source, and intermediate ratio of auxin to cytokinin.
Reference to aseptic conditions, e.g. surface sterilization.
Callus forms and divides by mitosis to form a mass of undifferentiated cells.
Isolate DNA containing rubisco that has a high efficiency from another plant
species.
Use same restriction enzyme to cut DNA containing gene for rubisco and (Ti)
plasmid;
Allow complementary sticky ends anneal, and DNA ligase to seal the nick by
forming phosphodiester bond;
Recombinant DNA transformed into Agrobacterium tumefaciens bacteria cell by
calcium chloride heat shock method;
Infect callus with transformed Agrobacterium tumefaciens bacteria which will
integrate gene for rubisco into plant chromosome;
Subculture to increase number of callus with desired gene;
Increase cytokinin to auxin ration for shoot formation.
Increase auxin to cytokinin ratio for root formation.
Acclimatize plantlets in greenhouse;
Transfer to sterile soil outside;
Select for plants that have increased rubisco activity by measuring rate of
photosynthesis;
Repeat whole process of plant tissue culture;
(Half mark each)

(b) Explain the basis of genetic engineering and how genetic engineering can improve the yield [8]
of one named crop plant and one named animal in solving the demand for food in the
world.

Basis (2 marks) Any two:


The genetic code is universal / will code for same amino acids in all organisms;
Central Dogma of molecular biology applies,
when gene from another species can still be expressed in the new host;
By transcription and translation for protein synthesis;
Nucleic acid is the basis of all life-foms;

3 marks for plant example and elaboration:


A named crop: Bt corn
209
13
The problem: The European corn borer is a pest that feeds on corn, hence reducing the
amount of corn being sold. Nature of the infestation makes it difficult to deal with despite
using chemical insecticides and biological control methods.
Source of the gene: Bacillus thuringiensis (Bt) is a soil bacterium that encodes a cry
protein (Bt toxin).
Gene product in the crop: Bt toxin
The solution: Transgenic corn plants containing the Bt gene will produce the protoxin
which when ingested by the insect, is cleaved in the insect gut and the active Bt toxin
causes gut cells to lyse, leading to the death of the insect.

OR

A named crop: Flavr Savr tomatoes


The problem: Polygalacturonase (PG) degrades pectin in the tomato cell wall, causing
tomatoes to soften, and hence a short shelf life.
Source of the gene: Antisense gene
Gene product in the crop: Antisense mRNA
The solution: Antisense mRNA binds by complementary base-pairing to mRNA,
preventing translation and hence reducing PG. This increases the shelf-life of tomato.

OR
Reject
A named crop: Golden rice
The problem: Childhood blindness in developing countries due to vitamin A deficiency in
the diet, which comprised of rice primarily. Outer coat of the dehusked grains (aleurone
layer) does not contain beta-carotene.
Source of the gene: Two beta-carotene biosynthesis genes/ phytoene synthase (psy)
gene and phytoene desaturase (crt 1) gene from the daffodil plant and soil bacterium,
Erwinia uredovora.
Gene product in the crop: beta-carotene produced 23 times more.
The solution: Prevent vitamin A deficiency as beta-carotene is a precursor of Vitamin A.

AND

3 marks for animal example and elaboration:


A named crop: GM salmon
The problem: Atlantic salmon can only grow in summer because their growth hormone
gene is switched off in winter.
Source of the gene: Antifreeze protein (AFP) promoter from ocean pout to drive
expression of Chinook salmon growth hormone gene.
Gene product in the crop: Cold conditions stimulate transcription of growth hormone
gene.
The solution: GM salmons are larger and reach sexual maturity sooner than wild salmon.
They inhibit increased feed use efficiency so they convert more food into tissues more
rapidly.

(c) Discuss how the human genome project can help parents customise their babies and the [6]
possible implications of doing so.
Info from HGP [1] must have both to gain 2 marks.
One can use the information from Human Genome Project to identify a wide
range /genome-wide range (or state number of genes);
of disease causing alleles, alleles that give desirable traits such as height,
intelligence, strength endurance etc;
(Important to state that it is a genome-wide range, since prior to HGP, other genetic
traits would have been known).
How info use to create Designer Babies [1] either one is a necessary point.
Based on info from HGP, Designer babies could 1) be selected for using Pre-
210
14
implantation Genetic Diagnosis (PGD)/(OWTTE) that are disease-free or have
desirable traits;
Or based on the HGP, designer babies could 2) go through genetic modification
for enhancements in desirable traits/removal of undesirable traits discovered
through info from HGP;
Possible Benefits (at least 2 points to gain full marks)
Babies that are born would not have to suffer from genetic defects as those that
have disease-causing alleles are not implanted or have their genes modified to
correct the disease;
Designer siblings of children who have genetic diseases may be created to
provide matching and healthy stem cells for therapy;
Determine the sex of the child as this might prevent babies who have diseases
like haemophilia and Duchenne Muscular Dystrophy, Fragile X syndrome, only
show themselves in male babies;
Ethical concerns (at least 2 points to gain full marks)
For the selection of embryos to implant in PGD, those that are not selected are
discarded or unused which will raise ethical concerns since these embryos have
been fertilized.
Designer siblings may be feel that their existence was for a cure rather than
parents wanting a child/feel inferior to the child who is sick
Choosing the sex of babies can raise ethical concerns especially in societies
where boys are valued more highly than girls, tipping the sex-ratio.

Designer babies with genetical enhancements (cf gene therapy) to add genes for
HIV resistance or longevity or a high IQ?
May lead to a social divide between between those who can afford designer
babies/designer genome and those that cannot.
[Total: 20]
211
NAME: ___________________________________________ CLASS: _________ INDEX: __________

CATHOLIC JUNIOR COLLEGE


JC2 PRELIMINARY EXAMINATIONS
Higher 2

BIOLOGY 9648/01
Paper 1 Multiple Choice WEDNESDAY 4 SEPT 2013
1 hour 15 minutes
Additional Materials: Multiple Choice Answer Sheet

READ THESE INSTRUCTIONS FIRST

Write in soft pencil.


Do not use staples, paper clips, highlighters, glue or correction fluid.
Write and/or shade your name, NRIC / FIN number and HT group on the Answer Sheet in the
spaces provided unless this has been done for you.

There are forty questions on this paper. Answer all questions. For each question, there are
four possible answers, A, B, C and D.

Choose the one you consider correct and record your choice in 2B pencil on the separate
Answer Sheet.

Read the instructions on the Answer Sheet very carefully.

Each correct answer will score one mark. A mark will not be deducted for a wrong answer.

Any rough working should be done in this booklet.


Calculators may be used.

This document consists of 22 printed pages.


[Turn over
212
1 Which of the following are the most likely consequences for a cell lacking Structure X?

I The cell dies because it is unable to make glycoproteins to detect stimuli from its
environment.
II The cell dies from a lack of enzymes to digest food taken in by endocytosis.
III The cell dies from the accumulation of worn out organelles within itself.
IV The cell is unable to reproduce itself.
V The cell is unable to export its enzymes or peptide hormones.

A II and III

B II and V

C III, IV and V

D I, II, III and V

2
213
2 The diagram below shows a biological molecule found in the living cells.

The following are some statements concerning the molecule shown above.

1 It is soluble in water.
2 Double bonds between the carbon and oxygen atoms increase membrane fluidity.
3 It can act as a buffer against extreme pH change.
4 It can undergo condensation to form a polymer.

Which of the above statements are true about this molecule?

A 1 and 3

B 1 and 4

C 2 and 3

D 2 and 4

3 An experiment was carried out to investigate the digestion of starch using amylase at two different
temperatures. A sample was removed from each mixture at 15 second intervals and place onto a
spotting tile well containing two drops of iodine in KI solution. The results are shown in the diagram.

Which shows the correct temperatures and times for the complete digestion of starch?

temperature / C time / s
A 30 3.15
10 0.45
B 30 45
10 195
C 10 45
30 195
D 10 3.15
30 0.45

3
214
4 The graph shows changes in the amount of DNA in a cell during one cell cycle. The letters U Z
marks out the different phases in the cell cycle.

Many drugs that are used to treat cancer work at different time periods during the cell cycle.

(i) Cisplatin binds to DNA and stops free DNA nucleotides from joining together.

(ii) Drug B stops spindle fibres from shortening.

With reference to the cell cycle above, determine where these 2 drugs work.

Cisplatin Drug B

A W X

B W Y

C U X

D U Z

4
215
5 The diagram shows how genetically identical frogs can be developed from unfertilised frog eggs.
The diploid number for frogs is 26.

Which combination of numbers correctly identifies the number of chromosomes in each type of cell?

V W X
A 13 13 26
B 13 26 13
C 13 26 26
D 26 26 13

6 The table below shows a list of characteristics displayed by mutant strains of E.coli during DNA
replication and the possible reasons.

No Characteristics of mutant E.coli Enzymes or functions affected by mutation


1 Okazaki fragments accumulate and DNA ligase activity is missing
DNA synthesis is never completed
2 Supercoils are found to remain at the DNA helicase is hyperactive
flanks of the replication bubbles
3 Synthesis is very slow. DNA polymerase keeps dissociating from the
DNA and has to re-associate
4 No initiation of replication occurs. A-T rich region at origin of replication deleted

Which of the reasons correctly explain the characteristics displayed by the mutant E. coli strains?

A 2 and 3
B 1 and 3
C 1, 3, and 4
D All the above

5
216
7 Part of the amino acid sequence of protein X in normal and disease patients are shown below.

Protein X in normal patient Protein X in disease patient

Lys-His-Pro-Asp-Thr Lys-His-Pro-Gly-Thr

mRNA codons for the amino acids shown in the sequence are as follows:

His: CAU, CAC Asp: GAU, GAC Lys: AAA, AAG

Thr: ACU, ACC, ACA, ACG Gly: GGU, GGC, GGA, GGG Pro: CCU, CCC, CCA, CCG

Which transfer RNA is involved in forming the modified part of protein X?

A B C D

8 Listed below are some antibacterial drugs that can affect the synthesis of bacterial proteins.

Anti-microbial drug Rifampicin Streptomycin Tetracycline

Binds to RNA mRNA misread Prevents binding of


Mode of action
polymerase during translation tRNA to ribosome

Which row shows the correct effects of these drugs on bacterial protein synthesis?

Rifampicin Streptomycin Tetracycline


A Defective protein mRNA does not bind to Polymerization of amino acids
synthesised ribosome does not occur

B mRNA not Defective protein Polymerization of amino acids


synthesised synthesised does not occur

C mRNA not mRNA does not bind to Transcription prevented


synthesised ribosome

D Transcription Defective protein mRNA does not bind to


prevented synthesised ribosome

6
217
9 The diagram represents a length of DNA which includes an operon. E, F, G and H have different
functions.

What correctly identifies E, F, G and H?

E F G H

A operator structural gene(s) regulatory gene promoter

B promoter regulatory gene structural gene(s) operator

C regulatory gene promoter operator structural gene(s)


D structural gene(s) operator promoter regulatory gene

10 The following statements describe bacterial conjugation.

I The F plasmid is made of single-stranded DNA.


II When an F+ donor gives an F plasmid to an F- recipient, both become F-
III When an F+ donor gives an F plasmid to an F- recipient, both become F+
IV When an F+ donor gives an F plasmid to an F- recipient, the donor becomes F-
V When F+ cells are mixed with F- cells, eventually all the cells will become F+.

Which of the statements are true?

A I, II and IV

B II and IV

C III, IV and V

D III and V

7
218
11 Which of the following correctly identifies the reproductive cycles X and Y and structures A and B?

X Y A B
A lytic lysogenic virus DNA provirus
B lysogenic lytic virus DNA prophage
C lytic lysogenic bacterial DNA prophage
D lysogenic lytic bacterial DNA provirus

12 Which feature occurs in the life cycle of the influenza virus?

A Host cell DNA is destroyed by lytic enzymes.

B Viral genome is integrated into the host genome.

C Viral DNA acts as a template for DNA synthesis.

D Viruses enter the host cell by receptor-mediated endocytosis.

8
219
13 The diagram below shows the reproductive cycle of the herpes virus which causes cold sores on the
mouth. With reference to the diagram below, which of the following statements best describes the
herpes virus?

A It is not a retrovirus as it does not contain RNA as its genetic material.


B Its mode of replication is similar to that of HIV.
C Its replication cycle includes a lysogenic phase
D Death of the host cell is necessary for the release of the viral progeny.

14 Which of the following is NOT a feature of eukaryotic gene expression?

A Polycistronic mRNAs are very rare.


B Many genes are interrupted by noncoding DNA sequences.
C RNA synthesis and protein synthesis are coupled.
D mRNA is often extensively modified before translation.

9
220
15 Human telomeres consist of repeating TTAGGG sequences which extend from the ends of the
chromosomal DNA.

When cells undergo mitotic division, some of these repeating sequences are lost. This results in a
shortening of the telomeric DNA.

The diagram shows a eukaryotic chromosome.

What is a consequence of the loss of repeating DNA sequences from the telomeres?

A The cell will begin the synthesis of different proteins.


B The cell will begin to differentiate as a result of the altered DNA.
C The number of mitotic divisions the cell can make will be limited.
D The production of mRNA will be reduced.

16 The acetylation of histones promotes loose chromatin structure. Recent evidence has shown that
chemical modification of histones play a direct role in regulation of gene expression.

Which of the following best explains how acetylation regulates gene expression?

A Helicase action is enhanced by acetylation.


B Acetylation of histones neutralizes their negative charges and encourages binding to DNA
polymerase.
C RNA polymerase works better by binding with acetyl groups.
D When nucleosomes are highly acetylated, chromatin becomes less compact and DNA is
more accessible for transcription.

17 The translation mixture contains a polynucleotide that directs the synthesis of Met-Gly-Gly-Phe-Leu-
Ala. In the presence of Azithromycin, this polymer directs the synthesis of Met-Gly only.

From the information given, which of the following deductions could you make about Azithromycin?

Control Stage Conclusion


A Translational It prevents formation of the initiation complex, which contains the
initiator tRNA and both ribosomal subunits.

B Post translational It inhibits binding of aminoacyl- tRNAs to the A site in the ribosome.

C Translational It blocks translocation of peptidyl transferase-rRNA from the A site to


the P site of the ribosome.

D Post translational It interferes with chain termination and release of the peptide.

10
221
18 Which of the following genetic changes might lead to lower risk of cancer?

1 amplification of a tumour suppressor gene


2 deletion of the promoter upstream of a proto-oncogene
3 loss of function mutation of a proto-oncogene
4 translocation of a tumour suppressor gene to downstream of a strong promoter

A 1 and 3 only
B 2 and 3 only
C 1, 2 and 4 only
D 1, 2, 3 and 4

19 What do chromosomal aberrations and gene mutations have in common and how are they different?

Similarity Difference
A Both may involve addition of Gene mutations always produce dominant
nucleotides. alleles but not chromosomal aberrations.
B Both may not result in disorders. Gene mutations do not involve inversions but
inversions of segments of chromosomes do
occur.
C Both affect nucleotide sequence in Gene mutations occur within a chromosome
DNA molecule. but chromosomal aberrations may occur
across chromosomes.
D Both may not result in a difference in Chromosomal aberrations are more harmful
protein expression. than gene mutations.

20 In poultry, males have two X chromosomes and females have one X chromosome and one Y
chromosome. The gene for feather-barring is sex-linked. The allele for barred feathers is dominant
to the allele for non-barred feathers. A non-barred male is crossed with a barred female.

What ratio of offspring would be expected?

A 1 barred male : 1 barred female


B 1 non-barred male : 1 non-barred female
C 1 barred male : 1 non-barred female
D 1 non-barred male : 1 barred female

11
222
21 Purple buds of the morning glory flower, Ipomoea, open into blue flowers. As the flower opens, the
pH on the vacuoles of the flower epidermal cells increases and this results in a change of colour
from purple to blue.

A mutant purple-flowered morning glory plant carries recessive alleles of a gene B/b, coding for a
membrane-bound ion pump, and is unable to increase the pH of the vacuole. Both normal blue
flowers and mutant purple flowers have the same anthocyanin pigment, coded by the dominant
allele of the gene A/a. Plants with aa cannot produce anthocyanin and they have white flowers.

The genes A/a and B/b are not linked.

A blue-flowered morning glory plant was crossed with a purple-flowered plant. Their offspring
consisted of plants which are blue-flowered, purple-flowered as well as white-flowered.

What were the genotypes of the blue-flowered and purple-flowered plants?

Blue-flowered plant Purple-flowered plant

A AABB AaBb
B AaBb Aabb
C aaBB AaBB
D aaBb aaBb

22 The table below shows the blood group phenotypes resulting from crosses between different
genotypes.

Genotype of 1st Genotype of 2nd parent


A A
parent I I IAIO
IAIO A A and O
IBIO A and AB A, B, B and AB
Two parents have a son who has blood group O and phenylketonuria. One parent has blood group
O and the other has blood group A. Neither parent has phenylketonuria.

What is the probability that the second child of these parents will be a girl with blood group A who
does not have phenylketonuria?

A 1 in 16
B 1 in 8
C 3 in 16
D 3 in 8

12
223
23 When a pure-breeding maize plant with yellow seeds and full endosperm is crossed with another
pure-breeding plant having white seeds and shrunken endosperm, the F1 maize plants are found
with yellow seeds and full endosperm. When the F1 plants are selfed, four classes of descendants
are obtained as shown in the table below:

Colour of seed Shape of endosperm Number of seeds observed in F2


generation (O)
Yellow Full 128
Yellow Shrunken 27
White Full 20
White Shrunken 17

The formula for the chi-squared (2) test is given as follows:

where = sum of
O = observed value
E = expected value

Degrees of Probability
freedom 0.10 0.05 0.01 0.001
1 2.71 3.84 6.64 10.83
2 4.69 5.99 9.21 13.82
3 6.25 7.82 11.35 16.27
4 7.78 9.49 13.28 18.47

Which of the following conclusions made about the inheritance of colour of seed and shape of
endosperm is correct?

A Since p < 0.05, the difference between the observed and expected results is not significant.
Thus the inheritance of colour of seed and shape of endosperm is following Mendels law of
independent assortment.
B Since p > 0.05, the difference between the observed and expected results is not significant.
Thus the inheritance of colour of seed and shape of endosperm is not following Mendels law
of independent assortment.
C Since p < 0.01, the difference between the observed and expected results is not significant.
Thus the inheritance of colour of seed and shape of endosperm is following Mendels law of
independent assortment.
D Since p < 0.01, the difference between the observed and expected results is significant.
Thus the inheritance of colour of seed and shape of endosperm is not following Mendels law
of independent assortment.

13
224
24 Solute Q moves across the plasma membrane via protein channels. Solute R moves across the
plasma membrane by simple diffusion. The rate of movement of each solute into a cell is recorded
and graphed.

The results are shown in the following graph.

It would be reasonable to conclude that

A solute Q is lipid soluble.


B solute Q saturates the protein channels.
C the gradient of the graph for solute R will increase as the temperature decreases.
D as solute R is metabolised within a cell, its rate of movement across the membrane
decreases.

25 The diagram shows a section through a chloroplast.

Where would the products of photophosphorylation be used?

14
225
26 The following graphs shows two separate experiments on carbon dioxide fixation in photosynthesis
using a unicellular green alga and radioactive isotope 14C to label the carbon dioxide. The algae
were actively photosynthesizing before the start of both experiments.

Which of the following correctly identify the graphs E, F, G and H?

E F G H
A RuBP GP GP RuBP
B GP RuBP GP RuBP
C GP RuBP RuBP GP
D RuBP GP RuBP GP

15
226
27 Rotene and oligomycin are two metabolic poisons which affect cellular respiration. The effects of
rotene and oligomycin on aerobic respiration are summarised in the table.

Ability to use glucose Ability to use oxygen ATP yield


Rotene Yes No Decreases
Oligomycin Yes Yes Decreases

Which of the following correctly identifies the specific functions of these two metabolic poisons?

Rotene Oligomycin
A Electron transport inhibitor Inhibits ATP synthase
B Inhibits ATP synthase Electron transport inhibitor
C Dissipate proton gradient Inhibits ATP synthase
D Inhibits ATP synthase Dissipate proton gradient

28 The flow chart shows a series of reactions occurring in an animal cell.

Which of the following statements correctly describes the flow chart?

A Reaction X, which occurs in the cytosol, is an anabolic reaction.

B Reaction Y involves the process of substrate-level phosphorylation, whereby pyruvate is first


converted to a compound called acetyl coenzyme A.

C Reaction Y occurs in the cytoplasm whereas reaction Z occurs in the mitochondria.

D Reaction Z is a catabolic pathway which occurs twice for every glucose molecule to be
completely oxidised.

16
227
29 Which sequence of events correctly describes evolution?

1 Differential reproduction of the spiders occurs.


2 A new selection pressure occurs.
3 Allele frequencies within the spider population change.
4 Poorly adapted spiders have decreased survivorship.

A 2 4 1 3
B 2 4 3 1
C 4 1 3 2
D 4 3 1 2

30 The graph below shows data on a population of a species of moth which shows considerable
variation in colour intensity

Which conclusion can be made from this graph?

A Colour variation is environmentally induced.


B Colour variation is genetically determined.
C Extreme forms are favoured by natural selection.
D The species shows discontinuous variation with respect to colour.

17
228
31 Darwins view of the process of evolution to form new species (speciation) has been reinforced by
more recent discoveries in genetics and cell biology. This is termed the neo-Darwinian view of
evolution.

In this view, which sequence of events is considered most likely to lead to speciation?

competition and
adaptation of behavioural sympatric
A predation leading
population   isolation  speciation
to natural selection

competition and
adaptation of behavioural allopatric
B predation leading
population   isolation  speciation
to natural selection

competition and
adaptation of
predation leading geographical sympatric
C isolated
to natural  isolation   speciation
populations
selection

competition and
adaptation of
predation leading geographical allopatric
D isolated
to natural  isolation   speciation
populations
selection

32 The table shows amino acid sequences from a segment of a viral protein that is found in a number
of different viruses. These viruses were isolated from patients suffering from either the common cold
caused by rhinovirus or flu caused by influenza. Each letter represents a single amino acid.

Virus Amino acid sequence

Rhinovirus 1 P E F P Y N A T T E P T K A V P F Q
Rhinovirus 2 P E F S Y S A V D D P I G E E P F K
Rhinovirus 3 P E F S Y S A G D D P A G E E P F N

Influenza A P E Y H T P V M E P L D P F T M D K
Influenza B P E Y H T P K M E P L D A F A M D N
Influenza C P E Y H T P V M E P L D V F D M D K

Which of the following statements is consistent with the data given?

A The rhinoviruses are more closely related to each other than the influenza viruses are to
each other.

B Influenza virus A is more closely related to influenza virus B than influenza virus B is to
influenza virus C.

C The influenza viruses are less closely related to each other than they are to the rhinoviruses.

D Rhinovirus 2 is more closely related to rhinovirus 3 than it is to rhinovirus 1.


18
229
33 The pBR322 vector is used to clone a eukaryotic gene, which has been digested by the restriction
endonuclease BamHI.

pBR322
Vector
(4361 bp)

Following transformation, bacterial cells were grown in four different media, as shown below:

1 Nutrient broth containing ampicillin

2 Nutrient broth containing tetracycline

3 Nutrient broth containing ampicillin and tetracycline

4 Nutrient broth without ampicillin and tetracycline

Which of the following media would bacterial cells containing the recombinant plasmids grow in?

A 4
B 1 and 2
C 2 and 3
D 1 and 4

19
230
34 Digestion of a 4 kb DNA molecule with EcoRI yields two fragments of 1 kb and 3 kb each. Digestion
of the same molecule with HindIII yields fragments of 1.5 kb and 2.5 kb. Finally, digestion with
EcoRI and HindIII in combination yields fragments of 0.5 kb, 1 kb and 2.5 kb. How would a
restriction map indicating the positions of the EcoRI and HindIII cleavage sites look like?

35 The dashed lines in the template sequence represent a long sequence of bases to be amplified.

Template
5 ATTCGGACTTG - - - - - - - - - - - - - - - - - - - - GTCCAGCTAGAGG 3
3 TAAGCCTGAAC - - - - - - - - - - - - - - - - - - - - CAGGTCGATCTCC 5

Which of the following sets of primers can be used in the PCR for the amplification of the following
DNA sequence?

A 5 GTCCAGC 3 & 5 CCTGAAC 3


B 5 ATTCGGA 3 & 5 CCTCTAG 3
C 5 GGACTTG 3 & 5 GCTGGAC 3
D 5 AUUCGGA 3 & 5 GAUCUCC 3

20
231
36 Which of the following statements about the human genome project (HGP) is false?

A HGP aims to identify all the genes in human and to determine the DNA sequences of these
genes.
B HGP aims to allow genetic testing to take place for earlier detection of genetic diseases
C HGP allows defective genes to be replaced through gene therapy
D HGP allows comparative studies to be made between humans and other organisms to
identify similar genes associated with diseases.

37 What is the role of stem cells with regard to the function of adult tissues and organs?

A Stem cells are undifferentiated cells that divide asymmetrically, giving rise to one daughter
cell that remains a stem cell and one daughter that will differentiate to replace damaged and
worn out cells in the adult tissue or organ.
B Stem cells are embryonic cells that persist in the adult, and can give rise to all of the cell
types in the body.
C Stem cells are differentiated cells that have yet to express the genes and produce proteins
characteristic of their differentiated state, and do so when needed for repair of tissues and
organs.
D Stem cells are fully differentiated cells that reside under the surface of epithelial tissue, in
position to take over the function of the tissue when the overlying cells become damaged or
worn out.

38 Which of the following regarding embryonic stem cells and hematopoietic stem cells is true?

A As embryonic stem cells develop, they turned into hematopoietic stem cells as they lose their
ability to differentiate into all types of cells.
B Embryonic stem cells have more genes than hematopoietic stem cells and thus are able to
form more types of cells.
C Under normal conditions, embryonic stem cells express more of their genes compared to the
hematopoietic stem cells.
D Both stem cells are derived from the zygotic stem cells with the hematopoietic stem cells
having a lowered telomerase activity compared to the embryonic stem cells.

39 Some scientists are concerned about the release of genetically modified microorganisms into their
natural habitat.

What is the most likely reason for this concern?

A The microorganisms may reproduce quickly.


B The microorganisms may not survive.
C The mutation rate of the microorganisms would increase.
D The transfer of changed genes to other organisms.

21
232

40 An attempt was made to produce Golden rice. To determine whether or not DNA from the daffodils
and the bacterium had been successfully incorporated in the DNA of the rice, scientists used PCR
and gel electrophoresis to produce DNA profiles.

The following DNA profiles belong to the original strain of rice, three strains I to III of genetically
modified Golden rice, and the species of daffodil and bacterium used to incorporate beta-carotene
genes in the rice.

Which one of the strain(s) of Golden rice has successfully incorporated DNA from both the daffodil
and the bacterium?

A Strain I only

B Strain II only

C Strain I and III only

D Strain II and III only

END OF PAPER

22
233
2013 CJC H2 Biology Prelim Paper 1 Answers

1 D 21 B
2 B 22 C
3 B 23 D
4 A 24 B
5 C 25 A
6 C 26 C
7 D 27 A
8 B 28 D
9 C 29 A
10 D 30 C
11 C 31 D
12 D 32 D
13 A 33 D
14 C 34 A
15 C 35 B
16 D 36 B
17 C 37 A
18 D 38 D
19 C 39 D
20 C 40 B

23
234
1

Name: Index Class:


Number:

DUNMAN HIGH SCHOOL


Preliminary Examination
Year 6

H2 BIOLOGY
9648/01
Paper 1 Multiple Choice Questions 23 September 2013
1 hour 15 mins
Additional Materials: OTAS sheet

INSTRUCTIONS TO CANDIDATES:
DO NOT TURN THIS PAGE OVER UNTIL YOU ARE TOLD TO DO SO.
READ THESE NOTES CAREFULLY.

Section A MCQ [40 marks]

There are forty questions in this paper. Answer all questions. For each question there are four
possible answers A, B, C and D.
Choose the one you consider correct and record your choice in soft pencil on the separate Answer
Sheet.

Each correct answer will score one mark. A mark will not be deducted for a wrong answer. Any
rough working should be done in this booklet.

This document consists of 20 printed pages and 0 blank page.

[Turn over
DHS 2013
235
2

Answer all questions in this section.

1 A piece of mammalian tissue was homogenised and subjected to differential centrifugation to


yield four subcellular fractions. The activity within each fraction, of four different types of
enzyme, A, B, C and D, was investigated.

Which bar chart shows the results of investigating hydrolytic enzyme activity?

2 Membranes in cells include the following components.


1 cholesterol
2 glycoproteins
3 phospholipids
4 proteins
Which component is the most important for these functions of membranes?

3 In amylopectin, the bonds found between glucose units are __________ .

A (14) glycosidic bonds only


B (14) glycosidic bonds and (16) glycosidic bonds only
C (14) glycosidic bonds only
D (14) glycosidic bonds and (16) glycosidic bonds only
236
3

4 The figure shows a series of reactions in a metabolic pathway.


D
D E
A B C
A B C

J K
J K L

Enzyme C catalyses the splitting of C into D and J. Assuming that product E is an allosteric
inhibitor of C, which of the following would likely happen if E were not consumed in a
subsequent reaction?

A The rate of production of D would increase.


B The rate of production of E would remain the same.
C The rate of production of L would remain the same.
D The rate of production of all products D, E, J, K would decrease.

5 The figure shows the life cycle of the water flea, Daphnia. The cells of individual R contain
10 chromosomes.
R

P
T

Which of the following are correct?

Individual Ploidy Number of Reason for choice


level chromosomes
I P 2n 20 The cells of P can undergo both mitosis
and meiosis.
II Q 2n 20 P produces eggs Q by mitosis which
develop into females.
III S n 10 The gametic cells of P have undergone
normal meiosis.
IV T 2n 20 Random fertilisation of haploid gametes
from R and S occurred to form zygote T.

A I and II only
B I and III only
C II and IV only
D All of the above
237
4

6 Bacteria grown in 15N for many generations were transferred to 14N for further replication.
DNA from the bacteria was extracted and separated by density gradient centrifugation.

Their results are summarised in the following diagram.

What are the percentages of DNA containing 15N and 14N in tube 4?

7 What is the basis for the difference in the synthesis of the leading and lagging strands of
DNA molecules?

I The anti-parallel arrangement of the DNA strands.


II The RNA primers are required to initiate DNA elongation.
III DNA polymerase joins new nucleotides to the 3 end of the growing strand.
IV Helicase and single-stranded binding proteins work at the 5end of the DNA strand.

A I and III

B II and IV

C I and IV

D II and III
238
5

8 The diagram below shows part of a molecule of mRNA bound to a ribosome.

Which of the following is FALSE about molecule X?

I. It is formed by RNA polymerase in the nucleus.


II. It is able to form hydrogen bonds with mRNA.
III. An amino acid was attached to it by the enzyme amino-acyl tRNA transferase.
IV. It is held in the amino-acyl tRNA binding site of the ribosome.

A III only
B IV only
C I and II
D III and IV

9 Which of the following shows the possible effects of a single nucleotide substitution in each
of the following locations in a gene on the production of the protein it codes for?

Promoter Transcription Start codon Stop codon Middle of an


terminator intron
A No protein Protein product Protein product Protein product Too much
product is is shorter than is longer than is normal protein product
produced normal normal is produced
B Too much Protein product No protein Protein product Protein product
protein product is normal product is is longer than is normal
is produced produced normal
C Protein product Protein product Protein product Too much Protein product
is normal is longer than is shorter than protein product is longer than
normal normal is produce normal
D Protein product Too much Protein product Protein product No protein
is longer than protein product is normal is shorter than product is
normal is produced normal produced
239
6

10 Two new viruses E and Z, which infect eukaryotic cells, have been identified.

In one experiment, the nucleic acid from each virus is isolated and analyzed over a range of
temperatures. The light absorbance of nucleic acids changes when denaturation or
annealing occurs. The behaviour of the nucleic acid from each virus is shown in the graph.

Light Virus E
absorbance
(arbitrary units)
Virus Z

Temperature (C)

In a second experiment, it is found that treatment with reverse transcriptase inhibitors or with
inhibitors of DNA synthesis blocks the ability of virus Z to infect cells. In contrast, reverse
transcriptase inhibitors have no effect on the ability of virus E to infect cells but DNA
synthesis inhibitors block infection by virus E.

Which of the following conclusions can be drawn from both the experiments?

A The genome of virus E is single-stranded RNA and that of virus Z is double-stranded


DNA.
B The genome of virus E is double-stranded DNA and that of virus Z is single-stranded
RNA.
C The genome of virus E is double-stranded RNA and that of virus Z is single-stranded
DNA.
D The genome of virus E is double-stranded DNA and that of virus Z is double-stranded
RNA.
240
7

11 Which of the following correctly identifies the reproductive cycles X and Y and structures A
and B?

X Y A B
A lytic lysogenic virus DNA provirus

B lysogenic lytic virus DNA prophage

C lytic lysogenic bacterial DNA prophage

D lysogenic lytic bacterial DNA provirus

12 Bacterial strain A has been infected with viruses. Upon lysis, the virions produced are
introduced to Bacterial strain B. After a short period of time, a new strain of bacteria is
detected that is very similar to strain A but has a few characteristics of Strain B. Which is the
process that leads to the production of the new strain?

A Conjugation
B Transduction
C Transformation
D Transposition
241
8

13 A student has two cultures of cells. Cell culture A consists of a culture of normal cells
whereas cell culture B consists of a culture of cells dividing uncontrollably (isolated from a
tumour). He wanted to determine if the uncontrolled growth (in culture B) was the result of
an oncogene or a mutated tumour suppressor gene. He decided to fuse the cells from the
two cultures (i.e. culture A with culture B) to form a hybrid cell line. As he was not certain of
the expected results, he wrote down all the possible combinations in terms of the expected
results and the gene that is involved as shown below.

Expected results of hybrid cells Gene involved

1 Grows normally Oncogene

2 Grows uncontrollably Oncogene

3 Grows normally Mutated tumour suppressor gene

4 Grows uncontrollably Mutated tumour suppressor gene

Of the four possibilities, only two of them are set correctly. Based on your understanding of
the genetic changes that has occurred in oncogenes and the mutated tumour suppressor
genes, choose one of the options below which shows the two correct possibilities.

A 1 and 3

B 1 and 4

C 2 and 3

D 2 and 4

14 Four different genes are regulated in different ways.

Gene 1 undergoes tissue-specific patterns of alternative splicing.


Gene 2 is part of a group of structural genes controlled by the same regulatory
sequences.
Gene 3 under some circumstances undergo methylation.
Gene 4 codes for a repressor protein which acts at an operator site close by.

Which row of the table correctly identifies which genes are prokaryotic and which are
eukaryotic?

prokaryotic eukaryotic
A 1 and 2 3 and 4
B 1 and 3 2 and 4
C 2 and 3 1 and 4
D 2 and 4 1 and 3
242
9

15 Which of the following contains repetitive DNA sequences?

I. Telomere
II. Centromere
III. Intron
IV. Control element

A I and II
B II and III
C III and IV
D I, II and III

16 A cross between a round-leafed, tall plant and round-leafed dwarf plant produced the
following offspring:
Key
121 round-leafed, tall plant R round leaf
124 round-leafed, dwarf plant r oval leaf
42 oval-leafed, tall plant T tall
37 oval-leafed, dwarf plant t dwarf

What were the genotypes of the parents?

A RrTt x Rrtt
B RrTt x RRtt
C RrTT x Rrtt
D RrTT x RRtt
243
10

17 The inheritance of a disease in a family is presented in a pedigree tree below.

What type of inheritance could this disease show?

I Autosomal dominant

II Autosomal recessive

III Sex-linked dominant

IV Sex-linked recessive

A I only
B II only
C I and III
D II and IV

18 Agoutic mice have banded hairs, giving a grey colour. Black mice have unbanded hairs.
White mice have no pigment. A cross between a homozygous black mouse and a white
mouse produced offspring with agouti hair. Another cross between the black mouse and
another white mouse produced some offspring with agoutic hair and some with black hair.

What explains these observations on the phenotype of hair of mice?

A There is a single gene with two codominant allele, black and white.
B There is a single gene with three alleles in a dominance series, black grey white.
C There are two epistatic genes, one controlling pigment production and one controlling
banding.
D There are two linked genes, one controlling pigment production and one controlling
banding.

244
11

19 1. Light joins carbon dioxide to an acceptor compound which is then reduced by


hydrogen obtained from water.
2. Carbon dioxide combines with an acceptor compound and this is reduced by
hydrogen split from water by light
3. Light splits carbon dioxide and the resulting carbon then combines with oxygen and
hydrogen obtained from water.
4. Light splits water and the resulting hydroxyl group combines with a compound which
has incorporated carbon dioxide

Which of the following statement is/are CORRECT outline of the main events in
photosynthesis?

A 1 only
B 2 only
C 1 and 2
D 3 and 4

20 Dinitrophenol is a metabolic poison that can lodge within the thylakoid membranes of
chloroplasts. It then provides an alternative route for H+ ions to diffuse across the thylakoid
membranes. In what way will the Calvin cycle be affected in chloroplasts poisoned by
dinitrophenol?

A No change in rate as Calvin cycle occurs in the stroma and not at thylakoid membranes.
B The rate of Calvin cycle will increase as pH in the stroma will decrease towards the
optimum for enzymes involved in the cycle.
C The rate of Calvin cycle will decrease with the accumulation of glycerate-3-phosphate.
D The rate of Calvin cycle will decrease with the accumulation of glyceraldehyde-3-
phosphate.

245
12

21 Isotopes of oxygen can be used to distinguish between oxygen absorbed by plants and
oxygen evolved.

A mixture of oxygen isotopes 16O2 and 18O2 was supplied to a suspension of the
unicellular alga Chlorella which had previously been exposed to 16O2 only. During the
following hour, changes in concentration of these gases in the suspension were measured
in light and dark conditions.

The graph shows the results.

16
O2

18
oxygen O2
concentration/
arbitrary units

dark light dark light

time
16
What caused the concentration of O2 to rise in light?

A H218O was being photolysed more rapidly than H216O.


16
B O2 was being absorbed at different rates in light and dark.
16
C O2 was being produced in photosynthesis but was not being absorbed in
respiration.
16
D O2 was being produced in photosynthesis faster than it was being absorbed in
respiration.

22 Both glucose and appropriate enzymes are necessary for the process of glycolysis to
begin.

Which additional compound must also be present?

A Acetylcoenzyme A
B ATP
C Pyruvate
D Reduced NAD
246
13

23 In which of the following setups would carbon dioxide be produced?

I II III
A Mitochondria + glucose Cell lysate + pyruvate Chloroplast + RuBP
carboxylase
B Mitochondria + starch + Cell lysate + RuBP Cell lysate + pyruvate
amylase carboxylase
C Cell lysate + glucose Cell lysate + pyruvate Mitochondria + pyruvate
D Cell lysate + pyruvate Chloroplast + RuBP Lysosome + starch
carboxylase

24 Cyanide is a poison that prevents the production of ATP.

What will happen if a resting axon is treated with such a poison?

A The charge outside the axon membrane does not change.


B The charge inside the axon membrane becomes more negative.
C The charge outside the axon membrane becomes less positive.
D The charge inside the axon membrane becomes less positive.

25 The diagram represents a transverse section through a neurone as seen under an electron
microscope. Which of the following most appropriately describes the function of X?

A Insulate the neurone to maintain homeostatic conditions.


B Provide mechanical protection to the axon.
C Prevents ion exchange between the axon and extracellular fluid.
D Speeds up nervous transmission by allowing impulse to travel along it.
247
14

26 A particular type of autoimmune disease causes antibodies to destroy acetylcholine


receptors of neurons. What effect will this have on the functioning of the nervous system?

A Presynaptic neurons will be unable to release neurotransmitters into the synaptic


clefts.
B Neurons will be unable to propagate action potentials along their axons.
C Depolarized neurons will be unable to reestablish an ionic gradient across their
membranes.
D Postsynaptic neurons will be unable to detect signals from presynaptic neurons.

27 These statements describe an action potential. Which one of the following is FALSE?

A Its speed can be increased by myelination of the neurone.


B Its depolarization phase is an example of a positive feedback.
C It is regarded as an all-or-nothing response.
D It is initiated by the opening of voltage-gated sodium and potassium channels.

28 The diagram represents a G-protein coupled receptor on the cell surface membrane. A
peptide hormone ligand is bound to the receptor and initiates the production of a second
messenger.

What is the second messenger?

A a peptide hormone
B ATP
C cyclic AMP
D kinase
248
15

29 Which of the following is a correct statement regarding the hormonal control of blood glucose
level?

Hormone Receptor which hormone Effect


binds to
A Insulin Tyrosine kinase receptor Increase glycogenesis
B Insulin G protein-linked receptor Increase gluconeogenesis
C Glucagon Tyrosine kinase receptor Increase gluconeogenesis
D Glucagon G protein-linked receptor Increase glycogenesis

30 The diagram below shows an example of cell signaling. Identify the stages P, Q and R.

P Q R
A amplification ligand-receptor interaction phosphorylation
B ligand-receptor interaction dephosphorylation amplification
C transduction phosphorylation amplification
D ligand-receptor interaction amplification dephosphorylation

249
16

31 Molecular homology of haemoglobin is used to determine evolutionary relatedness in a range


of mammals. Either nucleic acids or proteins can be sequenced. The sequence of nucleic
acids and proteins are compared.

Which sequence will vary LEAST in related mammals?

A Amino acid
B DNA
C Edited mRNA transcript
D Primary mRNA transcript

32 When organochlorine insecticides such as DDT were in widespread use, mosquitoes in


malarial regions developed resistance more rapidly than houseflies in Britain.

What could account for the difference in the rates of the development of resistance?

A More insecticides were used in Britain.


B More insecticides were used in malarial regions.
C Mosquitoes have fewer random mutations when exposed to insecticides.
D Mosquitoes have more random mutations when exposed to insecticides.

33 Fish in the Artic and Antarctica have antifreeze proteins. But these two population of fish
diverged long before the antifreeze genes evolved. It was found that the genes that code for
the antifreeze proteins in fish, at the Artic and Antarctica, are quite different. This is evidence
that separate and independent episodes of molecular evolution occurred, and obtained the
same functional result.

Which of the following processes is responsible for the antifreeze property in the two
populations of the fish?

A Reproductive isolation
B Divergent evolution
C Convergent evolution
D Geographical isolation
250
17

34 Which of the following is TRUE of a genomic library?

A It is a catalogue of the different RFLPs of organisms within a population.


B It is a collection of DNA fragments that make up the entire genome of a particular
organism.
C It is a collection of DNA fragments created by reverse transcriptase which are then
inserted into vectors.
D It is a catalogue of all genes of an organisms genome which have been sequenced
thus far.

35 The diagram below shows the changes in temperature in a thermocycler over time during
polymerase chain reaction.

X X

Z Z

Y Y

Which of the following statements is TRUE?

A Elongation of new daughter strand occurs during Y.


B Taq polymerase functions optimally at X.
C DNA primers are anneal to the DNA template during Z.
D Double stranded DNA template denatures into single strands during X.
251
18

36 The diagram below shows the position of four restriction sites J, K, L and M for four different
restriction enzymes in a single plasmid. The distances between these sites are measured in
kilobases of DNA.

The plasmid was cut using only two restriction enzymes. The resulting fragments were
separated by gel electrophoresis. The positions of the fragments are shown in the chart
below.

Which restriction sites were cut?

A J and K
B L and M
C J and M
D L and K
252
19

37 A synthetic plasmid having an origin of replication (ori), a restriction site (R), two genetic
markers, -galactosidase gene (-gal) and tetracycline-resistance gene (tetr), was used as a
vector to transfer the human growth hormone gene into bacteria. The -gal gene codes for an
enzyme -galactosidase, which breaks down a colourless compound called X-gal to a blue
compound.

At which position should the restriction site (R) be located on the plasmid in order to identify
the desired transformed bacteria on an agar medium containing both X-gal and tetracycline?

38 How is the polymerase chain reaction (PCR) similar to the replication of DNA?

I DNA is heated to break hydrogen bonds


II DNA unzips
III free nucleotides are used
IV DNA polymerase are required

A I only
B II and IV only
C I, II and III only
D II, III and IV only
253
20

39 The autoradiograms obtained below (after electrophoresis and Southern Blotting) show
human DNA digested with a specific restriction enzyme and probed with labeled rRNA.

On the left, the probe used was 28S rRNA. On the right, the probe used was 18S rRNA.

If the arrows in the following map show the location of the restriction sites of this restriction
enzyme, which map best explains the results shown above?

40 Which of the following explains why insulin expressed in a prokaryotic host cell is not
functional?

I. Prokaryotic cells cannot replicate human DNA.


II. Prokaryotic cells cannot splice mRNA.
III. Prokaryotic cells do not possess sER.
IV. Prokaryotic protein expression machinery stops at translation.

A I only
B II only
C II and III
D I, II, III and IV

END OF PAPER
254
21

2013 Y6 Preliminary Exam H2


MCQ Answer Scheme

1 C 21 D
2 B 22 B
3 B 23 C
4 D 24 A
5 D 25 C
6 C 26 D
7 A 27 D
8 D 28 C
9 B 29 A
10 B 30 B
11 C 31 A
12 B 32 B
13 C 33 C
14 D 34 B
15 A 35 D
16 A 36 B
17 B 37 A
18 C 38 D
19 B 39 B
20 C 40 B

255
1

Name: Index Number: Class:

DUNMAN HIGH SCHOOL


Preliminary Examination
Year 6

H2 BIOLOGY 9648/02
Paper 2 Structured and Free-Response Questions 16 September 2013
2 hours
Additional Materials: Writing paper

INSTRUCTIONS TO CANDIDATES:
DO NOT TURN THIS PAGE OVER UNTIL YOU ARE TOLD TO DO SO.
READ THESE NOTES CAREFULLY.

Section B Structured Questions For Examiners Use


Answer all questions.
Section A [40]
Write your answers on space provided in the
Section B [80]
Question Paper.
1 / 10
Section C Free-Response Questions
2 / 5
Answer one question. Your answer to Section C
3 / 5
must be in continuous prose, where appropriate.
Write your answers on the writing paper provided. 4 / 10
Answer (a) and (b) on separate pieces of paper.
5 / 10

Submit your answers to Sections B and Section 6 / 9


C (a) and (b) separately.
7 / 13

8 / 9
INFORMATION FOR CANDIDATES
9 / 9
Essential working must be shown.
Section C [20]
The intended marks for questions or parts of
questions are given in brackets [ ]. 1/2

Total [140]

This document consists of 23 printed pages and 1 blank page.


[Turn over
DHS 2013
256
2

Section B: Structtured Ques stions (80 marks)


m For
Answer
A all q
questions in this section
n. Examiners
E
use

Questio
on 1

Fig 1.1 below show ws a pancreatic acinarr cell, an ex


xocrine cell involved inn the synthe
esis and
secretio
on of pancrreatic diges
stive enzym
mes. It show densing vaccuoles in the Golgi
ws the cond
region a
and mature secretory vesicles
v at tthe cell ape
ex.

Fig 1.1
257
3

(a) Naame the org ganelles lab


belled A, B and C and
d explain their role in ppancreatic enzyme
e
syynthesis. [6]]

(b) Onne major pa


ancreatic pro
otease synth
hesized by th
he pancreatic acinar cellls is trypsin
n, which
is synthesize
ed and pa ackaged intto secretoryy vesicles as the innactive proenzyme
ypsinogen.
try
Fig 1.2 showss the structu
ure of trypsin
n.

Fig 1.2
2
258
4

(ii) Describ
be region X. [2]

Trypsinogen is con nverted to tryypsin by the


e removal off a hexapepptide from its
s amino
terminuus after it is released in
nto the diges F 1.3 shoows the structure of
stive tract. Fig
trypsin and trypsino ogen.

Activ
ve site

Fig 1.3

(ii) With re
eference to Fig 1.3, exxplain how the removal of a hexxapeptide frrom the
amino terminus
t of trypsinoge n can resultt in the activ
vation of thee enzyme. [2]

Total: [10]

259
5

on 2
Questio For
Examiners
E
use
Fig. 2.1 shows a fe A F) of m eiosis in the
ew stages (A e lily, Lilium
m longiflorum
m.

A B C

D E F
Fig. 2.1

(a) W
With referencce to Fig. 2.1,
2
(ii) arrange
e these stag
ges in the ccorrect sequence. [1]

(ii) describ
be the main events thatt will occur to
t complete
e meiosis affter stage A.
A [2]
260
6

(b) Explain how meiosis results in genetically non-identical gametes. [2]

Total: [5]
261
7

Questio
on 3 For
Examiners
use
A studyy of a bacte
erias chrommosome sh howed that it possesse es a lac opperon to allow the
bacteriu
um to metabbolise lactos
se. The stru
ucture of the n is shown in Fig 3.1 below:
e lac operon b

Fig 3.1

The graaph in Fig 3.2 below shows


s the growth of bacteria
b in the presencce of gluco
ose and
lactose.

Fig 3.2

(a) W 2, state the main respiratory substrate of the bacteria in Part I.


With referencce to Fig 3.2
[1]
262
8

(b) Explain the changes in graph C in Part II and Part III. [4]

Total: [5]
263
9

Question 4 For
Examiners
use
(a) Alpha tropomyosin (-TM) is a protein found in muscles as well as several other
tissues in different isoforms.

Fig 4.1 shows the organisation of the -TM gene with exons shown as boxes.

Fig 4.1

Fig 4.2 shows the -TM mRNA transcripts found in various tissues.

Fig 4.2

(i) Explain how the -TM gene can produce different -TM isoforms in different
cell types. [3]
264
10

(ii) Explain the advantage of the process mentioned in (i). [1]

(b) Telomeres are found at the ends of eukaryotic chromosomes. They contain a series of
tandem repeat sequences and end with a single-stranded section called the 3
overhang. This 3 overhang results from the limitation of DNA polymerase to replicate
all the way to the end of the chromosome. This is known as the end-replication
problem.

Fig 4.3 shows the structure of a telomere.

Fig 4.3

(i) Explain the end-replication problem. [3]

(ii) Explain the role of the T loop structure shown in Fig 4.3. [1]
265
11

An enzyme called telomerase catalyses the lengthening of telomeres. Telomerase is


typically present only in germ cells and some rapidly dividing somatic cells. It has also
been found to be present in high concentrations in cancerous cells.

(iii) Explain how high telomerase activity promotes cancer. [2]

Total: [10]
266
12

Question 5 For
Examiners
use
In Drosophila flies, the wing shape and eye colour are controlled by two different genes
found on separate chromosomes. The gene controlling wing shape is found on chromosome
2 and has 2 alleles. Curly wings were found to be dominant over normal wings. Fig 5.1 and
Fig 5.2 shows the normal wing and curly wing phenotype respectively.

Fig 5.1: Normal Wing Fig 5.2: Curly Wing

The gene controlling eye colour is found on the X chromosome and has two alleles. It was
observed that red eye trait is dominant over white eye trait.

(a) Explain what is meant by the term dominant? [1]

(b) Construct a genetic diagram showing the phenotypes of the offspring and the
expected ratio of phenotypes when a white eyed female is crossed with a red eyed
male, both are heterozygous for curly wings. [4]
267
13

(c) A geneticist conducted an experim


ment on the e cross mentioned earrlier. 240 offspring
we
ere obtaine
ed from the experiment
e . The pheno he offspring were as follows:
otypes of th

83
3 red-eyed females
f witth curly wing
gs
41
1 red-eyed females
f witth normal wwings
78
8 white-eyed males witth curly wing
gs
38
8 white-eyed males witth normal wwings

A Chi-square
e test was performed o n the resultts on the cro
oss.

Fig 5.3
3

((i) ulate the Chi-square va


Calcu ng your workings in thee space belo
alue, showin ow. [2]

((ii) Use your


y calculated Chi-squ uare value anda the tablle of probabbilities proviided in
Fig 5..3 to find the
e probabilityy that the difference is due to cha nce. [1]

(iiii) State what conclusions mayy be drawn from


f the Ch
hi-square teest. [2]

Tottal: [10]
268
14

Question 6 For
Examiners
use
C3 plants undergo photorespiration, a process by which RuBP has oxygen added to it by
the enzyme RuBisCO, instead of carbon dioxide during normal photosynthesis.
Photorespiration can occur when carbon dioxide levels are low and when temperatures are
high, but unlike respiration, photorespiration produces no ATP.

Fig. 6.1 shows the rate of photosynthesis (measured by rate of CO2 fixation, graph drawn in
solid line) and the rate of photorespiration (measured by rate of O2 fixation, graph drawn in
dotted line) in wheat over a typical summer day.

Fig. 6.1

(a) With reference to Fig. 6.1, explain how stomata size affects the overall rate of
photosynthesis in wheat. [3]

(b) State the type of inhibitor that oxygen is to the enzyme RuBisCO. [1]
269
15

(c) Some plants have mechanisms to reduce photorespiration, such as the C4 plants.
These plants undergo additional steps of capturing carbon dioxide using another
enzyme (PEP carboxylase) to form oxaloacetate, before undergoing the Calvin cycle.
PEP carboxylase has been found to have a lower Km than RuBisCO for carbon
dioxide. Examples of C4 plants include sugar cane and maize.

Describe the type of environment that C4 plants are usually found in and explain how
PEP carboxylase enables C4 plants to be better adapted to their environments. [3]

(d) Explain why Calvin cycle is still necessary in C4 plants. [2]

Total: [9]
270
16

on 7
Questio For
Examiners
E
use
Fig. 7.1
1 shows thee observed changes inn permeability of the ax
xonal plasmma membra
ane of a
neuronee to sodium
m ions and to
o potassium
m ions durin
ng a single action
a potenntial.

Fig. 7.1

(a) De
escribe and
d explain the f sodium ions betweeen 0.5 ms and
e shape of the curve for a 0.7
mss. [4]
271
17

(b) During an action potential, the membrane potential rises to +40 mV and then falls. Use
information from Fig. 7.1 to explain the fall in membrane potential. [3]

(c) Tetraethylammonium (TEA) is a drug that blocks voltage-gated potassium channels.

(i) Predict and explain the effect of TEA on the axonal plasma membrane of the
neurone. [2]

(ii) Predict and explain the effect of TEA if it could be applied selectively to a
presynaptic neurone that releases an excitatory neurotransmitter. [2]

(d) After exercise, some ATP is used to re-establish the resting potential in axons. Explain
how the resting potential is re-established. [2]

Total: [13]
272
18

Question 8 For
Examiners
use
(a) Fig. 8.1 shows the sequence of events in the inositol phosphate signaling pathway.

Fig. 8.1

With reference to Fig. 8.1,

(i) describe the main sequence of events that leads to the activation of protein
kinase C. [4]

(ii) explain why the signal molecule cannot act directly on the DNA in the nucleus.
[1]
273
19

(iii) suggestt and explain how an IP


P3-triggered
d response is terminateed. [2]

(b) Etthylene glyccol tetra-ace


etic acid (EG
GTA) is a chelating agent that is ccapable of bonding
b
2+
se
ecurely to io
ons such as s Ca . The pharmacolo ogical effec
cts of EGTAA have been n widely
stu
udied.

In one study, smooth muscle


m cellss were transferred to a medium ccontaining varying
co
oncentration n of EGTA. Fig. 8.2 sshows the effects
e of EGTA on th e released rate of
Caa2+ in an ino
ositol phosp
phate signallling pathwa
ay.

Fig. 8.2
2

Coomment if the data in Fig. 8.2 su pports the hypothesis that high cconcentratio on of up
to 14.3mM off EGTA can effectively inhibit the binding
b of IP
P3 to its recceptor. [2]

To
otal: [9]

274
20

on 9
Questio For
Examiners
E
use
(a) IIn Lake Tan
nganyika in Africa, therre are six sp
pecies of fis
sh of the geenus Tropheeus and
a much larg ger numberr of distincttly coloured
d subspecie es of each oof the six species.
s
TTropheus species
s are small fish that are co onfined to issolated rockky habitats around
tthe shores of
o Lake Tannganyika.

TThe six spe ecies evolve ed during th


he primary radiation phase whenn the lake was
w first
ffilled, aboutt 1.25 millio
on years aago. They arose
a from river dwelliing ancesto
ors and
a available niches in th
tthen filled all he lake.

SSecondary radiations into the m any subspe ecies occurrred during the last 2002 000
yyears. Sommetime durin ng this peri od, the water level in the lake feell, resulting
g in the
fformation of three sepaarate lake b basins. The
ese basins persisted
p foor many tho ousands
oof years beffore the watter level rosse again.

F
Fig. 9.1 shows an outtline map o of the lake and the loc
cation of thhe three tem
mporary
b
basins caussed by lowe
ering of lake
e levels.

Fig. 9.1

EExplain howw natural selection


s co
ould have caused
c the
e evolution of the six closely
rrelated speccies in the primary
p radiiation. [4]
275
21

(b) Recent research has compared DNA sequence of cytochrome b gene of the six
different species of Tropheus. Fig. 9.2 shows part of the cytochrome b DNA
sequence.

T. brichardi CTTTTAACTATGATAACAGCTTTTGTGGGCTACG
T. kasabae CTTTTAACTATGATAATAGCTTTTGTGGGCTACG
T. annectens CTTTTAACTATGATAACAGCTTTTGTGGGCTACG
T. moorii CTTTTAACTATGATAACAGCTTTTGTGGGTTACG
T. polli CTTTTAACTATAATAACAGCTTTTGTGGGCTACG
T. duboisi CTTTTAACCATGATAACGGCTTTTGTAGGCTATG
Fig. 9.2

A family tree based on differences between the DNA sequences is shown in Fig. 9.3.

Fig. 9.3

With reference to Fig. 9.2, explain why different species of Tropheus can have similar
DNA sequence in their cytochrome b gene. [2]
276
22

(c) With reference to Fig. 9.2 and Fig 9.3, describe how the time of divergence of the
species could be estimated. [3]

Total: [9]
277
23

Section C: Free-Response Questions (20 marks)

Answer only one question.


Write your answers on the writing paper provided.
Your answers should be illustrated by large, clearly labelled diagrams, where appropriate.
Your answers must be in continuous prose, where appropriate.
Your answers must be set out in sections (a), (b) etc., as indicated in the question.
A NIL RETURN is required.

Question 1

(a) Compare transcription and translation in eukaryotes. [10]

(b) (i) Describe how molecular methods can be used to classify organisms. [6]
(ii) Suggest how bacteria evolve. [4]

OR

Question 2

(a) Outline the differences between prokaryotic and eukaryotic control of gene expression. [10]

(b) (i) Describe Krebs Cycle. [5]


(ii) Compare oxidative phosphorylation and photophosphorylation. [5]

Total: [20]

Please answer (a) and (b) on separate sheets of writing paper.

You will need to hand in (a) and (b) separately.

END OF PAPER
278
24

BLANK PAGE


279
25

DUNMAN HIGH SCHOOL


PRELIMINARY EXAMINATION 2012
YEAR SIX
H2 BIOLOGY (9648)

Answer Scheme

Structured Answers

Question 1
(a)
A Rough Endoplasmic Reticulum;
Ribosomes on RER responsible for synthesis of proteins (enzymes) to be secreted;
OR
Packaging of proteins (enzymes) for transport to Golgi Apparatus in transport vesicles;

B Mitochondrion;
Aerobic cellular respiration for the synthesis of ATP necessary for protein synthesis;

C Nucleus;
Contains the genes that code for the various pancreatic enzymes which are transcribed into
mRNA for protein synthesis;

(b)(i)
Alpha helix,
held by H bonds located between CO and NH groups (of the main polypeptide chain);
Between n and n+4 residues in the chain;

(ii)
Cleavage of hexapeptide from amino terminus shortens primary structure which changes the
secondary structure such that pleated sheets can form;
this creates a pocket / groove for the substrate to fit into on the surface of the tertiary
structure that functions as the active site of the enzyme;

Question 2
(a) (i) D, F, C, B, A, E ;

(ii)
The chromosomes uncoil / decondense and become very indistinct to form chromatin;
Spindle fibres disassemble / disintegrate / dissassociate;
Nuclear envelopes reform around each nucleus;
2 max

(b)
Crossing over between non-sister chromatids / homologous chromosomes during prophase
I;
Independent assortment and random segregation of homologous pairs of chromosomes
during metaphase I and anaphase I respectively;
Independent assortment and random segregation of non-identical sister chromatids due to
crossing over during metaphase II and anaphase II respectively;
2 max
280
26

Question 3
(a) glucose as the level of glucose is decreasing;

(b)
In Part II, there is no increase in the cell count of the bacteria as glucose has been used up
and time is needed for the expression/activation of the lac operon to produce -
galactosidase before lactose can be used/metabolized;
The presence of lactose means that the small amount of -galactosidase present in the
bacterial cells will catalyse conversion of some lactose molecules to allolactose;
Allolactose binds to the lac repressor, causing a change in conformation, inactivating it such
that it no longer binds to the lac operator. Allowing RNA polymerase access to the promoter;
The absence of glucose means that cAMP levels in the cells are high;
cAMP binds to CAP, causing a change in conformation of CAP such that it binds to the CAP
binding site, increasing RNA polymerases affinity for the promoter, increasing the rate of
transcription of -galactosidase;
many copies of -galactosidase, allowing bacteria to efficiently use lactose as a respiratory
substrate to support growth, thus the increase in the bacterial cell count seen in part III. This
continues until lactose is used up and cell count levels off;
4 max

Question 4
(a)(i)
alternative RNA splicing of the pre-mRNA;
different spliceosomes are present in different cell types which recognize different base
sequences are introns and exons;
from 1 gene 1 pre-mRNA different mRNAs produced different proteins;

(ii)
1 gene can code for different versions of a protein, this increases the coding capacity of the
genome without needing to increase the amount of DNA within the cell;

(b)(i)
DNA polymerase can only add nucleotides to the 3 hydroxyl end of a pre-existing daughter
strand / cannot add nucleotides to the bare template;
In lagging strand synthesis, once that primer is removed, it cannot be replaced with DNA as
there is no 3 end onto which DNA polymerase can add deoxyribonucleotides;
Therefore the 5 ends of daughter DNA strands of the lagging strand cannot be completed /
repeated rounds of replication thus produce shorter and shorter DNA molecules at the 5
end;

(ii)
to prevent exonucleases from degrading the ends of the linear DNA molecules;
to prevent fusions of the ends of chromosome with other DNA molecules;
1 max

(iii)
Telomere length plays a role in regulating replicative cell senescence as the length of the
telomeres limits the number of times each cell can divide / ref to apoptosis;
Telomerase allows the generation of telomeres lost at each cell division. This allows cancer
cells to divide continuously and indefinitely;
281
27

Question 5
(a)
In a heterozygous individual, the phenotype of the dominant allele is always expressed and
masks the effect of the recessive allele;

(b)
Let A represent the dominant allele for curly wings
a represent the recessive allele for normal wings
XR represent the dominant allele for red eyes
Xr represent the recessive allele for white eyes

Parental White-eyed female Red-eyed male


X [1]
phenotype: with curly wings with curly wings
Parental XrXrAa XRYAa
X
genotype:
Gametes: r
XA r
Xa XRA XRa YA Ya
X

Punnets square:

XRA XRa YA Ya

[1]
XrA XRXrAA XRXrAa XrYAA XrYAa

Xra XRXrAa XRXraa XrYAa XrYaa

Offspring XRXrAA XRXraa XrYAA XrYaa


genotype: 2 XRXrAa 2 XrYAa
Offspring Red eyed Red eyed White eyed White eyed
phenotype: [1]
female with female with male with male with
curly wing normal wing curly wing normal wing

Offspring 3: 1: 3: 1 [1]
phenotypic ratio:

(c)(i)
Classes Observed Expected Expected (O E)2 (0 E)2
number (O) ratio number (E) E
Red eyed female 83 3 90 49 0.544
with curly wing
Red eyed female 41 1 30 121 4.033
with normal wing
White eyed male 78 3 90 144 1.6
with curly wing
White eyed male 38 1 30 64 2.133
with normal wing
Total 240 8 240 0 0
8.31
(Calculations / Workings [1] Correct Answer to 2d.p. [1])
282
28

(ii)
0.02 <p < 0.05;

(iii)
Critical 2 value (7.82) < calculated 2 value (8.31), therefore there is a significant difference
between observed and expected values;

cross does not follow Mendalian ratios and other factors may be at work;

Question 6
(a)
As average stomata size increases from 0 to 10 units, rate of photosynthesis increases
drastically by 550% / from 20 to 130 a.u.;

An increase in stomata size enables more atmospheric CO2 to enter, which is fixed by
RuBisCO;

Therefore, overall rate of photosynthesis increases as more CO2 is fixed as compared to


amount of O2 fixed;

(b)
Competitive;

(c)
Dry and hot;
Stomata closes and thus result in lower CO2 concentration entering leaves;
Since PEP carboylase has a higher affinity for carbon dioxide than RuBisCO/ PEP
carboylase not inhibited by oxygen;
Thus C4 plants are able to fix the carbon dioxide more efficiently;

(d)
Calvin cycle is the light-independent stage of photosynthesis;
Only through Calvin cycle can carbon dioxide be fixed to form glucose/ carbohydrate;
283
29

Question 7
(a)
permeability to sodium ions increases drastically from 0 to 28 a.u;
initial rise in permeability to sodium ions occurs when a stimulus opens some voltage-gated
sodium channels;
Influx of sodium ions through those channels depolarises the membrane, making the inside
of the neurone positive with respect to the outside / threshold;
As the influx of sodium ions continues, the membrane becomes further depolarised in a
positive-feedback loop, opening more voltage-gated sodium channels;
This allows even more sodium ions to diffuse into the neurone, explaining the sharp rise in
permeability to sodium ions;
4 max

(b)
The fall in membrane potential indicates repolarisation of the membrane as efflux of
potassium ions occurs without further influx of sodium ions;

Permeability to sodium ions decreases from 28 to 0 a.u. as most voltage-gated sodium


channels are inactivated / Closure of voltage gated sodium channels by the time the
membrane potential reaches +40 mV;

Permeability to potassium ions increases up to 12 arbitrary units as most voltage-gated


potassium channels are open by the time the membrane potential reaches +40 mV;

(c)(i)
Depolarised membrane would not be able to repolarise / return to the resting potential;
This is because efflux of potassium ions cannot occur when voltage-gated potassium
channels are blocked by TEA;
OR
Takes a long time for resting potential to return back to resting potential;
hence the next action potential will take a longer time to be generated / no action potential
can be generated;

(ii)
The presynaptic neurone would continue to release excitatory neurotransmitters until the
neurotransmitters in the presynaptic neurone are depleted;

This is because calcium ions would continue to enter the presynaptic neurone via the
voltage-gated calcium channels and trigger the release of excitatory neurotransmitters so
long as the presynaptic membrane remains depolarized;

OR

TEA prevents next action potential from being generated since neurone is perpetually in its
depolarised state / repolarisation cannot occur when efflux of potassium ions across the
voltage gated channels cannot take place.

hence the neurone is not able to release neurotransmitter across the synapse upon
stimulation. / action potential cannot be generated / decrease the occurrence of action
potential;

(d)
Sodium-potassium ion pump by active transport with the hydrolysis of ATP;
2 potassium ions in and 3 sodium ions out of the neurone;
284
30

Question 8

(a) (i)
Signal molecule binds to GPCR via ligand-receptor interaction which in turn activates G-
protein. G-protein is activated when it is attached to GTP nucleotide;

Upon activation, activated G-protein dissociates from the receptor, diffuses along the
membrane and then binds to and activates phospholipase C-;

PI 4,5 bisphosphate is cleaved into diacylglycerol and IP3 / Activation of phospholipase C-


catalyzes the formation of IP3;

IP3 serves as the second messenger and binds to ligand-gated Ca2+ channels in the ER
membrane, causing them to open;

Ca2+ ions flow down concentration gradient from ER lumen to cytosol and binds to protein
kinase C, thus activating the enzyme;
4 max

(ii)
Signal molecule is water soluble / lipid-insoluble, thus it cannot pass through the
hydrophobic lipid bilayer/ fatty acid tails of the plasma membrane;

(iii)
Dephosphorylation of IP3 by phosphatase which removes phosphate groups from proteins;
IP3 will no longer be able to bind to ligand-gated Ca2+ channels, hence leading to the inability
of Ca2+ ions to diffuse into the cytosol;
Presence of calcium ions that enter the cytosol is rapidly pumped out, mainly to the exterior
of the cell disruption to the ionic gradient of Ca2+ / answers pertaining to the disruption in
the Ca2+ concentration;
Breakdown of signal molecule such that it is unable to bind to GPCR, forming ligand-
receptor interaction. Hence, there will be no activation of G-protein and phospholipase C;
2 max

(b)
Data in Fig. 8.2 do not support the hypothesis that high concentration of EGTA can inhibit
the binding of IP3 to its receptor;
When total EGTA concentration was increased from 4.5 mM to 10.0 mM to 14.3 mM, the
calcium release rate remained relatively constant at approximately 0.020 sec-1;
285
31

Question 9
(a) (i)
variations in the Tropheus fish population due to mutation resulting in different alleles;
at least 6 different niches in the lake with different selection pressure;
fish with at selective advantage survive and reproduce viable offspring;
passing on advantageous genes/alleles to the next generation;
accumulation of many genetic changes over a long period of time to evolve into 6 species;
reproductive/ behavioural/ sympatric/ allopatric/ geographical isolation/ accept hundreds of
km apart thus no gene flow between different populations;
4 max

(b)
evolved from a common ancestor;
cytochrome b gene code for important protein involved in respiration thus any mutation will
result in non-functional protein which will be fatal to the organism;
this gene would have gone through the least mutation not counting silent mutation;
3 max

(c)
compare and note the number of differences in the DNA sequences between species;
determine the age of fossil record of each species by dating;
calculate the rate of mutation using calibrated molecular clock;
no. of mutation/ no. of difference in DNA sequence is used to calculate the length of time
since divergence;
3 max
286
32

Essay answers

1(a)
Similarities:
1. Function: Both processes are required for gene expression / protein synthesis;
2. Process: Both involve synthesis of polymers from monomers by condensation
reaction with the removal of 1 water molecule;
3. Process: both involve synthesis of polymer based on a reference molecule /
template;
4. Process: initiation of both processes requires the aid of factors in formation of
initiation complex;
5. Both require post-processing of products post-transcriptional modification for
pre-mRNA to become mature mRNA and post-translational modification of
polypeptide chain before formation of mature protein;

Differences:

Features Transcription Translation


6. Location Nucleus Free ribosomes in cytoplasm and
ribosomes bonds to the RER
7. Template DNA template strand of gene Mature mRNA
8. Direction of Template is read in 3 to 5 mRNA is read in 5 to 3 direction
reading of direction
template
9. Method of Template is read in individual Template is read in codons, with
reading of nucleotides, with incoming free incoming amino-acyl tRNA
template ribonucleotide added by anticodon recognizing the codon
complementary base pairing on mRNA by complementary base
pairing
10. Type of Free ribonucleotides Amino acids
monomer

11. Monomer no Yes


requires Requires activation / attachment to
activation tRNA
12. Bond Phosphodiester bonds between Peptide bonds between amino
between ribonucleotides acids
basic units

13. Enzyme RNA polymerase II Peptidyl transferase in large


(catalyst for ribosomal subunit
bond
formation)
14. Products mRNA, rRNA and tRNA Polypeptide chain
287
33

1 (b) (i)
1. Molecular methods used in classification of organisms involve using molecular homology
such as similarities in DNA or amino acid sequences;

2. molecular methods are objective and quantifiable ways to classify organisms base on
their phylogenetic relationships;

3. By comparing the DNA or amino acid sequence of different organisms , we can


determinate the amount of similarity in the sequences of each birds;

4. the higher the percentage of similarities in DNA or amino acid sequence, the more
closely related the organisms are; Accept: vice versa

5. If the species have evolved from the same common ancestors, their DNA or amino acid
sequence should show a high degree of homology with each others and also with that of
the original species;

6. based on the idea of molecular clock, different species can be arranged in chronological
order of time of evolution, thus creating a phylogeny and extrapolation in phylogenetic
studies;

7. Molecular methods can pick up in silent mutation thus the mutation will be reflected in
the DNA sequence;

8. New species are evolved when changes in DNA sequence are accumulated over time;
6 max

(b) (ii)

Variation in bacteria population due to random gene mutation;


To form new genes;
Lack proof reading mechanism during DNA replication;
Novel genes can also be introduced into bacteria via transformation, transduction and
conjugation;
New alleles can be incorporated into the bacteria genome via homologous recombination;
4 max
288
34

2(a)

Genetic code

Prokaryotes Eukaryotes

1. Gene Does not occur Occurs to increase the number of


amplification copies of the gene of interest, which
hence increases the overall rate of
transcription and the amount of gene
product.

Transcriptional control

Prokaryotes Eukaryotes

2. Access to Does not occur DNA methylation and histone


genes by acetylation / deacetylation can occur,
modification resulting in conversion between
of chromatin
euchromatin (transcribed) and
structure
heterochromatin (not transcribed).

3. Control Few More


elements Close to promoter / genes Can be located proximally upstream,
under their control or distally upstream / downstream of
the gene
E.g. operators located E.g. enhancer, silencer, proximal
near or within the promoter control elements, distal control
elements
4. General RNA polymerase core General transcription factors required
transcription enzyme can recognize and for RNA polymerase to bind to
factors bind to the promoter with the promoter and assembly of
aid of the sigma factor
transcription initiation complex

5. Specific Repressors bind to operator binding of activators to enhancers to


transcription preventing transcription up-regulate transcription
factors Activators bind to activator repressors binding to silencers /
binding sites to increase rate competing with for activator binding
of transcription sites to down -regulate transcription
6. RNA All RNAs synthesized by the Three different classes of RNA each
polymerase same RNA polymerase; synthesized by a different RNA
polymerase

7. Coordinated Genes or same metabolic No operons, each gene has own


transcription pathway organized into promoter resulting in monocistronic
of genes operons controlled by single mRNA,
promoter, resulting in Coordination of gene expression
transcription of single occurs by a combination of control
polycistronic mRNA element and specific TFs
289
35

Post-transcriptional

Prokaryotes Eukaryotes

8. Post- Does not occur Addition of 5 7-methylguanosine cap,


transcriptional 3 poly-A tail, splicing / alternative
modification splicing, editing of mature mRNA

Translational

Prokaryotes Eukaryotes

9. Simultaneous Translation is often begins No direct coupling of transcription and


transcription & before transcription is translation. (mRNA must pass across
translation complete as processes are nuclear envelope before translation in
not separated by nuclear the cytoplasm. RNA transcript is not
envelope free to associate with ribosomes prior
to the completion of transcription).

10. Half-life of Lower stability of transcript Higher stability of transcript,


mRNA / degradation within degradation after minutes / days
seconds or minutes
/ mRNAs shorter half life to controlled by length of poly-A tail
rapidly respond to (longer tail, slower degradation)
environmental changes
11. Binding of Production of anti-sense Production of small RNAs (miRNAs
complementary RNA from non-coding and siRNAs) which are cut into
sequence to DNA strand can bind to smaller pieces by Dicer and picked
RNA complementary mRNA up by RISC which will degrade any
and inhibit translation mature mRNA that has sequences
complementary to the small RNA.
12. Ribosome mRNAs have multiple start mRNAs have only one start codons
binding sites codons direct synthesis of only one kind of
direct the synthesis of polypeptide
several different
polypeptides

Post-translational

Prokaryotes Eukaryotes

13. Post- no/minimal post- Post-translational modifications


translational translational modifications usually required to get functional
modification occur protein

e.g. proteolysis, chemical


modification, ubiquitination for
degradation,
290
36

2 (b)(i)
1. Acetyl CoA and oxaloacetate undergo a condensation reaction. Acetyl CoA adds its two-
carbon acetyl group to oxaloacetate to form citrate. Enzyme = citrate synthetase;

2. Citrate is isomerised to isocitrate. Enzyme = aconitase;

3. Isocitrate undergoes oxidative decarboxylation to form -ketoglutarate. Carbon dioxide


is lost in this process. NAD+ is the hydrogen acceptor and NADH is formed. Enzyme =
isocitrate dehydrogenase

4. -ketoglutarate undergoes oxidative decarboxylation to form succinyl CoA. Carbon


dioxide is lost in this process. NAD+ is the hydrogen acceptor and NADH is formed.
Enzyme = -ketoglutarate dehydrogenase

5. Succinyl CoA is converted to succinate. ATP is formed by substrate level


phosphorylation. Enzyme = succinyl CoA synthetase

6. Succinate undergoes dehydrogenation to form fumarate. FAD is the hydrogen acceptor


and FADH2 is formed. Enzyme = succinate dehydrogenase

7. Fumarate undergoes hydration to form malate. Enzyme = fumarase

8. Malate undergoes dehydrogenation to form oxaloacetate. NAD+ is the hydrogen


acceptor and NADH is formed. Enzyme = malate dehydrogenase

2(b) (ii)
Similarities
Both processes synthesizes ATP by chemiosmosis, where energy released by the transport
of electrons down electron transport chain is used to pump H+ against a concentration
gradient. And as the H+ diffuses back down a concentration gradient, the energy released is
used to phosphorylate ADP to ATP;
Both processes involves the transport of electron down ETC.
2 max

Differences
Features Oxidative phosphorylation Photophosphorylation
Definition ;
Synthesis of ATP occurs as Synthesis of ATP occurs as
electrons are transferred from electrons are transferred from
NADH or FADH2 to oxygen by an water and chlorophyll
electron transport chain. The molecules to NADP by an
energy coupling occurs through a electron transport chain. The
proton gradient. energy coupling occurs
through a proton gradient.

Location On the inner membrane of on the thylakoid membrane ;


mitochondrion.

Reactions Involves a series of redox reactions hydrolysis of water molecule ;


via a series of electron carriers in on top of redox reaction in the
the electron transport chain. electron carriers.
Synthesis of by both cyclic and noncyclic ;
by Oxidative phosphorylation
ATP Photophosphorylation
3 max
291
1

Name: Index Number: Class:

DUNMAN HIGH SCHOOL


Preliminary Examination
Year 6

H2 BIOLOGY 9648/03
Paper 3 Applications Paper and SPA Planning Task 23 September 2013
2 hours
Additional Materials: Writing paper

INSTRUCTIONS TO CANDIDATES:
DO NOT TURN THIS PAGE OVER UNTIL YOU ARE TOLD TO DO SO.
READ THESE NOTES CAREFULLY.

Section A:
Consists of 4 Structured Questions
Answer all questions.
Write your answers in the space provided on the
question paper.

Section B:
Consists of 1 SPA Planning Task
Write your answers on the separate writing papers
provided. At the end of the examination, fasten all
your work securely together.
For Examiners Use
Section C: Section A [40]
Consists of 1 Free-Response Question.
Write your answers on the separate writing papers 1 / 12
provided. At the end of the examination, fasten all
your work securely together. 2 / 8

Sections A, B and C are to be submitted 3 / 13


separately. 4 / 7

INFORMATION FOR CANDIDATES Section B [12]

Essential working must be shown. Section C [20]

The intended marks for questions or parts of Total [72]


questions are given in brackets [ ].

This document consists of 13 printed pages and 1 blank page.


[Turn over
DHS 2012
292
2

Section A: Structured Questions (40 marks) For


Answer all questions in this section. Examiners
use

Question 1

Fig 1.1 shows two plasmids pAMP and pKAN. Ampr and Kanr confer resistance to the
antibiotics ampicillin and kanamycin respectively.

Bam HI Bam HI

Kanr
Nco I
784 bp
pAMP pKAN 1875 bp
3755 bp
4539 bp 4207 bp
2332 bp
Hind III

ori
Ampr
Hind III
Sca I
ori

Fig 1.1

In an experiment to make a cloning vector, solution containing the plasmids pAMP and
pKAN were treated with a solution containing restriction enzymes BamHI and HindIII. DNA
ligase was then added and the mixture was incubated at 37oC.

This experiment produced the plasmid pAK which can be used as a cloning vector. The
restriction map of pAK is shown in Fig 1.2 and the specific recognition sequence of its
restriction sites are shown in Table 1.3.
Bam HI
Restriction Recognition Sequence
Enzyme
Kanr Bam HI 5'-G^G A T C C-3'
3'-C C T A G^G-5'
Nco I
Hind III 5'-A^A G C T T-3'
3'-T T C G A^A -5'
1875 bp
pAK
Sca I 5'-A G T^A C T-3'
5630 bp 3'-T C A^T G A-5'
3755 bp
Nco I 5'-C^C A T G G-3'
3'-G G T A C^C-5'
Ampr
Hind III
^ indicates where the restriction enzyme cuts
Sca I ori
Fig 1.2 Table 1.3
293
3

(a) Restriction sites are palindromic. Explain the meaning of the term palindromic. [1]

(b) With reference to Fig 1.2 and Table 1.3, explain why Nco I is a more suitable choice of
restriction enzyme for use in genetic engineering with the pAK plasmid than Sca I. [2]

To clone a human gene with a molecular size of 600 bp in bacterial cells, the following
proceedure was carried out as shown in Fig 1.4.

Isolate mRNA

Procedure X: Synthesis of double stranded cDNA

Addition of linkers Cloning vector pAK

Cut with restriction enzyme Nco I

Ligation of cDNA and pAK

Transformation of recombinant
plasmids into E.coli

Culture E.coli on agar plates

Fig 1.4
294
4

(c) With reference to Fig 1.4, briefly describe procedure X. [3]

(d) Draw three possible gene products formed in the ligation mixture. [3]










295
5

(e) Explain how the plasmid pAK can aid in the successful selection of recombinant
colonies with the gene of interest. [3]

Total: [12]
296
6

on 2
Questio Foor
Exxaminers
usse
A lengthh of DNA frrom one of a pair of ho
omologous chromosom mes is show
wn in Fig 2.1. The
target sites of EcoR
RI are show
wn by arrowws and the le NA betweenn the target sites is
ength of DN
given in kilobases (kb).
(

Fig 2.1

A mutattion alters one


o base off the codingg sequencee at the site
e marked w with an asterrisk (*).
DNA froom two individuals, onne who is n
normal and the other whow is heteerozygous for this
mutation
n is cut with
h EcoRI and
d the DNA ffragments separated
s byy gel electroophoresis.

(a) On
n Fig. 2.2 below,
b
(i)) Draw all
a the fraggments of this length h of DNA produced from the pair of
homolo ogous chrom
mosomes in n each of the individualls tested annd label the size of
the frag
gments prod
duced. [2]

(ii) Circle the


t fragmen nts which w
will combine
e with the ra
adioactive pprobe and appear
as blacck bands on an autorad
diography. [1]

Fig 2.2
297
7

(b) Describe how DNA fragments may be separated by electrophoresis. [4]

(c) State the role of the DNA ladder seen in Fig 2.2. [1]

Total: [8]
298
8

Question 3 For
Examiners
use
(a) Cystic fibrosis (CF) is a genetic disease which affects many parts of the body. This
disease is caused by mutations of a gene coding for a transmembrane protein
(CFTR) which acts as an ion pore.

A study on ion transport was carried out on pancreatic cells by comparing ion
transport in cells expressing the normal allele for CFTR with that in cells expressing
mutant alleles. The pancreas secretes pancreatic juice which is alkaline to neutralize
the acidic gastric acid from the stomach.

The abilities of the cells to transport HCO3 and Cl were measured and expressed as
a HCO3 : Cl transport ratio. The results are shown in Table 3.1.

CF allele expressed by cells HCO3 : Cl transport ratio of cells

Normal alleles 1.0 : 1.0

Mutant alleles associated with CF with


All < 0.1 : 1.0
inadequate functioning of the pancreas

Table 3.1

(i) With reference to Table 3.1, explain the cause of inadequate functioning of the
pancreas in CF. [2]

(ii) Suggest one possible consequence of inadequate functioning of the pancreas


as a result of CF. [1]

(b) Gene therapy involves transferring normal CFTR genes into lung epithelial cells with
defective genes. Describe how a liposome can be used to insert the normal CFTR
gene into a CF patient. [2]
299
9

(c) The mucus produced in the lungs of someone with cystic fibrosis contains a lot of
DNA from dead cells. DNase, an enzyme which cuts DNA into short pieces has
recently been approved for the treatment of cystic fibrosis. In an investigation, different
concentrations of DNase were added to mucus collected from people with cystic
fibrosis. Fig. 3.1 shows the results.

Fig. 3.1

Using the information given in Fig. 3.1, explain why inhaling DNase helps a cystic
fibrosis patient to breathe more easily. [3]

(d) Describe how the isolated CFTR gene can be obtained from the genome using the
polymerase chain reaction. [3]

(e) About twenty percent of the DNA molecules produced by the PCR are copied
inaccurately. Explain why it is not safe to use the PCR to clone the normal CFTR gene
for use in treating cystic fibrosis. [2]

Total: [13]

300
10

Question 4 For
Examiners
use
EH1 is a hybrid derived from a cross between Eucalyptus grandis (2n = 22) and Eucalyptus
urophylla (2n = 22). Embryo culture is used to develop EH1 embryos into plantlets. At three
different stages of the embryonic development as shown in Fig. 4.1, the nutrients and plant
growth hormones concentrations in the medium are changed to allow development into
plantlets.

Fig. 4.1

(a) (i) Explain why the EH1 plantlets grown from different hybrid embryos are not
genetically identical. [2]

(ii) With reference to Fig. 4.1, explain why the concentrations of the plant growth
hormones have to be applied and changed to ensure development of plantlets.
[3]
301
11

The average growth of wood was recorded for EH1 grown in different parts of the
United States. Non-GM EH1 was grown in plot A, while GM EH1 was grown in plots B,
C, and D. The results are shown in Fig. 4.2.

Fig. 4.2

(b) Explain how the data shows that lignin gene expression level is difficult to control in
GM crops despite being placed under the control of an antifreeze promoter. [2]

Total: [7]
302
12

Section B: SPA Planning Task (12 marks)


Write your answers on the writing paper provided.
A NIL RETURN is required.

Question 5

Digestion of lipids usually occurs in the small intestines by the action of pancreatic lipase.
Unfortunately, dietary fats do not dissolve in water, and bile salts secreted by the liver are
required to emulsify fats. This provides a larger surface area for lipase to act on.

Phenolphthalein is an acid-base indicator which is colourless in acidic solutions and pink in


basic solutions such as sodium carbonate solution. A 1:1 ratio of sodium carbonate solution
to full cream milk will provide a slightly alkaline pH which is optimum for lipase action.

Using this information and your own knowledge, design an experiment to investigate the
effect of bile salt concentration on the rate of digestion of lipids.

Your planning must be based on the assumption that you have been provided with the
following equipment and materials.

Phenolphthalein
5% lipase solution
5% Bile salts solution
Sodium carbonate solution
Distilled water
Full cream milk
Stopwatch
A variety of different sized beakers, measuring cylinders and syringes for measuring
volumes
Access to standard laboratory equipment

Your plan should have a clear and helpful structure to include:

An explanation of theory to support your practical procedure


A description of the method used, including the scientific reasoning behind the method
An explanation of dependent and independent variables involved
How you will record your results and ensure they are accurate and reliable as possible
Proposed layout of results tables and graphs with clear headings and labels
The correct use of technical and scientific terms

Total: [12]

303
13

Section C: Free-Response Questions (20 marks)


Write your answers on the writing paper provided.
Your answers should be illustrated by large, clearly labelled diagrams, where appropriate.
Your answers must be in continuous prose, where appropriate.
Your answers must be set out in sections (a), (b) etc., as indicated in the question.
A NIL RETURN is required.

Question 6

(a) Describe the treatment of SCID using gene therapy. [10]

(b) Discuss the social and ethical considerations for gene therapy. [10]

Total: [20]

END OF PAPER
304
14

BLANK PAGE
305
15

DUNMMAN HIGH SCHOOL


S
PRELIMINA
P ARY EXAMINATION 2013
YEAR SIX
X
H2 BIOLOGY (9648)
PAPER 3
Answer Scheme

Structu
ured Answe ers
1(a)
nucleotide sequences that are e the same on both strands of the DNA moleccule when read r in
the sam
me direction (5 to 3) / read
r the sa me in the 5
5 to 3 direction on eacch strand of DNA.

(b)
Nco I prroduces sta
aggered / stticky ends w
whereas Sca I produce
es blunt endds;
This faccilitates the
e annealing of the gene
e of interes
st to plasmid
d when thee plasmid an
nd gene
are cut with Nco I byb complemmentary basse pairing;
Using S
Sca I would require the e additional procedure of
o adding lin
nker DNA too generate sticky
o facilitate annealing of gene of intterest.
ends to

(c)
Oligo-dT primers anneal
a to 3 poly-A tail o
of mRNA
Reversee transcripttase extends primers u
using mRNA
A as templatte to syntheesize single
strande
ed cDNA, prroduct is RNNA-DNA du plex.
Partial d
digestion off mRNA using alkaline solution / RNase
R H
ref to syynthesis of 2nd comple
ementary sttrand to form
m double stranded cDN
NA using DNA
polymerase
3 max.

(d)

1 mark each

(e)
AmpR g gene codess for ampic cillin resista
ance thus allows
a for first
f selectioon for succ
cessfully
transforrmed coloniies which ha
ave taken in n the plasm
mid;
Coloniees with rean
nnealed pla ct Kanr gene
asmids will have intac e which coodes for kan
namycin
nce and will survive on kanamycin
resistan n plate;
Coloniees with reco ombinant pla asmids will gene of inte erest inserted into Kannr gene wich
h results
in inserrtional inactiivation, and
d will not surrvive on kan
namycin plaate;
306
16

2(a)

Figure2.2

(i)
Normal individual 3 correct bands, labeeled and of the correct thickness;
Heterozzygous indivvidual 4 correct
c band
ds, labeled and
a of the correct
c thickkness;

(ii)
circling of 10kb and
d 5kb bands for norma
al individual & circling of
o 20kb, 10kkb and 5 kb bands
for heteerozygous in
ndividual;

(b)

A loadinng dye is addded to the DNA samp ples to help DNA to sink into the w wells / allowss the
processs of electropphoresis to be tracked;;
DNA lad dder and saamples are loaded intoo wells;
Gel is ssubmerged in a buffer solution
s tha
at will condu
uct electricity
y;
electric current is applied
a thou
ugh electroddes at oppoosite ends of the gel;
negatively-charged d DNA fragmments migra ate to the positively charged electtrode / anod de;
Gel elecctrophoresis separates s DNA fragm ments by sizze / molecu ular weightss, with the smallest
fragmen nts moving the fastest and furthesst;
4 max.

(c)

As a refference / sttandard, prooviding know


wn DNA fra
agments of specific
s lenggths to estim
mate
the size
e of DNA mo olecules in samples
307
17

3 (a) (i)
Transport ratio is all < 0.1 : 1.0;
Inability to transport HCO3 out of pancreatic cell;
increase in acidity / decrease in pH outside the cells in the intestine;
2 max

(ii)
Poor digestion of proteins/ lipid/starch in the intestines thus poor absorption of nutrients;
Decreases secretion of insulin/glucagon thus affect blood glucose level;
ref. damage to wall of intestine/ pain in the intestine;
1 max

(b)
Normal CFTR gene is inserted into liposomes which are inhaled/sprayed into nose and
mouth;
Liposomes fuse with the cell membrane and release normal CFTR gene into (cytoplasm of
lung epithelial ) cells;

(c)
A higher conc. of DNase will result in a greater number of shorter DNA pieces;
With shorter DNA pieces in the lungs, viscosity of the mucus is reduced;
more air can pass through the airways/airways are no longer blocked/mucus more easily
removed from airways;

(d)
Heat dsDNA to 95C for 1-2min to break the hydrogen bonds (between complementary
bases) / separate DNA strands;
Lower temperature to 55 C for 1-2min to allow annealing of forward and reverse primers to
ssDNA / to regions flanking the CFTR gene;
Increase temperature to 70C for 1-2min to allow Taq polymerase catalyses formation of
new DNA strand (using the template);

(e)
percentage risk is too high for human application/ ref to a wrong/mutant gene introduced into
CF patient;
incorrect mRNA formed changes sequence of amino acids in protein;
produce a toxic/harmful/non-functional/ mutated protein;
chloride ions not transported / thick mucus results;
2 max

4 (a) (i)
Crossing over and independent assortment and random segregation of chromosomes during
meioisis;
Formed gametes which showed variation / allelic recombination;

(ii)
Need to apply auxin and cytokinin to stimulate mitosis of undifferentiated embryonic cells;
Need to change to low auxin to cytokinin ratio to stimulate formation of shoot meristem;
Need to change to high auxin to cytokine ratio to stimulate formation of root meristem;

(b)
B, C and D experienced average winter temperature of 3 C, -1 C and 2 C respectively,
yet average growth are 5, 8 and 15 kg/ tree respectively;
There is no correlation between temperature and average growth;
308
18

Planning Answer

1. theory and expected trend


2. theory to support method to measure rate
3. independent variable
4.dependent variable
5. AVP temperature with method to control
6. dilution
7. appropriate volumes suggested
8. Measuring time taken for indicator to decolourise
9. 3 replicates, 2 repeats
10. method to calculate rate 1/t
11. appropriate table
12. appropriate graph
13. correct use of technical and scientific terms

Essay Answers

6 (a) Describe the treatment of SCID using gene therapy.

1. remove T-lymphocytes from patient with SCID and insert healthy genes in them using a
retrovirus, then transplanting them back into the patient;
2. gene therapy can be used to treat both ADA-SCID and sex-link-SCID;
3. An ex vivo gene therapy approach was used;
4. where lymphocytes obtained from the blood of the patient/bone marrow stem cell;
5. was stimulated to proliferate in culture;
6. A functional adenosine deaminase (ADA) cDNA/ IL2-RG cDNA in a retroviral vector was
then introduced into these cells;
7. The retrovirus had been inactivated to become non-pathogenic;
8. The transgenic lymphocytes were allowed to proliferate further;
9. retroviruses naturally integrate their genome into the genome of host cells;
10. thus the retrovirus was integrating the normal ADA gene / IL2-RG gene into the
chromosomes of the patients cells during this culturing;
11. After a short period of culturing, these ADA-containing lymphocytes were re-introduced
into the patient;
309
19

6 (b) Discuss the social and ethical considerations for gene therapy.

Arguments in favour of gene therapy:


1. Potential for treating ill patients or preventing onset of severe illness
2. corrects the genetic defect present in the individual alone
3. like any other new medical technology.
4. To prevent research on gene therapy is to infringe upon intellectual freedom of
researchers.
5. With current rate of progress in gene therapy techniques risks become more identifiable
and outcomes can be predicted rather precisely.
6. offers true cure, not just a relief of symptoms or a palliative treatment.
7. The expense of somatic-cell therapy for multiple generations is avoided.
8. Medicine should respond to the reproductive health needs of prospective parents at risk
for transmitting serious genetic diseases.
9. The scientific community has a right to free inquiry within the bounds of acceptable
human research.
10. Once successful techniques are developed, gene therapy could help parents and
researchers avoid the moral dilemma of disposing defective embryos in the laboratory

Arguments against gene therapy:


11. It is difficult to distinguish between "good" and "bad" uses of gene modification
techniques.
12. It is important to consider between genetic disease and genetic attribute. Consideration
should be given beyond the therapeutic if there is only a genetic tendency towards a
particular disease.
13. There is potential for harmful abuse of the technology.
14. There are many unresolved questions raised by gene therapy:
15. There is difficulty in following patients in long-term clinical research, including
surveillance of future generations to monitor the long-term effects of gene therapy.
16. Many gene therapy candidates are children who are too young to understand
ramifications of gene therapy
17. There are potential conflicts of interest of individuals who wish to protect their privacy
and confidentiality versus interests of insurance companies or society in not bearing the
financial burdens of caring for children with serious genetic defects.
18. There are issues of justice and allocation: can we afford the expense of gene therapy?
Who should receive gene therapy? If only those who can afford it, where is the principle
of justice? The procedures now being tested are expensive and require expertise and
equipment found only in major medical centers. A related question is, Should gene
therapy be reserved for treating serious diseases? Who should pay for their use?
Should we skew the distribution of desirable biological traits?
19. Long-term effects of germ-line therapy are not known.
20. It opens the door to attempts at altering human traits not associated with disease (i.e.,
eugenics), which in turn leads to issues of increased social inequality and a lowered
tolerance for genetic diversity.
21. Violates rights of future generations to inherit a genetic endowment which has not been
intentionally modified.
22. Too expensive.
23. Essentially mates generations of unconsenting research subjects (i.e., germ-line therapy
involves research on early embryos and affects their offspring).
310

Civics Group Index Number Name (use BLOCK LETTERS)


H2

ST. ANDREWS JUNIOR COLLEGE


2013 JC2 Preliminary Examinations

H2 BIOLOGY 9648/1

Paper 1: Multiple Choice


Friday 20 September 2013 1 hour 15 minutes

Additional Materials: Multiple Choice Answer Sheet


Soft clean eraser (not supplied)
Soft pencil (type B or HB is recommended)

READ THESE INSTRUCTIONS FIRST

Do not open this booklet until you are told to do so.

Write your name, civics group and index number on the multiple choice answer sheet in
the spaces provided.

There are 40 questions in this paper. Answer all questions. For each question, there are
four possible answers, A, B, C and D.

Choose the one you consider correct and record your choice in soft pencil on the separate
multiple choice answer sheet.

INFORMATION TO CANDIDATES

Each correct answer will score one mark. A mark will not be deducted for wrong answer.
Any rough working should be done in this booklet.

At the end of the examination, submit the multiple choice answer sheet only.

This document consists of 15 printed pages.


[Turn over
311
2

1 Th
he nuclei off both plan
nt and anim
mal cells co
ontain one or more deense bodie es known
as nucleoli. Which
W one of the follo
owing corrrectly describes the fuunction of nucleoli?

A The forrmation of new DNA molecules s.


B The orgganization of the spin
ndle during
g nuclear division.
d
C Synthe
esis of chro
omatin fibrees.
D The forrmation of ribosomess.

2 Whhat could be
b a possib
ble explana
ation for th
he ability off lysosomees to withsttand self-
dig
gestion?

A lysosommes contain inactive hydrolytic enzymes


B lysosomme membrane has numerous modified proteins w with carbo
ohydrate
side-ch
hains
C lysosommes do notn contain n lipases which are e the enzyymes cappable of
digestin
ng the lipid
d membran ne
D hydrola
ases in the t lysos omes aree inhibited
d by thee acidic internal
environ
nment of th he lysosommes

3 Th
he diagram
m below sho ows the ulttrastructure aryotic cell . Which orrganelle
e of a euka
do
oes not con
ntain nucleic acid?

4 ome organ
So nisms cha ange the compositiion of the
eir membbranes to maintain
me
embrane fluidity when tempera ture chang
ges.

Whhich chang
ges occur in the com
mposition of
o the mem
mbranes folllowing a change
c to
low
w temperatture?

A Increasse phosphoolipids, deccrease choolesterol.


B Increasse saturate
ed fatty acid
ds, increas
se cholesteerol.
C Increasse saturate
ed fatty acid
ds, decreaase cholestterol.
D Increasse unsatura
ated fatty a
acids, incre
ease chole esterol.

SAJC / H22 Biology 9648/1 JC2 Prelims 2013


312
3

5 What does a haemoglobin molecule contain?

A four iron (Fe2+) ions attached to each haem group


B four oxygen molecules attached to each haem group
C four polypeptide chains each with one attached haem group
D four polypeptide chains each with four attached haem groups

6 Liver tissue produces an enzyme called catalase which breaks down


hydrogen peroxide into water and oxygen.

2H2O2 2H2O + O2

The rate of this reaction can be determined by measuring the volume of


oxygen produced in a given length of time. Students added small cubes of
fresh liver tissue to a range of hydrogen peroxide solutions and measured the
volumes of oxygen produced. Their data were used to produce the graph
showing how changing the concentration of hydrogen peroxide affected the
rate of oxygen production.

Which statements are correct?

1 At P, the rate of reaction is limited by the concentration of enzyme.


2 At Q, all of the enzyme active sites are occupied by substrate molecules.
3 At Q, the rate of reaction is limited by the concentration of the substrate.
4 At S, all of the enzyme active sites are occupied by substrate molecules.

A 1 and 4
B 2 and 3
C 2 and 4
D 1, 3 and 4

SAJC / H2 Biology 9648/1 JC2 Prelims 2013


313
4

7 Thhe graph sh
hows the effect
e of two
o different compound
ds on the rrate of reac
ction of
the
e enzyme at
a differentt substrate
e concentraations.

hich statem
Wh ment is true
e?

A Compound which results


r in ccurve R is a competitiive inhibitoor which als
so
increases the Km of the enzym me.
B Compound which results
r in ccurve R is a non-competitive inhhibitor whicch did
not affecct the Km off the enzym me.
C Compound which results
r in ccurve S is a competitive inhibitoor which alsso
decrease es the Km of
o the enzyyme.
D Compound which results
r in ccurve S is a non-competitive inhhibitor whicch
increases the Km of the enzym me.

8 Ahhypothetical diploid cell


c has fouur chromos somes and
d total amoount of DNA
A in the
cell is denote
ed by X before
b DNA
A replicatio
on.

hich of the following is correct?


Wh ?

Cell Cycle Phase Num


mber of Am
mount of DNA
chromoso
omes per cell per celll
A Interphase 8 2X
B Telophase of mitosis 8 X
C Metapphase of meiosis
m I 4 X
D Anaph hase of me
eiosis II 4 X

9 Whhen identiical twins marry id entical tw


wins, the children
c oof both co
ouple are
ge
enetically

A identiccal unless crossing oover takes place


B identiccal becausse of the efffect of sem
mi-conserv vative DNA
A replicatio
on prior
to nucclear divisio
on
C differe
ent becaus se non-disjuunction of chromatids occurs
D differe
ent becaus se of rando
om segrega ation during parental meiosis

SAJC / H22 Biology 9648/1 JC2 Prelims 2013


314
5

10 To function as the heritable genetic code, DNA molecules must have all of the
following structural features except

A the ability to form complementary base-pairs with RNA molecules.


B exist as a very stable, double helical form when being replicated
C a sequence of nucleotides that can be decoded into a sequence of amino
acids in a protein
D ability to undergo semi-conservative replication

11 A particular eukaryotic protein is 300 amino acids long. Which of the following
could be the minimum number of nucleotides in the DNA molecule containing the
gene that codes for the amino acids in this protein?

A 100
B 300
C 900
D 1800

12 Which process in bacteria always allows chromosomal and non-chromosomal


DNA to be transferred?

A Binary fission.
B Conjugation.
C Transduction.
D Transformation.

13 Which of the following statements about binary fission is not true?

1 The mode of replication is semi-conservative


2 Chromosomes are separated to opposite ends of the cell.
3 DNA replicates using the rolling circle mechanism.
4 DNA replication at the ori is bi-directional.

A 3 only
B 1 and 2
C 2 and 4
D 3 and 4

14 Which of the following statements concerning lac operon is true?

1 Transcription of lac operon takes place all the time.


2 There is one single mRNA transcribed from the lac operon.
3 There is one start and one stop codon in the mRNA of lac operon.
4 The repressor molecule binds to the operator to turn off lac operon.

A 4 only
B 1 and 3
C 2 and 4
D 2, 3 and 4

SAJC / H2 Biology 9648/1 JC2 Prelims 2013


315
6

15 How do viruses cause diseases in animals?

1 They inhibit normal synthesis of host cell DNA, RNA, or protein.


2 They degrade the host cells chromosomes.
3 They disrupt the oncogenes of the host cell causing uncontrolled cell division.
4 Their viral proteins and glycoproteins on the surface membrane of host cells
cause them to be recognized and destroyed by the bodys immune defences.

A 1 and 3
B 2 and 3
C 1, 2 and 4
D 1, 2, 3 and 4

16 The pedigree shows the inheritance of a condition known as galactosemia.

1 2 3 4

5 6 7 8 9 10 11

12 13 14 15 16

What is the probability that the first child of individuals 15 and 16 will be a
normal boy?

A 0.125
B 0.25
C 0.50
D 0.75

SAJC / H2 Biology 9648/1 JC2 Prelims 2013


316
7

17 Agouti mice have banded hairs, giving a grey colour. Black mice have unbanded
hairs. White mice have no pigment. A cross between a homozygous black mouse
and a white mouse produced offspring with agouti hair. Another cross between the
black mouse and another white mouse produced some offspring with agouti hair
and some with black hair.

What explains these observations on the phenotype of hair of mice?

A There is a single gene with two codominant alleles, black and white.
B There is a single gene with three alleles in a dominance series, black
grey white.
C There are two epistatic genes, one controlling pigment production and one
controlling banding.
D There are two linked genes, one controlling pigment production and one
controlling banding.

18 With reference to prokaryotic and eukaryotic genomes, which of the following


statement is not true?

A They both have extrachromosomal DNA.


B They both have non-coding regions.
C They both have regulatory sequences.
D They are both associated with histones.

19 Four different genes are regulated in different ways.

Gene 1 undergoes tissue-specific patterns of alternative splicing


Gene 2 is part of a group of structural genes controlled by the same regulatory
sequences
Gene 3 is in some circumstances subject to methylation
Gene 4 codes for a repressor protein which acts at an operator site close by

Which role of the table correctly identifies which genes are prokaryotic and which
are eukaryotic?

prokaryotic eukaryotic
A 1 and 2 3 and 4
B 1 and 3 2 and 4
C 2 and 3 1 and 4
D 2 and 4 1 and 3

SAJC / H2 Biology 9648/1 JC2 Prelims 2013


317
8

20 Which of the following statements about the eukaryotic control elements are
correct?

1 Attachment of the RNA polymerase II at the TATA box is achieved with


the help of a series of specific transcription factors
2 Upstream regulatory elements (UREs) increase the basal level of
transcription
3 A given gene may have multiple enhancers, each active at a different time
or in a different cell type or location in the organism
4 Repressors bind to silencer regions of DNA far upstream of promoters to
repress transcription

A 1 and 4
B 2 and 3
C 2, 3 and 4
D 1, 2, 3 and 4

21 Cells taken from a human bone cancer multiplied readily in culture. Analysis
showed that the cells were homozygous for the deletion of an allele on
chromosome 13 coding for a protein, RB.

Addition of RB to these cells reduced their rate of division.

Which individual is not able to produce a tumour suppressor?

A Both chromosomes 13 carry alleles for RB.


B Both chromosomes 13 have the alleles for RB deleted.
C One chromosome 13 carries an allele for RB, but the other chromosome 13
carries the deletion.
D One chromosome 13 carries an allele for RB, but the other chromosome 13
carries a different mutation.

22 Which of the following processes could still occur in a chloroplast in the presence
of an inhibitor that prevents H+ from passing through ATP synthase complexes?

1 Sugar synthesis
2 Photolysis of water
3 Transfer of electrons down the electron transport chain
4 Oxidation of NADPH

A 1 and 2
B 1and 4
C 2 and 3
D 3 and 4

SAJC / H2 Biology 9648/1 JC2 Prelims 2013


318
9

23 Some photosynthetic organisms containing chloroplasts that lack PS II


(photosystem II) are able to survive. The best way to detect the lack of PS II in
these organisms would be to

A test for carbon fixation in the dark.


B test for liberation of oxygen in the light.
C determine the production of starch.
D determine if they have thylakoids in the chloroplasts.

24 A mitochondria suspension obtained from liver cells is prepared for investigations


of the products of respiration. Acetyl-CoA is added to the suspension.

Which of the following correctly matched the products of glycolysis and Krebs
cycle for every oxidation of one glucose molecule in this mitochondria suspension?

Product Glycolysis Krebs cycle


ATP 2 2
A Reduced NAD 2 6
Reduced FAD 0 2
CO2 0 4

Product Glycolysis Krebs cycle


ATP 0 2
B Reduced NAD 0 6
Reduced FAD 0 2
CO2 0 4

Product Glycolysis Krebs cycle


ATP 2 4
C Reduced NAD 2 4
Reduced FAD 0 2
CO2 0 2

Product Glycolysis Krebs cycle


ATP 0 4
D Reduced NAD 0 6
Reduced FAD 0 2
CO2 0 4

25 What would be the effect of inhibition of lactate dehydrogenase in a mammalian


cell under anaerobic conditions?

A A decrease in glycolysis, due to the lack of NAD.


B A decrease in cell pH, due to the accumulation of lactic acid.
C An increase in ATP production, due to increased amounts of reduced NAD.
D An increase in the activity of Krebs cycle, due to increased amounts of
pyruvate.

SAJC / H2 Biology 9648/1 JC2 Prelims 2013


319
10

26 Tetrdotoxin, a puffer fish toxin, blocks voltage-gated sodium channels. Black


widow spiders venom causes the voltage-gated calcium channels to be constantly
open. Crotoxin binds irreversibly to acetylcholine receptors. What will happen to
the nerve transmission if each toxin is applied?

Tetrodotoxin Black widow spiders Crotoxin


venom
A block action potentials reduce transmission of increase transmission
along axon impulse across of impulse across
synapse synapse
B increase transmission reduce transmission of block action potentials
of impulse across impulse across along axon
synapse synapse
C block action potentials increase transmission reduce transmission of
along axon of impulse across impulse across
synapse synapse
D reduce transmission of block action potentials increase transmission
impulse across along axon of impulse across
synapse synapse

27 Which one of the following statements about the transmission across a typical
chemical synapse is true?

A Neurotransmitter molecules are stored in vesicles in the dendrites.


B Vesicles containing neurotransmitter molecules diffuse to the post-
synaptic membrane.
C The binding of neurotransmitter molecules to receptors transmits an
impulse across a synapse.
D Action potentials trigger chemical changes that make the
neurotransmitter vesicles fuse with the post-synaptic membrane.

28 The diagram shows a hormone which affects metabolism in humans.

How does this hormone act on its target cell?

A It activates an enzyme cascade that amplifies the hormone signal.


B It alters specific receptor sites on the cell surface membrane.
C It enters the cell and binds to nuclear receptors.
D It inhibits the synthesis of cholesterol molecules.

SAJC / H2 Biology 9648/1 JC2 Prelims 2013


320
11

29 Some receptors for hormones like growth factors activate a protein kinase
cascade, with the participation of multiple enzymes catalysing multiple steps to
effect a change in gene expression. Which of the following statement about a
protein kinase cascade are true?

A Multiple steps in a process allow for the amplification of the signal.


B Multiple steps in an activation process means that abnormal stimulation of
a cellular response such as growth will not occur even with mutations in
more than one gene.
C Multiple steps in an activation process ensure that the chemical signal will
be sent at the corresponding magnitude as the original hormone signal.
D Having multiple steps in the process allows for an all-or-nothing
response.

30 Which of the following statements were not part of Darwins theory of evolution by
natural selection?

1 Characteristics acquired during an organisms lifetime are passed to its


offspring
2 Individuals in a sexually reproducing population are different.
3 Organisms produce more offspring than the environment can support.
4 Only those individuals best adapted to the environment survive and
reproduce.
5 Regions that encode portions of the polypeptide that are vital for structure
and function are less likely to incur mutations.

A 1 and 2
B 1 and 5
C 2 and 3
D 3 and 5

SAJC / H2 Biology 9648/1 JC2 Prelims 2013


321
12

31 A comparison was made between human, rabbit, mouse and chimpanzee of the

DNA coding sequence of the globin gene


DNA sequence in the introns of the globin gene
amino acid sequence of the globin polypeptide

The data is shown in the table below.

Sequence similarity (%)


Organisms being compared Coding DNA Intron Amino acid
sequence
Human globin / chimpanzee globin 100 98.4 100
Human globin / rabbit globin 89.3 67 92.4
Human globin / mouse globin 82.1 61 80.1

It is possible to conclude from this data that

A a human is more closely related to a mouse than to a rabbit.


B the variation between chimpanzees and humans occurs in a region of the
globin gene which would code for amino acids.
C the variation in the intron sequence between human and mouse would
account for some of the differences in the amino acid sequence.
D the comparison between rabbit and human indicates that the differences
in their DNA did not always make a difference to the amino acid
produced.

32 A researcher is interested to study the gene expression of a particular ATPase


gene in the liver of mice during development using an appropriate DNA library.
Which of the following steps are relevant in the creation of this library?

1 Restriction digestion of genomic DNA


2 Reverse transcription of mRNA
3 Ligation of single-stranded linkers to the ends of DNA
4 Restriction digestion of plasmid

A 1 and 4
B 2 and 4
C 1, 3, and 4
D 2, 3, and 4

33 Which of the following is the most powerful way of increasing the specificity of a
DNA profile analysis?

A Repeat the analysis multiple times.


B Increase the number of markers used.
C Analyze each marker by PCR rather than RFLP analysis.
D Select markers present on the sex chromosomes rather than on the
autosomes.

SAJC / H2 Biology 9648/1 JC2 Prelims 2013


322
13

34 The data below shows the results of electrophoresis of PCR fragments amplified
using primers for the site that has been shown to be altered in Huntington's
disease.

The inherited mutation in the Huntington's disease gene abnormally repeats the
nucleotide sequence CAG from 36 up to more than 120 times of that. The male
parent, shown as individual 2, had the onset of Huntington's disease when he was
40 years old.

Six of his children (individuals 3, 5, 7, 8, 10, 11) suffer from Huntington's disease,
and the age at which the symptoms first began is shown by the number below the
band from the PCR fragment.

What is the likely outcome for the normal individuals 4, 6, and 9?

A Individuals 4 and 9 do not have the trait, and will not get Huntington's
disease, but individual 6 is likely to start the disease when he reaches his
father's age of 40.
B Individuals 4, 6, and 9 have not inherited the defect causing Huntington's
disease.
C Individuals 4, 6, and 9 will still develop Huntington's disease at some point
in their lives, since the disease is inherited as a dominant trait.
D Two of the three will develop the disease, since it is inherited as a
dominant trait, but the data does not allow you to predict which two.

SAJC / H2 Biology 9648/1 JC2 Prelims 2013


323
14

35 Which of the following shows a correct comparison of the various processes?

Gene amplification Polymerase chain DNA replication


reaction
A Occurs in the nucleus Occurs in the Occurs in the nucleus
cytoplasm
B Requires nucleic acids Requires nucleic acid Does not require
nucleic acids
C Involves hydrogen Involves hydrogen Involves hydrogen
bonding bonding bonding
D Does not replicate the Replicates the entire Replicates the entire
entire template strand template strand template strand

36 Which of the following statements are true about adult stem cells?

1 They can undergo self-renewal


2 They can be totipotent, pluripotent or multipotent.
3 They can differentiate into almost any cell type.
4 They can give rise to specialised cells.

A 1 and 2
B 1 and 4
C 2 and 3
D 3 and 4

37 A patient underwent gene therapy using an adenoviral mediated delivery system.


The gene therapy failed and you were asked to find out the cause of the failure.
Which of the following could not be the explanation behind the failure in the gene
therapy?

A Failure of the expressed protein to be folded into the correct


conformation.
B The CFTR protein was not expressed in adequate amount.
C Integration of the target gene in the enhancer region.
D Rejection of the vector by the host immune system.

38 What is not expected to be an outcome of the human genome project?

A Identification of all genes present in human beings


B Identification of all alleles present in human beings
C Map and sequence the genetic makeup of every human being
D Map and sequence all the genetic material present in the chromosomes of
human beings

SAJC / H2 Biology 9648/1 JC2 Prelims 2013


324
15

39 Which uses of the information from the human genome project are generally
considered to be unethical?

1 an insurance company only giving cheap rates to people with genetic


predisposition to fewer diseases
2 genetic archaeologists identifying the earliest forms of genes to show
evolutionary relationships
3 cytologists developing tests for only some defective genes
4 doctors only giving specific drugs to block the actions of faulty genes to carriers
of those genes
5 genetic counsellors giving specific lifestyle information only to people
genetically predisposed to risks
6 parents choosing embryos for implantation only after antenatal tests for
acceptable genes

A 1 and 3
B 1 and 6
C 2 and 5
D 3 and 4

40 Which of the following is not a scientific concern relating to creating genetically


modified crops?

A Commercially important plants become resistant to drought.


B Herbicide resistance may spread to weedy species.
C Non-target insects may be affected.
D Genetically modified plants may lead to unknown risks to human health.

SAJC / H2 Biology 9648/1 JC2 Prelims 2013


325

Civics Group Index Number Name (use BLOCK LETTERS)


H2

ST. ANDREWS JUNIOR COLLEGE


2013 JC2 Preliminary Examinations

H2 BIOLOGY 9648/1

Paper 1: Multiple Choice (Mark Scheme) v3


Friday 20 September 2013 1 hour 15 minutes

Additional Materials: Multiple Choice Answer Sheet


Soft clean eraser (not supplied)
Soft pencil (type B or HB is recommended)

READ THESE INSTRUCTIONS FIRST

Do not open this booklet until you are told to do so.

Write your name, civics group and index number on the multiple choice answer sheet in
the spaces provided.

There are 40 questions in this paper. Answer all questions. For each question, there are
four possible answers, A, B, C and D.

Choose the one you consider correct and record your choice in soft pencil on the separate
multiple choice answer sheet.

INFORMATION TO CANDIDATES

Each correct answer will score one mark. A mark will not be deducted for wrong answer.
Any rough working should be done in this booklet.

At the end of the examination, submit the multiple choice answer sheet only.

This document consists of 15 printed pages.


[Turn over
326
2

1 Th
he nuclei off both plan
nt and anim
mal cells co
ontain one or more deense bodie es known
as nucleoli. Which
W one of the follo
owing corrrectly describes the fuunction of nucleoli?

A The forrmation of new DNA molecules s.


B The orgganization of the spin
ndle during
g nuclear division.
d
C Synthe
esis of chro
omatin fibrees.
D The forrmation of ribosomess.

2 Whhat could be
b a possib
ble explana
ation for th
he ability off lysosomees to withsttand self-
dig
gestion?

A lysosommes contain inactive hydrolytic enzymes


B lysosomme membrane has numerous modified proteins w with carbo
ohydrate
side-ch
hains
C lysosommes do notn contain n lipases which are e the enzyymes cappable of
digestin
ng the lipid
d membran ne
D hydrola
ases in the t lysos omes aree inhibited
d by thee acidic internal
environ
nment of th he lysosommes

3 Th
he diagram
m below sho ows the ulttrastructure aryotic cell . Which orrganelle
e of a euka
do
oes not con
ntain nucleic acid?

4 ome organ
So nisms cha ange the compositiion of the
eir membbranes to maintain
me
embrane fluidity when tempera ture chang
ges.

Whhich chang
ges occur in the com
mposition of
o the mem
mbranes folllowing a change
c to
low
w temperatture?

A Increasse phosphoolipids, deccrease choolesterol.


B Increasse saturate
ed fatty acid
ds, increas
se cholesteerol.
C Increasse saturate
ed fatty acid
ds, decreaase cholestterol.
D Increasse unsatura
ated fatty a
acids, incre
ease chole esterol.

SAJC / H22 Biology 9648/1 JC2 Prelims 2013


327
3

5 What does a haemoglobin molecule contain?

A four iron (Fe2+) ions attached to each haem group


B four oxygen molecules attached to each haem group
C four polypeptide chains each with one attached haem group
D four polypeptide chains each with four attached haem groups

6 Liver tissue produces an enzyme called catalase which breaks down


hydrogen peroxide into water and oxygen.

2H2O2 2H2O + O2

The rate of this reaction can be determined by measuring the volume of


oxygen produced in a given length of time. Students added small cubes of
fresh liver tissue to a range of hydrogen peroxide solutions and measured the
volumes of oxygen produced. Their data were used to produce the graph
showing how changing the concentration of hydrogen peroxide affected the
rate of oxygen production.

Which statements are correct?

1 At P, the rate of reaction is limited by the concentration of enzyme.


2 At Q, all of the enzyme active sites are occupied by substrate molecules.
3 At Q, the rate of reaction is limited by the concentration of the substrate.
4 At S, all of the enzyme active sites are occupied by substrate molecules.

A 1 and 4
B 2 and 3
C 2 and 4
D 1, 3 and 4

SAJC / H2 Biology 9648/1 JC2 Prelims 2013


328
4

7 Thhe graph sh
hows the effect
e of two
o different compound
ds on the rrate of reac
ction of
the
e enzyme at
a differentt substrate
e concentraations.

hich statem
Wh ment is true
e?

A Compound which results


r in ccurve R is a competitiive inhibitoor which als
so
increases the Km of the enzym me.
B Compound which results
r in ccurve R is a non-competitive inhhibitor whicch did
not affecct the Km off the enzym me.
C Compound which results
r in ccurve S is a competitive inhibitoor which alsso
decrease es the Km of
o the enzyyme.
D Compound which results
r in ccurve S is a non-competitive inhhibitor whicch
increases the Km of the enzym me.

8 Ahhypothetical diploid cell


c has fouur chromos somes and
d total amoount of DNA
A in the
cell is denote
ed by X before
b DNA
A replicatio
on.

hich of the following is correct?


Wh ?

Cell Cycle Phase Num


mber of Am
mount of DNA
chromoso
omes per cell per celll
A Interphase 8 2X
B Telophase of mitosis 8 X
C Metapphase of Meiosis
M I 4 X
D Anaph hase of Me
eiosis II 4 X

9 Whhen identiical twins marry id entical tw


wins, the children
c oof both co
ouple are
ge
enetically

A identiccal unless crossing oover takes place


B identiccal becausse of the efffect of sem
mi-conserv vative DNA
A replicatio
on prior
to nucclear divisio
on
C differe
ent becaus se non-disjuunction of chromatids occurs
D differe
ent becaus se of rando
om segrega ation during parental meiosis

SAJC / H22 Biology 9648/1 JC2 Prelims 2013


329
5

10 To function as the heritable genetic code, DNA molecules must have all of the
following structural features except

A the ability to form complementary base-pairs with RNA molecules.


B exist as a very stable, double helical form when being replicated
C a sequence of nucleotides that can be decoded into a sequence of amino
acids in a protein
D ability to undergo semi-conservative replication

11 A particular eukaryotic protein is 300 amino acids long. Which of the following
could be the minimum number of nucleotides in the DNA molecule containing the
gene that codes for the amino acids in this protein?

A 100
B 300
C 900
D 1800

12 Which process in bacteria always allows chromosomal and non-chromosomal


DNA to be transferred?

A Binary fission.
B Conjugation.
C Transduction.
D Transformation.

13 Which of the following statements about binary fission is not true?

1 The mode of replication is semi-conservative


2 Chromosomes are separated to opposite ends of the cell.
3 DNA replicates using the rolling circle mechanism.
4 DNA replication at the ori is bi-directional.

A 3 only
B 1 and 2
C 2 and 4
D 3 and 4

14 Which of the following statements concerning lac operon is true?

1 Transcription of lac operon takes place all the time.


2 There is one single mRNA transcribed from the lac operon.
3 There is one start and one stop codon in the mRNA of lac operon.
4 The repressor molecule binds to the operator to turn off lac operon.

A 4 only
B 1 and 3
C 2 and 4
D 2, 3 and 4

SAJC / H2 Biology 9648/1 JC2 Prelims 2013


330
6

15 How do viruses cause diseases in animals?

1 They inhibit normal synthesis of host cell DNA, RNA, or protein.


2 They degrade the host cells chromosomes.
3 They disrupt the oncogenes of the host cell causing uncontrolled cell division.
4 Their viral proteins and glycoproteins on the surface membrane of host cells
cause them to be recognized and destroyed by the bodys immune defences.

A 1 and 3
B 2 and 3
C 1, 2 and 4
D 1, 2, 3 and 4

16 The pedigree shows the inheritance of a condition known as galactosemia.

1 2 3 4

5 6 7 8 9 10 11

12 13 14 15 16

What is the probability that the first child of individuals 15 and 16 will be a
normal boy?

A 0.125
B 0.25
C 0.50
D 0.75

SAJC / H2 Biology 9648/1 JC2 Prelims 2013


331
7

17 Agouti mice have banded hairs, giving a grey colour. Black mice have unbanded
hairs. White mice have no pigment. A cross between a homozygous black mouse
and a white mouse produced offspring with agouti hair. Another cross between the
black mouse and another white mouse produced some offspring with agouti hair
and some with black hair.

What explains these observations on the phenotype of hair of mice?

A There is a single gene with two codominant alleles, black and white.
B There is a single gene with three alleles in a dominance series, black
grey white.
C There are two epistatic genes, one controlling pigment production and one
controlling banding.
D There are two linked genes, one controlling pigment production and one
controlling banding.

18 With reference to prokaryotic and eukaryotic genomes, which of the following


statement is not true?

A They both have extrachromosomal DNA.


B They both have non-coding regions.
C They both have regulatory sequences.
D They are both associated with histones.

19 Four different genes are regulated in different ways.

Gene 1 undergoes tissue-specific patterns of alternative splicing


Gene 2 is part of a group of structural genes controlled by the same regulatory
sequences
Gene 3 is in some circumstances subject to methylation
Gene 4 codes for a repressor protein which acts at an operator site close by

Which role of the table correctly identifies which genes are prokaryotic and which
are eukaryotic?

prokaryotic eukaryotic
A 1 and 2 3 and 4
B 1 and 3 2 and 4
C 2 and 3 1 and 4
D 2 and 4 1 and 3
20 Which of the following statements about the eukaryotic control elements are
correct?

1 Attachment of the RNA polymerase II at the TATA box is achieved with


the help of a series of specific transcription factors
2 Upstream regulatory elements (UREs) increase the basal level of
transcription
3 A given gene may have multiple enhancers, each active at a different time
or in a different cell type or location in the organism
4 Repressors bind to silencer regions of DNA far upstream of promoters to

SAJC / H2 Biology 9648/1 JC2 Prelims 2013


332
8

repress transcription

A 1 and 4
B 2 and 3
C 2, 3 and 4
D 1, 2, 3 and 4

21 Cells taken from a human bone cancer multiplied readily in culture. Analysis
showed that the cells were homozygous for the deletion of an allele on
chromosome 13 coding for a protein, RB.

Addition of RB to these cells reduced their rate of division.

Which individual is not able to produce a tumour suppressor?

A Both chromosomes 13 carry alleles for RB.


B Both chromosomes 13 have the alleles for RB deleted.
C One chromosome 13 carries an allele for RB, but the other chromosome 13
carries the deletion.
D One chromosome 13 carries an allele for RB, but the other chromosome 13
carries a different mutation.

22 Which of the following processes could still occur in a chloroplast in the presence
of an inhibitor that prevents H+ from passing through ATP synthase complexes?

1 Sugar synthesis
2 Photolysis of water
3 Transfer of electrons down the electron transport chain
4 Oxidation of NADPH

A 1 and 2
B 1and 4
C 2 and 3
D 3 and 4

SAJC / H2 Biology 9648/1 JC2 Prelims 2013


333
9

23 Some photosynthetic organisms containing chloroplasts that lack PS II


(photosystem II) are able to survive. The best way to detect the lack of PS II in
these organisms would be to

A test for carbon fixation in the dark.


B test for liberation of oxygen in the light.
C determine the production of starch.
D determine if they have thylakoids in the chloroplasts.

24 A mitochondria suspension obtained from liver cells is prepared for investigations


of the products of respiration. Acetyl-CoA is added to the suspension.

Which of the following correctly matched the products of glycolysis and Krebs
cycle for every oxidation of one glucose molecule in this mitochondria suspension?

Product Glycolysis Krebs cycle


ATP 2 2
A Reduced NAD 2 6
Reduced FAD 0 2
CO2 0 4

Product Glycolysis Krebs cycle


ATP 0 2
B Reduced NAD 0 6
Reduced FAD 0 2
CO2 0 4

Product Glycolysis Krebs cycle


ATP 2 4
C Reduced NAD 2 4
Reduced FAD 0 2
CO2 0 2

Product Glycolysis Krebs cycle


ATP 0 4
D Reduced NAD 0 6
Reduced FAD 0 2
CO2 0 4

25 What would be the effect of inhibition of lactate dehydrogenase in a mammalian


cell under anaerobic conditions?

A A decrease in glycolysis, due to the lack of NAD+.


B A decrease in cell pH, due to the accumulation of lactic acid.
C An increase in ATP production, due to increased amounts of reduced NAD.
D An increase in the activity of Krebs cycle, due to increased amounts of
pyruvate.

SAJC / H2 Biology 9648/1 JC2 Prelims 2013


334
10

26 Tetrdotoxin, a puffer fish toxin, blocks voltage-gated sodium channels. Black


widow spiders venom causes the voltage-gated calcium channels to be constantly
open. Crotoxin binds irreversibly to acetylcholine receptors. What will happen to
the nerve transmission if each toxin is applied?

Tetrodotoxin Black widow spiders Crotoxin


venom
A block action potentials reduce transmission of increase transmission
along axon impulse across of impulse across
synapse synapse
B increase transmission reduce transmission of block action potentials
of impulse across impulse across along axon
synapse synapse
C block action potentials increase transmissionreduce transmission of
along axon of impulse across impulse across
synapse synapse
D reduce transmission of block action potentials
increase transmission
impulse across along axon of impulse across
synapse synapse
27 Which one of the following statements about the transmission across a typical
chemical synapse is true?

A Neurotransmitter molecules are stored in vesicles in the dendrites.


B Vesicles containing neurotransmitter molecules diffuse to the post-
synaptic membrane.
C The binding of neurotransmitter molecules to receptors transmits an
impulse across a synapse.
D Action potentials trigger chemical changes that make the
neurotransmitter vesicles fuse with the post-synaptic membrane.

28 The diagram shows a hormone which affects metabolism in humans.

How does this hormone act on its target cell?

A It activates an enzyme cascade that amplifies the hormone signal.


B It alters specific receptor sites on the cell surface membrane.
C It enters the cell and binds to nuclear receptors.
D It inhibits the synthesis of cholesterol molecules.

SAJC / H2 Biology 9648/1 JC2 Prelims 2013


335
11

29 Some receptors for hormones like growth factors activate a protein kinase
cascade, with the participation of multiple enzymes catalysing multiple steps to
effect a change in gene expression. Which of the following statement about a
protein kinase cascade are true?

A Multiple steps in a process allow for the amplification of the signal.


B Multiple steps in an activation process means that abnormal stimulation of
a cellular response such as growth will not occur even with mutations in
more than one gene.
C Multiple steps in an activation process ensure that the chemical signal will
be sent at the corresponding magnitude as the original hormone signal.
D Having multiple steps in the process allows for an all-or-nothing
response.

30 Which of the following statements were not part of Darwins theory of evolution by
natural selection?

1 Characteristics acquired during an organisms lifetime are passed to its


offspring
2 Individuals in a sexually reproducing population are different.
3 Organisms produce more offspring than the environment can support.
4 Only those individuals best adapted to the environment survive and
reproduce.
5 Regions that encode portions of the polypeptide that are vital for structure
and function are less likely to incur mutations.

A 1 and 2
B 1 and 5
C 2 and 3
D 3 and 5

SAJC / H2 Biology 9648/1 JC2 Prelims 2013


336
12

31 A comparison was made between human, rabbit, mouse and chimpanzee of the

DNA coding sequence of the globin gene


DNA sequence in the introns of the globin gene
amino acid sequence of the globin polypeptide

The data is shown in the table below.

Sequence similarity (%)


Organisms being compared Coding DNA Intron Amino acid
sequence
Human globin / chimpanzee globin 100 98.4 100
Human globin / rabbit globin 89.3 67 92.4
Human globin / mouse globin 82.1 61 80.1

It is possible to conclude from this data that

A a human is more closely related to a mouse than to a rabbit.


B the variation between chimpanzees and humans occurs in a region of the
globin gene which would code for amino acids.
C the variation in the intron sequence between human and mouse would
account for some of the differences in the amino acid sequence.
D the comparison between rabbit and human indicates that the differences
in their DNA did not always make a difference to the amino acid
produced.

32 A researcher is interested to study the gene expression of a particular ATPase


gene in the liver of mice during development using an appropriate DNA library.
Which of the following steps are relevant in the creation of this library?

1 Restriction digestion of genomic DNA


2 Reverse transcription of mRNA
3 Ligation of single-stranded linkers to the ends of DNA
4 Restriction digestion of plasmid

A 1 and 4
B 2 and 4
C 1, 3, and 4
D 2, 3, and 4

33 Which of the following is the most powerful way of increasing the specificity of a
DNA profile analysis?

A Repeat the analysis multiple times.


B Increase the number of markers used.
C Analyze each marker by PCR rather than RFLP analysis.
D Select markers present on the sex chromosomes rather than on the
autosomes.

SAJC / H2 Biology 9648/1 JC2 Prelims 2013


337
13

34 The data below shows the results of electrophoresis of PCR fragments amplified
using primers for the site that has been shown to be altered in Huntington's
disease.

The inherited mutation in the Huntington's disease gene abnormally repeats the
nucleotide sequence CAG from 36 up to more than 120 times of that. The male
parent, shown as individual 2, had the onset of Huntington's disease when he was
40 years old.

Six of his children (individuals 3, 5, 7, 8, 10, 11) suffer from Huntington's disease,
and the age at which the symptoms first began is shown by the number below the
band from the PCR fragment.

What is the likely outcome for the normal individuals 4, 6, and 9?

A Individuals 4 and 9 do not have the trait, and will not get Huntington's
disease, but individual 6 is likely to start the disease when he reaches his
father's age of 40.
B Individuals 4, 6, and 9 have not inherited the defect causing Huntington's
disease.
C Individuals 4, 6, and 9 will still develop Huntington's disease at some point
in their lives, since the disease is inherited as a dominant trait.
D Two of the three will develop the disease, since it is inherited as a
dominant trait, but the data does not allow you to predict which two.

SAJC / H2 Biology 9648/1 JC2 Prelims 2013


338
14

35 Which of the following shows a correct comparison of the various processes?

Gene amplification Polymerase chain DNA replication


reaction
A Occurs in the nucleus Occurs in the Occurs in the nucleus
cytoplasm
B Requires nucleic acids Requires nucleic acid Does not require
nucleic acids
C Involves hydrogen Involves hydrogen Involves hydrogen
bonding bonding bonding
D Does not replicate the Replicates the entire Replicates the entire
entire template strand template strand template strand

36 Which of the following statements are true about adult stem cells?

1 They can undergo self-renewal


2 They can be totipotent, pluripotent or multipotent.
3 They can differentiate into almost any cell type.
4 They can give rise to specialised cells.

A 1 and 2
B 1 and 4
C 2 and 3
D 3 and 4

37 A patient underwent gene therapy using an adenoviral mediated delivery system.


The gene therapy failed and you were asked to find out the cause of the failure.
Which of the following could not be the explanation behind the failure in the gene
therapy?

A Failure of the expressed protein to be folded into the correct


conformation.
B The CFTR protein was not expressed in adequate amount.
C Integration of the target gene in the enhancer region.
D Rejection of the vector by the host immune system.

38 What is not expected to be an outcome of the human genome project?

A Identification of all genes present in human beings


B Identification of all alleles present in human beings
C Map and sequence the genetic makeup of every human being
D Map and sequence all the genetic material present in the chromosomes of
human beings

SAJC / H2 Biology 9648/1 JC2 Prelims 2013


339
15

39 Which uses f the information from the human genome project are generally
considered to be unethical?

1 an insurance company only giving cheap rates to people with genetic


predisposition to fewer diseases
2 genetic archaeologists identifying the earliest forms of genes to show
evolutionary relationships
3 cytologists developing tests for only some defective genes
4 doctors only giving specific drugs to block the actions of faulty genes to carriers
of those genes
5 genetic counsellors giving specific lifestyle information only to people
genetically predisposed to risks
6 parents choosing embryos for implantation only after antenatal tests for
acceptable genes

A 1 and 3
B 1 and 6
C 2 and 5
D 3 and 4

40 Which of the following is not a scientific concern relating to creating genetically


modified crops?

A Commercially important plants become resistant to drought.


B Herbicide resistance may spread to weedy species.
C Non-target insects may be affected.
D Genetically modified plants may lead to unknown risks to human health.

SAJC / H2 Biology 9648/1 JC2 Prelims 2013


340

Civics Index Name (use BLOCK LETTERS)


Group Number H2
ST. ANDREWS JUNIOR COLLEGE
2013 JC2 Preliminary Examinations

H2 BIOLOGY 9648/2

Paper 2: Core
Thursday 29 August 2013 2 hours

Additional Materials: Answer Paper


Cover Sheet for Section B

READ THESE INSTRUCTIONS FIRST

Write your name, civics group and index number on all the work you
hand in.
Write in dark blue or black pen on both sides of the paper.
You may use a soft pencil for any diagram, graph or rough working.
Do not use staples, paper clips, highlighters, glue or correction fluid.

Section A (Structured Questions)


Answer all seven questions.
Write your answers in the spaces provided on the question paper.

Section B (Essay Question) For Examiners Use


Answer one essay question.
Section A
Write your answers on the separate answer paper provided.
All working for numerical answers must be shown. 1
/11

2
/13
INFORMATION TO CANDIDATES 3
/11

At the end of the examination, 4


/12
1. Attach Section B answers to the cover sheet provided.
5
/12

6
/11
The number of marks is given in brackets [ ] at the end of each
question or part question. 7
/10

Total
/80

This document consists of 19 printed pages.


[Turn over
341
2
Section A

Answ
wer all ques
stions.

1. Fig. 1.1 show ws the intracellular annd extracellular eventts in the forrmation of a
collageen fibril. Co
ollagen fibrils are shoown assemmbling in thee extracell ular spacee
containned within a large info olding in th
he plasma membrane e. Collagenn fibrils cann form
ordered d arrays in the extraccellular spaace by further assemmbling into llarge collagen
fibre.

Fig 1.1

(a) P representss two organ


nelles foun
nd in the eu
ukaryotic cell.
c Identiffy these tw
wo
organe
elles.



...

....[1]

Procollagen peptidase remo oves the te


erminal peptides of th
he procollaagen to form
collage
en in Step 7 of Fig. 1..1.

(b) Exxplain the interaction between p


procollagen and proc
collagen peeptidase.



...

........



...

........



...

........



...

....[2]

SAJC / H2 B
Biology 9648/2 JC2 Prelims 2013
342
3
(c) Suggest why assembly of collagen fibril is done outside the cell.
...........
.......[1]

(d) Vitamin C is necessary in aiding the conversion of proline and lysine to


hydroxyproline and hydroxylysine. A deficiency in Vitamin C could lead to scurvy, a
disease displaying symptoms such as loss of teeth and easy bruising.

Collagen is an important matrix for the deposition of hard mineral crystals in the
formation of bones. Certain forms of osteogenesis imperfecta, or brittle-bone
disease, are caused by a single change of amino acid in the middle of the chain, from
glycine to arginine.

With reference to the structure of collagen, account for the following:

(i) Bleeding gum and easy bruising in scurvy


...........
...........
...........
...........
...........
...........
...........
.......[4]

(ii) weak and fragile bones in osteogenesis imperfecta


...........
...........
...........
...........
...........
.......[3]

[Q1 Total: 11]

SAJC / H2 Biology 9648/2 JC2 Prelims 2013


343
4
2.
(a) During semi-conservative DNA replication, both parental strands act as
templates for synthesis of complementary daughter strands. One of the daughter
strands, called the leading strand, is synthesized continuously. The other daughter
strand, called the lagging strand, is synthesized discontinuously in the form of short
fragments. The replicated DNA molecule consist of one new and one original strand
of DNA.

(i) State 3 structural differences in the formation of the leading and lagging strands.
...........
...........
...........
......[1]

(ii) Explain why the two new strands of DNA are synthesized in opposite directions.
...........
......[1]

(b) State three differences between the structures of DNA and RNA. [3]

DNA RNA

(c) In eukaryotic cells, the end-replication problem explains why normal somatic
cells are limited in the number of mitotic divisions before they die out.

(i) Outline the end-replication problem in eukaryotic chromosomes.


...........
...........
...........
...........
...........
..........[3]

SAJC / H2 Biology 9648/2 JC2 Prelims 2013


344
5
(ii) What are telomeres?
...........
..........[1]

(iii) State the role of telomeres in relation to the end-replication problem.


...........
......[]

(iv) Suggest why bacteria do not have telomeres.


...........
.......[1]

In bacteria cells, binary fission differs significantly from the process of mitosis seen in
eukaryotic cells.

(d) Explain the significance of binary fission in bacteria


...........
...........
...........
.......[2]

[Q2 Total: 13]

SAJC / H2 Biology 9648/2 JC2 Prelims 2013


345
6
3.
(a) Fig. 3.1 shows two different viruses.

Virus A Virus B
Fig. 3.1

(i) Relate the structure of virus A to its mechanism of release from the host.
...........
...........
...........
...[2]

(ii) Contrast the reproductive cycles of the Virus A and Virus B. [3]

Virus A Virus B

SAJC / H2 Biology 9648/2 JC2 Prelims 2013


346
7
(b) One of the most common treatment methods for patients infected with virus B is
the use of reverse transcriptase inhibitors. These inhibitors bind to an inhibitor-
binding site (other than the active site) on the reverse transcriptase.

However, virus B often develop resistance against these inhibitors due to a single
base substitution in the DNA sequence resulting in a change of an amino acid in a
subunit of reverse transcriptase. The mutant protein is full-length and retains reverse
transcriptase activity, but does not allow the binding of the inhibitors.

(ii) Explain why the mutated protein remains full-length and retains reverse
transcriptase activity, but does not allow the binding of the inhibitors.
...........
...........
...........
...........
..[2]

(c) In recent years, vaccines have been developed for virus B. Vaccines can either
be live but weakened forms of the virus, or its structural parts that are recognised by
the immune system. This provides immunity to the disease.

However, vaccine development for virus B has become more challenging due to the
viruss ability to undergo a high rate of antigenic drift.

(i) Suggest a reason for this high rate of antigenic drift and how it affects vaccine
development.
...........
...........
...........
...........
..[2]

(ii) A student tries to produce a vaccine by heating a human virus to 100C for five
minutes to inactivate its infectivity. Suggest why effective vaccines cannot be
produced using the virus that has been heated to 100C.
...........
...[1]

[Q3 Total: 11]

SAJC / H2 Biology 9648/2 JC2 Prelims 2013


347
8
4. Familial Dysautonomia (FD), is an autosomal disorder which affects the
development of sensory neurons in the autonomic and sensory nervous system,
resulting in various symptoms including the insensitivity to pain.

Mutation in another gene, transmembrane channel 1 gene (TMC1), results in a form


of DFNB7 hearing loss which is also inherited in an autosomal manner.

A world-wide, long term genetic study on the possible associations between FD and
hearing loss is implemented in 2003. Table 4.1 above shows the genotypes and
phenotypes of some subjects in this study.

Subject
Gene name Genotype Phenotype
number
FD Heterozygous No FD
1031
TMC1 Heterozygous Normal hearing
FD Homozygous recessive FD
1050
TMC1 Homozygous dominant Normal hearing
FD Homozygous dominant No FD
1088
TMC1 Homozygous recessive Hearing loss
FD Homozygous recessive FD
1092
TMC1 Homozygous recessive Hearing loss

Table 4.1

(a) Define the following terms:

(i) genotype
...........
...[1]

(ii) phenotype
...........
...[1]

(b) Subject 1031 and another subject listed in Table 4.1 are married and they have
their first child in 2005. This cross between them theoretically resembles a test
cross.

(i) Deduce the subject number of subject 1031s wife from Table 4.1.
......[]

SAJC / H2 Biology 9648/2 JC2 Prelims 2013


348
9
Subject 1031 has a hearing impaired father with FD; his mother has normal hearing
and does not suffer from FD (she is homozygous dominant for both gene locus).

A total of 200 couples with the same genotypes as subjects 1031 and his wife were
monitored in this study. Information on their children (total number of 186) was
collated 10 years after the implementation of this study and presented in Table 4.2.

Phenotype Number of children


No FD, Normal hearing 59
FD, Normal hearing 35
No FD, Hearing loss 26
FD, Hearing loss 66

Table 4.2

(ii) Using the information give in Table 4.2, calculate the 2 and state the
conclusions derived from this test.
2 =
Conclusion .............
...........
...........
...[2]

(O E) 2
=
2
v=c1
E
where = sum of O = observed value
v = degrees of freedom E = expected value
c = number of classes

A chi-squared table.
degrees of probability, p
freedom 0.10 0.05 0.02 0.01 0.001
1 2.71 3.84 5.41 6.64 10.83
2 4.61 5.99 7.82 9.21 13.82
3 6.25 7.82 9.84 11.35 16.27
4 7.78 9.49 11.67 13.28 18.47

(iii) Explain the discrepancy from the expected results.


...........
...........
...........
...........
...........
...[3]

SAJC / H2 Biology 9648/2 JC2 Prelims 2013


349
10
(iv) Using the symbols provided, draw a genetic diagram of the cross between
Subject 1031 and his wife. [2]

Symbols:
F dominant allele for no FD
f recessive allele for FD
T dominant allele for Normal hearing
t recessive allele for hearing loss

During gamete formation in Subject 1031s wife from a diploid germ cell, an error
occurred once. As a result, her children will have 50% chance of having 3 copies of
chromosome 9; and 50% of having only 1 copy of chromosome 9 in his/her cells.

(v) Explain how this could have happened.


...........
...........
...........
...[2]

[Q4 Total: 12]


SAJC / H2 Biology 9648/2 JC2 Prelims 2013
350
11
5.
(a) Fig
g. 5.1 showws the norm mal seque nce of cell signaling events whhen insulin
reaches its targett cell and binds
b to an
n insulin rec
ceptor.

Fig. 5.1

(i) Sta
ate the sou
urce of the insulin mo
olecule in Fig
F 5.1.



...

....[1]

(ii) With reference to the structure


s off the plasm
ma membra ane, descriibe how the
structural features of recepttor tyrosine
e kinase en nables it to
o be embeddded as a
transmembrane protein.
p



...

........



...

........



...

........



...

........



...

........



...

....[3]

(iii) W
With referen
nce to Fig. 5.1, descrribe the eve
ents occurrring in stagges A to D.
D



...

........



...

........



...

........



...

........



...

........



...

........



...

........



...

....[3]

SAJC / H2 B
Biology 9648/2 JC2 Prelims 2013
351
12
(iv) Based on your knowledge, describe the events that lead to an increase in the
number of glucose transporters in the cell surface membrane.
...........
...........
.......[1]

(v) A tissue may, over time, lose its responsiveness to insulin, even though insulin
concentration remains unchanged. Suggest possible reasons for this decrease in
responsiveness.
...........
.......[1]

An aqueous solution contains small membranous vesicles which may be derived


either from a thylakoid or a mitochondrial membrane. In these vesicles, the
orientation of the ATP synthase enzymes is preserved as it is in the cell.

Fig. 5.2 shows the various experimental systems that are prepared in an attempt to
synthesize ATP.

Fig. 5.2

(b) Which experimental system (A to D) shown in Fig. 5.2 would result in the
successful synthesis of ATP by thylakoid-derived vesicles? Explain your answer.
...........
...........
...........
...........
...........
......[3]

[Q5 Total: 12]


SAJC / H2 Biology 9648/2 JC2 Prelims 2013
352
13
6.
(a) During an action potential, the permeability of the cell-surface membrane of an
axon changes. Fig 6.1 shows changes in permeability of the membrane to sodium
ions (Na+) and to potassium ions (K+) during a single action potential.

Fig. 6.1

(i) Explain the shape of the curve for sodium ions between 0.5 ms and 0.7 ms.
...........
...........
...........
...[2]

(ii) During an action potential, the membrane potential rises to +40mV and then falls.
Using information from the graph, explain the fall in membrane potential.
...........
...........
...........
...[2]

(iii) After exercise, some ATP is used to re-establish the resting potential in axons.
Explain how the resting potential is re-established.
...........
...[1]

SAJC / H2 Biology 9648/2 JC2 Prelims 2013


353
14
(b) Th
he black mamba is a poisonouss snake wh
hich produc
ces a toxinn.

(i) The
e length off the sectio
on of DNA that codes
s for the co
omplete toxxin is longe
er than
the mR
RNA used for
f translattion. Explaain.



...
...[]

(ii) Thhe black mambas tox xin kills pre


ey by inhib
biting the en
nzyme aceetylcholinesterase
at neurromuscularr junctions that contro ols musclees involvedd in breathi ng. Explain how
this inh
hibition affe
ects breath
hing.


...

........



...

........



...

........



....[2]

(c) Do opamine iss a neurotraansmitter rreleased by dopamin nergic neurrons for the
e
transmission of nerve signa als importa nt for moto
or control. Dopaminee binds to and
a
activate
es its recep
ptors locatted on the post-synapptic membrane.

Fig 6.2 shows a crystallogra


c aphy of the
e dopamine
e receptor.

Fig 6.2

(i) Witth referencce of Fig. 6.2,


6 identifyy the type of
o receptorr which thee dopamine
e
recepto
or belongs to.



...
...[]

SAJC / H2 B
Biology 9648/2 JC2 Prelims 2013
354
15
Excesssive neurottransmissio
on of dopaamine is asssociated with
w schizoophrenia, a clinical
mental disorder. To
T relieve this disord
der, antipsy ugs are devveloped to bind to
ychotic dru
and ina
activate the
e post-syna
aptic dopa mine receptors.

Fig 6.3 shows the


e cell signa
alling pathw
way when dopamine binds to itts receptorrs.

Fig 6.3

(ii) With reference to Fig 6.3,


6 briefly describe how
h binding
g of the druugs to the
recepto
or affects downstream
d m signallin g pathway
y.



... ........



...

........



...

........



...

........



...

........



...

........



...[3]

otal: 11]
[Q6 To

SAJC / H2 B
Biology 9648/2 JC2 Prelims 2013
355
16
7. Fishes in the genus Anisotremus comprise of ten described species which occur
predominantly on coral reefs and subtropical rocky reefs in the Neotropics.

(a) Bernardi et.al. did a molecular phylogenetic study on such fishes in that area.
Results are shown in Fig. 7.1.

Fig. 7.1

(i) Explain how molecular methods can be used to elucidate the evolutionary
relationships of the different species of Anisotremus fishes.
...........
...........
...........
...........
..............[2]

SAJC / H2 Biology 9648/2 JC2 Prelims 2013


356
17
(ii) Explain why it may be more reliable to construct a phylogenetic tree of the ten
species of Anisotremus using molecular data instead of morphological comparisons.
...........
...[1]

(iii) With reference to Fig. 7.1 and molecular homology, comment on the
evolutionary relationship between A. taeniatus and A. virginicus.
...........
..............[1]

SAJC / H2 Biology 9648/2 JC2 Prelims 2013


357
18
(b) A. taeniatus is found in
n the Pacifiic Ocean, whereas
w A.
A virginicuss is found in the
Caribbe
ean Sea. These
T two species wwere derived due to th
he formatioon of the Is
sthmus
of Pana
ama aboutt 3.5 million
n years ag o. Before that
t event, the waters
rs of the Pa
acific
Ocean and Caribbean Sea mixed free ely.

(i) Expplain how the


t formation of the IIsthmus off Panama results
r in thhe emerge
ence of
A. taen
niatus and A.
A virginicu
us.



...

........



...

........



...

........



...

........



...

........



...

........



...

........



...

........



..
.[4]

(c) Exxplain why it is impos


ssible for evvolution to
o occur at the individuual level.



... ........



...

........



...

........



..
.[2]

[Q7 To
otal: 10]

SAJC / H2 B
Biology 9648/2 JC2 Prelims 2013
358
19
Section B

Answer one question.

Your answers should be illustrated by large, clearly labelled diagrams, where


appropriate.

Your answers must be in continuous prose, where appropriate.

Your answers must be set out in sections (a), (b) etc., as indicated in the question.

8 (a) Distinguish between the concepts of repressible and inducible systems


of gene regulation. [4]
(b) Outline the differences between prokaryotic control of gene expression
with the eukaryotic model. [10]
(c) Describe how gene amplification can occur and the significance of gene
amplification in development [6]

[Total: 20]

OR

9 (a) Describe the molecular structure of starch (amylose) and cellulose and
relate these structures to their functions in living organisms. [7]
(b) Explain, with two named examples, how the environment may affect the
phenotype. [6]
(c) Explain how biogeography and the fossil record support the
evolutionary deductions based on homologies. [7]

[Total: 20]

SAJC / H2 Biology 9648/2 JC2 Prelims 2013


359

Civics Index Name (use BLOCK LETTERS)


Group Number H2
ST. ANDREWS JUNIOR COLLEGE
2013 JC2 Preliminary Examinations

H2 BIOLOGY 9648/2

Paper 2: Core (Mark Scheme)


Thursday 29 August 2013 2 hours

Additional Materials: Answer Paper


Cover Sheet for Section B

READ THESE INSTRUCTIONS FIRST

Write your name, civics group and index number on all the work you
hand in.
Write in dark blue or black pen on both sides of the paper.
You may use a soft pencil for any diagram, graph or rough working.
Do not use staples, paper clips, highlighters, glue or correction fluid.

Section A (Structured Questions)


Answer all seven questions.
Write your answers in the spaces provided on the question paper.

Section B (Essay Question) For Examiners Use


Answer one essay question.
Section A
Write your answers on the separate answer paper provided.
All working for numerical answers must be shown. 1
/11

2
/13
INFORMATION TO CANDIDATES 3
/11

At the end of the examination, 4


/12
1. Attach Section B answers to the cover sheet provided.
5
/12

6
/11
The number of marks is given in brackets [ ] at the end of each
question or part question. 7
/10

Total
/80

This document consists of 30 printed pages.


[Turn over
360
2
Section A

Answ
wer all ques
stions.

1. Fig. 1.1 show ws the intracellular annd extracellular eventts in the forrmation of a
collageen fibril. Co
ollagen fibrils are shoown assemmbling in thee extracell ular spacee
containned within a large info olding in th
he plasma membrane e. Collagenn fibrils cann form
ordered d arrays in the extraccellular spaace by further assemmbling into llarge collagen
fibre.

Fig 1.1

(a) P representss two organ


nelles foun
nd in the eu
ukaryotic cell.
c Identiffy these tw
wo
organe
elles.

...

....[1]
1 roug
gh endopla
asmic retic
culum
2 Golgi body

Examineers comme ents:


More spe
ecific label must
m be given
n: ACCEPT b
bound riboso
omes

Procollagen peptidase remo oves the te he procollaagen to form


erminal peptides of th
collage
en in Step 7 of Fig. 1..1.

(b) Exxplain the interaction between p


procollagen and proc
collagen peeptidase.
... ....[2]
1 3D conforma ation/struc cture of proocollagen molecule is s complem mentary to
2 shaape of activve site of procollagen n peptidasee
3 formm enzyme--substrate complex
4 ref. lock-and-kkey hypoth hesis / induuced-fit hyp
pothesis
5 contact amino o acid in ac
ctive site fo
or brief bon
nding with substrate
/ via
a hydrogen n bonds an nd/or ionic bonds
6 cataalytic amino acids are e brought iinto correcct orientatio
ons in the aactive site
/ reff. inducing stress in bonds
b of suubstrate
/ reff. increasin
ng substratte reactivityy
SAJC / H2 B
Biology 9648/2 JC2 Prelims 2013
361
3
(c) Suggest why assembly of collagen fibril is done outside the cell.
.......[1]
1 collagen fibrils are too large (to be assembled inside the cell)
2 cannot be secreted out of cell
/ ref. affect internal cell environment

(d) Vitamin C is necessary in aiding the conversion of proline and lysine to


hydroxyproline and hydroxylysine. A deficiency in Vitamin C could lead to scurvy, a
disease displaying symptoms such as loss of teeth and easy bruising.

Collagen is an important matrix for the deposition of hard mineral crystals in the
formation of bones. Certain forms of osteogenesis imperfecta, or brittle-bone
disease, are caused by a single change of amino acid in the middle of the chain, from
glycine to arginine.

With reference to the structure of collagen, account for the following:

(i) Bleeding gum and easy bruising in scurvy


.......[4]
1 collagen is an important component in epidermis of gums and skin

2 lack of OH groups on proline


3 to form hydrogen bonds
4 to further stabilize tropocollagen structure

5 lack of hydroxylysine residues for formation of covalent bonds


6 between tropocollagen molecules forming collagen fibril
7 basic units of tropocollagen slide against each other

8 unable to form stable bundles of collagen fibres


9 leading to low tensile strength of collagen

Examiners comments:
Candidates should answer the question more directly, instead of phrasing the answers in a indirect,
reversed manner.
Order of packing: Tropocollagen (triple-helix) collagen fibrils collagen fibre
The terms microfibrils and macrofibrils are used to describe cellulose, NOT collagen!

(ii) weak and fragile bones in osteogenesis imperfecta


.......[3]
1 (R group of) arginine is too large
2 to fit into the crowded centre of the triple helix (REJECT: -helix)
3 unable to form hydrogen bonds between the NH groups of arginine residues on
each strand and CO groups on the other two strands.
4 unable to form stable tropocollagen
5 results in low tensile strength
6 lack stable matrix for deposition of minerals in bone (REJECT: no matrix)

Tutor: reject general statements about change in 3D structure due to amino acid change

[Q1 Total: 11]

SAJC / H2 Biology 9648/2 JC2 Prelims 2013


362
4
2.
(a) During semi-conservative DNA replication, both parental strands act as
templates for synthesis of complementary daughter strands. One of the daughter
strands, called the leading strand, is synthesized continuously. The other daughter
strand, called the lagging strand, is synthesized discontinuously in the form of short
fragments. The replicated DNA molecule consist of one new and one original strand
of DNA.

(i) State 3 structural differences in the formation of the leading and lagging strands.
......[1]
1 Presence of DNA ligase in lagging strand to ligate Okazaki fragments;
2 Presence of Okazaki fragments in lagging strand;
3 Presence of more than 1 primer/primase in lagging strand;
(REJECT: no primer needed in leading strand. This is incorrect!)
4 Strands are synthesized in opposite directions;

(ii) Explain why the two new strands of DNA are synthesized in opposite directions.
......[1]
1 DNA polymerase can only add nucleotides to the free 3 OH end of elongating
DNA strand
2 Template strands are antiparallel

(b) State three differences between the structures of DNA and RNA. [3]

DNA RNA
1 Exists as a double-stranded helix Single-stranded
2 Pentose sugar is deoxyribose Pentose sugar is ribose
3 Bases are A, T, C and G Bases are A, U, C and G
4 Ratio of A to T and G to C is always 1. Ratio varies.

(c) In eukaryotic cells, the end-replication problem explains why normal somatic
cells are limited in the number of mitotic divisions before they die out.

(i) Outline the end-replication problem in eukaryotic chromosomes.


.........[3]
1 DNA polymerase can only add nucleotides to the free 3 OH end
2 of a pre-existing/elongating polynucleotide (not DNA, since primer is RNA)
3 primer (bound to the very end of the template strand) removed
4 but cannot be replaced with DNA
5 because no 3-OH available (to which a DNA nucleotide can be added)
6 the 5 end of the DNA becomes shorter relative to that of the previous generation

SAJC / H2 Biology 9648/2 JC2 Prelims 2013


363
5
(ii) What are telomeres?
..........[1]
1 ends of the linear DNA molecules
2 of eukaryotic chromosomes
3 ref. short repeated sequences

(iii) State the role of telomeres in relation to the end-replication problem.


......[]
1 prevents genes from being lost as DNA shortens with each round of replication.

Examiners comments:
Candidates must answer the questions specifically for the role of telomeres in relation to the end-
replication problem. REJECT other roles of telomeres (i.e. protects the end of the chromosome from
degradation by nucleases; prevents end-joining of chromosome ends which may lead to chromosomal
mutations; prevents unintentional cell death).

Telomeres does not prevent DNA from shortening! It acts as a disposable buffer to protect the genes
from erosion as DNA shortens with each round of replication.

Telomeres Telomerase!

(iv) Suggest why bacteria do not have telomeres.


......[1]
1 bacteria have circular DNA
2 no degradation of chromosome ends at each replication of DNA
/ no end-replication problem

In bacteria cells, binary fission differs significantly from the process of mitosis seen in
eukaryotic cells.

(d) Explain the significance of binary fission in bacteria


......[2]
1 Ref. asexual reproduction for unicellular organism
2 Ensuring that offspring are genetically identical to the parent
3 Desirable alleles/traits are passed down

4 Rapid increase in cell numbers (under favourable conditions)


5 Important for survival of species

[Q2 Total: 13]

SAJC / H2 Biology 9648/2 JC2 Prelims 2013


364
6
3.
(a) Fig. 3.1 shows two different viruses.

Virus A Virus B
Fig. 3.1

(i) Relate the structure of virus A to its mechanism of release from the host.
...[2]
1 Haemagglutinin binds to sialic acid (receptor) on host cells
2 Neuraminidase removes/cleaves sialic acid on viral envelope
3 to prevent agglutination/clumping of virus
4 to allow release of virus from host cell
/ to prevent virus from attaching to host cell during release

(ii) Contrast the reproductive cycles of the Virus A and Virus B. [3]

Virus A Virus B
Enters host cell via endocytosis Enters host cell via membrane fusion
with plasma membrane of host cell

RNA genome is not reverse transcribed RNA genome is first reversed


/ negative-sense RNA is converted to transcribed by reverse transcriptase
positive-sense RNA into DNA

No integration of viral genome into DNA Double-stranded DNA is integrated as


of host cell provirus into DNA of host cell

No proteolysis; Proteolysis of viral polyprotein to form


mature viral proteins;

No latency phase Latency phase

Examiners comments:
REJECT structural differences e.g. reverse transcriptase vs no reverse transcriptase, enveloped virus
vs not enveloped virus;
REJECT metabolic machineries
Budding (which happens for both influenza virus and HIV) Exocytosis !!

SAJC / H2 Biology 9648/2 JC2 Prelims 2013


365
7
(b) One of the most common treatment methods for patients infected with virus B is
the use of reverse transcriptase inhibitors. These inhibitors bind to an inhibitor-
binding site (other than the active site) on the reverse transcriptase.

However, virus B often develop resistance against these inhibitors due to a single
base substitution in the DNA sequence resulting in a change of an amino acid in a
subunit of reverse transcriptase. The mutant protein is full-length and retains reverse
transcriptase activity, but does not allow the binding of the inhibitors.

(ii) Explain why the mutated protein remains full-length and retains reverse
transcriptase activity, but does not allow the binding of the inhibitors.
..[2]
Remain full-length
1 No frame-shift mutation
2 number of amino acids coded for remains the same

Does not allow binding of inhibitors


3 Tertiary structure/3D conformation of protein is altered
4 Inhibitor-binding site is no longer complementary to the 3D conformation of the
inhibitor (no interaction between the inhibitor and reverse transcriptase)

Retains reverse transcriptase activity


5 But active site is not altered

(c) In recent years, vaccines have been developed for virus B. Vaccines can either
be live but weakened forms of the virus, or its structural parts that are recognised by
the immune system. This provides immunity to the disease.

However, vaccine development for virus B has become more challenging due to the
viruss ability to undergo a high rate of antigenic drift.

(i) Suggest a reason for this high rate of antigenic drift and how it affects vaccine
development.
..[2]
1 Reverse transcriptase lacks proof-reading ability
2 Unable to remove mismatched deoxyribonucleotides and insert the correct one
(during DNA polymerisation)
3 Mutation in genes coding for protein/glycoprotein (gp120)
4 Produces altered (gp120) protein/glycoprotein
5 Antibodies induced by earlier strains of HIV / previously synthesised antibodies
can no longer bind to protein/glycoprotein
6 Hence, new vaccines must be developed

Examiners comments
Vaccines Antibodies Antibiotics!
Vaccines = A vaccine typically contains an agent that resembles a disease-causing
microorganism, and is often made from weakened or killed forms of the microbe, its toxins or
one of its surface proteins. The agent stimulates the body's immune system to recognize the
agent as foreign, destroy it, and "remember" it, so that the immune system can more easily
recognize and destroy any of these microorganisms that it later encounters.
Antibodies = Immunoglobulin proteins produced by the immune cells in response to foreign
antigen (eg. virus or vaccines). Antibodies will bind to the antigen.
Antibiotics = chemicals that are used to kill bacteria. They have no effect on viruses.

SAJC / H2 Biology 9648/2 JC2 Prelims 2013


366
8
(ii) A student tries to produce a vaccine by heating a human virus to 100C for five
minutes to inactivate its infectivity. Suggest why effective vaccines cannot be
produced using the virus that has been heated to 100C.
...[1]
1 heating denatures the viral glycoprotein/antigens/capsid/envelope proteins;
2 unable to elicit specific antibodies from the immune system that recognizes the
protein receptors/capsid from the viruses

Examiners comments
Function of vaccines is to stimulate the production of new antibodies by the immune system. Some
candidates mistakenly thought that vaccines will be targeted by existing antibodies.

[Q3 Total: 11]

SAJC / H2 Biology 9648/2 JC2 Prelims 2013


367
9
4. Familial Dysautonomia (FD), is an autosomal disorder which affects the
development of sensory neurons in the autonomic and sensory nervous system,
resulting in various symptoms including the insensitivity to pain.

Mutation in another gene, transmembrane channel 1 gene (TMC1), results in a form


of DFNB7 hearing loss which is also inherited in an autosomal manner.

A world-wide, long term genetic study on the possible associations between FD and
hearing loss is implemented in 2003. Table 4.1 above shows the genotypes and
phenotypes of some subjects in this study.

Subject
Gene name Genotype Phenotype
number
FD Heterozygous No FD
1031
TMC1 Heterozygous Normal hearing
FD Homozygous recessive FD
1050
TMC1 Homozygous dominant Normal hearing
FD Homozygous dominant No FD
1088
TMC1 Homozygous recessive Hearing loss
FD Homozygous recessive FD
1092
TMC1 Homozygous recessive Hearing loss

Table 4.1

(a) Define the following terms:

(i) genotype
...[1]
1 the genetic makeup of an organism;
2 the paired alleles that produces a phenotype;
3 can be either homozygous or heterozygous;

(ii) phenotype
...[1]
1 physical and chemical characteristic of an organism;
2 depend on genotype of the organism and the environment;

(b) Subject 1031 and another subject listed in Table 4.1 are married and they have
their first child in 2005. This cross between them theoretically resembles a test
cross.

(i) Deduce the subject number of subject 1031s wife from Table 4.1.
......[]
Subject 1092;

SAJC / H2 Biology 9648/2 JC2 Prelims 2013


368
10
Subject 1031 has a hearing impaired father with FD; his mother has normal hearing
and does not suffer from FD (she is homozygous dominant for both gene locus).

A total of 200 couples with the same genotypes as subjects 1031 and his wife were
monitored in this study. Information on their children (total number of 186) was
collated 10 years after the implementation of this study and presented in Table 4.2.

Phenotype Number of children


No FD, Normal hearing 59
FD, Normal hearing 35
No FD, Hearing loss 26
FD, Hearing loss 66

Table 4.2

(ii) Using the information give in Table 4.2, calculate the 2 and state the
conclusions derived from this test.
...[2]
1 23.42 (follow d.p. or sig.fig, whichever is applicable, of the 2-square table)
2 Since the calculated 2 value 23.42 > critical 2 value 7.82 at p = 0.05;
3 reject null hypothesis
4 The results of the 2 test suggest that there is a significant difference between the
observed and the expected values at the 5% level
5 Any difference is not due to chance
6 The predicted phenotype ratio of 1:1:1:1 is incorrect;

(O E) 2
=
2
v=c1
E
where = sum of O = observed value
v = degrees of freedom E = expected value
c = number of classes

A chi-squared table.
degrees of probability, p
freedom 0.10 0.05 0.02 0.01 0.001
1 2.71 3.84 5.41 6.64 10.83
2 4.61 5.99 7.82 9.21 13.82
3 6.25 7.82 9.84 11.35 16.27
4 7.78 9.49 11.67 13.28 18.47

(O E) 2
2 =
E
(59 46.5) 2 (35 46.5) 2 (26 46.5) 2 (66 46.5) 2
= + + +
46.5 46.5 46.5 46.5

Degrees of freedom =c1


=41=3

SAJC / H2 Biology 9648/2 JC2 Prelims 2013


369
11
(iii) Explain the discrepancy from the expected results.
...[3]
1 There is a larger number of parental phenotypes and smaller number of
recombinant phenotypes;
2 There is no independent assortment of genes;

3 There is partial linkage of FD and TMC1 genes;


4 the FD and TMC1 genes are located on the same chromosome;

5 Recombinants are formed when;


6 crossing over occasionally breaks the linkage between genes on the same
chromosome;
7 (Other scenario)If it is a normal dihybrid cross, the F2 generation results would
show 4 types of phenotypes with a phenotypic ratio of 1:1:1:1
8 (Other scenario)If it is tight gene linkage, the F2 generation results would show 2
types of phenotypes with a phenotypic ratio of 1:1

SAJC / H2 Biology 9648/2 JC2 Prelims 2013


370
12
(iv) Using the symbols provided, draw a genetic diagram of the cross between
Subject 1031 and his wife. [2]

Symbols:
F dominant allele for no FD
f recessive allele for FD
T dominant allele for Normal hearing
t recessive allele for hearing loss

Parental phenotypes: No FD, Normal x FD, Hearing loss


hearing
FT ft
Parental genotypes: ft ft

Parental gametes: FT ft Ft fT ft

Parental gametes recombinant gametes

Punnett square
showing F1 genotypes:

FT ft Ft fT

ft FT ft Ft fT
ft ft ft ft

Legend:

: No FD, normal hearing


: FD, hearing loss
: No FD, hearing loss
: FD, normal hearing

F1 No FD, normal hearing ; FD, hearing loss; No FD, hearing loss ; FD, Normal hearing
phenotypes:
Parental Recombinant

1 Parental genotypes ;
2 Parental gametes;
3 F1 genotypes;
4 F1 genotypes correspond to phenotypes / legend for Punnett square;
5 Correct presentation of genetic diagram (circled gametes, Indication of
recombinants and non-recombinant gametes, Indication of recombinants and
non-recombinant phenotypes);

SAJC / H2 Biology 9648/2 JC2 Prelims 2013


371
13
During gamete formation in Subject 1031s wife from a diploid germ cell, an error
occurred once. As a result, her children will have 50% chance of having 3 copies of
chromosome 9; and 50% of having only 1 copy of chromosome 9 in his/her cells.

(v) Explain how this could have happened.


...[2]
1 Non-disjunction
2 homologous chromosomes fail to separate (in anaphase I);
3 so two copies of chromosome 9 will be found in 2 out of 4 oocytes / eggs/
gametes;
4 remaining 2 oocytes/eggs/gametes will have no chromosome 9;
5 upon fertilisation by fusion with the male gamete; (there is 50% chance of having
3 copies of chromosome 9 and 50% chance of having none in the offspring)

Examiners comments:
Reject non-disjunction at anaphase II/separation of sister chromatids as an error that occurred once at
meiosis II will result in 50% chance of normal (2 copies of chromosome 9); 25% chance of getting 3
copies of chromosome 9; 25% chance of getting only 1 copy of chromosome 9 after fusion with sperm

[Q4 Total: 12]

SAJC / H2 Biology 9648/2 JC2 Prelims 2013


372
14
5.
(a) Fig
g. 5.1 showws the norm mal seque nce of cell signaling events whhen insulin
reaches its targett cell and binds
b to an
n insulin rec
ceptor.

Fig. 5.1

(i) Sta
ate the sou
urce of the insulin mo
olecule in Fig
F 5.1.

...

....[1]
1 -ceells of the islets of La
angerhans
2 of th
he pancrea as

(ii) With reference to the structure


s off the plasm
ma membra ane, descriibe how the
structural features of recepttor tyrosine
e kinase en nables it to
o be embeddded as a
transmembrane protein.
p
... ....[3]
1 hyddrophobic c region of protein connsist of am
mino acid with
w non-poolar R grou ups)
eracts with hydropho
2 inte obic core o of membra ane
3 consisting of fatty
f acid tail of phospholipid bilayer
b
4 hydrophilic reg
gion of pro
otein consisst of amino
o acids with polar/hyddrophilic R
grouups
5 inte
eracts with hydrophillic region oof membra ane
6 consisting of phosphate
p e heads off phospholipid bilayer

SAJC / H2 B
Biology 9648/2 JC2 Prelims 2013
373
15
(iii) With reference to Fig. 5.1, describe the events occurring in stages A to D.
.......[3]
1 insulin receptor exists as a dimer prior to the binding of insulin
2 insulin bind to insulin receptor
3 ref. complementary in shape to the ligand binding site of the insulin receptor
4 results in the activation of the tyrosine kinase domains in the cytoplasmic tail.
5 each receptor phosphorylates the tyrosine residues (at the cytoplasmic tails) of
the other receptor, hence activating it
6 auto-crossphosphorylation
7 receptor phosphorylates and activates IRS-1
8 which in turn phosphorylates and activates PI 3-kinase

Examiners comment
The question asks for a description of stages A, B C. Candidates must use the information given in the
diagram i.e. specifically referring to IRS-1, PI 3-kinase and phosphorylation rather than just proteins,
tyrosine kinases, relay proteins, activation.

(iv) Based on your knowledge, describe the events that lead to an increase in the
number of glucose transporters in the cell surface membrane.
.......[1]
1 movement of vesicles containing glucose transporters (GLUT4 transporters)
towards the cell surface membrane.
2 fusion of vesicle membrane with cell surface membrane results in incorporation
of glucose transporters on the cell membrane.

(v) A tissue may, over time, lose its responsiveness to insulin, even though insulin
concentration remains unchanged. Suggest possible reasons for this decrease in
responsiveness.
.......[1]
1 Mutation in the gene coding for insulin receptor / change in the shape of insulin
receptor as cell/tissue ages
2 Mutation in the gene coding for insulin receptor / change in the shape of Glut4
transporters as cell/tissue ages

3 Fewer insulin receptors are synthesised / ref. lower transcription rate of gene that
codes for insulin receptor
4 Fewer Glut4 transporters are synthesised/incorporated / ref. lower transcription
rate of gene that codes for Glut4 transporters

SAJC / H2 Biology 9648/2 JC2 Prelims 2013


374
16
(b) An aqueous solution contains small membranous vesicles which may be derived
either from a thylakoid or a mitochondrial membrane. In these vesicles, the
orientation of the ATP synthase enzymes is preserved as it is in the cell.

Fig. 5.2 shows the various experimental systems that are prepared in an attempt to
synthesize ATP.

Fig. 5.2

Which experimental system (A to D) shown in Fig. 5.2 would result in the successful
synthesis of ATP by thylakoid-derived vesicles? Explain your answer.
......[3]
1 Experimental system A
2 pH within the membrane is lower (pH 4 as compared to pH 8 outside the
membrane)
3 Concentration of H+ ions is higher within the membrane (than outside the
membrane)
4 ref. proton gradient
5 tendency for the H+ ions to diffuse out of the vesicle
6 via ATP synthase;
7 ATP is formed using ADP and Pi found outside the membrane

Examiners comments
pH 4 represents a higher concentration of H+. Some candidates made careless mistakes when relating
pH and H+ concentrations.
Some candidates merely describe the chloroplast without reference to the diagram. This will earn
them minimum marks.
Only ATP synthase can be seen in the diagram; no ETC or stalked particles are drawn.

[Q5 Total: 12]

SAJC / H2 Biology 9648/2 JC2 Prelims 2013


375
17
6.
(a) During an action potential, the permeability of the cell-surface membrane of an
axon changes. Fig 6.1 shows changes in permeability of the membrane to sodium
ions (Na+) and to potassium ions (K+) during a single action potential.

Fig. 6.1

(i) Explain the shape of the curve for sodium ions between 0.5 ms and 0.7 ms.
...[2]
1 Between 0.5 ms and 0.7 ms, permeability to Na+ ions increases from 0 a.u. to 28
a.u. (acceptable range 23 29 a.u.)
2 Some voltage-gated Na+ ion channels open
3 Na+ ions diffuse into the neurone
4 Changes membrane potential/makes inside of axon less
negative/positive/depolarisation/ reaches threshold
5 All voltage-gated Na+ ion channels open
6 More Na+ ions diffuse into the neurone

Examiners comments
The type of ion channels (eg. ungated, voltage-gated, ligand-gated) must be clearly stated.
In this question about action potential occurring in the axon, the ligand-gated ion channels are NOT
applicable.

(ii) During an action potential, the membrane potential rises to +40mV and then falls.
Using information from the graph, explain the fall in membrane potential.
...[2]
1 Repolarisation occurs;

2 Voltage-gated Na+ ion channels (in the membrane) are closed;


3 decreasing/stopping diffusion of Na+ ions into neurone;
4 ref. 0.7 ms to 1.5 ms, permeability to Na+ ions decreases from 28 a.u. to 0 a.u.
(acceptable range: 27-29 a.u.)

5 voltage-gated K+ ion channels (in the membrane) open;


6 increasing diffusion of K+ ions out of neurone;
7 ref. 0.5 ms to 1.2 ms, permeability to K+ ions increases from 0 a.u. to 12 a.u.
(acceptable range: 11-13 a.u.)

SAJC / H2 Biology 9648/2 JC2 Prelims 2013


376
18
(iii) After exercise, some ATP is used to re-establish the resting potential in axons.
Explain how the resting potential is re-established.
...[1]
1 Na+- K+ pump on membrane
2 Pumps 3Na+ out and 2K+ in neurone (by active transport)
3 restores the unequal distribution/concentration gradient of Na+ ions and K+ ions
(in the intracellular and extracellular fluids at resting potential)

(b) The black mamba is a poisonous snake which produces a toxin.

(i) The length of the section of DNA that codes for the complete toxin is longer than
the mRNA used for translation. Explain.
......[]
1 DNA contains introns/non- coding bases
/ mRNA only contains exons/coding bases

(ii) The black mambas toxin kills prey by inhibiting the enzyme acetylcholinesterase
at neuromuscular junctions that controls muscles involved in breathing. Explain how
this inhibition affects breathing.
...[2]
1 Acetylcholine not broken down
/ stays bound to receptor
2 Na+ ions continue to enter
/ continued depolarisation
/ Na+ channels continuously open
3 action potentials generated/impulses fired continuously;
4 Muscles involved in breathing remain contracted/cannot relax;

Examiners comments:
Muscles contract alone is not sufficient; muscles remain contracted.
Inability to breath is not due to lack of action potential generation. Students should read the question
carefully to note that acetylcholinesterase is inhibited.

SAJC / H2 Biology 9648/2 JC2 Prelims 2013


377
19
(c) Do opamine iss a neurotraansmitter rreleased by dopamin nergic neurrons for the
e
transmission of nerve signa als importa nt for moto
or control. Dopaminee binds to and
a
activate
es its recep
ptors locatted on the post-synapptic membrane.

Fig 6.2 shows a crystallogra


c aphy of the
e dopamine
e receptor.

Fig 6.2

(i) Witth referencce of Fig. 6.2,


6 identifyy the type of
o receptorr which thee dopamine
e
recepto
or belongs to.
...[]
...
G-prote
ein coupled
d receptor

SAJC / H2 B
Biology 9648/2 JC2 Prelims 2013
378
20
Excesssive neurottransmissio
on of dopaamine is asssociated with
w schizoophrenia, a clinical
mental disorder. To
T relieve this disord
der, antipsy ugs are devveloped to bind to
ychotic dru
and ina
activate the
e post-syna
aptic dopa mine receptors.

Fig 6.3 shows the


e cell signa
alling pathw
way when dopamine binds to itts receptorrs.

Fig 6.3

(ii) With reference to Fig 6.3,


6 briefly describe how h binding
g of the druugs to the
recepto
or affects downstream
d m signallin g pathway y.
...[3]
1 Prevents confformation change
c in rreceptor (h
hence no activation
a oof receptor)
2 Gp protein unable to bindd to receptoor
3 GDP not displlaced by GTP G
4 Gp mains inactive / -sub unit of G protein
protein rem p doe
es not dissoociate
5 and d bind to / activate
a ad
denylyl cyc lase
6 No conversion n of ATP to
o cAMP.
7 No activation of protein kinase C
8 No opening off Na+ ion channels
c

otal: 11]
[Q6 To

SAJC / H2 B
Biology 9648/2 JC2 Prelims 2013
379
21
7. Fishes in the genus Anisotremus comprise of ten described species which occur
predominantly on coral reefs and subtropical rocky reefs in the Neotropics.

(a) Bernardi et.al. did a molecular phylogenetic study on such fishes in that area.
Results are shown in Fig. 7.1.

Fig. 7.1

(i) Explain how molecular methods can be used to elucidate the evolutionary
relationships of the different species of Anisotremus fishes.
..............[2]
1 Neutral mutations do not confer any selective advantage or disadvantage;
2 Neutral mutations are accumulated at a relatively constant rate for any particular
gene and use as a molecular clock;
3 compare DNA sequence of a particular common gene (between different species
of fish);
/ comparison/alignment of homologous genetic sequences

SAJC / H2 Biology 9648/2 JC2 Prelims 2013


380
22
4 E.g. rRNA genes, cytochrome c genes, hypervariable regions of mtDNA
5 number of mutation in genetic sequence is used to calculate the length of time
since divergence
/ % sequence homology indicates degree of evolutionary closeness

(ii) Explain why it may be more reliable to construct a phylogenetic tree of the ten
species of Anisotremus using molecular data instead of morphological comparisons.
...[1]
1 Quantifiable ;
protein, nucleic acid sequence data are precise and accurate and easy to
quantify / convertible to numerical form for mathematical and statistical analysis;

OR

2 Objective;
based strictly on heritable material
/ can be easily described in an unambiguous manner
/ some morphological characteristics may be analogous / ref. convergent
evolution;

(iii) With reference to Fig. 7.1 and molecular homology, comment on the
evolutionary relationship between A. taeniatus and A. virginicus.
..............[1]
1 A. taeniatus and A. virginicus are closely related;
2 share a (recent) common ancestor;
3 high percentage homology in DNA sequence alignment;

SAJC / H2 Biology 9648/2 JC2 Prelims 2013


381
23
(b) A. taeniatus is found in
n the Pacifiic Ocean, whereas
w A.
A virginicuss is found in the
Caribbe
ean Sea. These
T two species wwere derived due to th
he formatioon of the Is
sthmus
of Pana
ama aboutt 3.5 million
n years ag o. Before that
t event, the waters
rs of the Pa
acific
Ocean and Caribbean Sea mixed free ely.

(i) Expplain how the


t formation of the IIsthmus off Panama results
r in thhe emerge
ence of
A. taen
niatus and A.
A virginicu
us.
.. .[4]
1 geo ographical isolation
/Istthmus of Panama
P is a physical barrier
/ re
ef. allopatric speciatio
on;
2 disruption to gene flow in the ance estral popu
ulation
/ noo interbree
eding betwe een the org
ganisms inn the Pacific Ocean aand Caribb
bean
Sea a;
3 proocesses of natural se election and
d genetic drift
d occur;

4 gen netic variattions exist within eacch sub-pop pulation;


5 diffferent selection press sures in Pa acific Ocea an and Carribbean Seea;
6 list 1 eg. food
d availabilitty/ salinity /temperatu ure/ different predatoors;
7 individuals wiith a selecttive advan tage in the e particularr environm ent survive ed till
repproductive age and pass on the o offspring;
eir alleles to
8 cha ange in alle
ele frequen ncy of gene e pool;
9 acccumulation n of genetic c difference es over timme;
10 diffferent popuulations ulttimately ca annot interb breed to prroduce via ble, fertile
offsspring;

(c) Exxplain why it is impos


ssible for evvolution to
o occur at the individuual level.
.. .[2]
1 Evo
olution = ch
hanges in allele
a frequ
uencies in a gene pool of a poppulation over time;

2 The ere must bee variation (in a popuulation) beffore selection can takke place;
3 Individuals aree selected for or aga inst by nattural selecttion;
4 Individuals can only pas ss alleles too the next generationn;
5 It is the popula
ations thatt actually e
evolve

[Q7 To
otal: 10]

SAJC / H2 B
Biology 9648/2 JC2 Prelims 2013
382
24
Section B

Answer one question.

Your answers should be illustrated by large, clearly labelled diagrams, where


appropriate.

Your answers must be in continuous prose, where appropriate.

Your answers must be set out in sections (a), (b) etc., as indicated in the question.

8 (a) Distinguish between the concepts of repressible and inducible systems


of gene regulation. [4]
(b) Outline the differences between prokaryotic control of gene expression
with the eukaryotic model. [10]
(c) Describe how gene amplification can occur and the significance of gene
amplification in development [6]

[Total: 20]

OR

9 (a) Describe the molecular structure of starch (amylose) and cellulose and
relate these structures to their functions in living organisms. [7]
(b) Explain, with two named examples, how the environment may affect the
phenotype. [6]
(c) Explain how biogeography and the fossil record support the
evolutionary deductions based on homologies. [7]

[Total: 20]

SAJC / H2 Biology 9648/2 JC2 Prelims 2013


383
25
8(a) Distinguish between the concepts of repressible and inducible systems
of gene regulation. [4]

Repressible system Inducible system


1 Repressible enzymes are produced Inducible enzymes are produced

2 Involved in anabolic (biosynthesis) Involved in catabolic (hydrolysis)


pathways pathways

3 Repressors are synthesized in Repressors are synthesized in active


inactive form form

4 Transcription of structural genes is Transcription of structural genes is


normally switched on normally switched off

5 Pathway end-product (tryptophan) Substrate molecule serves as inducer


serves as co-repressor that signals (allolactose) that signals the induction of
repression of the trp operon the lac operon

6 Operon is repressed to prevent over Operon is induced to stimulate


production of pathway product production of enzymes necessary for
breakdown substrate

8(b) Outline the differences between prokaryotic control of gene expression


with the eukaryotic model. [10]

For every comparison:


mark for correct comparison
mark for correct information

Chromosomal Level (max 2)


Prokaryotes Eukaryotes
1. Prokaryotic DNA not organised into Eukaryotic DNA is complexed with
chromatin / not associated with histones and other proteins to form
histones chromatin / associated with histones

2. DNA demethylation/methylation and DNA and histone modification can occur,


histone acetylation/deacetylation resulting in conversion between
cannot occur euchromatin and heterochromatin

3. DNA sequences, eg. promoters Structure of chromatin euchromatin,


and operators, serve as the on/off ready to be transcribed, or
switch heterochromatin and not available is
the major on/off switch for gene
regulation

/ Ease of transcription DNA made


accessible to RNA polymerase and other
regulatory proteins

SAJC / H2 Biology 9648/2 JC2 Prelims 2013


384
26
Transcriptional Level (max 2)
Prokaryotes Eukaryotes
4. One RNA polymerase consisting of Three different RNA polymerases, each
five subunits containing 10 or more subunits;
/ All RNAs synthesized by the same / Three different classes of RNA each
RNA polymerase; synthesized by a different RNA
polymerase (i.e. mRNA, tRNA, rRNA)
5. Simple regulatory sequence: Complex regulatory sequence:
Transcriptional regulatory protein / More extensive interaction between
Regulator protein binds to DNA- upstream DNA sequences and protein
binding sites upstream of the cluster factors involved to stimulate and initiate
of structural genes to regulate transcription. In addition to promoters,
initiation of transcription. enhancers and silencers control rate of
transcription.

6. Related genes are transcribed No operon


together as operons / each gene has own promoter
/ only 1 promoter / monocistronic mRNA
/ polycistronic mRNA

Post-transcriptional Level (max 2)


Prokaryotes Eukaryotes
7. Translation is often coupled to No direct coupling of transcription and
transcription translation
/ Transcription and translation take / mRNA must pass across nuclear
place in the same cellular envelope before translation in the
compartment simultaneously. cytoplasm. RNA transcript is not free to
associate with ribosomes prior to the
completion of transcription.

8. Primary transcripts are the actual Primary transcripts undergo processing to


mRNAs produce mature mRNAs methylated
/ no post-transcriptional modification guanosine cap at the 5 end, poly-A tail
at the 3 end, splicing

9. Lower stability of transcript Higher stability of transcript


/ degradation within seconds or / prevent transcript degradation
minutes / mRNAs longer half-life remaining much
/ mRNAs shorter half life to rapidly longer to orchestrate protein synthesis
respond to environmental changes prior to their degradation by nucleases in
the cell

SAJC / H2 Biology 9648/2 JC2 Prelims 2013


385
27
Translational level (max 2)
Prokaryotes Eukaryotes
10. Control at this level is unlikely; due Control at translational level:
to simultaneous transcription and phosphorylation of ribosomal translation
translation initiation factors
/ negative translational control through
regulatory proteins
/ cytoplasmic elongation of poly (A) tails
/ mRNA degradation
/ RNA interference and microRNA

11. mRNAs have multiple ribosome mRNAs have only one start site
binding sites / direct synthesis of only one kind of
/ direct the synthesis of several polypeptide
different polypeptides

Post-translational Level (max 2)


Prokaryotes Eukaryotes
12. no/minimal post-translational Post-translational modifications
modifications occur determine the functional abilities of the
protein
13. Proteolysis: Processing eukaryotic
polypeptides to yield functional protein
molecules e.g. cleavage of pro-insulin to
form the active insulin hormone
14. Chemical modification of proteins to yield
functional protein molecules
15. Phosphorylation of proteins to increase
or decrease its function
16. Transportation of proteins to target
destinations in the cell where it functions
is mediated by signal sequences at N-
terminus of some proteins. Once
transported to destination, signal
sequence is enzymatically removed from
the proteins
17. Ubiquitination marks protein for
degradation, ref to ubiquitin &
proteasome

SAJC / H2 Biology 9648/2 JC2 Prelims 2013


386
28
8(c) Describe how gene amplification can occur and the significance of gene
amplification in development . [6]

1 gene amplification is the the (selective) replication of certain genes


2 increases the number of templates for transcriptions
/ results in increased gene expression
/ allows more copies of mRNA to be made
3 increase amount of gene product / polypeptide in a short period of time
4 to carry out specialised functions in the cell

How gene amplification can occur

Faulty cytokinesis
5 faulty cytokinesis / non-disjunction
6 result in one or more extra sets of chromosomes ending up in a single cell
/ ref. polyploidy
/ ref. duplication of chromosomes sets

Errors in crossing over


7 errors in crossing over / unequal crossing over
8 of non-sister chromatids during prophase I of meiosis
9 even when their homologous gene sequences are not correctly aligned

Slippage during DNA replication


10 slippage occur during DNA replication
11 such that the template shifts with respect to the new complementary strand
12 one region of the template strand is copied twice

Significance in development
13 In developing ovum/egg cell, additional copies of the rRNA genes
14 for making enormous number of ribosomes
15 for active protein synthesis (once the egg is fertilized)

SAJC / H2 Biology 9648/2 JC2 Prelims 2013


387
29
9(a) Describe the molecular structure of starch (amylose) and cellulose and
relate these structures to their functions in living organisms. [7]

Starch

Structure
1 amylose is unbranched;
2 polymer of -glucose molecules linked by (1, 4) glycosidic bonds

3 hydrophilic hydroxyl groups of glucose residues project into interior of helices;


4 no cross-linking between amylose or glycogen chains
/ no cross-linking to form big bundles

Property and Function


5 insoluble;
6 can be stored in large quantities without having any great effect on the water
potential of cells
7 and can be prevented from diffusing out of cells

8 individual chains can fold into compact shape;


9 can be stored in large quantities (within a cell);

10 easily hydrolysed to monosaccharides when required;

Cellulose

Structure
11 unbranched;
12 polymer of -glucose molecules linked by (1, 4) glycosidic bonds;

13 alternative units are oriented 180 to each other


/ every other glucose monomer is upside down with respect to the others
14 hydroxyl groups project outwards from each cellulose chain;
15 form H-bonds with neighboring chains
/ cross-linking of chains to form macrofibrils;

Property and Function


16 great tensile strength
17 prevents plant cells from bursting when placed in solutions of higher water
potential
/ used as cell wall material for structural support in plants;

18 large intermolecular spaces between macrofibrils


19 allows passage of water and solute molecules;

SAJC / H2 Biology 9648/2 JC2 Prelims 2013


388
30
9(b) Explain, with two named examples, how the environment may affect the
phenotype. [6]

Example 1 [max 3]
1 Himalayan rabbit homozygous for the ch allele;
2 tyrosinase gene codes for heat-sensitive enzyme/tyrosinase that makes
melanin;

3 low temp /below 33C tyrosinase active / melanin produced;


4 eg. in parts of the body that are cool enough / the extremities / feet, nose, ears
and tail black;

5 high temp /above 33C tyrosinase inactive / no melanin produced ;


6 eg. in body regions that are massive enough to conserve a fair amount of heat
white;

7 if a small section of fur is shaved from a Himalayan rabbit, fur will grow back
either white or black depending on temp. of environment / Eg. experiment
involving ice-pack on shaved rabbit ;
8 just-born rabbits are all white as they had developed at temp. of mothers womb;
9 to produce rabbits with black extremities, the young are kept in the cold;

Example 2 [max 3]
1 In Drosophila homozygous for allele coding for vestigial wing;
2 allele for vestigial wing is recessive to that for long wing;
3 expression of vestigial wing is affected by the temperature at which the insect
develops;
4 The allele for vestigial wing is expressed only at low temperatures;
5 develop vestigial wings at 21C;
6 intermediate wings at 26C;
7 long wings at 31C;

Example 3 [max 3]
1 honeybee drones, queen and workers;
2 drones are males arise from unfertilised eggs;
3 queen and workers are females arise from fertilised eggs;

4 queen and workers have same amount of genetic material but phenotypically
different;
5 depends on diet of larva;

6 in the first 3 days of hatching, all female larvae feed on royal jelly;
7 after 3 days, those fed with diet consisting of honey/nectar and pollen would
develop into workers;
8 workers are sterile, smaller in size and have larger mouthparts and modified legs
as compared to the queen;
9 those fed with royal jelly develops into queens as high protein content of royal
jelly stimulates formation and maturation of female reproductive system;

SAJC / H2 Biology 9648/2 JC2 Prelims 2013


389
31
Example 4 [max 3]
1 people with functional pancreas/with no type I diabetes have functional genes for
insulin uptake;
2 insulin is secreted when blood glucose level increases;
3 overeating of sugary foods for a long period of time causes repeated stimulation
of the pancreas;
4 which responds by secreting high levels of insulin;
5 repeated exposure of target cells to large amounts of insulin desensitizes the cells
responsiveness to insulin;
6 result in the target cells failing to take in glucose;
7 resulting in type II diabetes;

SAJC / H2 Biology 9648/2 JC2 Prelims 2013


390
32
9(c) Explain how biogeography and the fossil record support the evolutionary
deductions based on homologies. [7]

Biogeography [max 3]
1 the study of the geographic distribution of species;
2 species that are closely related are found closer together;
3 and not found in similar environments far away due to geographical
barriers/convergent evolution;
4 e.g. Marsupials are not found anywhere else in the world except in Australia;

5 organisms found close together are evolved from a common ancestor;


6 e.g. Island Biogeography of the Galapagos: Most species of plants and animals
living on islands are closely related to neighbouring mainland species;

7 differences suggest evolution from mainland ancestral /descent with modification;


8 different species of finches on Galapagos Islands closely resemble one another;
9 they exhibit homology in terms of shape and size of beak;
10 due to natural selection: Favoured populations with beaks that enabled the birds
to acquire their particular food source at each island;

Fossil Records [max 4]


11 Fossils are the preserved remains of organisms.
12 allow scientists to study/compare morphological traits / homologous structures
with existing species;
13 coupled with dating of fossils based on radioactive isotopes to determine age of
fossil;
14 show that organisms evolved in a chronological manner / historical sequence
(fish amphibians reptiles birds and mammals);
15 show new species appearing and others disappearing;
16 reveal ancestral characteristics that have been lost over time;

17 discovery of transitional fossils linking older organisms to modern species


supports the idea of descent with modification.

18 provide evidence that accumulated changes over long period of time led to
diverse forms of life present today/diversity of life arose from evolution;

19 DNA/protein could be extracted from fossils and sequences compared;


20 higher molecular homology indicates closer relation;
21 can be used to prove that continental drift (land pieces which have drifted apart)
leads to divergent evolution;

SAJC / H2 Biology 9648/2 JC2 Prelims 2013


391

Civics Index Name (use BLOCK LETTERS)


Group Number H2
ST ANDREWS JUNIOR COLLEGE
2013 JC2 Preliminary Examinations

H2 BIOLOGY 9648/3

Paper 3: Applications and Planning Question

Wednesday 18 September 2013 2 hours

Additional Materials: Answer Paper


Cover Sheet for Section B

READ THESE INSTRUCTIONS FIRST

Write your civics group, index number and name on all the work you
hand in.
Write in dark blue or black pen on both sides of the paper.
You may use a soft pencil for any diagram, graph or rough working.
Do not use staples, paper clips, highlighters, glue or correction fluid.

Answer all questions.

At the end of the examination,


1. Attach Question 5 to the cover sheet provided.
For Examiners Use
The number of marks is given in brackets [ ] at the end of each
Paper 3
question or part question.
1
/14
2
/14
3
/12
5
/20
Total
/60

4
(Planning) /12
This document consists of 15 printed pages.
[Turn over

SAJC / H2 Biology 9648/3 JC2 Prelim 2013


392
2
Answer all questions.

1. A student researcher attempted to clone the Human Growth Hormone (HGH)


gene. Two separate samples of HGH DNA were obtained from DNA libraries and
subjected to agarose gel electrophoresis. Results were shown in Fig.1.1.

Sample 1 Sample 2

- well

+
Fig. 1.1

(a)(i) Describe how the DNA fragments were separated into visible bands.
.......
.......
.......
......[2]

Subsequent cloning procedures involving Escherichia coli host cells yield functional
HGH protein when Sample 2 was used, but not for Sample 1.

(ii) Based on the difference in the bands observed in Fig. 1.1, account for the failure
to produce functional HGH protein for Sample 1.
.......
.......
.......
......[2]

(iii) State the type of DNA library where Sample 2 was obtained from.
......[]

SAJC / H2 Biology 9648/3 JC2 Prelim 2013


393
3
The student researcher has designed the following plasmid (Fig. 1.2) for use in her
preliminary trials of a cloning experiment.

Restriction site for BamHI

Kanamycin resistant
(kanr) gene

Tetracycline
resistant (tetr) gene

Fig. 1.2

(b) With reference to the properties of bacterial plasmids, provide 2 reasons why this
plasmid in Fig 1.2 will not work well as a cloning vector.
.......
.......
.......
..........[2]

(c) The student researcher has since made improvements to the plasmid in Fig. 1.2
such that it is now a functional cloning vector.

Steps were then taken to create a recombinant plasmid using BamHI and the
improved plasmid.

Restriction enzyme Specific recognition site


5'-G^G A T C C-3'
BamHI
3'-C C T A G^G-5'

^ indicates where the restriction enzyme cuts

(i) Describe the natural function of BamHI restriction enzyme.


.......
...[1]

SAJC / H2 Biology 9648/3 JC2 Prelim 2013


394
4
(ii) Explain the main problem with using Sample 2 and BamHI in the formation of a
recombinant plasmid and suggest how this problem can be solved.
.......
.......
.......[1]

(iii) Describe the subsequent steps in the formation of recombinant plasmid.


.......
.......
.......
.......[2]

(iv) With reference to Fig. 1.2, explain how you would identify transformed bacterial
cells that have taken up the recombinant plasmid with the HGH gene using antibiotic
selection.
.......
.......
.......
.......
.......
.......
.......[3]

[Q1 Total: 14]

SAJC / H2 Biology 9648/3 JC2 Prelim 2013


395
5
2. Severe combined immunodeficiency (SCID) is a genetic disorder characterized
by lack of functional lymphocytes, resulting in defects in both T and B cells responses
of the immune system. Patients are susceptible to opportunistic infections while
children usually die if not treated.

Fig. 2.1 shows the inheritance of SCID in a family in which both parents are carriers.
Only one of the sons suffer from SCID.

Fig. 2.1

(a) State the form of SCID that is being inherited in the family and the mode of
inheritance.
.......
...[1]

(b) Two RFLP morphs have been found tightly linked to the adenosine deaminase
(ADA) gene which can be used for genetic screening. A radioactive probe has been
designed to reveal the two RFLP morphs.

Both RFLP morphs and radioactive probe are shown in Fig. 2.1 with the arrows
representing the PvuII restriction sites. The asterisk (*) indicates the position of a
mutation at the restriction site that results in the loss of the restriction site. Morph 2
was found to be linked to the mutant ADA gene.

Fig. 2.2

SAJC / H2 Biology 9648/3 JC2 Prelim 2013


396
6
(i) With reference to Fig 2.1 and Fig. 2.2, draw the resulting autoradiogram you
would expect to see in the space below. [3]

DNA
ladder Father Daughter 1 Son 2

200 bp

100 bp

20 bp
5 bp

(ii) Explain the role of the DNA ladder.


.......
...[1]

(c) Apart from disease detection, other applications of RFLP include genetic
fingerprinting and in genomic mapping.

(i) Explain why genes especially protein-encoding genes are seldom used as
markers for genetic fingerprinting.
.......
.......
.......
......[2]

(ii) Explain the use of genetic markers in genomic mapping in terms of linkage
mapping.
.......
.......
.......
.......
.......
.......
......[3]

SAJC / H2 Biology 9648/3 JC2 Prelim 2013


397
7
(d) Another method to detect ADA-SCID is through the use of PCR. Fig 2.3 shows
the number of DNA molecules made using PCR, starting with one molecule.

Fig 2.3

(i) Explain three advantages of PCR.


.......
.......
.......
.......
.......
.......
.......[3]

(ii) Suggest two reasons for the leveling off of the graph from cycles 17 to 20.
.......
...[1]

[Q2 Total: 14]

SAJC / H2 Biology 9648/3 JC2 Prelim 2013


398
8
3.
(a) Many varieties of rice, Oryza sativa, die quickly when totally submerged by flood
water during the monsoon season. Typically, submergence-intolerant plants respond
to submerged conditions by elongating rapidly.

In 2006, a gene, Sub1A, on rice chromosome 9 was found to be involved in tolerance


of prolonged submergence.

An allele, Sub1A-1, was found only in submergence-tolerant rice. Varieties that are
submergence-intolerant either have a second allele, Sub1A-2, or lack the gene
altogether.

(i) Sub1A-1 codes for a protein that controls the transcription of the gene for alcohol
dehydrogenase. This gene is activated when the plant is in anaerobic conditions.

Explain the importance to submerged rice plants of producing alcohol dehydrogenase.


.......
.......
.......
..........[2]

A variety of submergence-intolerant rice, which lacked the gene Sub1A, was


genetically engineered to express Sub1A-1.

The mean heights of three types of rice plants were compared before and after 10
days of submergence in water. The types of rice plants were:

I : submergence-intolerant plants
I+ : submergence-intolerant plants genetically engineered to express Sub1A-1
T : submergence-tolerant plants

The results are shown in Fig. 3.1.

Fig. 3.1

SAJC / H2 Biology 9648/3 JC2 Prelim 2013


399
9
(ii) With reference to Figure 3.1, compare the effect of submergence on the
elongation of the three types of rice plants.
.......
.......
.......
.......
.......
.......[2]

The allele Sub1A-1 could be introduced into high-yielding variety of submergence-


intolerant rice either by genetic engineering or by selective breeding.

(iii) Suggest two advantages of using genetic engineering, rather than selective
breeding, to introduce the allele.
.......
..........[1]

(b) Round-Up ReadyTM soybeans are genetically modified to be resistant towards


Round-UpTM herbicide, a glyphosate-based herbicide that normally inhibits an
important enzyme in the aromatic amino acid synthesis pathway in plants. This allows
farmers to freely apply Round-UpTM to destroy weeds without affecting the Round-Up
ReadyTM soybeans.

Round-Up ReadyTM soybeans may be produced by first transfecting soybean callus


cells with the resistance gene from Agrobacterium.

(i) To obtain a callus culture, suitable explants must first be chosen. Suggest a
suitable explant and explain your choice.
.......
.......[1]

(ii) Explain three advantages of growing soyabean plants from tissue culture rather
than from seeds.
.......
.......
.......
.......
.......
.......
.......[3]

SAJC / H2 Biology 9648/3 JC2 Prelim 2013


400
10
(iii) Soyabean plants can reproduce sexually by means of seeds. Soyabean plants
grown from seeds are very variable in their yield. Explain why.
.......
.......
.......
.......[2]

Genetically modifying soybean plants to be resistant towards Round-UpTM herbicide


might threaten the environment and human safety.

(iv) State one possible adverse effect on:

The environment
.......
.........[]

Human safety
.......
.........[]

[Q3 Total: 12]

SAJC / H2 Biology 9648/3 JC2 Prelim 2013


401
11
4. Planning question

The goshawk is an endangered species protected by the Convention of International


Trade in Endangered Species of Wild Fauna and Flora (CITES).

The Royal Society for the Protection of Birds (RSPB) has found a man in possession
of four young goshawks and an adult female. He claimed to have bred the young
birds using his adult female and a male bird borrowed from another keeper of these
birds of prey. The RSPB suspects that the young goshawks were poached from the
wild and not bred in captivity as claimed by the man.

Plan an investigation using DNA fingerprinting to verify if the young goshawks were
bred in captivity or poached from the wild.

[Total: 12]

You planning must be based on the assumption that you have been provided with the
following equipment and materials

tissue samples (feather tips) from the four young gohawks, the adult female
and the adult male under investigation
pestle and mortar
DNA extraction buffer solution
ice-cold ethanol
glass rods
microcentrifuge tubes
centrifuge
restriction enzyme
PCR reagents and equipment
agarose gel electrophoresis reagents and equipment
suitable source of electrical current
radioactive probe
nitrocellulose membrane
autoradiography equipment

Your plan should have a clear and helpful structure to include

an explanation of the theory to support your practical procedure

SAJC / H2 Biology 9648/3 JC2 Prelim 2013


402
12
a description of the method used, including the scientific reasoning behind the
method
the type of data generated by the experiment
how the results will be analysed including how the origin of the organism can
be determined.
relevant risks and precautions taken

.......
.......
.......
.......
.......
.......
.......
.......
.......
.......

SAJC / H2 Biology 9648/3 JC2 Prelim 2013


403
13

.......
.......
.......
.......
.......
.......
.......
.......
.......
.......
.......
.......
.......
.......
.......
.......
.......
.......
.......
.......
.......
.......
.......
.......
.......
.......
.......
.......
.......
.......
.......
.......
.......
.......
.......
.......
SAJC / H2 Biology 9648/3 JC2 Prelim 2013
404
14

.......
.......
.......
.......
.......
.......
.......
.......
.......
.......
.......
.......
.......
.......
.......
.......
.......
.......
.......
.......
.......
.......
.......
.......
.......
.......
.......
.......
.......
.......
.......
.......
.......
.......
.......
.......
SAJC / H2 Biology 9648/3 JC2 Prelim 2013
405
15
Free-response question

Write your answer to this question on the separate answer paper provided.

Your answer:

should be illustrated by large, clearly labeled diagrams, where appropriate;


must be in continuous prose, where appropriate;
must be set out in sections (a), (b) etc., as indicated in the question.

5 (a) Compare the properties, features and functions of zygotic and


embryonic stem cells. [5]

(b) Using one named example of an adult stem cell, describe its normal
role and how it could be used in gene therapy against SCID using a
retrovirus. [8]

(c) Discuss the social and ethical considerations for the use of gene
therapy. [7]

[Total: 20]

SAJC / H2 Biology 9648/3 JC2 Prelim 2013


406

Civics Index Name (use BLOCK LETTERS)


Group Number H2
ST ANDREWS JUNIOR COLLEGE
2013 JC2 Preliminary Examinations

H2 BIOLOGY 9648/3

Paper 3: Applications and Planning Question


(Mark Scheme)
Wednesday 18 September 2013 2 hours

Additional Materials: Answer Paper


Cover Sheet for Section B

READ THESE INSTRUCTIONS FIRST

Write your civics group, index number and name on all the work you
hand in.
Write in dark blue or black pen on both sides of the paper.
You may use a soft pencil for any diagram, graph or rough working.
Do not use staples, paper clips, highlighters, glue or correction fluid.

Answer all questions.

At the end of the examination,


1. Attach Question 5 to the cover sheet provided.
For Examiners Use
The number of marks is given in brackets [ ] at the end of each
Paper 3
question or part question.
1
/14
2
/14
3
/12
5
/20
Total
/60

4
(Planning) /12
This document consists of 15 printed pages.
[Turn over

SAJC / H2 Biology 9648/3 JC2 Prelim 2013


407
2
Answer all questions.

1. A student researcher attempted to clone the Human Growth Hormone (HGH)


gene. Two separate samples of HGH DNA were obtained from DNA libraries and
subjected to agarose gel electrophoresis. Results were shown in Fig.1.1.

Sample 1 Sample 2

- well

+
Fig. 1.1
(a)(i) Describe how the DNA fragments were separated into visible bands.
......[2]
1 negatively-charged DNA fragments migrate to the positively-charged electrode;
2 through an agarose gel/matrix;
3 DNA fragments separated by size / molecular weights
/ shorter fragments move faster than longer ones;
4 invisible DNA bands revealed under UV light by staining the gel with a dye such
as ethidium bromide;

Examiners comments:
Many candidates failed to read the question properly and missed out the EtBr staining followed by
visualization under UV light in order to obtain visible bands

Subsequent cloning procedures involving Escherichia coli host cells yield functional
HGH protein when Sample 2 was used, but not for Sample 1.

(ii) Based on the difference in the bands observed in Fig. 1.1, account for the failure
to produce functional HGH protein for Sample 1.
......[2]
1 sample 1 has a higher molecular weight/larger size compared to sample 2;
2 due to presence of introns in sample 1;
3 bacterial host cells do not have RNA splicing machinery;
4 ref. HGH gene not expressed correctly;

(iii) State the type of DNA library where Sample 2 was obtained from.
......[]
1 cDNA library;

SAJC / H2 Biology 9648/3 JC2 Prelim 2013


408
3
The student researcher has designed the following plasmid (Fig. 1.2) for use in her
preliminary trials of a cloning experiment.

Restriction site for BamHI

Kanamycin resistant
(kanr) gene

Tetracycline
resistant (tetr) gene

Fig. 1.2

(b) With reference to the properties of bacterial plasmids, provide 2 reasons why this
plasmid in Fig 1.2 will not work well as a cloning vector.
..........[2]
1 no origin of DNA replication ;
2 no independent replication of the plasmid (and target gene)
/ cannot result in multiple copies of the plasmid (and target gene) within one
bacterium;

3 no prokaryotic promoter;
4 which allows recognition by host cells RNA polymerase (which is also prokaryotic)
/ no transcription/ expression of genes;

(c) The student researcher has since made improvements to the plasmid in Fig. 1.2
such that it is now a functional cloning vector.

Steps were then taken to create a recombinant plasmid using BamHI and the
improved plasmid.

Restriction enzyme Specific recognition site


5'-G^G A T C C-3'
BamHI
3'-C C T A G^G-5'

^ indicates where the restriction enzyme cuts

(i) Describe the natural function of BamHI restriction enzyme.


...[1]
1 protect bacteria from attack by viruses/bacteriophages ;
2 cleave any foreign DNA that enters bacterial cell
/ by cutting up intruding DNA from other organism

SAJC / H2 Biology 9648/3 JC2 Prelim 2013


409
4
(ii) Explain the main problem with using Sample 2 and BamHI in the formation of a
recombinant plasmid and suggest how this problem can be solved.
.......[1]
1 cDNA has blunt ends;
2 ref. incompatibility with sticky ends of plasmid after BamHI cuts;
3 ligate linker DNA;
4 to both ends of cDNA fragment;

(iii) Describe the subsequent steps in the formation of recombinant plasmid.


.......[2]
1 Use the same restriction enzyme to digest HGH cDNA and plasmid to generate
complementary sticky ends
2 The plasmid and the HGH cDNA are mixed together
3 the sticky ends will anneal by complementary base pairing
4 DNA ligase added to seal the nicks by catalyzing the formation of phosphodiester
bonds (between adjacent nucleotides on the sugar phosphate backbone)

(iv) With reference to Fig. 1.2, explain how you would identify transformed bacterial
cells that have taken up the recombinant plasmid with the HGH gene using antibiotic
selection.
.......[3]
1 grow bacteria on LB/tet plate ;
2 transformed cells can survive;
3 as they contain plasmid with intact with tetracycline resistance gene;

4 transfer a small amount of each colony to an identified spot on LB/kan plate


/ replica plate on LB/kan plate;
5 bacterial colonies that cannot survive on LB/kan plates have the recombinant
plasmid;
6 insertion of human gene disrupts sequence of kanamycin resistance gene;

Examiners comments
REJECT recombinant plasmid cannot survive. The correct phrasing should be bacterial colonies
containing the recombinant plasmid cannot survive.

[Q1 Total: 14]

SAJC / H2 Biology 9648/3 JC2 Prelim 2013


410
5
2. Severe combined immunodeficiency (SCID) is a genetic disorder characterized
by lack of functional lymphocytes, resulting in defects in both T and B cells responses
of the immune system. Patients are susceptible to opportunistic infections while
children usually die if not treated.

Fig. 2.1 shows the inheritance of SCID in a family in which both parents are carriers.
Only one of the sons suffer from SCID.

Fig. 2.1

(a) State the form of SCID that is being inherited in the family and the mode of
inheritance.
...[1]
1 ADA-SCID
2 autosomal recessive

(b) Two RFLP morphs have been found tightly linked to the adenosine deaminase
(ADA) gene which can be used for genetic screening. A radioactive probe has been
designed to reveal the two RFLP morphs.

Both RFLP morphs and radioactive probe are shown in Fig. 2.1 with the arrows
representing the PvuII restriction sites. The asterisk (*) indicates the position of a
mutation at the restriction site that results in the loss of the restriction site. Morph 2
was found to be linked to the mutant ADA gene.

Fig. 2.2

SAJC / H2 Biology 9648/3 JC2 Prelim 2013


411
6
(i) With reference to Fig 2.1 and Fig. 2.2, draw the resulting autoradiogram you
would expect to see in the space below. [3]

DNA
ladder Father Daughter 1 Son 2

200 bp

100 bp

20 bp
5 bp

For each individual,


1 Correct bands
2 Correct thickness

(ii) Explain the role of the DNA ladder.


...[1]
1 as a reference/standard
/ providing known DNA fragments of specific lengths
2 to estimate the size of unknown DNA molecules

(c) Apart from disease detection, other applications of RFLP include genetic
fingerprinting and in genomic mapping.

(i) Explain why genes especially protein-encoding genes are seldom used as
markers for genetic fingerprinting.
......[2]
1 sequence of protein-encoding genes are conserved
2 due to its important biological function;
3 mutation rate lower than non-coding region
4 not able to differentiate between individuals;

(ii) Explain the use of genetic markers in genomic mapping in terms of linkage
mapping.
......[3]
1 Genetic markers are scattered throughout each of the chromosomes
2 used to determine the order/position of genes
3 Frequency with which two genetic markers are inherited together
4 is a measure of the closeness of the two loci on a chromosome

5 Recombination frequency / frequency of crossing over between 2 genetic markers


6 indicates the distance between two loci on a chromosome in map units /
centiMorgan;

SAJC / H2 Biology 9648/3 JC2 Prelim 2013


412
7
(d) Another method to detect ADA-SCID is through the use of PCR. Fig 2.3 shows
the number of DNA molecules made using PCR, starting with one molecule.

Fig 2.3

(i) Explain three advantages of PCR.


.......[3]
1 Millions of copies of target DNA can be obtained in a relatively short amount of
time
2 Target DNA is doubled after every round of PCR / exponential increase / 2n

3 Easy to set up a PCR reaction and the use of a thermocycler / PCR machine
4 availability of computer software for primer design / commercial synthesis of
primers

5 Sensitivity of PCR
6 capable of amplifying sequences from minute amounts of target DNA

7 Specificity in amplifying only target sequences


8 as primers hydrogen-bond only to complementary sequences

9 A broad range of nucleic acid sources are suitable templates for PCR
amplification / Robustness
10 permit amplification of specific sequences from material in which the DNA is badly
degraded or embedded in a medium

11 Cell-free method of DNA replication


12 requires no cleanup of unwanted cellular debris or vector DNA

(ii) Suggest two reasons for the leveling off of the graph from cycles 17 to 20.
...[1]
1 Nucleotides are used up (hence is unable to make complementary chains)
2 Primers are used up
3 the template DNA strands anneal with each other instead of with the primer
4 Taq DNA polymerase gradually becomes denatured ref. no elongation

[Q2 Total: 14]


SAJC / H2 Biology 9648/3 JC2 Prelim 2013
413
8
3.
(a) Many varieties of rice, Oryza sativa, die quickly when totally submerged by flood
water during the monsoon season. Typically, submergence-intolerant plants respond
to submerged conditions by elongating rapidly.

In 2006, a gene, Sub1A, on rice chromosome 9 was found to be involved in tolerance


of prolonged submergence.

An allele, Sub1A-1, was found only in submergence-tolerant rice. Varieties that are
submergence-intolerant either have a second allele, Sub1A-2, or lack the gene
altogether.

(i) Sub1A-1 codes for a protein that controls the transcription of the gene for alcohol
dehydrogenase. This gene is activated when the plant is in anaerobic conditions.

Explain the importance to submerged rice plants of producing alcohol dehydrogenase.


..........[2]
1 Lack of oxygen in submerged conditions
2 leads to anaerobic respiration
3 Alcohol dehydrogenase catalyse conversion of ethanal to ethanol
/ regeneration of NAD+
/ hydrogen atoms from NADH donated to ethanol to form NAD+
4 allows for generation of ATP (by glycolysis)

Examiners comments
In plants (i.e. alcoholic fermentation), alcohol dehydrogenase enzyme catalyses the reduction of
ethanol/acetaldehyde to produce ethanol, with NAD+ being regenerated in the process. In animals,
alcohol dehydrogenase is used in the liver in an opposite reaction to breakdown ethanol..

A variety of submergence-intolerant rice, which lacked the gene Sub1A, was


genetically engineered to express Sub1A-1.

The mean heights of three types of rice plants were compared before and after 10
days of submergence in water. The types of rice plants were:

I : submergence-intolerant plants
I+ : submergence-intolerant plants genetically engineered to express Sub1A-1
T : submergence-tolerant plants

The results are shown in Fig. 3.1.

Fig. 3.1

SAJC / H2 Biology 9648/3 JC2 Prelim 2013


414
9
(ii) With reference to Figure 3.1, compare the effect of submergence on the
elongation of the three types of rice plants.
.......[2]
1 All three plants elongate during submergence ;
2 ref. comparison of elongation of I, I+ and T plants
/ eg. Expression of Sub1-A1 in I+ and T resulted in reduced elongation ;

3 I 28 cm elongation / 2.4 times elongation


4 I+ 5 cm elongation / 1.5 times elongation
5 T 2 cm elongation / 1.08 times elongation

The allele Sub1A-1 could be introduced into high-yielding variety of submergence-


intolerant rice either by genetic engineering or by selective breeding.

(iii) Suggest two advantages of using genetic engineering, rather than selective
breeding, to introduce the allele.
..........[1]
1 More specific in transfer of allele conferring high yield
2 Does not dilute desirable traits of high yield variety
3 Genes from different species can be combined (to produce a transgenic
organism)
4 Shorter time required to see results compared to traditional selective breeding
which may take several generations

(b) Round-Up ReadyTM soybeans are genetically modified to be resistant towards


Round-UpTM herbicide, a glyphosate-based herbicide that normally inhibits an
important enzyme in the aromatic amino acid synthesis pathway in plants. This allows
farmers to freely apply Round-UpTM to destroy weeds without affecting the Round-Up
ReadyTM soybeans.

Round-Up ReadyTM soybeans may be produced by first transfecting soybean callus


cells with the resistance gene from Agrobacterium.

(i) To obtain a callus culture, suitable explants must first be chosen. Suggest a
suitable explant and explain your choice.
.......[1]
1 eg. apical or axillary buds / shoot tips / root tips
2 contain meristematic tissues which are (actively dividing, undifferentiated and)
disease-free

SAJC / H2 Biology 9648/3 JC2 Prelim 2013


415
10
(ii) Explain three advantages of growing soyabean plants from tissue culture rather
than from seeds.
.......[3]
1 genetically identical
2 possess the desirable features of the stock plants

3 allows the rapid multiplication of plants


4 giving rise to bulk production of genetically identical plants

5 new plants are disease-free;


6 leading to higher yield / better quality

7 genetic modifications possible (with help of protoplast cultures)


8 desirable traits can be introduced

9 Take up little space when compared with plants growing in fields


10 can be grown intensively

11 Plantlets are light and small in size


12 can be air-freighted and transported easily/cheaply/in large quantities, increasing
international trade

13 Independent of climate changes so plants can be produced continuously/at any


time of year
14 flexibility in meeting consumer demand

15 possible to standardise the conditions for growth and obtain many batches of
identical plants
16 ensures product uniformity

(iii) Soyabean plants can reproduce sexually by means of seeds. Soyabean plants
grown from seeds are very variable in their yield. Explain why.
.......[2]
1 there is genetic variation due to

During gamete formation, there is


2 Crossing over
3 between non-sister chromatids of homologous chromosomes during prophase I
4 Independent assortment of homologous chromosomes

5 Seeds formed through random fusion of gametes

SAJC / H2 Biology 9648/3 JC2 Prelim 2013


416
11
Genetically modifying soybean plants to be resistant towards Round-UpTM herbicide
might threaten the environment and human safety.

(iv) State one possible adverse effect on:

The environment
.........[]
1 Seeds from GM crops might be carried to other places and establish themselves
as weeds
2 Cross-pollination between the GM crops and their wild relatives may spread the
resistance to weeds
/ Emergence of vigorous weeds with herbicide-resistant genes

Examiners comments
Some candidates were confused between herbicide-resistant plants and pesticide-resistant plants.

Human safety
.........[]
1 Introduction of foreign gene(s) may result in production of secondary metabolites
which may be toxic to animals themselves and/or livestock/humans that consume
them.
2 New proteins in GM plants may be potentially allergenic to humans that consume
them.
3 Vectors used in introducing the herbicide resistant gene can contain genes for
antibiotic resistance which may pass from the plant to the E. coli in the gut when
the soybean is eaten, making the bacteria resistant to antibiotics

[Q3 Total: 12]

SAJC / H2 Biology 9648/3 JC2 Prelim 2013


417
12
4. Planning question

The goshawk is an endangered species protected by the Convention of International


Trade in Endangered Species of Wild Fauna and Flora (CITES).

The Royal Society for the Protection of Birds (RSPB) has found a man in possession
of four young goshawks and an adult female. He claimed to have bred the young
birds using his adult female and a male bird borrowed from another keeper of these
birds of prey. The RSPB suspects that the young goshawks were poached from the
wild and not bred in captivity as claimed by the man.

Plan an investigation using DNA fingerprinting to verify if the young goshawks were
bred in captivity or poached from the wild.

[Total: 12]

You planning must be based on the assumption that you have been provided with the
following equipment and materials

tissue samples (feather tips) from the four young gohawks, the adult female
and the adult male under investigation
pestle and mortar
DNA extraction buffer solution
ice-cold ethanol
glass rods
microcentrifuge tubes
centrifuge
restriction enzyme
PCR reagents and equipment
agarose gel electrophoresis reagents and equipment
suitable source of electrical current
radioactive probe
nitrocellulose membrane
autoradiography equipment

Your plan should have a clear and helpful structure to include

an explanation of the theory to support your practical procedure

SAJC / H2 Biology 9648/3 JC2 Prelim 2013


418
13
a description of the method used, including the scientific reasoning behind the
method
the type of data generated by the experiment
how the results will be analysed including how the origin of the organism can
be determined.
relevant risks and precautions taken

Theoretical consideration or rationale of the plan to justify the practical


procedure (1 mark)
1 RFLP refers to differences in the restriction sites on homologous chromosomes
that leads to different restriction fragment patterns.
2 DNA fingerprints makes use of polymorphisms that consist of repeated short
tandem repeats (STRs).

Method of DNA extraction including homogenization and use of buffers (2


marks)
1 Homogenise the feather tips of a goshawk by grinding using the pestle and
mortar in the presence of DNA extraction buffer solution.
2 Centrifuge homogenised mixture to separate DNA from the rest of the cell debris.
Use a micropipette to transfer the supernatant which contains the DNA into
another clean microcentrifuge tube
3 Add ice-cold ethanol to precipitate the DNA out of solution. Using the glass rod
to spool up the precipitated DNA and transfer it to a fresh microcentrifuge tube.
4 Resuspend the DNA with restriction enzyme buffer.
5 Repeat steps the preparation of DNA from the other goshawk tissue samples
and. ensure that the same mass of tissue is used to ensure an equivalent amount
of DNA can be produced for analysis.

Selection of restriction enzyme and reason for the selection (1 mark)


1 Add 5 l of EcoRI to sample. Incubate enzyme with DNA samples for 2 hours at
37C.
2 EcoRI will cleave the DNA segments at different sites, generating different sized
fragments for different individual goshawks.

Amplification of DNA fragment using PCR including detail of PCR (2 marks)


1 The DNA is added in a PCR tube with Taq polymerase, free deoxyribonucleotides
and primers designed to flank the DNA sequence to be amplified.
2 The mixture is heated to 95C to break the hydrogen bonds so that the double-
stranded DNA separates into single strands.
3 The mixture is then cooled to 55C to enable the annealing of DNA primers to
complementary sequences flanking the target sequence to be amplified
4 The mixture is heated to 72C for Taq polymerase to catalyse the synthesis of a
complementary strand of DNA for each of the single strands of the target DNA.
5 PCR is needed to generate sufficient DNA for genetic fingerprinting or analysis
/ Repeat cycles for 30 times. The amount of DNA produced will double each
time, resulting in amplification of target DNA sequence.

SAJC / H2 Biology 9648/3 JC2 Prelim 2013


419
14
Separation of fragments by agarose gel electrophoresis and the principles
behind the separation (2 marks)
1 Prepare agarose gel cast and set up electrophoresis chamber.
2 Add 5 l loading dye to DNA samples. Spin the samples down using centrifuge.
3 Load 10 ul of DNA ladder into the first slot. Load 10 ul of individual DNA samples
into the subsequent slots.
4 Conduct gel electrophoresis at 100V till tracking dye move to length of gel.
5 DNA is negatively-charged and will migrate to the positive electrode. Smaller
fragments move through the gel faster than larger fragments.

Transfer of DNA onto nitrocellulose membrane (3 marks)


1 The gel is treated with sodium hydroxide to cause the double-stranded DNA to
denature, separating it into single strands. Denaturation is necessary so that the
DNA will stick to the membrane and be hybridized by the single stranded DNA
probe.
2 A sheet of nitrocellulose membrane is placed on top of the gel. Pressure is
applied evenly to the gel by placing a stack of paper towels and a weight on top of
the membrane and gel. This causes the DNA to move from the gel onto the
nitrocellulose membrane by capillary action, where it sticks.
3 The membrane is then baked to permanently crosslink the DNA to the membrane.

Hybridisation with radioactive labeled DNA probe


4 The membrane is treated with a single-stranded radioactive hybridization probe
which is complementary to the target STR sequence.

Autoradiography method and method of band visualization


5 After hybridization, excess probe is washed from the membrane
6 The pattern of hybridization is visualized on X-ray film by autoradiography.

Significance of matching bands and proposed method of result feedback to


RSPB (1 mark)
1 If the young goshawks are bred in captivity, all bands in the offspring can be
assigned as coming from either the mother or the father.
2 If the young goshawks are poached from the wild, there will be unassigned bands.
3 Autoradiograph diagram

DNA Male Female Young Young Young Young


ladder goshawk goshawk goshawk 1 goshawk 1 goshawk 3 goshawk 4

SAJC / H2 Biology 9648/3 JC2 Prelim 2013


420
15
Risks and safety measures (1 mark)
1 Radioactive probe is carcinogenic. Use isolated area for radioactive probe
hybridisation, work behind a radioactive shield and use latex gloves when
handling samples.
2 To prevent electrical shock, ensure hands are dry before using the electrical
appliance such as the electrical power supply of agarose gel electrophoresis.
Clean all spillage before running the gel to ensure all surfaces near the
electrophoresis gel chamber are dry.

Control (1 mark)
1 Extract DNA from random gohawks and conduct the same DNA fingerprinting
experiment. Unrelated gohawks should have unassigned bands compared to the
male and female parents.
2 Otherwise, new probes for other regions of the genome should be designed to
differentiate between individuals.

Reliability ( mark)
1 ref. At least 3 replicates and at least 3 repeats

SAJC / H2 Biology 9648/3 JC2 Prelim 2013


421
16
1 Free-response question

Write your answer to this question on the separate answer paper provided.

Your answer:

should be illustrated by large, clearly labeled diagrams, where appropriate;


must be in continuous prose, where appropriate;
must be set out in sections (a), (b) etc., as indicated in the question.

5 (a) Compare the properties, features and functions of zygotic and


embryonic stem cells. [5]

(b) Using one named example of an adult stem cell, describe its normal
role and how it could be used in gene therapy against SCID using a
retrovirus. [8]

(c) Discuss the social and ethical considerations for the use of gene
therapy. [7]

[Total: 20]

SAJC / H2 Biology 9648/3 JC2 Prelim 2013


422
17
(a) Compare the properties, features and functions of zygotic and embryonic
stem cells. [5]

Similarities (max 1)
1 Self-renewal ability / divide continuously by mitosis to produce new stem cells
2 Unspecialized / do not have cell-specific structures to carry out specific functions
3 Undifferentiated

Differences
Zygotic stem cells Embryonic stem cells
Isolated from morula / zygote; Isolated from inner cell mass of
blastocyst;

Totipotent, pluripotent and multipotent; Pluripotent, not totipotent but are


multipotent;

able to differentiate into any cell type to able to differentiate into almost any cell
form any organ or type of cells, type to form any organ or type of cells,
including extra-embryonic membranes; except those of the extra-embryonic
membranes;

ability to differentiate into any cell type to give rise to various organs in organism /
form whole organisms; multiple specialized cell types that make
up the heart, lung, skin and other tissues
in the developing foetus

SAJC / H2 Biology 9648/3 JC2 Prelim 2013


423
18
(b) Using one named example of an adult stem cell, describe its normal role
and how it could be used in gene therapy against SCID using a retrovirus. [8]

1 Named example: blood stem cells;


2 Differentiate into cells of the same lineage;
3 E.g. red blood cells, white blood cells and platelets;
4 Replace cells worn out from normal wear and tear;

5 (blood stem cells) isolated from bone marrow (of patient);


6 No cell rejection;
7 Cultured in vitro first to increase numbers;

8 Attenuate retroviral vector by removing viral genes that cause harm (eg. lysis);
9 mRNA of target gene inserted in vector;
10 ref. ADA gene / IL2RG gene;

11 Recombinant vector allowed to infect cultured blood stem cells;


12 Viral genome is reverse transcribed;
13 and resulting cDNA is then integrated into host cell DNA;
14 can be stably propagated by chromosomal replication following cell division

15 Selection of transformed blood stem cells on selection plates;


16 Transformed blood stem cells grown in culture to increase number of cells
[e.c.f from pt 7]
17 Transgenic blood stem cells grown reimplanted/injected into patient;

18 ref. normal functional protein product, thereby restoring the correct function of the
target cells and altering the phenotype
19 functional T and B lymphocytes in the patient;
20 to fight off infections / development of functional immune system;

(c) Discuss the social and ethical considerations for the use of gene therapy.[7]

Benefit of gene therapy


1 Potential for tremendous patient benefit
2 Germline therapy can eradicate disease from subsequent generation and
prevent transmission of serious genetic diseases from parent to offspring

Interfere with evolution


3 elimination of unwanted alleles from the gene pool will decrease genetic
variation
4 genes that are damaging under some conditions may be advantageous under
other conditions
/ germ-line gene therapy would forever change the genetic makeup of an
individuals descendants / the human gene pool would be permanently affected
/ detrimental to the survival of our species in future

SAJC / H2 Biology 9648/3 JC2 Prelim 2013


424
19
Safety
5 question of safety and efficiency of treatment and therefore unknown long term
consequences to the well-being of the recipients.
6 raises questions over who should participate in a human trial the terminally ill
with no treatment options or anyone who understands the potential risk but is a
willing patient

Cost
7 gene therapy is currently very expensive, require expertise and equipment found
only in major medical centres, concerns about gene therapy become a luxury
available only to the rich and powerful.
8 possible genetic enhancements creating an advantage for those who can
afford the treatment

Eugenics social policies


9 type of circumstances that determines whether genomes should be altered
/ what is normal or a disability or disorder, and who decides
/ potential for non-therapeutic enhancement possibilities / eugenic social
policies
10 concerns about the widespread use of gene therapy making society less
accepting of people who have mild disorders / genetic diseases or less able
/ lead to new definitions of normal (e.g. intellect / height / strength etc.)

Privacy
11 concerns about the protection of privacy and confidentiality of medical
information of patients involved in clinical trials
12 which may have implications on (medical) insurance coverage / employability

Interfering with life


13 opposition to gene therapy based on religious grounds, believing that altering
genetic material is against Gods will
14 the unborn child who would be affected by germline gene therapy cannot
choose whether to have the treatment

SAJC / H2 Biology 9648/3 JC2 Prelim 2013


425
HWA CH HONG INST TITUTION
JC2 Preliminary Examinatio
ons
Higher 2

CANDIDA
ATE
CT
C GROUP
P 12S7___
__
NAME

CENTRE INDEX
R
NUMBER NUMBER
N

BIOLOG
GY 9648/01
Paper 1 M
Multiple Cho
oice 27 Septe
ember 2013
3
Additional Materials: Optical Marrk Sheet 1 hour 15
1 minutes
s

CTIONS TO CANDIDAT
INSTRUC TES

1. Writee your nam me, CT grou up, Centre number an


nd index number in t he spaces provided att
the ttop of this cover
c page.
2. Fill iin your parrticulars on the Optica heet. Write your NRIC
al Mark Sh C number and shade
e
acco ordingly.
3. Therre are forty
y questions in this pap ons. For ea ch question
per. Answerr all questio n, there are
e
four possible an
nswers, A, B,
B C and D..
Chooose the onee you consider correctt and record
d your choic
ce in soft p
pencil on th
he separate
e
Opticcal Mark Sh
heet.
4. he end of the paper, yo
At th ou are to sub
bmit only th
he Optical Mark
M Sheet .

ATION FOR
INFORMA R CANDIDA
ATES

Each corre
ect answer will score one n be deducted for a w
o mark. A mark will not wrong answ
wer.
Any rough
h working sh
hould be do
one in this b
booklet.
Calculatorrs may be used.
u

This document cconsists of 26


2 printed pages.
p
426
2

1 Which organellle(s) is / arre required for the forrmation of the hydrolyytic enzyme
es found in
n
lysosomes?

2 Lipid
d membranes can be formed in tthe laborato
ory by paintting phosphholipids ove
er a supportt
(PTF w a hole in it.
FE sheet) with

Such a lipid membrane


m is
s impermea able to watter-soluble materials inncluding ch
harged ionss
+ + +
suchh as Na or K . In one experimentt with Na io ons, no currrent flowed across the membrane e
untill a substancce called grramicidin wa
as added.

Which stateme
ent is cons
sistent with this inform
mation and
d your know
wledge of membrane
e
struccture?

micidin beccomes incorrporated into


Gram o the memb
brane and is
s

A a carbohyydrate mole
ecule found only on the
e outside of the membrrane.

B a non-pollar lipid which passes a


all the way through the
e membranee.

C a protein molecule with drophilic an


w both hyd nd hydropho
obic regionss.

D a protein molecule which


w has on
nly hydroph
hobic region
ns.

Hwa C
Chong Institu
ution 2013 9648 H2
H Biology / JC2 Preliminaary Examination / Paper 1
427
3

3 The table shows the results of food tests carried out to identify two types of biomolecules.

reagent added to test-tube

test-tube
sodium hydroxide +
Benedict's solution iodine in potassium iodide
copper sulphate

X purple brick red brown

Y blue blue blue-black

Z purple blue blue-black

Which conclusion is consistent with the results?

A Only albumen and sucrose had been placed in tube X.

B Only maltose and starch had been placed in tube Z.

C Only starch and sucrose had been placed in tube Y.

D Only starch and sucrose had been placed in tube Z.

Hwa Chong Institution 2013 9648 H2 Biology / JC2 Preliminary Examination / Paper 1
428
4

4 The graph shoows the res sults of an investigatio


on using th
he enzyme invertase that
t breakss
3
dowwn sucrose into
i glucose
e and fructo
ose. 1 g of sucrose
s was dissolvedd in 100 cm water and
d
3
2 cm
m of a 1 % invertase solution
s wass added.

Which conclusion can be drawn


d from this information?

A Between 0 and 60 min, o the substtrate remainns constant.


m the con centration of

B After 60 min,
m the con
ncentration of enzymes
s becomes the limiting factor.

C At 140 min, some off the enzym e molecules are denattured.

D Between 60 and 140


0 min, the cconcentratio
on of the sub
bstrate is thhe limiting fa
actor.

Hwa C
Chong Institu
ution 2013 9648 H2
H Biology / JC2 Preliminaary Examination / Paper 1
429
5

5 The diagram shows


s a sttage of mi tosis in a cell. The amount oof DNA pre
esent is 12
2
pico
ograms (pg)).

umber of DNA molecu


Which row corrrectly identtifies the nu ules and am
mount of DN
NA in each
h
nuclleus at diffe s of nuclear division?
erent stages

telophase
e of mitosis telophase II of meiosis

number of DNA amount o


of DNA / pg numbe
er of DNA amount of
o DNA / pg
g
ecules
mole mole
ecules

A 1
12 6 6 3

B 2
24 1
12 12 6

C 1
12 1
12 6 3

D 2
24 6 12 6

Hwa C
Chong Institu
ution 2013 9648 H2
H Biology / JC2 Preliminaary Examination / Paper 1
430
6

6 Nocodazole is a chemical used in the study of mitosis. It causes all mitotic cells to be
arrested at metaphase.

Which statement(s) correctly identify how nocodazole might work?

1 Inhibits chromatin condensing in the nucleus


2 Prevents replication of the centrioles
3 Stops sister chromatids migrating to opposite poles

A 3 only B 1 and 2 only C 1 and 3 only D All of the above

7 A double-stranded DNA molecule is 10 kb long.

Which row correctly describes the structure of this DNA molecule?

number of nucleotides number of complete turns length of DNA molecule / m

A 10000 1000 3.4

B 20000 1000 0.34

C 20000 1000 3.4

D 20000 10000 3.4

8 An alien organism was found and investigated. Like eukaryotes, it has a linear double-
stranded DNA genome. When DNA replication was examined, it revealed that although the
process is semi-conservative, no Okazaki fragments were observed in all the multiple
replication forks. In addition, the end-replication problem was not observed.

Which statement correctly explains this phenomenon?

A Alien DNA is antiparallel.

B Alien DNA replication only starts at the 3 end of each template strand.

C Alien DNA polymerase synthesises DNA in both 5 to 3 and 3 to 5 directions.

D Alien DNA ligase is not involved in the DNA replication process.

Hwa Chong Institution 2013 9648 H2 Biology / JC2 Preliminary Examination / Paper 1
431
7

9 You have three different double-stranded DNA molecules, each of which you incubate
(separately) in solution that is heated from 25 C to 100 C to denature the double-stranded
molecules.

Molecule 1 is 1000 bp long and has a G-C content of 70 %.


Molecule 2 is 1000 bp long and has a G-C content of 50 %.
Molecule 3 is 2000 bp long and has a G-C content of 40 %.

Which statement is likely to be the correct prediction of the denaturation of these molecules?

A Molecule 2 will denature at the highest temperature.

B Molecule 3 will denature at the highest temperature.

C Molecules 1 and 2 will denature at a higher temperature than Molecule 3.

D All three molecules will denature at the same temperature.

10 Which statements correctly describe the structure and function of prokaryote ribosomes?

1 Prokaryote ribosomes are smaller than eukaryote ribosomes and sediment at


70S.
2 A prokaryote ribosome consists of two subunits, one of 50S and one of 30S.
3 In prokaryotes, ribosomes translate mRNA in the same cellular compartment in
which it is transcribed.
4 In prokaryotes, ribosomes can begin translating mRNA before its synthesis has
been completed.
5 A prokaryote ribosome can accommodate only one tRNA at a time.

A 1, 3 and 5 only

B 2, 4 and 5 only

C 1, 2, 3 and 4 only

D All of the above

Hwa Chong Institution 2013 9648 H2 Biology / JC2 Preliminary Examination / Paper 1
432
8

11 Whicch statements about th


he genetic ccode are co
orrect?

1 e genetic co
The ode has red undancy an
nd is degenerate.
2 The
ere is only one or the amino acid meth
o codon fo hionine.
3 Cod
dons act as stop and start signa
als during tra
anscription and transla
ation.
4 Prokaryotes ge
enerally use
e the same genetic cod
de as eukarryotes.
5 RNA codons have the ssame nucleo
mR otide sequence as DNA
A triplet cod
des.

A 1, 2 and 3 only
o B 1, 2 and 4 only C 1, 3 and
d 5 only D 2, 4 an
nd 5 only

12 During the proccess of transcription, errrors somettimes occurr such that ccertain nucleotides are
e
eated. Thesse errors are
repe a often re eferred to as duplicattions. Dupl ications of nucleotide e
basees within ge enes vary in severity. Some result in non-functional prooteins and others mayy
havee little or no
o effect on function. Th
he diagramm shows a strand
s of m RNA produ uced from a
particular gene.

Whicch duplicatiion event is most likelyy to lead to the


t synthes
sis of a non--functional protein?
p

A One base
e pair is dup
plicated at Z
Z.

B Three basse pairs are


e duplicated
d at X.
C One base
e pair is dup
plicated at Y
Y.
D Three basse pairs are
e duplicated
d at Y.

Hwa C
Chong Institu
ution 2013 9648 H2
H Biology / JC2 Preliminaary Examination / Paper 1
433
9

13 Which statement(s) about bacterial genetic transfer is / are not correct?

1 In transformation, only bacterial cells which possess competence factors in their


chromosomes can take up naked DNA from the surroundings.
2 In generalised transduction, bacterial genes near the prophage insertion site on
the donor chromosome can be transferred to the recipient cell via a T4 phage.
3 In conjugation, DNA replication of the F plasmid occurs in both donor and
recipient cells.
4 Binary fission does not lead to variation in bacterial cells.

A 2 only B 1 and 2 only C 2 and 3 only D 3 and 4 only

14 Which statements about viruses are correct?

1 They can only be seen under the electron microscope.


2 They possess DNA but not RNA.
3 They have a simple structure based on nucleic acids, proteins and
polysaccharides.
4 They are obligate parasites.
5 All possess a capsid made of capsomeres.

A 1 and 3 only B 2 and 5 only C 1, 4 and 5 only D 2, 4 and 5 only

Hwa Chong Institution 2013 9648 H2 Biology / JC2 Preliminary Examination / Paper 1
434
10

15 The diagram sh hows stage es (1 to 8) o


of the repro
oductive cyc
cle of a hum
man immunodeficiencyy
viruss (HIV) in in
nfecting a human CD4+ + lymphocy yte.

Which row corrrectly explains how tthe reprodu


uctive cycle
e of HIV caan be arres
sted at the
e
ous stages?
vario ?

sta
age 1 sta
age 2 sta
age 4 sta
age 7

A drugs th
hat bind to inhibitio
on of viral mutation
n in the HIV inhib
bition of
gpp120 repllicase gennome exoc
cytosis

B restriction inhibitio
on of viral inhib
bition of inhib
bition of
endonuccleases to repllicase inte
egrase buddding
cut th
he viral
gennome

C drugs th
hat bind to inhibition
n of reverse inhib
bition of inhib
bition of
gpp120 transccriptase inte
egrase buddding

D restriction inhibition
n of reverse mutation
n in the HIV inhib
bition of
endonuccleases to transccriptase gennome exoc
cytosis
cut th
he viral
gennome

Hwa C
Chong Institu
ution 2013 9648 H2
H Biology / JC2 Preliminaary Examination / Paper 1
435
11

16 Drug
g R is a DN
NA methyltrransferase inhibitor annd drug Q iss a histonee deacetylas
se inhibitor..
An e
experiment was carried
d out to inve
estigate the
e effects of drug R andd Q on exprression of a
gene
e. The grap
ph shows the experime ental results.

Whicch are posssible explan


nations to th
he results sh
hown?

1 Druug Q controls modificattion of histo


ones, resulting in weakker binding of histoness
to DNA.
D
2 Dru
ug Q incre eases gen ne express
sion by in
ncreasing aaccessibility
y of RNA
A
polyymerase to the promotter.
3 Dru
ug R increasses gene e
expression by
b preventing methylattion at CpG
G islands att
the promoter.
4 Inhiibiting DNAA methylatio
on is more effective
e in increasing gene expre
ession than
n
inhibiting histon
ne deacetyllation.

A 1 and 2 onlly B 2 and 4 o


only C 1, 2 and
d 3 only D All of the
t above

Hwa C
Chong Institu
ution 2013 9648 H2
H Biology / JC2 Preliminaary Examination / Paper 1
436
12

17 The table compares the genomes of various organisms.

organism classification number of size of approximate


chromosomes genome number of protein-
/ Mb coding genes

Haemophilus bacteria 1 1.8 1700


influenzae

Saccharomyces eukarya 16 12.1 5900


cerevisiae
(yeast)

Drosophila eukarya 4 180 13000


melanogaster
(fruit fly)

Oryza sativa eukarya 12 440 50000


(rice)

Canis familiaris eukarya 39 2400 19000


(dog)

Homo sapiens eukarya 23 3000 20000


(human)

Which statement can be concluded from the table?



A There is no correlation between chromosome number and genome size.

B Presence of introns in the eukaryotes resulted in larger genomes and more


chromosomes.

C The more genes there are, the larger the genome.

D The genes are smaller in O. sativa, hence it has a smaller genome.

Hwa Chong Institution 2013 9648 H2 Biology / JC2 Preliminary Examination / Paper 1
437
13

18 The flowchart shows


s the mechanism
m of telomera
ase action.

Whicch row corrrectly descriibes the role


es of the respective mo
olecules in telomerase
e action?

telomerase RN
NA lagging stra
and templatte teelomerase protein

A acts as
a a ribozym
me acts as a template acts as a DNA polymerase
p

B actss as a prime
er acts as a template aacts as a re
everse
transcripttase

C acts as a templa
ate acts as
s a primer aacts as a re
everse
transcripttase

D acts as a ribozyme and a acts as


s a primer aacts as a te
erminal
template transfera
ase

Hwa C
Chong Institu
ution 2013 9648 H2
H Biology / JC2 Preliminaary Examination / Paper 1
438
14

19 The diagram sh
hows cance
erous cells d
dividing in between
b norrmal cells.

Whicch statements can be concluded


c ffrom the dia
agram?

1 ncerous cells can unde


Can ergo self-ren
newal due to
t the preseence of telom
merase.
2 Can
ncerous cells invade no
ormal cells by undergo
oing excess ive mitosis.
3 A tu
umour is a localised ma
ass of cells.
4 A tu
umour is forrmed after u
undergoing through me
etastasis annd angiogen
nesis.

A 1 and 2 onlly B 2 and 3 o


only C 1, 2 and
d 4 only D All of the
t above

20 A po opulation of
o cats conntains indiviiduals with shortened tails know wn as Manx x cats andd
indivviduals with
h normal tails known a s non-Manx x cats. Mating of two nnon-Manx cats
c alwayss
prodduced non-M Manx cats. Mating of twwo Manx cats always produced a mixture off Manx catss
and non-Manx cats, where e the numbeer of Manx cats
c is usua
ally twice th at of non-M
Manx cats.

Whicch hypothessis is consis


stent with th
hese observ
vations?

A The gene
es for shorte
ened tails an
nd normal tails
t are link
ked on the ssame chrom
mosome.

B The gene
es for shorte
ened tails an
nd normal tails
t are sex
x-linked.

C The allele
e for shorttened tails is dominan e allele for normal tails, but the
nt over the e
homozygous domina ant genotyp pe is lethal.

D e for shorttened tails is dominan


The allele nt over the
e allele for normal tails, but the
e
homozygous recessive genotyp pe is lethal.

Hwa C
Chong Institu
ution 2013 9648 H2
H Biology / JC2 Preliminaary Examination / Paper 1
439
15

21 The pedigree shows


s inheritance of a dominant trrait.

key

normal male

normal female

d male
affected

affected
d female

Which identifie ect probabil ities of the trait appea


es the corre aring in thee offspring for the two
o
marrriages?

obability of trait inherita


pro ance

m
marriage 1: III - 1 x III - 3 marriage 2:: III - 2 x III - 4

A 0% 50 % ooffspring

B 50 % offspring
o 0%

C 0% 50 % femaale offspring

D 50 % male offspring 50 % femaale offspring

22 In th
he fruit fly, re
ecessive mutations in either of tw wo independently assorrting genes, brown andd
purpple, preventts the synth hesis of red d pigments in the eyes
s. Thus, hommozygotes of either off
thesse mutation ns have brownish-purrple eyes. However, heterozygot
h tes for botth of these
e
muta ations have e dark red, that is, wild--type eyes.

wo flies which are both


If tw h homozyg gous for a different
d mu
utation and heterozygous for the e
othe nding mutattion are crosssed, what proportion of the progeeny will be mutants?
er correspon

7 1 2 3
A B C D
16 2 3 4

Hwa C
Chong Institu
ution 2013 9648 H2
H Biology / JC2 Preliminaary Examination / Paper 1
440
16

23 In a breed of ra
abbits, multiple alleles with the following dom
minance relaationships control
c coatt
ch h
colouration: C (agouti) > c (chinchillla) > c (Him malayan) > c (albino)

An e
experimenta
al cross be
etween ago
outi and Himalayan prroduced 500 % agouti and 50 %
Hima
alayan prog
geny.

Whicch cross wo
ould have th
he lowest prrobability off producing this result??

A Cch x chch B Cc x chc C Cch x chc D Cc x chch

24 The phenotype
es of three experimenttal populations of plan
nts X, Y annd Z are sh
hown in the
e
grap
phs.

The three populations X, Y and Z reprresent, resp


pectively, th
he

A F1, F2 and
d F3 generations.

B P, F1 and F2 generations.

C F2, P and F1 generations.

D F3, F1 and
d F2 generations.

Hwa C
Chong Institu
ution 2013 9648 H2
H Biology / JC2 Preliminaary Examination / Paper 1
441
17

25 The graph show


ws the abso
orption specctra of some
e pigments found in chhloroplasts.

Which statement is not co


orrect?

A Having several pigments,


p rather th
han one, increases the efficiency off
nthesis.
photosyn

B Most leavves are gre


een as chlo rophyll abs
sorbs light in
n the blue aand red reg
gions of the
e
spectrum
m.

C Photosynnthesis will be fastest iin light at th


he red end of the specctrum, as re
ed light hass
higher en
nergy than blue
b light.

D Prior to leaf fall, chlorophyll iis broken down, leav


ving carotennoids which makes
leaves look yellow or red.

26 hows the relationship b


The diagram sh between varrious metab
bolic processses.

Glucos
se

CO2 + H2O Pyruvate Lactate


Q R

Whicch statemen
nts are corrrect?

1 Ace
etyl coenzym
me A is requ
uired for ste
ep Q.
2 Sub
bstrate level phosphoryylation occu
urs in both steps
s P andd Q.
3 The
ere is a net increase in ATP during
g steps P, Q and R.
4 Oxid
dised form of NAD is u
used in step
ps P and R.

A 1 and 2 only B 1 and 3 o nly C 2 and 4 only D 1, 2 an


nd 4 only

Hwa C
Chong Institu
ution 2013 9648 H2
H Biology / JC2 Preliminaary Examination / Paper 1
442
18

27 Mammalian liver cells were homogenised and the resulting homogenate centrifuged.
Portions containing only mitochondria and cytosol (residual cytoplasm) were each isolated.
Samples of each portion, and of the complete homogenate, were incubated in four ways:

1 with glucose;
2 with pyruvate;
3 with glucose plus cyanide; and
4 with pyruvate plus cyanide.

Cyanide inhibits carriers in the electron transport chain. After incubation, the presence or
absence of carbon dioxide and lactate in each sample was determined. The results are
summarised in the table below.

samples of homogenate

complete mitochondria only cytosol

carbon carbon carbon


lactate lactate lactate
dioxide dioxide dioxide

1 glucose

2 pyruvate

3 glucose +

cyanide

4 pyruvate +

cyanide

= absent, = present

Which statement can be concluded from the table?

A Carbon dioxide was not formed when mitochondria was incubated with glucose as
there was no oxygen present.

B Both aerobic and anaerobic respiration were occurring in 3.

C Lactate formation in mitochondria was inhibited by the presence of cyanide.

D The mode of action of cyanide would be similar to that of a cell experiencing anaerobic
respiration.

Hwa Chong Institution 2013 9648 H2 Biology / JC2 Preliminary Examination / Paper 1
443
19

28 A te
est was admministered too evaluate b
blood glucose levels. Subjects
S weere asked to
o fast for 12
2
hourrs before th
he start of the
t test. Th hey were ad dministeredd glucose att time 0 an
nd then had d
bloo
od samples drawn everry 30 min fo or a total of 4 h 30 min.

A grraph of bloo
od glucose levels for n
normal, hyp
perglycaemic and hypooglycaemic individualss
is sh
hown. The values reprresent averaaged data from
f ten sub
bjects for eaach conditio
on.

Whyy would a physician


p diet consisting of severral small meeals to hypoglycaemicc
prrescribe a d
indivviduals?

A to cause the pancrea


as to releasse more glucagon
B to cause the liver to convert mo
ore glucose to glycogen
n
C t blood gllucose conc
to rapidlyy decrease the centration
D to mainta
ain a steady
y blood gluccose concen
ntration

Hwa C
Chong Institu
ution 2013 9648 H2
H Biology / JC2 Preliminaary Examination / Paper 1
444
20

29 The diagram sh
hows a neuromuscularr junction.

Which row show


ws the corre
ect structurre to function relationsh
hip?

Q R S

A axo
on allows foor p
post-synapttic membrane musscle cell will undergo
transmisssion of electrical allows for exocytosis
e of
o contrraction upon receiving
impulse neurotraansmitters chemical signals

B axon allo
ows for movvement pre-synaptic membran ne siggnal transduuction will
of neurootransmitters from allows for exocytosis
e of
o occcur in cytop
plasm of
cell bo
ody to the post-
p neurotraansmitters musccle cell upon
n activation
synapptic membra ane oof chemicallyy-gated
receptoors

C axo
on allows foor pre-synaptic membran ne siggnal transduuction will
transmisssion of electrical allows for exocytosis
e of
o occcur in cytop
plasm of
impulse neurotraansmitters musccle cell upon
n activation
oof chemicallyy-gated
receptoors

D axon allo
ows for mov vement p
post-synapttic membrane musscle cell will undergo
of neurootransmitters from allows for exocytosis
e of
o contrraction upon receiving
cell body to the pre-
p neurotraansmitters chemical signals
synap ptic membraane

Hwa C
Chong Institu
ution 2013 9648 H2
H Biology / JC2 Preliminaary Examination / Paper 1
445
21

30 Gammma-aminob butyric acidd (GABA) is a neuro otransmitterr that funcctions as a signalling


g
moleecule in thee central neervous systtem. GABA A binds to a receptor protein located in the e
plasm
ma membra ane of targe
et cells as shown in Fig.1.
F Binding of a GA
ABA molecu ule opens a

chan ows chloride ions (Cl ) to enter th
nnel that allo he cell.

In a treatment for
f seizuress, Valium, a drug, acts s as alloste
eric regulatoor by bindinng at a site
e
that is away from
m the GABAA-binding ssite. Fig. 2 shows
s the movement
m oof chloride io
ons throughh
the cchannel as GABA is inc
creased witth and witho out the drugg being pressent.

Whicch statemen
nts are correct?

1 The
e GABA rece
eptor is a m
multimeric prrotein.
2 Bind
ding of Valiu
um enhance
es the open
ning of the channel.
c
3 A de
epolarizatio
on occurs in the target cell.
c
4 Bind
ding of Valiu
um helps to
o open the channel
c en if GABA is not prese
eve ent.

A 1 and 2 onlyy B 2 and 3 on


nly C 1, 2 and
d 4 only D 2, 3 and
a 4 only

Hwa C
Chong Institu
ution 2013 9648 H2
H Biology / JC2 Preliminaary Examination / Paper 1
446
22

31 Before the settlement of California in the 1800s, the elk population was very large. By about
1900 there were only a few dozen elk left. Owing to protection, there are now about 3000 elk
living in a small number of isolated herds. Unfortunately, some of the elk in all the herds
have difficulty grazing due to a shortened lower jaw.

Which statements best explain this?

1 The early settlers only hunted elk that could graze.


2 There was a mutation affecting jaw size.
3 There is random mating within each herd.
4 The current elk population demonstrates genetic drift.
5 There was directional selection favouring short jaws.

A 1, 2 and 4 only B 1, 3 and 5 only C 2, 3 and 4 only D 2, 3 and 5 only

32 The following statements relate to molecular phylogenetics.

1 Lines of descent from a common ancestor to present-day organisms have


undergone similar, fixed rates of DNA mutation.
2 Organisms with similar base sequences in their DNA are closely related to each
other.
3 The number of differences in the base sequences of DNA of different organisms
can be used to construct evolutionary trees.
4 The proportional rate of fixation of mutations in one gene relative to the rate of
fixation of mutations in other genes stays the same in any given line of descent.

Which statements, when taken together, suggest the existence of a molecular clock that
enables scientists to estimate the time at which one species might have diverged from
another?

A 1 and 2 only B 1 and 4 only C 2 and 3 only D 3 and 4 only

Hwa Chong Institution 2013 9648 H2 Biology / JC2 Preliminary Examination / Paper 1
447
23

33 Twoo organismss, P and Q are from the same order but from
f differeent families. Therefore
e
orga
anisms P an
nd Q belong
g to

A different kingdoms.
k

B different phyla.
p

C the same
e genus.

D the same
e class.

34 Cyto i a protein found in mo


ochrome c is ost organisms. The am
mino acid seequence of this protein
n
varie
es between
n different species
s and
d can be us sed to dete
ermine evol utionary relationships..
ws the numbers of diffferences in the amino acid sequeences of cytochrome c
The table show
ween three species (X, Y, and Z).
betw

sY
species speciees Z

Sp
pecies X 8 2

Sp
pecies Y 0 9

Which diagram best repres


sents the evvolutionary relationship
ps betweenn species X,, Y and Z?

Hwa C
Chong Institu
ution 2013 9648 H2
H Biology / JC2 Preliminaary Examination / Paper 1
448
24

35 Hind
d III is a resstriction end
donuclease
e commonly
y used to cu
ut human D
DNA into pie
eces before
e
inse
erting it into a plasmid.

Which is most likely


l to be the recogniition sequen
nce for this enzyme?

A AAGGAA
A

B AAGAAG
G

C AAGTTC
C

D AAGCTT
T

36 The diagram sh
hows a vecttor, pUC18,, used in ge
enetic engin
neering.

Which statements are true


e for the use
e of pUC18 in genetic engineering
e g?

1 ne of interest is inserte
Gen ed into the la
ac promoter.
2 Gen
ne of interest is inserte
ed into the polylinker.
p
3 Baccteria colon
ny with succcessfully inserted gene of interesst will appe
ear white in
n
colo
our.
4 Baccteria colony with this vvector will grow
g in agarr plate with ampicillin.

A 1 and 3 only B 2 and 4 o


only C 1, 2 and
d 4 only D 2, 3 an
nd 4 only

Hwa C
Chong Institu
ution 2013 9648 H2
H Biology / JC2 Preliminaary Examination / Paper 1
449
25

37 Which statement is correct?

A Clones isolated from cDNA libraries contain promoter sequences.

B Restriction endonucleases cut DNA at specific sites that are always located between
genes.

C To make a cDNA library, either a reverse transcriptase or a restriction endonuclease,


but not both, can be used.

D It is possible for a coding region of a gene to be present in a genomic library prepared


from a particular tissue but absent from cDNA library prepared from the same tissue.

38 DNA fingerprinting is often used to confirm the identity of an individual. The diagram shows
a small section of the gel electrophoresis results from a DNA fingerprinting analysis of a
family.

Which of the children is least likely to be the offspring of both parents?

A child 1

B child 2

C child 3

D all of the children are definitely offspring of both parents

Hwa Chong Institution 2013 9648 H2 Biology / JC2 Preliminary Examination / Paper 1
450
26

39 The diagram sh
hows a gene therapy a
approach for SCID.

Which factors prevent


p this gene thera
apy approac
ch from bec
coming an eeffective trea
atment for
the patient?

1 e patient ma
The ay develop leukaemia.
2 The
e virus may lyse the tarrget T lymphocyte.
3 The
e normal AD
DA allele ma
ay be passe
ed to the ne
ext generati on.
4 Thiss approach may not be nt, thus conttinual gene therapy is necessary.
e permanen

A 1 and 4 only
o

B 2 and 3 only
o

C 1, 2 and 4 only

D 1, 3 and 4 only

40 Wha
at are the arguments against
a the u etically modified organissms (GMOs
use of gene s)?

1 ufficient tes
Insu sting of gene
etically mod
dified crop for their sidee effects
2 Unfforeseen lon
ng-term effe
ects of gene
etic manipulation
3 Acccidental genetic recom
mbination in humans as a resuult of consu
uming food
d
derrived from GMOs
G
4 ntrol of food supply b
Con by a small number of companiess that have
e access to
o
gen
netic engine
eering techn
nology

A 1 and 2 only B 2 and 3 o


only C 1, 2 and
d 3 only D All of the
t above

--- END
D OF PA
APER ---
Hwa C
Chong Institu
ution 2013 9648 H2
H Biology / JC2 Preliminaary Examination / Paper 1
451

HWA CHONG INSTITUTION (COLLEGE SECTION)


2013 JC2 PRELIMINARY EXAMINATIONS
9648 H2 BIOLOGY PAPER 1 ANSWER

Question Solution Question Solution


1 A 21 A
2 C 22 D
3 C 23 B
4 D 24 C
5 A 25 C
6 A 26 A
7 C 27 D
8 C 28 D
9 B 29 C
10 C 30 A
11 B 31 C
12 C 32 B
13 B 33 D
14 C 34 B
15 C 35 D
16 D 36 D
17 A 37 D
18 C 38 A
19 B 39 A
20 C 40 D

452
HW
WA CHONG G INSTITU
UTION
JC2
2 Prelimin
nary Exam
mination
Hig
gher 2

CAN
NDIDATE CT
12S7__
___
NAM
ME GROUP

NTRE
CEN INDEX
NUM
MBER NUMBERR

BIO
OLOGY 9648 / 02
Pap
per 2 Core Paper 3 Sep
ptember 20
013
Add
ditional Mate
erials: Writting Paper 2 hou
urs

INSTRUCTION
NS TO CAN
NDIDATES
The
ere are fourr question booklets
b (I to IV) to thiss paper. Write
W your na
ame, CT grroup, Centrre number and
inde
ex numberr in the spaces provide
ed at the to p of this co
over page, and
a your naame and CT
T group on
n the
lines
s provided a
at the top of
o the cover page of Bo
ooklets II, III and IV.

SEC
CTION A
This
s section co
ontains seve
en structure
ed question
ns. Answerr all questio
ons.
Writte your answ
wers on the
e lines / in th
he spaces p
provided.

CTION B
SEC
This ontains two questions. Answer a ny one que
s section co estion. For Exa
aminers' Us
se
You
ur answers m
must be in continuous
c prose, whe
ere appropriiate.
Question
n Mark
ks
Writte your answ
wers on the
e writing pap
per provide d.
BEG
GIN EACH PART ON A FRESH SHEET
S OF WRITING PAPER.
P 1 / 12

A NIL RETURN
N is required. 2 / 12

3 / 10
INFORMATION
N FOR CAN
NDIDATES
4 / 12
The
e number off marks is given
g in bra
ackets [ ] a
at the end of
o each que
estion or
partt question. 5 / 11

You
u may use a calculator.. 6 / 10
You
u are remind
ded of the need
n for go
ood English and clear presentation
p n in your 7 / 13
answers.
8/9 / 20

Total / 100

This document
d consists
c off 22 printed
d pages an
nd 4 blank pages.

HWA CHONG INSTITUTION 453
2013 JC2 H2 BIOLOGY
PRELIMINARY EXAMINATION MARK SCHEME

PAPER 2

QUESTION 1

(a)(i) Identify the class of carbohydrate molecule of which agarose is an example. [1]
polysaccharide

(ii) Plants contain a carbohydrate called amylose. Amylose does not contain galactose. Describe one similarity and
another difference in structure between agarose and amylose. [2]

similarity:
1. monosaccharides of both agarose and amylose are held together by glycosidic bonds
2. both agarose and amylose are unbranched

difference:
1a. coiled/helical structures in amylose
1b. but linear/straight structures in agarose

2a. alternate glucose is not inverted/ same orientation in amylose


o
2b. but alternate galactose is inverted 180 in agarose

3a. (1,4) glycosidic bond is present in amylose


3b. different types of glycosidic bonds are present in agarose

(iii) Suggest why bacteria are unable to metabolise agarose. [2]

1. lack of bacterial enzyme


2. to hydrolyse the type of glycosidic bonds present in agarose
3. substrate not complementary to active site of enzyme

(b) (i) Describe how the structure of domain A of the cholera toxin is maintained. [3]

1. primary structure folds into secondary structures


2. maintained by hydrogen bonds formed between C=O and NH groups
3. further folding
4. into a unique 3D conformation
5. maintained by hydrogen bonds, ionic bonds, disulfide bonds and hydrophobic interactions between R-
groups of amino acid residues

(ii) Explain how a globular protein like cholera toxin differs from a fibrous protein, such as collagen. [2]

1a. cholera toxin being a globular protein is soluble due to the presence of hydrophilic amino acids on the
surface of the protein
1b. collagen being a fibrous protein is insoluble due to the presence of mainly hydrophobic amino acids in the
protein

2a. cholera toxin being a globular protein is more compact/ spherical in shape
2b. collagen being a fibrous protein is elongated in shape/forms multimolecular parallel filament to strands/
collagen fibrils and collagen fibres

3a. cholera toxin being a globular protein is made up of fixed, non-repetitive specific sequence of amino acids
3b. collagen being a fibrous protein is made up of repetitive sequence of amino acids

4a. cholera toxin being a globular protein has a relatively unstable structure ref. to weak non-covalent bonds
4b. collagen being a fibrous protein has a stable structure ref. to extensive intra- and inter-molecular
hydrogen bonds/ covalent crosslinks

5a. cholera toxin being a globular protein performs metabolic functions since they are soluble
5b. collagen being a fibrous protein performs structural functions since they have high tensile strength

HWA CHONG INSTITUTION / 2013 JC2 H2 BIOLOGY / PRELIMINARY EXAMINATION / MARK SCHEME 1
(c) Suggest h
how the chole
era toxin enters epithelia l cells. 454 [2]

1. receeptor bindingg domain B ofo cholera toxxin is compleementary to receptors


r onn intestinal epithelial cellss
2. bind
ds/attach to receptors
r on
n cell membra ane
3. resuulting in a co
onformational change in rreceptor
4. choolera toxin ennters by receptor mediate ed endocytos
sis
5. inva
agination/ inffolding of epiithelial cell m
membrane
6. form
mation of a vesicle
v enclossing cholera toxin

[Total: 12 marks]
ESTION 2
QUE

(a) (i) Name stru


uctures X and Y. [2]

X: Heterocchromatin
Y: Euchro
omatin

(ii) Account ffor the differe


ence in the structures
s Xa
and Y. [2]

1. Reggion X is morre electron dense


d / darklyy stained thaan region Y
2. Thu
us heterochroomatin is mo ore condense ed / tightly pa
acked / coiled
d than euchrromatin
3. Hetterochromatin is transcrip
ptionally inacctive as comp pared to euchromatin
4. Hetterochromatin is wound around
a deaceetylated histoones

Fig. 2.2 shows p


part of a DNA
A molecule.

5 ; 3 ;

3 5

Fig. 2b
2

(b) (i) Complete Fig. 2.2 by indicating the


e polarity of tthe DNA molecule in the boxes providded. [1]

(ii) State the importance of


o hydrogen bonds in DN
NA structure. [2]

1. Hyd
drogen bonds allow for th
he formation of double sttranded DNA
A / a double hhelix

HW
WA CHONG INS
STITUTION / 2013 JC2 H2 BIOLOGY / PRELIM
MINARY EXAM
MINATION / MAR
RK SCHEME 2
2. Hydrogen bonds hold the two polynucleotide strands together 455
3. Hydrogen bonds hold the complementary nucleotides / bases together
4. Many hydrogen bonds give stability to DNA molecule
5. Hydrogen bonds can be broken for transcription / DNA replication to occur

(c) Explain how the data in Table 2.1 helps to confirm the arrangement of bases in DNA. [3]

1. In all the organisms, percentage of adenine to thymine and percentage of guanine to cytosine are
approximately 1:1 ratio / equal / similar
2. For example, in yeast, percentage of adenine is 31.3% which is very similar to thymine with percentage at
32.9% and percentage of guanine is 18.7% which is very similar to cytosine with percentage at 17.1%.
Accept any one example
3 . This shows that adenine and guanine base pair with thymine and cytosine respectively

(d) (i) State how the result for the virus differs from those for all the organisms given in table 2.1. [1]
1. The percentage of adenine to thymine and guanine to cytosine are not similar / not 1:1

(ii) Give a reason for your answer to d(i). [1]


1. Virus is made up of single-stranded DNA / DNA that is not a double helix

[Total: 12 marks]

QUESTION 3

(a) Describe the role of lactose in the regulation of lac operon. [3]

1. Lactose / allolactose acts as an inducer that binds the lac repressor


2. Lac repressor is inactivated and dissociates from the operator
3. RNA polymerase can bind to the promoter, resulting in transcription of structural genes

(b) Identify the location of the point mutation in the operon of strain A. Give a reason for your answer. [2]

1. The mutation occurred in the promoter / operator


2. RNA polymerase cannot recognise / bind to mutant promoter and hence, transcription initiation cannot
occur or mutant operator is permanently bound to lac repressor and hence RNA polymerase cannot
recognise and bind promoter
3. Plasmids used in rescue experiments bear functional promoter and operator and hence, RNA
polymerase can bind to this functional promoter and initiate transcription

(c) Suggest why strain B cannot be rescued at all. [2]

1. Mutation occurred in the lacI gene


2. Resulting in expression of a constitutively active / hyperactive repressor / constitutively active repressor
cannot be inactivated by allolactose
3. Hence, even if the genes are inserted under the control of the functional lac promoter and operator, the
constitutively active repressor will always bind to lac operator and prevent RNA polymerase from binding
to the promoter / continues to inhibit the transcription of the new genes

(d) Describe how the formation of separate mature proteins from these polycistronic mRNAs in HIV maturation and
in bacteria differs. [3]

Bacteria HIV maturation


1. The polycistronic mRNA in bacteria has 1. The polycistronic mRNA in HIV only has one
individual start and stop codons for the coding start and stop codon shared by the coding
region for each polypeptide regions of all polypeptides
2. Coding regions in the polycistronic mRNAs are 2. Coding regions in the polycistronic mRNAs are
translated separately translated as a whole / to give one polyprotein
3. There is no need for cleavage / post- 3. Each polyprotein is cleaved by the protease to
translational modifications of the polypeptide give separate proteins
4. Polypeptides are translated separately 4. Each polyprotein is cleaved by the protease to
give separate proteins

[Total: 10]

HWA CHONG INSTITUTION / 2013 JC2 H2 BIOLOGY / PRELIMINARY EXAMINATION / MARK SCHEME 3
456
QUESTION 4

(a) Describe how an individual can inherit a recessive disorder like albinism or LDHA deficiency from his parents.
[2]
1. Ref to both parents having at least 1 recessive allele each
2. Ref to individual receiving 2 recessive alleles

(b) (i) Using the symbols provided, draw a genetic diagram in the space provided below to show the cross between
the couple. For each of their children, state their genotypes. [5]

Let A represent the dominant allele for normal LDHA production


a represent the recessive allele for LDHA deficiency
N represent the dominant allele for normal skin pigment
n represent the recessive allele for no skin pigment

Parental Woman with normal skin x albino man with LDHA


phenotypes pigment and LDHA production deficiency

AN an
an an

Gametes AN an An aN an

Random fertilization
Female gametes

AN an An aN
Male AN an An aN
an
gametes an an an an

AN AN
Child I Child II
an an

aN an
Child III Child IV
an an

(ii) Suggest how the genotype of Child III has arisen. [2]

1. Ref to incomplete linkage


2. Ref to crossing over

(c) (i) Define the term chromosomal aberrations. [1]

Ref to changes in number or structure of chromosomes

(ii) With reference to Fig. 4.1, identify the parent who have the abnormal karyotype and suggest how the
abnormality arises. [2]

1. Mother
2. Ref to chromosomal translocation
[Total: 12 marks]

QUESTION 5

(a) (i) Describe the effect of flashing light on ATP synthesis in the absence of venturicidin. [2]

1. As the number of light flashes increases from 1 3 flashes, the concentration of ATP increases from
25nM 190 nM
2. which eventually plateaus

(ii) Explain your answer in a (i) [4]

1. Increasing the number of flashes of light result in an increase in the number of excited electrons

HWA CHONG INSTITUTION / 2013 JC2 H2 BIOLOGY / PRELIMINARY EXAMINATION / MARK SCHEME 4
2. and results in more excited electrons travelling down the electron transport chain 457
3. The energy released is coupled to pump/ actively transport H+
4. from the stroma into the thylakoid space/ thylakoid lumen
5. resulting in the formation of a proton motive force/ electrochemical gradient (OWTTE)
6. H+ then diffuses down the concentration gradient from the thylakoid space to the stroma
7. via the ATP synthase complex
8. releasing more free energy which is coupled for synthesise more ATP

(b) Describe and explain the effect of venturicidin on the plant and its ability to produce sugars. [4]

1. In the presence of ventruricidin, ATP concentration remains at close to 0nM even when 3 flashes of light
are used
2. Since ATP synthase is inhibited, H+ cannot diffuse down the electrochemical gradient
3. from thylakoid space to stroma
4. thus there is no supply of energy to drive ATP synthesis/ no energy released for formation of ATP
5. This prevents the progression of the Calvins cycle / no G3P/ no hexose sugar being produced (OWTTE)
6. since ATP is not available for conversion of GP to 1,3-bisphosphoglycerate / regeneration of RuBP

(c) Distinguish the role of ATP synthesis in mitochondria and chloroplasts [1]

1. In mitochondria, ATP synthesised is transported into the cytosol and used for various cellular functions/
active transport/ protein synthesis/ @ref to specific process
2. While ATP synthesised in chloroplasts is transported into the stroma for use in the light-independent
reaction / Calvin cycle for synthesis of glucose
R: Photosynthesis/ light dependent reaction

[Total: 11 marks]

QUESTION 6

(a) State what is meant by an intracellular receptor. [1]

1. Receptor proteins that are located in the cytoplasm / cytosol / nucleus / cell
2. Ligands, e.g. estrogens, that binds such receptors are hydrophobic / lipid soluble and small enough to
diffuse across the hydrophobic core of the cell membrane

(b) Describe stages A, B and C. [5]

Stage A:

1a. Estrogen binds estrogen receptor / ER


1b. via complementary binding (accept: at specific binding site)
1c. resulting in a change in conformation / shape or resulting in ER activation
2. ER bound with estrogens dimerises (essential for full credit)
3. and enters the nucleus

Stage B:

4. Estrogen-ER complex binds estrogen response element (ERE)


Accept: reference to ER as a specific transcription factor / activator that binds ERE
Reject: ER activates ERE

Stage C:

5a. Binding of ER complex to ERE causes DNA to bend


5ai activator/ERE is brought close to the promoter
5b. recruitment of mediators / coactivators / M1 and M2
5c. recruitment of general transcription factors / G

6. RNA polymerase is able to bind to the promoter

7. resulting in the formation of stable transcription initiation complex at the promoter

HWA CHONG INSTITUTION / 2013 JC2 H2 BIOLOGY / PRELIMINARY EXAMINATION / MARK SCHEME 5
8. to initiate transcription/rate of transcription increases 458

(c) With reference to Table 6.1,

(i) explain how SERMs can be used in the treatment of breast cancer. [2]

1a. SERMs are structurally similar to estrogen


1b compete with estrogens for binding with ER

2a. SERMs recruit M1 but not M2 / prevent complete recruitment / assembly of mediators
2b. Formation of stable transcription initiation complex is reduced / prevented
Accept: positioning / stability of TIC

3.* This reduces rate of oncogene/cancer critical gene expression OR reduce rate of transcription of gene
that result in excessive cell division/rate of cell division exceeding rate of cell death
* Point 3 is essential to achieve full credit because it specifically address treatment of cancer.

(ii) suggest why ERDRs might be more effective than SERMs. [2]

1. ERDRs prevent movement of ERDR-ER complexes to the nucleus


2. ER remaining in the cytoplasm are degraded
3. This completely prevents up-regulation of oncogene expression / transcription (mark not awarded if
concept in point 1 is wrong)
4. SERMs still allows movement of ER into the nucleus/enables recruitment of M1 and general transcription
factors / transcription initiation complex to be formed for transcription/gene expression

[Total: 10 marks]

QUESTION 7

(a) Discuss how cladograms can be used in classification of organisms. [5]

1. A cladogram is used in classification to represent evolutionary relationship /descent with modification


among organisms
2. A cladogram is made up of nested hierarchies
3. Each clade consists of an ancestral species and all its descendants
4. Organisms within a clade are grouped based on synapomorphies/shared derived characters/homology
5. Shared derived characters originate from a recent common ancestor and are present in all its
descendants
6. Within a cladogram, living organism may be classified based on molecular homologies
7. Therefore, classification based on cladograms may correspond to traditional classification where each
clade may correspond to a genus, family or some broader taxon

(b) State what is meant by transient polymorphism in moths. [1]

1. The existence of two phenotypes / phenotypically distinct forms (melanic and non-melanic) of moths in a
particular species

(c) Explain the binomial nomenclature of Biston betularia. [2]

1. Biston betularia is the unique two-part name of a particular moth species, recognised internationally
2. Biston indicates the genus and betularia is the specific epithet

(d) Explain for the trends observed for the three moths species from 1987 to 2002. [3]

1. From 1987 to 2002, the % melanic B. betularia declined from 95% to 5% as compared to the % melanic
A. crenata which ranged between 63% to 60% and % melanic O. bidentata declined from 60% to 32%
2. There was a change from polluted (industrial) to clean (post-industrial) environment leading to growth of
pale coloured lichens on the bark of trees
3. B. betularia are most vulnerable to in the day as they lay on the branches
4. O. bidentata are hidden under the leaves or within cracks less likely to be vulnerable

HWA CHONG INSTITUTION / 2013 JC2 H2 BIOLOGY / PRELIMINARY EXAMINATION / MARK SCHEME 6
5. Gra uickly / colour not influencced by polluttion so least change in / less selectio
ass grows qu on pressure
459 o A.
on
cren
nata
6. AVPP

(e) Account forr the type of speciation th


hat has led to
o the existen
nce of the thrree different moth specie
es in Manche ester.
[2]
1. Symmpatric speciation
2. Thee three moth species occ cupy differentt ecological niches
n
[Total: 13 marks]

QUE ESTION 8
(a) Describe tthe structure
e of a chromo
osome prior tto the start of
o mitosis. [5]

1. Prio
or to the sta art of mitosis, each chrromosome contains
c DNA molecules joined at the
two identical D
cen
ntromeres
2. Thiss is due to DNA
D replicatio
on during S p phase of inte
erphase
3. Thee chromosom me exists in n the form o of unconden nsed chroma atin and dis crete chrom
mosomes are e not
obsserved in the cell
4. Eacch DNA mole ecule is tightlly coiled arou
und the posittively charge
ed histone occtamer
5. Thiss forms the 10-nm
1 chrommatin fibre/ nuucleosome fiibre/ beads--on-a-string
6. DNA A is further coiled
c to give
e a 30-nm ch hromatin fibre
e / solenoid
7. Thee chromatin within a nuc cleus prior too mitosis caan be organiized into difffused euchro
omatin, or highly
h
con
ndensed hete erochromatin n.

(b) e aid of large


With the e and clearly labeled diag
grams, expla ain the signifficance of thee behaviour of chromoso
omes
during m
meiosis, whicch can lead to
o genetic varriation in plan
nts. [10]

1.
PL1 Propha
ase I

PL2. Homologo
ous chromossomes

PL3
3. Crossing over occurss between
er chromatidds at the
non-siste
chiasmatta

PL4
4. Different alleles
a

PD1. Hommologous chromosomes pair up


PD2. Hom mologous ch hromosomes have the sa ame size, sh
hape and centromere loccation, but different
d allele
es at
the same gene loci
PD3. Croossing over
PS1. Excchange of ge enetic materia
al (alleles) be
etween homologous secttions of non--sister chrom
matids
PS2. Givve rise to new
w combination of paterna l and matern nal alleles
PS3. Gam metes will ha
ave chromosomes that arre different frrom parental cells
PS4. AVP P

ML1. Metaph
hase I

HW
WA CHONG INS
STITUTION / 2013 JC2 H2 BIOLOGY / PRELIM
MINARY EXAM
MINATION / MAR
RK SCHEME 7
460

ML2.
M Kinetochore spinndle fibre

ML3.
M Metaphase plate

ML6. Rec
combinant chhromatid

ML4. Paternal homolog ue

ML5. Maternal homoloogue


MD1. Pairs of homolo
ogous chromo osomes alignn at the meta
aphase plate
e
MD2. Eacch homologuue on one sid
de of the mettaphase plate
e
MD3. Twoo or more paairs of homo
ologous chroomosomes having
h differe
ent orientatioons of patern
nal and maternal
reco
ombinant chromosomes towards the poles

MS1. Independentt assortmentt


ere is random
MS2. The m distributionn of paternal and maternaal chromosom
mes in each daughter cell
MS3. Num
mber of posssible combination of gam metes due to independennt assortmen t is 2n for a diploid organ
nism,
whe
ere n=numbe er of sets of homologous
h chromosom mes
MS4. Dau
ughter cells have
h differen
nt combinatioon of chromo
osomes comp pared to pareental cells
MS5. AVP
P

(c) Discuss how the dysre


egulation of checkpoints
c of cell divisio
on can resultt in cancer.
[5]
1. Cell cycle checkkpoints contrrol/ regulate tthe cell cycle e progression n / cell divisioon
2. Muttations in genes E.g., cyc clins/ cdks/ RRb/ E2F/ p21/p53/ AVP controlling
c ceell cycle may lead to cell cycle
c
proggression
3. Dyssregulation inn cell cycle signalling patthways may also a lead to cell
c cycle proogression
4. Dyssregulation of
o G1 checkpo oint may resu ult in replicattion of damaged/ defectivve DNA
5. Dyssregulation of
o G2 checkpo oint may resu ult in cell cyc
cle progression despite D DNA damage e during DNA A
replication/ cellss escaping apoptosis eve en if there is irreparable damage
d
6. Dyssregulation of
o M checkpo oint may resu ult in cell cycle progressioon even if theere is non-dissjunction
7. Theerefore, resu ulting in more copies off protooncog gnes /absen nce of tumoour suppress sor genes in n the
dauughter cell
8. Genne mutationss/ chromoso omal aberrattions are pa assed on to daughter ceells as the cells c do not stop
dividing to repair damage
9. Succcessive rounds of cell division allow w accumulatio on of mutatioons in other ccancer critical genes succh as
Rass/ p53 in a single cell line
eage
10. Eveentually there e is uncontro
olled cell divi sion that thee rate of cell division is g reater than that
t of cell death,
lead
ding to cance er

HW
WA CHONG INS
STITUTION / 2013 JC2 H2 BIOLOGY / PRELIM
MINARY EXAM
MINATION / MAR
RK SCHEME 8
QUESTION 9 461

(a) Discuss the principles of homeostasis. [5]

1. Homeostasis is the maintenance of a constant internal environment of an organism providing it with a


degree of independence from the external environment
2. This involves the constant monitoring of multiple physiological conditions or variables within the body to
maintain the steady state value / range of values about a specific set point
3. Any fluctuations / deviation above or below the set point serve as the stimulus, which is detected by a
receptor
4. This information is integrated at the integrating centre to initiate a response in the form of signals and will
act upon target / effector cells
5. Homeostatic control systems usually operate via negative feedback mechanisms where the response
carried out returns the variable to the set point

(b) Explain the roles of insulin and glucagon in the regulation of blood glucose. [10]

1. The control of blood glucose concentration operates by a negative feedback system via insulin and
glucagon, which are antagonistic hormones
2. When blood glucose concentration is higher than the set point of 90 mg/100ml of blood, the change is
detected by the -cells of the islets of Langerhans of pancreas and insulin is released to decrease blood
glucose concentration towards the set point
3. Insulin binds to insulin receptors on target cells and accelerates rate of glucose uptake
4. In muscle, liver and adipose cells, the number of glucose transporters increases in cell membrane
5. In liver and skeletal muscle cells, glucose utilisation and storage increases such that there is an
increased rate of glycolysis for production of ATP, glycogenesis stimulated by activating glycogen
synthase and glycogenolysis inhibited
6. In skeletal muscle cells, amino acid absorption and protein synthesis stimulated and gluconeogenesis
inhibited
7. In adipose tissues, lipogenesis is stimulated
8. When blood glucose concentration is lower than the set point of 90 mg/100ml of blood, change is
detected by the -cells of the islets of Langerhans of pancreas, glucagon is released to increase blood
glucose concentration towards the set point
9. Glucagon binds to glucagon receptors on target cells and stimulates glycogenolysis in liver and muscle
cells
10. Gluconeogenesis stimulated in liver cells and lipolysis in adipose tissues

(c) Describe how two named membrane proteins are involved in impulse transmission along the axon of a motor
neuron. [5]

L1. Sodium and potassium leak channels


+ + + +
L2. There are more K leak channels than Na leak channels, leading to more diffusion of K than Na down
their respective concentration gradient via facilitated diffusion
L3. This results in a net loss of positive charge
L4. Resting membrane potential of neurons restored / maintained at about -70mV, so that it can be excited to
produce action potential when there is a stimulus
P1. Sodium-potassium pumps
P2. Three sodium ions are pumped out of the neurone while two potassium ions are pumped into the neuron
via active transport against their respective concentration gradient
P3 It restores the original ionic gradients and results in a net loss of positive charges
P4. Resting membrane potential of neurons restored / maintained at about -70mV, so that it can be excited to
produce action potential when there is a stimulus
S1. Voltage-gated sodium channels
+ +,
S2. A stimulus causes some voltage-gated Na channels to open, which allows influx of Na
S3. and results in depolarisation of membrane
S4. If the depolarisation reaches threshold potential of -55mV, an action potential is generated, so that more/
all voltage-gated Na+ channels will open
S5. depolarising the membrane potential to +40mV
K1. Voltage-gated potassium channels
K2. At the peak of an action potential, voltage-gated K+ channels opens, which allows efflux of K+
K3. and hence repolarisation
K4. Voltage-gated K+ channels are slow to close, so that excessive K+ efflux causes the membrane potential
to hyperpolarised

HWA CHONG INSTITUTION / 2013 JC2 H2 BIOLOGY / PRELIMINARY EXAMINATION / MARK SCHEME 9
462
HW
WA CHONG G INSTITU
UTION
JC2
2 Prelimin
nary Exam
mination
Hig
gher 2

CAN
NDIDATE CT
12S7__
___
NAM
ME GROUP

NTRE
CEN INDEX
NUM
MBER NUMBERR

BIO
OLOGY 9648 / 03
Pap
per 3 Applications Pap
per and Pla
anning Quesstion 23 Sep
ptember 20
013
Add
ditional Mate
erials: Writting Paper 2 hou
urs

INSTRUCTION
NS TO CAN
NDIDATES
The
ere are fourr question booklets
b (I to IV) to thiss paper. Write
W your na
ame, CT grroup, Centre numberr and
inde
ex numberr in the spaces provide
ed at the to p of this co
over page, and
a your naame and CT
T group on
n the
lines
s provided a
at the top of
o the cover page of Bo
ooklets II, III and IV.

STR
RUCTURED
D QUESTIO
ONS
Ans
swer all thre ur answers on the lines
ee questionss. Write you s provided.

ANNING QU
PLA UESTION
Ans
swer the que
estion in bo
ooklet IV. Write
W your an
nswers on th
he lines / in the spacess provided.

FRE
EE RESPONSE QUES
STION
Ans estion. Your answers must
swer the que m be in ccontinuous prose, where appropriaate.
Writte your answ
wers on the
e writing pap
per provide d.
BEG
GIN EACH PART ON A FRESH SHEET
S OF WRITING PAPER.
P
A NIL RETURN
N is required. For Examiners' Us
se

Question Mark
ks
INFORMATION
N FOR CAN
NDIDATES
1 / 14
e number off marks is given in brac
The ckets [ ] at tthe end of each
e question
or part
p question
n. 2 / 12

Calc
culators ma
ay be used. 3 / 14
You
u are remind
ded of the need
n for goo
od English a
and clear presentation
p n in
4 / 12
your answers.
5 / 20

Total / 72

This document
d consists
c off 15 printed
d pages an
nd 2 blank pages.

463
2

B
BOOKLET
TI

STRUCT
TURED QU
UESTIONS

QUESTIO
ON 1

Tobacco p plants are vulnerable to tobacco o mosaic virus (TMV). It is a RN NA virus, which
w upon
eads to dea
infection le ath. Scientists have uused genetic engineerring to overrcome this problem
p byy
creating ttransgenic plants tha at are resisstant to TMV.
T This technique involves the t use off
Agrobacte erium tumeffaciens (thaat naturally infects plan
nt cells) and
d the Ti plassmid as shown in Fig.
1.1. These e transgeniic plants ca
an express the viral ge ene coding for TMV cooat protein. The coatt
protein willl provoke a response from
f ants, resulting in resista
the pla ance to TM
MV.

step 1: Issolation of viral RNA and RNA


coonversion to cDNA
c cDN
NA
Coat

step 2: Formation off recombina


ant Ti
plasmid
T
TMV
Leaf-specific
Vira
al gene coding
g pro
omoter
for TMV
T coat
prottein

step 3: Transformation
n of A. tumeffaciens R
Recombinantt Ti plasmid
w
with recombinant Ti plasmid

step 4: Innfection of plant cells by


trransformed A.
A tumefacien
ns and Trransformed A.
A tumefaciens
s
in
ntegration off Ti plasmid
d DNA
in
nto plant chro
omosome

step 5: R
Recombinant plant cells grrown in
Infected plant
p cell Ti plasmiid DNA
tissue culture into plantletts and
hen whole tra
th ansgenic plannts carrying viral
v gene
for TMV coat
protein within
w plant

step 6: TMV rubbeed


onto th
he
su urface of
le
eaves of
trransgenic
l t

Tra
ansgenic plan t Control

Fig. 1.1
Hwa C
Chong Institu
ution 2013 9648 H2
H Biology / JC2
J Preliminaary Examinattion / Paper 3

464
3

(a) Outline how the isolated viral RNA can be converted into a double-stranded cDNA in step 1.
[2]

(b) (i) Explain how the recombinant Ti plasmid in step 2 can be formed. [3]

(ii) Explain the purpose of having a leaf-specific promoter in the recombinant Ti plasmid
formed in step 2. [2]

One important role of A. tumefaciens is to infect plant cells to facilitate the integration of Ti plasmid
DNA into plant chromosomes in step 4.

(c) Suggest another important role of A. tumefaciens in the process of creating transgenic
plants which are resistant to TMV.
[1]

Hwa Chong Institution 2013 9648 H2 Biology / JC2 Preliminary Examination / Paper 3

465
4

Recombinant plant cells are grown using tissue culture into plantlets and then whole transgenic
plants in step 5. The initial stages involve growing the plant cells in growth media supplemented
with different ingredients, such as inorganic nutrients, before a change of environment is required
for the newly formed plantlets in the final stage.

(d) (i) Describe the functions of two other named ingredients in the growth media in the
development of plant tissues. [4]

(ii) Explain the significance of the change of environment for the newly formed plantlets in
the final stage of tissue culture. [2]

[Total: 14]

Hwa Chong Institution 2013 9648 H2 Biology / JC2 Preliminary Examination / Paper 3

466
5

NAME: __________________________________ CT GROUP: 12S7 ___

BOOKLET II

STRUCTURED QUESTIONS (continued)

BLANK PAGE

Hwa Chong Institution 2013 9648 H2 Biology / JC2 Preliminary Examination / Paper 3

467
6

QUESTIO
ON 2

A coupless first child is affected by cystic fibbrosis. As they are try


ying for a s econd child
d by in vitro
o
fertilisation
n, they un nderwent genetic
g he mutant cystic fibrrosis trans
tessting for th smembrane e
conductan nce regulato
or (CFTR) allele
a in thre
ee-day-old embryos.
e

(a) State what is meant


m by gen
netic testing
g. [1]

The CFTRR alleles of each family y member a are amplified by polymerase chainn reaction (PCR). Fig.
2.1 showss the resultss of gel elec
ctrophoresiss performedd on each DNA samplee.

Fig. 2.1

(b) ng the inform


Usin mation given in Fig. 2.1
1, explain why
w

(i) the three--day-old em


mbryo will de
evelop into a child affected by cysstic fibrosis. [2]

Hwa C
Chong Institu
ution 2013 9648 H2
H Biology / JC2
J Preliminaary Examinattion / Paper 3

468
7

(ii) the positiion of allele 2 on the


e agarose gel indicate
es that it ccontains a deletion in
comparison with alle ele 1. [2]

The couplles first child undergo


oes liposom me-mediated d gene therrapy. Fig. 22.2 shows the normal
CFTR alle
eles being trransferred into his traccheal epithelial cells.

Fig. 2.2

(c) he child cannot pass on


Suggest why th n the norma
al CFTR alle
ele to his offfspring. [1]

Hwa C
Chong Institu
ution 2013 9648 H2
H Biology / JC2
J Preliminaary Examinattion / Paper 3

469
8

(d) State one limitation of lipo


osome-med
diated gene
e therapy and
a suggestt one impro
ovement to
o
this limitation.
[2]

limitation:

imprrovement:

Linkage a
analysis usin
ng RFLP markers
m provvided evideence for the existence of the CFT TR locus on
human ch hromosome e 7. Fig. 2.3
2 shows the positio on of the CFTR
C locuss in relation to RFLPP
markers. The asterrix (*) indicates the poosition of a Taq I resstriction sitee that can be lost byy
mutation.

Fig. 2.3

(e) Describe how RFLP


R analy
ysis facilitate
ed linkage mapping
m of the CFTR gene on ch
hromosomee
7. [4]

Hwa C
Chong Institu
ution 2013 9648 H2
H Biology / JC2
J Preliminaary Examinattion / Paper 3

470
9

[Total: 12]

Hwa Chong Institution 2013 9648 H2 Biology / JC2 Preliminary Examination / Paper 3

471
10

NAME: __________________________________ CT GROUP: 12S7 ___

BOOKLET III

STRUCTURED QUESTIONS (continued)

BLANK PAGE

Hwa Chong Institution 2013 9648 H2 Biology / JC2 Preliminary Examination / Paper 3

472
11

QUESTIO
ON 3

A new claass of RNA A, called microRNA


m ((miRNA), was
w discove
ered in 19993. These small RNA A
moleculess have an important roole in contrrolling the translation of
o mRNA. T This type of
o control iss
called RNAA interferen
nce.

Fig. 3.1 shows how miRNA is formed


f fromm a precurs sor RNA molecule
m thaat folds into
o a double--
stranded hairpin struucture. The e hairpin iss then proc cessed to give a shoorter molec cule by the
e
enzymes Drosha and d Dicer. Onne strand off this short molecule attaches to RRISC proteins and the e
resulting ccomplex bin
nds to target mRNA mo olecule.

structural gene miR


RNA gene DNA
A

transcriptio
on transcrription

mRNA miRNA precursor

Drosh
ha enzyme

DGCR8

nuclear nucleus
memmbrane

cytoplasm
m Dicer enz
zyme

ddouble-stran
nded
mmiRNA

RISC
R
one strand of miRNA
ribosome destroyed

RISC
R RISSC protein with
w miRNA
m
mRNA bin ds to mRNAA

Transla
ation ON Tra
anslation OFF

Fig. 3.1

Hwa C
Chong Institu
ution 2013 9648 H2
H Biology / JC2
J Preliminaary Examinattion / Paper 3

473
12

(a) Describe how the expression of the miRNA gene controls the expression of the structural
gene. [3]

(b) Researchers believe that cancer cells may have mutations in some of the miRNA genes.
With reference to proto-oncogenes, suggest how these mutations may have contributed to
the progression of cancer.
[2]

Recent research has investigated the importance of miRNA in controlling the fate of stem cells in
adult organisms. To determine the role of miRNAs in these processes, stem cells were modified to
knock out miRNA production. These miRNA knockout cells lack the protein DGCR8, an activator
of Drosha.

(c) State a role of stem cells in an adult organism. [1]

(d) Describe how the knockout of DGCR8 affects RNA interference. [2]

Hwa Chong Institution 2013 9648 H2 Biology / JC2 Preliminary Examination / Paper 3

474
13

In further work, the differentiatio


d on of knocko out and normal cells was
w studiedd by inducin ng the stem
cells to differentiate. Analysis was
w carried out on the levels of specific
s marrker molecuules whosee
presence is associate ed with either self-rene
ewal or diffe
erentiation. Results
R aree shown in Fig.
F 3.2.

Fig. 3.2

(e) (i) Describe how self-re


enewal in sstem cells allows
a for th
heir proliferration and subsequent
s t
ation.
differentia [2]

(ii) Account for


f the results obtained
d. [4]

Hwa C
Chong Institu
ution 2013 9648 H2
H Biology / JC2
J Preliminaary Examinattion / Paper 3

475
14

[Total: 14]

Hwa Chong Institution 2013 9648 H2 Biology / JC2 Preliminary Examination / Paper 3

476
15

NAME: __________________________________ CT GROUP: 12S7 ___

BOOKLET IV

PLANNING QUESTION

QUESTION 4

The yield of many cultivated varieties of crop plants such as the onion, Allium cepa, is reduced by
the presence of relatively high concentrations of salts, particularly sodium chloride, in the soil.
There are, however, salt-tolerant varieties of onion, which can be grown successfully in such
places. One possible explanation for this is that the fluid in the cell vacuoles of plants of the
tolerant varieties differs in solute potential from that of non-tolerant varieties. To measure the
solute potential of the cell vacuole, one can make use of the concept of incipient plasmolysis.
Incipient plasmolysis is the point where 50 % of plant cells are plasmolysed. At incipient
plasmolysis, the water potential of plant cells is equal to the solute potential as the pressure
potential is zero.

Plan an investigation to find out whether or not the solute potentials of the cell vacuoles of
epidermal cells of the bulbs of salt-tolerant onions differ from those of their non-tolerant counter
parts.

You are provided with the following equipment. Choose your equipment from this list. You may
not use any additional equipment.

an unlimited supply of 1.0 mol dm-3 sodium chloride solution


an unlimited supply of bulbs of two varieties of onion, one salt-tolerant and the other non-
salt tolerant
unlimited supply of distilled or deionized water
beakers of various sizes
glass specimen tubes
microscopic slides and coverslips
measuring cylinders of various sizes
graduated pipettes of various sizes, and pipette fillers
dropping pipettes
light microscope with low (x100), medium (x200) and high (x400) magnification and built in
illumination
wash bottle
marker pen
stopwatch
scalpel
forceps
glass rods for stirring
mounted needles
iodine in potassium iodide solution as a stain

Hwa Chong Institution 2013 9648 H2 Biology / JC2 Preliminary Examination / Paper 3

477
16

Your plan should:

have a clear and helpful structure such that the method you use is able to be repeated by
anyone reading it,
identify the independent and dependent variables,
describe the method with the scientific reasoning used to decide the method so that results
are as accurate and reliable as possible,
include layout of results tables and graphs with clear headings and labels,
use the correct technical and scientific terms,
include reference to safety measures to minimise any risks associated with the proposed
experiment.

[Total: 12]

Hwa Chong Institution 2013 9648 H2 Biology / JC2 Preliminary Examination / Paper 3

478
17

Hwa Chong Institution 2013 9648 H2 Biology / JC2 Preliminary Examination / Paper 3

479
18

Hwa Chong Institution 2013 9648 H2 Biology / JC2 Preliminary Examination / Paper 3

480
19

FREE RESPONSE QUESTION

Your answers must be in continuous prose, where appropriate.


Your answers should be illustrated by large, clearly labelled diagrams, where appropriate.
Write your answers in the writing paper provided.
BEGIN EACH PART ON A FRESH SHEET OF WRITING PAPER.
A NIL RETURN is required.

QUESTION 5

(a) Describe the polymerase chain reaction and explain one limitation of this procedure. [8]

(b) Explain the problems associated with the expression of eukaryotic genes in prokaryotes and
how these problems are overcome. [6]

(c) Discuss the benefits and difficult ethical concerns of the human genome project for humans.
[6]


--- END OF PAPER---

Hwa Chong Institution 2013 9648 H2 Biology / JC2 Preliminary Examination / Paper 3

481

HWA CHONG INSTITUTION


2013 JC2 H2 BIOLOGY
PRELIMINARY EXAMINATION MARK SCHEME

PAPER 3

QUESTION 1

(a) Outline how the isolated viral RNA can be converted into a double-stranded cDNA in step 1. [2]

1. Reverse transcriptase catalyses synthesis of the single-stranded cDNA using the isolated viral RNA
template
2. DNA polymerase converts the single-stranded cDNA into a double-stranded cDNA

(b) (i) Explain how the recombinant Ti plasmid in step 2 can be formed. [3]

1. The same restriction enzyme is used to cleave both the viral gene coding for TMV coat protein from the
cDNA formed in Step 1 and the Ti plasmid
2. to produce complementary sticky ends which facilitate annealing of / formation of hydrogen bonds
between the viral gene and linearized Ti plasmid
3. DNA ligase is added to covalently link / form phosphodiester bonds between the viral gene and linearized
Ti plasmid to produce a recombinant Ti plasmid

(ii) Explain the purpose of having a leaf-specific promoter in the recombinant Ti plasmid formed in step 2. [2]

1. The leaf-specific promoter will ensure that the viral gene coding for TMV coat protein is only expressed in
the leaves of transgenic plants
2. to ensure production of TMV coat protein

(c) Suggest another important role of Agrobacterium tumefaciens in the process of creating transgenic plants which
are resistant to TMV. [1]

1. Agrobacterium tumefaciens allows for in vivo amplification / production of numerous / multiple copies of
the recombinant Ti plasmid

(d) (i) Describe the functions of two other named ingredients in the growth media in the development of plant tissues.
[4]
Ingredient Use in developing tissue
1a. Carbon source, 1b. As respiratory substrate due to lack in autotrophic ability for proliferation of
e.g. glucose / sucrose cells and regeneration of shoots
2a. Plant growth 2b. Stimulates root growth by promoting cell growth & differentiation
regulators, auxins
3a. Plant growth 3b. Stimulates shoot growth by promoting cell growth & differentiation
regulators, cytokinins
4a. Amino acids 4b. Precursors for protein synthesis to stimulate cell growth
5a. Vitamins 5b. As catalysts / coenzymes / enzyme activators in various metabolic processes
6a. Antibiotics 6b. Prevent growth of systemic microbes / effectively control systemic infection

(ii) Explain the significance of the change of environment for the newly formed plantlets in the final stage of tissue
culture. [2]

1. The change of environment represents the shift from heterotrophic / sugar-requiring to autotrophic / free-
living condition
2. and the acclimatization of the plantlets to the outdoor environment before they develop into whole
transgenic plants

[Total: 14 marks]

HWA CHONG INSTITUTION / 2013 JC2 H2 BIOLOGY / PRELIMINARY EXAMINATION / MARK SCHEME 1
482

QUESTION 2

(a) State what is meant by genetic testing. [1]

1. identification of changes in chromosomes/ genes/ alleles/ DNA


2. to detect suspected genetic disease

(b) Using the information given in Fig. 2.1, explain why


(i) the three-day-old embryo will develop into a child affected by cystic fibrosis. [2]

1. reference to cystic fibrosis as a recessive condition


2. requiring two copies of mutant allele/ allele 2
3. Three-day-old embryo has same banding pattern as affected child

(ii) the position of allele 2 on the agarose gel indicates that it contains a deletion in comparison with allele 1. [2]

1. allele 2 is shorter / lighter / has a lower molecular mass than allele 1


2. allele 2 is able to migrate a longer distance / move faster through agarose gel

(c) Suggest why the child cannot pass on the normal CFTR allele to his offspring. [1]

1. tracheal epithelial cells are somatic cells / are not germ cells

(d) State one limitation of liposome-mediated gene therapy and suggest one improvement to this limitation. [2]

limitation:
1. transient expression of CFTR allele
2. CFTR allele delivered is not integrated into chromosome
3. short-lived/ transient nature/ repeated treatment needed
4. not specific in delivering CFTR allele into target cells

improvement:
1. Use retrovirus-mediated gene therapy
2. design liposomes carrying glycoproteins/glycolipids in their membranes that are recognised by cell
receptors

(e) Describe how RFLP analysis facilitated linkage mapping of the CFTR gene on chromosome 7. [4]

1. restriction sites occur more than once outside the CFTR locus, i.e. Taq I
2. point mutations that create / destroy the restriction sites
3. restriction digestion / cutting with Taq I produce unique patterns of restriction fragments respectively
4. each pattern / set of restriction fragment is an RFLP marker
5. determine the COV between each RFLP marker and the CF condition
6. the longer the distance between an RFLP marker and a particular CFTR allele, the higher the frequency
of crossing over
7. Application: to determine order of CFTR gene in relation to RFLP markers and relative distances
between the loci

[Total: 12 marks]

HWA CHONG INSTITUTION / 2013 JC2 H2 BIOLOGY / PRELIMINARY EXAMINATION / MARK SCHEME 2
483

QUESTION 3

(a) Describe how the expression of the miRNA gene controls the expression of the structural gene. [3]

1. Transcription of the miRNA gene gives a double-stranded miRNA precursor/ RNA folding onto itself/
forming a hairpin loop
2. Drosha cleaves/cut the free ends of the double stranded miRNA precursor and this is exported out of the
nucleus
OR
Dicer then cleaves/ cut the loop and gives a double stranded miRNA
Reject: Non-specific terms (e.g processed/ modify miRNA)

3. RISC binds to one of the miRNA strands


4. and becomes complementary to the mRNA of the structural gene, preventing its translation/
downregulating the expression of structural gene

(b) Researchers believe that cancer cells may have mutations in some of the miRNA genes. With reference to
proto-oncogenes, suggest how these mutations may have contributed to the progression of cancer. [2]

1. (i) Mutation of miRNA gene results in miRNA that are no longer complementary to target mRNA and thus
cannot bind to mRNA to down-regulate expression
OR
(ii) Mutation of miRNA gene results in miRNA that cannot form double stranded miRNA thus
Drosha/Dicer/RISC cannot bind to it
OR
(iii) Mutation of miRNA gene results in miRNA promoter of the miRNA is mutated resulting in no miRNA
being produced
Reject: Vague description (inactive/ non-functional miRNA etc) or reference to nonsense mutation resulting in
truncated miRNA.

2. hence leading to overexpression of the proto-oncogene products resulting in excessive proliferation

Reject: Ref to disruption of tumour-supressor gene or lack of ref to proto-oncogene.

(c) State a role of stem cells in an adult organism. [1]

1. maintain and repair the tissue in which they are found


2. replace cells that die because of injury or disease

(d) Describe how the knockout of DGCR8 affects RNA interference [2]

1. Drosha is inactivated
2. miRNA/precursor is not processed / not cleaved
3. RISC cant bind miRNA
4. RNA interference is reduced / translation is left on

(e)(i) Describe how self-renewal in stem cells allows for their proliferation and subsequent differentiation. [2]

1. By process of mitosis / symmetric divisions


2. to produce a pool of undifferentiated cells

3. Through assymmetric division


4. to generate committed precursors that then become committed to specific lineages

5. Differentiation occurs when precursor cells are exposed to different (internal and external) signals /
chemicals
6. and undergo differential gene expression
Note: point 1 and 2 are required to obtain full marks

(ii) Account for the results obtained. [4]

HWA CHONG INSTITUTION / 2013 JC2 H2 BIOLOGY / PRELIMINARY EXAMINATION / MARK SCHEME 3
484

Trend 1
T1a. In normal cells, self-renewal marker decreases from 1 unit (day 0) to 0.8 unit (day 8) to 0.5 unit (day 16)
T1b. Differentiation marker increases from 0.8 units (day 0) to 8 unit (day 8) to 16 units (day 16)

Explanation 1
E1a. This shows that self-renewal is switched off as differentiation proceeds
E1b. The inverse relationship is due to presence of miRNA which regulated the translation of mRNA of self-
renewal and differentiation markers
E1c. AVP

Trend 2
T2a. In KO cells, self-renewal marker increases from 1.5 unit (day 0) to 1.8 units (day 8) before decreasing to
1 unit (day 16)
T2b. Differentiation marker increases from 1.2 unit (day 0) to 4 units (day 8) before decreasing to 2 units (day
16)
OR
T2c. AVP

Explanation 2
E2a. The interaction of self-renewal and differentiation is abnormal in knockout cells
E2b. No relationship is observed due to the absence of miRNA, thus the loss of regulation in translation of
mRNA of self-renewal and differentiation markers
E2c. AVP
[Total=14 marks]

HWA CHONG INSTITUTION / 2013 JC2 H2 BIOLOGY / PRELIMINARY EXAMINATION / MARK SCHEME 4
485

QUESTION 4

1. Cell vacuoles of salt-tolerant onions are of more negative solute potential than that of non-tolerant onions.
2. Use the scalpel to cut 1 section of 2 cm by 1 cm epidermal tissue from salt-tolerant onion.
3. Using the forceps, put the tissue into 5 cm3 of 0.1 mol dm-3 NaCl solution in a glass specimen tube.
4. Incubate the tube for 15 minutes as timed using a stopwatch.
5. Remove the onion epidermal tissue and place it on a microscopic slide.
6. Add a drop of iodine in potassium iodide solution using a dropper.
7. Cover the sample with a coverslip using mounting needle and place the slide onto the microscope.
8. Count the number of cells undergoing plasmolysis in the first 30 cells seen in this field of view.
9. Repeat for the remaining salt-tolerant onion samples in other concentrations of NaCl solution and for non-
tolerant onion samples.
10. Calculate percentage of plasmolysed cells at each NaCl concentration.
11. The NaCl concentration at which there is incipient plasmolysis can be used to further determine the
respective solute potential.
12. Care must be taken when using the scalpel to avoid cutting oneself.
13. AVP

[Total: 12 marks]

HWA CHONG INSTITUTION / 2013 JC2 H2 BIOLOGY / PRELIMINARY EXAMINATION / MARK SCHEME 5
486

QUESTION 5

(a) Describe the polymerase chain reaction and explain one limitation of this procedure. [8]

Stage 1
1. DNA denatures / H bonds broken, resulting in separation of DNA strands
2. by heating to 95 C

Stage 2
3. Primers / oligonucleotides anneal / hybridize to single-stranded DNA template
4. by cooling to 54 C

Stage 3
5. new strands (of DNA) are synthesized from the position of the primers / amplification of target region
flanked by primers
6. by TAQ DNA polymerase
7. by heating to 72 C

General
8. entire 3 step cycle repeat 20 30 times in a thermocycler
9. PCR products serve as templates for further amplification

Any one pair:

L1a. Highly sensitive to contamination


L1b. hence non-target sequences can be amplified

L2a. Taq polymerase used in PCR lack proof-reading / 3 to 5 exonuclease activity


L2b. resulting in infidelity / inaccuracy of DNA replication in vitro / possible errors in DNA replication
L3a. Loss of Taq polymerase activity resulting in accumulation of inaccuracies in longer PCR products
L3b. Therefore, PCR products are shorter / are synthesised in smaller / limited amount

L4a. Specific primers required to selectively amplify target sequence


L4b. Hence, some prior sequence information is needed

L5. It can only be applied to amplify nucleic acids but not proteins

(b) Explain the problems associated with the expression of eukaryotic genes in prokaryotes and how these
problems are overcome. [6]

A. Presence of introns
PA1. Eukaryotic genes contain introns OR prokaryotic genes do not contain introns
PA2. There is no RNA splicing machinery OR introns are not removed in prokaryotes
PA3. pre-mRNA not converted into mature mRNA OR additional amino acid incorporated
SA1. cDNA for the gene of interest is synthesised using mRNA as a template via reverse transcriptase

B. Transcriptional initiation
PB1. Bacterial transcriptional machinery does not recognise eukaryotic promoter OR lack enhancers and
activators
PB2. hence eukaryotic gene is not expressed OR expressed at very low rates when it is placed in a prokaryote
host
SB1. Eukaryotic gene is placed under the control of a strong bacterial promoter to allow for expression of the
eukaryotic gene

C. Translational initiation
PC1. Eukaryotic mRNA lack a Shine-Dalgarno sequence
PC2. hence prokaryotic ribosomes / translation machinery cannot recognise the start site
SC1. Insert the DNA form of the Shine-Dalgarno sequence OR leader sequence
OR
different 5 UTR sequence in the 5UTR of the eukaryotic gene / upstream of gene of interest

D. Post-translational modifications
PD1. Prokaryotes do not have membrane-bound organelles
OR
have no / different post-translational modifications
HWA CHONG INSTITUTION / 2013 JC2 H2 BIOLOGY / PRELIMINARY EXAMINATION / MARK SCHEME 6
487

PD2. hence proteins are not folded into their native / specific 3D conformations/ glycosylation
SD1. Recombinant proteins have to be further purified and modified manually

(c) Discuss the benefits and difficult ethical concerns of the human genome project for humans. [6]

B1a. allows for early detection of mutant genes that causes disease / risk assessment for disease
B1b. screening allows for diagnosis of inherited mutations that predisposes them to specific diseases

B2a. provides insight into the types of protein encoded by genes


B2b. able to treat genetic and acquired diseases by gene therapy

B3a. allows genetic profile of an individual to be determined


B3b. customised drugs / molecular medicine / OWTTE can be designed

B4a. associated techniques eg. genetic testing / RFLP / DNA fingerprinting / DNA sequencing techniques
can be used
B4bi. in forensic science eg. analysis of DNA at crime scenes
OR
B4bii. to establish family relationships e.g. paternity testing
OR
B4biii. to assess risk of mutagenesis / health damage posed by exposure to radiation or carcinogens

B5a. Comparative genomics where the genes in the human genome project of unknown functions can be used
as comparison against known sequences of other organisms
B5b. for identification of gene functions in HGP

B6a. To elucidate evolutionary history


B6b to seek out relationship between species and to check for degree of relatedness eg: between the 3
domains of life

B7. AVP

E1a. Ownership of personal genetic information


E1b. Difficult to determine who owns and controls the genetic information or who should have access to it
OR
E1c. Legislation needed to ensure that there is no discrimination on the basis of genetic information; e.g. at
work or for health insurance

E2a. Commercialisation of products


E2b. Difficult to determine who own genes and other pieces of DNA
OR
E2c. To ascertain if patenting DNA will limit the accessibility and development into useful products

E3a. Use of genetic information in reproductive decision making and reproductive rights
E3b. Concerns that prenatal genetic testing could lead to genetic manipulation or a decision to abort based
on undesirable traits disclosed by the tests

E4a. Physiological impacts and stigmatisation


E4b. due to an individuals genetic differences, may affect the societys perceptions and treatment of an
individual

E5a. Clinical issues including the professional education of doctors and other health service providers and
patients; as well as public education
E5b. in genetic capabilities, scientific limitations, and social risks and implementation of standards and quality-
control measures in testing procedures

E6a. Treatment versus enhancement of humans


E6b. No clear distinction between medical treatment and enhancement

E7. AVP

[Total: 20 marks]

HWA CHONG INSTITUTION / 2013 JC2 H2 BIOLOGY / PRELIMINARY EXAMINATION / MARK SCHEME 7
488

INNO
OVA JUNIOR COLLLEGE
JC 2 PRELIMINARY E
EXAMINA
ATION II
in prep
paration for General Ce
ertificate of Education Advanced
A LLevel
Higher 2

CANDIDA
ATE
NAME

CLASS INDEX NUMBER

BIOLO
OGY 96
648/01
Paper 1 Multiple Choiice 225 Septem
mber 2013
1 hour 15 minutes
Additional Materials: Multiiple Choice Answer Sh
heet

READ THE
ESE INSTR
RUCTIONS FIRST

Write yourr name and class on all the work yyou hand in.
Write in so
oft pencil.
Do not usee staples, paper clips, highlighters
h s, glue or co
orrection fluid.

There are forty quesstions on this paper. A


Answer all questions.
q For
F each q uestion theere are fourr
possible answers A, B,
B C and D..
Choose thhe one you u consider correct and our choice in soft peencil on the separate
d record yo
Answer Sh heet.

Read the instruction


ns on the Answer
A eet very carefully.
She

Each corre
ect answer will
w score one mark. A mark will not
n be deduc
cted for a w
wrong answe
er.
Any rough working sh
hould be done in this bo
ooklet.

Calculatorss may be ussed.

This document consists o


of 27 printed
d pages and
d 1 blank paage.

Innova Junio
or College [Turn
[ over
2 489

1 Which molecules, found in cell surface membranes, contribute to cell recognition?

A Cholesterol and glycolipids


B Cholesterol and phospholipids
C Glycolipids and glycoproteins
D Phospholipids and glycoproteins

2 An enzyme is a globular protein held together by several different types of bonds,


giving the enzyme primary, secondary and tertiary levels of structure.
Which combination correctly summarizes the types of bond involved in each level of
structure?

Disulfide bonds Hydrogen bonds Ionic bonds Peptide bonds

A Primary Tertiary Secondary Tertiary


B Primary, tertiary Primary Secondary, tertiary Secondary
C Secondary Secondary Tertiary Primary
D Tertiary Secondary, tertiary Tertiary Primary

@INNOVA 9648/01/PRELIM II/2013 [Turn over


3 490

3 In an experiment using starch and amylase, the concentrations of starch and


maltose present in the reacting mixture are measured every minute for 20 minutes.
1% hydrochloric acid is added after 10 minutes and the mixture is heated to 60C at
14 minutes.
Which graph represents the results of this experiment?

A B

C D

@INNOVA 9648/01/PRELIM II/2013 [Turn over


4 491

4 The
e graph beelow show ws the rate of an enzzyme catalyzed reacction with increasing
sub
bstrate con
ncentration arried out at 37oC annd a pH of
n. The reacction is ca o 7 for all
sub
bstrate concentrations.

Which of the following(s


f s) would re
esult in a de
ecrease in the rate of reaction at
a W?

1 Decrea
ase in enzy
yme conce
entration
2 Decrea
ase in temp o 27oC
perature to
3 Increasse in pH to
o9
4 Additio
on of competitive inhi bitor

A 1 and 4 on
nly
B 2 and 3 on
nly
C 2 , 3 and 4 only
D 1, 2, 3 and
d4

@INNOVA
A 9648
8/01/PRELIM II/2013 [Turn ov
ver
5 492

5 The
e diagram shows
s a se
ection of a cell surfac
ce membra
ane.

What causes the phospholipid mo


olecules to be arrange
ed as show
wn?

A The headss are hydro


ophilic and point towa
ards waterr moleculess.
B The headss are hydro
ophobic an
nd point aw
way from water
w moleccules.
C The tails are
a hydroph
hilic and po
oint away from
f waterr moleculess.
D The tails are
a hydroph
hobic and point towa
ards water moleculess.

6 Which pair(s) of factors is inverselly proportio


onal to the rate of diffffusion?

1 Concentration gradient and


d surface area
a over which
w diffussion occurs
s.
2 Distancce over wh
hich diffusio
on occurs and size of
o diffusing molecule.
3 Size off diffusing molecule
m a
and concen
ntration gra
adient.
4 Surface
e area ove
er which d
diffusion oc
ccurs and distance oover which
h diffusion
occurs.

A 2 only
B 1, 2 and 4 only
C 2, 3 and 4 only
D All of the above
a

@INNOVA
A 9648
8/01/PRELIM II/2013 [Turn ov
ver
6 493

7 The
e figure bellow shows 6 stages o
of meiosis occurring in a plant ccell, where
e 2n = 18.

Wha
at is the co er of these 6 stages?
orrect orde

A (iii), (v), (ii)), (vi), (iv), (i)


B (iii), (i), (v)), (ii), (vi), (iv)
(
C (ii), (iii), (i), (v), (vi), (iv)
(
D (i), (iii), (v)), (ii), (vi), (iv)
(

8 A ccell contain
ning 92 chrromatids a
at metapha
ase of mito
osis wouldd produce two
t nuclei
eacch containinng ____ ch
hromosom es?

A 92
B 46
C 23
D 16

@INNOVA
A 9648
8/01/PRELIM II/2013 [Turn ov
ver
7 494

9 Cancer is a multi-step process. Some of the stages in cancer are listed.

1 Mutated cell undergo uncontrolled cell division by mitosis.


2 Formation of tumour.
3 Mutations in tumour suppressor genes and proto-oncogenes.
4 Cells from the primary tumour metastasize to other sites via the bloodstream.

What is the correct sequence of events?

A 1432
B 1234
C 3124
D 3214

10 Cells taken from a human bone cancer multiplied readily in culture. Analysis showed
that the cells were unable to produce the protein, RB. Addition of RB to these cells
reduced their rate of division.
Which of the following statement accounts for the observation?

A Both chromosomes in the cancer cell carry alleles for tumour suppressor gene.
B Both chromosomes in the cancer cell have the allele for tumour suppressor gene
deleted.
C Both chromosomes in the cancer cell carry alleles for proto-oncogene.
D Both chromosomes in the cancer cell have the allele for proto-oncogene deleted.

@INNOVA 9648/01/PRELIM II/2013 [Turn over


8 495

11 A cculture of bacteria
b ha NA labelled with the heavy isottope of nitrrogen, 15N.
ad all its DN
14
The w then allowed to rreproduce using nucleotides coontaining normal
e culture was n N.
Thee DNA wass examined using a centrifuge after one generationn and again after two o
gennerations.
Thee diagram shows the e position of the DNA band at Z in the ccentrifuge tube when
n
the
e DNA was first labelled.

In w
which patttern would
d the DNA
A be found after the first and aafter the second
s celll
gennerations?

After first
f genera
ation Afte
er second generation
g

A Half att X and hallf at Y Quarter at


a X and at Z and halff at Y
B Half att X and hallf at Z Quarter at
a X and at Z and halff at Y
C All at X Half at X and half at Y
D All at Y Half at X and half at Y

12 hich of the following diagrams


Wh d iss the structure of a ribose sugaar?

A B

C D

@INNOVA
A 9648
8/01/PRELIM II/2013 [Turn ov
ver
9 496

13 Wh
hich of the
e following
g statemen
nt is true of the end
d-replicatioon problem
m of linear
DN
NA?

1 When a linear DNA


D moleccule replica
ates, a ga
ap is left aat the 3 en
nd of each
h
new sttrand because DNA ppolymerase can only add nucleeotides to a 5 end.
2 Telome
erase prev
vents the e
end-replication proble
em from occcurring.
3 Repea
ated rounds
s of replica
ation produ
uce shorterr and shortter DNA molecules.
4 Prokarryotes do not
n have th
he end-replication pro
oblem.

A 1 and 2
B 1 and 4
C 2 and 3
D 3 and 4

14 If th
here were
e 35 aminoo acids an
nd DNA on
nly contain
ned three ttypes of nitrogenous
n s
basses, what would
w be the
t minimu um numbeer of bases
s per codonn that could code for
prooteins?

A 3
B 4
C 6
D 8

15 The
e RNA trip plet UAG acts as a stop codon which terminatess the syntthesis of a
polyypeptide. The
T diagraam shows a strand off DNA whic
ch codes foor four amino acids.
In w
which position will an
n insertion of a thymine nucleo
otide resultt in the term
mination of
tran
nslation?

A B C D

@INNOVA
A 9648
8/01/PRELIM II/2013 [Turn ov
ver
10 497

16 Rib
bonucleasee is an ennzyme tha at digests RNA. The
e first five amino ac
cids of the
e
functioning molecule
m of ribonuclea
ase are
lys glu thr ala ala
The e mRNA of
o the gene
e coding fo
or ribonucle
ease, for th
he first 15 nucleotide
es, has the
e
follo
owing sequence.
AU G A A GG AA ACUG C U
Ag
genetic cod
de, showing mRNA ccodons, is shown
s belo
ow.

Wh
hich event occurs
o to explain
e the
e informatio
on given above?

A There is no tRNA with an anticcodon com


mplementarry to the firsst codon.
B The first codon is removed from the mRNA tra
anscript inn post-tran
nscriptiona
al
modificatio
on.
C The mRNA
A binds to the rRNA in the seco
ond codon position.
D The first amino
a acid
d on the p
polypeptide
e chain is removed in post-trranslationa
al
modificatio
on.

@INNOVA
A 9648
8/01/PRELIM II/2013 [Turn ov
ver
11 498

17 Which of the following statements are correct?

1 A mutation in an allele occupying a gene locus may result in a change in the


individuals phenotype.
2 Gene mutations always give rise to recessive alleles.

3 Chromosomal mutations can give rise to new alleles.

4 Aneuploids have extra chromosomes in their genome.

A 1 only

B 1 and 4

C 1, 3 and 4

D 2, 3 and 4

@INNOVA 9648/01/PRELIM II/2013 [Turn over


12 499

18 The diagram below shows the reproductive cycle of the herpes virus which causes
cold sores on the mouth.

With reference to the diagram above, which of the following statements best
describes the herpes virus?

A It is not a retrovirus as it contains DNA as its genetic material.

B Its mode of replication is similar to that of HIV.

C Its replication cycle includes a lysogenic phase.

D Death of the host cell is necessary for the release of the viral progeny.

@INNOVA 9648/01/PRELIM II/2013 [Turn over


13 500

19 The diagram represents the structure of the human immunodeficiency virus (HIV).

Which combination correctly identifies the chemical constituents of the lettered


components?

V W X Y Z

amino acids
A amino acids and lipids nucleotides amino acids
carbohydrates
amino acids
B amino acids lipids amino acids amino acids and
carbohydrates
amino acids
C and lipids amino acids nucleotides amino acids
carbohydrates
amino acids amino acids
D and amino acids lipids nucleotides and
carbohydrates carbohydrates

@INNOVA 9648/01/PRELIM II/2013 [Turn over


14 501

20 The e diagram below sho ows 2 spe ecies of ba


acteria. The
e bacteria that are shaded
s are
e
ressistant to th
he antibiotic penicillin
n.

Wh
hich one off the follow
wing statem ely to be true?
ments is like

A Bacterium
m D is resistant to pe ue the transfer of thee complete F plasmid
enicillin du d
and peniccillin resista
ant gene th
hrough transduction involving B Bacterium A.
B Bacterium
m D is resiistant to pe
enicillin du
ue to horizontal genee transfer involving a
bacteriop
phage and Bacterium A.
C Bacteriumm E acquireed resistan
nce to pen
nicillin as a result of rrandom mu
utation and
d
genetic trransformation.
D Bacteria E and F are resisttant to pe
enicillin as a result of conjug
gation from
m
m D.
Bacterium

@INNOVA
A 9648
8/01/PRELIM II/2013 [Turn ov
ver
15 502

21 The
e diagram shows som
me of the ffactors affe
ecting the lac operon .

Whhich statem
ment descrribes the ro
ole of the regulator
r protein
p in tthe expres
ssion of the
e
lac operon?

A Activated regulator binding sta


abilizes the
e transcriptional com
mplex.
B Activated regulator binding to the operattor increas
ses mRNA formation.
C Inactive regulator
r in
nhibits tran scription.
D Regulatorr protein is
s activated when ATP
P level is high.

@INNOVA
A 9648
8/01/PRELIM II/2013 [Turn ov
ver
16 503

22 The
e diagram shows a gene in a h uman cell.

In o
one form of
o the bloo
od conditio
on thalassa
aemia, there is a muutation in an
a intron of
o
the
e haemogloobin gene.
Wh
hich stage of
o gene ex
xpression w
would this mutation directly
d affeect?

A Activation
n of the gene by a ste
eroid horm
mone
B Attachme
ent of mRN
NA to a ribo
osome
C Binding of
o RNA poly
ymerase to
o DNA
D Splicing of
o pre-mRN
NA

23 Thee following
g diagram shows th he express sion of a particular gene to its proteinn
pro
oduct in a eukaryotic
e cell. Whi ch of the following
f combination
c n correctly
y describess
steps 1 4?

1 2 3 4

RNase does not


Histones are 5 cappping Initiation
n factors
A degrade 5
5 capped
acetyylated occuurs bind to ribosome
r
mRNNA
Histones are Poly(A) tail is Alternative
e splicing Initiation
n factors
B
methyylated added to 3 end occuurs bind to ribosome
r
Activatorr binds to Intronss are Initiation factors Ribosomme binds
C
enhaancer removved bind to rib
bosome to 5 UTR
Represssor does
Intronss are Initiation
n factors
D not biind to 5 capping
g occurs
removved bind to ribosome
r
operrator

@INNOVA
A 9648
8/01/PRELIM II/2013 [Turn ov
ver
17 504

24 The
e pedigree
e chart belo
ow shows tthe inherita
ance of a genetic
g dissease in a family.
f

Wh
hat is the nature of the allele tha
at causes this
t diseas
se?

A dominantt and sex linked


B dominantt and non-s
sex linked
C recessive
e and sex linked
D recessive
e and non-s
sex linked

@INNOVA
A 9648
8/01/PRELIM II/2013 [Turn ov
ver
18 505

25 The statements are descriptions of aspects of genetics.

1 The phenotype is affected by both alleles at the same locus of a


heterozygous individual.
2 The combined effects of alleles at two or more gene loci equal the sum of
their individual effects.
3 Many different alleles present in a gene pool can occupy the same gene
locus.
4 Alleles of one gene mask the effects of the alleles of another gene at a
different locus.

Which of the following correctly describes the statements?

codominant alleles epistatic alleles multiple alleles additive genes

A 1 4 2 3
B 1 4 3 2
C 2 1 4 3
D 3 1 4 2

26 Which one of the following is a correct outline of the main events in photosynthesis?

A Carbon dioxide combines with an acceptor compound and this is reduced by


hydrogen split from water by light.
B Light joins carbon dioxide to an acceptor compound which is then reduced by
hydrogen obtained from water.
C Light splits carbon dioxide and the resulting carbon then combines with oxygen
and hydrogen obtained from water.
D Light splits water and the resulting hydroxyl group combines with a compound
which has incorporated carbon dioxide.

@INNOVA 9648/01/PRELIM II/2013 [Turn over


19 506

27 The
e diagram shows the e reversible
e dehydrog
genase co
onversion oof pyruvate
e to lactate
e
by tthe enzym
me lactate dehydrogen
d nase.

Whhat would be
b the effe
ect of inhibiition of lac
ctate dehyd
drogenase in a mammalian cell
und
der anaero
obic conditiions?

A Decrease
e in cell pH
H, due to th
he accumulation of lactic acid.
B Decrease
e in glycoly
ysis, due to
o the lack of
o NAD.
C Increase in ATP pro
oduction, d
due to incre
eased amo
ounts of redduced NAD
D.
D Increase in activity of
o the Kreb
bs cycle, due to incre
eased amoounts of py
yruvate.

28 Froom yeast cells, a mixture


m off enzymess can be obtained that cata alyses the
e
con nversion off glucose to carbon dioxide an
nd ethanol. Pyruvatee is produced during
g
thiss process.
Whhich combination of substances
s s would increase the
e rate of eethanol pro
oduction byy
the
ese enzyme
es?

A ATP and lactate


B ATP and NAD
C NAD and coenzyme
eA
D NAD and lactate

29 Whhich of the
e following
g statemennts best describes
d the outcoome of the
e negative
e
fee
edback loop
p in a homeostatic m
mechanism??

A Blood glu
ucose level increasess to the no
ormal level in responnse to the release of
o
glucagon.
B Glucagonn is released by the pancreas in respon
nse to a drrop in bloo
od glucose
e
concentra
ation below
w the norm
mal level.
C Release of glucago on is stop pped in re
esponse to
o blood gllucose con
ncentration
n
returning to the norm
mal level.
D The liver cells break
k down glyycogen to glucose
g in response tto glucago
on.

@INNOVA
A 9648
8/01/PRELIM II/2013 [Turn ov
ver
20 507

30 Homeostasis often involves multi-step pathways. An example is given in the diagram


below, which shows the hormonal feedback mechanism that controls body
temperature. [TRF = thyrotrophin-releasing factor ; TSH = thyroid-stimulating
hormone]

(-)
hypothalamus

TRF

(-)
anterior pituitary
gland

TSH

thyroid gland

Thyroxine

Increases
metabolic rate

A rise in temperature causes a decrease in the amount of TRF released by the


hypothalamus. As a result of this, the level of thyroxine will

A result in the inhibition of TRF production.


B result in the inhibition of TSH production.
C rise with a consequent fall in the basic metabolic rate.
D fall with a consequent fall in the basic metabolic rate.

@INNOVA 9648/01/PRELIM II/2013 [Turn over


21 508

31 An experiment was performed on laboratory mice to test the effects of bisphenol A


(BPA), a chemical commonly used to make polycarbonate bottles. BPA was fed
orally to the mice and their serum glucose levels were tested at various time
intervals.
The graph shows the results of the experiment.

Based on the results shown, which of the following cannot be a long-term effect of
BPA?

A BPA-treated animals will secrete more insulin than the control.


B Functions of cells of the islets of Langerhans will be affected.
C Hypoglycaemia will develop with long term use of BPA.
D Insulin resistance will develop.

@INNOVA 9648/01/PRELIM II/2013 [Turn over


22 509

32 The
e diagram shows some bioche emical path
hways in a liver cell . Some off the pointss
whe
ere hormones affect the pathw belled 1 to 5.
ways are lab

At w
which poin
nts would the
t hormon
ne insulin accelerate
a the pathw
ways in the
e directionss
indicated?

A 1 and 2
B 2 and 5
C 1, 2 and 5
D 2, 3 and 4

33 Ap particular tyype of auto


oimmune ddisease ca auses antib
bodies to ddestroy ac
cetylcholine
e
recceptors of neurons. What
W effecct will this have on the functiooning of th
he nervouss
sysstem?

A Presynapptic neurons will be u nable to re


elease neu
urotransmittters into th
he synapticc
clefts.
B Neurons will
w be una
able to prop
pagate acttion potentials along their axons.
C Depolarizzed neuron
ns will be u
unable to reestablish
r h an ionic gradient across
a theirr
membran nes.
D Postsynaptic neurons will be u
unable to detect
d sign
nals from ppresynaptic
c neurons.

@INNOVA
A 9648
8/01/PRELIM II/2013 [Turn ov
ver
23 510

34 Tetrodotoxin, a puffer fish toxin, blocks voltage-gated sodium channels. Black widow
spiders venom causes the voltage-gated calcium channels to be constantly open.
Crotoxin binds irreversibly to acetylcholine receptors.
What will happen to the nerve transmission if each toxin is applied?

black widow spiders


tetrodotoxin crotoxin
venom

block action potentials reduce transmission of increase transmission of


A
along axon impulse across synapse impulse across synapse
block action potentials increase transmission of reduce transmission of
B
along axon impulse across synapse impulse across synapse
increase transmission of reduce transmission of block action potentials
C
impulse across synapse impulse across synapse along axon
reduce transmission of block action potentials increase transmission of
D
impulse across synapse along axon impulse across synapse

35 In the summer of 1995, at least 15 iguanas survived Hurricane Marilyn on a raft of


uprooted trees. They rode the high seas for a month before colonizing the
Caribbean island, Anguilla. These few individuals were perhaps the first of their
species, Iguana iguana, to reach the island.

When one such healthy iguana was mated with another from its source of origin in
the Virgin Islands, the offspring were infertile. Which of the following best explains
this?

A Allopatric speciation
B Artificial selection
C Diversifying selection
D Founder effect

@INNOVA 9648/01/PRELIM II/2013 [Turn over


24 511

36 The graph below shows the distribution of huntsman spiders at a forest boundary:

One species of huntsman spider (Isopeda isopedella) varies in body colour from
dark brown to light grey. In one community at the forest boundary, two populations of
this species were found. Some were found living in the leaf litter inside the forest and
others were found living in the grass just outside the forest. The relative numbers of
dark brown adult spiders and light grey adult spiders found at certain distances from
the forest boundary are shown in the graph above.

The best explanation for this distribution is that the

A differences in the two habitats had changed the physical appearance of


individual spiders.
B particular body colour provided a selective advantage to spiders in a particular
habitat.
C two populations of spiders were once different species.
D two populations of spiders were unable to interbreed and individuals were
adapting to suit their habitats.

@INNOVA 9648/01/PRELIM II/2013 [Turn over


25 512

37 In separate reactions, PCR was performed using a pair of primers with sequences
complementary to parts of separate exons of a gene suspected to be involved in a
genetic disease.
Using genomic DNA and RNA isolated from healthy (wild-type) and diseased cells,
separate PCR reactions were done. The PCR products were analysed on
polyacrylamide gels. The sizes of the bands in the molecular weight marker (M) are
indicated in base-pairs.

GENOMIC RNA

Which of the following best explains the results obtained?

A Deletion of an exon
B Deletion of several introns
C Deletion of several codons
D Deletion of a splice site

@INNOVA 9648/01/PRELIM II/2013 [Turn over


26 513

38 As part of the procedure to produce recombinant proteins in E. coli, you are asked to
insert the gene encoding for the Bt toxin protein into the pSEK-T vector. The
restriction sites and selectable markers on the vector are shown below.

Nah I
Ptz II
Wyn II
AdrL II
Oli II
Ampr lacZ Eva II
Multiple Chk II
pSEK-T cloning site AriC II
Vector
AcmI III

If the gene for Bt toxin were to be inserted into the multiple cloning site, what should
be added to the agar plate in order to screen for recombinant clones and how would
the recombinant clones appear?

chemicals colour of colonies

A ampicillin X-gal blue


B ampicillin X-gal white
C -galactosidase X-gal blue
D -galactosidase lactose white

@INNOVA 9648/01/PRELIM II/2013 [Turn over


27 514

39 One therapy to treat -thalassaemia is to transplant bone marrow cells from a


genetically compatible donor into a patient. A potential gene therapy involves adding
the normal, dominant allele for -globin to the patients cells.
Which of the following would ensure that the normal gene is passed on to the next
generation?

A Using a retrovirus to introduce the normal -globin gene into bone marrow cells.
B Using a retrovirus to introduce the normal -globin gene into an egg cell.
C Using an adenoviral vector to introduce the normal -globin gene into bone
marrow cells.
D Using an adenoviral vector to introduce the normal -globin gene into an egg
cell.

40 Which genetic modification could increase the yield of crop measured as mass of
crop produced per unit area per year?

A Delayed ripening in the fruits


B Herbicide resistant plants
C More essential amino acids in the seeds
D More vitamin A in the grain

@INNOVA 9648/01/PRELIM II/2013 [Turn over


28 515

BLANK PAGE

@INNOVA 9648/01/PRELIM II/2013


2013 JC2 Prelim I ANSWER KEY 29 516

Qn Answer Qn Answer
1 C 21 A
2 D 22 D
3 C 23 A
4 C 24 B
5 A 25 B

6 A 26 A
7 D 27 B
8 B 28 B
9 C 29 C
10 B 30 D

11 D 31 C
12 C 32 C
13 D 33 D
14 B 34 B
15 B 35 A

16 D 36 B
17 B 37 D
18 A 38 B
19 C 39 B
20 B 40 B

@INNOVA 9648/01/PRELIM II/2013


517

INN
NOVA JU
UNIOR COLLEGEE
C 2 PRELIMINARY
JC Y EXAMIN
NATION II
in preparation
p for Genera l Certificate
e of Educatio
on Advanceed Level
Hig
gher 2

CANDID
DATE
NAME

CLASS INDEX NUMBER

BIOL
LOGY 9648/02
Paper 2 Core Pape
er 177 Septemb
ber 2013
2 hours
Addition
nal Materialls: Answerr Paper
Cover Page
P

READ T
THESE INS
STRUCTION
NS FIRST
F
For Examin
ners Use
Write yoour name and
a class on n all the worrk you handd in.
Write in o black pen on both s ides of the paper.
n dark blue or Seection A
You ma ay use a soft
s pencil for any dia agrams, grraphs or roough
1
workingg. 10
0
Do not use staple es, paper clips,
c highlig
ghters, glue
e or correc
ction
2
fluid. 14
4
3
nA
Section 16
6
Answerr all questio
ons.
4
14
4
nB
Section
5
Answerr any one question.
q 15
5
6
At the end of the examinatio on, fasten aall your wo
ork securelyy 11
1
togethe
er. Seection B
The nummber of maarks is given
n in the bra
ackets [ ] at the end of
each qu
uestion or part
p question n. 7/8
20
0

T
Total
100
0
Perrcentage
G
Grade

This document
d cconsists of 18
1 printed pages.
p

Innova Ju
unior College
e [Tu
urn over
518
2 For
Examiners
Use

Section A
Answ
wer all ques
stions.

1 Fig. 1 is an electron micrograph


m of a cell fo
ound in the
e pancreass.

Fig. 1

a) Describe how org


(a ganelle X iis adapted for genetic expressioon.

[2]

b) Describe how in
(b nsulin, afte
er being synthesized
s d by ribossomes on Y, is
ed out of th
exporte he cell.

[4]

OVA
@INNO 9648/02
2/PRELIM II/20
013 n over
[Turn
519
3 For
Examiners
Use

c) When insulin is released


(c r in
nto the bloodstream, it exerts m
many effec
cts on
carboh
hydrate, lip
pid and pro
otein metabbolism.

One of its effectts is to acctivate the enzyme glycogen


g ssynthase, which
w
functio
ons to polym
merize glu cose mono omers into
o glycogen..

(i) Explain ho ow the strructure of glycogen enables itt to perforrm its


function in
n animals.

[2]

(ii) Describe two


t ways in which th
he structure of glycoggen differs
s from
the structu
ural polysa
accharide fo
ound in pla
ant cell waalls.

[2]

[Tota
al: 10]

2 Fig. 2.1 sho


ows the structure of a protease enzyme fo
ound in a re
retrovirus.

Fig 2.1

OVA
@INNO 9648/02
2/PRELIM II/20
013 n over
[Turn
520
4 For
Examiners
Use

(a
a) (i) With referrence to FFig. 2.1, sta
ate the ev
vidences thhat the enz
zyme
has both a tertiary a nd quatern
nary protein structuree.

[2]

(ii) Explain what


w determ
mines the 3-dimensional confiiguration of
o the
enzyme.

[2]

b) The ra
(b ate of reac
ction of H
HIV proteas
se may be
e decreassed by diffferent
inhibito
ors.

Fig. 2.2 shows th he effect o


of substrate
e concentrration on thhe initial ra
ate of
an enzzyme catalysed reacction with and without the adddition of a non-
compe etitive inhib
bitor.

Fig 2.2
2

(i) On Fig. 2.2


2 draw a and label thet curve you wouldd expect in the
presence of a comp
petitive inhiibitor. [1]

OVA
@INNO 9648/02
2/PRELIM II/20
013 n over
[Turn
521
5 For
Examiners
Use

(ii) Describe and explain the difference in the shape of the curve you
have drawn and that of the non-competitive inhibitor.

[3]

(c) A similar protease is found in blackflies. A proteinaceous toxin found in


insecticides used to control its population can be activated by this
protease.

The active toxin binds to receptor molecules on the larvaes gut


epithelial cell membranes. This damages the epithelial cells and kills the
larvae.

Fig. 2.3 shows the results of an experiment to determine the optimum


pH for activation of the toxin.

Fig. 2.3

@INNOVA 9648/02/PRELIM II/2013 [Turn over


522
6 For
Examiners
Use

(i) With referrence to F Fig. 2.3, describe th


he effect oof pH on mean
m
activity of the toxin.

[2]

(ii) Explain wh
hy there iss no activity
y of toxin when
w pH faalls below 8.5.
8

[2]

(iii) Suggest how


h the pro
otease in larvas
l digestive systtem cause
es the
toxin to be
e activated .

[2]

[Tota
al: 14]

3 Fig. 3.1 sho


ows a T4 bacterioph h has been modifiedd to contain the
hage which
D
DNA of Mu bacterioph
hage, a knoown tempe
erate phage.

Fig. 3.1

OVA
@INNO 9648/02
2/PRELIM II/20
013 n over
[Turn
523
7 For
Examiners
Use

(a) Suggest how this bacteriophage attaches to the correct host cell.

[2]

(b) Outline the main events which occur following the entry of the phage
DNA into the host cell until new phage particles are released.

[4]

@INNOVA 9648/02/PRELIM II/2013 [Turn over


524
8 For
Examiners
Use

While the T4 ba acteriophagge infects bacterial cells, the influenza virus


infectss cells of the respirato
ory tract in animals.

H1N1 influenza virus


v cause es swine fllu in pigs. In a numb er of instances,
people
e have dev veloped th he swine flu
f infection when thhey are closely
associated with pigs,
p for exxample farm
mers and porkp proceessors.

Figure 3.2 shows


s a H1N1 vvirus.

M-protein neeuraminidas
se

envvelope enzyme Q

ribonu
ucleoprotein
n hemaggluttinin

Fig. 3.2

c) Describe the significance o


(c of enzyme
e Q to the reproductiive cycle of
o the
H1N1 virus.
v

[3]

OVA
@INNO 9648/02
2/PRELIM II/20
013 n over
[Turn
525
9 For
Examiners
Use

(d) A person is infected with the H1N1 influenza virus but does not develop
the symptoms. Examination shows that the persons immune system
has produced an antibody that binds to the haemagglutinin molecules.

(i) Suggest why the person does not develop influenza-like


symptoms.

[2]

(ii) After a few days and many viral generations, the person begins to
develop influenza symptoms. Tests for the antibody referred to
above show that it is still present in the blood.

Account for the observations and comment on the implication this


has for developing H1N1 influenza vaccines.

[2]

(e) Contrast the reproductive cycles of the H1N1 influenza virus and Human
Immunodeficieny Virus (HIV).

[3]

[Total: 16]

@INNOVA 9648/02/PRELIM II/2013 [Turn over


526
10 For
Examiners
Use

4 O
Oxytocin (OOT) is a neuropeptidee secreted from the posterior
p ppituitary gla
and in
th
he brain an
nd is best known
k for iits role in mammalian
m n birth and lactation.

In
n response e to a varriety of stim
muli such as sucklin
ng, parturiition, or ce
ertain
kiinds of strress, the processed d OT pepttide is rele
eased fromm the pos sterior
pituitary into
o the syste
emic circulaation.

The OT re eceptors activity


a is mediated by G prroteins whhich activa
ate a
phosphatidyylinositol-calcium seccond mess
senger systtem.

Fig. 4.1

a) The blo
(a ood brain barrier
b is sset by the endothelial
e cells liningg the vessels in
the bra
ain.

Suggest why the ese cells ca


an prevent oxytocin secreted
s froom the gla
and to
er the brain
re-ente n.

[2]

OVA
@INNO 9648/02
2/PRELIM II/20
013 n over
[Turn
527
11 For
Examiners
Use

(b) With reference to Fig. 4.1, describe and explain how a molecule of
oxytocin can lead to signal amplification leading to cellular responses.

[5]

@INNOVA 9648/02/PRELIM II/2013 [Turn over


528
12 For
Examiners
Use

-Amminobutyricc acid (GABA) is tthe chief inhibitory neurotrannsmitter inn the


mam mmalian ce entral nerv
vous syste
em involving maturee neuronss. During foetal
f
deveelopment, the
t release e of GABA
A leads to an
a excitato
ory post-syynaptic potential
in im
mmature neeurons.

Durinng labour, findings shown


s tha
at high leveels of matternal oxyttocin inducced a
switcch in the action
a of GABA
G fromm excitatoryy to inhibitory in the neurons of
o the
foetu
us at birth hence incrreases theiir resistanc
ce to dama age during delivery.

Fig. 4.2 showss the gene


eration of the post-s
synaptic po
otentials inn immature
e and
matuure neuron
ns.

Fig. 4.2

c) Describe and explain how


(c w GABA is release ed from thhe pre-syn naptic
ne to the post-syna ptic neuro
neuron one to result in an inhibitory post-
synapttic potentia
al.

[4]

OVA
@INNO 9648/02
2/PRELIM II/20
013 n over
[Turn
529
13 For
Examiners
Use

(d) With reference to Fig. 4.2, explain how GABA could have differing
effects on the post-synaptic membrane in different neurons.

[3]

[Total: 14]

5 Nail-patella syndrome (NPS) is a genetic disorder that results in small, poorly


developed nails and kneecaps, but can also affect many other areas of the
body, such as the elbows, chest, and hips. The genetic linkage between the
NPS and ABO blood group loci was the third to be identified in humans.

The NPS gene codes for a transcription factor and a gain-of-function


mutation in one of its alleles leads to NPS.

An unaffected man with blood group O marries an affected female with blood
group B. Fig. 5 shows the resultant pedigree.

Fig. 5

@INNOVA 9648/02/PRELIM II/2013 [Turn over


530
14 For
Examiners
Use

(a) With reference to Fig. 5,

(i) State and explain the mode of inheritance of NPS with respect to
the ABO locus.

[3]

(ii) Explain why the mating between individual 6 and 7 in generation II


produces 100% unaffected offsprings.

[4]

(b) (i) State the genotype of the individuals 5 and 8 in Generation II with
respect to both the NPS and ABO loci.

[2]

@INNOVA 9648/02/PRELIM II/2013 [Turn over


531
15 For
Examiners
Use

(ii) Suggest reasons for your answers in (i).

[3]

(c) Apart from the factors due to the inheritance of genes, environment
plays a very important role in determining phenotype of certain
organisms.

Explain, with one example, how the environment may affect the
phenotype.

[3]

[Total: 15]

@INNOVA 9648/02/PRELIM II/2013 [Turn over


532
16 For
Examiners
Use

6 Cranes are large birds and one of the endangered species, Grus americana
commonly known as the Whooping crane is the tallest North American bird
which faced near extinction due to unregulated hunting and loss of habitat.

Cranes can be categorised based on their behaviours and one of the earliest
methods of classifying cranes was based on the calls they make during the
breeding season.

(a) Suggest why biologists could use calls to investigate relationships


between different species of crane.

[3]

More recently, biologists have used DNA hybridisation to confirm the


relationships between different species of crane. They made samples of
hybrid DNA from the same and from different species. They measured
the percentage of hybridisation of each sample. The results are shown
in Table 6.

Table 6

Species of crane from which hybrid DNA was Percentage DNA


made hybridisation/%

Grus americana and Grus monachus 97.4


Grus monachus and Grus rubicunda 95.7
Grus americana and Grus rubicunda 35.5

Grus rubicunda and Grus rubicunda 99.9

Grus americana and Grus americana 99.9

Grus monachus and Grus monachus 99.8

@INNOVA 9648/02/PRELIM II/2013 [Turn over


533
17 For
Examiners
Use

(b) Describe the structure of the hybrid DNA between the same species.

[4]

(c) With reference to Table 6, which of the two species seemed to be the
most closely related? Explain your answer.

[2]

(d) The biologists measured the temperatures at which the samples of


hybrid DNA separated into single strands.

Explain why these temperatures could be used to find the percentage of


DNA hybridisation.

[2]

[Total: 11]

@INNOVA 9648/02/PRELIM II/2013 [Turn over


534
18

Section B
Answer one question

Write your answers on the separate answer paper provided.


Your answers should be illustrated by large, clearly labelled diagrams, where
appropriate.
Your answers must be in continuous prose, where appropriate.
Your answers must be set out in sections (a), (b) etc., as indicated in the question.

7 (a) Describe the eukaryotic processing of pre-mRNA in terms of intron splicing,


polyadenylation and 5capping. [8]

(b) Define control elements and explain how they influence transcription in the
eukaryotic genome. [6]

(c) State the various ways in which gene expression of an enzyme may be
controlled at translational and post-translational level. [6]

[Total: 20]

8 (a) Describe the role of mitosis in maintaining genetic stability. [4]

(b) Explain how meiosis can lead to genetic variation. [8]

(c) With one named example, describe the role of proto-oncogenes and tumour
suppressor genes in the control of normal cellular division. [8]

[Total: 20]

@INNOVA 9648/02/PRELIM II/2013


535
19

BLANK PAGE

@INNOVA 9648/02/PRELIM II/2013


536

INN
NOVA JU
UNIOR COLLEGEE
C 2 PRELIMINARY
JC Y EXAMIN
NATION II
in preparation
p for Genera l Certificate
e of Educatio
on Advanceed Level
Hig
gher 2

CANDID
DATE
NAME

CLASS INDEX NUMBER

BIOL
LOGY 9648/02
Paper 2 Core Pape
er 177 Septemb
ber 2013
2 hours
Addition
nal Materialls: Answerr Paper
Cover Page
P

READ T
THESE INS
STRUCTION
NS FIRST
F
For Examin
ners Use
Write yoour name and
a class on n all the worrk you handd in.
Write in o black pen on both s ides of the paper.
n dark blue or Seection A
You ma ay use a soft
s pencil for any dia agrams, grraphs or roough
1
workingg. 10
0
Do not use staple es, paper clips,
c highlig
ghters, glue
e or correc
ction
2
fluid. 14
4
3
nA
Section 16
6
Answerr all questio
ons.
4
14
4
nB
Section
5
Answerr any one question.
q 15
5
6
At the end of the examinatio on, fasten aall your wo
ork securelyy 11
1
togethe
er. Seection B
The nummber of maarks is given
n in the bra
ackets [ ] at the end of
each qu
uestion or part
p question n. 7/8
20
0

T
Total
100
0
Perrcentage
G
Grade

This document
d cconsists of 18
1 printed pages.
p

Innova Ju
unior College
e [Tu
urn over
537
2 For
Examiners
Use

Section A
Answ
wer all ques
stions.

1 Fig. 1 is an electron micrograph


m of a cell fo
ound in the
e pancreass.

Fig. 1

a) Describe how org


(a ganelle X iis adapted for genetic expressioon.
1. Nuc clear pores
s with proteeins controolling move ement of R RNA and
prooteins betw
ween nucleu us and cytoplasm;; (M MUST)
2. Nuc clear envellope separrates nuclear compartment from m cytoplasm m (in
eukkaryotes) which
w allow
ws spatial separation
s of the trannscription and
a
nslation prrocess c
tran creates mic cro-environ nment that allows for
posst-transcrip
ptional leveel of contro
ol / modificcation of traanscripts
beffore transla
ation;;
3. Con ntain genees which co ode for prooducts invo olved in geenetic
exppression (eeg. HATs);;
4. Contain enzym mes e.g. HA ATs that chemically modify
m chro omatin,
affe
ecting commpactness a and access sibility of promoter
p to
o
tran
nscription factors and d RNA poly ymerase, hence
h allowwing for
gennomic level of controll of gene expression;
e ;; (KIV 3-4) [2]

OVA
@INNO 9648/02
2/PRELIM II/20
013 n over
[Turn
538
3 For
Examiners
Use

(b) Describe how insulin, after being synthesized by ribosomes on Y, is


exported out of the cell.
1. protein travels thru cisternae to bud off in a transport vesicle at
transitional ER, transport vesicle travels through cytoplasm & fuse
with GA at cis face;;

2. protein enters cisternal space & modified by


adding/deleting/substituting of sugar monomers in oligosaccharide
chains &/or adding phosphate groups;; post-translational mod

3. modified & tagged protein travels from cisternae to cisternae via


budding & fusion of Golgi vesicle, sorted protein leaves GA in a
secretory vesicle that buds off at trans face;;
4. secretory vesicle moves near cell surface membrane via
cytoskeleton, membrane of secretory vesicle fuses with plasma
membrane insulin released into exterior of cell via exocytosis;; [4]

(c) When insulin is released into the bloodstream, it exerts many effects on
carbohydrate, lipid and protein metabolism.

One of its effects is to activate the enzyme glycogen synthase, which


functions to polymerize glucose monomers into glycogen.

(i) Explain how the structure of glycogen enables it to perform its


function in animals.

Structure of glycogen Function


1. Composed of several hundred / 1. Stores large amount of energy in glucose
thousands of glucose molecules molecules i.e. main respiratory substrate,
as monomers which are linked by -1,4 glycosidic bonds can be hydrolyzed
glycosidic bonds by enzymes present in plants & animals
to release glucose molecules for
respiration;

2. Highly branched 2. Easily hydrolyzed to by enzymes to


release glucose monomers when energy
is needed;
3. Large molecule, high molecular 3. Kept within cell, cannot diffuse out of cell;
weight insoluble, therefore osmotically inactive;

4. -OH groups not protruding 4. Does not interact with water, insoluble,
outwards; osmotically inactive, can be stored
without causing influx of water;
Any 2, 1 mark each

[2]

@INNOVA 9648/02/PRELIM II/2013 [Turn over


539
4 For
Examiners
Use

(ii) Describe two


t ways in which th
he structure of glycoggen differs
s from
the structu
ural polysa
accharide fo
ound in pla
ant cell waalls.

Gly
ycogen Cellulose
C
1. Madde up of glucose monomers; 1. Mad de up of glucose
g moonomers;
2. -1,,4 and -1,6 glycosid
dic bonds 2. - 1,4
4 glycosidic
c bonds linnk glucose
link
k glucose monomers
m together; monomers together;
3. highhly branchhed structu
ure; 3. long, unbranche ed linear ch
hains;
4. OHH groups not
n protrud ding 4. OH groups
g pro
ojecting ou
ut of cellulo
ose
outtwards of macromolec
m cule/ face chains tot allow cro
oss-linkage
ges to be foormed
inte
erior of maccromolecuule; b/w OH groups of parallel ceellulose cha ains;
5. gllucose monomers no ot flipped 5. gluucose mon pped 180o with
nomers flip w
1800o with resp
pect to prev
vious resppect to prev
vious mono omer;
mon nomer;
Any 2, 1 mark eac ch

[2]

[Tota
al: 10]

2 Fig. 2.1 sho


ows the structure of a protease enzyme fo
ound in a re
retrovirus.

Fig 2.1

a) (i)
(a With referrence to FFig. 2.1, sta
ate the ev
vidences thhat the enz
zyme
has both a tertiary a nd quatern
nary protein structuree.
1. Folding
g of polyppeptide cha g -helicess and -plleated
ain to bring
sheets into speciific 3-D strructure (via ds)
a intramoleecular bond
shows tertiary strructure;;
2. Enzymme made up p of 2 poly ypeptide subunits
s heeld together by
intermo
olecular boonds shows quaterrnary struccture;;
3o/4o but circle
Accept 3
[2]

OVA
@INNO 9648/02
2/PRELIM II/20
013 n over
[Turn
540
5 For
Examiners
Use

(ii) Explain what


w determ
mines the 3-dimensional confiiguration of
o the
enzyme.
1. Proteinn structurre maintaiined by boonds betw
ween R grroups
of amiino acids iin polypepptide chaiin;;
2. Hydroophobic interactio ons betw ween non n-polar R
groups, Hydrogen
H n bonds between
b R groupss of polarr
amino acids, Ionic bonds be etween oppositely ccharged R
groups, diS
d bonds between SH groups;; (Any T TWO) [2]

b) The ra
(b ate of reac
ction of H
HIV proteas
se may be
e decreassed by diffferent
inhibito
ors.

Fig. 2.2 shows th he effect o


of substrate
e concentrration on thhe initial ra
ate of
an enzzyme catalysed reacction with and without the adddition of a non-
compe etitive inhib
bitor.

Fig 2.2
2

(i) On Fig. 2.2


2 draw a and label thet curve you wouldd expect in the
presence of a comp
petitive inhiibitor. [1]
curve with
h initial rate
e < w/o I that reaches Vmax;;

(ii) Describe and


a explai n the difference in th
he shape oof the curve
e you
have draw
wn and thatt of the non
n-competittive inhibitoor.
1. Quote Vmax (and
d Km) for both
b graphs
s;;
2. At low [S],
Compe etitive inhib
bitor has siimilar struc
ctural shap
pe to S/ clo
ose
structuural resemb blance
VS
Non-co ompetitive inhibitor iss structurally differen
nt from S, binds
b
at site away from m enz activee site
BOTH inhibit S binding / E-S compllex formatiion thus
decrea asing rate oof rxn;;

OVA
@INNO 9648/02
2/PRELIM II/20
013 n over
[Turn
541
6 For
Examiners
Use

3. At high [S],
Competitive inhibition: higher chances of S binding to the
active site outcompeting inhibitor hence effect of inhibitor
becomes negligible / WTE
VS
non-competitive inhibition: effective enz concentration
reduced;; [3]

(c) A similar protease is found in blackflies. A proteinaceous toxin found in


insecticides used to control its population can be activated by this
protease.

The active toxin binds to receptor molecules on the larvaes gut


epithelial cell membranes. This damages the epithelial cells and kills the
larvae.

Fig. 2.3 shows the results of an experiment to determine the optimum


pH for activation of the toxin.

Fig. 2.3

(i) With reference to Fig. 2.3, describe the effect of pH on mean


activity of the toxin.
1. Optimum pH at 10 at which theres highest mean activity of
toxin between 5-6 a.u (specify one value);;
2. either an increase in pH from 10 to 11 or decrease in pH from
10 to 9.5 results in drastic decrease of mean activity to 2 a.u (or
less);;
3. further decrease in pH from 9.5 to 8.5 results in mean
activity decreasing from 2 a.u to 0 a.u [2]

@INNOVA 9648/02/PRELIM II/2013 [Turn over


542
7 For
Examiners
Use

(ii) Explain wh
hy there iss no activity
y of toxin when
w pH faalls below 8.5.
8
1. increasse H+ conc centration disrupts
d in
ntramolecu ular ionic bo
onds
and hyydrogen bo onds that help maintain the speccific 3D
conforrmation of tthe proteasse;; toxin
n
2. Disruption of seccondary an nd tertiary structures/
s /
denatu protease loss of sp
uration of p pecific 3D
conforrmation of eenz active site toxiin (substraate) cannott
bind to
o active site ase no ac
e of protea ctivation o
of toxin;; [2]

(iii) Suggest how


h the pro
otease in larvas
l digestive systtem cause
es the
toxin to be
e activated .
1. proteas
se cleaves
s toxin wheen catalyticc aa at its a ctive site
hydroly
yses peptidde bonds in
i toxin;;
2. toxin molecule
m ac
cquires com mplementa ary configu uration for
binding
g receptor molecule/ exposes binding
b sitee for
recepto
or moleculle on gut ep
pithelial ce
ell membran ne;; [2]

[Tota
al: 14]

3 Fig. 3.1 sho


ows a T4 bacterioph h has been modifiedd to contain the
hage which
D
DNA of Mu bacterioph
hage, a knoown tempe
erate phage.

Fig. 3.1

a) Suggest how this


(a s bacteriop
phage attac
ches to the
e correct hoost cell.
1. Lon
ng tail fibrees recognis ses specifiic receptorrs on host ccell surface
e
membrrane and bind via wea ak interacttions;;
2. (Sh
hort tail fibrres)/ base p
plate pins interact
i strrongly to aanchor
pha
age to hostt;; [2]

b) Outline
(b e the main n events w which occu ur followingg the entry
ry of the phage
p
DNA innto the hosst cell until new phage particles s are releassed.
1. Mu phage
p genoome integrrates into host
h hromosomee propha
cell ch age
repllicated toge
ether with h host cell ch
hromosom me;;
2. Prop
phage increease in nummber of co
opies until exposure
e tto certain
chemiccals/ UV, prrophage in
nduced to exit
e bacteriial chromo osome;;

OVA
@INNO 9648/02
2/PRELIM II/20
013 n over
[Turn
543
8 For
Examiners
Use

3. Prop
phage trans scribed intto structuraal proteins (capsid prroteins, head
and taiil/ tail fibre structuress) using hoost cell RNAA pol and rribosomes
these together
t wiith viral ge
enome asse emble to foorm compleete Mu vira al
particle
e;;
4. phaage directs s the prod duction of an enzym me that bre reaks down n the
bacteriial cell walll from with hin fluid en
nters the baacteria, cauusing it to swell
and unndergo celll lysis re elease neww Mu phage particles;;;
everrything if T4 is relea
ased
[4]

While the T4 ba acteriophagge infects bacterial cells, the influenza virus


infectss cells of the respirato
ory tract in animals.

H1N1 influenza virus


v cause es swine fllu in pigs. In a numb er of instances,
e have dev
people veloped th he swine flu
f infection when thhey are closely
associated with pigs,
p for exxample farm
mers and porkp proceessors.

Figure 3.2 shows


s a H1N1 vvirus.

M-protein neeuraminidas
se

envvelope enzyme Q

ribonu
ucleoprotein
n hemaggluttinin

Fig. 3.2

c) Describe the significance o


(c of enzyme
e Q to the reproductiive cycle of
o the
H1N1 virus.
v
1. RNA
NA-depende ent RNA po olymerase uses viral ssRNA as template forfo
tran
nscribing complemen
c ntary RNA strands;;
2. whiich are useed as (1) te
emplates fo
or transcrip
ption of new
w RNA straands
whiich form viiral genom e, and (2) used
u as mRRNA for traanslation of
o
vira
al proteins;;;
3. eg. capsid proteins/
p g
glycoproteiins/ enzymmes en nable virall
assembly into neew progeny y viruses;; [3]

OVA
@INNO 9648/02
2/PRELIM II/20
013 n over
[Turn
544
9 For
Examiners
Use

(d) A person is infected with the H1N1 influenza virus but does not develop
the symptoms. Examination shows that the persons immune system
has produced an antibody that binds to the haemagglutinin molecules.

(i) Suggest why the person does not develop influenza-like


symptoms.
1. antibodies produced bind to HA preventing them from
recognizing and binding to (complementary) sialic acid
residues on cell surface receptors on epithelial cells/
antibody-coated/ tagged viruses are recognized by phagocytes;;
2. virus is unable to enter the cells to cause cellular damage/
engulfed and destroyed by phagocytes before viral entry
occurs;; [2]

(ii) After a few days and many viral generations, the person begins to
develop influenza symptoms. Tests for the antibody referred to
above show that it is still present in the blood.

Account for the observations and comment on the implication this


has for developing H1N1 influenza vaccines.
1. virus mutated due to the lack of proofreading by the viral RNA
pol structurally different HA that is not complementary to
the antibody;;
2. Need for regularly designing new/ different vaccines;;
[2]

(e) Contrast the reproductive cycles of the H1N1 influenza virus and Human
Immunodeficieny Virus (HIV).

H1N1 Influenza Virus HIV


1. Host cell Epithelial cells of respiratory tract CD4 T-lymphocytes
2. Attachment HA attaches to sialic-acid Gp120 binds to CD4
containing residues on cell receptors on T-lymphocytes,
surface receptors of host cell undergoes conformational
RME change to expose fusion
domain of gp41
membrane fusion
3. Penetration Acidification of endosome via M2 Viral capsid released into
ion channel conformational host cytoplasm, capsid
change of HA release viral proteins removed to release
capsid into host cytoplasm, viral RNA
capsid proteins removed to
release viral RNA
4. Integration No integration of viral genome Viral genome/ DNA
of viral into host chrom integrated into host cell
genome chrom via integrase
5. Reverse No reverse transcription of viral Reverse transcription of
transcription genome/ ssRNA used as template ssRNA dsDNA via RTase
of viral by RNA pol to synthesize new
genome viral genome and mRNA for
translation into new viral proteins
(Any 3, 1 mark each)

@INNOVA 9648/02/PRELIM II/2013 [Turn over


545
10 For
Examiners
Use

[3]

[Tota
al: 16]

4 O
Oxytocin (OOT) is a neuropeptidee secreted from the posterior
p ppituitary gla
and in
th
he brain an
nd is best known
k for iits role in mammalian
m n birth and lactation.

In
n response e to a varriety of stim
muli such as sucklin
ng, parturiition, or ce
ertain
kiinds of strress, the processed d OT pepttide is rele
eased fromm the pos sterior
pituitary into
o the syste
emic circulaation.

The OT re eceptors activity


a is mediated by G prroteins whhich activa
ate a
phosphatidyylinositol-calcium seccond mess
senger systtem.

Fig. 4.1

a) The blo
(a ood brain barrier
b is sset by the endothelial
e cells liningg the vessels in
the bra
ain.

Suggest why the ese cells ca


an prevent oxytocin secreted
s froom the gla
and to
er the brain
re-ente n.
1. CSSM of cellls made u up of lipid d bilayer with hydrdrophobic core
maade up of hydrocarb
h bon tail off fatty acid
ds;;
2. Oxyytocin is peptide
p hoormone wh hich is hyydrophilic unable e
to trav
verse lipid
d bilayer/ too large// absence of transm membrane e
transpport proteiin for oxyttocin;; [2]

OVA
@INNO 9648/02
2/PRELIM II/20
013 n over
[Turn
546
11 For
Examiners
Use

b) With reference
(b r to
t Fig. 4.1 1, describe and exp plain how a molecu ule of
oxytoccin can lead
d to signal amplification leadingg to cellula
ar response
es.
1. Oxy
xytocin bin nds to GP PCR (at extracellul
e lar face), which ha as an
asssociated inactive G G-protein change in co onformatioon of
GPPCR;;
2. stimmulates GDP
G excha ange for GTP
G at - subunit o of G-proteein
trannslocatedd along cy ytoplasmiic face off CSM tow wards effeector/
pho ospholipaase;;
3. pho ospholipaase cleav ves many y PIP2 molecules
m into second
me essengers DAG and d IP3 / sign
nal amplification at tthis stagee;;
4. DAAG activate es protein n kinase C which ca arries out target cellular
ressponses;;
5. IP3 stimulate es the rellease of CaC 2+ ions from (s)E ER, Ca2+ forms
fo
com mplex wiith calmo odulin wh hich carriies out ttarget cellular
ressponses;;
Refereence to siggnal casca ade
*mech h of action
n: literaturre seems tot imply 1 DAG 1 PKC [5]

-Amminobutyricc acid (GABA) is tthe chief inhibitory neurotrannsmitter inn the


mam mmalian ce entral nerv
vous syste
em involving maturee neuronss. During foetal
f
deveelopment, the
t release e of GABA
A leads to an
a excitato
ory post-syynaptic potential
in im
mmature neeurons.

Durinng labour, findings shown


s tha
at high leveels of matternal oxyttocin inducced a
switcch in the action
a of GABA
G fromm excitatoryy to inhibitory in the neurons of
o the
foetu
us at birth hence incrreases theiir resistanc
ce to dama age during delivery.

Fig. 4.2 showss the gene


eration of the post-s
synaptic po
otentials inn immature
e and
matuure neuron
ns.

Fig. 4.2

OVA
@INNO 9648/02
2/PRELIM II/20
013 n over
[Turn
547
12 For
Examiners
Use

(c) Describe and explain how GABA is released from the pre-synaptic
neurone to the post-synaptic neurone to result in an inhibitory post-
synaptic potential.
1. Depolarization of/ arrival of A.P. at pre-synaptic knob causes
opening of voltage-gated Ca2+ channels Ca2+ influx;;
2. Induces migration of synaptic vesicles containing GABA to
CSM and fuse release GABA into synaptic cleft via
exocytosis;;
3. GABA diffuses across synaptic cleft (down conc grad) and
binds to ligand-gated Cl- channels, causing them to open;;
4. influx of Cl- occurs hyperpolarized of post-synaptic
memb IPSP;; [4]

(d) With reference to Fig. 4.2, explain how GABA could have differing
effects on the post-synaptic membrane in different neurons.
1. 25mM in immature neurone higher than that in ECF vs 7mM in
mature neurone lower than that in ECF;;
2. Binding of GABA to ligand-gated Cl- channels result in Cl-
efflux in immature neurone leads to memb depolarization =
EPSP;;
3. but Cl- influx in mature neurone leads to memb
hyperpolarization = IPSP;;
Accept IPSP / EPSP
[3]

[Total: 14]

@INNOVA 9648/02/PRELIM II/2013 [Turn over


548
13 For
Examiners
Use

5 Nail-patella syndrome (NPS) is a genetic disorder that results in small, poorly


developed nails and kneecaps, but can also affect many other areas of the
body, such as the elbows, chest, and hips. The genetic linkage between the
NPS and ABO blood group loci was the third to be identified in humans.

The NPS gene codes for a transcription factor and a gain-of-function


mutation in one of its alleles leads to NPS.

An unaffected man with blood group O marries an affected female with blood
group B. Fig. 5 shows the resultant pedigree.

Fig. 5

(a) With reference to Fig. 5,

(i) State and explain the mode of inheritance of NPS with respect to
the ABO locus.
1. autosomal linkage between NPS and ABO loci, mutated
NPS allele is dominant;;
2. all individuals with blood group B except individuals II-5
and III-3 are affected;;
3. mutant NPS allele is linked to IB tends to be
inherited together;; [3]

(ii) Explain why the mating between individual 6 and 7 in generation II


produces 100% unaffected offsprings.
1. Genotype of 6 is NnIBIO, 7 could be nnIAIA /nnIAIO;;

2. Offpsrings inherited the IA allele from the parent and the


IO allele from the parent;;
3. IO allele is not linked to the mutated NPS allele;;

4. (Idea) of random segregration, independent


assortment and random fertilisation;; [4]

@INNOVA 9648/02/PRELIM II/2013 [Turn over


549
14 For
Examiners
Use

(b) (i) State the genotype of the individuals 5 and 8 in Generation II with
respect to both the NPS and ABO loci.
1. Individual 5 nnIBIO with both copies of the normal allele for
NPS;;
2. Individual 8 NnIOIO with one copy of the normal allele
and one mutated copy;; [2]

(ii) Suggest reasons for your answers in (i).


1. Recombinants formed from crossing over between the
homologous chromosomes in prophase I;;
2. IB allele and the mutated NPS allele separated due to
chiasmata formation between their loci;
3. Some gametes contain the chromosome where the
mutated NPS allele is linked to the IO allele (accept
reverse argument);; [3]

(c) Apart from the factors due to the inheritance of genes, environment
plays a very important role in determining phenotype of certain
organisms.

Explain, with one example, how the environment may affect the
phenotype.
1. Stating of a specific e.g. and the environmental conditions;;

2. How the phenotype differs with same genotype;;

3. Functionality of the difference in phenotype;;


[3]

[Total: 15]

@INNOVA 9648/02/PRELIM II/2013 [Turn over


550
15 For
Examiners
Use

6 Cranes are large birds and one of the endangered species, Grus americana
commonly known as the Whooping crane is the tallest North American bird
which faced near extinction due to unregulated hunting and loss of habitat.

Cranes can be categorised based on their behaviours and one of the earliest
methods of classifying cranes was based on the calls they make during the
breeding season.

(a) Suggest why biologists could use calls to investigate relationships


between different species of crane.
1. Calls are species specific, only members of the same species
will be able to recognise the calls made;;
2. Characteristics of notes in the calls could be used to
differentiate between different species;;
3. Greater the similarity in the calls, the closer the
relationships between the different species;; [3]

More recently, biologists have used DNA hybridisation to confirm the


relationships between different species of crane. They made samples of
hybrid DNA from the same and from different species. They measured
the percentage of hybridisation of each sample. The results are shown
in Table 6.

Table 6

Species of crane from which hybrid DNA was Percentage DNA


made hybridisation/%

Grus americana and Grus monachus 97.4


Grus monachus and Grus rubicunda 95.7
Grus americana and Grus rubicunda 35.5

Grus rubicunda and Grus rubicunda 99.9

Grus americana and Grus americana 99.9

Grus monachus and Grus monachus 99.8

(b) Describe the structure of the hybrid DNA between the same species.
1. 2 polynucleotides held in a double helix;;

2. held by H-bond between complementary DNA bases, A=T, CG;;

3. 2nm diameter, 3.4nm per complete turn;;

4. Nucleotides held by phosphodiester bonds btw phosphate grp


at 5 C and 3OH of adjacent nucleotide;;

@INNOVA 9648/02/PRELIM II/2013 [Turn over


551
16 For
Examiners
Use

5. anti//, 3 5, 5 3;;
[4]

(c) With reference to Table 6, which of the two species seemed to be the
most closely related? Explain your answer.
1. Grus americana and Grus monachus;;

2. % DNA hybridisation at 97.4%, indicating most number of


bases able to form complementary base pairs;; [2]

(d) The biologists measured the temperatures at which the samples of


hybrid DNA separated into single strands.

Explain why these temperatures could be used to find the percentage of


DNA hybridisation.
1. Higher the temp, K.E required to separate DNA strands, means
higher the degree of relatedness/ hybridisation;;
2. Many regions able to form H-bonds between
complementary base pairs;; [2]

[Total: 11]

@INNOVA 9648/02/PRELIM II/2013 [Turn over


552
17

Section B
Answer one question

Write your answers on the separate answer paper provided.


Your answers should be illustrated by large, clearly labelled diagrams, where
appropriate.
Your answers must be in continuous prose, where appropriate.
Your answers must be set out in sections (a), (b) etc., as indicated in the question.

7 (a) Describe the eukaryotic processing of pre-mRNA in terms of intron splicing,


polyadenylation and 5capping. [8]
1. Introns are non-coding regions within a gene, hence pre-mRNA
introns will be spliced out via spliceosomes;;
2. Constitutive splicing removes all introns and joins together all
exons to form one polypeptide from a gene;;
3. Alternative splicing joins together different combinations of
exons to form different types of polypeptides from a single
gene;;
4. Spliceosome recognises highly conserved regions/ splice sites
between the 3 end of the exon and 5 end of the intron and
cleave the phosphodiester bond between the nucleotides;;
5. After the synthesis of the polyadenylation signal, (AAUAAA),
terminal transferase will bind to the 3 end of the pre-mRNA;;
6. Attach a long chain of nucleotides with the base adenine,
forming the poly(A) tail which is not encoded in the genome;;
7. 5 end of the pre-mRNA will be capped by the addition of a 5
modified guanine nucleotide;;
8. Formation of mature mRNA which will be protected from
enzymatic degradation and exported out of nucleus for
translation;;

(b) Define control elements and explain how they influence transcription in the
eukaryotic genome. [6]
1. Control elements consist of cis-acting elements which are on the
same chromosome and trans-acting elements which are on
different chromosomes as the gene to be regulated;;
2. Cis-acting elements includes promoters core and proximal,
enhancers and silencers;;
3. Trans-acting elements includes genes coding for basal and
specific TFs activators and repressors;;
4. Core promoters are adjacent DNA sequences where the basal TF
and RNA pol will bind to initiate transcription;;
5. Proximal promoters, which are DNA sequences that are close to
the core promoter allow the binding of specific TFs to increase
transcription;;
6. Enhancers allow the binding of activators to recruit DNA
bending protein so that the enhancer is brought nearer to the
core promoter to stabilise the TIC, effectively increasing the rate
of transcription/ amount of mRNA produced/time;;
7. Silencers allow the binding of repressors to result in a decrease

@INNOVA 9648/02/PRELIM II/2013


553
18

in transcription rate, effectively reducing the rate of


transcription/ amount of mRNA produced/time;;
(c) State the various ways in which gene expression of an enzyme may be
controlled at translational and post-translational level. [6]
Translational level
1. mRNA stability by having a long poly(A) tail and 5 cap,
shortening of poly (A) tail & removal of cap results in the
recruitment of nucleases to degrade the mRNA;;
2. Gene silencing when microRNA associates with the RISC (spell
in full);;
3. results in either degradation of the target mRNA, or the
translation inhibition of the target mRNA;;

Post-translational level
4. Concentration/ stability of the translated product could be
regulated via the degradation of the protein;;
5. Proteins designated for destruction are conjugated with a small
protein called ubiquitin;;
6. Ubiquitinylated proteins are subsequently recognized and
degraded by proteasome;;
7. GA modification of the translated polypeptide by cleaving the
polypeptide or modifying the sugar monomers on the oligosacc
chains/ phosphorylation;;

[Total: 20]

8 (a) Describe the role of mitosis in maintaining genetic stability. [4]


1. formation of two nuclei, with same no. of chromosomes as the
parent cell;;
2. During replication, each strand acts as template for synthesis of
new daughter strands;;
3. via complementary bp, A-T 2H-bonds, CG 3H-bonds;;
4. =separation of sister chromatids at anaphase ensure =
distribution of genetic materials to both daughter cells;
5. genetically identical daughter cells with no variation;;
6. production of clones during asexual reproduction to colonise;;

(b) Explain how meiosis can lead to genetic variation. [8]


1. diagrams showing prophase I, metaphase I, anaphase I,
metaphase II, anaphase II
2. independent assortment of homologous chromosomes at
metaphase plate in anaphase 1;
3. possible gametic combinations, 2n
4. crossing over at chiasmata during prophase I between non-
sister chromatids of homologues;;
5. linked genes separated at chiasmata, formed new combinations
of alleles;;
6. sister chromatids no longer identical, will segregate randomly

@INNOVA 9648/02/PRELIM II/2013


554
19

during anaphase II;;


7. new gametes produced from parents not identical, with only one
of each pair of chromosomes;;
8. recombinant genotypes gave rise to genetic variation;;
9. un= separation due to erroneous spindle fibre formation leads to
polyploidy if there are no separation of sis chromatids;
10. Aneuploidy may result if there are un= separation in one pair of
sister chromatids;;

(c) With one named example, describe the role of proto-oncogenes and tumour
suppressor genes in the control of normal cellular division. [8]
1. proto-oncogene = genes that code for proteins that stimulate
normal cell growth and division.
2. RAS, codes for a G prot, role in G1 check point where the
presence of growth factor is being checked for;;
3. Binding of growth factor result in phosphorylation cascade to
result in the increase in levels of cyclins in cell;;
4. Cyclins bind to CDKs, forming complexes to remove the
inhibition of other proteins that prevents progress to S phase;;
5. In the absence of growth factor, the RAS is not activated, hence
cells do not progress to S phase/ presence of growth factor, Ras
activated cells progress from G1 S phase;;
6. p53, tumour suppressor genes = code for proteins that normally
inhibit cell division (when conditions for cell division not met);;
7. by activating DNA repair proteins to repair damaged DNA /
controlling proper cell anchorage;;
8. activating transcription of p21 gene p21 binds to Cdks
prevent entry into S phase;;
9. triggering apoptosis (when DNA damage is beyond repair);;

[Total: 20]

BLANK PAGE

@INNOVA 9648/02/PRELIM II/2013


555

INN
NOVA JU
UNIOR COLLEGEE
C2 PRELIMINARY
JC Y EXAMIN
NATION II
in preparation
p for Genera l Certificate
e of Educatio
on Advanceed Level
Hig
gher 2

CANDID
DATE
NAME

CLASS INDEX NUMBER

BIOL
LOGY 9648/03
2 5 Septemb
ber 2013
Paper 3 Applications Paper
2 hours

Addition
nal Materialls: Answerr Paper
Cover Page
P

READ T
THESE INS
STRUCTION
NS FIRST
F
For Examin
ners Use
Write yoour name and
a class on n all the worrk you handd in.
Write in o black pen on both s ides of the paper.
n dark blue or 1 15
5
You ma ay use a soft
s pencil for any dia agrams, grraphs or roough
workingg. 2
15
5
Do not use staple es, paper clips,
c highlig
ghters, glue
e or correc
ction
fluid. 3
10
0

Answerr all questio


ons. 4 12
2
5 20
0
At the end of the examinatio on, fasten aall your wo
ork securelyy
T
Total 72
2
togethe
er.
The nummber of maarks is given
n in the bra
ackets [ ] at the end of Perrcentage
each qu
uestion or part
p question n.

G
Grade

This document
d cconsists of 16
1 printed pages.
p

Innova Ju
unior College
e [Tu
urn over
556
2 For
Examiners
Use

Section A
Answ
wer all ques
stions.

1 In fla
ax plants, the
t flowers s are white
e, lilac or blue. Fig.1 shows
s the pathway by
b
whicch the flow
wer cells pro
oduce colooured pigm ments.

Fig. 1

(a) A single ba
ase deletio
on occurs in
n gene 1.

DDescribe and
a explain the effecct of this mutation
m o the coloour of flax plant
on
fflowers.

[4]

OVA
@INNO 9648/0 3/PRELIM II/20
013 n over
[Turn
557
3 For
Examiners
Use

(b) G
Gel electroophoresis was used d to separate the enzymes
e i nvolved in
n this
p
pathway. When ex xtracts of the diffeerently colloured flax
ax petals were
a
analyzed, four differeent pattern
ns of band
ds were prroduced. Inn Table 1, only
b
bands thatt contain fu
unctional e nzymes arre shown.

Table 1

R
Results of electrophore
esis Colo
our of petalss

White

((i) With
h reference
e to Fig. 1
1, complete Table 1 to give thhe colour of
o the
al from which each exxtract was taken.
peta [3]

((ii) Desscribe how the enzym


mes were separated
s by
b gel elecctrophoresiis.

[5]

OVA
@INNO 9648/0 3/PRELIM II/20
013 n over
[Turn
558
4 For
Examiners
Use

(c) Genomic libraries and cDNA libraries can be created for the flax plant
genome.

State three differences between the two gene libraries.

[3]

[Total: 15]

@INNOVA 9648/03/PRELIM II/2013 [Turn over


559
5 For
Examiners
Use

2 Diabetes mellitus is a human disease condition in which the patients do not


produce sufficient insulin. A way to treat diabetic patients is to inject purified
recombinant human insulin, produced using genetically engineered bacteria,
into their bloodstream.

Insulin is a relatively small protein comprising two polypeptides, one of 21 amino


acids (the A chain) and one of 30 amino acids (the B chain). Trinucleotides
representing all the codons are synthesized and joined together to form two
artificial genes for the respective chains. Fig. 2.1 shows the two artificial genes,
including the positions of the recognition sequences of two restriction enzymes,
PstI and EcoRI, and distances (in base pairs) between the sites.

Each artificial gene is cloned into a separate bacterial plasmid pUC21, using the
restriction enzyme HindIII, and placed under the control of a strong lac
promoter. Fig. 2.2 shows the plasmid map of pUC21. It includes the positions of
the restriction sites for the three restriction enzymes, PstI, EcoRI and HindIII,
and distances (in base pairs) between the sites. The shaded regions represent
two marker genes, AmpR and lac Z.
6 28 35 8 40 48

Start EcoRI PstI Stop Start PstI EcoRI Stop


codon codon codon codon

Artificial gene for A chain Artificial gene for B chain

Fig. 2.1

PstI

Amp
800 1600

pUC21

EcoRI ORI
Lac Z

400 lac promoter


HindIII

lac operator

Fig. 2.2

@INNOVA 9648/03/PRELIM II/2013 [Turn over


560
6 For
Examiners
Use

Both recombinant plasmids containing the two artificial genes are transformed
separately into E. coli cells. The bacterial cells from the transformation process are
grown on an agar plate containing ampicillin and X-Gal.

(a) State the substance that must be added to induce the expression of the
artificial genes in E. coli.

[1]

(b) Explain what is meant by a marker gene.

[1]

(c) Heat shock treatment is applied to E.coli cells to render them competent to
take up recombinant plasmids.

Describe three ways in which this process of artificial transformation of the


recombinant plasmid into bacterial cells differs from natural transformation.

[3]

(d) Describe and explain the procedure to select for bacterial cells containing
the recombinant plasmid.

[4]

@INNOVA 9648/03/PRELIM II/2013 [Turn over


561
7 For
Examiners
Use

Once the transformed bacterial colonies have been identified, nucleic acid
hybridization is carried out to further identify the colonies which contained
the artificial genes.

The plasmid DNA from these colonies is then isolated and subjected to
digestion by a particular restriction enzyme to determine which colonies
have the artificial genes inserted in the correct orientation.

A student who is unsure of which particular restriction enzyme to use carried


out digestion of all the four possible recombinant plasmids numbered A1,
A2, B1 and B2 using either enzyme EcoRI or PstI.

Table 2 shows all the results obtained.

Table 2

Digestion of plasmid DNA Digestion of plasmid DNA with


with artificial gene for artificial gene for
A chain B chain
Plasmid A1 A2 A1 A2 B1 B2 B1 B2

Length of 406 463 1234 1235 1288 1208 448 448


fragments
(bp) 2463 2406 1635 1634 1608 1688 2448 2448

Restriction
enzyme
used

(e) With reference to Fig. 2.1 and Fig. 2.2, complete Table 2 by identifying the
particular restriction enzyme used for digestion of the respective plasmid
DNA. [2]

(f) With reference to Fig. 2.1, Fig. 2.2 and Table 2, state the appropriate
restriction enzyme that has to be used for the digestion of the respective
plasmid DNA in order to determine which has the artificial genes inserted in
the correct orientation.

(i) Restriction enzyme to digest


plasmid DNA with artificial gene
for A chain

(ii) Restriction enzyme to digest


plasmid DNA with artificial gene
for B chain

[2]

@INNOVA 9648/03/PRELIM II/2013 [Turn over


562
8 For
Examiners
Use

(g) Provide an explanation for your answer in (f).

[2]

[Total: 15]

3 Duchenne muscular dystrophy (DMD) is a common lethal X-linked human


genetic disease caused by the absence of the protein dystrophin in muscle
fibres.

Possible treatments included the transplanting of normal cells from genetically


compatible donors or gene therapy which involves incorporating recombinant
DNA with part of the normal allele of the gene for dystrophin into a muscle.

(a) Explain briefly what is meant by genetically compatible donors.

[2]

@INNOVA 9648/03/PRELIM II/2013 [Turn over


563
9 For
Examiners
Use

The dystrophin gene was inserted into an adenovirus and injected into a
strain of mice with the symptoms of muscular dystrophy.

Eight mice with DMD were injected once in a muscle in one hind limb with
different doses of the modified virus 7 days after birth. The mice were then
killed at the age of 2 to 25 weeks and sections of the injected muscle and
uninjected muscle from the other hind limb were obtained.

These tissues were then treated with a fluorescent antibody specific to


dystrophin to show gene expression and the results are shown in Table 3.

Table 3

Increase in muscles
Virus dose/ Death/weeks
Mouse fibres with
particles*109 cm-3 after birth
dystrophin/%
1 1.0 2 6
2 2.0 3 8
3 2.5 5 10
4 3.0 8 22
5 4.0 10 30
6 5.0 14 50
7 5.0 18 35
8 5.0 25 15

(b) Describe and explain how the adenovirus delivers the dystrophin gene.

[3]

@INNOVA 9648/03/PRELIM II/2013 [Turn over


564
10 For
Examiners
Use

(c) With reference to Table 3,

(i) suggest why uninjected muscle from each mouse was also treated
with fluorescent antibody specific to dystrophin.

[1]

(ii) describe the effect of the injected dosage of virus on the percentage
of muscle fibres producing dystrophin.

[2]

(iii) explain why this gene therapy needs to be repeated.

[2]

[Total: 10]

@INNOVA 9648/03/PRELIM II/2013 [Turn over


565
11

4 Planning question

Malt is germinated cereal grains which have been dried in a process known as
malting. Malted barley contain amylases which digest starch into maltose units by
cleaving the -1,4-glycosidic bonds.

The digestion of starch is an exothermic process and releases energy. This energy
released can be detected by the largest increase in temperature of the reaction
mixture over a period of time.

Using this information and your own knowledge, design an experiment to determine
the concentration of active amylase in malted barley samples that have undergone
different heat treatments.

You must use:


1.0 moldm-3 starch solution
10% amylase solution
Malted barley which have heat treated for 10 minutes at 40.0 C, labelled
M1
Malted barley which have been heat treated for 10 minutes at 80.0C,
labelled M2
distilled water
any normal laboratory glassware e.g. test-tubes, beakers, measuring
cylinders, graduated pipettes, glass rods, etc.
syringes, pipette fillers
timer e.g. stopwatch or stopclock
mercury thermometer

Your plan should:


have a clear and helpful structure such that the method you use is able to
be repeated by anyone reading it,
be illustrated by relevant diagrams, if necessary,
identify the independent and dependent variables,
describe the method with the scientific reasoning used to decide the
method so that the results are as accurate and reliable as possible,
include layout of results tables and graphs with clear headings and labels,
use the correct technical and scientific terms, include reference to safety
measures to minimise any risks associated with the proposed experiment.
[Total: 12]

@INNOVA 9648/03/PRELIM II/2013 [Turn over


566
12
For
Examiners
Use

@INNOVA 9648/03/PRELIM II/2013 [Turn over


567
13
For
Examiners
Use

@INNOVA 9648/03/PRELIM II/2013 [Turn over


568
14
For
Examiners
Use

@INNOVA 9648/03/PRELIM II/2013 [Turn over


569
15
For
Examiners
Use

@INNOVA 9648/03/PRELIM II/2013 [Turn over


570
16

Free-response question

Write your answers to this question on the separate answer paper provided.

Your answer:
should be illustrated by large, clearly labelled diagrams, where appropriate;
must be in continuous prose, where appropriate;
must be set out in sections (a), (b), etc., as indicated in the question.

5 (a) Explain the normal functions of stem cells in a living organism. [6]

(b) Discuss the advantages and limitations of plant cloning. [8]

(c) Comment on the statement GM crops can make a relevant contribution to


solving health problems in developing countries. [6]

[Total: 20]

@INNOVA 9648/03/PRELIM II/2013


571
17

BLANK PAGE

@INNOVA 9648/03/PRELIM II/2013


572

INN
NOVA JU
UNIOR COLLEGEE
C2 PRELIMINARY
JC Y EXAMIN
NATION II
in preparation
p for Genera l Certificate
e of Educatio
on Advanceed Level
Hig
gher 2

CANDID
DATE
NAME

CLASS INDEX NUMBER

BIOL
LOGY 9648/03
2 5 Septemb
ber 2013
Paper 3 Applications Paper
2 hours

Addition
nal Materialls: Answerr Paper
Cover Page
P

READ T
THESE INS
STRUCTION
NS FIRST
F
For Examin
ners Use
Write yoour name and
a class on n all the worrk you handd in.
Write in o black pen on both s ides of the paper.
n dark blue or 1 15
5
You ma ay use a soft
s pencil for any dia agrams, grraphs or roough
workingg. 2
15
5
Do not use staple es, paper clips,
c highlig
ghters, glue
e or correc
ction
fluid. 3
10
0

Answerr all questio


ons. 4 12
2
5 20
0
At the end of the examinatio on, fasten aall your wo
ork securelyy
T
Total 72
2
togethe
er.
The nummber of maarks is given
n in the bra
ackets [ ] at the end of Perrcentage
each qu
uestion or part
p question n.

G
Grade

This document
d cconsists of 16
1 printed pages.
p

Innova Ju
unior College
e [Tu
urn over
573
2 For
Examiners
Use

Section A
Answ
wer all ques
stions.

1 In fla
ax plants, the
t flowers s are white
e, lilac or blue. Fig.1 shows
s the pathway by
b
whicch the flow
wer cells pro
oduce colooured pigm ments.

Fig. 1

(a) A single ba
ase deletio
on occurs in
n gene 1.

DDescribe and
a explain the effecct of this mutation
m o the coloour of flax plant
on
fflowers.
11. framesh hift mutatioon/ changee in reading
g frame of gene/ chan nge in codo
ons
downsttream of mutation;;
22. change e in aa seq. of enz 1 change in n R groupss changee in R grou up
interacttions in po
olypeptide;;;
33. leading g to differen
nt folding tthus 3D con n of enz 1 likely to be
nfiguration
non-funnctional / does
d not ca
atalyze syn
nthesis of liilac pigmen
nt;;
44. lack of substrate
s fo
or enz 2 to act on to synthesize
s ment flax
blue pigm
fflowers app pear white;; [4]

OVA
@INNO 9648/0 3/PRELIM II/20
013 n over
[Turn
574
3 For
Examiners
Use

(b) G
Gel electroophoresis was used d to separate the enzymes
e i nvolved in
n this
p
pathway. When ex xtracts of the diffeerently colloured flax
ax petals were
a
analyzed, four differeent pattern
ns of band
ds were prroduced. Inn Table 1, only
b
bands thatt contain fu
unctional e nzymes arre shown.

Table 1

R
Results of electrophore
esis Colo
our of petalss

White

blue

lilac

white
w

((i) With
h reference
e to Fig. 1
1, complete Table 1 to give thhe colour of
o the
al from which each exxtract was taken.
peta [3]

((ii) Desscribe how the enzym


mes were separated
s by
b gel elecctrophoresiis.
1. polyacrylam
p mide gel ellectrophorresis for separation off proteins;;
2. S
SDS to dennature / line
earize prott to removee influence of shape ono
m
migration speed
s and confer equ ual negativve charge p per unit ma
ass/
l
length usin
ng sodium dodecyl su ulfate (SDS
S) / anionicc detergentt;;
3. Buffer
B used
d to provid
de medium for ion con
nductivity;;;
4. lload prot sample
s in w
wells near thet negativ ve electrode e
de / cathode
a
apply voltaage / electri
ric field/ dirrect current across geel;;
5. -v
vely chargeed prot miggrate towarrds anode, polyacrylaamide gel
actss as molecuular sieve tto impede movementt of larger p prot/
larg
ger prot trav
vels shorteer distance e compared d to smallerr prot;; [5]

OVA
@INNO 9648/0 3/PRELIM II/20
013 n over
[Turn
575
4 For
Examiners
Use

(c) Genomic libraries and cDNA libraries can be created for the flax plant
genome.

State three differences between the two gene libraries.

Feature Genomic library cDNA library


1. Content Contains entire DNA content of Contains entire protein-
an organism including all encoding DNA content/ exons
coding and non-coding only
sequences/ introns + exons
2. Starting Requires chromosomal/ total Requires total mRNA isolation
genetic genomic DNA isolation reverse transcribed into
material to cDNA;
be isolated
3. Starting Can be from any cell/ tissue Total mRNA should be isolated
cellular from a specific cell/ tissue
material where the particular protein of
interest is likely to be produced
in large quantities
4. Intactness A gene may be cut in between Intact genes are obtained as
of genes. when there is a restriction site cDNAs are derived from
within the sequence mRNAs
5. Size of Larger number of different Smaller number of different
library clone types in the library as clone types in the library (less
fragments originate from whole than 1% of genome)
genome
6. Functional use to study gene expression usually to decipher aa seq /
use of due to presence of reg seq like clone to produce polypeptide
library in promoters
relation to
type of
sequences
studied.
7. Functional Cannot be used to study Can be used for tracing
use of physiological or changes in patterns of gene
library in developmental-based changes expression under different
relation to in gene expression developmental or physiological
patterns of conditions
gene
expression
studied.
(Any 3, 1 mark each)

[3]

[Total: 15]

@INNOVA 9648/03/PRELIM II/2013 [Turn over


576
5 For
Examiners
Use

2 Diabetes mellitus is a human disease condition in which the patients do not


produce sufficient insulin. A way to treat diabetic patients is to inject purified
recombinant human insulin, produced using genetically engineered bacteria,
into their bloodstream.

Insulin is a relatively small protein comprising two polypeptides, one of 21 amino


acids (the A chain) and one of 30 amino acids (the B chain). Trinucleotides
representing all the codons are synthesized and joined together to form two
artificial genes for the respective chains. Fig. 2.1 shows the two artificial genes,
including the positions of the recognition sequences of two restriction enzymes,
PstI and EcoRI, and distances (in base pairs) between the sites.

Each artificial gene is cloned into a separate bacterial plasmid pUC21, using the
restriction enzyme HindIII, and placed under the control of a strong lac
promoter. Fig. 2.2 shows the plasmid map of pUC21. It includes the positions of
the restriction sites for the three restriction enzymes, PstI, EcoRI and HindIII,
and distances (in base pairs) between the sites. The shaded regions represent
two marker genes, AmpR and lac Z.
6 28 35 8 40 48

Start EcoRI PstI Stop Start PstI EcoRI Stop


codon codon codon codon

Artificial gene for A chain Artificial gene for B chain

Fig. 2.1

PstI

Amp
800 1600

pUC21

EcoRI ORI
Lac Z

400 lac promoter


HindIII

lac operator

Fig. 2.2

@INNOVA 9648/03/PRELIM II/2013 [Turn over


577
6 For
Examiners
Use

Both recombinant plasmids containing the two artificial genes are transformed
separately into E. coli cells. The bacterial cells from the transformation process are
grown on an agar plate containing ampicillin and X-Gal.

(a) State the substance that must be added to induce the expression of the
artificial genes in E. coli.
Lactose/ allolactose/ IPTG;;
[1]

(b) Explain what is meant by a marker gene.


Gene which will give a visible/ detectable phenotype / trait
/characteristic to the bacteria for identification/ selection of bacteria
containing recombinant plasmid/ gene of interest;; [1]

(c) Heat shock treatment is applied to E.coli cells to render them competent to
take up recombinant plasmids.

Describe three ways in which this process of artificial transformation of the


recombinant plasmid into bacterial cells differs from natural transformation.

Artificial Transformation Natural Transformation

1. plasmids enter via pores 1. plasmids enter by binding to


induced from heat shock competence factor / cell surface
receptor of bacterial cell
2. dsDNA enters 2. ssDNA enters
3. does not undergo homologous 3. allele may undergo
recombination, exist as recombination with homologous
plasmid/ extrachromosomal region in host chrom
DNA

[3]

(d) Describe and explain the procedure to select for bacterial cells containing
the recombinant plasmid.
1. Bacterial cells successfully transformed with plasmids will be resistant to
ampicillin hence can survive/ grow/ form colonies;;
2. bacterial cells containing non-recombinant plasmids appear blue since
lacZ gene is intact/ not disrupted by insertion of insulin genes
functional -galactosidase produced
3. bacterial cells containing recombinant plasmids appear white since lacZ
gene has been disrupted/ insertional inactivation of lacZ gene has
occurred no functional -galactosidase produced
4. X-gal hydrolyzed/ cleaved/ X-gal not hydrolyzed/ cleaved;; (only
awarded if pt2/3 is correct) [4]

@INNOVA 9648/03/PRELIM II/2013 [Turn over


578
7 For
Examiners
Use

Once the transformed bacterial colonies have been identified, nucleic acid
hybridization is carried out to further identify the colonies which contained
the artificial genes.

The plasmid DNA from these colonies is then isolated and subjected to
digestion by a particular restriction enzyme to determine which colonies
have the artificial genes inserted in the correct orientation.

A student who is unsure of which particular restriction enzyme to use carried


out digestion of all the four possible recombinant plasmids numbered A1,
A2, B1 and B2 using either enzyme EcoRI or PstI.

Table 2 shows all the results obtained.

Table 2

Digestion of plasmid DNA Digestion of plasmid DNA with


with artificial gene for artificial gene for
A chain B chain
Plasmid A1 A2 A1 A2 B1 B2 B1 B2

Length of 406 463 1234 1235 1288 1208 448 448


fragments
(bp) 2463 2406 1635 1634 1608 1688 2448 2448

Restriction
EcoRI; PstI; PstI; EcoRI;
enzyme
used

(e) With reference to Fig. 2.1 and Fig. 2.2, complete Table 2 by identifying the
particular restriction enzyme used for digestion of the respective plasmid
DNA. [2]

(f) With reference to Fig. 2.1, Fig. 2.2 and Table 2, state the appropriate
restriction enzyme that has to be used for the digestion of the respective
plasmid DNA in order to determine which has the artificial genes inserted in
the correct orientation.

(i) Restriction enzyme to digest


plasmid DNA with artificial gene EcoR1;;
for A chain

(ii) Restriction enzyme to digest


plasmid DNA with artificial gene Pst1;;
for B chain

[2]

@INNOVA 9648/03/PRELIM II/2013 [Turn over


579
8 For
Examiners
Use

(g) Provide an explanation for your answer in (f).


1. artificial genes must be cut in such a manner that the fragments
generated are of different lengths / not of the same length;;
2. EcoRI cleavage generates 406kb fragment for chain A/1208kb
fragment for chain B if wrong orientation indicating start codon
attached @ a further site from promoter (accept reverse aug) [2]

[Total: 15]

3 Duchenne muscular dystrophy (DMD) is a common lethal X-linked human


genetic disease caused by the absence of the protein dystrophin in muscle
fibres.
Possible treatments included the transplanting of normal cells from genetically
compatible donors or gene therapy which involves incorporating recombinant
DNA with part of the normal allele of the gene for dystrophin into a muscle.

(a) Explain briefly what is meant by genetically compatible donors.


1. individuals who are genetically similar to recipient / blood relations/
close relatives;;
2. transplanted cells from donor would not elicit cell rejection / trigger
immune response;;
3. antigens on CSM are not recognized as foreign / donor cells
with similar glycoprot on CSM;; [2]

The dystrophin gene was inserted into an adenovirus and injected into a
strain of mice with the symptoms of muscular dystrophy.

Eight mice with DMD were injected once in a muscle in one hind limb with
different doses of the modified virus 7 days after birth. The mice were then
killed at the age of 2 to 25 weeks and sections of the injected muscle and
uninjected muscle from the other hind limb were obtained.

These tissues were then treated with a fluorescent antibody specific to


dystrophin to show gene expression and the results are shown in Table 3.

Table 3
Increase in muscles
Virus dose/ Death/weeks
Mouse fibres with
particles*109 cm-3 after birth
dystrophin/%
1 1.0 2 6
2 2.0 3 8
3 2.5 5 10
4 3.0 8 22
5 4.0 10 30
6 5.0 14 50
7 5.0 18 35
8 5.0 25 15

@INNOVA 9648/03/PRELIM II/2013 [Turn over


580
9 For
Examiners
Use

(b) Describe and explain how the adenovirus delivers the dystrophin gene.
1. adenovirus binds to specific receptors found on CSM of muscle
cells, enters by receptor-mediated endocytosis;;
2. viral capsid enters cytoplasm, enzymes remove capsid to release
viral genome along with normal dystrophin gene;;
3. normal dystrophin gene travels into nucleus of muscle cell,
undergoes homologous recombination with host chromosome
to replace defective allele / exist and expressed extra-
chromosomally to express dystrophin prot;; [3]

(c) With reference to Table 3,

(i) suggest why uninjected muscle from each mouse was also treated
with fluorescent antibody specific to dystrophin.
act control
to calculate % increase in muscle fibres w dystrophin by
subtracting away baseline production from uninjected
muscle;; [1]

(ii) describe the effect of the injected dosage of virus on the percentage
of muscle fibres producing dystrophin.
1. higher dosage of virus, higher the increase in % of muscle
fibre producing dystrophin
1.0 x 109 cm-3 with 6% increase at 2 weeks to 5.0 x 109 cm-3
with 50% increase at 14 weeks;;
2. virus dose of 5.0 x 109 cm-3 for mice killed in 14 25 weeks
caused decreased in % increase of muscle fibre producing
dystrophin
from 50% to 15%;;
[2]

(iii) explain why this gene therapy needs to be repeated.


1. transient expression / short term effect;
2. at high dose of virus @ 5.0 x 109 cm-3 , % increase drops from
50% for mice in week 14 to 15% in week 25;
3. normal dystrophin gene did not integrate into host
chromosome;
4. not segregated equally into daughter cells during
mitosis / lost when the cell undergoes replicative
senescence / apoptosis;; [2]
(any 2 points to score 1m)
[Total: 10]

@INNOVA 9648/03/PRELIM II/2013 [Turn over


581
10

4 Planning question

Malt is germinated cereal grains which have been dried in a process known as
malting. Malted barley contain amylases which digest starch into maltose units by
cleaving the -1,4-glycosidic bonds.

The digestion of starch is an exothermic process and releases energy. This energy
released can be detected by the largest increase in temperature of the reaction
mixture over a period of time.

Using this information and your own knowledge, design an experiment to determine
the concentration of active amylase in malted barley samples that have undergone
different heat treatments.

You must use:


1.0 moldm-3 starch solution
10% amylase solution
Malted barley which have heat treated for 10 minutes at 40.0 C, labelled
M1
Malted barley which have been heat treated for 10 minutes at 80.0C,
labelled M2
distilled water
any normal laboratory glassware e.g. test-tubes, beakers, measuring
cylinders, graduated pipettes, glass rods, etc.
syringes, pipette fillers
timer e.g. stopwatch or stopclock
mercury thermometer

Your plan should:


have a clear and helpful structure such that the method you use is able to
be repeated by anyone reading it,
be illustrated by relevant diagrams, if necessary,
identify the independent and dependent variables,
describe the method with the scientific reasoning used to decide the
method so that the results are as accurate and reliable as possible,
include layout of results tables and graphs with clear headings and labels,
use the correct technical and scientific terms, include reference to safety
measures to minimise any risks associated with the proposed experiment.
[Total: 12]

@INNOVA 9648/03/PRELIM II/2013 [Turn over


582
11

Theoretical consideration

1. [mode of enzyme action = 1 mark]


Starch is a polysaccharide consisting -glucose molecules linked by -1,4 and -1,6
glycosidic bonds.
Amylase is an enzyme which has an active site complementary to substrate starch,
and is able to hydrolyze the -1,4 glycosidic bonds to produce maltose, a
disaccharide of 2 -glucose monomers joined together.

2. [effect of temp on enz = 1 mark]


Amylase is a protein molecule with a specific 3D configuration maintained by
intramolecular and intermolecular bonds between amino acid R-groups (eg. H-bonds,
ionic bonds, disulfide bonds, hydrophobic interactions).
Increase in temp and the increase in kinetic energy causes enzyme molecule to
vibrate vigorously/ undergo thermal agitation such that intermolecular bonds are
disrupted enzyme loses specific 3D configuration active site no longer able to
accommodate substrate starch less E-S complexes formed per unit time
enzyme is said to be denatured.

3. [Identification of variables & methods of measuring the dependent variable = 1


mark]

Dependent variable: Rate of reaction measured by the largest rise in temperature of


amylase-starch reaction over a 5-min period, in oC min-1.
Independent variable: concentration of amylase / %
Other variables to be kept constant:
o pH by adding citrate phosphate buffer into reaction mixture/ w.t.e.
o temp at which amylase-starch reaction is occurring by conducting
experiment in automated thermostatically water bath maintained at 37oC.
Reasons for keeping variables constant:
o pH and temperature must be kept constant because rate of enzyme
activity is affected by changes in pH and temp.

Precaution:
4. [Precaution & safety = 1 mark any 2]
Mercury is toxic. Exercise caution in handling thermometer so as to prevent
breakage.
Glassware is fragile, handle with care to prevent breakage.
Buffer is a skin irritant, wear gloves and googles when handling.

@INNOVA 9648/03/PRELIM II/2013


583
12

Procedure

5. [specify appropriately at least 5 different concentrations of amylase at regular


intervals in % = 1 mark]
Step 1:
Dilution table required.
Dilute the 10% amylase solution using distilled water/ buffer shown in the dilution
table.

Dilution Table

Volume of 10% Final vol/ cm3 Final conc of


Boiling Volume of distilled
amylase solution/ amylase
tubes water / cm3
cm3 solution/ %
A 10.0 0.0 10.0 10.0
B 8.0 2.0 10.0 8.0
C 6.0 4.0 10.0 6.0
D 4.0 6.0 10.0 4.0
E 2.0 8.0 10.0 2.0
Control 0.0 10.0 10.0 0.0
No need to indicate control in table, but must appear in text

6. [acclimatisation timing]
Step 2:
Set up a water bath maintained at 37oC and immerse boiling tube A containing 3 cm3
of 10% amylase solution into it.
Immerse a test tube containing 3 cm3 of 1.0 moldm-3 starch solution into the same
water bath.
Allow both amylase and starch solutions to acclimatize to the temperature for 2-10
minutes before adding the starch solution into boiling tube A.

7. [measurement of increase in temp = 1 mark]


Step 3:
Upon adding the starch solution into amylase solution, start the stopwatch and record
the initial temperature, To.
Record temperature readings every 30 seconds (Tx) until 3 minutes is over.
Calculate the largest increase in temperature (Tx To) over the stated period of time.
Repeat the experiment with boiling tubes B, C, D, and E using the same volume of
starch solution.
Repeat the experiment using equal volumes of M1 and M2 to replace amylase
solution, and record the increase in temperature.

@INNOVA 9648/03/PRELIM II/2013


584
13

8. [collection & processing of data = 4 marks]


Step 4:
a) Record raw data in Table 1 (including correct table headings + formulas). [1m]
b) Calculate the rate of increase in temperature in oC min-1 by taking the average of a least
3 readings then divided by 3mins [1m]
c) Plot rate of temp increase under different amylase concentration on Graph 1 [1m]
d) Use graph to find concentration of active amylase present in M1 and M2 [1m]

9. [replicates and repeats = 1 mark]


Step 5:
Repeat Steps 2-3 twice obtain two further readings.
Repeat the entire experiment once.

10. [control with distilled water = 1 mark]

same setup as the experiment except that the 3cm3 amylase solution has been
replaced by distilled water.

@INNOVA 9648/03/PRELIM II/2013


585
14
For
Examiners
Use
Temperature/oC
Average
largest Rate of
Amylase increase in increase of
0s 30s 60s 90s 120s 150s 180s
Concentr temperature temp / oC min-
(T0) (T1) (T2) (T3) (T4) (T5) (T6) 1
ation /% (TA)
(TA /3)
(Tx-T0)/3
1 2 3 1 2 3 1 2 3 1 2 3 1 2 3 1 2 3 1 2 3

10.0
8.0
6.0
4.0
2.0
0.0
M1
M2
Formulas must be shown in table heading.
Table 1

@INNOVA 9648/03/PRELIM II/2013


586
15
For
Examiners
Use
Free-response question

Write your answers to this question on the separate answer paper provided.

Your answer:
should be illustrated by large, clearly labelled diagrams, where appropriate;
must be in continuous prose, where appropriate;
must be set out in sections (a), (b), etc., as indicated in the question.

5
(a) Explain the normal functions of stem cells in a living organism. [6]
1. undifferentiated, unspecialised cells with no specific tissue structures;;
2. unlimited self-renewal capacity;
due to high level of telomerase acty ;
capable of undergoing cont division / mitosis;
to replace damaged or worn out tissues;
3. totipotent;
zygotic SC;
4. able to dy/dx into all cell types including extra-embryonic tiss / placenta;
extracellular mem
5. pluripotent;
ESCs found in inner cell mass of embryo / blastocyst;
6. able to dy/dx into any cell type except extra-embryonic tiss / placenta;;
7. into the 3 dermal layers: ectoderm, mesoderm, endoderm;;
8. multipotent;
ASC e.g. HSCs, found in the bone marrow;
9. able to dy/dx into grp of cells with related f(x) / limited cell lineages / myeloid,
lymphoid progenitor;
e.g. give rise to RBCs, WBCs, lymphocytes;
10. SCs dy/dx in response to internal & external signals;
that influences gene expression;

(b) Discuss the advantages and limitations of plant cloning. [8]

Advantages (max. 4)
1. rapid pdtn of many plants in short period of time;
which are genetically identical;
plants grow faster
2. able to generate whole plant from a small portion of explant;
facilitate growing of GM plants;
3. prod hybrid offspring from sexually incompatible plants with desirable traits fr

@INNOVA 9648/03/PRELIM II/2013


587
16
For
Examiners
Use
both plants;
using protoplast culture;
4. able to prod disease free plants;
by meristemic culture using rapidly proliferating bact / virus-free meristematic
tiss as explant;
5. to prod. plant spp that are difficult to propagate;
e.g. difficult to pollinate / low seed pdtn / low seed germination;
6. propagate plants limited by seasonal variations;
possible to meet consumer demand for out-of-season plants;
idea of independent of climate as plants are grown in lab. Plants are
eventually transplanted into field.

Limitations (max 4)
7. high risk of contamination / genetic uniformity increases susceptibility
resulting in high losses;
8. exposure to high conc. of PGRs;
plants tend to undergo mutation;
9. plant cells may possess physiological memory;
plants generated from non-juvenile cells may mature before they are fully
grown;
10. high cost due to need for sophisticated facilities & high labour input;
e.g. sterile lab conditions / tiss culture room / special nutrient media /
trained personnel / transplanting plantlets;
(c) Comment on the statement GM crops can make a relevant contribution to
solving health problems in developing countries. [6]

1. state appropriate example:


Golden Rice;
GM to contain (crt 1 & psy gene that syn) -carotene (precursor to vit A);
2. state health problem GMO can solve:
treat vit A deficiency in children in developing countries;
which may lead to vision impairment / blindness / other infectious diseases;
poor eye-sight
3. For
a. rice is staple diet;
can be consumed in larger amt / easier access to ensure adequate vit A
intake as cf to o/r food sources;
b. rice is relatively inexpensive & non-seasonal;
cf to o/r vit A sources like green leafy veg;

@INNOVA 9648/03/PRELIM II/2013


588
17
For
Examiners
Use
c. sustainable & cost-effective;
cf to alternative like vit supplements;
d. may be biofortified / enhanced with o/r micronutrients;
e.g. I2 / Zn to reduce problem of malnutrition;
4. Against
a. -carotene is lipid soluble;
malnourished children w/o enough fat stores to absorb;
b. may not be the long term solution;
cf to improved & varied diets, rising socioeconomic status;
poor infrastructure & govt policies may prevent access to rice;
c. may topple ecological balance;
levels of -carotene may impact organism downstream of food chain;
d. rice eaten in large quantities could xcessive intake of vit A;
hypervitaminosis / vit A toxicity;
GM salmon which is not a crop
Bt corn which aims to increase yield which may improve food shortage but not
targeted to solve health problems
[Total: 20]

@INNOVA 9648/03/PRELIM II/2013


589
18
For
Examiners
Use

@INNOVA 9648/03/PRELIM II/2013


590
19
For
Examiners
Use
BLANK PAGE

@INNOVA 9648/03/PRELIM II/2013


591

JURONG JUNIOR COLLEGE


JC 2 PRELIMINARY EXAMINATIONS
Higher 2

CANDIDATE
NAME

CLASS

BIOLOGY 9648/01
Paper 1 Multiple Choice 20 September 2013
1 hour 15 minutes
Additional Materials: Multiple Choice Answer Sheet

READ THESE INSTRUCTIONS FIRST


Write in soft pencil.
Do not use staples, paper clips, highlighters, glue or correction fluid.
Write your name and class on the Answer Sheet in the spaces provided unless this has been done
for you.

There are forty questions on this paper. Answer all questions. For each question there are four
possible answers A, B, C and D.
Choose the one you consider correct and record your choice in soft pencil on the separate Answer
Sheet.

Read the instructions on the Answer Sheet very carefully.

Each correct answer will score one mark. A mark will not be deducted for a wrong answer.
Any rough working should be done in this booklet.
Calculators may be used.

This document consists of 19 printed pages.


[Turn over

9648 / 01
592
2

1 Which row correctly identifies all the locations of ribosomes in a eukaryotic cell?

attached to attached to
free in in
endoplasmic nuclear in nucleus
cytoplasm mitochondria
reticulum envelope
A
B
C
D

2 What are the features of triglycerides?

higher energy lower proportion


less dense than
polar value than of hydrogen than
water
carbohydrates in carbohydrates
A
B
C
D

Jurong JC/JC2 H2 Biology/Prelims/2013


593
3

3 Liver tissue produces an enzyme called catalase which breaks down hydrogen peroxide
into water and oxygen.

2H2O2 2H2O + O2

The rate of this reaction can be determined by measuring the volume of oxygen
produced in a given length of time.

Students added small cubes of fresh liver tissue to a range of hydrogen peroxide
solutions and measured the volumes of oxygen produced. Their data were used to
produce the graph showing how changing the concentration of hydrogen peroxide
affected the rate of oxygen production.

Which statements are correct?

1 At P, the rate of reaction is limited by the concentration of enzyme.


2 At Q, all of the enzyme active sites are occupied by substrate molecules.
3 At Q, the rate of reaction is limited by the concentration of the substrate.
4 R represents Km where the reaction rate = Vmax / 2.
5 At S, all of the enzyme active sites are occupied by substrate molecules.

A 2 and 3 only B 1, 2, 3 and 4 only C 1, 2, 4 and 5 only D 1, 3, 4 and 5 only

4 Which statement is incorrect for mitotic cell division?

A DNA is replicated semi-conservatively during mitosis.


B DNA is normally unchanged from one generation of cells to the next.
C The daughter cells have the potential to produce the same enzymes as the parent
cell.
D The same quantity of DNA is distributed to the nuclei of two new cells.

Jurong JC/JC2 H2 Biology/Prelims/2013


594
4

5 For organisms undergoing sexual reproduction, a reduction division occurs before


fertilisation.

Which reason(s) explain why this is necessary?

1 increase genetic variation


2 prevent doubling of the chromosome number
3 reduce the chances of mutation

A 1 only B 2 only C 1 and 2 only D 1, 2 and 3

6 Which features of DNA enable it to meet these requirements as a molecule of


inheritance?

requirement of DNA molecule


ability to remain ability to contain ability to transfer ability to replicate
stable information information
A complementary formation of sequences of sugar-phosphate
base pairing mRNA for nucleotides backbone
translation
B formation of complementary sugar-phosphate sequences of
mRNA for base pairing backbone nucleotides
translation
C sequences of sugar-phosphate complementary formation of
nucleotides backbone base pairing mRNA for
translation
D sugar-phosphate sequences of formation of complementary
backbone nucleotides mRNA for base pairing
translation

7 Which statements about the genetic code are correct?

1 The genetic code has redundancy and is degenerate.


2 There is only one codon for the amino acid methionine.
3 Codons act as stop and start signals during transcription and translation.
4 Prokaryotes generally use the same genetic code as eukaryotes.

A 1 and 2 only B 1, 2 and 3 only C 1, 2 and 4 only D 1, 2, 3 and 4

Jurong JC/JC2 H2 Biology/Prelims/2013


595
5

8 Ribonuclease is an enzyme that digests RNA. The first five amino acids of the functional
molecule of ribonuclease are:

lys glu thr ala ala

The mRNA of the gene coding for ribonuclease, for the first 15 nucleotides, has the
following sequence.

AUGAAGGAAACUGCU

The genetic code table is shown below.

Which event occurs to explain the information given above?

A The first amino acid on the polypeptide chain is removed in post-translational


modification.
B The first codon is removed from the mRNA transcript in post-transcriptional
modification.
C The mRNA binds to the rRNA in the second codon position.
D There is no tRNA with an anticodon complementary to the first codon.

9 Base substitution mutations can have the following molecular consequence except

A changes one codon for an amino acid into another codon for that same amino acid.
B codon for one amino acid is changed into a codon of another amino acid.
C reading frame changes downstream of the mutant site.
D codon for one amino acid is changed into a translation termination codon.

Jurong JC/JC2 H2 Biology/Prelims/2013


596
6

10 What happens when a bacterial cell that is infected by lambda phage divides?

A Viral RNA is replicated.


B Viral RNA is transcribed from viral DNA.
C Viral DNA is replicated.
D Viral DNA is made from viral RNA.

11 The following describes events in the life cycle of an influenza virus. Which statement is
correct?

A Acidification results in the viral envelope fusing with the exocytic vesicle membrane,
releasing the capsid in the process.
B Viral genome acts as a template for the making of new copies of viral RNA genome.
C Complementary RNA strand is translated into viral proteins.
D Glycoproteins synthesised by ribosomes are transported to the plasma membrane
via endocytic vesicles whereby the glycoproteins are incorporated into the host cells
plasma membrane.

12 A scientist is using an ampicillin-resistant strain of bacteria which has a non-functional


lacZ gene.

She has a F plasmid containing a functional copy of lacZ gene. She then adds a high
concentration of the plasmid to a tube of the bacteria in a medium for bacterial growth
that contains glucose or lactose as the only energy source. This tube (+) and a control
tube (-) with similar bacteria but no plasmid are both incubated under the appropriate
conditions for growth and plasmid uptake. The scientist then spreads a sample of each
bacterial culture (+ and -) on each of the three types of plates indicated below.

Plate A: Glucose medium


Plate B: Glucose medium with ampicillin
Plate C: Lactose medium with ampicillin

Which row correctly identifies the plates with growth () or no growth ()?

Bacterial strain with added plasmid (+) Bacterial strain with no plasmid (-)
Plate A Plate B Plate C Plate A Plate B Plate C
A
B
C
D

Jurong JC/JC2 H2 Biology/Prelims/2013


597
7

13 The diagram represents a length of DNA which forms a structure called an operon. Parts
of the operon are labelled E, F, G and H. They have different functions.

What identifies the functions of parts E, F, G and H?

E F G H
A operator structural gene(s) regulator promoter
B promoter regulator structural gene(s) operator
C regulator promoter operator structural gene(s)
D structural gene(s) operator promoter regulator

14 Which statement correctly describes the role of histone proteins?

A All eukaryotic genes are transcribed continuously because they are not packaged by
histones.
B A DNA must be selectively released from its histone packaging before transcription
can occur in bacteria.
C Histones package prokaryote chromatin into the nucleosomes that form the bulk of the
chromosome.
D The organisation of DNA by histones in eukaryotes allows some gene control
sequences to be thousands of base pairs away from the gene concerned.

15 In transcriptional control in eukaryotic cells

1 a different combination of DNA binding proteins may regulate the activity of


a particular gene.
2 enhancers may be involved in the promotion as well as regulation of gene
transcription.
3 phosphorylation of transcriptional factors by a kinase may occur.
4 enhancers may be some distance from the promoter sites they control.

A 1, 2 and 3 only B 1, 3 and 4 only C 2, 3 and 4 only D 1, 2, 3 and 4

16 Which of the following is least likely to cause a proto-oncogene to become an oncogene?

A A gene is incorporated into a retroviral genome.


B A gene is moved close to an enhancer, causing excess product to be made.
C A gene is truncated, yielding a protein with modified activity.
D A gene is moved into centromeric heterochromatin, silencing its transcription.

Jurong JC/JC2 H2 Biology/Prelims/2013


598
8

17 A student has 2 cultures of cells. Culture A consists of normal cells, while culture B
consists of cells which were isolated from a tumour and divide uncontrollably. He wants
to determine if the uncontrolled growth of cells in culture B was the result of activation of
an oncogene or the mutation of a tumour suppressor gene. He decides to fuse the cells
from the two cultures to form a hybrid line containing DNA from both cells.

The list of possible results and the conclusions drawn from the experiment are
summarised by the student in the table below:

resulting growth of hybrid cells type of gene mutated in the tumour


1 grows normally activated oncogene
2 grows uncontrollably activated oncogene
3 grows normally mutated tumour suppressor gene
4 grows uncontrollably mutated tumour suppressor gene

Only two of the conclusions are correctly matched with the possible results. The two
correct possibilities are

A 1 and 3 B 1 and 4 C 2 and 3 D 2 and 4

18 A parent organism of unknown genotype is mated in a test cross. Half of the offspring
have the same phenotype as the parent.

What can be concluded from this result?

A The parent is heterozygous for the trait.


B The parent is homozygous dominant for the trait.
C The parent is homozygous recessive for the trait.
D The trait being inherited is polygenic.

19 The table shows the results of a series of crosses in a species of small mammal.

coat colour phenotype


male parent female parent offspring
dark grey light grey dark grey, light grey, albino
light grey albino light grey, white with black patches
dark grey white with black patches dark grey, light grey
light grey dark grey dark grey, light grey, white with black patches

What explains the inheritance of the range of phenotypes shown by these crosses?

A one gene with a pair of co-dominant alleles


B one gene with multiple alleles
C sex linkage of the allele for grey coat colour
D two genes, each with a dominant and recessive allele

Jurong JC/JC2 H2 Biology/Prelims/2013


599
9

20 Two
o genes A and
a B are lin
nked togeth
her as show
wn below.

If th
he genes area far enou ugh apart ssuch that crossing ove
er between the alleles
s occurs
occa asionally, which
w ment is true of the gam
statem metes?

A All of the gametes will be Ab and aB.


B There will be
b 25% Ab,, 25% aB, 2 25% ab and 25% AB.
C There will be
b approxim mately equaal numbers of Ab and ab
a gametes .
D The numbe er of Ab gam
metes will b
be greater th
han the num
mber of ab ggametes.

21 In the fruit fly, Drosophila


a melanoga aster, four genes who ose recessivve alleles code
c for
blacck body (B / b), curved wings (C / c), purple eyes
e (P / p) and vestigiial wings (V
V / v) are
linke
ed on chrom mosome 2.

The
e table shows some distances
d b
between the
e gene loci, as determ
mined by breeding
b
experiments.

at is the corrrect sequence of the lo


Wha oci on chromosome 2?
?

A B/b P/p V/v C/c


B C/c V/v B/b P/p
C P/p V/v C/c B/b
D V/v B/b P/p C/c

Jurong J
JC/JC2 H2 Biology/Prelims/2013
600
10

22 The following pedigree depicts the inheritance of a rare hereditary disease affecting
muscles.

What is the mode of inheritance of this disease?

A autosomal dominant
B autosomal recessive
C X-linked dominant
D X-linked recessive

23 The phenotypes of 200 offspring of a dihybrid test cross were recorded. The cross
involved petal colour and fertility of the anthers of sweet pea flowers. The table shows
the observed and expected numbers of each phenotype.

A chi-squared (2) test was performed and the probability of the difference between the
observed and expected results being due to chance was found to be <0.001.

Which conclusions may be drawn from this probability?

1 The difference is significant.


2 The difference is due to chance.
3 The difference is not due to chance.
4 The difference is due to some factor such as linkage of the genes
concerned.

A 2 only B 3 and 4 only C 1, 2 and 4 only D 1, 3 and 4 only

Jurong JC/JC2 H2 Biology/Prelims/2013


601
11

24 The diagram summarises the reactions during photosynthesis.

Which of the following correctly identifies the substances involved?

1 2 3 4 5 6

A H2O ADP, NAD CO2 O2 ATP, NADH glucose


B O2 ADP, NADP glucose H2O ATP, NADPH CO2
C H2O ATP, NADPH CO2 O2 ADP, NADP glucose
D H2O ADP, NADP CO2 O2 ATP, NADPH glucose

Jurong JC/JC2 H2 Biology/Prelims/2013


602
12

25 The figure shows the arrangement of photosystems, protein complexes containing


chlorophyll molecules, on the thylakoid membrane of a plant chloroplast.

Which row correctly identifies the structure or function of the labelled regions?

site of photoactivation site of photolysis site of ATP stroma thylakoid


of chlorophyll of water synthesis lumen

A 1 and 2 1 and 2 4 6 5

B 1 and 2 1 4 6 5

C 1, 2 and 3 1 3 and 4 5 6

D 1, 2 and 3 1 and 2 3 and 4 6 5

26 During substrate-level phosphorylation, ATP is synthesised from ADP and inorganic


phosphate.

What is the immediate source of energy for this reaction?

A chemical bond energy released during the light independent stage of photosynthesis
B chemical bond energy released during glycolysis and the Krebs cycle
C potential energy of protons diffusing through the inner mitochondrial membranes in
mitochondria
D potential energy of protons diffusing through thylakoid membranes in chloroplasts

Jurong JC/JC2 H2 Biology/Prelims/2013


603
13

27 The diagram shows part of a membrane in a cell.

Which of these descriptions is true?

A X is the intermembrane space, Y is the inner mitochondrial membrane and the


diagram shows ATP synthesis in a chloroplast.
B Y is the inner mitochondrial membrane, Z is the cytosol and the diagram shows ATP
synthesis in a mitochondrion.
C X is the intermembrane space, Y is the inner mitochondrial membrane and the
diagram shows ATP synthesis in a mitochondrion.
D Z is the intermembrane space, X is the matrix and the diagram shows ATP synthesis
in a mitochondrion.

Jurong JC/JC2 H2 Biology/Prelims/2013


604
14

28 Lipid membranes can be formed in the laboratory by painting phospholipids over a PTFE
sheet with a hole in it.

Such a lipid membrane is impermeable to water-soluble materials including charged ions


such as Na+ or K+.

In one experiment with Na+ ions, no current flowed across the membrane until a
substance called gramicidin was added, at which time current flowed.

Which statement is consistent with this information and your knowledge of membrane
structure?

Gramicidin becomes incorporated into the membrane and is

A a carbohydrate molecule found only on the outside of the membrane.


B a non-polar lipid which passes all the way through the membrane.
C a protein molecule with both hydrophilic and hydrophobic regions.
D a protein molecule which has only hydrophobic regions.

Jurong JC/JC2 H2 Biology/Prelims/2013


605
15

29 The graph shows how the rate of entry of glucose into a cell changes as the
concentration of glucose outside the cell changes.

What is the cause of the plateau at X?

A All the carrier proteins are saturated with glucose.


B The carrier proteins are denatured and no longer able to function.
C The cell has used up its supply of ATP.
D The concentrations of glucose inside and outside the cell are equal.

30 Which two features contribute to the great tensile strength of cellulose?

1 glycosidic bonds linking the long chains of 1,4 -glucose molecules


2 the -OH groups of the glucose molecules project outwards and form H
bonds with neighbouring chains
3 the strength of the glycosidic bonds between the neighbouring chains of
molecules
4 the successive glucose molecules are orientated at 180 to each other

A 1 and 3 only B 1 and 4 only C 2 and 3 only D 2 and 4 only

31 During which stage of a nerve impulse does the opening of the voltage-gated sodium
channels play an important role?

A resting potential
B depolarisation
C repolarisation
D hyperpolarisation

Jurong JC/JC2 H2 Biology/Prelims/2013


606
16

32 The heart, like most internal organs, is supplied with nerves from both the sympathetic
and parasympathetic nervous systems. The neurones in these nerves form synapses
with muscle tissue in the sino-atrial node (SAN) of the heart.

The diagram shows how the rate of heartbeat of a mammal changed when the
sympathetic and parasympathetic nerves were stimulated at different frequencies. The
frequency of stimulation was measured in hertz (Hz), i.e. the number of stimulations per
second.

Which of these statements 1 to 4 are true?

1 The parasympathetic nerves released less transmitters substances than the


sympathetic nerves.
2 The two nerves released different transmitter substances.
3 The stimulation of the sympathetic nerves decreases the heart rate while the
stimulation of the parasympathetic nerves increases the heart rate.
4 The stimulation of the parasympathetic nerves decreases the heart rate
while the stimulation of the sympathetic nerves increases the heart rate.

A 1 and 3 only B 1 and 4 only C 2 and 4 only D 1, 2 and 4 only

33 Which of the following is true about G-protein signalling?

1 During activation of G-protein, subunit of the G-protein dissociates from


the activated G-protein to activate adenylyl cyclase
2 During activation of G-protein, the active subunit is terminated by the
hydrolysis of the bound GTP caused by GTPase
3 Testosterone can bind to the cell membrane receptor to activate G-protein
4 The ratio of G-protein coupled receptor to G-protein is 1:1

A 2 only B 1 and 3 only C 2 and 3 only D 2, 3 and 4 only

Jurong JC/JC2 H2 Biology/Prelims/2013


607
17

34 Before the settlement of California in the 1800s, the elk population was very large. By
about 1900 there were only a few dozen elk left.

Owing to protection, there are now about 3000 elk living in a small number of isolated
herds.

Unfortunately, some of the elk in all the herds have difficulty grazing due to a shortened
lower jaw.

Which statements best explain this?

1 The early settlers only hunted elk that could graze.


2 There was a mutation affecting jaw size in one of the herds.
3 There is random mating within each herd.
4 The current elk population demonstrates a bottleneck effect.
5 There was directional selection favouring short jaws.

A 2 and 5 only
B 3 and 4 only
C 1, 2 and 4 only
D 2, 3 and 5 only

35 Darwins view of the process of evolution to form new species (speciation) has been
reinforced by more recent discoveries in genetics and cell biology.

In this view, which sequence of events is considered most likely to lead to speciation?

competition and
adaptation of behavioural sympatric
A predation leading to
population isolation speciation
natural selection
competition and
B
adaptation of predation leading to behavioural allopatric
population isolation speciation
natural selection
competition and adaptation
geographical sympatric
C predation leading to of isolated
isolation speciation
natural selection populations
competition and adaptation
D predation leading to
geographical of isolated allopatric
isolation speciation
natural selection populations

Jurong JC/JC2 H2 Biology/Prelims/2013


608
18

36 Restriction enzymes can be used to cut DNA at specific sites. Genes such as the gene
for insulin can be cut from the chromosome of one species and as a result of ligation
joined to the chromosome of another species forming recombinant (hybrid) DNA.

The figure shows two chromosomes from different species. The specific restriction
enzyme sites (R1, R2, and R3) are shown.

Both chromosomes were cut at restriction site R1 with EcoRI which recognises the
sequence GAATTC and produces restriction fragments with sticky ends. The fragments
were mixed and allowed to join to form recombinant (hybrid) DNA.

What is the maximum number of ways in which the fragments can join?

A 2 B 4 C 6 D 8

37 In which way(s) is/are the polymerase chain reaction (PCR) similar to the replication of
DNA?

1 DNA is heated to break hydrogen bonds


2 DNA unwinds
3 RNA primers are used
4 DNA polymerase are required

A 2 only B 2 and 4 only C 1 and 3 only D 2, 3 and 4

38 Some of the steps involved in DNA fingerprinting are listed below:

1 transfer segments of DNA to nitrocellulose membrane


2 extraction of DNA
3 gel electrophoresis
4 treating DNA with restriction enzymes
5 autoradiography
6 hybridise with probe

The correct sequence is

A 243165
B 423165
C 421365
D 243156

Jurong JC/JC2 H2 Biology/Prelims/2013


609
19

39 A key feature of most multicellular organisms is the ability to differentiate and produce
specialised cells.

Which row best describes the ability of zygotic cells to differentiate?

totipotent pluripotent multipotent


A
B
C
D

Key: = ability, = no ability

40 Which row correctly states the effects and possible consequences of producing the
various genetically-modified organisms?

Bt corn GM salmon Golden rice


possible possible possible
Effect effect effect
consequence consequence consequence
might
improve might become improve displace native improve
A become
quality weeds yield species yield
weeds

improve toxic to closely improve displace native improve cause


B
yield related species yield species quality allergies

improve might become improve improve cause


C cause allergies
yield weeds quality yield allergies

might
improve toxic to closely improve improve
D cause allergies become
yield related species yield quality
weeds

Jurong JC/JC2 H2 Biology/Prelims/2013


610

JURONG JUNIOR COLLEGE


JC 2 PRELIMINARY EXAMINATIONS
Higher 2

CANDIDATE
NAME

CLASS

BIOLOGY 9648/01
Paper 1 Multiple Choice 20 September 2013
1 hour 15 minutes
Additional Materials: Multiple Choice Answer Sheet

READ THESE INSTRUCTIONS FIRST


Write in soft pencil.
Do not use staples, paper clips, highlighters, glue or correction fluid.
Write your name and class on the Answer Sheet in the spaces provided unless this has been done
for you.

There are forty questions on this paper. Answer all questions. For each question there are four
possible answers A, B, C and D.
Choose the one you consider correct and record your choice in soft pencil on the separate Answer
Sheet.

Read the instructions on the Answer Sheet very carefully.

Each correct answer will score one mark. A mark will not be deducted for a wrong answer.
Any rough working should be done in this booklet.
Calculators may be used.

This document consists of 19 printed pages.


[Turn over

9648 / 01
611
2

1 Which row correctly identifies all the locations of ribosomes in a eukaryotic cell?

attached to attached to
free in in
endoplasmic nuclear in nucleus
cytoplasm mitochondria
reticulum envelope
A
B
C
D

2 What are the features of triglycerides?

higher energy lower proportion


less dense than
polar value than of hydrogen than
water
carbohydrates in carbohydrates
A
B
C
D

Jurong JC/JC2 H2 Biology/Prelims/2013


612
3

3 Liver tissue produces an enzyme called catalase which breaks down hydrogen peroxide
into water and oxygen.

2H2O2 2H2O + O2

The rate of this reaction can be determined by measuring the volume of oxygen
produced in a given length of time.

Students added small cubes of fresh liver tissue to a range of hydrogen peroxide
solutions and measured the volumes of oxygen produced. Their data were used to
produce the graph showing how changing the concentration of hydrogen peroxide
affected the rate of oxygen production.

Which statements are correct?

1 At P, the rate of reaction is limited by the concentration of enzyme.


2 At Q, all of the enzyme active sites are occupied by substrate molecules.
3 At Q, the rate of reaction is limited by the concentration of the substrate.
4 R represents Km where the reaction rate = Vmax / 2.
5 At S, all of the enzyme active sites are occupied by substrate molecules.

A 2 and 3 only B 1, 2, 3 and 4 only C 1, 2, 4 and 5 only D 1, 3, 4 and 5 only

4 Which statement is incorrect for mitotic cell division?

A DNA is replicated semi-conservatively during mitosis.


B DNA is normally unchanged from one generation of cells to the next.
C The daughter cells have the potential to produce the same enzymes as the parent
cell.
D The same quantity of DNA is distributed to the nuclei of two new cells.

Jurong JC/JC2 H2 Biology/Prelims/2013


613
4

5 For organisms undergoing sexual reproduction, a reduction division occurs before


fertilisation.

Which reason(s) explain why this is necessary?

1 increase genetic variation


2 prevent doubling of the chromosome number
3 reduce the chances of mutation

A 1 only B 2 only C 1 and 2 only D 1, 2 and 3

6 Which features of DNA enable it to meet these requirements as a molecule of


inheritance?

requirement of DNA molecule


ability to remain ability to contain ability to transfer ability to replicate
stable information information
A complementary formation of sequences of sugar-phosphate
base pairing mRNA for nucleotides backbone
translation
B formation of complementary sugar-phosphate sequences of
mRNA for base pairing backbone nucleotides
translation
C sequences of sugar-phosphate complementary formation of
nucleotides backbone base pairing mRNA for
translation
D sugar-phosphate sequences of formation of complementary
backbone nucleotides mRNA for base pairing
translation

7 Which statements about the genetic code are correct?

1 The genetic code has redundancy and is degenerate.


2 There is only one codon for the amino acid methionine.
3 Codons act as stop and start signals during transcription and translation.
4 Prokaryotes generally use the same genetic code as eukaryotes.

A 1 and 2 only B 1, 2 and 3 only C 1, 2 and 4 only D 1, 2, 3 and 4

Jurong JC/JC2 H2 Biology/Prelims/2013


614
5

8 Ribonuclease is an enzyme that digests RNA. The first five amino acids of the functional
molecule of ribonuclease are:

lys glu thr ala ala

The mRNA of the gene coding for ribonuclease, for the first 15 nucleotides, has the
following sequence.

AUGAAGGAAACUGCU

The genetic code table is shown below.

Which event occurs to explain the information given above?

A The first amino acid on the polypeptide chain is removed in post-translational


modification.
B The first codon is removed from the mRNA transcript in post-transcriptional
modification.
C The mRNA binds to the rRNA in the second codon position.
D There is no tRNA with an anticodon complementary to the first codon.

9 Base substitution mutations can have the following molecular consequence except

A changes one codon for an amino acid into another codon for that same amino acid.
B codon for one amino acid is changed into a codon of another amino acid.
C reading frame changes downstream of the mutant site.
D codon for one amino acid is changed into a translation termination codon.

Jurong JC/JC2 H2 Biology/Prelims/2013


615
6

10 What happens when a bacterial cell that is infected by lambda phage divides?

A Viral RNA is replicated.


B Viral RNA is transcribed from viral DNA.
C Viral DNA is replicated.
D Viral DNA is made from viral RNA.

11 The following describes events in the life cycle of an influenza virus. Which statement is
correct?

A Acidification results in the viral envelope fusing with the exocytic vesicle membrane,
releasing the capsid in the process.
B Viral genome acts as a template for the making of new copies of viral RNA genome.
C Complementary RNA strand is translated into viral proteins.
D Glycoproteins synthesised by ribosomes are transported to the plasma membrane
via endocytic vesicles whereby the glycoproteins are incorporated into the host cells
plasma membrane.

12 A scientist is using an ampicillin-resistant strain of bacteria which has a non-functional


lacZ gene.

She has a F plasmid containing a functional copy of lacZ gene. She then adds a high
concentration of the plasmid to a tube of the bacteria in a medium for bacterial growth
that contains glucose or lactose as the only energy source. This tube (+) and a control
tube (-) with similar bacteria but no plasmid are both incubated under the appropriate
conditions for growth and plasmid uptake. The scientist then spreads a sample of each
bacterial culture (+ and -) on each of the three types of plates indicated below.

Plate A: Glucose medium


Plate B: Glucose medium with ampicillin
Plate C: Lactose medium with ampicillin

Which row correctly identifies the plates with growth () or no growth ()?

Bacterial strain with added plasmid (+) Bacterial strain with no plasmid (-)
Plate A Plate B Plate C Plate A Plate B Plate C
A
B
C
D

Jurong JC/JC2 H2 Biology/Prelims/2013


616
7

13 The diagram represents a length of DNA which forms a structure called an operon. Parts
of the operon are labelled E, F, G and H. They have different functions.

What identifies the functions of parts E, F, G and H?

E F G H
A operator structural gene(s) regulator promoter
B promoter regulator structural gene(s) operator
C regulator promoter operator structural gene(s)
D structural gene(s) operator promoter regulator

14 Which statement correctly describes the role of histone proteins?

A All eukaryotic genes are transcribed continuously because they are not packaged by
histones.
B A DNA must be selectively released from its histone packaging before transcription
can occur in bacteria.
C Histones package prokaryote chromatin into the nucleosomes that form the bulk of the
chromosome.
D The organisation of DNA by histones in eukaryotes allows some gene control
sequences to be thousands of base pairs away from the gene concerned.

15 In transcriptional control in eukaryotic cells

1 a different combination of DNA binding proteins may regulate the activity of


a particular gene.
2 enhancers may be involved in the promotion as well as regulation of gene
transcription.
3 phosphorylation of transcriptional factors by a kinase may occur.
4 enhancers may be some distance from the promoter sites they control.

A 1, 2 and 3 only B 1, 3 and 4 only C 2, 3 and 4 only D 1, 2, 3 and 4

16 Which of the following is least likely to cause a proto-oncogene to become an oncogene?

A A gene is incorporated into a retroviral genome.


B A gene is moved close to an enhancer, causing excess product to be made.
C A gene is truncated, yielding a protein with modified activity.
D A gene is moved into centromeric heterochromatin, silencing its transcription.

Jurong JC/JC2 H2 Biology/Prelims/2013


617
8

17 A student has 2 cultures of cells. Culture A consists of normal cells, while culture B
consists of cells which were isolated from a tumour and divide uncontrollably. He wants
to determine if the uncontrolled growth of cells in culture B was the result of activation of
an oncogene or the mutation of a tumour suppressor gene. He decides to fuse the cells
from the two cultures to form a hybrid line containing DNA from both cells.

The list of possible results and the conclusions drawn from the experiment are
summarised by the student in the table below:

resulting growth of hybrid cells type of gene mutated in the tumour


1 grows normally activated oncogene
2 grows uncontrollably activated oncogene
3 grows normally mutated tumour suppressor gene
4 grows uncontrollably mutated tumour suppressor gene

Only two of the conclusions are correctly matched with the possible results. The two
correct possibilities are

A 1 and 3 B 1 and 4 C 2 and 3 D 2 and 4

18 A parent organism of unknown genotype is mated in a test cross. Half of the offspring
have the same phenotype as the parent.

What can be concluded from this result?

A The parent is heterozygous for the trait.


B The parent is homozygous dominant for the trait.
C The parent is homozygous recessive for the trait.
D The trait being inherited is polygenic.

19 The table shows the results of a series of crosses in a species of small mammal.

coat colour phenotype


male parent female parent offspring
dark grey light grey dark grey, light grey, albino
light grey albino light grey, white with black patches
dark grey white with black patches dark grey, light grey
light grey dark grey dark grey, light grey, white with black patches

What explains the inheritance of the range of phenotypes shown by these crosses?

A one gene with a pair of co-dominant alleles


B one gene with multiple alleles
C sex linkage of the allele for grey coat colour
D two genes, each with a dominant and recessive allele

Jurong JC/JC2 H2 Biology/Prelims/2013


618
9

20 Two
o genes A and
a B are lin
nked togeth
her as show
wn below.

If th
he genes area far enou ugh apart ssuch that crossing ove
er between the alleles
s occurs
occa asionally, which
w ment is true of the gam
statem metes?

A All of the gametes will be Ab and aB.


B There will be
b 25% Ab,, 25% aB, 2 25% ab and 25% AB.
C There will be
b approxim mately equaal numbers of Ab and ab
a gametes .
D The numbe er of Ab gam
metes will b
be greater th
han the num
mber of ab ggametes.

21 In the fruit fly, Drosophila


a melanoga aster, four genes who ose recessivve alleles code
c for
blacck body (B / b), curved wings (C / c), purple eyes
e (P / p) and vestigiial wings (V
V / v) are
linke
ed on chrom mosome 2.

The
e table shows some distances
d b
between the
e gene loci, as determ
mined by breeding
b
experiments.

at is the corrrect sequence of the lo


Wha oci on chromosome 2?
?

A B/b P/p V/v C/c


B C/c V/v B/b P/p
C P/p V/v C/c B/b
D V/v B/b P/p C/c

Jurong J
JC/JC2 H2 Biology/Prelims/2013
619
10

22 The following pedigree depicts the inheritance of a rare hereditary disease affecting
muscles.

What is the mode of inheritance of this disease?

A autosomal dominant
B autosomal recessive
C X-linked dominant
D X-linked recessive

23 The phenotypes of 200 offspring of a dihybrid test cross were recorded. The cross
involved petal colour and fertility of the anthers of sweet pea flowers. The table shows
the observed and expected numbers of each phenotype.

A chi-squared (2) test was performed and the probability of the difference between the
observed and expected results being due to chance was found to be <0.001.

Which conclusions may be drawn from this probability?

1 The difference is significant.


2 The difference is due to chance.
3 The difference is not due to chance.
4 The difference is due to some factor such as linkage of the genes
concerned.

A 2 only B 3 and 4 only C 1, 2 and 4 only D 1, 3 and 4 only

Jurong JC/JC2 H2 Biology/Prelims/2013


620
11

24 The diagram summarises the reactions during photosynthesis.

Which of the following correctly identifies the substances involved?

1 2 3 4 5 6

A H2O ADP, NAD CO2 O2 ATP, NADH glucose


B O2 ADP, NADP glucose H2O ATP, NADPH CO2
C H2O ATP, NADPH CO2 O2 ADP, NADP glucose
D H2O ADP, NADP CO2 O2 ATP, NADPH glucose

Jurong JC/JC2 H2 Biology/Prelims/2013


621
12

25 The figure shows the arrangement of photosystems, protein complexes containing


chlorophyll molecules, on the thylakoid membrane of a plant chloroplast.

Which row correctly identifies the structure or function of the labelled regions?

site of photoactivation site of photolysis site of ATP stroma thylakoid


of chlorophyll of water synthesis lumen

A 1 and 2 1 and 2 4 6 5

B 1 and 2 1 4 6 5

C 1, 2 and 3 1 3 and 4 5 6

D 1, 2 and 3 1 and 2 3 and 4 6 5

26 During substrate-level phosphorylation, ATP is synthesised from ADP and inorganic


phosphate.

What is the immediate source of energy for this reaction?

A chemical bond energy released during the light independent stage of photosynthesis
B chemical bond energy released during glycolysis and the Krebs cycle
C potential energy of protons diffusing through the inner mitochondrial membranes in
mitochondria
D potential energy of protons diffusing through thylakoid membranes in chloroplasts

Jurong JC/JC2 H2 Biology/Prelims/2013


622
13

27 The diagram shows part of a membrane in a cell.

Which of these descriptions is true?

A X is the intermembrane space, Y is the inner mitochondrial membrane and the


diagram shows ATP synthesis in a chloroplast.
B Y is the inner mitochondrial membrane, Z is the cytosol and the diagram shows ATP
synthesis in a mitochondrion.
C X is the intermembrane space, Y is the inner mitochondrial membrane and the
diagram shows ATP synthesis in a mitochondrion.
D Z is the intermembrane space, X is the matrix and the diagram shows ATP synthesis
in a mitochondrion.

Jurong JC/JC2 H2 Biology/Prelims/2013


623
14

28 Lipid membranes can be formed in the laboratory by painting phospholipids over a PTFE
sheet with a hole in it.

Such a lipid membrane is impermeable to water-soluble materials including charged ions


such as Na+ or K+.

In one experiment with Na+ ions, no current flowed across the membrane until a
substance called gramicidin was added, at which time current flowed.

Which statement is consistent with this information and your knowledge of membrane
structure?

Gramicidin becomes incorporated into the membrane and is

A a carbohydrate molecule found only on the outside of the membrane.


B a non-polar lipid which passes all the way through the membrane.
C a protein molecule with both hydrophilic and hydrophobic regions.
D a protein molecule which has only hydrophobic regions.

Jurong JC/JC2 H2 Biology/Prelims/2013


624
15

29 The graph shows how the rate of entry of glucose into a cell changes as the
concentration of glucose outside the cell changes.

What is the cause of the plateau at X?

A All the carrier proteins are saturated with glucose.


B The carrier proteins are denatured and no longer able to function.
C The cell has used up its supply of ATP.
D The concentrations of glucose inside and outside the cell are equal.

30 Which two features contribute to the great tensile strength of cellulose?

1 glycosidic bonds linking the long chains of 1,4 -glucose molecules


2 the -OH groups of the glucose molecules project outwards and form H
bonds with neighbouring chains
3 the strength of the glycosidic bonds between the neighbouring chains of
molecules
4 the successive glucose molecules are orientated at 180 to each other

A 1 and 3 only B 1 and 4 only C 2 and 3 only D 2 and 4 only

31 During which stage of a nerve impulse does the opening of the voltage-gated sodium
channels play an important role?

A resting potential
B depolarisation
C repolarisation
D hyperpolarisation

Jurong JC/JC2 H2 Biology/Prelims/2013


625
16

32 The heart, like most internal organs, is supplied with nerves from both the sympathetic
and parasympathetic nervous systems. The neurones in these nerves form synapses
with muscle tissue in the sino-atrial node (SAN) of the heart.

The diagram shows how the rate of heartbeat of a mammal changed when the
sympathetic and parasympathetic nerves were stimulated at different frequencies. The
frequency of stimulation was measured in hertz (Hz), i.e. the number of stimulations per
second.

Which of these statements 1 to 4 are true?

1 The parasympathetic nerves released less transmitters substances than the


sympathetic nerves.
2 The two nerves released different transmitter substances.
3 The stimulation of the sympathetic nerves decreases the heart rate while the
stimulation of the parasympathetic nerves increases the heart rate.
4 The stimulation of the parasympathetic nerves decreases the heart rate
while the stimulation of the sympathetic nerves increases the heart rate.

A 1 and 3 only B 1 and 4 only C 2 and 4 only D 1, 2 and 4 only

33 Which of the following is true about G-protein signalling?

1 During activation of G-protein, subunit of the G-protein dissociates from


the activated G-protein to activate adenylyl cyclase
2 During activation of G-protein, the active subunit is terminated by the
hydrolysis of the bound GTP caused by GTPase
3 Testosterone can bind to the cell membrane receptor to activate G-protein
4 The ratio of G-protein coupled receptor to G-protein is 1:1

A 2 only B 1 and 3 only C 2 and 3 only D 2, 3 and 4 only

Jurong JC/JC2 H2 Biology/Prelims/2013


626
17

34 Before the settlement of California in the 1800s, the elk population was very large. By
about 1900 there were only a few dozen elk left.

Owing to protection, there are now about 3000 elk living in a small number of isolated
herds.

Unfortunately, some of the elk in all the herds have difficulty grazing due to a shortened
lower jaw.

Which statements best explain this?

1 The early settlers only hunted elk that could graze.


2 There was a mutation affecting jaw size in one of the herds.
3 There is random mating within each herd.
4 The current elk population demonstrates a bottleneck effect.
5 There was directional selection favouring short jaws.

A 2 and 5 only
B 3 and 4 only
C 1, 2 and 4 only
D 2, 3 and 5 only

35 Darwins view of the process of evolution to form new species (speciation) has been
reinforced by more recent discoveries in genetics and cell biology.

In this view, which sequence of events is considered most likely to lead to speciation?

competition and
adaptation of behavioural sympatric
A predation leading to
population isolation speciation
natural selection
competition and
B
adaptation of predation leading to behavioural allopatric
population isolation speciation
natural selection
competition and adaptation
geographical sympatric
C predation leading to of isolated
isolation speciation
natural selection populations
competition and adaptation
D predation leading to
geographical of isolated allopatric
isolation speciation
natural selection populations

Jurong JC/JC2 H2 Biology/Prelims/2013


627
18

36 Restriction enzymes can be used to cut DNA at specific sites. Genes such as the gene
for insulin can be cut from the chromosome of one species and as a result of ligation
joined to the chromosome of another species forming recombinant (hybrid) DNA.

The figure shows two chromosomes from different species. The specific restriction
enzyme sites (R1, R2, and R3) are shown.

Both chromosomes were cut at restriction site R1 with EcoRI which recognises the
sequence GAATTC and produces restriction fragments with sticky ends. The fragments
were mixed and allowed to join to form recombinant (hybrid) DNA.

What is the maximum number of ways in which the fragments can join?

A 2 B 4 C 6 D 8

37 In which way(s) is/are the polymerase chain reaction (PCR) similar to the replication of
DNA?

1 DNA is heated to break hydrogen bonds


2 DNA unwinds
3 RNA primers are used
4 DNA polymerase are required

A 2 only B 2 and 4 only C 1 and 3 only D 2, 3 and 4

38 Some of the steps involved in DNA fingerprinting are listed below:

1 transfer segments of DNA to nitrocellulose membrane


2 extraction of DNA
3 gel electrophoresis
4 treating DNA with restriction enzymes
5 autoradiography
6 hybridise with probe

The correct sequence is

A 243165
B 423165
C 421365
D 243156

Jurong JC/JC2 H2 Biology/Prelims/2013


628
19

39 A key feature of most multicellular organisms is the ability to differentiate and produce
specialised cells.

Which row best describes the ability of zygotic cells to differentiate?

totipotent pluripotent multipotent


A
B
C
D

Key: = ability, = no ability

40 Which row correctly states the effects and possible consequences of producing the
various genetically-modified organisms?

Bt corn GM salmon Golden rice


possible possible possible
Effect effect effect
consequence consequence consequence
might
improve might become improve displace native improve
A become
quality weeds yield species yield
weeds

improve toxic to closely improve displace native improve cause


B
yield related species yield species quality allergies

improve might become improve improve cause


C cause allergies
yield weeds quality yield allergies

might
improve toxic to closely improve improve
D cause allergies become
yield related species yield quality
weeds

Jurong JC/JC2 H2 Biology/Prelims/2013


629

hh JURONG JUNIOR COLLEGE


JC 2 PRELIMINARY EXAMINATIONS
Higher 2

CANDIDATE
NAME

CLASS

BIOLOGY 9648/02
Paper 2 Core Paper 2 September 2013

Additional Materials: Answer Paper 2 hours

READ THESE INSTRUCTIONS FIRST

Write your name and class on all the work you hand in.
Write in dark blue or black pen on both sides of the paper.
You may use soft pencil for any diagrams, graphs or rough working.
Do not use staples, paper clips, highlighters, glue or correction fluid.

Section A
Answer all questions.
For Examiners Use
Section B
Answer any one question. Section A
Circle the question number of the question attempted.
1

3
At the end of the examination, fasten all your work securely together.
The number of marks is given in brackets [ ] at the end of each question or part 4
question.
5

Section B

8 / 9

Total

This document consists of 23 printed pages and 1 blank page.


[Turn over

9648 / 02
630

Section A

Answer all the questions in this section.

1 In translation, mRNA that is produced in transcription is used as the template for the
synthesis of a polypeptide chain which will be further coiled and folded to form a
functional protein. Fig. 1.1 shows a stage during translation.

Fig. 1.1

(a) Name the stage of translation shown in Fig. 1.1. [1]

___________________________________________________________________

(b) State the number of amino acids found on the peptide chain. [1]

___________________________________________________________________

(c) During translation, Enzyme X was used to catalyse the reaction shown in Fig. 1.2.

Fig. 1.2

(i) State the name of Enzyme X. [1]

________________________________________________________________

Jurong JC/JC2 H2 Biology/Prelim/2013 - 2 -


631

(ii) Using the information provided in Fig. 1.1 and Fig. 1.2, describe the process
catalysed by Enzyme X. [3]

________________________________________________________________

________________________________________________________________

________________________________________________________________

________________________________________________________________

________________________________________________________________

________________________________________________________________

________________________________________________________________

________________________________________________________________

(iii) Enzyme X is able to act on all the aminoacyl-tRNA found in the body. Describe
the model by which the aminoacyl-tRNA binds to the enzyme. [2]

________________________________________________________________

________________________________________________________________

________________________________________________________________

________________________________________________________________

________________________________________________________________

________________________________________________________________

Jurong JC/JC2 H2 Biology/Prelim/2013 - 3 -


632

The rate of reaction of Enzyme X is affected by pH within the cytosol where translation is
taking place. Fig. 1.3 was the result of an experiment to investigate how pH affected the
rate of translation in normal cells.

Fig. 1.3

(d) With reference to Fig. 1.3, describe how pH affects the rate of enzyme reaction. [2]

___________________________________________________________________

___________________________________________________________________

___________________________________________________________________

___________________________________________________________________

(e) A toxin introduced into normal cells changes the pH of the cells, leading to a reduced
rate of translation and hence bringing about cell death.

Suggest what the pH of the cells was after the introduction of the toxin and explain
why protein synthesis was affected. [2]

___________________________________________________________________

___________________________________________________________________

___________________________________________________________________

___________________________________________________________________

[Total: 12]

Jurong JC/JC2 H2 Biology/Prelim/2013 - 4 -


633

2 Fig. 2.1 showss a cell thaat has unde


ergone semmi-conserva
ative replicaation and is in the
proccess of und
dergoing telo
ophase and
d cytokinesis.

Fig. 2.1

(a) (i) State precisely when DNA rep


plication occ c cycle. [11]
curs in the cell

______
__________
__________
__________
_________
____________________
______

(ii) State what


w is mean
nt by semi-cconservatio
on replication. [2]

______
__________
__________
__________
_________
____________________
______

______
__________
__________
__________
_________
____________________
______

______
__________
__________
__________
_________
____________________
______

______
__________
__________
__________
_________
____________________
______

______
__________
__________
__________
_________
____________________
______

______
__________
__________
__________
_________
____________________
______

(iii) Describ
be the roles of enzymess used in th
he process. [3]

______
__________
__________
__________
_________
____________________
______

______
__________
__________
__________
_________
____________________
______

______
__________
__________
__________
_________
____________________
______

______
__________
__________
__________
_________
____________________
______

______
__________
__________
__________
_________
____________________
______

______
__________
__________
__________
_________
____________________
______

Jurong J
JC/JC2 H2 Biology/Prelim/2013 - 5 -
634

(b) Describe the events that occur during the stage shown in Fig. 2.1. [3]

___________________________________________________________________

___________________________________________________________________

___________________________________________________________________

___________________________________________________________________

___________________________________________________________________

___________________________________________________________________

An enzyme called telomerase is produced by some cells in the human body. Telomerase
adds nucleotides to the ends of chromosomes.

Table 2.1 shows the telomerase activity and telomere length of some types of cell after
cell division.
Table 2.1

type of cell number of cell telomerase activity telomere length


divisions
bone cell limited absent shortened
cancer cell unlimited high maintained
embryonic stem cell unlimited high maintained
epidermis cell limited absent shortened
liver cell partially limited low slightly shortened
neurone limited absent shortened

(c) Using the information in Table 2.1,

(i) State the consequences of high telomerase activity in a cell. [1]

________________________________________________________________

________________________________________________________________

Jurong JC/JC2 H2 Biology/Prelim/2013 - 6 -


635

(ii) Explain the relationship between high levels of telomerase expression and
cancer. [2]

________________________________________________________________

________________________________________________________________

________________________________________________________________

________________________________________________________________

________________________________________________________________

________________________________________________________________

[Total: 12]

Jurong JC/JC2 H2 Biology/Prelim/2013 - 7 -


636

3 Durring the firstt phase of the


t cell cyclle, G1, therre is a crucial cell-cyclee checkpoin
nt that a
cell must pass to begin th he process tthat leads to cell division. If changges in the structure
s
of D
DNA are dettected durin ng G1, the ccell cycle is halted until DNA is reppaired.

p53
3 gene loca ated on ch
hromosome e 17 codess for p53 and is knnown as a tumour
suppressor geene. Mutatio
ons of one of the two e cause a hhereditary disorder
o p53 gene
known as Li-Frraumeni syn
ndrome.

Fig. 3.1 is a simplified


s flo
ow chart shhowing the sequence of events tthat occurs
s after a
change in the structure
s of DNA is dettected.

1
Fig. 3.1

(a) What is a tu
umour supp
pressor gen
ne? [1]

_________
__________
__________
__________
_________
____________________
______

_________
__________
__________
__________
_________
____________________
______

Jurong J
JC/JC2 H2 Biology/Prelim/2013 - 8 -
637

In the inactive form, p53 forms a p53-protein complex. In the active form, p53 is no
longer attached to the protein.

(b) Suggest why only the active form of p53 can enter the nucleus. [1]

___________________________________________________________________

___________________________________________________________________

(c) Using the information in Fig. 3.1, explain how mutation of p53 gene could lead to
cells continuing to divide even after a change in the structure of DNA is detected. [3]

___________________________________________________________________

___________________________________________________________________

___________________________________________________________________

___________________________________________________________________

___________________________________________________________________

___________________________________________________________________

Jurong JC/JC2 H2 Biology/Prelim/2013 - 9 -


638

The effectiveness of anti-cancer drugs against metastatic colorectal cancer may be


determined by growing human tumours in culture. The effectiveness of two drugs,
T138067 and vinblastine, on the tumours was assessed. The two drugs were added to
the tumours in culture on days 5, 12 and 19. The volumes of the tumours were compared
with the volumes of tumours that were not treated with any drugs.

The results are shown in Fig. 3.2.

Fig. 3.2

(d) With reference to Fig. 3.2,

(i) conclude the effectiveness of T138067 on tumours. [1]

___________________________________________________________________

___________________________________________________________________

___________________________________________________________________

___________________________________________________________________

Jurong JC/JC2 H2 Biology/Prelim/2013 - 10 -


639

(ii) compare the effectiveness of the two drugs when used to treat patients suffering
from metastatic colorectal cancer. [2]

________________________________________________________________

________________________________________________________________

________________________________________________________________

________________________________________________________________

________________________________________________________________

________________________________________________________________

Vinblastine disrupts the formation of the spindle apparatus

(e) Explain how vinblastine acts as an anti-cancer drug. [2]

___________________________________________________________________

___________________________________________________________________

___________________________________________________________________

___________________________________________________________________

[Total: 10]

Jurong JC/JC2 H2 Biology/Prelim/2013 - 11 -


640

4 Fig. 4.1 shows a Human Immunodeficiency Virus.

Fig. 4.1

(a) Name the structures A and B in Fig. 4.1. [1]

A __________________________________________________________________

B __________________________________________________________________

(b) Describe how the viral genome is released into host cell after a conformational
change has occurred in gp41. [2]

___________________________________________________________________

___________________________________________________________________

___________________________________________________________________

___________________________________________________________________

Jurong JC/JC2 H2 Biology/Prelim/2013 - 12 -


641

(c) Fig. 4.2 shows a generalized graph of the relationship between HIV RNA copies and
T lymphocytes counts over the average course of untreated HIV infection.

Fig. 4.2

(i) With reference to Fig. 4.2, describe and explain what happens during the period
of clinical latency. [3]

________________________________________________________________

________________________________________________________________

________________________________________________________________

________________________________________________________________

________________________________________________________________

________________________________________________________________

Jurong JC/JC2 H2 Biology/Prelim/2013 - 13 -


642

(ii) Explain how HIV infection causes death in the 11th year from primary infection. [2]

________________________________________________________________

________________________________________________________________

________________________________________________________________

________________________________________________________________

(d) The management of AIDS normally includes the use of multiple anti-retroviral drugs
that act on different stages of the HIV life cycle in an attempt to control HIV infection.
One such drug is alpha interferon which inhibits an enzyme necessary in the
maturation and release stage of the HIV life cycle.

Identify the enzyme and explain how HIV infection is controlled. [2]

___________________________________________________________________

___________________________________________________________________

___________________________________________________________________

___________________________________________________________________

[Total: 10]

Jurong JC/JC2 H2 Biology/Prelim/2013 - 14 -


643

BLANK PAGE

Jurong JC/JC2 H2 Biology/Prelim/2013 - 15 -


644

5 A plant breeder carried out three crosses as follows:

cross 1 - pure breeding tomato plants with uniformly pigmented red-coloured fruit
were crossed with pure breeding tomato plants with dark patches at the base of the
orange-coloured fruit
cross 2 - the offspring of cross 1 (offspring 1) were crossed with each other
cross 3 - offspring 1 were crossed with pure breeding tomato plants with dark
patches at the base of the orange-coloured fruit

The number of offspring in each phenotype was counted.


Table 5.1 shows the results of the three crosses.

Table 5.1

total number of offspring of each


cross parental phenotype
offspring phenotype

pure breeding pure breeding


plants with red plants with orange
1 X 150 150 red uniformly pigmented fruit
uniformly fruit with dark
pigmented fruit patches

Offspring 1 Offspring 1 24 red fruit with dark patches


plants with red plants with red 98 red uniformly pigmented fruit
2 uniformly X uniformly 172
46 orange fruit with dark patches
pigmented fruit pigmented fruit
4 orange uniformly pigmented fruit

Offspring 1 pure breeding 23 red fruit with dark patches


plants with red plants with orange 83 red uniformly pigmented fruit
3 uniformly X fruit with dark 210
patches 85 orange fruit with dark patches
pigmented fruit
19 orange uniformly pigmented fruit

Jurong JC/JC2 H2 Biology/Prelim/2013 - 16 -


645

(a) Using the symbols R for red fruit and r for orange fruit, D for uniform pigment and d
for dark patches, draw a genetic diagram to explain the results shown in Cross 3. [4]

(b) Explain the deviation of the observed results from the expected dihybrid ratio. [3]

___________________________________________________________________

___________________________________________________________________

___________________________________________________________________

___________________________________________________________________

___________________________________________________________________

___________________________________________________________________

___________________________________________________________________

___________________________________________________________________

Jurong JC/JC2 H2 Biology/Prelim/2013 - 17 -


646

Observations showed that in tomato plants, there are 3 different types of pigmentation at
the base of the fruit. Pigmentation in these fruits is controlled by 2 proteins involved in
the same metabolic pathway for production of pigments, Inhibitor E and Enzyme B.
Uniform pigmentation is produced in the presence of two copies of e alleles and one
copy of B alleles. Dark patches are produced in the presence of two copies of e alleles.
No pigmentation is produced in the presence of at least one copy of E allele.

(c) Suggest a possible mode of inheritance for pigmentation at the base of the fruit. [1]

___________________________________________________________________

(d) Explain how the interaction between the two genes E and B gives rise to
pigmentation. [3]

___________________________________________________________________

___________________________________________________________________

___________________________________________________________________

___________________________________________________________________

___________________________________________________________________

___________________________________________________________________

___________________________________________________________________

___________________________________________________________________

(e) Explain the biochemical basis for this form of epistasis. [2]

[Total: 13]

Jurong JC/JC2 H2 Biology/Prelim/2013 - 18 -


647

6 Fig. 6.1 shows an electron micrograph of a double membranous organelle,


mitochondrion.

Fig. 6.1

(a) Describe the structure of membrane. [5]

___________________________________________________________________

___________________________________________________________________

___________________________________________________________________

___________________________________________________________________

___________________________________________________________________

___________________________________________________________________

___________________________________________________________________

___________________________________________________________________

___________________________________________________________________

___________________________________________________________________

___________________________________________________________________

___________________________________________________________________

(b) From your biological knowledge, explain the adaptation of the mitochondrions double
membrane for its role in the production of energy. [2]

___________________________________________________________________

___________________________________________________________________

___________________________________________________________________

___________________________________________________________________

Jurong JC/JC2 H2 Biology/Prelim/2013 - 19 -


648

Fig. 6.2 shows a stage in respiration that takes place during the oxidation of glucose.

Fig. 6.2

(c) Explain what happens to Molecule X. [3]

___________________________________________________________________

___________________________________________________________________

___________________________________________________________________

___________________________________________________________________

___________________________________________________________________

___________________________________________________________________

___________________________________________________________________

___________________________________________________________________

(d) List two ways in which photophosphorylation differs from oxidative phosphorylation.
[2]

___________________________________________________________________

___________________________________________________________________

___________________________________________________________________

___________________________________________________________________

___________________________________________________________________

___________________________________________________________________

[Total: 12]

Jurong JC/JC2 H2 Biology/Prelim/2013 - 20 -


649

7 The takah, Porphyrio hochstetteri, is a flightless bird that is restricted to a small area of
the South Island of New Zealand. It is one of only two remaining species of large,
flightless, herbivorous birds from New Zealand. The other remaining species is found on
the North Island of New Zealand. Their flighted ancestor came over from Australia
millions of years ago.

The takah was thought to be extinct, but a small population with low genetic variation
was discovered in 1948 among the remote mountains of the South Island. The takah is
a grassland specialist and lives in alpine grassland dominated by tussock grasses as
shown in Fig. 7.1.

Fig. 7.1

(a) Explain why it has been possible for flightless birds, such as the takah, to evolve in
New Zealand. [3]

___________________________________________________________________

___________________________________________________________________

___________________________________________________________________

___________________________________________________________________

___________________________________________________________________

___________________________________________________________________

___________________________________________________________________

___________________________________________________________________

Jurong JC/JC2 H2 Biology/Prelim/2013 - 21 -


650

(b) Differences in the DNA sequences that encode cytochrome b in the two species of
flightless birds from New Zealand have been measured.

(i) When differences in cytochrome b gene are plotted against time since
colonisation, the plot of the line is straight. Describe how this supports the neutral
theory of molecular evolution. [2]

________________________________________________________________

________________________________________________________________

________________________________________________________________

________________________________________________________________

(ii) Suggest why the cytochrome b gene is used to measure changes in DNA
sequences between the two species of flightless birds. [2]

________________________________________________________________

________________________________________________________________

________________________________________________________________

________________________________________________________________

(c) The takah is similar in appearance to another flightless bird found on the
Desventuradas Islands off the coast of Chile even though molecular studies have
shown that they do not share a common ancestor. Explain how biogeography
support the evolutionary deductions based on molecular homology. [2]

___________________________________________________________________

___________________________________________________________________

___________________________________________________________________

___________________________________________________________________

Jurong JC/JC2 H2 Biology/Prelim/2013 - 22 -


651

Another method to determine evolutionary relationships of different animals would be


based on anatomical homology.

Fig. 7.2 shows the relationship between some animals which was determined by
comparing the arrangement of the bones in their forelimbs, which had a basic
pentadactyl structure.

Fig. 7.2

(d) Explain how the anatomical homology in Fig. 7.2 provides evidence to support the
theory of evolution. [2]

___________________________________________________________________

___________________________________________________________________

___________________________________________________________________

___________________________________________________________________

[Total: 11]

Jurong JC/JC2 H2 Biology/Prelim/2013 - 23 -


652

Section B

Answer one question.

Write your answers on the separate answer paper provided.

Your answers should be illustrated by large, clearly labelled diagrams, where appropriate.

Your answers must be in continuous prose, where appropriate.

Your answers must be set out in sections (a), (b) etc., as indicated in the question.

8 (a) Describe the main structural features and function(s) of collagen and glycogen. [8]

(b) Explain how a change in the nucleotide sequence of DNA leads to cystic fibrosis. [8]

(c) Explain how the recessive allele that results in cystic fibrosis may be preserved in a
natural population. [4]

[Total: 20]

9 (a) Describe the main structural features of the lac operon and explain how the presence
of lactose in the growth medium affects the lac operon. [6]

(b) Describe the role of insulin and glucagon in regulating blood glucose concentration.
[8]

(c) Explain how a nerve impulse is passed across a cholinergic synapse. [6]

[Total: 20]

Jurong JC/JC2 H2 Biology/Prelim/2013 - 24 -


653

hh JURONG JUNIOR COLLEGE


JC 2 PRELIMINARY EXAMINATIONS
Higher 2

CANDIDATE
NAME

CLASS

BIOLOGY 9648/02
Paper 2 Core Paper 2 September 2013

Additional Materials: Answer Paper 2 hours

READ THESE INSTRUCTIONS FIRST

Write your name and class on all the work you hand in.
Write in dark blue or black pen on both sides of the paper.
You may use soft pencil for any diagrams, graphs or rough working.
Do not use staples, paper clips, highlighters, glue or correction fluid.

Section A
Answer all questions.
For Examiners Use
Section B
Answer any one question. Section A
Circle the question number of the question attempted
1

3
At the end of the examination, fasten all your work securely together.
The number of marks is given in brackets [ ] at the end of each question or part 4
question.
5

Section B

8 / 9

Total

This document consists of 22 printed pages and 2 blank page.


[Turn over

9648 / 02
654

Section A

Answer all the questions in this section.

1 In translation, mRNA that is produced in transcription is used as the template for the
synthesis of a polypeptide chain which will be further coiled and folded to form a
functional protein. Fig. 1.1 shows a stage during translation.

Fig. 1.1

(a) Name the stage of translation shown in Fig. 1.1. [1]

Elongation;;

(b) State the number of amino acids found on the peptide chain. [1]

5;;

(c) During translation, Enzyme X was used to catalyse the reaction shown in Fig. 1.2.

Fig. 1.2

(i) State the name of Enzyme X. [1]

1. Peptidyl transferase;;

Jurong JC/Prelim/2013 - 2 -
655

(ii) Using the information provided in Fig. 1.1 and Fig. 1.2, describe the process
catalysed by Enzyme X. [3]

1. peptidyl transferase catalyses the formation of a peptide bond;


2. The peptide chain/polypeptide bound to the tRNA in the P site separates
from the tRNA;
3. and attaches (by a peptide bond) to the amino acid carried by the
tRNA/aminoacyl tRNA in the A site;
4. The (peptide bond) is formed between the amino end of the amino acid in
the A site;
5. and the carboxyl end of the peptide/growing chain in the P site;
6. one more amino acid is added to the polypeptide chain;
7. transferring the peptide/growing chain to the tRNA in the A site;

(iii) Enzyme X is able to act on all the aminoacyl-tRNA found in the body. Describe
the model by which the aminoacyl-tRNA binds to the enzyme. [2]

1. According to the induced fit hypothesis/model;


2. The initial shape of the active site of enzyme may not be exactly
complementary to that of the substrate;
3. However, as the substrate enters (and binds) to the active site, it induces a
conformational change of the enzyme;
4. Such that the substrate can fit even more snugly into the active site;
5. Allowing enhanced interaction between the chemical groups of the
substrate and the amino acid residues at active site for successful reaction;

Jurong JC/Prelim/2013 - 3 -
656

The rate of reaction of Enzyme X is affected by pH within the cytosol where translation is
taking place. Fig. 1.3 was the result of an experiment to investigate how pH affected the
rate of translation in normal cells.

Fig. 1.3

(d) With reference to Fig. 1.3, describe how pH affects the rate of enzyme reaction. [2]

1. The rate of translation is at maximum of 22.5 a.u. at optimum pH of pH 7.25;;


2. At pH other than pH 7.25, the rate of translation decreases, reaching 8 a.u.
at pH 5.75 and 20 a.u. at pH 7.75;;

(e) A toxin introduced into normal cells changes the pH of the cells, leading to a reduced
rate of translation and hence bringing about cell death.

Suggest what the pH of the cells were after the introduction of the toxin and explain
why protein synthesis was affected. [2]

1. pH 5.75 - pH 6.75 or pH 7.75; (state a value)


2. low pH alters the ionic charge of the acidic and basic R-groups of amino
acids (of peptidyl transferase);
3. results in breakage of ionic bonds (and hydrogen bonds) between R groups
of amino acids;
4. loss of 3-D conformation of (active site of) peptidyl transferase;
5. peptidyl transferase is denatured (and affects protein synthesis);
6. substrate/aminoacyl tRNA can no longer bind to active site of enzyme to
form enzyme-substrate complex, decreased rate of translation;

[Total: 12]

Jurong JC/Prelim/2013 - 4 -
657

2 Fig. 2.1 showss a cell thaat has unde


ergone semmi-conserva
ative replicaation and is in the
proccess of und
dergoing telo
ophase and
d cytokinesis.

Fig. 2.1

(a) (i) State precisely when DNA rep


plication occ c cycle. [11]
curs in the cell

S phase off interphas


se ;

(ii) State what


w is mean
nt by semi-cconservatio
on replication. [2]

1. The two paarental DNAA strands u eparate intto 2 strandss;


unwinds/se
2. by the brea
aking of hy
ydrogen bo onds betweeen comple ementary b bases;
3. and each parental strand ac ct as a te
emplate fo or the syn nthesis of a new
compleme entary stran
nd;
4. 2 new DNA A moleculees formed, each cons sisting of one
o originaal strand and
a one
newly syntthesized sttrand of DN
NA;

(iii) Describ
be the roles of enzymess used in th
he process. [3]

1. Helicase;
2. unwinding g of DNA double he elix (at orrigins of replication)
r ) by breaking of
hydrogen bonds bettween com mplementarry base pa airs, (allow
wing separaation of
the two strrands);
OR
3. Primase;
4. synthesize es RNA primmer to initiiate replica
ation proce
ess;
OR
5. DNA polym merase;
6. catalyses the form mation off phosphodiester bonds beetween adjacent a
nucleotide owing stra nd, synthe
es (of gro esising daaughter sttrand in 55 to 3
direction);
OR
7. another DN NA polymeerase;
8. removed RNAR primerrs and repllaced with DNA nucle eotides;
OR
9. DNA ligase e;
10. anneal sug gar phosphate backb bone / seals nicks be etween (Okkazaki) fra
agments
via formation of phos
sphodieste er bonds to o form conttinuous strrands;

Jurong J
JC/Prelim/201
13 - 5 -
658

(b) Describe the events that occur during the stage shown in Fig. 2.1. [3]

1. Daughter chromosomes reach opposite poles of the cell;


2. Chromosomes uncoil, (lengthen and become indistinct) to form chromatin;
3. Spindle fibres disintegrate;
4. Nuclear envelope reforms (around the chromatin in each daughter cell);
5. Nuclei formed;
6. Nucleoli reappear (in each daughter nucleus);
7. cleavage furrow begins to form / Cell starts to divide by furrowing;
8. Constriction/contraction of ring of microfilaments in middle/center of the cell
for outside inward

An enzyme called telomerase is produced by some cells in the human body. Telomerase
adds nucleotides to the ends of chromosomes.

Table 2.1 shows the telomerase activity and telomere length of some types of cell after
cell division.

Table 2.1

type of cell number of cell telomerase activity telomere length


divisions
bone cell limited absent shortened
cancer cell unlimited high maintained
embryonic stem cell unlimited high maintained
epidermis cell limited absent shortened
liver cell partially limited low slightly shortened
neurone limited absent shortened

(c) Using the information in Table 2.1,

(i) State the consequences of high telomerase activity in a cell. [1]

1. telomere length is maintained;


2. cell is able to continue to divide indefinitely/unlimited number of cell division;
3. cell might become cancerous;

(ii) Explain the relationship between high levels of telomerase expression and
cancer. [2]

1. telomerase expression is a step in the multi-step process of cancer (where


more than one somatic mutation is involved);
2. telomerase adds specific DNA sequence repeats to telomeres;
3. preventing the shortening of chromosomes to the critical length (beyond which
cell division can no longer occur);
4. apoptosis is not triggered / cells with high levels of telomerase do not undergo
apoptosis/replicative cell senescence;
5. cells continue to divide infinitely/uncontrollably and do not undergo apoptosis;
6. contributing to the development of cancer;
[Total: 12]

Jurong JC/Prelim/2013 - 6 -
659

3 Durring the firstt phase of the


t cell cyclle, G1, therre is a crucial cell-cyclee checkpoin
nt that a
cell must pass to begin th he process tthat leads to cell division. If changges in the structure
s
of D
DNA are dettected durin ng G1, the ccell cycle is halted until DNA is reppaired.

p53
3 gene loca ated on ch
hromosome e 17 codess for p53 and is knnown as a tumour
suppressor geene. Mutatio
ons of one of the two e cause a hhereditary disorder
o p53 gene
known as Li-Frraumeni syn
ndrome.

Fig. 3.1 is a simplified


s flo
ow chart shhowing the sequence of events tthat occurs
s after a
change in the structure
s of DNA is dettected.

1
Fig. 3.1

(a) What is a tu
umour supp
pressor gen
ne? [1]

1. encodes protein
p thatt prevent u ncontrolled cell divis
sion;;

In the inactive
e form, p53 3 forms a p
p53-protein complex. In the activve form, p5
53 is no
long
ger attachedd to the protein.

hy only the active form


(b) Suggest wh m of p53 can
n enter the nucleus.
n [1]

1. active p53 is not com mplexed w with/joined tot the proteein;


2. small enou ugh to pass s through nnuclear poore;
(Cre
edit: revers
se argumen nt for inacttive form)
OR
3. compleme entary shap pe;
4. to the bindding site off the carrie r / channell protein;

Jurong J
JC/Prelim/201
13 - 7 -
660

(c) Using the information in Fig. 3.1, explain how mutation of p53 gene could lead to
cells continuing to divide even after a change in the structure of DNA is detected. [3]

1. non functional p53 synthesized;


2. unable to recognise and bind to DNA;
3. no p21 synthesized;
4. cyclins not blocked/can activate cyclin-dependent kinases;
5. cell cycle not halted/goes beyond G1;
6. progresses into S phase of interphase;
7. DNA replication continues, cell continues into cellular division;

The effectiveness of anti-cancer drugs against metastatic colorectal cancer may be


determined by growing human tumours in culture. The effectiveness of two drugs,
T138067 and vinblastine, against the tumours was assessed. The two drugs were added
to the tumours in culture on days 5, 12 and 19. The volumes of the tumours were
compared with the volumes of tumours that were not treated with any drugs.

The results are shown in Fig. 3.2.

Fig. 3.2

(d) With reference to Fig. 3.2,

(i) conclude the effectiveness of T138067 on tumours. [1]

1. T138067 is effective in treating of tumours / not completely effective in


stopping tumour growth/tumours continue to grow;
2. At 25th day, the volume of tumour treated with T138067 is much smaller at
120mm3 compared to volume of untreated tumour at 630(10)mm3;
3. Difference of 510(10)mm3 between tumour treated with T138067 compared to
untreated tumour / 5.25X smaller;

Jurong JC/Prelim/2013 - 8 -
661

(ii) compare the effectiveness of the two drugs when used to treat patients suffering
from metastatic colorectal cancer. [2]

1. T138067 is a more effective drug than vinblastine / vinblastine is a less


effective drug against tumours;

2. At 25th day, volume of tumour treated with T138067 is 120mm3 while volume of
tumour treated with vinblastine is 370mm3;
3. Tumour treated with T138067 is 3.1X smaller;
4. Tumour treated with T138067 grew at a slower rate of 4.8mm3 while that treated
with vinblastine grew at a faster rate of 14.8mm3;
OR
5. Compared to tumour that is untreated, T138067 slowed down growth by
510(10)mm3/41.3% while vinblastine slowed down growth by
260(10)mm3/81.0%;
6. T138067 is more effective than vinblastine by 1.96X;

7. Up to the 10th/15th day, little difference/similar effectiveness in effectiveness


between vinblastine and T138067 against tumour;
8. Difference of only 10-20mm3;
(Credit: any other valid data analysis)

Vinblastine disrupts the formation of the spindle apparatus

(e) Explain how vinblastine acts as an anti-cancer drug. [2]

1. growth of tumour/cell division involves mitosis;


2. spindle apparatus required for stages of mitosis / use of vinblastine results in
absence of spindle apparatus;
3. no attachment of spindle apparatus to attach to centromeres of chromosomes
during prophase;
4. Chromosomes will not align along metaphase plate during metaphase;
5. Centromeres will not divide and chromatids will not separate during anaphase;
6. mitosis stops / metaphase to anaphase to telophase, cannot proceed / cell
division stops / prevents uncontrolled cell division;
7. preventing tumour cells from undergoing further mitosis/cell division;
(pt 3-5: max 1)
[Total: 10]

Jurong JC/Prelim/2013 - 9 -
662

4 Fig. 4.1 shows a Human Immunodeficiency Virus.

Fig. 4.1

(a) Name the structures A and B in Fig. 4.1. [1]

1. A glycoprotein / gp120;
2. B reverse transcriptase;

(b) Describe how the viral genome is released into host cell after a conformational
change has occurred in gp41. [2]

1. Fusion of viral envelope with the host cells plasma membrane;


2. allows the nucleocapsid to enter the target cell;
3. Uncoating occurs;
4. Capsid is removed by cellular enzymes;

Jurong JC/Prelim/2013 - 10 -
663

(c) Fig. 4.2 shows a generalized graph of the relationship between HIV RNA copies and
T lymphocytes counts over the average course of untreated HIV infection.

Fig. 4.2

(i) With reference to Fig. 4.2, describe and explain what happens during the period
of clinical latency. [3]

1. From the 12th week to 6th year (since primary infection), the HIV RNA copies
remain constant at 3/ less than 10/~ 0 copies per ml plasma;

2. Replication occurs;
3. Reverse transcriptase catalyses the synthesis of a complementary DNA
strand (cDNA) from viral RNA;
4. synthesis of a second DNA strand which is complementary to cDNA;
5. double stranded DNA is integrated/incorporated into the T lymphocytes
DNA/genome (as a provirus) by integrase;
6. number of free infectious virus is low;
7. number of HIV RNA copies is low;

(ii) Explain how HIV infection causes death in the 11th year from primary infection. [2]

1. Replication of HIV destroys T lymphocytes;


2. resulting in low T lymphocytes counts;
3. the immune system weakens and develops AIDS;
4. susceptible to opportunistic infections which can result in death;

Jurong JC/Prelim/2013 - 11 -
664

(d) The management of AIDS normally includes the use of multiple anti-retroviral drugs
that act on different stages of the HIV life cycle in an attempt to control HIV infection.
One such drug is alpha interferon which inhibits an enzyme necessary in the
maturation and release stage of the HIV life cycle.

Identify the enzyme and explain how HIV infection is controlled. [2]

1. HIV protease;
2. HIV polyprotein cannot be cleaved (into individual functional proteins);
3. like capsid proteins, reverse transcriptase and integrase (any 2);
4. virus remained immatured / non-infectious / non-functional;
5. unable to infect other cells/ unable to carry out further infection;

[Total: 10]

Jurong JC/Prelim/2013 - 12 -
665

5 A plant breeder carried out three crosses as follows:

cross 1 - pure breeding tomato plants with uniformly pigmented red-coloured fruit
were crossed with pure breeding tomato plants with dark patches at the base of the
orange-coloured fruit
cross 2 - the offspring of cross 1 (offspring 1) were crossed with each other
cross 3 - offspring 1 were crossed with pure breeding tomato plants with dark
patches at the base of the orange-coloured fruit

The number of offspring in each phenotype was counted.


Table 5.1 shows the results of the three crosses.

Table 5.1

total number of offspring of each


cross parental phenotype
offspring phenotype
pure breeding pure breeding
plants with red plants with orange
1 X 150 150 red uniformly pigmented fruit
uniformly fruit with dark
pigmented fruit patches
Offspring 1 Offspring 1 24 red fruit with dark patches
plants with red plants with red 98 red uniformly pigmented fruit
2 X 172
uniformly uniformly 46 orange fruit with dark patches
pigmented fruit pigmented fruit 4 orange uniformly pigmented fruit
Offspring 1 pure breeding 23 red fruit with dark patches
plants with red plants with orange 83 red uniformly pigmented fruit
3 X 210
uniformly fruit with dark 85 orange fruit with dark patches
pigmented fruit patches 19 orange uniformly pigmented fruit

(a) Using the symbols R for red fruit and r for orange fruit, D for uniform pigment and d
for dark patches, draw a genetic diagram to explain the results in Cross 3. [4]

parental
red uniformly pigmented fruit X orange fruit with dark patches
phenotypes:
parental genotypes: RD rd ;;
X
rd rd
Gametes RD Rd rd ;;
rD rd

Punnett Square
RD Rd rD rd
rd RD Rd rD rd
rd rd rd rd

offspring RD Rd rD rd
: : : ;;
genotype : rd rd rd rd
offspring red uniformly red fruit with orange uniformly orange fruit with ;
: : :
phenotype : pigmented fruit dark patches pigmented fruit dark patches
offspring
83 : 23 : 19 : 85 ;
phenotypic ratio :

Jurong JC/Prelim/2013 - 13 -
666

(b) Explain the deviation of the observed results from the expected dihybrid ratio. [3]

1. Expected dihybrid ratio is 1:1:1:1 while observed test cross results is approx.
4:1:1:4 which deviates from the expected ratio;
2. The 2 genes for colour of fruit and pigmentation are linked;
3. which means they form a linkage group and are usually pass into the same
gamete and are inherited together /do not undergo independent assortment
and segregation during meiosis;
4. crossing over between the 2 linked genes on non-sister chromatids of
homologous chromosomes during prophase 1 of meiosis 1 occur ;
5. to form recombinant gametes Rd and rD;
6. since crossing over is a chance event, there are lower chances of getting
recombinant gametes;
7. thus the observed offspring of test cross are majority of parental combinations
and minority of recombinants combinations;

Observations showed that in tomato plants, there are 3 different types of pigmentation at
the base of the fruit. Pigmentation in these fruits is controlled by 2 proteins involved in
the same metabolic pathway for production of pigments, Inhibitor E and Enzyme B.
Uniform pigmentation is produced in the presence of two copies of e alleles and one
copy of B alleles. Dark patches are produced in the presence of two copies of e alleles.
No pigmentation is produced in the presence of at least one copy of E allele.

(c) Suggest a possible mode of inheritance for pigmentation at the base of the fruit. [1]

Dominant epistasis;;

(d) Explain how the interaction between the two genes E and B gives rise to
pigmentation. [3]

1. The pigmentation is controlled by a pair of genes (gene E/e and gene B/b)
occupying different loci;
2. the epistatic gene determines the production of pigments;
3. the dominant allele (E_) results in no pigmentation;
4. homozygous for the recessive allele (ee) results in uniform pigmentation or
dark patches, depending on the genotype of the gene B/b;
5. the hypostatic gene determines the type/intensity of pigmentation;
6. the dominant allele (B_) determines uniform pigmentation;
7. homozygous for the recessive allele (bb) determines dark patches;
8. In the interaction between these two genes, the presence of allele E hides the
effects of either B_ or bb, producing no pigmentation;
9. E_ is epistatic to (any genotype of the) B/b gene;

Jurong JC/Prelim/2013 - 14 -
667

(e) Explain the biochemical basis for this form of epistasis. [2]

gene e gene B

inhibitor e enzyme B

Precursor dark patches uniform pigmentation;;


(colourless)

Award 1 mark for correct genes and gene products;;


[Total: 13]

Jurong JC/Prelim/2013 - 15 -
668

6 Fig. 6.1 shows an electron micrograph of a double membranous organelle,


mitochondrion.

Fig. 6.1

(a) Describe the structure of membrane. [5]

1. Made up of a phospholipid bilayer;


2. The thickness is about 7nm;
3. The (hydrophilic) phosphate heads of the phospholipids face outwards into the
aqueous environment/water (inside and outside the cell);
4. The (hydrophobic) hydrocarbon chains/ fatty acid tails face inwards and create
a hydrophobic interior (core)/face away from aqueous medium;
5. Hydrophobic fatty acid tails are buried inside the two hydrophilic layers formed
by the phosphate groups;
6. Phospholipids are held by (weak) hydrophobic interactions;
7. Most protein molecules embedded in/float about in the phospholipid bilayer;
8. In an irregular pattern/randomly/forming a fluid mosaic pattern;
9. Peripheral proteins (loosely) bind to the surface of the membrane / attached
regions of the integral proteins;
10. Integral proteins penetrate/span all the way through the membrane;
11. Glycoproteins and glycolipids have (short branching) carbohydrate chains;
12. Contain cholesterol;
13. Membranes are asymmetric/the two sides of a membrane can differ in
composition and function;

Jurong JC/Prelim/2013 - 16 -
669

(b) From your biological knowledge, explain the adaptation of the mitochondrions double
membrane for its role in the production of energy. [2]

1. Membranes impermeable to H+/protons / contain proton pumps;


2. to create proton reservoir/gradient;;
OR
3. Highly folded inner membrane to increase surface area;
4. For embedding of ATP synthase / electron carriers;
5. Movements of protons through the stalked particles release energy for the
synthesis of ATP
OR
6. Compartmentalisation/separation within the mitochondrion;
7. so that reactions may occur in different areas;

Fig. 6.2 shows a stage in respiration that takes place during the oxidation of glucose.

Fig. 6.2

(c) Explain what happens to Molecule X. [3]

1. Molecule X is acetyl CoA


2. Acetyl CoA and oxaloacetate are condensed to form citrate;
3. Oxaloacetate displaces the coenzyme A;
4. Citrate undergoes oxidative decarboxylation to form -ketoglutarate;
5. A molecule of CO2 is lost, (and the remaining 5C compound is oxidised by
NAD+)
6. -ketoglutarate undergoes oxidative decarboxylation to form succinate;
7. A molecule of CO2 is lost, (and the remaining 4C compound is oxidised by
NAD+);
8. Succinate undergoes oxidation to form fumarate;
9. Fumarate is converted to form malate;
10. Malate undergoes oxidation to regenerate oxaloacetate;
Pt 4 and 6: award once

Jurong JC/Prelim/2013 - 17 -
670

(d) List two ways in which photophosphorylation differs from oxidative phosphorylation.
[3]

Features No. Photophosphorylation No. Oxidative phosphorylation

Process 1a In photosynthesis 1b In aerobic respiration

Location 2a Thylakoid membrane of chloroplast 2b Inner membrane of mitochondrion

Involvement of 3a Light energy is required for 3b Light energy is not required


light energy photolysis / required to energise
electrons in special chl a

Source of 4a Energy for synthesis of ATP comes 4b Energy for synthesis of ATP comes
energy for ATP from light from the oxidation of glucose
synthesis

Electron donors 5a Water is the electron donor in non- 5c NADH and FADH2 are electron
cyclic reaction donors

5b Photosystem I is the electron donor


in the cyclic reaction

Electron 6a NADP+ is the final electron acceptor 6c Oxygen is the final electron
acceptors in the non-cyclic reaction acceptor

6b PSI is the electron acceptor in the


cyclic reaction
(Credit: any other valid comparisons)

[Total: 12]

Jurong JC/Prelim/2013 - 18 -
671

7 The takah, Porphyrio hochstetteri, is a flightless bird that is restricted to a small area of
the South Island of New Zealand. It is one of only two remaining species of large,
flightless, herbivorous birds from New Zealand. The other remaining species is found on
the North Island of New Zealand. Their flighted ancestor came over from Australia
millions of years ago.

The takah was thought to be extinct, but a small population with low genetic variation
was discovered in 1948 among the remote mountains of the South Island. The takah is
a grassland specialist and lives in alpine grassland dominated by tussock grasses as
shown in Fig. 7.1.

Fig. 7.1

(a) Explain why it has been possible for flightless birds, such as the takah, to evolve in
New Zealand. [3]

1. Flighted birds migrated to New Zealand;


2. (genetic) variation in the takahs ability to fly due to (spontaneous) mutation;
3. Some birds were flightless while some were flighted;
4. Selection pressure absence of mammals, abundant food / resources on
ground / nesting sites available on the ground / few or no land predators;
5. As flight requires considerable energy;
6. Flightless birds with higher fitness/are better adapted to the environment will
be at a selective advantage;
7. They will survive to maturity, mate / reproduce and pass down favourable
alleles to their offspring;
8. Over time, change in allele frequency occurs, leading to evolution (and
flightless birds predominates);

Jurong JC/Prelim/2013 - 19 -
672

(b) Differences in the DNA sequences that encode cytochrome b in the two species of
flightless birds from New Zealand have been measured.

(i) When differences in cytochrome b gene is plotted against time since colonisation,
the plot of the line is straight. Describe how this supports the neutral theory of
molecular evolution. [2]

1. Plot of line being straight indicates that the rate of mutation is constant /
steady;
2. Mutations are silent and there are small number of changes;
3. There is no effect on the phenotype i.e. cytochrome b of the organisms;
4. No selective advantage or disadvantage is involved;

(ii) Suggest why the cytochrome b gene is used to measure changes in DNA
sequences between the two species of flightless birds. [2]

1. The cytochrome b gene was present in both species and is therefore a


good basis for comparison;;
2. Changes in gene occur very slowly (as mutations are silent);;
3. Changes in the gene is restricted as unfavourable mutations will result in
gene being eliminated from gene pool;;

(c) The takah is similar in appearance to another flightless bird found on the
Desventuradas Islands off the coast of Chile even though molecular studies have
shown that they do not share a common ancestor. Explain how biogeography
support the evolutionary deductions based on molecular homology. [2]

1. Biogeography is the study of the geographical distribution of species;


2. Unrelated species in distinct region may look similar to each other/different
species evolved and have common features;
3. due to convergent evolution;
4. Unrelated species faced similar selection pressures in their environment;
5. due to similar conditions on the different islands;

Jurong JC/Prelim/2013 - 20 -
673

Another method to determine evolutionary relationships of different animals would be


based on anatomical homology.

Fig. 7.2 shows the relationship between some animals which was determined by
comparing the arrangement of the bones in their forelimbs, which had a basic
pentadactyl structure.

Fig. 7.2

(d) Explain how the anatomical homology in Fig. 7.2 provides evidence to support the
theory of evolution. [2]

1. Although the forelimbs had different functions;


2. they basically have the same arrangement/similar structure;
3. which is a basic pentadactyl/ five digit forelimb structure;
4. Therefore they were likely to have originated/derived from a common ancestor;
5. Which is the ancient lobe-finned fish;
6. and had been modified over time/ descent with modification;
[Total: 11]

Jurong JC/Prelim/2013 - 21 -
674

Section B

Answer one question.

Write your answers on the separate answer paper provided.


Your answers should be illustrated by large, clearly labelled diagrams, where appropriate.
Your answers must be in continuous prose, where appropriate.
Your answers must be set out in sections (a), (b) etc., as indicated in the question.

8 (a) Describe the main structural features and function(s) of collagen and glycogen. [8]

(b) Explain how a change in the nucleotide sequence of DNA leads to cystic fibrosis. [8]

(c) Explain how the recessive allele that results in cystic fibrosis may be preserved in a
natural population. [4]

[Total: 20]

9 (a) Describe the main structural features of the lac operon and explain how the presence
of lactose in the growth medium affects the lac operon. [6]

(b) Describe the role of insulin and glucagon in regulating blood glucose concentration.
[8]

(c) Explain how a nerve impulse is passed across a cholinergic synapse. [6]

[Total: 20]

Jurong JC/Prelim/2013 - 22 -
675

Section B

Answer one question.

Write your answers on the separate answer paper provided.


Your answers should be illustrated by large, clearly labelled diagrams, where appropriate.
Your answers must be in continuous prose, where appropriate.
Your answers must be set out in sections (a), (b) etc., as indicated in the question.

8 (a) Describe the main structural features and function(s) of collagen and glycogen. [8]

structural features of glycogen


1. A polysaccharide made from glucose;
2. A backbone of -glucose residues held together by -1, 4 glycosidic bonds;
3. Highly branched with branches formed by -1, 6 glycosidic bonds;
4. angle of these bonds causes the chain to coil helically into a more compact shape;
5. Most of the -OH groups are projected into the interior of the helix;

functions of glycogen
6. food and energy stores in animals / compact nature of glycogen allows packing of
many monomers per unit volume;
7. provides a useful energy reserve for liver and muscles;
8. Hydrolysis of glycogen in liver and muscle cells releases glucose when the
demand for glucose increases. E.g. During exercise;
9. can be hydrolysed by enzymes more efficiently / easily broken down into its
glucose monomers (to be used as respiratory substrates);
10. Large and insoluble nature of glycogen does not allow its diffusion out of the cell
and do not change the osmotic potential within the cell;

structural features of collagen


1. made up of amino acids monomers linked together by peptide bonds;
2. Each (helical) polypeptide is in the shape of a loosely wound helix;
3. high proportion of proline residues;
4. almost every 3rd amino acid in each chain is glycine;
5. glycines small size allows the three strands of the polypeptide to (lie close
together and) form a tight coil;
6. The three polypeptides are wound around each other / twisted into / form a
tropocollagen/triple helix;
7. bound to one another by inter-molecular hydrogen bonds;
8. tropocollagen runs parallel to each other to form fibrils and the ends of the
molecule are staggered;
9. Covalent cross-links form between (the carboxyl end of) one tropocollagen and
(the amino end of) another tropocollagen;
10. Fibrils unite to form fibres;

function of collagen
11. structural protein found in connective tissue / in the basement membrane (in skin,
tendons, cartilage, bones, teeth and the walls of blood vessels of humans and
almost all animals);
[Total: 8]

Jurong JC/Prelim/2013 - 23 -
676

(b) Explain how a change in the nucleotide sequence of DNA leads to cystic fibrosis. [8]

1. Cystic fibrosis (CF) is an inherited disease where the mucus glands are
affected.
2. caused by a mutation of the cystic fibrosis transmembrane conductance
regulator (CFTR) gene found on chromosome 7;
3. a mutation results in deletion of 3 nucleotides CTT;
4. malformed CFTR protein loses an amino acid, phenylalanine;
5. at position 508
6. CFTR protein is a transmembrane chloride channel that allows efflux of
chloride ions out of the epithelial cells;
7. mutation of the CFTR gene results in a change in the 3D conformation/
configuration of the chloride channel;
8. channel is no longer able to transport chloride ions;
9. prevent normal efflux of chloride ions out of epithelial cells;
10. chloride ions accumulate in the epithelial cells;
11. attracting water into the epithelial cells causing thick and sticky mucus;
12. cells that line the passageways of the lungs/pancreas/other organs produce
thick and sticky mucus.
13. mucus obstruct the airways
14. leading to severe problems with breathing/ resulting in bacterial infections in
the lungs/ permanent lung damage/ formation of scar tissue (fibrosis) and
cysts in the lungs;
15. mucus blocks the duct / interferes with the function of the pancreas;
16. preventing enzymes and hormones from reaching the intestines to aid
digestion and assimilation, leading to digestive problems;
17. reduces life expectancy;

(c) Explain how the recessive allele that results in cystic fibrosis may be preserved in a
natural population. [4]

1. Diploidy ;
2. individuals have 2 sets of chromosomes;
3. The diploid nature of (most) eukaryotes hides a considerable amount of
genetic variation from selection;
4. Recessive allele that results in cystic fibrosis is less favourable / harmful in the
present environment than its dominant counterparts/the dominant allele hence
the recessive allele can persist/exist in the heterozygotes individuals in a
population;;
5. Only alleles present in the homozygous recessive organism will be expressed
in the phenotype;
6. and be exposed to (environmental) selection / elimination;
7. Very few of the recessive alleles can be eliminated from the population in each
generation;
8. Known as heterozygote protection;

[Total: 20]

Jurong JC/Prelim/2013 - 24 -
677

9 (a) Describe the main structural features of the lac operon and explain how the presence
of lactose in the growth medium affects the lac operon. [6]

1. An operon is a cluster of genes that make up a single transcription unit;


2. The lac operon encode the enzymes involved in lactose metabolism;

The lac operon consist of:


3. lacP (Promoter:): binding site of RNA polymerase;
4. lacO (Operator): binding site of the lac repressor;
5. CAP Binding Site, binding site of CAP (catabolite activator protein);
6. lacZ codes for -galactosidase;
7. which hydrolyses lactose into glucose and galactose;
8. lacY codes for lac permease;
9. which is a membrane protein for the transport of lactose into the cell;
10. lacA codes for -galactoside transacetylase;
11. which is involved in the hydrolysis of lactose;

12. allolactose is formed when lactose enters the cell and is cleaved by -
galactosidase;
13. allolactose recognises and binds to the allosteric site of the lac repressor;
14. lac repressor changes its conformation from active to inactive;
15. lac repressor unable to bind to the operator;
16. RNA polymerase recognises and binds to promoter;
17. transcribe the genes of the lac operon/ transcription occurs;

Jurong JC/Prelim/2013 - 25 -
678

(b) Describe the role of insulin and glucagon in regulating blood glucose concentration.
[8]

1. Blood glucose concentration increases above normal level of 90 mg per 100 cm3
of blood;
2. Islets of Langerhans of the pancreas detect the deviation;
3. cells of islets of Langerhans secrete insulin;
4. Insulin recognises and binds to receptor sites of receptors on the surface
membrane of target cells;
5. to stimulate glycogenesis, an increase in the rate of conversion of glucose to
glycogen;
6. for storage in liver cells and muscle cells;
7. to stimulate an increase in the rate of respiration using glucose;
8. as respiratory substrate which will be broken down and oxidised to form carbon
dioxide and water;
9. to stimulate an increase in the permeability of plasma membrane to glucose;
10. to increase the rate of uptake of glucose from the blood by almost all body cells
especially muscle cells;
11. to stimulate an increase in the rate of conversion of excess glucose to fats;
12. for storage in adipose tissues;
13. Blood glucose concentration decreases and return back to normal level;
[Max 6m]

14. Blood glucose concentration decreases below normal level of 90 mg per 100 cm3
of blood;
15. Islets of Langerhans of the pancreas detect the deviation;
16. cells of islets of Langerhans secrete glucagon;
17. Glucagon recognises and binds to receptor sites of receptors on the surface
membrane of liver/target cells;
18. To stimulate glycogenolysis, an increase in the rate of breakdown of glycogen to
glucose;
19. to stimulate gluconeogenesis, an increase in the rate of conversion of amino
acids and glycerol to glucose;
20. Blood glucose concentration increases and return back to normal level;

Jurong JC/Prelim/2013 - 26 -
679

(c) Explain how a nerve impulse is passed across a cholinergic synapse. [6]

1. Before the presynaptic membrane is depolarised, there are few Ca2+ ions inside
the synaptic knob;
2. The arrival of an impulse/action potential at the synaptic knob causes voltage-
gated Na+ channels and voltage-gated Ca2+ channels to open;
3. resulting in influx of Na+ ions and Ca2+ ions into the synaptic knob from the
synaptic cleft;
4. The influx of Ca2+ ions causes vesicles containing neurotransmitters/
acetylcholine to move to the presynaptic membrane and fuse with it;
5. emptying the neurotransmitters into the synaptic cleft by exocytosis;
6. The Ca2+ ions are pumped out of the synaptic knob by active transport using ATP;
7. Acetylcholine diffuse across the 20nm synaptic cleft;
8. usually in less than 0.5 ms;
9. Acetylcholine is complementary in shape to specific binding sites of receptors;
10. and recognises and binds to the binding sites of receptors on the postsynaptic
membrane;
11. This changes the shape of the receptor protein, opening chemically-gated Na+
channels on the postsynaptic membrane;
12. Influx of Na+ ions into the cytoplasm along concentration gradient depolarise the
postsynaptic membrane;
13. The membrane potential becomes more positive than resting potential;
14. The local potential that develops is known as an excitatory postsynaptic potential
(EPSP);
15. If the membrane potential of the postsynaptic membrane reaches threshold
potential (-50mV), an action potential is generated in the postsynaptic neurone
and travels down the axon to the next synapse;
16. The synaptic cleft contains an enzyme, acetylcholinesterase, which hydrolyses
each acetylcholine molecule into acetate and choline. The choline is taken back
into the presynaptic neurone to combine with acetyl coA to form acetylcholine;

[Total: 20]

Jurong JC/Prelim/2013 - 27 -
680

JURONG JUNIOR COLLEGE


JC 2 PRELIMINARY EXAMINATIONS
Higher 2

CANDIDATE
NAME

CLASS

BIOLOGY 9648/03
Applications Paper and Planning Question 16 September 2013
Paper 3 2 hours
Additional Materials: Answer Booklet/Paper

READ THESE INSTRUCTIONS FIRST

Write your name and class on all the work you hand in.
Write in dark blue or black pen on both sides of the paper.
You may use soft pencil for any diagrams, graphs or rough working.
Do no use staples, paper clips, highlighters, glue or correction fluid.

Answer all questions.

At the end of the examination, fasten all your work securely together.
The number of marks is given in brackets [ ] at the end of each question or part question.

For Examiners Use

Total

This document consists of 17 printed pages and 1 blank page.


[Turn over
681

Answer all questions.

1 Plasmids used in genetic engineering as a cloning vector include pMOD2-Neo shown in


Fig. 1.1. This plasmid contains genes coding for antibiotic resistance ampicillin
resistance gene (amp) and neomycin resistance gene (neo).

Fig. 1.1

With reference to the plasmid shown in Fig. 1.1,

(a) State two properties of the plasmid which allows it to be used as a cloning vector. [2]

___________________________________________________________________

___________________________________________________________________

___________________________________________________________________

___________________________________________________________________

___________________________________________________________________

___________________________________________________________________

(b) The plasmid pMOD2-Neo and the human DNA containing the gene of interest are cut
with restriction enzymes and cloned using bacteria in genetic engineering.

(i) Explain what is meant by a restriction enzyme. [2]

________________________________________________________________

________________________________________________________________

________________________________________________________________

________________________________________________________________

Jurong JC/JC2 H2 Biology/Prelim/2013 -2-


682

(ii) Suggest why the restriction enzyme will cut the human DNA in many places but
will only cut the plasmid DNA once. [1]

________________________________________________________________

________________________________________________________________

________________________________________________________________

(iii) Suggest why the plasmid should only be cut once. [1]

________________________________________________________________

________________________________________________________________

Jurong JC/JC2 H2 Biology/Prelim/2013 -3-


683

(c) Disease X is an autosomal disease resulting from a single base substitution which
removes a recognition site of the restriction enzyme, HaeIII, as shown in Fig. 1.2.
The enzymes recognition sites on allele 1 and allele 2 are indicated by arrows.

Fig. 1.2

The mutation can be detected by restriction fragment length polymorphism after


cleaving with HaeIII. Fig. 1.3 shows the banding patterns produced by four
individuals.

Fig. 1.3

With reference to Fig. 1.2 and Fig. 1.3,

(i) On allele 1, draw the probe at the position it will hybridise to. [1]

(ii) On allele 2, draw the position of the polymorphic restriction site by marking it with
an asterisk (*). [1]

(iii) State the mode of inheritance for Disease X. [1]

________________________________________________________________

Jurong JC/JC2 H2 Biology/Prelim/2013 -4-


684

(iv) Explain how gel electrophoresis can be used to distinguish between the two
alleles. [4]

________________________________________________________________

________________________________________________________________

________________________________________________________________

________________________________________________________________

________________________________________________________________

________________________________________________________________

________________________________________________________________

________________________________________________________________

________________________________________________________________

________________________________________________________________

(d) Information from the Human Genome Project (HGP) has allowed for better
understanding of the genetic basis of diseases such as Disease X.

Discuss two other benefits arising from HGP. [2]

___________________________________________________________________

___________________________________________________________________

___________________________________________________________________

___________________________________________________________________

[Total: 15]

Jurong JC/JC2 H2 Biology/Prelim/2013 -5-


685

2 Haemophilia B is a blood clotting disorder. Haemophilia B patients are unable to produce


enough human clotting factor IX, also referred to as FIX, which is an essential protein in
the blood clotting process.

The dominant allele coding for FIX was packaged in a modified adeno-associated virus,
AAV8, that has been designed to infect liver cells specifically. The modified AAV8 was
injected into six patients with severe haemophilia B who were producing FIX at less than
1% of normal levels.

Fig. 2.1

Prior to undergoing gene therapy, all six patients were receiving the standard treatment
for severe haemophilia Binjection of purified FIX produced by recombinant DNA
technologyseveral times a month. After receiving the FIX gene therapy, each patient
produced FIX at between 2% to 12% of normal levels. For at least six months, four of the
six patients no longer required purified FIX injections for routine bleeding. The other two
patients required purified FIX injections less frequently than before the therapy.

(a) Suggest and explain why two out of the six patients still require purified FIX injection.
[2]

___________________________________________________________________

___________________________________________________________________

___________________________________________________________________

___________________________________________________________________

(b) Explain why it is sufficient to insert a single dominant allele coding for FIX into each
liver cell. [2]

___________________________________________________________________

___________________________________________________________________

___________________________________________________________________

___________________________________________________________________

Jurong JC/JC2 H2 Biology/Prelim/2013 -6-


686

(c) Explain why individuals who have been treated by this method of gene therapy do
not pass on the dominant allele coding for FIX to their children. [1]

___________________________________________________________________

___________________________________________________________________

(d) State two disadvantages of this type of gene therapy. [2]

___________________________________________________________________

___________________________________________________________________

___________________________________________________________________

___________________________________________________________________

(e) Suggest and explain the advantages of using a viral delivery system for the dominant
allele coding for FIX. [2]

___________________________________________________________________

___________________________________________________________________

___________________________________________________________________

___________________________________________________________________

[Total: 9]

Jurong JC/JC2 H2 Biology/Prelim/2013 -7-


687

3 Investigators often report success in culturing plant tissue using root tips. This technique
has many commercial applications which include the production of large numbers of
genetically identical plants and the culturing of plant cells in a fermenter with culture
medium for the production of secondary products.

(a) (i) Suggest one reason why plant tissue culture was successful using organs such
as root tips. [1]

________________________________________________________________

________________________________________________________________

(ii) Suggest why it is advantageous that plants produced by plant tissue culture are
genetically identical. [1]

________________________________________________________________

________________________________________________________________

In many stages of plant tissue culture, strict aseptic techniques need to be used. One of
the difficulties of culturing plant tissue is in carrying out surface sterilisation of the
explant.

To investigate a suitable method of culturing explants, plant tissue samples were taken
at three different times of the year and placed on a medium containing no fungicide or
antibiotic. Two different sampling methods were used:

discs cut from leaves wiped with bleach;


stem tissues extracted with a sterile hypodermic syringe.

The results of culturing the explants are shown in Fig. 3.1.

Fig. 3.1

Jurong JC/JC2 H2 Biology/Prelim/2013 -8-


688

With reference to Fig. 3.1,

(b) compare the effect of the different methods of taking tissue samples and the time of
the year on fungal or bacterial contamination of the cultured explants. [3]

___________________________________________________________________

___________________________________________________________________

___________________________________________________________________

___________________________________________________________________

___________________________________________________________________

___________________________________________________________________

Jurong JC/JC2 H2 Biology/Prelim/2013 -9-


689

(c) Early attempts to culture plant tissue on a culture medium were not always
successful. Studies have shown that the culture medium must contain a range of
substances - mineral ions, a sugar suitable for plant cells to use as well as plant
growth regulators, to bring about successful plant cell growth and division.

To investigate the role of plant growth regulators, equal masses of plant callus were
cultured for four weeks on media containing different concentrations of two plant
growth regulators: auxin and cytokinin. The results are shown in Table 3.1.

Table 3.1

concentration of plant growth


regulators / mg dm-3 effect of plant growth regulators
treatment
on callus growth
auxin cytokinin
A 2.00 0.00 little or no growth
B 2.00 0.02 growth of roots
increased growth of callus
C 2.00 0.20
with no differentiation
D 2.00 0.50 growth of shoots
E 0.00 0.20 little or no growth

(i) Suggest why a sugar was included in the culture medium. [1]

________________________________________________________________

________________________________________________________________

Jurong JC/JC2 H2 Biology/Prelim/2013 - 10 -


690

With reference to Table 3.1,

(ii) describe the effects of auxin and cytokinin on callus growth. [3]

________________________________________________________________

________________________________________________________________

________________________________________________________________

________________________________________________________________

________________________________________________________________

________________________________________________________________

(iii) Suggest and explain what treatment should be given to the shoots developed
from callus in treatment D to turn them into plantlets. [2]

________________________________________________________________

________________________________________________________________

________________________________________________________________

________________________________________________________________

Plantlets often do not have fully functional cuticle and this causes them to be
inefficient in their use of water. Hence, plantlets might not be able to survive the ex-
vivo acclimatisation process if directly transferred from the culture medium to the
greenhouse.

(iv) Suggest what steps need to be taken to increase the viability of the plantlets. [2]

________________________________________________________________

________________________________________________________________

________________________________________________________________

________________________________________________________________

________________________________________________________________

________________________________________________________________

Jurong JC/JC2 H2 Biology/Prelim/2013 - 11 -


691

(d) In many regions of temperate rice production, severe weed infestations have
removed land from economic rice production. LLRICE62 is adapted for planting in the
temperate rice production regions.

LLRICE62 is a variety of Oryza sativa L. rice produced through genetic modification


to confer tolerance to the herbicide glufosinate ammonium. Herbicide tolerance is
based upon the bar gene, a bialaphos resistance gene, isolated from the soil
microorganism, Streptomyces hygroscopicus. The bar gene encodes the production
of the enzyme, Phosphinothricin-Acetyl-Transferase (PAT). When sprayed with
glufosinate ammonium herbicides, the LLRICE62 plants can continue to grow while
surrounding weeds rapidly die.

Based on the above information, suggest

(i) the benefit of cultivating LLRICE62. [1]

________________________________________________________________

________________________________________________________________

(ii) two ethical implications of this genetic engineering of LLRICE62. [2]

________________________________________________________________

________________________________________________________________

________________________________________________________________

________________________________________________________________

[Total: 16]

Jurong JC/JC2 H2 Biology/Prelim/2013 - 12 -


692

BLANK PAGE

Jurong JC/JC2 H2 Biology/Prelim/2013 - 13 -


693

4 Planning question

You are required to plan, but not carry out, an investigation into the effect of alcohol
concentration on the membrane permeability of beetroot tissue. Beetroot is a starchy
edible root from the Beta vulgaris plant whose cells contain a water-soluble red pigment,
betacyanin, in their vacuoles. This pigment cannot pass through membranes unless the
membrane is damaged.

Fig. 4.1 shows a spectrophotometer which can be used to measure colour intensity by
measuring absorbance of different wavelengths of light as they pass through the sample.
A cuvette filled with sample to be placed into the sample holder is also shown. Generally,
a sample of higher colour intensity will absorb more light energy at a specific wavelength.

Fig. 4.1

Your plan should:

have a clear and helpful structure such that the method you use is able to be repeated
by anyone reading it,
be illustrated by relevant diagram(s), if necessary,
identify the independent and dependent variable,
describe the method with the scientific reasoning used to decide the method so that
the results are as accurate and reliable as possible,
include layout of results tables and graphs with clear headings and labels,
use the correct technical and scientific terms,
include reference to safety measures to minimise any risks associated with the
proposed experiment.

Jurong JC/JC2 H2 Biology/Prelim/2013 - 14 -


694

Your planning must be based on the assumption that you have been provided with the
following equipment and material, which you must use:

a large piece of beetroot tissue with the skin removed,


test tubes 100mm tall with 11mm inside diameter,
scalpel,
10.0% alcohol,
Spectrophotometer set at 460nm,
cuvettes,
distilled water.

You should select from the following apparatus:


white tile,
ruler,
timer e.g. stopwatch or stopclock,
any normal laboratory glassware e.g. different sized beakers, measuring cylinders or
syringes for measuring volume.

[Total: 12]

Jurong JC/JC2 H2 Biology/Prelim/2013 - 15 -


695

Jurong JC/JC2 H2 Biology/Prelim/2013 - 16 -


696

Jurong JC/JC2 H2 Biology/Prelim/2013 - 17 -


697

Free-response question

Write your answer to this question on the separate answer paper provided.

Your answer:

should be illustrated by large, clearly labelled diagrams, where appropriate;


must be in continuous prose, where appropriate;
must be set out in sections (a), (b) etc., as indicated in the question.

5 (a) Describe the polymerase chain reaction (PCR) and explain the advantages of this
procedure. [8]

(b) Describe, with examples, how a eukaryotic gene can be incorporated into E.coli. [6]

(c) Explain how RFLP analysis facilitates the process of DNA fingerprinting and genomic
mapping. [6]

[Total: 20]

Jurong JC/JC2 H2 Biology/Prelim/2013 - 18 -


698

JURONG JUNIOR COLLEGE


JC 2 PRELIMINARY EXAMINATIONS
Higher 2

CANDIDATE
NAME

CLASS

BIOLOGY 9648/03
Applications Paper and Planning Question 16 September 2013
Paper 3 2 hours
Additional Materials: Answer Booklet/Paper

READ THESE INSTRUCTIONS FIRST

Write your name and class on all the work you hand in.
Write in dark blue or black pen on both sides of the paper.
You may use soft pencil for any diagrams, graphs or rough working.
Do no use staples, paper clips, highlighters, glue or correction fluid.

Answer all questions.

At the end of the examination, fasten all your work securely together.
The number of marks is given in brackets [ ] at the end of each question or part question.

For Examiners Use

Total

This document consists of 17 printed pages and 1 blank page.


[Turn over

9648 / 3
699

BLANK PAGE

Jurong JC/JC2 H2 Biology/Prelim/2013 -2-


700

Answer all questions.

1 Plasmids used in genetic engineering as a cloning vector include pMOD2-Neo shown in


Fig. 1.1. This plasmid contains genes coding for antibiotic resistance ampicillin
resistance gene (amp) and neomycin resistance gene (neo).

Fig. 1.1

With reference to the plasmid shown in Fig 1.1,

(a) State two properties of the plasmid which allows it to be used as a cloning vector. [2]

1. Has an origin of replication;


2. Allows pMOD2-Neo/plasmid and foreign gene/GOI to replicate independently of
bacterial genome within the host cell;
3. To give multiple copies of the plasmid and gene within one bacterium;
OR
4. Has an ampicillin resistance gene
5. Allows for the selection of (successfully) transformed bacteria containing (both
recombinant and non-recombinant) plasmids;;
R: Selection of transformed bacteria with recombinant plasmid, survival of bacteria
not plasmids
OR
6. Has a neo gene
7. Allows for the selection of (transformed) cells with recombinant plasmids;;
OR
8. Has multiple cloning site/restriction sites;
9. can be cut by specific restriction enzymes to insert foreign gene/GOI;
OR
10. has genetic markers
11. Allows for the selection of transformed cells with both recombinant and non -
recombinant) plasmids;;

Jurong JC/JC2 H2 Biology/Prelim/2013 -3-


701

(b) The plasmid pMOD2-Neo and the human DNA containing the gene of interest are cut
with restriction enzymes and cloned using bacteria in genetic engineering.

(i) Explain what is meant by a restriction enzyme. [2]

1. Restriction enzyme recognises and binds;


2. to short sequences of bases of DNA (restriction sites);
3. The restriction site is 4-6 base pairs in length;
4. Catalyses the breaking of phosphodiester bonds between specific
nucleotides / cut DNA at specific points;
5. To produce restriction fragments with blunt/sticky ends;

(ii) Suggest why the restriction enzyme will cut the human DNA in many places but
will only cut the plasmid DNA once. [1]

1. restriction enzymes only cut DNA at specific base sequence/recognition


site;
2. sequence of bases/recognition site (on which enzyme acts) occurs once in
plasmid but many times in human DNA;

(iii) Suggest why the plasmid should only be cut once. [1]

1. To prevent non-specific and random DNA fragments from being generated;;


2. To transfer the gene of interest into a specific location in the plasmid;;
OWTTE

Jurong JC/JC2 H2 Biology/Prelim/2013 -4-


702

(c) Disease X is an autosomal disease resulting from a single base substitution which
removes a recognition site of the restriction enzyme, HaeIII as shown in Fig. 1.2. The
enzymes recognition sites on allele 1 and allele 2 are indicated by arrows.

Fig 1.2

The mutation can be detected by restriction fragment length polymorphism after


cleaving with HaeIII. Fig 1.3 shows the banding patterns produced by four individuals.

Fig. 1.3

With reference to Fig 1.2 and Fig. 1.3,

(i) On allele 1, draw the probe at the position it will hybridise to. [1]

(ii) On allele 2, draw the position of the polymorphic restriction site by marking it with
an asterisk (*). [1]

(iii) State the mode of inheritance for Disease X. [1]

Autosomal recessive;;

Jurong JC/JC2 H2 Biology/Prelim/2013 -5-


703

(iv) Explain how gel electrophoresis can be used to distinguish between the two
alleles. [4]

1. The two alleles Allele 1 (A1) and Allele 2 (A2), differ in their base sequences
such that A1 has 3 HaeIII restriction sites while A2 has 2 HaeIII restriction
sites;;
2. Restriction digestion of the DNA with A1 by HaeIII will produce 3 restriction
fragments of 3.2kb, 2.8kb and 1.7kb;
3. Restriction digestion of the DNA with A2 by HaeIII will produce 2 restriction
fragments of 3.2kb and 4.5kb;
4. These fragments are separated out by gel electrophoresis, the 4.5kb
fragment from A2 is a longer fragment, faces more resistance from gel
matrix, will move slower and be found closest to the well (or vice versa);;
5. Different band patterns for A1 and A2 will result after gel electrophoresis;
6. and when the DNA fragments are stained with ethidium bromide, they can
be visualised under UV light;

(d) Information from the Human Genome Project (HGP) has allowed for better
understanding of the genetic basis of diseases such as Disease X.

Discuss two other benefits arising from HGP. [2]

1. Ref to Genetic Testing + elaboration;;


2. Ref to Gene Therapy + elaboration;;
3. Ref to Pharmacogenomics+ elaboration;;
4. Ref to DNA forensics + elaboration;;

[Total: 15]

Jurong JC/JC2 H2 Biology/Prelim/2013 -6-


704

2 Haemophilia B is a blood clotting disorder. Haemophilia B patients are unable to produce


enough human clotting factor IX, also referred to as FIX, which is an essential protein in
the blood clotting process.

The dominant allele coding for FIX was packaged in a modified adeno-associated virus,
AAV8, that has been designed to infect liver cells specifically. The modified AAV8 was
injected into six patients with severe haemophilia B who were producing FIX at less than
1% of normal levels.

Fig 2.1

Prior to undergoing gene therapy, all six patients were receiving the standard treatment
for severe haemophilia Binjection of purified FIX produced by recombinant DNA
technologyseveral times a month. After receiving the FIX gene therapy, each patient
produced FIX at between 2% to 12% of normal levels. For at least six months, four of the
six patients no longer required purified FIX injections for routine bleeding. The other two
patients required purified FIX injections less frequently than before the therapy.

(a) Suggest and explain why two out of the six patients still require purified FIX injection.
[2]

1. Treatment is not as successful;

2. most liver cells had not taken up the dominant allele coding for FIX/functional
FIX allele during viral gene delivery / Only some of liver cells contains
dominant allele coding for FIX/functional FIX allele;
3. leading to low expression of the dominant allele coding for FIX/functional FIX
allele;;
4. Lower concentration of FIX produced;
OR
5. Gene delivered is not long lasting;
6. because the dominant allele coding for FIX/functional FIX allele delivered
remained in the cytoplasm is degraded after some time/earlier;;
7. AAV8 have low integration efficiency;

Jurong JC/JC2 H2 Biology/Prelim/2013 -7-


705

(b) Explain why it is sufficient to insert a single dominant allele coding for FIX into each
liver cell. [2]

1. Normal functional allele is dominant/the dominant allele of the gene


concerned;
2. a single copy of dominant allele is sufficient/can mask the effect of the
recessive/mutant allele ;
3. Expression of the normal allele would result in the synthesis of a normal
functional protein product;
4. restoring target cell to normal state/correcting functions/altering the phenotype
of the target cell;

(c) Explain why individuals who have been treated by this method of gene therapy do
not pass on the dominant allele coding for FIX to their children. [1]

1. Treatment targets the liver/somatic cells and not germ cells/gametes;;


OR
2. gametes of these individuals who have been treated will not contain the
dominant allele coding for FIX, so it will not get inherited;;

(d) State two disadvantages of this type of gene therapy. [2]

1. allele may be inserted into tissue other than target with unknown
consequences;;
2. Gene therapy is commonly conducted on somatic cells. The natural death of
treated cells as well as the rapidly dividing nature of somatic cells makes the
gene therapy short-lived hence patients need to be treated on a frequent
basis;;
3. Viral vector introduced into the tissue may be recognized as foreign particle
and trigger immune system to mount an attack which reduces the
effectiveness of gene therapy;;
4. Viral vector may regain virulence and cause an immune response in the
patient;;
5. low integration efficiency;;
(Credit: any other valid data analysis / reverse answers)

(e) Suggest and explain the advantages of using a viral delivery system for the dominant
allele coding for FIX. [2]

1. Specific targeting, the virus is able to deliver genes to specific target tissues;;
2. Greater efficiency, the virus is able to transfer the gene into the cell more
effectively;;
3. The virus is able to introduce the normal DNA / allele into the nucleus;;

[Total: 9]

Jurong JC/JC2 H2 Biology/Prelim/2013 -8-


706

3 Investigators often report success in culturing plant tissue using root tips. This technique
has many commercial applications which include the production of large numbers of
genetically identical plants and the culturing of plant cells in a fermenter with culture
medium for the production of secondary products.

(a) (i) Suggest one reason why plant tissue culture was successful using organs such
as root tips. [1]

1. meristematic cells which are undifferentiated cells/rapidly dividing cells;;


2. totipotent cell able to divide indefinitely / able to differentiate into all cell
types of the new plant;;

(ii) Suggest why it is advantageous that plants produced by plant tissue culture are
genetically identical. [1]

1. all possess the desirable features of the stock plants/transgenic plants with
improved / new desired traits;;
OR
2. ensures product uniformity, this reliability is an important sales
advantage;;

In many stages of plant tissue culture, strict aseptic techniques need to be used. One of
the difficulties of culturing plant tissue is in carrying out surface sterilisation of the
explant.

To investigate a suitable method of culturing explants, plant tissue samples were taken
at three different times of the year and placed on a medium containing no fungicide or
antibiotic. Two different sampling methods were used:

discs cut from leaves wiped with bleach;


stem tissues extracted with a sterile hypodermic syringe.

The results of culturing the explants are shown in Fig 3.1.

Fig. 3.1

Jurong JC/JC2 H2 Biology/Prelim/2013 -9-


707

With reference to Fig. 3.1,

(b) compare the effect of the different methods of taking tissue samples and the time of
the year on fungal or bacterial contamination of the cultured explants. [3]

different methods
1. stem tissue has less contamination than leaf discs;
2. average 66.0% vs average 7.33% lesser contaminations / 52% - 65% lesser
contaminations;
3. stem tissue has average 9X lesser contamination than leaf discs / 7.5X to 12.8X
lesser contamination than leaf discs;
(Credit: any other valid data analysis / reverse answers)
4. extraction with a sterile hypodermic syringe is a better method of taking
samples;

time of year
5. both treatments has highest % non-contamination in January;
6. 74% for stem tissue and 9% for leaf disc;

7. lowest % non-contamination for stem tissue is in April while the lowest % non-
contamination for leaf discs is in August;
8. 60% for stem tissue and 5% for leaf disc;
(Credit: any other valid data analysis / reverse answers)
9. culturing of explants should be done in January;

(c) Early attempts to culture plant tissue on a culture medium were not always
successful. Studies have shown that the culture medium must contain a range of
substances - mineral ions, a sugar suitable for plant cells to use as well as plant
growth regulators, to bring about successful plant cell growth and division.

To investigate the role of plant growth regulators, equal masses of plant callus were
cultured for four weeks on media containing different concentrations of two plant
growth regulators: auxin and cytokinin. The results are shown in Table 3.1.

Table 3.1

concentration of plant growth


regulators / mg dm-3 effect of plant growth regulators
treatment
on callus growth
auxin cytokinin
A 2.00 0.00 little or no growth
B 2.00 0.02 growth of roots
increased growth of callus
C 2.00 0.20
with no differentiation
D 2.00 0.50 growth of shoots
E 0.00 0.20 little or no growth

Jurong JC/JC2 H2 Biology/Prelim/2013 - 10 -


708

(i) Suggest why a sugar was included in the culture medium. [1]

1. Photosynthesis is not yet occurring / occurring at a low rate, therefore


sugars is not being synthesised;
2. Sugars are required as energy source for respiration/ATP production/
respiratory substrate;
OR
3. Source of carbon for carbohydrates like starch, sucrose, and organic
molecules proteins or enzymes/other suitable named organic molecules;

With reference to Table 3.1,

(ii) describe the effects of auxin and cytokinin on callus growth. [3]

1. both cytokinin and auxin needed for cell division/growth / little or no growth
if only one present;;
2. highest conc. of auxin cf. cytokinin, ratio of 100:1/100X more auxin, only
triggers formation of roots growth;;
3. lowest conc. of auxin cf. cytokinin, ratio of 4:1/4X more auxin, only triggers
formation of shoots;;
4. intermediate auxin cf. cytokinin concentration, ratio of 10:1/10X more auxin,
triggers formation of more callus/stimulates cell division/growth/mitosis;;

(iii) Suggest and explain what treatment should be given to the shoots developed
from callus in treatment D to turn them into plantlets. [2]

1. remove from treatment D and implement treatment B/description of


treatment B / increase conc of auxin to 50mg dm-3;;
2. stimulates the growth of roots to become a plantlet;;

Jurong JC/JC2 H2 Biology/Prelim/2013 - 11 -


709

Plantlets often do not have fully functional cuticle and this causes them to be
inefficient in their use of water. Hence, plantlets might not be able to survive the ex-
vivo acclimatisation process if directly transferred from the culture medium to the
greenhouse.

(iv) Suggest what steps need to be taken to increase the viability of the plantlets. [2]

1. Plantlets are removed from the culture vessel;


2. washed thoroughly in running tap water to remove traces of agar;
3. transferred to plastic cups containing enriched soil covered over by
polythene bags for hardening;
4. Slowly weaning the plantlets from a high-humidity, low light, warm
environment to a normal growth environment for the species;
5. After two weeks in enriched soil, the acclimatized plantlets can be
transplanted to a potting mixture;

(d) In many regions of temperate rice production, severe weed infestations have
removed land from economic rice production. LLRICE62 is adapted for planting in the
temperate rice production regions.

LLRICE62 is a variety of Oryza sativa L. rice produced through genetic modification


to confer tolerance to the herbicide glufosinate ammonium. Herbicide tolerance is
based upon the bar gene, a bialaphos resistance gene, isolated from the soil
microorganism, Streptomyces hygroscopicus. The bar gene encodes the production
of the enzyme, Phosphinothricin-Acetyl-Transferase (PAT). When sprayed with
glufosinate ammonium herbicides, the LLRICE62 plants can continue to grow while
surrounding weeds rapidly die.

Based on the above information, suggest

(i) the benefit of cultivating LLRICE62 [1]

1. Due to less competition from weeds, there is increase in crop yield for the
farmers;;
2. Allows for removal of weeds that are difficult to control, to make lands
available for farming/ allow rice cultivation to return;;

(ii) the ethical implications of this genetic engineering of LLRICE62. [2]

1. Violation of natural organisms' intrinsic values;;


2. Tampering with nature by mixing genes among species;;
3. Introduced genes escaping from a transgenic crop into related weeds
through crop-to-weed hybridization, leading to herbicide-resistant weeds.
(Credit: any other valid points)

[Total: 16]

Jurong JC/JC2 H2 Biology/Prelim/2013 - 12 -


710

4 Planning question

You are required to plan, but not carry out, an investigation into the effect of alcohol
concentration on the membrane permeability of beetroot tissue. Beetroot is a starchy
edible root from the Beta vulgaris plant whose cells contain a water-soluble red pigment,
betacyanin, in their vacuoles. This pigment cannot pass through membranes unless the
membrane is damaged.

Fig. 4.1 shows a spectrophotometer which can be used to measure colour intensity by
measuring absorbance of different wavelengths of light as they pass through the sample,
as well as the cuvette with sample to be placed into the sample holder. Generally, a
sample of higher colour intensity will absorb more light energy at a specific wavelength.

Fig. 4.1

Your plan should:

have a clear and helpful structure such that the method you use is able to be repeated
by anyone reading it,
be illustrated by relevant diagram(s), if necessary,
identify the independent and dependent variable,
describe the method with the scientific reasoning used to decide the method so that
the results are as accurate and reliable as possible,
include layout of results tables and graphs with clear headings and labels,
use the correct technical and scientific terms,
include reference to safety measures to minimise any risks associated with the
proposed experiment.

Jurong JC/JC2 H2 Biology/Prelim/2013 - 13 -


711

Your planning must be based on the assumption that you have been provided with the
following equipment and material, which you must use:

a large piece of beetroot tissue with the skin removed,


test tubes 100mm tall with 11mm inside diameter,
scalpel,
10.0% alcohol,
Spectrophotometer set at 460nm,
cuvettes,
distilled water.

You should select from the following apparatus:


white tile,
ruler,
timer e.g. stopwatch or stopclock,
any normal laboratory glassware e.g. different sized beakers, measuring cylinders or
syringes for measuring volume.

[Total: 12]

Jurong JC/JC2 H2 Biology/Prelim/2013 - 14 -


712

Mark Scheme for Experiment


1. Theoretical consideration or rationale of the plan to justify the practical procedure-
prediction

2. Theoretical consideration or rationale of the plan to justify the practical procedure-


explanation

3. Identification of variables independent and dependent variables.

4. Description of method used i.e. using a spectrophotometer, cuvettes, calibration


of spectrophotometer using distilled water.

5. Identification of other variables to be kept constant e.g. washing and rinsing cut
beetroot cylinders before start of experiment

6. Alcohol concentration as measured by at least 5 different alcohol concentrations


(A! range of 1%-10%).

7. Serial dilution shown.

8. Description of how beetroot tissues are to be used in experiment e.g. cutting into
cylinders of diameter 5 mm using corkborer, trimming to length of 1.0 cm
cylinders OR cut to specific dimensions

9. Description of control.

10. Replicates and at least two repeats.

11. Calculation of mean/average (average calculated in table or in graph).

12. Layout of results table with appropriate headings.

13. Graph with average absorbance/ A.U. (y-axis) against alcohol concentration/ % (x-
axis).

14. Safety precautions / risk hazards identified.

15. The correct use of technical and scientific terms.

[12 marks]

Jurong JC/JC2 H2 Biology/Prelim/2013 - 15 -


713

(3) Variables
The independent variable in this experiment is alcohol concentration while the
dependent variable is absorbance.

(5) Other variables to be kept constant (any 1)


washing and rinsing cut beetroot cylinders before start of experiment.
calibration of spectrophotometer using distilled water.
Length (if using corkborer) / dimensions / shape and size of beetroot tissue used
(if using scalpel).
Time the beetroot cylinders were immersed in the alcohol solution.
Same volume of ethanol used (e.g. 10-15cm3)
Same cuvette used
Same beetroot used
Constant temperature (e.g. 28C -30C)
(Any other valid variables)

(1) Prediction of the likely outcome of the experiment


As concentration of alcohol increases, membrane permeability of beetroot tissue
increases, leading to an increase in absorbance value.

(2) Explain the likely outcome of the experiment (b+c)


Pigment molecule is (a)large/ hydrophilic and cannot pass through (b)hydrophobic
core of phospholipid bilayer.

Increasing alcohol concentration increases (c)damage to/destroy/disrupting the


integrity of cell membrane/regular arrangement of the membrane due to
(d)denaturation/disruption of tertiary structure of (membrane) proteins/
phospholipids dissolve.

vacuole membrane/tonoplast AND cell surface membrane (e)lose selective


permeability/become more permeable, allowing pigment to diffuse out of the cell.

*(a) to (e) Pt 15.

(4) Method (any 2)


Use a spectrophotometer to measure absorbance value.
Use cuvettes to contain samples.
Calibration of spectrophotometer with distilled water.

Jurong JC/JC2 H2 Biology/Prelim/2013 - 16 -


714

Procedure (includes safety and reliability of results)


1. Prepare 10 cm3 each of 2.0%, 4.0%, 6.0%, 8.0% and 10.0% alcohol solution (Pt 6: 5
conc 0%-10%) and a control of 0.0% alcohol solution (Pt 9: Control) by diluting the
10.0% stock alcohol solution as indicated in the table below. Place them in
labelled test tubes.

(7) Dilution table


Concentration of alcohol
Test Volume of 10% Volume of distilled
solution/%
tubes alcohol/cm3 water/cm3
(Pt 6: 5 conc 0%-10%)
A 2.0 2.0 8.0
B 4.0 4.0 6.0
C 6.0 6.0 4.0
D 8.0 8.0 2.0
E 10.0 10.0 0.0
Control
0.0 0.0 20.0
(Pt 9)

2. (Pt 8: preparation of beetroot) Cut the beetroot tissue into cylinders of a diameter
of 5 mm (less than 11mm) using a cork borer and trim the length to 1.0 cm (less
than 100mm) on a white tile using a scalpel OR cut to specific dimensions.
3. (Pt 5: variables kept constant) Wash and rinse beetroot cylinders in distilled water
to remove any pigments which may leak out during cutting. There should be no
further leakage of colour from the beetroot tissue.
4. (Pt 4: Method) Calibrate the spectrophotometer with cuvette containing distilled
water.
5. Place 2 beetroot cylinders (appropriate dimension) into test tube A.
6. (Pt 5: variables kept constant) Immediately start timing for 5 minutes using a
stopwatch.
7. (Pt 4: Method) After 5 minutes, remove 1 cm3 of solution from test tube A and
transfer to a cuvette to measure absorbance value using a spectrophotometer.

(Pt 10)
8. Repeat step 5-7 to obtain another 2 readings.
9. Repeat step 5-8 for 2%, 4%, 6%, 8%, 10% alcohol solutions.
10. Repeat entire experiment twice.

11. Plot a graph of average absorbance / A.U. against alcohol concentration /%.

Jurong JC/JC2 H2 Biology/Prelim/2013 - 17 -


715

(12) Table showing absorbance value at varying alcohol concentrations

Absorbance / A.U.
Alcohol concentration/% Average
Reading 1 Reading 2 Reading 3
(Pt 11)
0.0
2.0
4.0
6.0
8.0
10.0

(13) Graph showing average absorbance/ AU against alcohol concentration/ %

(14) Risk Assessment (any 1)


State: Handle sharp objects such as scalpel and cork borer carefully or place them
away from main work area after use
Explain: to avoid injuring/cutting oneself.

State: Handle liquid chemicals such as the 10% alcohol with care or wear protective
goggles
Explain: to prevent eyes from being in contact with alcohol.

State: Handle fragile objects like test tubes with care. Arrange test tubes neatly in the
test tube rack, ensuring that they are placed where they cannot be knocked over or
roll off the bench.
Explain: To prevent breakage

Jurong JC/JC2 H2 Biology/Prelim/2013 - 18 -


716

Free-response question

Write your answer to this question on the separate answer paper provided.

Your answer:

should be illustrated by large, clearly labelled diagrams, where appropriate;


must be in continuous prose, where appropriate;
must be set out in sections (a), (b) etc., as indicated in the question.

5 (a) Describe the polymerase chain reaction (PCR) and explain the advantages of this
procedure. [8]

Stage 1 [2]
1. Denaturation;
2. Hydrogen bonds holding the double-stranded DNA fragment break;
3. The double-stranded DNA separate and dissociates into single-stranded DNA;
4. By heating to 95oC; (A) 90-100oC

Stage 2 [2]
5. Annealing of DNA primers/oligonucleotides;
6. DNA primers base pair with complementary sequences;
7. At the 3 end of the single stranded DNA;
8. By cooling to about 65oC; (A) 30-65oC

Stage 3 [2]
9. Primer extension;
10. Using the DNA primer, the heat stable polymerase-Taq polymerase
synthesizes the rest of the fragment (new strand of DNA);
11. Nucleotides are added to the 3' ends on both primers;
12. By heating to 72oC; (A) 60-75oC

Advantages [2]
13. High sensitivity: PCR can amplify minute amount of DNA, this is especially
useful in forensic work where DNA may not be present in high quality;;
14. Speed and ease of use: PCR can amplify many copies/very large amounts of
DNA in a few hours;;
15. Robustness: PCR can permit amplification of specific sequences from
material in which the DNA is badly degraded or embedded in a medium from
which conventional DNA isolation is problematic;;
16. Automation of PCR: use of thermostable Taq polymerase, can withstand high
temperatures without being denatured allows for the automation of PCR;;

Jurong JC/JC2 H2 Biology/Prelim/2013 - 19 -


717

(b) Describe, with examples, how a eukaryotic gene can be incorporated into E.coli. [6]

1. Isolate plasmids;
2. Named example of plasmid-e.g. pBR322/pVector;
3. Named example of human gene-e.g. insulin gene

Either
4. Isolate insulin gene from human DNA;
Or
5. Isolate insulin gene using PCR/reverse transcription using mRNA to form cDNA
and add linker DNA;

6. Cut both the human DNA and plasmid with the same restriction enzyme;
7. to get complementary sticky ends;
8. Named example of RE e.g. EcoRI/BamHI/PstI/HindIII;
9. Mix the desired gene and plasmid vector in vitro, allowing the sticky ends to
reanneal (in a specific, complementary manner);
10. via complementary base pairing;
11. formation of hydrogen bonds between the complementary bases;
12. DNA ligase is added to the mixture;
13. to seal the nicks between the fragments/join sugar-phosphate backbone
together/join adjacent nucleotides together;
14. via the formation of phosphodiester bonds between adjacent nucleotides;
15. so that a recombinant plasmid is formed;
16. Add CaCl2 to the mixture and subject the mixture to heat
shock/electroporation;
17. so that transient pores formed on the bacteria cell membrane;
18. Allowing the recombinant plasmid to enter host E.coli cells via transformation;

Jurong JC/JC2 H2 Biology/Prelim/2013 - 20 -


718

(c) Explain how RFLP analysis facilitates the process of DNA fingerprinting and genomic
mapping. [6]

DNA fingerprinting:
1. DNA fingerprinting is a technology that identifies particular individuals using
properties of their DNA;
2. Certain loci within human chromosomes contain tandemly repeated
sequences (e.g. minisatellites);
3. The number of tandem repeats (at each minisatellite) varies substantially
among individuals;
4. and is highly polymorphic;
5. (minisatellites are also referred to as loci with a variable number of tandem
repeats (VNTRs));
6. Different number of tandem repeats leads to generation of (different)
restriction fragments;
7. that are of different molecular weight and length;
8. giving rise to unique DNA fingerprint;
9. Each individuals combination is unique, having inherited one allele for each
locus from each parent;

10. DNA fingerprint of an individual is then analysed / compared against another


individual to detect matching band pattern;
OR
11. Used for forensic analysis for crime investigation / paternity determination /
trace evolutionary lineage;

Genomic Mapping:
12. Genomic mapping is a reconstruction of the entire set of chromosomes for a
given organism;
13. showing the relative position of every gene / genetic markers on a
chromosome;
14. RFLPs greatly increased the number of genetic markers available for mapping
the human genome;
Reject: Gene markers / genome markers / marker genes;
15. An RFLP serves as a genetic marker for a particular locus in the genome,
(analogous to an allele of a particular gene);
16. RFLPs are used as genetic markers to construct linkage map;
17. The frequency with which two RFLP markers or an RFLP marker and a certain
allele for a gene is inherited together is a measure of the closeness of the 2
gene loci on a chromosome and the relative distance between 2 gene loci;;
18. Based on the recombination frequencies / frequency of crossing over between
2 RFLP markers / cross-over values of 2 RFLP markers, the linkage map is
constructed;
19. The order of the genetic markers / the relative distances between genetic
markers along a chromosome can be determined;

Jurong JC/JC2 H2 Biology/Prelim/2013 - 21 -


719
Class Adm
A No

Candid
date Name
e:

2013 Prelimin
P nary Examinaation 2
Pre-U
University 3

Biolog
gy Highe
er 2 Sylllabus 96
648/01
255 September 2013

1 hrr 15 min

Addition
nal materiall: Multiple
e Choice An
nswer Shee
et

READ T
THESE INS
STRUCTION
NS FIRST

Write in
n soft pencil.
Do not use stapless, paper clip
ps, highlightters, glue orr correction fluid.
Write yyour name, Centre number and index numb ber on the Answer Shheet in the spaces
provideed unless this has beenn done for yyou.

There a uestions on this paper . Answer all questions


are forty qu s. For each question th
here are
four posssible answ
wers A, B, C and D.

Choose
e the one yo
ou considerr correct an
nd record yo
our choice in
i soft pen
ncil on the separate
s
Answerr Sheet.

he instructtions on the
Read th e Answer S
Sheet very carefully.

Each co
orrect answ
wer will score educted for a wrong answer.
e one markk. A mark wiill not be de
Any rou
ugh working
g should be done in thiss booklet.

Calcula
ators may be
e used.

This ques
stion pape r consists of 22 printed pages.

[Tu
urn over
720
2

Answer all questions.

1. Vicilin is the major protein in pea.

Which row correctly describes the structure of vicilin?

primary secondary tertiary quaternary


structure structure structure structure

-helix and
amino acid association of
-pleated sheet folding of each
A. sequence of one three
regions of each polypeptide
polypeptide polypeptides
polypeptide

-helix and
amino acid association of
-pleated sheet folding of each
B. sequence of one three
regions of each polypeptide
polypeptide polypeptides
polypeptide

-helix and
association of amino acid
-pleated sheet folding of each
C. three sequence of one
regions of each polypeptide
polypeptides polypeptide
polypeptide

-helix and
association of amino acid
folding of each -pleated sheet
D. three sequence of one
polypeptide regions of each
polypeptides polypeptide
polypeptide

9648/01/PU3 Prelim 2/2013


721
3
2. The diameters of some atoms when they form bonds are given in the table.

Atom Single bond / nm Double bond / nm


H 0.060 -
O 0.132 0.110
N 0.140 0.120
C 0.154 0.134

The approximate length of the amino acid shown below was estimated using the
figures in the table.

What would be the approximate length of a dipeptide formed using this amino acid?

A. 0.8 nm

B. 1.2 nm

C. 1.5 nm

D. 1.9 nm

9648/01/PU3 Prelim 2/2013


[Turn over
722
4

3. The diagrams show the results of an investigation into the composition of different
mixtures of amino acids. Each mixture of amino acids was separated using
chromatography. Each chromatogram was then turned through 90 and the amino
acids separated again by electrophoresis.

Which diagram shows an amino acid mixture in which the solubility of some of the
amino acids is the same but the charge on those particular amino acids is different?

9648/01/PU3 Prelim 2/2013


723
5

4. Fractionation
n is used to
o separate plant cell componentts accordingg to their size
s and
de
ensity.

m shows the main stepss in fractiona


The diagram ation of a plant cell.

D
DCPIP and buffer soluttion were a added to eaach sedimen nt and the mixtures le
eft in the
lig
ght for fiftee he oxidised DCPIP to bbe reduced.
en minutes. Sediment 2 caused th

W
Which organ
nelles are prresent in se
ediment 2?

A
A. nuclei

B
B. mitochon
ndria

C
C. chloropla
asts

D
D. ribosome
es
9648/0
01/PU3 Prelim 2/2013
[Tu
urn over
724
6

5. Nocodazole is a chemical used in the study of mitosis. It causes all mitotic cells to stop
dividing at metaphase.

Which statements correctly identify how this chemical might work?

1 inhibits chromatin condensing in the nucleus


2 prevents replication of the centrioles
3 stops sister chromatids migrating to opposite poles

A. 3 only

B. 1, 2 and 3

C. 1 and 2 only

D. 1 and 3 only

6. What is the maximum number of hydrogen bonds in a length of DNA containing 600
base pairs?

A. 300

B. 600

C. 1200

D. 1800

7. Which correctly describes the structure and role of a type of nucleic acid?

A. double stranded, partly base paired, unpaired bases to bind to a ribosome

B. double stranded, base sequence acts as a template for DNA synthesis

C. single stranded, partly base paired, binds to tRNA

D. single stranded, base sequence acts as a template for RNA synthesis

9648/01/PU3 Prelim 2/2013


725
7

8. In order to replicate, the ends of a eukaryotic chromosome contain a special sequence


of DNA called a telomere. Human telomeres consist of repeating TTAGGG sequences
which extend from the ends of the chromosomal DNA.

When cells undergo mitotic division, some of these repeating sequences are lost. This
results in a shortening of the telomeric DNA.

What is a consequence of the loss of repeating DNA sequences from the telomeres?

A. The cell will begin the synthesis of different proteins.

B. The cell will begin to differentiate as a result of the altered DNA.

C. The number of mitotic divisions the cell can make will be limited.

D. The production of mRNA will be reduced.

9. The table gives tRNA anticodons for four amino acids.

amino acid tRNA anticodon


asparagine UUA
glutamic acid CUU
proline GGA
threonine UGG

A cell makes a polypeptide with the amino acid sequence:

glutamic acid asparagine threonine proline

What was the sequence of bases on the strand of the DNA which was complementary
to the mRNA from which this polypeptide was formed?

A. CTTTTATGGGGA

B. CUUUUAUGGGGA

C. GAAAATACCCCT

D. GAAAAUACCCCU

9648/01/PU3 Prelim 2/2013


[Turn over
726
8

10. Which statements about the genetic code are correct?

1 The genetic code has redundancy and is degenerate.


2 There is only one codon for the amino acid methionine.
3 Prokaryotes generally use the same genetic code as eukaryotes.
4 Codons act as stop and start signals during transcription and translation.
5 mRNA codons have the same nucleotide sequence as DNA triplet codes.

A. 1, 2 and 3

B. 1, 2 and 4

C. 1, 4 and 5

D. 2 and 4 only

11. The diagram shows the major components of the lac operon.

Which statement is correct?

A. 1 is a ribosome, 2 is a t-RNA molecule and 3 is a phosphorylated amino acid, the


activator, so lactose-digesting enzyme can be made.

B. 1 is mRNA polymerase, 2 is -galactosidase, the inducer and 3 is the repressor,


so lactose-digesting enzyme cannot be made.

C. 1 is the repressor, 2 is a -galactosidase molecule and 3 is lactose, the promoter,


so lactose-digesting enzyme can be made.

D. 1 is mRNA polymerase, 2 is the repressor and 3 is lactose, the inducer, so lactose


digesting enzyme can be made.

9648/01/PU3 Prelim 2/2013


727
9

12. Which type of control of gene expression is commonly found in both prokaryotes and
eukaryotes?

A. transcriptional

B. post translational

C. post transcriptional

D. translational

13. What is the normal function of tumour suppressor genes?

A. to change the coding sequences in proto-oncogenes so that they become


oncogenes

B. to encode proteins that stimulate cell proliferation

C. to prevent the stimulating activity of cellular oncogenes

D. to prevent replication of cell's DNA

14. What is an example of translational control of gene expression?

A. activation of proteins by folding or enzymatic cleavage

B. addition of chemical groups, such as phosphate groups, to free amino acids in the
cytoplasm

C. binding of protein factors to specific sequences in mRNA preventing ribosomes


attaching

D. formation of disulphide bridges in the protein being formed

15. Which type of enzyme enables a retrovirus to prepare its genetic information for
insertion into a host cell's DNA?

A. DNA polymerase

B. integrase

C. restriction enzyme

D. reverse transcriptase

9648/01/PU3 Prelim 2/2013


[Turn over
728
10

16. A symbiont may be defined as a species in which individuals live in a long-term,


intimate and beneficial relationship with hosts of a different species. As the name
suggests, endosymbionts live within their hosts.

Which statement best provides evidence that mitochondria and chloroplasts are
endosymbionts?

A. Proteins encoded by the nucleus are exported to these organelles.

B. Their inner membrane has different structure from other intracellular membranes.

C. They are surrounded by more than one membrane.

D. They contain their own ribosomes.

17. Gene mutations in either the BRCA1 or the BRCA2 genes are responsible for the
majority of hereditary breast cancer in humans.

The proteins produced by the two genes migrate to the nucleus where they interact
with other proteins, such as those produced by the tumour suppressor gene, p53 and
the DNA repair gene, RAD51.

Which combination of gene activity is most likely to result in breast cancer?

Key: gene produces normal protein


X gene produces abnormal protein or no protein

gene
BRCA1 or
p53 RAD51
BRCA2
A.
B. X
C. X
D. X X X

9648/01/PU3 Prelim 2/2013


729
11

18. Human immunodeficiency virus (HIV) is a retrovirus. After infecting a host cell, viral
DNA is produced which is incorporated into the DNA of the host cell. The modified host
genome now codes for the production of new HIV particles.

Which could be used as a potential treatment to slow down the spread of HIV?

1 inhibitors of restriction endonucleases


2 inhibitors of reverse transcriptase
3 restriction endonucleases
4 reverse transcriptase

A. 1 and 4 only

B. 2 and 3 only

C. 1 only

D. 2 only

19. One hypothesis about the origin of viruses is that they existed before cellular life forms,
later evolving into parasites of cellular organisms. Groups with an ancient, common
origin tend to share conserved sequences of DNA or RNA.

Which observation about virus genomes supports the view that viruses existed before
cellular life forms and only much later evolved into parasites?

A. All viruses have genes that code for capsid components.

B. Introns of eukaryotic genes have common features with viral genomes.

C. Large virus genomes have genes that originate in host cells.

D. Viral genomes have conserved genes not found in any other genomes

9648/01/PU3 Prelim 2/2013


[Turn over
730
12

20. The diagram shows an investigation into bacterial genetics

To grow mutant bacteria X To grow mutant bacteria Y


need methionine (met-) do not need methionine
and biotin (bio-) (met+) and biotin (bio+)
do not need threonine need threonine (thr-) and
(thr+) and leucine (leu+) leucine (leu-)


Grow together in the presence of methionine,

biotin, threonine and leucine


Grow together without methionine,
biotin, threonine and leucine


Growth of some bacteria that
are met+, bio+, thr+, leu+

Which process or processes could explain these results?

1 transformation
2 transduction
3 conjugation

A. 1 only

B. 3 only

C. 1 and 2

D. 1 and 3

9648/01/PU3 Prelim 2/2013


731
13

21. What is necessary for an organism to respond to changes in its internal environment?

A. a communication system between receptors and effectors

B. a communication system involving nerve cells

C. negative feedback from receptor to effector

D. positive feedback from effector to receptor

22. A snake venom causes death by leading to paralysis of muscles. It exerts its effect at
synapses.

The statements below were put forward by scientists as possible explanations for the
effects of this venom.

1 It interferes with the binding of neurotransmitter vesicles to the membranes.


2 It binds with neurotransmitter receptor sites.
3 It blocks calcium and sodium channels.
4 It destroys the myelin sheath of the neurone.
5 It binds with neurotransmitter.

Which statements should be investigated further?

A. 1, 2, 3 and 5 only

B. 2, 4 and 5 only

C. 1 and 4 only

D. 1, 2, 3, 4 and 5

23. Vibrio cholerae produces a toxin that binds to a cell surface membrane receptor on
intestinal cells of the host. The toxin permanently activates the G protein in target cells,
causing them to lose water rapidly. When a person is infected with cholera they suffer
severe dehydration.

What statement best describes the V. cholerae toxin

A. It is an example of a second messenger molecule.

B. It disrupts normal signal transduction in the cell.

C. It is a lipid-soluble molecule.

D. It acts as a neurotransmitter.

9648/01/PU3 Prelim 2/2013


[Turn over
732
14

24. The graphs show the response of sensory neurones to increasing pressure on the
Pacinian corpuscles (receptors found in deep layers of the skin that senses vibratory
pressure and touch.) in mammalian skin.

Which conclusion about increasing pressure is correct?

A. As pressure increases the depolarisation of the corpuscle and sensory neurone


increases.

B. As pressure increases the axon terminal depolarisation reaches a value that


depolarises the sensory neurone.

C. If the corpuscle is stimulated for long enough the depolarisation of the sensory
neurone increases.

D. If the corpuscle is stimulated for long enough the axon terminal depolarisation
reaches a value that depolarises the sensory neurone.

9648/01/PU3 Prelim 2/2013


733
15

25. The following graph shows the relationship between light intensity and net oxygen
uptake or output by a particular green plant.

Which of the following statement is true?

A. At a light intensity of 10 units, the rate of photosynthesis is zero.

B. At a light intensity of 10 units, the rate of aerobic respiration is zero.

C. At a light intensity of 10 units, oxygen produced by photosynthesis is equal to the


oxygen used by aerobic respiration.

D. At a light intensity of 10 units, oxygen produced by photosynthesis is equal to


twice the oxygen used by aerobic respiration.

26. What causes phenotypic variation among organisms of the same genotype?

A. continuous variation within the species

B. different varieties of the same species

C. living in different environments

D. mutation

9648/01/PU3 Prelim 2/2013


[Turn over
734
16

27. One of the many recessive mutations of the CFTR gene changes one amino acid in
the region of the CFTR protein that binds ATP. The graph shows the effect of different
concentrations of ATP on normal and mutant CFTR proteins.

Which correctly describes individuals who are homozygous for this mutation?

1 Their CFTR protein cannot bind ATP and cannot act as an ion channel.
2 Their CFTR protein binds ATP less readily than normal CFTR protein.
3 They produce CFTR protein that must bind ATP to function as an ion
channel.
4 They produce a mixture of normal and mutant CFTR protein, both of which
can act as an ion channel.

A. 1 only

B. 2 only

C. 2 and 3 only

D. 2 and 4 only

9648/01/PU3 Prelim 2/2013


735
17

28. Many plants are not fertilised by pollen from their own flowers. This is known as self-
incompatibility. In any individual species, a single gene, the S gene, is responsible and
it may have many different alleles.

If a pollen grain has an S allele which matches an allele in the genotype of the stigma
then the pollen grain fails to germinate or the pollen tube fails to grow through the style.

The genotype of the stigma of a flower is S3S4.

Which pollen grains would germinate?

A. 4 only

B. 2 and 4 only

C. 1 only

D. 3 and 4 only

29. Two areas of molecular biology that have received considerable attention in
evolutionary studies are the genetic code and cytochrome C. Cytochrome C is an
essential component of all respiratory electron transport chains.

Which statements lend evidence to the ideas that

all living organisms are related


there is a single, rather than a multiple, origin of life?

1 The almost universal nature of the genetic code is a result of evolutionary


convergence from multiple lineages.
2 The sequence of amino acids in cytochrome C is similar in organisms that
are from similar environments or with similar metabolic demands.
3 The majority of organisms have the same, or similar, amino acid
sequences for cytochrome C.
4 When transferred into a very dissimilar organism, a gene coding for
cytochrome C will lead to the expression of a protein that will function in the
other organism.

A. 2 and 3 only

B. 1 and 2 only

C. 3 and 4 only

D. 1, 3 and 4 only

9648/01/PU3 Prelim 2/2013


[Turn over
736
18

30. The following statements relate to molecular phylogenetics.

1 Lines of descent from a common ancestor to present-day organisms have


undergone similar, fixed rates of DNA mutation.

2 Organisms with similar base sequences in their DNA are closely related to
each other.

3 The number of differences in the base sequences of DNA of different


organisms can be used to construct evolutionary trees.

4 The proportional rate of fixation of mutations in one gene relative to the


rate of fixation of mutations in other genes stays the same in any given line
of descent.

Which statements, when taken together, suggest the existence of a molecular clock
that enables scientists to estimate the time at which one species might have diverged
from another?

A. 1 and 2

B. 1 and 4

C. 2 and 3

D. 3 and 4

31. Based on binomial nomenclature, which two species are most closely related?

1 Common barberry (Berberis vulgaris)


2 Canadian bunchberry (Cornus canadensis)
3 Smooth blackberry (Rubus canadensis)
4 Canadian barberry (Berberis canadensis)

A. 1 and 4

B. 1 and 3

C. 2 and 4

D. 3 and 4

9648/01/PU3 Prelim 2/2013


737
19

32. The graphs represent the frequency of alleles in species X, Y and Z during and after
selection.

In which species does evolution take place?

A. X only

B. Y only

C. Y and Z

D. none of X, Y nor Z

33. Why can restriction fragment length polymorphism (RFLP) be used to detect inherited
diseases?

A. Different alleles vary in the sequence and number of base pairs.

B. The DNA fragments are the same length within each gene.

C. The restriction site is unaffected by the number of base pairs.

D. Alleles of different genes have different molecular masses.

9648/01/PU3 Prelim 2/2013


[Turn over
738
20

34. Ripening of fruit, for example figs, is caused by ethylene gas, a lipid-soluble compound
that diffuses easily from cell to cell as well as to neighbouring plants. Plant tissues
produce ethylene at certain developmental stages and in response to stressful
conditions, such as heat or injury.

Plant cells have special ethylene-binding receptors. When ethylene is detected, a


series of reactions follows inside the cell. This results in the production of pectinases to
break down cell walls and amylases to convert carbohydrates into simple sugars.

Which of the following statements is consistent with the information above?

A. Ethylene stimulates cell elongation which enables fruit to grow.

B. The scratching of the surfaces of harvested figs speeds up ripening.

C. Changes triggered by ethylene would discourage animals from consuming fruit.

D. Ethylene receptors are located on the external surfaces of the cell surface
membrane.

35. Stem cells are widely used in medical research.

Which property of stem cells makes them particularly useful in this research?

A. They can be fused together to form a zygote.

B. They can divide and eventually give rise to a whole organism.

C. They can divide and be made to differentiate into various types of cell.

D. They will continue to divide indefinitely.

36. Which of these processes could increase crop yield?

P inserting genes for vitamin A production into rice


Q inserting genes for pest resistance into cotton
R inserting genes for herbicide resistance into oilseed rape
S inserting genes for nitrogen fixation into soya beans

A. P and Q

B. Q and R

C. R and S

D. S and P

9648/01/PU3 Prelim 2/2013


739
21

37. Which genetic modification could increase the yield of a crop measured as mass of
crop produced per unit area per year?

A. winter resistant plants

B. delayed ripening in the fruits

C. more essential amino acids in the seeds

D. more vitamin A in the grain

38. The following figures show two different methods used to sequence DNA.

What of the following statements is correct?

A. Method 1 requires 4 slots for the loading of the DNA.

B. Method 2 uses a laser beam to separate the individual nucleotides.

C. Method 1 uses fluorescent dyes to distinguish between the different nucleotides.

D. Method 2 requires the DNA to be loaded at the positive end on the sequencing
gel.
9648/01/PU3 Prelim 2/2013
[Turn over
740
22

39. Gene therapy is a way of treating genetic disease by introducing a piece of DNA into
the cells of an affected individual. Liposomes can be used for gene therapy as they
target the cells affected by a genetic disease.

What feature of a cell surface membrane allows the liposome to target cells affected by
a genetic disease?

A. carrier molecules

B. phosphate groups

C. receptor molecules

D. protein channels

40. Small samples from crime scenes can be genetically profiled (DNA finger printed).

Which are necessary parts of a successful genetic profiling process?

Key: present
X absent
crime scene ethidium bromide
PCR
sample and X-rays

A. red blood cells X


B. saliva X
C. semen X
D. skin cells X

9648/01/PU3 Prelim 2/2013


741
Class Adm
A No

Candid
date Name
e:

2013 Prelimin
P nary Examinaation 2
Pre-U
University 3

Biolog
gy Highe
er 2 Sylllabus 96
648/01
255 September 2013

1 hrr 15 min

Addition
nal materiall: Multiple
e Choice An
nswer Shee
et

READ T
THESE INS
STRUCTION
NS FIRST

Write in
n soft pencil.
Do not use stapless, paper clip
ps, highlightters, glue orr correction fluid.
Write yyour name, Centre number and index numb ber on the Answer Shheet in the spaces
provideed unless this has beenn done for yyou.

There a uestions on this paper . Answer all questions


are forty qu s. For each question th
here are
four posssible answ
wers A, B, C and D.

Choose
e the one yo
ou considerr correct an
nd record yo
our choice in
i soft pen
ncil on the separate
s
Answerr Sheet.

he instructtions on the
Read th e Answer S
Sheet very carefully.

Each co
orrect answ
wer will score educted for a wrong answer.
e one markk. A mark wiill not be de
Any rou
ugh working
g should be done in thiss booklet.

Calcula
ators may be
e used.

This ques
stion pape r consists of 22 printed pages.

[Tu
urn over
742
2

Answer all questions.

1. Vicilin is the major protein in pea.

Ans: B
Which row correctly describes the structure of vicilin?

primary secondary tertiary quaternary


structure structure structure structure

-helix and
amino acid association of
-pleated sheet folding of each
A. sequence of one three
regions of each polypeptide
polypeptide polypeptides
polypeptide

-helix and
amino acid association of
-pleated sheet folding of each
B. sequence of one three
regions of each polypeptide
polypeptide polypeptides
polypeptide

-helix and
association of amino acid
-pleated sheet folding of each
C. three sequence of one
regions of each polypeptide
polypeptides polypeptide
polypeptide

-helix and
association of amino acid
folding of each -pleated sheet
D. three sequence of one
polypeptide regions of each
polypeptides polypeptide
polypeptide

9648/01/PU3 Prelim 2/2013


743
3
2. The diameters of some atoms when they form bonds are given in the table.

Atom Single bond / nm Double bond / nm


H 0.060 -
O 0.132 0.110
N 0.140 0.120
C 0.154 0.134

The approximate length of the amino acid shown below was estimated using the
figures in the table.

What would be the approximate length of a dipeptide formed using this amino acid?

A. 0.8 nm
Ans: B
B. 1.2 nm

C. 1.5 nm

D. 1.9 nm

9648/01/PU3 Prelim 2/2013


[Turn over
744
4

3. The diagrams show the results of an investigation into the composition of different
mixtures of amino acids. Each mixture of amino acids was separated using
chromatography. Each chromatogram was then turned through 90 and the amino
acids separated again by electrophoresis.

Which diagram shows an amino acid mixture in which the solubility of some of the
amino acids is the same but the charge on those particular amino acids is different?

Ans: B

9648/01/PU3 Prelim 2/2013


745
5

4. Fractionation
n is used to
o separate plant cell componentts accordingg to their size
s and
de
ensity.
Ans: C
m shows the main stepss in fractiona
The diagram ation of a plant cell.

D
DCPIP and buffer soluttion were a added to eaach sedimen nt and the mixtures le
eft in the
lig
ght for fiftee he oxidised DCPIP to bbe reduced.
en minutes. Sediment 2 caused th

W
Which organ
nelles are prresent in se
ediment 2?

A
A. nuclei

B
B. mitochon
ndria

C
C. chloropla
asts

D
D. ribosome
es
9648/0
01/PU3 Prelim 2/2013
[Tu
urn over
746
6

5. Nocodazole is a chemical used in the study of mitosis. It causes all mitotic cells to stop
dividing at metaphase.

Which statements correctly identify how this chemical might work?

1 inhibits chromatin condensing in the nucleus


2 prevents replication of the centrioles
3 stops sister chromatids migrating to opposite poles

A. 3 only

B. 1, 2 and 3
Ans: A
C. 1 and 2 only

D. 1 and 3 only

6. What is the maximum number of hydrogen bonds in a length of DNA containing 600
base pairs?

A. 300
Ans: D
B. 600

C. 1200

D. 1800

7. Which correctly describes the structure and role of a type of nucleic acid?

A. double stranded, partly base paired, unpaired bases to bind to a ribosome Ans B

B. double stranded, base sequence acts as a template for DNA synthesis

C. single stranded, partly base paired, binds to tRNA

D. single stranded, base sequence acts as a template for RNA synthesis

9648/01/PU3 Prelim 2/2013


747
7

8. In order to replicate, the ends of a eukaryotic chromosome contain a special sequence


of DNA called a telomere. Human telomeres consist of repeating TTAGGG sequences
which extend from the ends of the chromosomal DNA.

When cells undergo mitotic division, some of these repeating sequences are lost. This
results in a shortening of the telomeric DNA.

What is a consequence of the loss of repeating DNA sequences from the telomeres?

A. The cell will begin the synthesis of different proteins.


Ans: C
B. The cell will begin to differentiate as a result of the altered DNA.

C. The number of mitotic divisions the cell can make will be limited.

D. The production of mRNA will be reduced.

9. The table gives tRNA anticodons for four amino acids.

amino acid tRNA anticodon


asparagine UUA
glutamic acid CUU
proline GGA
threonine UGG

A cell makes a polypeptide with the amino acid sequence:

glutamic acid asparagine threonine proline

What was the sequence of bases on the strand of the DNA which was complementary
to the mRNA from which this polypeptide was formed?

A. CTTTTATGGGGA
Ans: A
B. CUUUUAUGGGGA

C. GAAAATACCCCT

D. GAAAAUACCCCU

9648/01/PU3 Prelim 2/2013


[Turn over
748
8

10. Which statements about the genetic code are correct?

1 The genetic code has redundancy and is degenerate.


2 There is only one codon for the amino acid methionine.
3 Prokaryotes generally use the same genetic code as eukaryotes.
4 Codons act as stop and start signals during transcription and translation.
5 mRNA codons have the same nucleotide sequence as DNA triplet codes.

A. 1, 2 and 3
Ans: A
B. 1, 2 and 4

C. 1, 4 and 5

D. 2 and 4 only

11. The diagram shows the major components of the lac operon.
Ans: D

Which statement is correct?

A. 1 is a ribosome, 2 is a t-RNA molecule and 3 is a phosphorylated amino acid, the


activator, so lactose-digesting enzyme can be made.

B. 1 is mRNA polymerase, 2 is -galactosidase, the inducer and 3 is the repressor,


so lactose-digesting enzyme cannot be made.

C. 1 is the repressor, 2 is a -galactosidase molecule and 3 is lactose, the promoter,


so lactose-digesting enzyme can be made.

D. 1 is mRNA polymerase, 2 is the repressor and 3 is lactose, the inducer, so lactose


digesting enzyme can be made.

9648/01/PU3 Prelim 2/2013


749
9

12. Which type of control of gene expression is commonly found in both prokaryotes and
eukaryotes?

A. transcriptional Ans A

B. post translational

C. post transcriptional

D. translational

13. What is the normal function of tumour suppressor genes?

A. to change the coding sequences in proto-oncogenes so that they become


oncogenes

B. to encode proteins that stimulate cell proliferation Ans C

C. to prevent the stimulating activity of cellular oncogenes

D. to prevent replication of cell's DNA

14. What is an example of translational control of gene expression?


Ans: C

A. activation of proteins by folding or enzymatic cleavage

B. addition of chemical groups, such as phosphate groups, to free amino acids in the
cytoplasm

C. binding of protein factors to specific sequences in mRNA preventing ribosomes


attaching

D. formation of disulphide bridges in the protein being formed

15. Which type of enzyme enables a retrovirus to prepare its genetic information for
insertion into a host cell's DNA?
Ans D
A. DNA polymerase

B. integrase

C. restriction enzyme

D. reverse transcriptase

9648/01/PU3 Prelim 2/2013


[Turn over
750
10

16. A symbiont may be defined as a species in which individuals live in a long-term,


intimate and beneficial relationship with hosts of a different species. As the name
suggests, endosymbionts live within their hosts.

Which statement best provides evidence that mitochondria and chloroplasts are
endosymbionts?
Ans: A
A. Proteins encoded by the nucleus are exported to these organelles.

B. Their inner membrane has different structure from other intracellular membranes.

C. They are surrounded by more than one membrane.

D. They contain their own ribosomes.

17. Gene mutations in either the BRCA1 or the BRCA2 genes are responsible for the
majority of hereditary breast cancer in humans.

The proteins produced by the two genes migrate to the nucleus where they interact
with other proteins, such as those produced by the tumour suppressor gene, p53 and
the DNA repair gene, RAD51.

Which combination of gene activity is most likely to result in breast cancer?

Key: gene produces normal protein


Ans: D

X gene produces abnormal protein or no protein

gene
BRCA1 or
p53 RAD51
BRCA2
A.
B. X
C. X
D. X X X

9648/01/PU3 Prelim 2/2013


751
11

18. Human immunodeficiency virus (HIV) is a retrovirus. After infecting a host cell, viral
DNA is produced which is incorporated into the DNA of the host cell. The modified host
genome now codes for the production of new HIV particles.

Which could be used as a potential treatment to slow down the spread of HIV?

1 inhibitors of restriction endonucleases


Ans: D
2 inhibitors of reverse transcriptase
3 restriction endonucleases
4 reverse transcriptase

A. 1 and 4 only

B. 2 and 3 only

C. 1 only

D. 2 only

19. One hypothesis about the origin of viruses is that they existed before cellular life forms,
later evolving into parasites of cellular organisms. Groups with an ancient, common
origin tend to share conserved sequences of DNA or RNA.

Which observation about virus genomes supports the view that viruses existed before
cellular life forms and only much later evolved into parasites?
Ans: D
A. All viruses have genes that code for capsid components.

B. Introns of eukaryotic genes have common features with viral genomes.

C. Large virus genomes have genes that originate in host cells.

D. Viral genomes have conserved genes not found in any other genomes

9648/01/PU3 Prelim 2/2013


[Turn over
752
12

20. The diagram shows an investigation into bacterial genetics

To grow mutant bacteria X To grow mutant bacteria Y


need methionine (met-) do not need methionine
and biotin (bio-) (met+) and biotin (bio+)
do not need threonine need threonine (thr-) and
(thr+) and leucine (leu+) leucine (leu-)


Grow together in the presence of methionine,
Ans: D

biotin, threonine and leucine


Grow together without methionine,
biotin, threonine and leucine


Growth of some bacteria that
are met+, bio+, thr+, leu+

Which process or processes could explain these results?

1 transformation
2 transduction
3 conjugation

A. 1 only

B. 3 only

C. 1 and 2

D. 1 and 3

9648/01/PU3 Prelim 2/2013


753
13

21. What is necessary for an organism to respond to changes in its internal environment?

A. a communication system between receptors and effectors Ans A

B. a communication system involving nerve cells

C. negative feedback from receptor to effector

D. positive feedback from effector to receptor

22. A snake venom causes death by leading to paralysis of muscles. It exerts its effect at
synapses.

The statements below were put forward by scientists as possible explanations for the
effects of this venom.

1 It interferes with the binding of neurotransmitter vesicles to the membranes.


2 It binds with neurotransmitter receptor sites.
3 It blocks calcium and sodium channels.
Ans: A
4 It destroys the myelin sheath of the neurone.
5 It binds with neurotransmitter.

Which statements should be investigated further?

A. 1, 2, 3 and 5 only

B. 2, 4 and 5 only

C. 1 and 4 only

D. 1, 2, 3, 4 and 5

23. Vibrio cholerae produces a toxin that binds to a cell surface membrane receptor on
intestinal cells of the host. The toxin permanently activates the G protein in target cells,
causing them to lose water rapidly. When a person is infected with cholera they suffer
severe dehydration. Ans: B

What statement best describes the V. cholerae toxin

A. It is an example of a second messenger molecule.

B. It disrupts normal signal transduction in the cell.

C. It is a lipid-soluble molecule.

D. It acts as a neurotransmitter.

9648/01/PU3 Prelim 2/2013


[Turn over
754
14

24. The graphs show the response of sensory neurones to increasing pressure on the
Pacinian corpuscles (receptors found in deep layers of the skin that senses vibratory
pressure and touch.) in mammalian skin.

Ans: B

Which conclusion about increasing pressure is correct?

A. As pressure increases the depolarisation of the corpuscle and sensory neurone


increases.

B. As pressure increases the axon terminal depolarisation reaches a value that


depolarises the sensory neurone.

C. If the corpuscle is stimulated for long enough the depolarisation of the sensory
neurone increases.

D. If the corpuscle is stimulated for long enough the axon terminal depolarisation
reaches a value that depolarises the sensory neurone.

9648/01/PU3 Prelim 2/2013


755
15

25. The following graph shows the relationship between light intensity and net oxygen
uptake or output by a particular green plant.

Ans: C

Which of the following statement is true?

A. At a light intensity of 10 units, the rate of photosynthesis is zero.

B. At a light intensity of 10 units, the rate of aerobic respiration is zero.

C. At a light intensity of 10 units, oxygen produced by photosynthesis is equal to the


oxygen used by aerobic respiration.

D. At a light intensity of 10 units, oxygen produced by photosynthesis is equal to


twice the oxygen used by aerobic respiration.

26. What causes phenotypic variation among organisms of the same genotype?

A. continuous variation within the species


Ans C
B. different varieties of the same species

C. living in different environments

D. mutation

9648/01/PU3 Prelim 2/2013


[Turn over
756
16

27. One of the many recessive mutations of the CFTR gene changes one amino acid in
the region of the CFTR protein that binds ATP. The graph shows the effect of different
concentrations of ATP on normal and mutant CFTR proteins.

Which correctly describes individuals who are homozygous for this mutation?

1 Their CFTR protein cannot bind ATP and cannot act as an ion channel.
2 Their CFTR protein binds ATP less readily than normal CFTR protein.
3 They produce CFTR protein that must bind ATP to function as an ion
channel.
4 They produce a mixture of normal and mutant CFTR protein, both of which
can act as an ion channel.

A. 1 only
Ans: C
B. 2 only

C. 2 and 3 only

D. 2 and 4 only

9648/01/PU3 Prelim 2/2013


757
17

28. Many plants are not fertilised by pollen from their own flowers. This is known as self-
incompatibility. In any individual species, a single gene, the S gene, is responsible and
it may have many different alleles.

If a pollen grain has an S allele which matches an allele in the genotype of the stigma
then the pollen grain fails to germinate or the pollen tube fails to grow through the style.

The genotype of the stigma of a flower is S3S4.

Which pollen grains would germinate?

A. 4 only
Ans: C
B. 2 and 4 only

C. 1 only

D. 3 and 4 only

29. Two areas of molecular biology that have received considerable attention in
evolutionary studies are the genetic code and cytochrome C. Cytochrome C is an
essential component of all respiratory electron transport chains.

Which statements lend evidence to the ideas that

all living organisms are related


there is a single, rather than a multiple, origin of life?

1 The almost universal nature of the genetic code is a result of evolutionary


convergence from multiple lineages.
2 The sequence of amino acids in cytochrome C is similar in organisms that
are from similar environments or with similar metabolic demands.
3 The majority of organisms have the same, or similar, amino acid
sequences for cytochrome C.
4 When transferred into a very dissimilar organism, a gene coding for
cytochrome C will lead to the expression of a protein that will function in the
other organism.

A. 2 and 3 only Ans: C

B. 1 and 2 only

C. 3 and 4 only

D. 1, 3 and 4 only

9648/01/PU3 Prelim 2/2013


[Turn over
758
18

30. The following statements relate to molecular phylogenetics.

1 Lines of descent from a common ancestor to present-day organisms have


undergone similar, fixed rates of DNA mutation.

2 Organisms with similar base sequences in their DNA are closely related to
each other.

3 The number of differences in the base sequences of DNA of different


organisms can be used to construct evolutionary trees.

4 The proportional rate of fixation of mutations in one gene relative to the


rate of fixation of mutations in other genes stays the same in any given line
of descent.

Which statements, when taken together, suggest the existence of a molecular clock
that enables scientists to estimate the time at which one species might have diverged
from another?

A. 1 and 2
Ans: B
B. 1 and 4

C. 2 and 3

D. 3 and 4

31. Based on binomial nomenclature, which two species are most closely related?

1 Common barberry (Berberis vulgaris) Ans A


2 Canadian bunchberry (Cornus canadensis)
3 Smooth blackberry (Rubus canadensis)
4 Canadian barberry (Berberis canadensis)

A. 1 and 4

B. 1 and 3

C. 2 and 4

D. 3 and 4

9648/01/PU3 Prelim 2/2013


759
19

32. The graphs represent the frequency of alleles in species X, Y and Z during and after
selection.

Ans: B

In which species does evolution take place?

A. X only

B. Y only

C. Y and Z

D. none of X, Y nor Z

33. Why can restriction fragment length polymorphism (RFLP) be used to detect inherited
diseases?

A. Different alleles vary in the sequence and number of base pairs.


Ans: A

B. The DNA fragments are the same length within each gene.

C. The restriction site is unaffected by the number of base pairs.

D. Alleles of different genes have different molecular masses.

9648/01/PU3 Prelim 2/2013


[Turn over
760
20

34. Ripening of fruit, for example figs, is caused by ethylene gas, a lipid-soluble compound
that diffuses easily from cell to cell as well as to neighbouring plants. Plant tissues
produce ethylene at certain developmental stages and in response to stressful
conditions, such as heat or injury.

Plant cells have special ethylene-binding receptors. When ethylene is detected, a


series of reactions follows inside the cell. This results in the production of pectinases to
break down cell walls and amylases to convert carbohydrates into simple sugars.
Ans: B
Which of the following statements is consistent with the information above?

A. Ethylene stimulates cell elongation which enables fruit to grow.

B. The scratching of the surfaces of harvested figs speeds up ripening.

C. Changes triggered by ethylene would discourage animals from consuming fruit.

D. Ethylene receptors are located on the external surfaces of the cell surface
membrane.

35. Stem cells are widely used in medical research.

Which property of stem cells makes them particularly useful in this research?

A. They can be fused together to form a zygote. Ans C

B. They can divide and eventually give rise to a whole organism.

C. They can divide and be made to differentiate into various types of cell.

D. They will continue to divide indefinitely.

36. Which of these processes could increase crop yield?

P inserting genes for vitamin A production into rice Ans B


Q inserting genes for pest resistance into cotton
R inserting genes for herbicide resistance into oilseed rape
S inserting genes for nitrogen fixation into soya beans

A. P and Q

B. Q and R

C. R and S

D. S and P

9648/01/PU3 Prelim 2/2013


761
21

37. Which genetic modification could increase the yield of a crop measured as mass of
crop produced per unit area per year?
Ans: A
A. winter resistant plants

B. delayed ripening in the fruits

C. more essential amino acids in the seeds

D. more vitamin A in the grain

38. The following figures show two different methods used to sequence DNA.

Ans: C
What of the following statements is correct?

A. Method 1 requires 4 slots for the loading of the DNA.

B. Method 2 uses a laser beam to separate the individual nucleotides.

C. Method 1 uses fluorescent dyes to distinguish between the different nucleotides.

D. Method 2 requires the DNA to be loaded at the positive end on the sequencing
gel.
9648/01/PU3 Prelim 2/2013
[Turn over
762
22

39. Gene therapy is a way of treating genetic disease by introducing a piece of DNA into
the cells of an affected individual. Liposomes can be used for gene therapy as they
target the cells affected by a genetic disease.

What feature of a cell surface membrane allows the liposome to target cells affected by
a genetic disease?
Ans C
A. carrier molecules

B. phosphate groups

C. receptor molecules

D. protein channels

40. Small samples from crime scenes can be genetically profiled (DNA finger printed).

Which are necessary parts of a successful genetic profiling process?


Ans: D

Key: present
X absent
crime scene ethidium bromide
PCR
sample and X-rays

A. red blood cells X


B. saliva X
C. semen X
D. skin cells X

9648/01/PU3 Prelim 2/2013


763
Class Adm
A No

Candid
date Name
e:

2013 Prelimin
P nary Examinaation 2
Pre-U
University 3
H2 Biology 9648/02
2

17 Septe
ember 2013
3

2 hours
s
Additionall Materials: Writing paper

READ TH
HESE INSTR
RUCTIONS
S FIRST

Do not op ooklet until you are to


pen this bo old to do so
o.

Write yourr Index num


mber and na ame on all th he work you u hand in.
Write in da
ark blue or black pen on
o both side es of the paaper.
You may u use a soft pencil
p for an
ny diagramss, graphs orr rough work king.
Do not usee staples, paper
p clips, highlighterss, glue or co
orrection flu
uid.

Section A
Answer alll questionss.
Fo
or Examiner's Use
Section B
ny one question.
Answer an Secction A

At the end
d of the examination, faasten all you
ur work seccurely together. 1
The numb ber of marks is given n in brackeets [ ] at th
he end of each
e
question oor part queestion. At th
he end of t he examina ation, fasten all 2
your workk securely to
ogether.
3

Secction B

T
Total

This ques
stion pape r consists of 24 printed pages.

[Tu
urn over
764
2 For
Examiners
Use
Section A

Answer all questions in this section.

1. A long-term breeding experiment to investigate the genetic basis of tame (friendly)


behaviour was carried out in a population of silver foxes. The foxes were bred each
year and the resulting young foxes assessed each month between the ages of 1 and
8 months to see how tame they were.

Table 1.1 shows how the foxes were put into categories according to their tameness.

Table 1.1

tameness description of behaviour


class towards humans
Not tame - these foxes run
3 away from humans or bite
when handled.
Neutral - these foxes allow
handling by humans but show
2
no emotionally friendly
response.
Tame - these foxes are friendly
1 to humans. They wag their
tails and whine for attention.
Very tame - these foxes are
eager for human elite contact.
elite They whimper to attract
attention and sniff and lick
humans.

The tamest 5% of the male foxes and the tamest 20% of the female foxes in each
generation were used for breeding to produce the next generation. This was
repeated for over forty generations.

(a)

(i) State the name given to the process in which only a certain percentage of
adult foxes were chosen by humans to breed in each generation.

...... [1]

(ii) Suggest why 20% of the female foxes were used for breeding but only 5%
of the male foxes.

......

......

......

......

......

...... [2]

9648/02/PU3 Prelim 2/2013


765 For
3 Examiners
Use

(b) Table 1.2 shows the number of foxes in the elite tameness class during the
long-term experiment.

Table. 1.2

Number of generations Foxes in elite class (%)


10 18
20 35
35 75

Discuss what the results shown in Table 1.2 suggest about the causes of
variation in tameness behaviour in silver foxes.

.........

.........

.........

.........

.........

........ [3]

9648/02/PU3 Prelim 2/2013


[Turn over
766
4 For
Examiners
Use
(c) As tameness increased in the silver fox population over the years, it was
noticed that other phenotypic traits also became more common.

Table 1.3 compares the frequency of these traits in a control group of silver
foxes that had not been used in this long-term breeding experiment and in the
tame population of foxes.

Table. 1.3

Phenotypic trait Animals showing trait Percentage


(per 100 000) increase in trait
Control Tame
Population Population
White patch of 710 12 400 1 646
fur on head
Floppy ears 170 230 35
Short tail 2 140 6 900
Curly tail 830 9 400 1 033

Students were asked to suggest a variety of genetic hypotheses to explain


why these traits become more common in tame foxes. Their suggestions
were:

Linkage Epistasis Inbreeding Genetic Drift

Select one hypothesis from the list and explain how it could account for the
data in Table 1.3.

.........

.........

.........

.........

.........

........ [2]

9648/02/PU3 Prelim 2/2013


767 For
5 Examiners
Use

(d) Similar changes in tameness, colour and body shape are believed to have
occurred in the 11 000 year period during which the grey wolf species, Canis
lupus, evolved into the domesticated dog species, Canis familiaris.

Suggest how different types of isolating mechanism allowed dogs to evolve


separately to wolves.

.........

.........

.........

.........

.........

.........

........ [3]

(e) Interbreeding between members of the wolf species and some dogs has been
reported.

However, there are some large breeds of dogs that cannot breed successfully
with small dog breeds.

Use this information and your own knowledge to explain the problems of
classifying wolves and different dog breeds according to:

the biological species concept

and

the phylogenetic species concept.

.........

.........

.........

.........

.........

.........

.........

.........

........ [4]

[Total: 15]

9648/02/PU3 Prelim 2/2013


[Turn over
768
6 For
Examiners
Use
2.
(a) The brain depends on a constant supply of oxygen for aerobic respiration.
Anaerobic respiration is not sufficient to keep neurones in the brain alive. This
is because neurones require especially large amounts of ATP. Up to 80% of
the ATP is used to provide energy for the Na+ / K+ pump.

When a person suffers a stroke, blood flow to part of the brain is stopped, so
some neurones receive no oxygen. ATP production by oxidative
phosphorylation stops. Fig. 2.1 shows some of the ways in which the lack of
ATP affects a neurone in the brain.

No ATP

Na+ / K+ pump stops working

membrane depolarises

voltage-gated Ca2+ channels open

Ca2+ ions flood into the neurone

2+
Ca ions activate enzymes that eventually
destroy the neurone

Fig. 2.1

(i) Explain why the membrane of the neurone depolarises when the Na+ / K+
pump stops working.

......

......

......

......

......

......

......

...... [4]

9648/02/PU3 Prelim 2/2013


769 For
7 Examiners
Use

(ii) Suggest why calcium ions flood into the neurone when the Na+ / K+ pump
stops working.

......

......

......

...... [2]

(b) The freshwater turtle, Trachemys scripta, is able to survive for long periods in
conditions of very low oxygen concentration. As in humans, the rate of activity
of the Na+ / K+ pump in the neurones in its brain falls sharply. However, in
turtles this does not result in damage to these cells.

A better understanding of how the neurones in the turtles brain survive in


these conditions could lead to new treatments for people who have suffered a
stroke.

Experiments show that, in turtle brain neurones, in conditions of low oxygen


availability:

most ion channels in the cell surface membranes immediately close

after about four hours, the quantity of mRNA involved in the synthesis
of proteins used to build ion channels, falls to less than one fifth of
normal concentrations.

(i) Suggest how the closure of ion channels in the neurones of the turtle in
very low oxygen concentrations could allow the cells to survive.

......

......

......

...... [2]

(ii) Suggest what causes the quantity of mRNA for protein channels to fall.

......

......

......

...... [2]

9648/02/PU3 Prelim 2/2013


[Turn over
770
8 For
Examiners
Use
(c) Some synapses in the brain use the neurotransmitter dopamine. After the
postsynaptic membrane has been depolarised, dopamine leaves the receptor
proteins and moves back into the presynaptic neurone through specific
transporter proteins.

Schizophrenia is a condition in which there is a higher than usual


concentration of dopamine in certain areas of the brain. The drug
phenothiazine has a similar shape to dopamine and is used to treat
schizophrenia.

Suggest and explain what occurs at the synapse when phenothiazine is used
in the treatment of schizophrenia.

.........

.........

.........

........ [2]

(d) DRD4 is a dopamine receptor in humans. The gene coding for the DRD4
receptor has a large number of alleles, of which an individual can have only
two.

Three alleles of the DRD4 receptor gene have the following mutations:

a single base substitution


a 21 base-pair deletion
a 13 base-pair deletion.

The 13 base-pair deletion has the most serious consequences for the
structure of the DRD4 receptor protein. Suggest why this is so.

.........

.........

.........

.........

.........

........ [3]

9648/02/PU3 Prelim 2/2013


771 For
9 Examiners
Use

(e) One allele of the DRD4 gene has been found more frequently amongst
individuals whose personality is described as impulsive and exploratory.

Describe the mechanism whereby an allele such as this could have become
common in the human population.

.........

.........

.........

.........

.........

........ [3]

[Total: 18]

9648/02/PU3 Prelim 2/2013


[Turn over
772
10 For
Examiners
Use
3. Bone marrow contains stem cells that divide by mitosis to form blood cells. Each time
a stem cell divides it forms a replacement stem cell and a cell that develops into a
blood cell.

Fig. 3.1 shows changes in the mass of DNA in a human stem cell from the bone
marrow during three cell cycles.

Fig. 3.1

(a) With reference to Fig. 3.1, state:

(i) what happens to bring about the changes in the mass of DNA per cell at K
and at L.

K ......

......

L ...

...... [2]

(ii) how many blood cells are formed from the stem cell in the time shown.

...... [1]

(iii) what happens to the number of chromosomes in the stem cell.

...... [1]

9648/02/PU3 Prelim 2/2013


773 For
11 Examiners
Use

Stem cells in bone marrow give rise to phagocytes, B-lymphocytes and T-


lymphocytes.

(b) During an immune response, cells divide by mitosis.

Suggest how mitosis is involved in an immune response.

.........

.........

.........

.........

.........

........ [3]

A protein, mitosis-promoting factor (MPF), has been identified in cells. MPF is a


globular protein made from two polypeptide chains.

The presence of MPF is known to cause prophase to start.

(c) Describe the changes that occur during prophase in an animal cell.

.........

.........

.........

.........

.........

.........

.........

........ [4]

9648/02/PU3 Prelim 2/2013


[Turn over
774
12 For
Examiners
Use
(d) MPF normally begins to break down and stops functioning during anaphase.

Suggest the possible consequences of MPF not breaking down.

.........

.........

.........

.........

.........

........ [3]

[Total: 14]

9648/02/PU3 Prelim 2/2013


775 For
13 Examiners
Use

BLANK PAGE

9648/02/PU3 Prelim 2/2013


[Turn over
776
14 For
Examiners
Use
4. A type of pheasant occurs in a range of colours, especially when bred in captivity. It
may, for example, have green or purple plumage.

Sometimes when a green male is crossed with a green female all the offspring, male
and female, are green. However, sometimes a green male crossed with a green
female results in offspring in which the majority of the offspring are green, but in
which some of the females are purple, as shown in Table 4.1.

Table. 4.1

Phenotype Number of
offspring
green male 7
green female 3
purple female 4

(a)
(i) Using the results shown in Table 4.1, a geneticist proposed a hypothesis
which predicted a 3:1 ratio of green:purple phenotypes. She used the 2
test to test her hypothesis

Complete Table 4.2 to calculate 2 .

Table. 4.2

Green Plumage Purple Plumage

Observed (O) 7 + 3 = 10 4

Expected (E) 10.5 3.5

OE

(O E)2

(O E)2
E

(O E)2
= 2
E

[2]

9648/02/PU3 Prelim 2/2013


777 For
15 Examiners
Use

(ii) Draw a conclusion from your calculation in Table 4.2, given that the critical
value for 2 at p = 0.05 with one degree of freedom is 3.84. Explain why
this conclusion should be treated with caution.

......

......

......

......

......

...... [3]

(b) Plumage colour in pheasants is sex-linked.

In birds, the sex chromosomes are referred to as W and Z, rather than Y and
X as in mammals. The W chromosome has no genes that affect plumage
colour. The heterogametic sex is the female, not the male. Thus the male has
two Z chromosomes (ZZ) and the female has one W and one Z chromosome
(WZ).

(i) Use a genetic diagram to explain the results in Table 4.1.

[3]

9648/02/PU3 Prelim 2/2013


[Turn over
778
16 For
Examiners
Use
(ii) Using the same symbols as in (b)(i), indicate the genotypes and
phenotypes of the parents which could give rise to purple male offspring.

......

...... [2]

[Total: 10]

9648/02/PU3 Prelim 2/2013


779 For
17 Examiners
Use

BLANK PAGE

9648/02/PU3 Prelim 2/2013


[Turn over
780
18 For
Examiners
Use
5.
(a) An experiment was carried out to investigate the effect of temperature on the
rate of oxygen consumption of the lizard, Sauromalus hispidus. The body
temperature of a lizard varies with environmental temperature.

Several lizards were fitted with small, airtight masks that covered their heads.
Air was supplied inside the mask through one tube, and collected through
another. The differences between oxygen concentrations in the air supplied
for inhalation and the exhaled air enabled the researchers to measure the
rate of oxygen consumption of the lizards.

The rate of oxygen consumption of each lizard was measured when it was at
rest and when it was running. Measurements were made at different
temperatures ranging from 15 C to 40 C.

Fig. 5.1 shows the results.

Fig. 5.1

9648/02/PU3 Prelim 2/2013


781 For
19 Examiners
Use

(i) Running requires rapid use of ATP by muscle cells in the legs and heart of
a lizard.

With reference to the events occurring inside a mitochondrion, explain


why a faster use of ATP requires a greater rate of oxygen consumption.

......

......

......

......

......

......

......

...... [4]

(ii) Explain the effect of temperature on the rate of oxygen consumption in


Sauromalus when at rest.

......

......

......

......

......

...... [3]

9648/02/PU3 Prelim 2/2013


[Turn over
782
20 For
Examiners
Use
(b) In the light-independent stage of photosynthesis, the enzyme rubisco
catalyses the combination of carbon dioxide with ribulose bisphosphate,
RuBP. When the carbon dioxide concentration within the leaf is very low,
rubisco tends to combine oxygen, rather than carbon dioxide, with RuBP. This
process is called photorespiration. It reduces carbon dioxide assimilation and
therefore reduces crop yields.

Photorespiration is most likely to happen in hot, dry conditions.

Suggest why photorespiration is most likely to take place in hot, dry


conditions

.........

.........

.........

.........

.........

.........

.........

........ [3]

(c) It is expected that the carbon dioxide concentration in the atmosphere will
increase in the future, which would be expected to increase rates of
photosynthesis in many crop plants.

Investigations were carried out into the effect of increased carbon dioxide
concentration on the rate of photosynthesis in maize.

Maize plants were grown in open-air trials, in the same field and were
exposed to the same changes in the weather.
50% of the plants were exposed to a normal carbon dioxide
concentration.
50% of the plants were exposed to an increased carbon dioxide
concentration.
The rate of photosynthesis was measured as the net assimilation rate
of carbon dioxide.
Measurements were made at three-hourly intervals between 0700
hours and 1900 hours on three different days.

9648/02/PU3 Prelim 2/2013


783 For
21 Examiners
Use

The results are shown in Fig. 5.2.

Fig. 5.2

(i) Suggest an explanation for the lack of effect of carbon dioxide


concentration on the rate of photosynthesis in maize plants, shown by
these results.

......

......

......

...... [2]

(ii) Suggest one explanation for the changes in the rate of photosynthesis
between 0700 hours and 1900 hours on day 1.

......

......

......

...... [2]

[Total: 14]

9648/02/PU3 Prelim 2/2013


[Turn over
784
22 For
Examiners
Use
6. A study finding published in the journal PLoS Genetics in 2013 was the first to show
exercise alters DNA methylation related to fat storage.

The finding adds to a growing body of knowledge about epigenetics that show though
we cannot change our DNA we can change the way our genes are expressed to fight
disease.

(a) Describe and explain the effect of DNA methylation in eukaryotic genome.

.........

.........

.........

.........

.........

........ [3]

(b) Suggest how DNA methylation enables cells to differentiate.

.........

........ [1]

In the 1950s, Erwin Chargaff determined the relative quantities of the four bases in
DNA in different organisms. His results provided important evidence for the model of
DNA proposed by James Watson and Francis Crick in 1953. Some of Chargaffs data
is shown in Table 6.1.

Table. 6.1

9648/02/PU3 Prelim 2/2013


785 For
23 Examiners
Use

(c) Explain how the data in Table 6.1 helps to confirm the arrangement of bases
in DNA.

.........

.........

.........

.........

.........

........ [3]

(d) Table 6.2 shows Chargaffs data for a virus.

Table. 6.2

(i) State how the result for the virus differs from the results for all the
organisms given in Table 6.1.

......

......

......

...... [1]

(ii) Suggest why the results for the virus are different from all the other
organisms.

......

......

......

...... [1]

[Total: 9]

9648/02/PU3 Prelim 2/2013


[Turn over
786
24 For
Examiners
Use
Section B

Answer one question.

Write your answers on the separate answer paper provided.


Your answers should be illustrated by large, clearly labelled diagrams, where appropriate.
Your answers must be in continuous prose, where appropriate.
Your answers must be set out in sections (a), (b) etc., as indicated in the question.

7.
(a) Outline the molecular structure of phospholipids in relation to their function in cell
membranes. [7]

(b) Explain the role of the Golgi body and its link to the rough endoplasmic reticulum.
[7]

(c) Describe the structure of ribosomes. [6]

[Total: 20]

8.
(a) Explain the effects of competitive and non-competitive inhibitors on the rate of
enzymatic activity. [6]

(b) Explain the mode of action of enzymes in terms of specificity and activation
energy. [10]

(c) Describe the effect of temperature on enzyme-catalyzed reactions. [4]

[Total: 20]

9648/02/PU3 Prelim 2/2013


787
Class Adm
A No

Candid
date Name
e:

2013 Prelimin
P nary Examinaation 2
Pre-U
University 3
H2 Biology 9648/02
2

17 Septe
ember 2013
3

2 hours
s
Additionall Materials: Writing paper

READ TH
HESE INSTR
RUCTIONS
S FIRST

Do not op ooklet until you are to


pen this bo old to do so
o.

Write yourr Index num


mber and na ame on all th he work you u hand in.
Write in da
ark blue or black pen on
o both side es of the paaper.
You may u use a soft pencil
p for an
ny diagramss, graphs orr rough work king.
Do not usee staples, paper
p clips, highlighterss, glue or co
orrection flu
uid.

Section A
Answer alll questionss.
Fo
or Examiner's Use
Section B
ny one question.
Answer an Secction A

At the end
d of the examination, faasten all you
ur work seccurely together. 1
The numb ber of marks is given n in brackeets [ ] at th
he end of each
e
question oor part queestion. At th
he end of t he examina ation, fasten all 2
your workk securely to
ogether.
3

Secction B

T
Total

This ques
stion pape r consists of 24 printed pages.

[Tu
urn over
788
2 For
Examiners
Use
Section A

Answer all questions in this section.

1. A long-term breeding experiment to investigate the genetic basis of tame (friendly)


behaviour was carried out in a population of silver foxes. The foxes were bred each
year and the resulting young foxes assessed each month between the ages of 1 and
8 months to see how tame they were.

Table 1.1 shows how the foxes were put into categories according to their tameness.

Table 1.1

tameness description of behaviour


class towards humans
Not tame - these foxes run
3 away from humans or bite
when handled.
Neutral - these foxes allow
handling by humans but show
2
no emotionally friendly
response.
Tame - these foxes are friendly
1 to humans. They wag their
tails and whine for attention.
Very tame - these foxes are
eager for human elite contact.
elite They whimper to attract
attention and sniff and lick
humans.

The tamest 5% of the male foxes and the tamest 20% of the female foxes in each
generation were used for breeding to produce the next generation. This was
repeated for over forty generations.

(a)

(i) State the name given to the process in which only a certain percentage of
adult foxes were chosen by humans to breed in each generation.
artificial selection / selective breeding ;
...... [1]

(ii) Suggest why 20% of the female foxes were used for breeding but only 5%
of the male foxes.

......

idea that females produce only a few offspring ;


......
(so) more females (than males) needed to maintain numbers (each
generation) ;
......
(20% females chosen as) inbreeding / genetic problems, if breeding
population is too small;
......
(5% males chosen as) selection pressure stronger if fewer (tamest) are used;

......

...... [2]

9648/02/PU3 Prelim 2/2013


789 For
3 Examiners
Use

(b) Table 1.2 shows the number of foxes in the elite tameness class during the
long-term experiment.

Table. 1.2

Number of generations Foxes in elite class (%)


10 18
20 35
35 75

Discuss what the results shown in Table 1.2 suggest about the causes of
variation in tameness behaviour in silver foxes.

.........
1 (mostly) genetic ;
2 as can be selected for / selective breeding increases frequency ;
3 allele(s) for tameness ;
.........
4 (from) mutation ;
5 query role of environment / learning ;
.........
6 ref. DRD4 / dopamine receptor ;
.........
R environment also given as cause
R genetic drift
.........

........ [3]

9648/02/PU3 Prelim 2/2013


[Turn over
790
4 For
Examiners
Use
(c) As tameness increased in the silver fox population over the years, it was
noticed that other phenotypic traits also became more common.

Table 1.3 compares the frequency of these traits in a control group of silver
foxes that had not been used in this long-term breeding experiment and in the
tame population of foxes.

Table. 1.3

Phenotypic trait Animals showing trait Percentage


(per 100 000) increase in trait
Control Tame
Population Population
White patch of 710 12 400 1 646
fur on head
Floppy ears 170 230 35
Short tail 2 140 6 900
Curly tail 830 9 400 1 033

Students were asked to suggest a variety of genetic hypotheses to explain


why these traits become more common in tame foxes. Their suggestions
were:

Linkage Epistasis Inbreeding Genetic Drift

Select one hypothesis from the list and explain how it could account for the
data in Table 1.3.

.........
linkage
tameness.........
genes and genes for these traits on same chromosome ;
(so) inherited together ;
.........
epistasis
(product.........
of) one gene affects expression of another ;
via enzyme pathway ;
.........
inbreeding
(hidden ........
/ masked) recessive alleles ; [2]
selected for, as well / unintentionally ;
more chance homozygous as, small gene pool / parents related ;

ACCEPT idea of (recessive )allele inherited from both parents because, they are closely-
related / small gene pool / reduced genetic diversity

genetic drift
random / chance (which alleles, present / passed on) ;
(effect stronger because) small breeding population ;

9648/02/PU3 Prelim 2/2013


791 For
5 Examiners
Use

(d) Similar changes in tameness, colour and body shape are believed to have
occurred in the 11 000 year period during which the grey wolf species, Canis
lupus, evolved into the domesticated dog species, Canis familiaris.

Suggest how different types of isolating mechanism allowed dogs to evolve


separately to wolves.
1 geographic ;
2 wolves avoid human settlements / dogs confined by humans ;
.........

3 behavioural ;
.........
4 detail / description ; eg differences in, pheromones / courtship
.........
5 mechanical ;
6 idea of different size of wolves and some small dogs ; ACCEPT different
.........
genitalia

.........
7 gamete incompatibility ;
8 possibility of different chromosome numbers ;
.........
9 seasonal / temporal ;
10 different breeding, seasons / times ; idea that dogs breed all year round /
........ [3]
wolves breed once a year

(e) Interbreeding between members of the wolf species and some dogs has been
reported.

However, there are some large breeds of dogs that cannot breed successfully
with small dog breeds.

Use this information and your own knowledge to explain the problems of
classifying wolves and different dog breeds according to:

the biological species concept

and

the phylogenetic species concept.

.........
biological species concept

1 (members of same species) need can interbreed to produce fertile offspring ;


.........
2 not all dog breeds can do this therefore not same species ;
3 dog and wolf can so they should be same species ;
.........
phylogenetic species concept
.........
4 idea that dogs and wolves monophyletic group / tip of phylogeny ;
.........
5 genetic differences, between dogs and wolves small ;
6 gene flow between wolves big dogs little dogs / analogous to ring species ;
.........
7 (PSC) one species (with a lot of phenotypic variation) ;

.........

.........

........ [4]

[Total: 15]

9648/02/PU3 Prelim 2/2013


[Turn over
792
6 For
Examiners
Use
2.
(a) The brain depends on a constant supply of oxygen for aerobic respiration.
Anaerobic respiration is not sufficient to keep neurones in the brain alive. This
is because neurones require especially large amounts of ATP. Up to 80% of
the ATP is used to provide energy for the Na+ / K+ pump.

When a person suffers a stroke, blood flow to part of the brain is stopped, so
some neurones receive no oxygen. ATP production by oxidative
phosphorylation stops. Fig. 2.1 shows some of the ways in which the lack of
ATP affects a neurone in the brain.

No ATP

Na+ / K+ pump stops working

membrane depolarises

voltage-gated Ca2+ channels open

Ca2+ ions flood into the neurone

2+
Ca ions activate enzymes that eventually
destroy the neurone

Fig. 2.1

(i) Explain why the membrane of the neurone depolarises when the Na+ / K+
pump stops working.

......
1. (when pump stops working), resting potential not maintained
or
......
pump usually maintains the resting potential;

......
2. (during resting potential) membrane polarised
or
positive charge outside (neurone) / negative charge inside (neurone) / -70mV
......
inside neurone relative to outside / potential difference across membrane;
......
3. (when pump stops working), ions (only) move by diffusion;
4. Na+ into the neurone;
......
5. outward diffusion of K+ is limited / K+ stay in neurone;
6. ref. non voltage-gated channels;
......
7. (eventually) inside of the neurone, becomes less negative / contains
......
(relatively) more positive ions [4]
or
there is a reduced potential difference across the membrane;

9648/02/PU3 Prelim 2/2013


793 For
7 Examiners
Use

(ii) Suggest why calcium ions flood into the neurone when the Na+ / K+ pump
stops working.

......
1. voltage gated (calcium) channels open;

......
2. (calcium ions move in) by diffusion / move down their concentration
gradient;
......

...... [2]

(b) The freshwater turtle, Trachemys scripta, is able to survive for long periods in
conditions of very low oxygen concentration. As in humans, the rate of activity
of the Na+ / K+ pump in the neurones in its brain falls sharply. However, in
turtles this does not result in damage to these cells.

A better understanding of how the neurones in the turtles brain survive in


these conditions could lead to new treatments for people who have suffered a
stroke.

Experiments show that, in turtle brain neurones, in conditions of low oxygen


availability:

most ion channels in the cell surface membranes immediately close

after about four hours, the quantity of mRNA involved in the synthesis
of proteins used to build ion channels, falls to less than one fifth of
normal concentrations.

(i) Suggest how the closure of ion channels in the neurones of the turtle in
very low oxygen concentrations could allow the cells to survive.

......
1. Na+ / K+, cannot move through membrane;
2. so potential across membrane maintained even when pump stops / so membrane depolarisation does not
happen; ......
3. calcium ions cannot enter cell;
......
4. so, (destructive) enzymes not activated;

...... [2]

(ii) Suggest what causes the quantity of mRNA for protein channels to fall.

......
1. gene (for protein channels), expressed less / switched off;
2. transcription, reduced / stopped;
......
3. AVP; e.g. reduced aerobic respiration / less ATP, for transcription

......

...... [2]

9648/02/PU3 Prelim 2/2013


[Turn over
794
8 For
Examiners
Use
(c) Some synapses in the brain use the neurotransmitter dopamine. After the
postsynaptic membrane has been depolarised, dopamine leaves the receptor
proteins and moves back into the presynaptic neurone through specific
transporter proteins.

Schizophrenia is a condition in which there is a higher than usual


concentration of dopamine in certain areas of the brain. The drug
phenothiazine has a similar shape to dopamine and is used to treat
schizophrenia.

Suggest and explain what occurs at the synapse when phenothiazine is used
in the treatment of schizophrenia.

binds to/blocks, dopamine receptors (on postsynaptic membrane);


.........
prevents depolarization (of postsynaptic membrane);
reduces effect of dopamine;
.........
R reduces amount of dopamine
.........

........ [2]

(d) DRD4 is a dopamine receptor in humans. The gene coding for the DRD4
receptor has a large number of alleles, of which an individual can have only
two.

Three alleles of the DRD4 receptor gene have the following mutations:

a single base substitution


a 21 base-pair deletion
a 13 base-pair deletion.

The 13 base-pair deletion has the most serious consequences for the
structure of the DRD4 receptor protein. Suggest why this is so.

.........
ref 13 base deletion
frame shift/alters reading frame (after mutation);
(so) all amino acids different after mutation;
.........
3-D shape/tertiary structure, of protein changed;
(whereas) 21 base-pair deletion, loses 7 amino acids/no frame shift;
.........
(whereas) substitution, may change only one amino acid/may be silent;
.........

.........

........ [3]

9648/02/PU3 Prelim 2/2013


795 For
9 Examiners
Use

(e) One allele of the DRD4 gene has been found more frequently amongst
individuals whose personality is described as impulsive and exploratory.

Describe the mechanism whereby an allele such as this could have become
common in the human population.

.........
increased chances of, survival/breeding/mating;
.........
provides a selective advantage;
allele passed on (to next generation);
.........
allele increases in frequency over time;
natural selection;
.........

.........

........ [3]

[Total: 18]

9648/02/PU3 Prelim 2/2013


[Turn over
796
10 For
Examiners
Use
3. Bone marrow contains stem cells that divide by mitosis to form blood cells. Each time
a stem cell divides it forms a replacement stem cell and a cell that develops into a
blood cell.

Fig. 3.1 shows changes in the mass of DNA in a human stem cell from the bone
marrow during three cell cycles.

Fig. 3.1

(a) With reference to Fig. 3.1, state:

(i) what happens to bring about the changes in the mass of DNA per cell at K
and at L.

K ......
K (DNA) replication / synthesis / described ;

L cytokinesis / cytoplasmic division / cell division ;


......

L ...

...... [2]

(ii) how many blood cells are formed from the stem cell in the time shown.
3;
...... [1]

(iii) what happens to the number of chromosomes in the stem cell.


remain the same / stays constant / stay at 46 / AW ; ignore description of events
......
occurring before and during mitosis [1]

9648/02/PU3 Prelim 2/2013


797 For
11 Examiners
Use

Stem cells in bone marrow give rise to phagocytes, B-lymphocytes and T-


lymphocytes.

(b) During an immune response, cells divide by mitosis.

Suggest how mitosis is involved in an immune response.

.........
transcription (of specific genes) ; A reference to gene switching
protein / polypeptide, synthesis ; A translation
.........
lymphocytes / B -cells / T-cells / divide (by mitosis) ;
producing, clone / many cells ;
.........

.........

.........

........ [3]

A protein, mitosis-promoting factor (MPF), has been identified in cells. MPF is a


globular protein made from two polypeptide chains.

The presence of MPF is known to cause prophase to start.

(c) Describe the changes that occur during prophase in an animal cell.

.........

.........
appearance of chromosomes / condensation of chromatin / AW ;
chromosomes visible as two, sister chromatids / chromatids joined by a centromere ;
.........
spindle formation / spindle fibres form / microtubules assemble / AW ;
centrioles, move to / reach, opposite poles ;
.........
R sides / ends
disappearance of nucleolus ;
.........
disassembly / breakdown of, nuclear envelope ;
A nuclear membrane
.........

.........

........ [4]

9648/02/PU3 Prelim 2/2013


[Turn over
798
12 For
Examiners
Use
(d) MPF normally begins to break down and stops functioning during anaphase.

Suggest the possible consequences of MPF not breaking down.

mitosis / prophase, will begin again, too soon / immediately ;


.........
uncontrolled / repeated, cell division / mitosis ;
ignore (risk of), tumour formation / cancerous growth
.........
ref. to consequences on the timing of the cell cycle ;
.........

.........

.........

........ [3]

[Total: 14]

9648/02/PU3 Prelim 2/2013


799 For
13 Examiners
Use

BLANK PAGE

9648/02/PU3 Prelim 2/2013


[Turn over
800
14 For
Examiners
Use
4. A type of pheasant
p oc
ccurs in a ra
ange of colours, espec
cially when bred in cap
ptivity. It
ve green or purple plum
may, for exxample, hav mage.

Sometimess when a grreen male iss crossed with


w a green n female all the offsprin
ng, male
and female
e, are gree en. Howeve er, sometimmes a green n male croossed with a green
female results in offspring in wh
hich the ma ajority of th
he offspringg are green n, but in
which somee of the fem
males are pu
urple, as sh
hown in Table 4.1.

Table. 4.1

Pheno
otype Numberr of
offsprin
ng
green male 7
green ffemale 3
purple ffemale 4

(a)
(i) Using the results
r showwn in Tablee 4.1, a gen
neticist propposed a hyp
pothesis
which pred dicted a 3:1 ratio of green:purple phenotypess. She used the 2
test to test her hypothe
esis

Complete Table
T o calculate 2 .
4.2 to

Table. 4.2

Gre
een Plumage
e P
Purple Plum
mage

Obserrved (O) 7 + 3 = 10 4

Expec
cted (E) 10.5 3.5

OE

(O E)2

(O E)2
E

(O E)2
= 2
E

[2]

9648/0
02/PU3 Prelim 2/2013
801 For
15 Examiners
Use

(ii) Draw a con nclusion from


m your calc
culation in Table
T 4.2, giiven that the
e critical
value for 2 at p = 0.005 with one
e degree of freedom iss 3.84. Expllain why
this conclus
sion shouldd be treated with cautio
on.

...
..

.
1 not significant
s ;
2 no difference
d... betw
ween observed
.. d and expecte
d which canno
ot be explaineed
by chance
;
.
3 (obsserved ratio) iss a valid approoximation to a 3:1 ratio ;
4 (shoould be treated
d with cautionn since it is) baased on a smalll sample ;
...
..

.

...
..

.

.....


.

.....


. [3]

(b) Plum
mage colou
ur in pheasa
ants is sex-linked.

In birds,
b the se
ex chromos omes are referred to as
a W and Z Z, rather tha
an Y and
X as
a in mamm mals. The W chromos some has non genes thhat affect plumage
p
colo
our. The hetterogameticc sex is the female, no
ot the male. Thus the male
m has
two Z chromos somes (ZZ) and the fem male has one W and oone Z chrom mosome
(WZZ).

(i) Use a gene


etic diagram
m to explain the results in Table 4. 1.

[3]

9648/0
02/PU3 Prelim 2/2013
[Tu
urn over
802
16 For
Examiners
Use
(ii) Using the same sym mbols as in (b)(i), indicate thhe genotyp pes and
phenotypes rents which could give rise to purpple male offs
s of the pare spring.

.....


.

.....


. [2]

[T
Total: 10]

9648/0
02/PU3 Prelim 2/2013
803 For
17 Examiners
Use

BLANK PAGE

9648/02/PU3 Prelim 2/2013


[Turn over
804
18 For
Examiners
Use
5.
(a) An experiment was carried out to investigate the effect of temperature on the
rate of oxygen consumption of the lizard, Sauromalus hispidus. The body
temperature of a lizard varies with environmental temperature.

Several lizards were fitted with small, airtight masks that covered their heads.
Air was supplied inside the mask through one tube, and collected through
another. The differences between oxygen concentrations in the air supplied
for inhalation and the exhaled air enabled the researchers to measure the
rate of oxygen consumption of the lizards.

The rate of oxygen consumption of each lizard was measured when it was at
rest and when it was running. Measurements were made at different
temperatures ranging from 15 C to 40 C.

Fig. 5.1 shows the results.

Fig. 5.1

9648/02/PU3 Prelim 2/2013


805 For
19 Examiners
Use

(i) Running requires rapid use of ATP by muscle cells in the legs and heart of
a lizard.

With reference to the events occurring inside a mitochondrion, explain


why a faster use of ATP requires a greater rate of oxygen consumption.

......
1. ATP is made, in the electron transport chain/by oxidative
phosphorylation;
......
2. oxygen is the final electron acceptor;
3. in the, inner membrane of the mitochondrion/cristae;
......
4. transfer of electron (between electron carriers) provides energy;
5. energy used to pump hydrogen ions (into intermembrane space);
6. creates proton gradient;
......
7. diffusion of hydrogen ions down their electrochemical gradient
causes ATP to be synthesised;
......
8. ref. chemiosmosis/ATP synthase/stalked particles;
9. idea that if less oxygen (consumed/available) then fewer
......
electrons transferred along the chain;

......

...... [4]

(ii) Explain the effect of temperature on the rate of oxygen consumption in


Sauromalus when at rest.
1. at high temperatures, reactions/enzyme activity/metabolism,
......
faster;
2. because, molecules/enzymes/substrates, have more kinetic
......
energy;
3. more frequent collisions;
......
4. therefore, respiration/Krebs cycle/electron transport
chain/production
......
of reduced NAD, take place at a faster rate;
5. idea of increase in rate of anabolic reactions (requiring more
ATP);
......

...... [3]

9648/02/PU3 Prelim 2/2013


[Turn over
806
20 For
Examiners
Use
(b) In the light-independent stage of photosynthesis, the enzyme rubisco
catalyses the combination of carbon dioxide with ribulose bisphosphate,
RuBP. When the carbon dioxide concentration within the leaf is very low,
rubisco tends to combine oxygen, rather than carbon dioxide, with RuBP. This
process is called photorespiration. It reduces carbon dioxide assimilation and
therefore reduces crop yields.

Photorespiration is most likely to happen in hot, dry conditions.

Suggest why photorespiration is most likely to take place in hot, dry


conditions

1. stomata closed ;
.........
2. to reduce transpiration / to avoid too much loss of water ;
3. so carbon dioxide cannot enter the leaf ;
.........
4. so carbon dioxide concentration (in leaf / in chloroplast) becomes very low ;
.........

.........

.........

.........

.........

........ [3]

(c) It is expected that the carbon dioxide concentration in the atmosphere will
increase in the future, which would be expected to increase rates of
photosynthesis in many crop plants.

Investigations were carried out into the effect of increased carbon dioxide
concentration on the rate of photosynthesis in maize.

Maize plants were grown in open-air trials, in the same field and were
exposed to the same changes in the weather.
50% of the plants were exposed to a normal carbon dioxide
concentration.
50% of the plants were exposed to an increased carbon dioxide
concentration.
The rate of photosynthesis was measured as the net assimilation rate
of carbon dioxide.
Measurements were made at three-hourly intervals between 0700
hours and 1900 hours on three different days.

9648/02/PU3 Prelim 2/2013


807 For
21 Examiners
Use

The results are shown in Fig. 5.2.

Fig. 5.2

(i) Suggest an explanation for the lack of effect of carbon dioxide


concentration on the rate of photosynthesis in maize plants, shown by
these results.

......
1. CO2 concentration is always high ;
2. CO2 not limiting ;
......
3. another factor / light intensity / temperature, limiting ;
4. no photorespiration ;
......

...... [2]

(ii) Suggest one explanation for the changes in the rate of photosynthesis
between 0700 hours and 1900 hours on day 1.

......
1. idea of change in temperature ;
2. affects, light independent / light dependent, stage (of photosynthesis) ;
......
or
3. idea of change in light intensity ;
......
4. affects light dependent stage (of photosynthesis) ;

...... [2]

[Total: 14]

9648/02/PU3 Prelim 2/2013


[Turn over
808
22 For
Examiners
Use
6. A study finding published in the journal PLoS Genetics in 2013 was the first to show
exercise alters DNA methylation related to fat storage.

The finding adds to a growing body of knowledge about epigenetics that show though
we cannot change our DNA we can change the way our genes are expressed to fight
disease.

(a) Describe and explain the effect of DNA methylation in eukaryotic genome.

.........
DNA methylation leads to a more compact chromatin structure / chromatin
with a closed conformation / heterochromatin;
.........
Addition of methyl group to certain bases (usually cytosine) of DNA;
.........
DNA methytransferase catalyze the transfer of a methyl group to DNA;
.........
Transcription factors and RNA polymerase unable to bind to DNA;
.........
Methylated DNA attracts other proteins which and in turn recruits histone
deacetylation enzymes;
........ [3]

(b) Suggest how DNA methylation enables cells to differentiate.

.........
Unrequired genes are silenced/not expressed/switched off when methylated;

........ [1]

In the 1950s, Erwin Chargaff determined the relative quantities of the four bases in
DNA in different organisms. His results provided important evidence for the model of
DNA proposed by James Watson and Francis Crick in 1953. Some of Chargaffs data
is shown in Table 6.1.

Table. 6.1

9648/02/PU3 Prelim 2/2013


809 For
23 Examiners
Use

(c) Explain how the data in Table 6.1 helps to confirm the arrangement of bases
in DNA.

.........
1 (named) base / nucleotide, pairing ;
.........
2 purine pyrimidine ;
3 percentage of A = percentage of T ; A very similar
4 percentage of C = percentage of G ; A very similar
.........
5 data quote in support ;
.........

.........

........ [3]

(d) Table 6.2 shows Chargaffs data for a virus.

Table. 6.2

(i) State how the result for the virus differs from the results for all the
organisms given in Table 6.1.

......
idea that
percentages of, A and T / C and G, are not the same / three percentages are
......
similar;

......

...... [1]

(ii) Suggest why the results for the virus are different from all the other
organisms.

......
single-stranded DNA / not double-stranded / not a double helix ;
A may be other bases ;
......

......

...... [1]

[Total: 9]

9648/02/PU3 Prelim 2/2013


[Turn over
810
24 For
Examiners
Use
Section B

Answer one question.

Write your answers on the separate answer paper provided.


Your answers should be illustrated by large, clearly labelled diagrams, where appropriate.
Your answers must be in continuous prose, where appropriate.
Your answers must be set out in sections (a), (b) etc., as indicated in the question.

7.
(a) Outline the molecular structure of phospholipids in relation to their function in cell
membranes. [7]

(b) Explain the role of the Golgi body and its link to the rough endoplasmic reticulum.
[7]

(c) Describe the structure of ribosomes. [6]

[Total: 20]

8.
(a) Explain the effects of competitive and non-competitive inhibitors on the rate of
enzymatic activity. [6]

(b) Explain the mode of action of enzymes in terms of specificity and activation
energy. [10]

(c) Describe the effect of temperature on enzyme-catalyzed reactions. [4]

[Total: 20]

9648/02/PU3 Prelim 2/2013


811 For
25 Examiners
Use

BLANK PAGE

9648/02/0/N/12 Qns 8a 9648/02/0/N/12 Qns 8b 9648/02/0/N/12 Qns 8c


Outline the molecular structure of Explain the role of the Golgi body Describe the structure of
phospholipids in relation to their and its link to the rough ribosomes. [6]
function in cell membranes. [6] endoplasmic reticulum. [7]
(i) 2 separate subunits -
Structure Role Large unit and small unit
a) 2, fatty acid / 1. processing/modification/AW (ii) subunits bind together
hydrocarbon, chain / / packaging, of, proteins/ during protein synthesis
tails ; molecules ; (iii) made up of ribosomal
b) a phosphate group ; 2. description of modification RNA (rRNA) and protein
c) (most) contain, nitrogen / e.g. glycosylation (iv) Free ribosomes
choline (attached to 3. production of, secretory/ suspended in the cytosol
phosphate in, head / Golgi, vesicles (v) bound (fixed or
polar portion) ; 4. production of lysosomes attached) ribosomes
attached to the outside
Function Link to rER of the endoplasmic
- can form a bilayer ; (i) Proteins from the ER enter reticulum or nuclear
- link between, at its cis face (entry face), envelope
hydrophobic core / AW, which is convex and usually (vi) two types of ribosomes
and barrier to water- oriented toward the that differ in size and
soluble substances ; A nucleus. mass: 70S (found in
polar / ionic (ii) They are then transported prokaryotic cells) and
- idea of, hydrophilic / through the Golgi and exit 80S (found in eukaryotic
phosphate, head, forming from its concave trans face cells)
H bonds with water ; (exit face). (vii)Usually smallest
- A facing, water / watery (iii) protein synthesized in organelle in a cell.
environment / aqueous Rough ER
environment / cytoplasm / (iv) creates transport vesicle by
cytosol ref. contribution to pinching off a portion of its
fluid nature of own membrane.
membrane ; (v) vesicle then moves either to
- further detail ; e.g. mainly the cell surface membrane
saturated fatty acids, less or to the Golgi Apparatus.
fluid e.g. mainly
unsaturated fatty acids,
more fluid
- ref. to control over
membrane protein
orientation ; e.g.
hydrophobic

9648/02/PU3 Prelim 2/2013


[Turn over
812
26 For
Examiners
Use
Explain the effects of competitive Explain the mode of action of Describe the effect of
and non-competitive inhibitors enzymes in terms of specificity and temperature on enzyme-
on the rate of enzymatic activity. activation energy. [10] catalyzed reactions. [4]
[6]
Enzymes are Specific Temperature
Competitive Enzyme specificity depends on Heating increases
competitive inhibitor / a close fit between substrate molecular motion which
effect described in terms of molecules and their binding facilitates a higher rate of
competition ; sites on an enzyme successful collision
complementary shape Enzymes are highly specific between the enzymes and
to active site R same / due to the three dimensional substrates.
similar, structure to active configuration of its active site. As a result, there is
site fits into active site ; This is described by the lock greater probability of a
blocking entry of substrate / and key hypothesis, where the reaction occurring.
prevents formation of ES substrate is imagined being A temperature increases,
complex ; like a key whose shape is rate of enzyme-catalysed
lower rate at low substrate complementary to the enzyme reactions increase.
concentrations, but at high or lock. The temperature that
substrate concentration will The active site is made up promotes maximum
reach the same plateau ; contact residues and the activity is referred to as
increasing substrate catalytic residues the optimum temperature.
concentration reverses The catalytic residues are If the temperature is
inhibition ; responsible for the ability of increased above this
the enzyme to catalyse a level, then a rapid
Non-Competitive particular metabolic reaction decrease in the rate of the
non-competitive inhibitor / and they act on the bonds in reaction occurs despite
described in terms of not the substrate(s). the increasing frequency
competing ; The contact residues are of collisions.
fits into, a site other than responsible for the specificity This is because the
active site / allosteric site ; of the enzyme and form a thermal agitation of the
shape of enzyme changes / shape that is complementary to enzyme molecule
shape of active site the shape of the substrate. disrupts the hydrogen
changes ; Molecules which do not fit bonds, ionic bonds, and
active site no longer nicely into the active site will hydrophobic interactions
complementary shape to not undergo a catalysed that stabilize the active
substrate ; reaction conformation.
prevents formation of ES When the correct substrate The secondary and
complex ; enters the active site, tertiary structures of the
lower rate / no activity / noncovalent or secondary active site are gradually
does not reach the same rate forces bring substrate lost and the protein
at high substrate molecules into close contact molecule eventually
concentrations ; with catalytic residues. denatures.
increase substrate Once the products are formed, The denaturation of
concentration does not the products no longer fit into enzymes has a larger
reverse inhibition ; the active site and escaped into effect over the increase in
the surrounding medium, molecular motion once
leaving the active site free to the temperature rise past
receive other substrate. the optimum temperature.
This accounts for the
Enzymes lower Activation Energy rapid decrease in reaction
1. Activation energy is the rate
energy required to make If temperature is reduced
the substances react. to near or below freezing
2. The greater the activation point, enzymes are
energy, the slower the inactive, not denatured.
reaction at any particular
temperature.
3. If the activation energy of
a reaction decreased, the
rate of reaction would be
increased.
4 Enzymes by functioning

9648/02/PU3 Prelim 2/2013


813
Class Adm
A No

Candid
date Name
e:

2013 Prelimin
P nary Examinaation 2
Pre-U
University 3
H2 Biology 9648/03
3

20 Septe
ember 2013
3

2 hours
s
Additionall Materials: Writing paper

READ TH
HESE INSTR
RUCTIONS
S FIRST

Do not op ooklet until you are to


pen this bo old to do so
o.

Write yourr Index num


mber and na ame on all th he work you u hand in.
Write in da
ark blue or black pen on
o both side es of the paaper.
You may u use a soft pencil
p for an
ny diagramss, graphs orr rough work king.
Do not usee staples, paper
p clips, highlighterss, glue or co
orrection flu
uid.

Answer alll questionss.

At the end
d of the examination, faasten all you
ur work sec curely together.
The numb ber of markss is given in
n brackets [ ] at the end
d of each qu
uestion or ppart question
n.
At the end
d of the examination, faasten all you
ur work sec curely together.

Fo
or Examiner's Use
1

T
Total

This ques
stion pape r consists of 16 printed pages.

[Tu
urn over
814
2 For
Examiners
Use

Answer all questions

1. The pink bollworm moth, Pectinophora gossypiella, is a pest of cotton crops. The size
of its population can be reduced by releasing large numbers of sterile male moths
into cotton fields. The sterile male moths mate with wild females from the cotton
fields, but no offspring are produced.

Over a period of three years, 20 million genetically modified (GM) sterile male moths
were released in the USA. Each insect contained a gene coding for a red fluorescent
protein (DsRed) taken from a species of reef coral. The added DNA also included a
promoter.

(a) Explain why, in gene technology:

(i) genes for fluorescent proteins such as DsRed are now more commonly
used as markers than are genes for antibiotic resistance

......

......

......

...... [2]

(ii) a promoter needs to be included when transferring a gene from a coral


into an insect.

......

......

......

......

......

...... [3]

9648/03/PU3 Prelim 2/2013


815 For
3 Examiners
Use

(b) DsRed is visible at all stages of the life cycle of the moth, but the presence of
the gene in a particular individual can be confirmed by genetic fingerprinting,
using gel electrophoresis.

(i) Outline the principles of gel electrophoresis.

......

......

......

......

......

......

......

...... [4]

(ii) Explain how the presence of the gene for DsRed in a moth can be
confirmed once electrophoresis is complete.

......

......

......

......

...... [2]

(c) DsRed allows sterile male moths to be distinguished from wild moths when
caught in an insect trap in a field of cotton plants.

Suggest why it is important to be sure whether a moth caught in such a trap is


a released sterile male or a wild insect.

.........

.........

.........

........ [2]

9648/03/PU3 Prelim 2/2013


[Turn over
816
4 For
Examiners
Use
(d) The United States Department of Agriculture has ruled that the release of
sterile males to control insect pest numbers is environmentally preferable to
all other alternatives.

Suggest what information would be needed to determine whether the release


of the sterile male moths, carrying the gene for DsRed, has a damaging effect
on the environment.

.........

.........

.........

........ [2]

[Total: 15]

9648/03/PU3 Prelim 2/2013


817 For
5 Examiners
Use

BLANK PAGE

9648/03/PU3 Prelim 2/2013


[Turn over
818
6 For
Examiners
Use
2.
(a) Plant responses to environmental changes are co-ordinated by plant growth
substances (plant hormones).

Suggest why plants need to be able to respond to their environment.

.........

.........

.........

........ [2]

(b) The following investigation was carried out into the effects of plant growth
substances on germination

a large number of lettuce seeds was divided into eight equal batches
each batch of seeds was placed on moist filter paper in a Petri dish
and given a different treatment

The different treatments are shown in Table 2.1. Each tick represents one of
the eight batches of seeds.

Table. 2.1

Treatment concentration of gibberellin (mol dm-3)


0.00 0.05 0.50 5.00
A Water
B Abscisic
acid

The batches of seeds were left to germinate at 25C in identical conditions


and the percentage germination was calculated

Fig. 2.1 shows the results of this investigation

Fig. 2.1

9648/03/PU3 Prelim 2/2013


819 For
7 Examiners
Use

(i) Describe, with reference to Fig. 2.1, the effects of the plant growth
substances on the germination of lettuce seeds.

......

......

......

......

......

......

......

...... [4]

(ii) Explain why all the lettuce seeds were kept at 25 oC.

......

......

......

...... [2]

(iii) State three variables, other than temperature, that needed to be controlled
in the investigation.

1 ...

2 ...

3 ...[3]

(c) Propose two commercial uses of plant growth substances.

.........

.........

.........

........ [2]

[Total: 13]

9648/03/PU3 Prelim 2/2013


[Turn over
820
8 For
Examiners
Use
3. Genetic information in humans can be obtained by DNA profiling.

In DNA profiling, the polymerase chain reaction is used by a scientist to amplify a


particular sequence of DNA.

(a) Briefly describe the steps of this technique.

.........

.........

.........

.........

.........

.........

.........

........ [4]

Scientists investigating the performance of athletes found that one gene contributing
to the performance of sprinters is the ACTN3 gene. There are two alleles of the gene,
the 577R allele and the 577X allele. The 577X allele codes for a very short protein
fragment in muscle fibres due to a stop codon mutation.

Table 3.1 summarises the athletic potential for the three possible genotypes for the
ACTN3 gene.

Table. 3.1

ACTN3 genotype Athletic potential

577R / 577R outstanding sprinter

577R / 577X good sprinter or long-


distance runner

577X / 577X very good long-distance


runner

A scientist tested sprinters to see if they possessed the 577R allele. Samples were
obtained from athletes muscle fibres. A standard containing proteins of the same
lengths as the proteins coded for by both alleles 577X and 577R was used as a
comparison. The standard and the samples were exposed to gel electrophoresis. In
gel electrophoresis, protein molecules separate in the same way as DNA molecules.

9648/03/PU3 Prelim 2/2013


821 For
9 Examiners
Use

The result for the standard is shown in Fig. 3.1.

Standard Outstanding Good


Sprinter Sprinter

Fig. 3.1

(b) On Fig. 3.1, draw the bands expected for an outstanding sprinter and for a
good sprinter.

Explain why you have placed the bands in these positions.

.........

.........

.........

.........

.........

.........

.........

........ [4]

9648/03/PU3 Prelim 2/2013


[Turn over
822
10 For
Examiners
Use
A cybrid (cytoplasmic hybrid cell) is produced as shown in Fig. 3.2.

Fig. 3.2

9648/03/PU3 Prelim 2/2013


823 For
11 Examiners
Use

(c) The DNA of such a cybrid is 99.6% human. The remaining 0.4% of the DNA
is in the cytoplasm. Explain why there is DNA in the cytoplasm of the cybrid.

.........

.........

.........

........ [1]

(d) When the Human Fertilisation and Embryology Bill was considered by the UK
Parliament in 2008, some people argued that it is unethical to allow the
production of cybrids.

State whether you agree or disagree that this is unethical and explain why
you reached this decision.

.........

.........

.........

.........

.........

.........

.........

........ [3]

[Total: 12]

9648/03/PU3 Prelim 2/2013


[Turn over
824
12 For
Examiners
Use
4. The enzyme urease is a catalyst of the hydrolysis of urea in solution, forming
ammonia and carbon dioxide, for example in the breakdown of urea in soils by
microorganisms.

You are required to plan an investigation to compare the activity of urease free in
solution and urease immobilised in alginate beads.

As the reaction proceeds, the ammonia released dissolves, causing the pH to


increase.

You are provided with the following equipment which you may use or not in your plan,
as you wish. You may not use any additional equipment in your plan.

an unlimited supply of calcium alginate beads all of uniform size prepared with
a 50 g dm-3 urease solution (you may call this immobilised urease)
an unlimited volume of 50 g dm-3 urease solution (you may call this free
urease)
an unlimited volume of 1.0 mol dm-3 urea solution
an unlimited volume of distilled water
beakers and flasks of different sizes
stop watch or electronic timer
broad and narrow range pH papers and liquids with appropriate colour charts,
pH probes and meters
colorimeter and tubes/cuvettes
thermometer
thermostatically-controlled water baths
graduated pipettes and pipette fillers
filter funnels
syringes
glass rods for stirring
test-tubes and boiling tubes
test-tube racks

Your plan should include a clear statement of the hypothesis or prediction and should
identify key variables.

Your plan should be written in clear scientific language and should be illustrated with
a diagram.

Your plan should give full details and explanations of the procedures that you would
adopt to ensure that the results are as precise and reliable as possible. A brief risk
assessment should be included.
[Total: 12]

9648/03/PU3 Prelim 2/2013


825 For
13 Examiners
Use

..........

..........

..........

..........

..........

..........

..........

..........

..........

..........

..........

..........

..........

..........

..........

..........

..........

9648/03/PU3 Prelim 2/2013


[Turn over
826
14 For
Examiners
Use
..........

..........

..........

..........

..........

..........

..........

..........

..........

..........

..........

..........

..........

..........

..........

..........

..........

..........

..........

..........

..........

..........

..........

..........

..........

..........

..........

..........

9648/03/PU3 Prelim 2/2013


827 For
15 Examiners
Use

..........

..........

..........

..........

..........

..........

..........

..........

..........

..........

..........

..........

..........

..........

..........

..........

..........

..........

..........

..........

..........

..........

..........

..........

..........

..........

..........

..........

9648/03/PU3 Prelim 2/2013


[Turn over
828
16 For
Examiners
Use
Free-response question

Write your answer to this question on the separate answer paper provided.
Your answer:
should be illustrated by large, clearly labelled diagrams, where appropriate;
must be in continuous prose, where appropriate;
must be set out in sections (a), (b) etc., as indicated in the question.

5.

(a) Describe the genetic diseases SCID (severe combined immunodeficiency) and
cystic fibrosis. [12]

(b) Discuss the factors which prevent gene therapy from becoming an effective
treatment. [8]

[Total: 20]

9648/03/PU3 Prelim 2/2013


829
Class Adm
A No

Candid
date Name
e:

2013 Prelimin
P nary Examinaation 2
Pre-U
University 3
H2 Biology 9648/03
3

20 Septe
ember 2013
3

2 hours
s
Additionall Materials: Writing paper

READ TH
HESE INSTR
RUCTIONS
S FIRST

Do not op ooklet until you are to


pen this bo old to do so
o.

Write yourr Index num


mber and na ame on all th he work you u hand in.
Write in da
ark blue or black pen on
o both side es of the paaper.
You may u use a soft pencil
p for an
ny diagramss, graphs orr rough work king.
Do not usee staples, paper
p clips, highlighterss, glue or co
orrection flu
uid.

Answer alll questionss.

At the end
d of the examination, faasten all you
ur work sec curely together.
The numb ber of markss is given in
n brackets [ ] at the end
d of each qu
uestion or ppart question
n.
At the end
d of the examination, faasten all you
ur work sec curely together.

Fo
or Examiner's Use
1

T
Total

This ques
stion pape r consists of 16 printed pages.

[Tu
urn over
830
2 For
Examiners
Use

Answer all questions

1. The pink bollworm moth, Pectinophora gossypiella, is a pest of cotton crops. The size
of its population can be reduced by releasing large numbers of sterile male moths
into cotton fields. The sterile male moths mate with wild females from the cotton
fields, but no offspring are produced.

Over a period of three years, 20 million genetically modified (GM) sterile male moths
were released in the USA. Each insect contained a gene coding for a red fluorescent
protein (DsRed) taken from a species of reef coral. The added DNA also included a
promoter.

(a) Explain why, in gene technology:

(i) genes for fluorescent proteins such as DsRed are now more commonly
used as markers than are genes for antibiotic resistance

......
1. easier to, identify / screen;
2. more economical / time saving / labour saving / harmless;
3. resistance gene(s) can be passed to other bacteria;
......
4. idea of antibiotics no longer effective
or
......
requiring development of new antibiotics;

...... [2]

(ii) a promoter needs to be included when transferring a gene from a coral


into an insect.
1. promoter, initiates transcription / switches on gene /causes gene expression / AW;
......
2. ref. binding of, RNA polymerase / transcription factors;
3. otherwise gene has to be inserted near an existing promoter;
4. this is difficult to do / this may disrupt expression of existing gene;
......
5. in eukaryotes precise position of promoter important;
6. idea that you need a coral promoter to switch on a coral gene;
......

......

......

...... [3]

9648/03/PU3 Prelim 2/2013


831 For
3 Examiners
Use

(b) DsRed is visible at all stages of the life cycle of the moth, but the presence of
the gene in a particular individual can be confirmed by genetic fingerprinting,
using gel electrophoresis.

(i) Outline the principles of gel electrophoresis.

......
1. DNA fragmented by, restriction enzyme(s) / endonuclease(s);
2. loaded (into wells) at, negative end / cathode end, (of gel);
3. ref. buffer / electrolyte;
......
4. phosphate groups of DNA give negative charge;
5. (negatively charged) DNA attracted to, anode / positive electrode;
6. separation due to, electric field / PD / potential difference;
......
7. short pieces / smaller mass, move further (in unit time) / move faster; ora
8. ref. impedance of gel / AW;
......

......

......

......

...... [4]

(ii) Explain how the presence of the gene for DsRed in a moth can be
confirmed once electrophoresis is complete.
1. idea of comparison of position with reference DNA;
......
2. ref. staining / fluorescence in UV;
3. by use of DNA probe;
4. ref. single-stranded / complementary base pairing;
......

......

......

...... [2]

(c) DsRed allows sterile male moths to be distinguished from wild moths when
caught in an insect trap in a field of cotton plants.

Suggest why it is important to be sure whether a moth caught in such a trap is


a released sterile male or a wild insect.
1. allows estimate of numbers of each type;
.........
2. to check success (of release of sterile males);
3. if sterile males wrongly identified as wild;
4. there will be a waste of resources, e.g. pesticides;
.........
5. if wild males wrongly identified as sterile males;
6. a potential infestation may be missed;
7. AVP; e.g. to determine which moths to (re)release
.........

........ [2]

9648/03/PU3 Prelim 2/2013


[Turn over
832
4 For
Examiners
Use
(d) The United States Department of Agriculture has ruled that the release of
sterile males to control insect pest numbers is environmentally preferable to
all other alternatives.

Suggest what information would be needed to determine whether the release


of the sterile male moths, carrying the gene for DsRed, has a damaging effect
on the environment.
1. that DsRed is not toxic to predators of the moth;
.........
2. that DsRed does not persist in the environment;
3. that the gene cannot pass to other organisms;
4. does not alter, food web / ecosystem, (in harmful way);
.........

.........

........ [2]

[Total: 15]

9648/03/PU3 Prelim 2/2013


833 For
5 Examiners
Use

BLANK PAGE

9648/03/PU3 Prelim 2/2013


[Turn over
834
6 For
Examiners
Use
2.
(a) Plant responses to environmental changes are co-ordinated by plant growth
substances (plant hormones).

Suggest why plants need to be able to respond to their environment.


to cope with changing conditions / AW ; R adapt
.........
avoid abiotic (non-living chemical and physical factors in the environment) stress ;
.........
to maximise photosynthesis
or
.........
to obtain more, light / water / minerals ; ora

avoid, herbivory / grazing ;


........
(methods of preventing grazing could include producing more toxins / more spines / encouraging stinging [2]
ants) R predation
(b)to ensure,
The following investigation was carried out into the effects of plant growth
germination in suitable conditions / pollination /
substances on germination
seed dispersal ;
a large number of lettuce seeds was divided into eight equal batches
each batch of seeds was placed on moist filter paper in a Petri dish
and given a different treatment

The different treatments are shown in Table 2.1. Each tick represents one of
the eight batches of seeds.

Table. 2.1

Treatment concentration of gibberellin (mol dm-3)


0.00 0.05 0.50 5.00
A Water
B Abscisic
acid

The batches of seeds were left to germinate at 25C in identical conditions


and the percentage germination was calculated

Fig. 2.1 shows the results of this investigation

Fig. 2.1

9648/03/PU3 Prelim 2/2013


835 For
7 Examiners
Use

(i) Describe, with reference to Fig. 2.1, the effects of the plant growth
substances on the germination of lettuce seeds.

......
in water / in A / with no abscisic acid,
germination increases as conc. GA increases ;
......
when abscisic acid present / in B, no germination ;

maximum germination 90% with 5 mol dm-3 GA,


......
in water / without abscisic acid ; A 91% ( 2%) for 90%

......
2 comparative figures (x and y refs. plus units) ;

GA concentration increases,
......
logarithmically / by a factor of 10, on x axis ;

10 times more GA gives,


......
3 (conc 0.05 to 0.5) / 0.5 (conc 0.5 to 5),
times more germination ;
......

...... [4]

(ii) Explain why all the lettuce seeds were kept at 25 oC.

so temperature doesnt affect results /


......
so only desired variable(s) changed /
to show just the effect of plant hormones ; A fair test
......
since temperature affects enzyme activity / optimum temperature for enzyme activity /
this is the temperature when enzymes work best;
......
suitable / optimum, temperature for (lettuce) germination ;
...... [2]
volumes of liquid(s) ;
(iii) State three
ABAvariables,
concentration ;other than temperature, that needed to be controlled
in the investigation.
oxygen availability ;
age of seeds ;
previous storage of seeds / viability idea ;
1 ...
genotype / variety, of seeds ;
size / type of, petri dish / filter paper ;
length of time experiment left for (before recording results) ;
2 ...
space between seeds ;
light qualified (duration / intensity / wavelength);
3 use of distilled water;
...[3]
all lids, off / on;

(c) Propose two commercial uses of plant growth substances.

.........
seedless, fruits / grapes ;
weedkillers ;
.........
rooting powder / to grow cuttings / used in tissue culture ;
control fruit ripening ;
controls fruit drop ;
.........
restrict hedge growth ;
preserve, cut flowers / green vegetables ;
specific example of improved fruit quality ;
........
producing malt / in brewing ; [2]

[Total: 13]

9648/03/PU3 Prelim 2/2013


[Turn over
836
8 For
Examiners
Use
3. Genetic information in humans can be obtained by DNA profiling.

In DNA profiling, the polymerase chain reaction is used by a scientist to amplify a


particular sequence of DNA.

(a) Briefly describe the steps of this technique.

.........
heat DNA to approximately 90 C or to separate strands;
cool to approximately 50 C or to anneal/attach primers;
.........
heat to approx 72 C or Taq/DNA polymerase copies strands;
repeat cycle;
.........

.........

.........

.........

.........

........ [4]

Scientists investigating the performance of athletes found that one gene contributing
to the performance of sprinters is the ACTN3 gene. There are two alleles of the gene,
the 577R allele and the 577X allele. The 577X allele codes for a very short protein
fragment in muscle fibres due to a stop codon mutation.

Table 3.1 summarises the athletic potential for the three possible genotypes for the
ACTN3 gene.

Table. 3.1

ACTN3 genotype Athletic potential

577R / 577R outstanding sprinter

577R / 577X good sprinter or long-


distance runner

577X / 577X very good long-distance


runner

A scientist tested sprinters to see if they possessed the 577R allele. Samples were
obtained from athletes muscle fibres. A standard containing proteins of the same
lengths as the proteins coded for by both alleles 577X and 577R was used as a
comparison. The standard and the samples were exposed to gel electrophoresis. In
gel electrophoresis, protein molecules separate in the same way as DNA molecules.

9648/03/PU3 Prelim 2/2013


837 For
9 Examiners
Use

The result for the standard is shown in Fig. 3.1.

Standard Outstanding Good


Sprinter Sprinter

Fig. 3.1

(b) On Fig. 3.1, draw the bands expected for an outstanding sprinter and for a
good sprinter.

Explain why you have placed the bands in these positions.

.........
On Fig 3.1,

.........
Outstanding Spinter 1 band near well
Good Sprinter 2 bands, same as standard
.........
An outstanding sprinter would have only one band as only one allele is
present/homozygous.
.........

.........
A good sprinter has two different alleles/heterozygous.

.........

.........

........ [4]

9648/03/PU3 Prelim 2/2013


[Turn over
838
10 For
Examiners
Use
A cybrid (cytoplasmic hybrid cell) is produced as shown in Fig. 3.2.

Fig. 3.2

9648/03/PU3 Prelim 2/2013


839 For
11 Examiners
Use

(c) The DNA of such a cybrid is 99.6% human. The remaining 0.4% of the DNA
is in the cytoplasm. Explain why there is DNA in the cytoplasm of the cybrid.

.........
1 (mitochondrial DNA) from the cow / not of human origin ;

.........

.........

........ [1]

(d) When the Human Fertilisation and Embryology Bill was considered by the UK
Parliament in 2008, some people argued that it is unethical to allow the
production of cybrids.

State whether you agree or disagree that this is unethical and explain why
you reached this decision.

unethical: .........
1 presence of human and animal DNA means it is partly human and partly cow / a
humananimal .........
hybrid which would not happen in nature ;
2 claims of the benefits of embryonic stem cell research are over-rated / few (if any)
.........
examples of success in medical applications ;
3 dangerous precedent / thin end of the wedge argument / current applications (may be
.........
benign) but will lead to abuse in future ;
4 possibility of unforeseen (irreversible) consequences ;
5 unnecessary.........
because, there already are / will soon be, alternative techniques (to the use
of embryonic stem cells) ;
6 e.g. use of adult stem cells/umbilical cord cells ;
.........
ethical: .........
1 the amount of non-human DNA is very small/negligible ;
2 protocol limits keeping embryo to 14 days (to guard against abuse) so cannot develop
........ [3]
into a whole organism ;
3 provides a more ethical alternative to the use of embryonic stem cells from humans ;
4 can provide many more stem cells than is possible from embryonic stem cells ; [Total: 12]
5 it would be unethical not to use cybrids to relieve, human suffering / e.g. diseases such
as Parkinsons disease ;
6 techniques which offer alternatives to embryonic stem cells builds on work done initially
on embryonic stem cells ;
7 idea that ovum has bovine proteins which would be rejected at implantation ;

Allow other reasonable arguments on either side. If a candidate does not express an opinion,
limit score to one mark unless an attempt has been made to argue that it is a finely balanced
matter. Ignore references to playing God and against religion.

9648/03/PU3 Prelim 2/2013


[Turn over
840
12 For
Examiners
Use
4. The enzyme urease is a catalyst of the hydrolysis of urea in solution, forming
ammonia and carbon dioxide, for example in the breakdown of urea in soils by
microorganisms.

You are required to plan an investigation to compare the activity of urease free in
solution and urease immobilised in alginate beads.

As the reaction proceeds, the ammonia released dissolves, causing the pH to


increase.

You are provided with the following equipment which you may use or not in your plan,
as you wish. You may not use any additional equipment in your plan.

an unlimited supply of calcium alginate beads all of uniform size prepared with
a 50 g dm-3 urease solution (you may call this immobilised urease)
an unlimited volume of 50 g dm-3 urease solution (you may call this free
urease)
an unlimited volume of 1.0 mol dm-3 urea solution
an unlimited volume of distilled water
beakers and flasks of different sizes
stop watch or electronic timer
broad and narrow range pH papers and liquids with appropriate colour charts,
pH probes and meters
colorimeter and tubes/cuvettes
thermometer
thermostatically-controlled water baths
graduated pipettes and pipette fillers
filter funnels
syringes
glass rods for stirring
test-tubes and boiling tubes
test-tube racks

Your plan should include a clear statement of the hypothesis or prediction and should
identify key variables.

Your plan should be written in clear scientific language and should be illustrated with
a diagram.

Your plan should give full details and explanations of the procedures that you would
adopt to ensure that the results are as precise and reliable as possible. A brief risk
assessment should be included.
[Total: 12]

9648/03/PU3 Prelim 2/2013


841 For
13 Examiners
Use

Hypothesis or prediction ;
e.g. rate of hydrolysis is faster using free enzyme/quantity of urea hydrolysed over time is
greater with free enzyme/immobilised urease catalyses reaction over much longer period of time

Theory to support candidates hypothesis or prediction ;


e.g. refs to accessible active sites/diffusion of substrate into alginate beads/stability of enzyme
in alginate beads

Outline of strategy ;
e.g. method of following the reaction taking samples at intervals and calculating the initial rate

Justification/evaluation, of strategy ;
e.g. can only alter concentration of immobilised enzyme by changing number of
beads/limitations of colour comparison these could be awarded at the end of the plan

Method of determining, pH / (the concentration) of ammonium carbonate, at intervals ;


e.g. use of pH indicator, to follow colour change

At least two control variables ;


e.g. temperature, concentration of urea solution, volumes used, number of beads

Risk assessment ;
ref to hazard and precaution

use range of concentrations of urea ;

use range of concentrations of urease ;


..........
to find suitable concentrations to make comparison

dilution table(s) included ; A ratios


..........
method to ensure concentration of urease in reaction mixtures is the same for both free and
..........
immobilised enzyme ;

..........
urea solution mixed with pH indicator ;

equilibration in water bath ;


..........
mixing, urease/beads, and urea solution at time = 0 ;
..........
staggered start ;
..........
samples taken at stated intervals ;

..........
uncertainty/precision, of results ;

plot results and take gradient to give initial rate ;


..........
colour standard set up at known pH ;
..........
time taken (t) to reach colour standard recorded ;
..........
rate = 1/t ; A 1000/t, etc.
..........
colour change followed in colorimeter ;

..........
repeats/replicates (calculate means) ;

calculate, standard deviation/standard error ;


..........
ref to use of t-test to see if rates are significantly different ;
..........
plot results on appropriate graph (bar or line) ;
..........
12max

..........

9648/03/PU3 Prelim 2/2013


[Turn over
842
14 For
Examiners
Use
..........

..........

..........

..........

..........

..........

..........

..........

..........

..........

..........

..........

..........

..........

..........

..........

..........

..........

..........

..........

..........

..........

..........

..........

..........

..........

..........

..........

9648/03/PU3 Prelim 2/2013


843 For
15 Examiners
Use

..........

..........

..........

..........

..........

..........

..........

..........

..........

..........

..........

..........

..........

..........

..........

..........

..........

..........

..........

..........

..........

..........

..........

..........

..........

..........

..........

..........

9648/03/PU3 Prelim 2/2013


[Turn over
844
16 For
Examiners
Use
Free-response question

Write your answer to this question on the separate answer paper provided.
Your answer:
should be illustrated by large, clearly labelled diagrams, where appropriate;
must be in continuous prose, where appropriate;
must be set out in sections (a), (b) etc., as indicated in the question.

5.

(a) Describe the genetic diseases SCID (severe combined immunodeficiency) and
cystic fibrosis. [12]

(b) Discuss the factors which prevent gene therapy from becoming an effective
treatment. [8]

(a) describe SCID (severe combined immunodeficiency) and cystic fibrosis. [12]
[Total: 20]
SCID
S1 more males affected (than females);
S2 recessive;
S3 (most common form is) sex-linked (X chromosome);
S4 mutation of IL2RG (interleukin-2 receptor gamma) gene (for gamma chain on
lymphocyte receptor);
S5 ADA (adenosine deaminase) (deficiency);
S6 mutation of gene on chromosome 20;
S7 prevents formation of, lymphocytes / T cells / B cells;
S8 toxic metabolites not broken down so lymphocytes killed;
S9 so susceptible to opportunistic infections;
S10 children usually die (if not treated);
S11 pneumonia / PCP / meningitis / chickenpox / measles / other valid;

CF
C1 autosomal/chromosome 7;
C2 recessive;
C3 mutation of CFTR/CFTCR gene;
C4 deletion of 3 base pair/codons;
C5 CFTR/CFTCR, (protein) has lost phenylalanine;
C6 (channels) no longer able to transport chloride ions
C7 causing thick mucus
C8 resulted in stated problems in lungs/pancreas/reproductive system
C9 reduces life expectancy

(b) discuss the factors which prevent gene therapy from becoming an effective
treatment. [8]

1. difficult to get DNA to integrate into the target cell genome;


2. results can be short-lived / patients need to be treated on a frequent basis;
3. ref. problem with controlling the activity of gene expression;
4. risk of stimulating immune response;
5. virus vector may, regain / develop, virulence;
6. incorrect insertion of gene may cause, named / described, problem to patient;
7. possible problem with finding vector for large gene;
8. many genetic diseases are a result of presence of many defective genes;
9. impossible to introduce many genes at same time;
10. problem dealing with, dominant condition / non dividing cells;
11. ref. problem with liposomes; (toxicity / low efficiency)

9648/03/PU3 Prelim 2/2013


845

Civics Group Reg Number



Candidates Name _______________________________________

MERIDIAN JUNIOR COLLEGE


JC2 Preliminary Examinations
Higher 2

______________________________________________________________________
H2 BIOLOGY 9648/01
Paper 1 Multiple Choice 27 September 2013
1 hour 15 minutes
Additional Materials: OMR
______________________________________________________________________

READ THESE INSTRUCTIONS FIRST

Do not open this booklet until you are told to do so.

Write in soft pencil.


Do not use staples, paper clips, highlighters, glue or correction fluid.
Write your name, civics group and register number in the spaces at the top of this page and on the
OMR.

There are 40 questions in this section. Answer all questions. For each question, there are four
possible answers labelled A, B, C and D.
Choose the one you consider correct and record your choice in soft pencil on the separate OMR
answer sheet.

_________________________________________________________________________
This paper consist of 25 printed pages
[Turn over]

846

1 The figure below shows two disaccharides, X and Y.

Which statements correctly describe X and Y as shown in the figure?

1 Glucose monomer(s) that make up X contains both -glucose and -glucose,


while the glucose monomer(s) that make up Y contains only -glucose.

2 The glycosidic bond that is formed in X is -1,2 glycosidic bond.

3 The glycosidic bond that is formed in Y is similar to the glycosidic bond that is
found in cellulose.

4 The general formula of a monomer is (CH2O2)n.

A 1 and 3 only

B 2 and 4 only

C 1, 3 and 4 only

D 1, 2, 3 and 4

847

2 The electron micrograph below shows a liver cell.

Which statements correctly describe the labelled structures?

1 Structure A transports proteins from Structure B to Golgi Apparatus.

2 Proteins enter the lumen of Structure B, where they undergo chemical


modifications such as glycosylation.

3 When Benedicts reagent is added to Structure C, the Benedicts reagent


would form a brick red precipitate.

4 The process shown in structure D is autolysis.

A 2 only

B 1 and 2 only

C 2 and 3 only

D 2, 3 and 4 only

848

3 Which one of the following statements about the cell surface membrane is true?

A It is totally impermeable to water due to the large number of hydrophobic


molecules it contains.

B It is a bilayer, 7 10nm thick, containing two layers of intrinsic proteins


sandwiched between two layers of phospholipids.

C It sometimes contains cholesterol which regulates membrane fluidity by


preventing it from becoming too fluid at high temperatures or too rigid at low
temperatures.

D Glycolipids and glycoproteins are biological markers which act as antibodies to


destroy foreign antigens of the infecting pathogens.

4 The table below summarises the results from experiments on the rate of absorption of certain
monosaccharides by pieces of mammalian intestine.

Cyanide inhibits cytochrome c oxidases.


Relative rate of absorption by intestine
(arbitrary units)
In physiological In physiological
saline saline with added
cyanide
Hexoses Glucose 1.00 0.33

Galactose 1.10 0.53

Fructose 0.43 0.37

Pentoses Xylose 0.30 0.31

Arabinose 0.29 0.29

Which of the following statements are consistent with these results?

1 Active transport is not involved in the uptake of pentoses.

2 Xylose and arabinose enter by osmosis.

3 Aerobic respiration is necessary for the maximal rate of uptake of glucose.

4 Smaller molecular weight sugars are absorbed more easily.

A 1 and 3 only

B 1, 2 and 3 only

C 1 and 4 only

D 2 and 4 only

849

5 The following figure shows nuclear division occurring in a lily plant.

Which of the following shows the correct sequence of events?

A 1>4>3>2

B 1>3>4>2

C 4>1>3>2

D 4>2>3>1

850

6 A biologist counted 2,500 cells from an embryo on a microscope slide and recorded the
following data.

Stage Number of cells

Prophase 125

Metaphase 50

Anaphase 50

Telophase 25

Interphase 2,250

From the table above, it could be reasonably concluded that

A the cells are undergoing differentiation.

B the cells are cancerous.

C the cells are dividing randomly.

D interphase is the longest stage of nuclear division.

7 Which statement about the cell cycle is not correct?

A The G2 checkpoint checks for DNA damage or mutation.

B Rapidly dividing cells have a shorter G1 phase than non-dividing cells.

C Protein synthesis mainly occurs during the G1 and G2 phases of the cell cycle.

D Dysregulation in cell cycle checkpoints causes chromosomal translocations and


cancer.

851

8 Curve X represents the course of an enzyme-catalysed reaction under optimum


conditions. Curve Y shows the action of the same enzyme on the same substrate but
with one alteration to the reaction conditions.

Which of the following factors, operating to a constant extent throughout the


experiment, could give the results shown by curve Y?

A Addition of a competitive inhibitor

B Addition of an end-product inhibitor

C An increased substrate concentration

D A lower temperature

852

9 The fig
gure below shows a DN
NA moleculle.

Which
h statementss correctly describe
d the
e polynucleotide?

1T
The structurre labelled A correspo
onds to a pu
urine, while
e the structuure labelled
dB
ccorresponds to a pyrim
midine.

2 T
The antiparrallel nature
e of DNA double helix aligns th
he DNA strrands so th hat
p
phosphodieester bonds s can form properly be
etween the nitrogenouus bases fro
om
o
opposite strrands.

3 D
Distance between
b ad
djacent de oxyribonucleotides is 3.4 and with 10
d
deoxyribonu
ucleotides per ote: 10
p turn. (No = 1 nm)

4 T
The wound DNA doub ble helix coonsists of alternating
a major
m grovees and min
nor
g
groves alon
ng its axis, that is essen
ntial for the binding witth proteins.

A 1 only

B 1 and 2 onlyy

C 3 and 4 only

D 1, 3 and 4 only

853

10 The figure below shows structures that are formed along a DNA molecule of a
eukaryotic cell during S phase of the cell cycle.

Which statement(s) correctly describe the labelled structure and the process that is
taking place?

1 Many such structures can also be found in prokaryotic DNA.

2 DNA replication requires primase to synthesize primers to provide free 3 OH for


the elongation of daughter strands by RNA polymerase.

3 Eukaryotic chromosomes face the end-replication problem as they are linear.


Prokaryotic chromosomes do not have the end-replication problem as they are
circular.

4 The end-replication problem occurs only on the lagging strands.

A 1 and 2 only

B 1 and 3 only

C 3 only

D 3 and 4 only

854

11 The electron micrograph below shows several labelled structures present in a


mitochondrion.

Which of the statements below correctly describes the labelled structures?


1 The structure labelled A is the polypeptide chain.

2 The structures labelled B are polyribosomes. Each ribosome consists of a 50S


large subunit and a 30S small subunit.

3 The structure labelled C is the 3 end of template DNA strand.

4 The structure labelled C is the 5 end of the mRNA strand.

A 1 and 2 only

B 1 and 4 only

C 2 and 3 only

D 1, 2 and 4 only

10

855

12 Which of the following statements correctly describes the type of mutation that gives rise to
sickle cell anaemia?

A Deletion of a codon resulting in the deletion of amino acid phenylalanine at position 508
of the polypeptide chain.

B Deletion of a base pair resulting in the deletion of amino acid phenylalanine at position
508 of the polypeptide chain.

C Base pair substitution of thymine by adenine on the non-template strand of DNA


causes a change in amino acid from glutamic acid to valine.

D Base pair substitution of thymine by adenine on the template strand of DNA causes a
change in amino acid from glutamic acid to valine.

13 Which of the following statement correctly describes introns and/or exons in a eukaryotic cell?

A Mutations to introns have absolutely no effect on the primary structure of protein.

B Different combinations of exons and introns allow more than one type of proteins to be
coded for by a gene.

C Exons are always translated into proteins.

D Single base-pair deletion on the first exon of a gene is likely to be more harmful than one
which occurs on the last exon of a gene.

14 The following statements describe a ribosome and a telomerase.

1 The genes coding for the ribosomal proteins are found in the nucleolus whereas the
genes coding for the spliceosomal proteins are found in other parts of the nucleus.

2 A ribosome is involved in condensation reactions whereas a telomerase is involved in


hydrolysis reactions.

3 Both function in the cytosol.

4 In performing their functions, both involve the formation of hydrogen bonds.

5 Both are found in the cells which constitute the inner cell mass of a blastocyst.

6 Both are ribonucleoproteins.

Which of the statements are true?

A 1, 2 and 3
B 1, 4 and 5
C 2, 3 and 6
D 4, 5 and 6

11

856

15 The figure below shows the structure of a virus.

Which of the following matches the functions of structures W Z?

W X Y Z

Ensures the integrity


A Entry of virus into Specificity of host
of the viral genome Assembly of viruses
host cell cell
is maintained

Ensures the integrity


B Entry of virus into Specificity of host
of the viral genome Assembly of viruses
host cell cell
is maintained

Ensures the integrity


C Specificity of host Entry of virus into
Assembly of viruses of the viral genome
cell host cell
is maintained.

Ensures the integrity


D Entry of virus into Specificity of host
Assembly of viruses of the viral genome
host cell cell
is maintained.

16 To date, more than 10 different strains of influenza virus (e.g. H1N1, H2N3, H5N1, H7N9 and
so on) have been documented.

Which of the following structural characteristic of influenza virus makes this possible?

A Single-stranded RNA as its genetic material


B Presence of an envelope that is derived from the host cell
C Eight separate segments of genetic material
D Presence of error-prone reverse transcriptase within the virus

12

857

17 During heat-shock transformation, competent E. coli cells take up plasmid DNA which contains
ampicillin-resistant gene. Successfully transformed cells are allowed to divide for several
generations in a medium without ampicillin. It is observed that some of the E. coli cells are no
longer resistant to ampicillin.

Which of the following best accounts for the observation?

A Error during DNA replication causes a loss-of-function mutation to the ampicillin-resistant


gene.

B Incorporation of the plasmid DNA into the heterochromatin region of the bacterial
genome renders the ampicillin-resistant gene inactive.

C Restriction endonucleases present in E. coli cells recognize specific sequences on the


plasmid DNA and hydrolyze it into fragments.

D Unequal distribution of plasmid during binary fission results in some daughter cells
without plasmids.

18 The following shows a process taking place among prokaryotic cells.

Which of the following best describes the process?

A A polynucleotide can be found within Structure A.

B Structure A is made up of protein subunits that are coded for by the genes carried on the
chromosomal DNA in Bacterium X.

C At the end of the process, Bacterium Y will be genetically identical to Bacterium X.

D DNA replication is taking place only in Bacterium X but not in Bacterium Y.

13

858

19 In a certain species of bacteria, the gene coding for enzyme X is not transcribed when the
repressor protein binds to the operator gene.

Which protein is prevented from functioning during this binding?

A Enzyme X
B TATA-binding protein
C RNA polymerase
D Repressor protein

20 A fisherman was surprised to catch a fish which had no scales (nude). To investigate the origin
of this phenotype the nude fish was mated several times to fish with scales and the result of
each cross was recorded. In the crosses of nude with scaled, a third phenotype appeared,
which was later called linear. The linear phenotype has only a single line of scales down one
side of the body.

The outcomes of these crosses are shown in the table.

Cross Parents Offspring phenotype and ratio


1 scaled x nude all offspring linear
2 linear x linear 1 scaled : 2 linear : 1 nude

Which of the following is conclusive from the above data?

A There is incomplete dominance between the nude and scaled phenotype.


B The environment is the reason for the loss of scales in the nude fish.
C All of the linear fish are homozygous.
D The nude fish are heterozygous.

14

859

21 Three gene loci in mice are shown below.

Locus 1 Locus 2 Locus 3


Coat colour Tail appearance Coat appearance
agouti kinky non-frizzy
albino straight frizzy

Crosses involving two loci at a time were set up and their outcomes are shown in the table below.

Parents Offspring of a test


F1
(pure breeding) cross of the F1

cross 1 agouti, non-frizzy coat 44


agouti, non-frizzy coat albino, frizzy coat 46
agouti, non-frizzy coat
x agouti, frizzy coat 5
albino, frizzy coat albino, non-frizzy coat 5

cross 2 agouti, straight tail 23


agouti, straight tail albino, kinky tail 27
agouti, kinky tail
x agouti, kinky tail 24
albino, kinky tail albino, straight tail 26

Which of the following statement is true about cross 1 and cross 2?

Cross 1 Cross 2

The agouti coat and kinky-tailed offspring of


A The frequency of crossing over between the
the test cross of the F1 are heterozygous at
locus 1 and locus 3 is 10%
both loci.

B Locus 1 and locus 3 undergo independent The albino coat and straight-tailed offspring
assortment. of the test cross are pure breeding.

C Locus 1 and locus 3 are located on the The F1 mice were test crossed with agouti
same chromosome. coat and straight-tailed mice.

D The interaction between locus 1 and locus 3 Locus 1 and locus 2 undergo independent
is an example of epistasis. assortment.

15

860

22 The graph below e results of a breeding


w shows the g experimennt with the vinegar fly Drosophila
a
melan nogaster. In each of the first 25 g
generations the smallest flies werre selected to produce
e
the neext generation. After 25
5 generatio ns the sele
ection was reversed.
r Frrom genera ation 2535
5
the larrgest flies were
w chosen
n to breed th
he next gen
neration.

h of the follo
Which owing can be concluded
d from the result
r of the
e experimennt?

A T
The flies fro
om generation 25 onwa
ards are phenotypically
y identical.

B B
Body size iss inherited in a polygen
nic manner..

C B ne with morre than two alleles.


Body size iss controlled by one gen

D B
Body size iss a sex-linke
ed trait.

23 In Primmula, the dominant allele,


a A, o
of a gene for anthoc cyanin pigm ment production givess
magen nta-coloured
d flowers. The rece ssive allele e, a, gives yellow-cooloured flo owers. The e
dominant allele, B,
B of anothe er unlinked gene inhib bits the effec
ct of allele A
A, while the
e recessive
e
allele, b, has no effect.
e Plantts in which a
allele A is in
nhibited by allele B havve yellow flowers.

A yello
ow-floweredd plant is kn
nown to be homozygou ve at the A/aa locus. The genotype
us recessiv e
at the B/b locus iss unknown.

Whichh of the following gen


notype of th
he test cro
oss will reveal the ge notype of the yellow--
flowerred plant?

A abb
aa B AABB C AAbb D aaBB

16

861

M N
24 One of the human blood groups is the MN group. There are two alleles, L and L , at this
gene locus which determine the presence of an antigen, M or N, on the surface of the red
M N
blood cells. The heterozygote L L has a different phenotype from each of the homozygotes.

The frequencies of the phenotypes of the MN blood group were measured in a European
population. Of the 100 individuals sampled, 40 were blood type M, 20 were blood type MN and
40 were blood type N.

Which of the following is conclusive from this data?

N
A The frequency of the L allele in the European population is 0.3.
M
B The frequency of the L allele in the European population is 0.5.
M
C There are 60 L alleles in the European population.
D There is a total pool of 200 alleles at this locus in the European population.

25 In the graph below, the rate of CO2 uptake by plant cells is shown to vary with increasing light
intensity.

Which of the following is true at point X?

A The plant is photosynthesising.


B Respiration equals photosynthesis.
C CO2 is a limiting factor.
D There is not enough light for photosynthesis to have commenced.

17

862

26 Removal of the source of carbon dioxide from photosynthesising chloroplasts results in rapid
changes in the concentration of certain chemicals. Which one of the following represents the
correct combination of concentration changes?

ATP ribulose bisphosphate glycerate-3-phosphate

A decreases decreases increases

B decreases increases no change

C increases increases decreases

D increases no change increases

27 Both glucose and appropriate enzymes are necessary for the process of glycolysis to begin.
Which additional compound must also be present?

A acetylcoenzyme A
B ATP
C pyruvate
D reduced NAD

28 What happens to most of the reduced NADP molecules in cell metabolism?

A They act as oxidising agents in glycolysis.


B They are oxidised in thylakoid membrane for ATP formation.
C They are oxidised in the Calvin cycle.
D They combine with succinic acid as part of Krebs cycle.

29 A populations gene pool was found to have remained unaltered for many generations. Which
of the following conditions must have existed in the population?

A mating had always been random


B genetic drift had often occurred
C inbreeding had been commonly practised
D certain alleles are at selective advantage

18

863

30 Competition, isolation and selection are thought to be events that take place during the
formation of new species. In which order did these events take place in the process of
speciation to form 13 species of Darwins finches?

A competition, selection, isolation


B selection, isolation, competition
C isolation, competition, selection
D competition, isolation, selection

31 The dendrite end of the axon is stimulated at the site 1 (see figure below). The excitation is
transferred from site 1 to 2 and then to 3 and 4. The excitation is measured at these sites.
Which statement of impulse frequencies (I) measured at these sites is correct?

A I(1) > I(2) > I(3), I(3) = I(4), I(3) + I(4) = I(2).

B I(1) < I(2) < I(3), I(3) = I(4).

C I(1) = I(2) > I(3), I(3) = I(4), I(3) + I(4) = I(2).

D I(1) = I(2) = I(3) = I(4).

19

864

32 Drosophila flies homozygous for the shake mutation are extremely sensitive to diethyl ether
that causes convulsions in homozygous individuals. Convulsions are caused by abnormalities
in nerve impulse conduction as shown in the graph below. Which of the following structures is
impaired in the shake mutations?

A Na+ channels
B K+ channels
C Ca2+ channels
D Na+/ K+ pump

20

865

33 Plasmid X can serve as a vector for the insertion of genes to be cloned.

Which of the following options will allow the selection of the colonies containing the
recombinant form of plasmid X?

Selection medium Phenotype of colonies that contain the inserted gene

A Containing ampicillin and


White colonies
lactose

B Containing ampicillin and


Colonies that emit light
luciferase

C Containing ampicillin,
White colonies
lactose and luciferin

D Containing ampicillin,
Colonies that emit light
lactose and luciferin

21

866

34 A scientist planned to insert a gene of approximately 2719 base pairs into a plasmid vector
using restriction enzyme EcoRI. The diagram below shows the recombinant plasmid with the
gene in the desired orientation.

Restriction mapping can be used to check the orientation of the gene inserted into a plasmid.
The recombinant plasmid is cleaved with restriction enzymes(s) and the fragments are run on
an electrophoresis gel to ascertain their size.

Which of the following results will confirm that the gene was not inserted in the desired
orientation?

Restriction enzyme(s) used Fragment size(s)


A PstI 1399bp, 6182bp
B PstI 2832bp, 4749bp
C EcoRI 2719bp, 4862bp
D PstI and EcoRI 643bp, 756bp, 2076bp, 4106bp

35 A sequence of DNA is to be amplified by the Polymerase Chain Reaction is shown below.

Which option shows suitable primers for amplifying this sequence?

Primer 1 Primer 2
A 5 ATTCTCGATCGG 3 5 TCGATATGATCG 3
B 5 CTTCCGATCGAG 3 5 ATTCTCGATCGG 3
C 5 GAAGGCTAGCTC 3 5 GCTAGTATAGCT 3
D 5 CCGATCGAGAAT 3 5 TCGATATGATCG 3

22

867

36 Restriction Fragment Length Polymorphism (RFLP) was used to analyse the inheritance of a
rare mutation that causes the pre-mature degeneration of the nervous system. A genetic
marker on Chromosome 5 was discovered to be associated with the disease and used in the
analysis. Genomic DNA was digested with a restriction enzyme BamHI and the results of the
gel electrophoresis and nucleic acid hybridisation are shown in the diagram below.

Which of the following observations cannot be made from the information provided?

A The mutation that causes the disease is probably recessive.


B The mutation that causes the disease is likely to be located in a coding region of the
genome.
C The children of II-1 are unlikely to inherit the disease.
D There can only be a maximum of three BamHI restriction sites on Chromosome 5 in the
affected individual.

37 Cloning plants by tissue culture is more advantageous as compared to traditional vegetative


propagation methods as

A The plantlets will require less space to grow to a saleable age.


B The plantlets can be grown regardless of seasonal variation.
C The plantlets will be genetically identical to the parent plant.
D The plantlets will be more resistant to fungi and diseases.

23

868

38 A proposed method of stem cell gene therapy is illustrated in the diagram below.

This treatment may not work because


A the patients immune system recognises the transplanted cells as foreign particles and
an immune response is triggered.

B only one copy of the normal allele is inserted to treat a recessively inherited disorder.

C the genetic disorder is caused by multiple genes.

D repeated transplants may be required.

39 Recent advances in the field of stem cell research have shown that induced pluripotent stem
cells (iPS cells) can be artificially derived from adult somatic cells. iPS cells are mostly similar
to natural pluripotent cells. This implies that

A iPS cells can theoretically differentiate into all cell types.


B iPS cells can theoretically differentiate into any of the three germ layers.
C iPS cells can theoretically differentiate into gametes.
D iPS cells are theoretically capable of transdifferentiation.

24

869

40 Which of the following theoretical application(s) of information obtained from the Human
Genome Project is/are unethical?

I. Gene therapies for only certain genetic diseases are developed.

II. Potential parents undergoing genetic testing and genetic counselling prior to
conceiving.

III. Potential parents choosing embryos for implantation only after ante-natal tests for
selected genes.

IV. Increasing insurance premiums for people who are genetically predisposed to cancer.

V. A scientist studying phylogeny and early human migration patterns.

A I and V
B I and III
C II and III
D III and IV

THE END

25

870

Civics Group Reg Number



Candidates Name _______________________________________

MERIDIAN JUNIOR COLLEGE


JC2 Preliminary Examinations
Higher 2

______________________________________________________________________
H2 BIOLOGY 9648/01
Paper 1 Multiple Choice 27 September 2013
1 hour 15 minutes
Additional Materials: OMR
______________________________________________________________________

READ THESE INSTRUCTIONS FIRST

Do not open this booklet until you are told to do so.

Write in soft pencil.


Do not use staples, paper clips, highlighters, glue or correction fluid.
Write your name, civics group and register number in the spaces at the top of this page and on the
OMR.

There are 40 questions in this section. Answer all questions. For each question, there are four
possible answers labelled A, B, C and D.
Choose the one you consider correct and record your choice in soft pencil on the separate OMR
answer sheet.

_________________________________________________________________________
This paper consist of 25 printed pages
[Turn over]

871

H2 PRELIM 2013 PAPER 1


ANSWER SCHEME

1. A 9. D 17. D 25. A 33.C

2. A 10. D 18. A 26. C 34. B

3. C 11. C 19. C 27. B 35. A

4. A 12. D 20. A 28. C 36. D

5. C 13. D 21. A 29. A 37. B

6. C 14. D 22. B 30. C 38. C

7.D 15. B 23. C 31. D 39. B

8. B 16. C 24. B 32. B 40. D

872

1 The figure below shows two disaccharides, X and Y.

Which statements correctly describe X and Y as shown in the figure?

1 Glucose monomer(s) that make up X contains both -glucose and -glucose,


while the glucose monomer(s) that make up Y contains only -glucose.

2 The glycosidic bond that is formed in X is -1,2 glycosidic bond.

3 The glycosidic bond that is formed in Y is similar to the glycosidic bond that is
found in cellulose.

4 The general formula of a monomer is (CH2O2)n.

A 1 and 3 only

B 2 and 4 only

C 1, 3 and 4 only

D 1, 2, 3 and 4

873

2 The electron micrograph below shows a liver cell.

Which statements correctly describe the labelled structures?

1 Structure A transports proteins from Structure B to Golgi Apparatus.

2 Proteins enter the lumen of Structure B, where they undergo chemical


modifications such as glycosylation.

3 When Benedicts reagent is added to Structure C, the Benedicts reagent


would form a brick red precipitate.

4 The process shown in structure D is autolysis.

A 2 only

B 1 and 2 only

C 2 and 3 only

D 2, 3 and 4 only

874

3 Which one of the following statements about the cell surface membrane is true?

A It is totally impermeable to water due to the large number of hydrophobic


molecules it contains.

B It is a bilayer, 7 10nm thick, containing two layers of intrinsic proteins


sandwiched between two layers of phospholipids.

C It sometimes contains cholesterol which regulates membrane fluidity by


preventing it from becoming too fluid at high temperatures or too rigid at low
temperatures.

D Glycolipids and glycoproteins are biological markers which act as antibodies to


destroy foreign antigens of the infecting pathogens.

4 The table below summarises the results from experiments on the rate of absorption of certain
monosaccharides by pieces of mammalian intestine.

Cyanide inhibits cytochrome c oxidases.


Relative rate of absorption by intestine
(arbitrary units)
In physiological In physiological
saline saline with added
cyanide
Hexoses Glucose 1.00 0.33

Galactose 1.10 0.53

Fructose 0.43 0.37

Pentoses Xylose 0.30 0.31

Arabinose 0.29 0.29

Which of the following statements are consistent with these results?

1 Active transport is not involved in the uptake of pentoses.

2 Xylose and arabinose enter by osmosis.

3 Aerobic respiration is necessary for the maximal rate of uptake of glucose.

4 Smaller molecular weight sugars are absorbed more easily.

A 1 and 3 only

B 1, 2 and 3 only

C 1 and 4 only

D 2 and 4 only

875

5 The following figure shows nuclear division occurring in a lily plant.

Which of the following shows the correct sequence of events?

A 1>4>3>2

B 1>3>4>2

C 4>1>3>2

D 4>2>3>1

876

6 A biologist counted 2,500 cells from an embryo on a microscope slide and recorded the
following data.

Stage Number of cells

Prophase 125

Metaphase 50

Anaphase 50

Telophase 25

Interphase 2,250

From the table above, it could be reasonably concluded that

A the cells are undergoing differentiation.

B the cells are cancerous.

C the cells are dividing randomly.

D interphase is the longest stage of nuclear division.

877

7 Which statement about the cell cycle is not correct?

A The G2 checkpoint checks for DNA damage or mutation.

B Rapidly dividing cells have a shorter G1 phase than non-dividing cells.

C Protein synthesis mainly occurs during the G1 and G2 phases of the cell cycle.

D Dysregulation in cell cycle checkpoints causes chromosomal translocations and


cancer.

8 Curve X represents the course of an enzyme-catalysed reaction under optimum


conditions. Curve Y shows the action of the same enzyme on the same substrate but
with one alteration to the reaction conditions.

Which of the following factors, operating to a constant extent throughout the


experiment, could give the results shown by curve Y?

A Addition of a competitive inhibitor

B Addition of an end-product inhibitor

C An increased substrate concentration

D A lower temperature

878

9 The fig
gure below shows a DN
NA moleculle.

Which
h statementss correctly describe
d the
e polynucleotide?

1T
The structurre labelled A correspo
onds to a pu
urine, while
e the structuure labelled
dB
ccorresponds to a pyrim
midine.

2 T
The antiparrallel nature
e of DNA double helix aligns th
he DNA strrands so th hat
p
phosphodieester bonds s can form properly be
etween the nitrogenouus bases fro
om
o
opposite strrands.

3 D
Distance between
b ad
djacent de oxyribonucleotides is 3.4 and with 10
d
deoxyribonu
ucleotides per ote: 10
p turn. (No = 1 nm)

4 T
The wound DNA doub ble helix coonsists of alternating
a major
m grovees and min
nor
g
groves alon
ng its axis, that is essen
ntial for the binding witth proteins.

A 1 only

B 1 and 2 onlyy

C 3 and 4 only

D 1, 3 and 4 only

879

10 The figure below shows structures that are formed along a DNA molecule of a
eukaryotic cell during S phase of the cell cycle.

Which statement(s) correctly describe the labelled structure and the process that is
taking place?

1 Many such structures can also be found in prokaryotic DNA.

2 DNA replication requires primase to synthesize primers to provide free 3 OH for


the elongation of daughter strands by RNA polymerase.

3 Eukaryotic chromosomes face the end-replication problem as they are linear.


Prokaryotic chromosomes do not have the end-replication problem as they are
circular.

4 The end-replication problem occurs only on the lagging strands.

A 1 and 2 only

B 1 and 3 only

C 3 only

D 3 and 4 only

10

880

11 The electron micrograph below shows several labelled structures present in a


mitochondrion.

Which of the statements below correctly describes the labelled structures?


1 The structure labelled A is the polypeptide chain.

2 The structures labelled B are polyribosomes. Each ribosome consists of a 50S


large subunit and a 30S small subunit.

3 The structure labelled C is the 3 end of template DNA strand.

4 The structure labelled C is the 5 end of the mRNA strand.

A 1 and 2 only

B 1 and 4 only

C 2 and 3 only

D 1, 2 and 4 only

12 Which of the following statements correctly describes the type of mutation that gives rise to
sickle cell anaemia?

A Deletion of a codon resulting in the deletion of amino acid phenylalanine at position 508
of the polypeptide chain.

B Deletion of a base pair resulting in the deletion of amino acid phenylalanine at position
508 of the polypeptide chain.

C Base pair substitution of thymine by adenine on the non-template strand of DNA


causes a change in amino acid from glutamic acid to valine.

D Base pair substitution of thymine by adenine on the template strand of DNA causes a
change in amino acid from glutamic acid to valine.

11

881

13 Which of the following statement correctly describes introns and/or exons in a eukaryotic cell?

A Mutations to introns have absolutely no effect on the primary structure of protein.

B Different combinations of exons and introns allow more than one type of proteins to be
coded for by a gene.

C Exons are always translated into proteins.

D Single base-pair deletion on the first exon of a gene is likely to be more harmful than one
which occurs on the last exon of a gene.

14 The following statements describe a ribosome and a telomerase.

1 The genes coding for the ribosomal proteins are found in the nucleolus whereas the
genes coding for the spliceosomal proteins are found in other parts of the nucleus.

2 A ribosome is involved in condensation reactions whereas a telomerase is involved in


hydrolysis reactions.

3 Both function in the cytosol.

4 In performing their functions, both involve the formation of hydrogen bonds.

5 Both are found in the cells which constitute the inner cell mass of a blastocyst.

6 Both are ribonucleoproteins.

Which of the statements are true?

A 1, 2 and 3
B 1, 4 and 5
C 2, 3 and 6
D 4, 5 and 6

12

882

15 The figure below shows the structure of a virus.

Which of the following matches the functions of structures W Z?

W X Y Z

Ensures the integrity


A Entry of virus into Specificity of host
of the viral genome Assembly of viruses
host cell cell
is maintained

Ensures the integrity


B Entry of virus into Specificity of host
of the viral genome Assembly of viruses
host cell cell
is maintained

Ensures the integrity


C Specificity of host Entry of virus into
Assembly of viruses of the viral genome
cell host cell
is maintained.

Ensures the integrity


D Entry of virus into Specificity of host
Assembly of viruses of the viral genome
host cell cell
is maintained.

16 To date, more than 10 different strains of influenza virus (e.g. H1N1, H2N3, H5N1, H7N9 and
so on) have been documented.

Which of the following structural characteristic of influenza virus makes this possible?

A Single-stranded RNA as its genetic material


B Presence of an envelope that is derived from the host cell
C Eight separate segments of genetic material
D Presence of error-prone reverse transcriptase within the virus

13

883

17 During heat-shock transformation, competent E. coli cells take up plasmid DNA which contains
ampicillin-resistant gene. Successfully transformed cells are allowed to divide for several
generations in a medium without ampicillin. It is observed that some of the E. coli cells are no
longer resistant to ampicillin.

Which of the following best accounts for the observation?

A Error during DNA replication causes a loss-of-function mutation to the ampicillin-resistant


gene.

B Incorporation of the plasmid DNA into the heterochromatin region of the bacterial
genome renders the ampicillin-resistant gene inactive.

C Restriction endonucleases present in E. coli cells recognize specific sequences on the


plasmid DNA and hydrolyze it into fragments.

D Unequal distribution of plasmid during binary fission results in some daughter cells
without plasmids.

18 The following shows a process taking place among prokaryotic cells.

Which of the following best describes the process?

A A polynucleotide can be found within Structure A.

B Structure A is made up of protein subunits that are coded for by the genes carried on the
chromosomal DNA in Bacterium X.

C At the end of the process, Bacterium Y will be genetically identical to Bacterium X.

D DNA replication is taking place only in Bacterium X but not in Bacterium Y.

14

884

19 In a certain species of bacteria, the gene coding for enzyme X is not transcribed when the
repressor protein binds to the operator gene.

Which protein is prevented from functioning during this binding?

A Enzyme X
B TATA-binding protein
C RNA polymerase
D Repressor protein

20 A fisherman was surprised to catch a fish which had no scales (nude). To investigate the origin
of this phenotype the nude fish was mated several times to fish with scales and the result of
each cross was recorded. In the crosses of nude with scaled, a third phenotype appeared,
which was later called linear. The linear phenotype has only a single line of scales down one
side of the body.

The outcomes of these crosses are shown in the table.

Cross Parents Offspring phenotype and ratio


1 scaled x nude all offspring linear
2 linear x linear 1 scaled : 2 linear : 1 nude

Which of the following is conclusive from the above data?

A There is incomplete dominance between the nude and scaled phenotype.


B The environment is the reason for the loss of scales in the nude fish.
C All of the linear fish are homozygous.
D The nude fish are heterozygous.

15

885

21 Three gene loci in mice are shown below.

Locus 1 Locus 2 Locus 3


Coat colour Tail appearance Coat appearance
agouti kinky non-frizzy
albino straight frizzy

Crosses involving two loci at a time were set up and their outcomes are shown in the table below.

Parents Offspring of a test


F1
(pure breeding) cross of the F1

cross 1 agouti, non-frizzy coat 44


agouti, non-frizzy coat albino, frizzy coat 46
agouti, non-frizzy coat
x agouti, frizzy coat 5
albino, frizzy coat albino, non-frizzy coat 5

cross 2 agouti, straight tail 23


agouti, straight tail albino, kinky tail 27
agouti, kinky tail
x agouti, kinky tail 24
albino, kinky tail albino, straight tail 26

Which of the following statement is true about cross 1 and cross 2?

Cross 1 Cross 2

The agouti coat and kinky-tailed offspring of


A The frequency of crossing over between the
the test cross of the F1 are heterozygous at
locus 1 and locus 3 is 10%
both loci.

B Locus 1 and locus 3 undergo independent The albino coat and straight-tailed offspring
assortment. of the test cross are pure breeding.

C Locus 1 and locus 3 are located on the The F1 mice were test crossed with agouti
same chromosome. coat and straight-tailed mice.

D The interaction between locus 1 and locus 3 Locus 1 and locus 2 undergo independent
is an example of epistasis. assortment.

16

886

22 The graph below e results of a breeding


w shows the g experimennt with the vinegar fly Drosophila
a
melan nogaster. In each of the first 25 g
generations the smallest flies werre selected to produce
e
the neext generation. After 25
5 generatio ns the sele
ection was reversed.
r Frrom genera ation 2535
5
the larrgest flies were
w chosen
n to breed th
he next gen
neration.

h of the follo
Which owing can be concluded
d from the result
r of the
e experimennt?

A T
The flies fro
om generation 25 onwa
ards are phenotypically
y identical.

B B
Body size iss inherited in a polygen
nic manner..

C B ne with morre than two alleles.


Body size iss controlled by one gen

D B
Body size iss a sex-linke
ed trait.

23 In Primmula, the dominant allele,


a A, o
of a gene for anthoc cyanin pigm ment production givess
magen nta-coloured
d flowers. The rece ssive allele e, a, gives yellow-cooloured flo owers. The e
dominant allele, B,
B of anothe er unlinked gene inhib bits the effec
ct of allele A
A, while the
e recessive
e
allele, b, has no effect.
e Plantts in which a
allele A is in
nhibited by allele B havve yellow flowers.

A yello
ow-floweredd plant is kn
nown to be homozygou ve at the A/aa locus. The genotype
us recessiv e
at the B/b locus iss unknown.

Whichh of the following gen


notype of th
he test cro
oss will reveal the ge notype of the yellow--
flowerred plant?

A abb
aa B AABB C AAbb D aaBB

17

887

M N
24 One of the human blood groups is the MN group. There are two alleles, L and L , at this
gene locus which determine the presence of an antigen, M or N, on the surface of the red
M N
blood cells. The heterozygote L L has a different phenotype from each of the homozygotes.

The frequencies of the phenotypes of the MN blood group were measured in a European
population. Of the 100 individuals sampled, 40 were blood type M, 20 were blood type MN and
40 were blood type N.

Which of the following is conclusive from this data?

N
A The frequency of the L allele in the European population is 0.3.
M
B The frequency of the L allele in the European population is 0.5.
M
C There are 60 L alleles in the European population.
D There is a total pool of 200 alleles at this locus in the European population.

25 In the graph below, the rate of CO2 uptake by plant cells is shown to vary with increasing light
intensity.

Which of the following is true at point X?

A The plant is photosynthesising.


B Respiration equals photosynthesis.
C CO2 is a limiting factor.
D There is not enough light for photosynthesis to have commenced.

18

888

26 Removal of the source of carbon dioxide from photosynthesising chloroplasts results in rapid
changes in the concentration of certain chemicals. Which one of the following represents the
correct combination of concentration changes?

ATP ribulose bisphosphate glycerate-3-phosphate

A decreases decreases increases

B decreases increases no change

C increases increases decreases

D increases no change increases

27 Both glucose and appropriate enzymes are necessary for the process of glycolysis to begin.
Which additional compound must also be present?

A acetylcoenzyme A
B ATP
C pyruvate
D reduced NAD
28 What happens to most of the reduced NADP molecules in cell metabolism?

A They act as oxidising agents in glycolysis.


B They are oxidised in thylakoid membrane for ATP formation.
C They are oxidised in the Calvin cycle.
D They combine with succinic acid as part of Krebs cycle.

29 A populations gene pool was found to have remained unaltered for many generations. Which
of the following conditions must have existed in the population?

A mating had always been random


B genetic drift had often occurred
C inbreeding had been commonly practised
D certain alleles are at selective advantage

19

889

30 Competition, isolation and selection are thought to be events that take place during the
formation of new species. In which order did these events take place in the process of
speciation to form 13 species of Darwins finches?

A competition, selection, isolation


B selection, isolation, competition
C isolation, competition, selection
D competition, isolation, selection

31 The dendrite end of the axon is stimulated at the site 1 (see figure below). The excitation is
transferred from site 1 to 2 and then to 3 and 4. The excitation is measured at these sites.
Which statement of impulse frequencies (I) measured at these sites is correct?

A I(1) > I(2) > I(3), I(3) = I(4), I(3) + I(4) = I(2).

B I(1) < I(2) < I(3), I(3) = I(4).

C I(1) = I(2) > I(3), I(3) = I(4), I(3) + I(4) = I(2).

D I(1) = I(2) = I(3) = I(4).

20

890

32 rosophila flies homozygous for the shake mutation are extremely sensitive to diethyl ether that
causes convulsions in homozygous individuals. Convulsions are caused by abnormalities in
nerve impulse conduction as shown in the graph below. Which of the following structures is
impaired in the shake mutations?

A Na+ channels
B K+ channels
C Ca2+ channels
D Na+/ K+ pump

21

891

33 Plasmid X can serve as a vector for the insertion of genes to be cloned.

Which of the following options will allow the selection of the colonies containing the
recombinant form of plasmid X?

Selection medium Phenotype of colonies that contain the inserted gene

A Containing ampicillin and


White colonies
lactose

B Containing ampicillin and


Colonies that emit light
luciferase

C Containing ampicillin,
White colonies
lactose and luciferin

D Containing ampicillin,
Colonies that emit light
lactose and luciferin

22

892

34 A scientist planned to insert a gene of approximately 2719 base pairs into a plasmid vector
using restriction enzyme EcoRI. The diagram below shows the recombinant plasmid with the
gene in the desired orientation.

Restriction mapping can be used to check the orientation of the gene inserted into a plasmid.
The recombinant plasmid is cleaved with restriction enzymes(s) and the fragments are run on
an electrophoresis gel to ascertain their size.

Which of the following results will confirm that the gene was not inserted in the desired
orientation?

Restriction enzyme(s) used Fragment size(s)


A PstI 1399bp, 6182bp
B PstI 2832bp, 4749bp
C EcoRI 2719bp, 4862bp
D PstI and EcoRI 643bp, 756bp, 2076bp, 4106bp

35 A sequence of DNA is to be amplified by the Polymerase Chain Reaction is shown below.

Which option shows suitable primers for amplifying this sequence?

Primer 1 Primer 2
A 5 ATTCTCGATCGG 3 5 TCGATATGATCG 3
B 5 CTTCCGATCGAG 3 5 ATTCTCGATCGG 3
C 5 GAAGGCTAGCTC 3 5 GCTAGTATAGCT 3
D 5 CCGATCGAGAAT 3 5 TCGATATGATCG 3

23

893

36 Restriction Fragment Length Polymorphism (RFLP) was used to analyse the inheritance of a
rare mutation that causes the pre-mature degeneration of the nervous system. A genetic
marker on Chromosome 5 was discovered to be associated with the disease and used in the
analysis. Genomic DNA was digested with a restriction enzyme BamHI and the results of the
gel electrophoresis and nucleic acid hybridisation are shown in the diagram below.

Which of the following observations cannot be made from the information provided?

A The mutation that causes the disease is probably recessive.


B The mutation that causes the disease is likely to be located in a coding region of the
genome.
C The children of II-1 are unlikely to inherit the disease.
D There can only be a maximum of three BamHI restriction sites on Chromosome 5 in the
affected individual.

37 Cloning plants by tissue culture is more advantageous as compared to traditional vegetative


propagation methods as

A The plantlets will require less space to grow to a saleable age.


B The plantlets can be grown regardless of seasonal variation.
C The plantlets will be genetically identical to the parent plant.
D The plantlets will be more resistant to fungi and diseases.

24

894

38 A proposed method of stem cell gene therapy is illustrated in the diagram below.

This treatment may not work because


A the patients immune system recognises the transplanted cells as foreign particles and
an immune response is triggered.

B only one copy of the normal allele is inserted to treat a recessively inherited disorder.

C the genetic disorder is caused by multiple genes.

D repeated transplants may be required.

39 Recent advances in the field of stem cell research have shown that induced pluripotent stem
cells (iPS cells) can be artificially derived from adult somatic cells. iPS cells are mostly similar
to natural pluripotent cells. This implies that

A iPS cells can theoretically differentiate into all cell types.


B iPS cells can theoretically differentiate into any of the three germ layers.
C iPS cells can theoretically differentiate into gametes.
D iPS cells are theoretically capable of transdifferentiation.

25

895

40 Which of the following theoretical application(s) of information obtained from the Human
Genome Project is/are unethical?

I. Gene therapies for only certain genetic diseases are developed.

II. Potential parents undergoing genetic testing and genetic counselling prior to
conceiving.

III. Potential parents choosing embryos for implantation only after ante-natal tests for
selected genes.

IV. Increasing insurance premiums for people who are genetically predisposed to cancer.

V. A scientist studying phylogeny and early human migration patterns.

A I and V
B I and III
C II and III
D III and IV

THE END

26

896

MERIDIAN JUNIOR COLLEGE


JC2 Preliminary Examinations 2013
Higher 2

CANDIDATE
NAME

CIVICS INDEX
GROUP 1 2 S NUMBER

_______________________________________________________________________________

H2 BIOLOGY 9648/02
Paper 2 Core Paper 18 September 2013
2 hours
Additional Materials: Answer papers
_______________________________________________________________________________

READ THESE INSTRUCTIONS FIRST

Do not open this booklet until you are told to do so.


Write your name, civics group and index number on all the work you hand in.
Write in dark blue or black pen on both sides of the paper.
You may use a soft pencil for any diagrams, graphs or rough working.
Do not use staples, paper clips, highlighters, glue or correction fluid/tape.

Section A
Answer all questions.
For examiners Use
Section B Section A
Answer any one question.
1 / 10

At the end of the examination, 2 / 12


1. Fasten your answer papers to section B securely together.
2. Hand in the following separately:
3 / 12
Section A (Part I)
Section A (Part II)
Section B 4 / 14

The number of marks is given in brackets [ ] at the end of each 5 /9


question or part question.
6 / 13

7 / 10
Section B
8/9 / 20

Total /100

________________________________________________________________________________
This paper consists of 21 printed pages.
[Turn over]
JC2Prelim2013Biology9648/02 1
897

Section A (Part I) For


Answer all the questions in this section. Examiners
Use

QUESTION 1
Heavy metal ions are waste products of many industrial processes and at times may leak into the
ocean and cause environmental pollution. The toxic effects of heavy metals on organisms stems
from their effect on metabolically-important enzymes, such as lactate dehydrogenase. Lactate
dehydrogenase catalyses the conversion of pyruvate into lactate.

Fig. 1.1 shows the effect of varying concentrations of mercury ions on the rate of reaction
catalysed by lactate dehydrogenase isolated from Oreochromis mossambicus, a species of fish.

Fig. 1.1

JC2Prelim2013Biology9648/02 2
898

(a) Describe and explain the effect of mercury ions on the rate of reaction catalysed by lactate For
dehydrogenase. [4] Examiners
Use

.........

.........

.........

.........

.........

.........

.........

.........

(b) Explain how mercury ions may affect the survival of the fish when oxygen concentrations are
low. [3]

.........

.........

.........

.........

.........

.........

JC2Prelim2013Biology9648/02 3
899

Human lysozyme is an important enzyme which is part of the human body's defence against For
some bacteria. It is found in abundance in tears, saliva and mucous. The main action of Examiners
Use
lysozyme is to catalyse the hydrolysis of -(1,4) glycosidic bonds between N-acetylmuramate
(NAM) and N-acetylglucosamine (NAG) sugar residues in the peptidoglycan cell walls of bacteria.
With the weakened cell wall, the bacterial cells are more likely to undergo lysis.

Fig. 1.2 shows the structure of lysozyme.

Fig. 1.2

(c) With reference to Fig. 1.2 and with your knowledge of haemoglobin, state two structural
differences between lysozyme and haemoglobin. [2]

1. ...

...

2. ...

...

(d) Suggest why some bacteria are more resistant to digestion by lysozymes. [1]

.........

.........

[Total: 10]

JC2Prelim2013Biology9648/02 4
900

QUESTION 2 For
Fig. 2.1 illustrates the Central Dogma, which describes the flow of genetic information from DNA Examiners
Use
to RNA to proteins.

Fig. 2.1

(a) Explain, with an example, how the Central Dogma may not always hold true. [2]

.........

.........

.........

.........

(b) Describe how the information carried on mRNA allows the synthesis of a complete
polypeptide chain. [3]

.........

.........

.........

.........

.........

.........

JC2Prelim2013Biology9648/02 5
901

A ribonucleoprotein is a protein that is associated with RNA. The ribosome is an example of a For
ribonucleoprotein. Prokaryotes have 70S ribosomes while eukaryotes have 80S ribosomes, each Examiners
Use
consisting of a small and large subunit. In prokaryotes, the small subunit is made up of a 16S
rRNA associated with 21 proteins. Fig. 2.2 shows the structure of a 16S rRNA.

Fig. 2.2
JC2Prelim2013Biology9648/02 6
902

(c) 16S rRNA and tRNA are both made up of four types of ribonucleotides linked by For
phosphodiester bonds. Examiners
Use

State two other similarities between the structures of the 16S rRNA and tRNA. [2]

1. ...

...

2. ...

...

(d) Describe how eukaryotic ribosomes are formed. [4]

.........

.........

.........

.........

.........

.........

.........

.........

Unlike eukaryotes, prokaryotes do not have endoplasmic reticulum or vesicles.

(e) Suggest how prokaryotes are still able to secrete proteins. [1]

.........

.........

[Total: 12]

JC2Prelim2013Biology9648/02 7
903

QUESTION 3 For
The Human Immunodeficiency Virus (HIV) weakens the immune system, causing the patient to Examiners
Use
succumb to secondary infections, leading to the development of AIDS. The first recognized case
of AIDS was reported in 1981 in Los Angeles, U.S.A.

Fig. 3.1 shows the binding of a HIV particle to a T cell. The viral receptor gp120 trimer binds to
both CD4 and a co-receptor called CCR5 on the T cell surface.

Fig. 3.1

(a) Explain why the composition of the viral envelope is similar to the composition of the T cell
membrane. [1]

.........

.........

(b) Individuals who carry a mutation known as CCR532 in both copies of the CCR5 gene are
resistant to certain strains of HIV. The CCR532 allele is a result of a deletion of a particular
sequence of 32 base-pairs within the exon of CCR5 gene.

Explain how an individual who is homozygous for the CCR532 allele is resistant to HIV. [4]

.........

.........

.........

.........

.........

.........

.........

JC2Prelim2013Biology9648/02 8
904

(c) It was observed that this strain of HIV that uses CCR5 as the co-receptor quickly evolves into For
another strain which bind to another co-receptor known as CXCR4. Examiners
Use

Suggest and explain how HIV can evolve to use CXCR4 as the co-receptor. [2]

.........

.........

.........

.........

(d) The allele frequency of CCR5-32 is unusually high in the western European countries, with
up to 20% of ethnic western Europeans carrying this allele, which is rare or absent in all other
ethnic groups throughout the world.

Suggest a reason for this phenomenon. [1]

.........

.........

(e) A technique called RNA interference has been shown to slow down the replication of HIV in
vitro. Double-stranded RNA molecules, 21 to 23 nucleotides long, were added to a culture of
T cells infected with HIV. The sequence of this small interfering RNA (siRNA) matches part of
the HIV protease gene. Once inside the T cell, the two strands of the siRNA separate into
single strands. One of the strands is identical in sequence to the mRNA of the HIV protease.

Explain how this siRNA is able to interfere with HIV replication. [4]

.........

.........

.........

.........

.........

.........

.........

[Total: 12]

JC2Prelim2013Biology9648/02 9
905

QUESTION 4 For
Gene expression in eukaryotic cells can be controlled in many ways, one of which is the Examiners
Use
regulation of mRNA production. Fig. 4.1 shows how transcription can be controlled.

Fig. 4.1

(a) Using the information in Fig. 4.1 and with your knowledge on the control of eukaryotic gene
expression, explain how transcription is initiated. [4]

.........

.........

.........

.........

.........

.........

.........

JC2Prelim2013Biology9648/02 10
906

Ubiquitin-dependent protein degradation in the proteasome is one way in which the amount of For
cytosolic proteins in a cell can be controlled at the post-translational level. An amino acid Examiners
Use
sequence of proline, glutamic acid, serine and threonine (PEST) in a protein are often recognized
by enzymes and tagged with ubiquitin proteins. Globular proteins in their correct conformation
are not tagged for degradation.

(b) Using the above information, suggest why denatured cytosolic proteins can be tagged with
ubiquitin. [1]

.........

.........

(c) Transmembrane proteins are usually not degraded by proteasome.

Suggest how transmembrane proteins on the cell surface are degraded. [2]

.........

.........

.........

JC2Prelim2013Biology9648/02 11
907

Ovalbumin
O g
gene from chicken wa as isolated and made single-stra anded, whicch was then mixed For
to
ogether witth its maturre mRNA. The
T results were obse erved underr an electroon microsco
ope. Fig. Examiners
E
Use
4.2
4 shows the electron n micrograph and its co orrespondin
ng diagrammatic repreesentation, showing
s
th
he binding oof the mRNA to certain
n regions off the DNA.

F
Fig. 4.2

(d) Explain w
why loops (A
( G) were observed . [3]







.........







.........







.........







.........






.........







.........

(e
e) Explain w
why the reg
gion of the mRNA, ind
dicated by th
he arrow in Fig. 4.2, reemains unb
bound to
DNA. [2]







.........






.........







.........







.........

JC 02
C2Prelim2013Biology9648/0 12
908

Repeating nucleotide sequences are common in the genome of eukaryotes. These repeating For
sequences have been commonly referred to as junk DNA. Examiners
Use

(f) Suggest why the term junk DNA is misleading. [2]

.........

.........

.........

[Total: 14]

JC2Prelim2013Biology9648/02 13
909

QUESTION 5 For
Hereditary hypophosphatemic rickets is a genetic disorder that results on low level of phosphate Examiners
Use
in the blood (hypophosphatemia).

Fig. 5.1 shows the inheritance of this disease over four generations in an extended family.

Fig. 5.1

(a) Explain the mode of inheritance of this disease. [2]

.........

.........

.........

.........

(b) Using the symbols D/d to represent the alleles, construct a genetic diagram to show how
individual III-7 and III-8 can pass on the disease to their children. [4]

JC2Prelim2013Biology9648/02 14
910

Fur colour of Labrador dogs is controlled by two genes, B/b and E/e. The genes are on different For
pairs of chromosomes. Examiners
Use

Allele B produces black fur while allele b produces brown fur. However, the colour is expressed
only in the presence of allele E. In the absence of allele E, the dogs are beige in colour, hence
the name Golden Labrador.

Crossing two heterozygotes typically gives the following phenotypic ratio:

9 black : 3 brown : 4 golden

(c) Explain how the phenotypic ratio will differ if genes B/b and E/e are located on the same
homologous pair of chromosomes, as shown in Fig. 5.2. [3]



Fig. 5.2

.........

.........

.........

.........

.........

[Total: 9]

End of Section A (Part I)

JC2Prelim2013Biology9648/02 15
911

Candidate Name Civics Group Index Number For


Examiners
Use

Meridian Junior College Question 6 Question 7
JC2 Preliminary Examinations 2013
H2 Biology (9648/02) / 13 / 10

Section A (Part II)
Answer all the questions in this section.

QUESTION 6
(a) Describe the role of the following ions in impulse transmission along a nerve cell.
+
(i) Na [2]

...

...

...
+
(ii) K [2]

...

...

...

2+
(b) Describe the role of Ca in the passage of impulses across a synapse. [3]

.........

.........

.........

.........

.........

.........

2+
(c) Explain why the concentration of Ca in the cytosol of the synaptic knob remains low despite
frequent opening of channels. [2]

.........

.........

.........

JC2Prelim2013Biology9648/02 16
912

A water-soluble toxin, muscarine, derived from a mushroom, causes deceleration of the heart For
rate, resulting in death of individuals. Fig. 6.1 shows the signalling pathway of muscarine on a Examiners
Use
postsynaptic membrane.

Fig. 6.1

(d) Describe how muscarine results in inhibition of nerve transmission in heart muscle cells. [4]

.........

.........

.........

.........

.........

.........

.........

.........

[Total: 13]

JC2Prelim2013Biology9648/02 17
913

QUESTION 7 For
Fig. 7.1 shows the distribution of malaria and the allele frequency of the sickle cell allele. Examiners
Use

Fig. 7.1

(a) Describe and explain the correlation between the occurrence of malaria and the frequency of
the sickle cell allele. [4]

.........

.........

.........

.........

.........

.........

.........

.........

JC2Prelim2013Biology9648/02 18
914

The evolution of haemoglobin molecules has been studied extensively by comparing the amino For
acid sequences in both myoglobin and haemoglobin. Myoglobin is used for oxygen storage while Examiners
Use
haemoglobin is used for oxygen transport. Ancient prehistoric animals had a single chain of
simple globin for oxygen storage and transport.

About 500 million years ago, a gene duplication event occurred and one copy became the
present day myoglobin and the other evolved into an oxygen-transport protein that gave rise to
the present day haemoglobin (Fig. 7.2).

Fig. 7.2

(b) State how many years ago the haemoglobin molecule diverged into chains and chains. [1]

.........

(c) Suggest why changes observed in the sequence of amino acids may lead to an under-
estimation of the actual number of mutations. [2]

.........

.........

.........

.........

JC2Prelim2013Biology9648/02 19
915

(d) Explain how variations in the haemoglobin gene can be used as a molecular clock. [3] For
Examiners
Use
.........

.........

.........

.........

.........

.........

[Total: 10]

End of Section A (Part II)

JC2Prelim2013Biology9648/02 20
916

Section B For
Answer ONE question Examiners
Use

Write your answers on the separate answer paper provided.


Your answers should be illustrated by large, clearly labeled diagrams, where appropriate.
Your answers must be in continuous prose, where appropriate.
Your answers must be in set out in questions (a), (b), etc., as indicated in the question.

QUESTION 8

(a) Describe the importance of ATP in cellular processes. [8]

(b) Distinguish between a specialized cell and a cancerous cell. [5]

(c) Discuss, with a named example, how mutation to centromeres can result in genetic disorders.
[7]

[Total: 20]

QUESTION 9

(a) Describe and explain how the structures of chloroplasts and mitochondria are similarly
adapted for their functions. [8]

(b) Discuss the factors that contribute to speciation. [7]

(c) Explain the concept of negative feedback in controlling insulin levels in the body. [5]

[Total: 20]

End of Section B

JC2Prelim2013Biology9648/02 21
917

MERIDIAN JUNIOR COLLEGE


JC2 Preliminary Examinations 2013
Higher 2

CANDIDATE
NAME

CIVICS INDEX
GROUP 1 2 S NUMBER

_______________________________________________________________________________

H2 BIOLOGY 9648/02
Paper 2 Core Paper 18 September 2013
2 hours
Additional Materials: Answer papers
_______________________________________________________________________________

READ THESE INSTRUCTIONS FIRST

Do not open this booklet until you are told to do so.


Write your name, civics group and index number on all the work you hand in.
Write in dark blue or black pen on both sides of the paper.
You may use a soft pencil for any diagrams, graphs or rough working.
Do not use staples, paper clips, highlighters, glue or correction fluid/tape.

Section A
Answer all questions.

Section B For examiners Use


Answer any one question. Section A
1 / 10
At the end of the examination,
1. Fasten your answer papers to section B securely together. 2 / 12
2. Hand in the following separately:
Section A (Part I)
3 / 12
Section A (Part II)
Section B
4 / 14
The number of marks is given in brackets [ ] at the end of each
question or part question. 5 /9

6 / 13

ANSWERSCHEME 7
Section B
/ 10

8/9 / 20

Total /100
________________________________________________________________________________
This paper consists of ___ printed pages.
[Turn over]
JC2Prelim2013Biology9648/02 1
918

Section A (Part I)
Answer all the questions in this section.

QUESTION 1
Heavy metal ions are waste products of many industrial processes and at times may leak into the
ocean and cause environmental pollution. The toxicity of heavy metals on organisms stems from
their effect on metabolically-important enzymes, such as lactate dehydrogenase. Lactate
dehydrogenase catalyses the conversion of pyruvate into lactate.

Fig. 1.1 shows the effect of varying concentrations of mercury ions on the rate of reaction
catalysed by lactate dehydrogenase isolated from Oreochromis mossambicus, a species of fish.

Fig. 1.1

JC2Prelim2013Biology9648/02 2
919

(a) Describe and explain the effect of mercury ions on the rate of reaction catalysed by lactate
dehydrogenase. [4]

In the presence of 10g/ml of mercury, the maximum rate (Vmax) of reaction decreased
from 0.048 arbitrary units to 0.033 arbitrary units.

Mercury ions act as a non-competitive inhibitor

Has a different structure from the substrate pyruvate and binds permanently / irreversibly
to another site away from the active site on the enzyme

Forms a strong covalent bond to an amino acid R group (e.g. -SH group) on the enzyme.

This causes a conformational change on the enzyme which alters the shape of the active
site.

Hence the enzyme is rendered non-functional / no longer able to bind to substrate /


decreased rate of formation of enzyme-substrate complex and hence decreasing the rate
of reaction.

Increasing substrate/pyruvate concentration will not allow the maximum rate of reaction to be
achieved.

(b) Explain how mercury ions may affect the survival of the fish when oxygen concentrations are
low. [3]

When oxygen concentrations are low, aerobic respiration cannot produce sufficient ATP.

In addition, the fish cannot carry out anaerobic respiration in the presence of mercury
ions

as lactate dehydrogenase cannot catalyse the conversion of pyruvate to lactate/lactic


acid to regenerate NAD+ in the cytoplasm due to inhibition by mercury ions.

Hence, glycolysis cannot continue to produce ATP via substrate level phosphorylation.

Thus ATP-requiring cellular activities are inhibited, causing the death of the fish.

JC2Prelim2013Biology9648/02 3
920

Human lysozyme is an important enzyme which is part of the human body's defence against
some bacteria. It is found in abundance in tears, saliva and mucous. The main action of
lysozyme is to catalyse the hydrolysis of -(1,4) glycosidic bonds between N-acetylmuramate
(NAM) and N-acetylglucosamine (NAG) sugar residues in the peptidoglycan cell walls of bacteria.
With the weakened cell wall, the bacterial cells are more likely to undergo lysis.

Fig. 1.2 shows the structure of lysozyme.

Fig. 1.2

(c) With reference to Fig. 1.2 and with your knowledge of haemoglobin, state two structural
differences between lysozyme and haemoglobin. [2]

The secondary structure of lysozyme consists of -helices and -pleated sheets, whereas
the secondary structure of haemoglobin consists of only -helices.

Lysozyme is a protein with a tertiary structure as it consists of only one polypeptide chain,
whereas haemoglobin is a protein with a quaternary structure as it consists of four
polypeptide chains.

Lysozyme does not carry a prosthetic group, whereas each polypeptide chain in
haemoglobin carries a prosthetic (non-proteinaceous) haem group containing Fe2+ which
binds a molecule of oxygen each.

(d) Suggest why some bacteria are more resistant to digestion by lysozymes. [1]

These bacteria cells have a slime layer that makes the -(1,4) glycosidic bonds less
accessible to the lysozymes.

These bacteria cells (gram negative bacteria) have a lower proportion of peptidoglycan in
their cell walls.

These bacteria (gram negative bacteria) have an additional outer membrane that makes the
-(1,4) glycosidic bonds less accessible to the lysozymes.

AVP (e.g. different set of monomers/bond making up the cell wall)

[Total: 10]

JC2Prelim2013Biology9648/02 4
921

QUESTION 2
Fig. 2.1 illustrates the Central Dogma, which describes the flow of genetic information from DNA
to RNA to proteins.

Fig. 2.1

(a) Explain, with an example, how the Central Dogma may not always hold true. [2]

There can be reverse flow of genetic information from RNA to DNA


E.g. in HIV virus, reverse transcriptase converts viral RNA to DNA / reverse transcription
occurs in HIV

RNA instead of DNA can store genetic information


E.g. the influenza virus has several segments of linear () single stranded RNA genome.

RNA can undergo replication.


E.g. influenza virus contains RNA-dependent RNA polymerase synthesizes (+)RNA
using ()RNA as the template via complementary base pairing.

JC2Prelim2013Biology9648/02 5
922

(b) Describe how the information carried on mRNA allows the synthesis of a complete
polypeptide chain. [3]

5 UTR allows recognition by small ribosomal subunit / translation factors

Start codon signals the start of translation

Three bases on the mRNA correspond to a codon

Each codon on mRNA codes for a specific amino acid

The stop codon (UAG, UAA, UGA) terminates translation by allowing the binding of a
release factor that adds a water molecule to the polypeptide chain

JC2Prelim2013Biology9648/02 6
923

A ribonucleoprotein is a protein that is associated with RNA. The ribosome is an example of a


ribonucleoprotein. Prokaryotes have 70S ribosomes while eukaryotes have 80S ribosomes, each
consisting of a small and large subunit. In prokaryotes, the small subunit is made up of a 16S
rRNA associated with 21 proteins. Fig. 2.2 shows the structure of a 16S rRNA.

Fig. 2.2

JC2Prelim2013Biology9648/02 7
924

(c) 16S rRNA and tRNA are both made up of four types of ribonucleotides linked by
phosphodiester bonds.

State two other similarities between the structures of the 16S rRNA and tRNA. [2]

Both rRNA and tRNA are single stranded molecules.

Both have double-stranded regions / can form loops

Both have intra-molecular hydrogen bonds between complementary bases / between A&U
and C&G.

(d) Describe how eukaryotic ribosomes are formed. [4]

rRNA is produced through transcription of the rRNA genes in the nucleolus

Genes of ribosomal proteins are transcribed in nucleus to form mRNA

which is translated in cytoplasm to form ribosomal proteins

Ribosomal protein is transported from the cytoplasm into nucleus via nuclear pore

Partial assembly of rRNA and ribosomal proteins into large and small subunits occurs in
nucleolus

The large and small subunits are transported to the cytoplasm

Unlike eukaryotes, prokaryotes do not have endoplasmic reticulum or vesicles.

(e) Suggest how prokaryotes are still able to secrete proteins. [1]

Prokaryotes have protein channels in the cell surface membrane that allows secretion of
proteins

[Total: 12]

JC2Prelim2013Biology9648/02 8
925

QUESTION 3
The Human Immunodeficiency Virus (HIV) weakens the immune system, causing the patient to
succumb to secondary infections, leading to the development of AIDS. The first recognized case
of AIDS was reported in 1981 in Los Angeles, U.S.A.

Fig. 3.1 shows the binding of a HIV particle to a T cell. The viral receptor gp120 trimer binds to
both CD4 and a co-receptor called CCR5 on the T cell surface.

Fig. 3.1

(a) Explain why the composition of the viral envelope is similar to the composition of the T cell
membrane. [1]

Viral membrane is derived from the host cell when the virus exits the host cell via
budding

JC2Prelim2013Biology9648/02 9
926

(b) Individuals who carry a mutation known as CCR532 in both copies of the CCR5 gene are
resistant to certain strains of HIV. The CCR532 allele is a result of a deletion of a particular
sequence of 32 base-pairs within the exon of CCR5 gene.

Explain how an individual who is homozygous for the CCR532 allele is resistant to HIV. [4]

Loss-of-function mutation

The deletion causes a conformational change / mis-folding in the CCR5 co-receptor /


gp120 binding domain of CCR5

The shape of the mutant CCR5 co-receptor no longer complementary to the shape of
gp120 hence cannot bind

Individual is homozygous hence no normal CCR5 co-receptor is produced at all.

Entry of HIV is inhibited

Loss-of-function mutation

The deletion causes mis-folding of the CCR5 co-receptor, leading to its degradation.

Absence of CCR5 co-receptor on the cell surface

Individual is homozygous hence no normal CCR5 co-receptor is produced at all.

Entry of HIV is inhibited

(c) It was observed that this strain of HIV that uses CCR5 as the co-receptor quickly evolves into
another strain which bind to another co-receptor known as CXCR4.

Suggest and explain how HIV can evolve to use CXCR4 as the co-receptor. [2]

HIV reverse transcriptase is error-prone / has no proofreading function

Results in mutation to gp120 gene

The conformation of the mutated gp120 protein is complementary to and binds CCXR4

(d) The allele frequency of CCR5-32 is unusually high in the western European countries, with
up to 20% of ethnic western Europeans carrying this allele, which is rare or absent in all other
ethnic groups throughout the world.

Suggest a reason for this phenomenon. [1]

Founders effect a small group of individuals from a larger population, some of whom carry
the CCR5-32 allele, colonized/established a new population in the western European
countries.

JC2Prelim2013Biology9648/02 10
927

(e) A technique called RNA interference has been shown to slow down the replication of HIV in
vitro. Double-stranded RNA molecules, 21 to 23 nucleotides long, were added to a culture of
T cells infected with HIV. The sequence of this small interfering RNA (siRNA) matches part of
the HIV protease gene. Once inside the T cell, the two strands of the siRNA separate into
single strands. One of the strands is identical in sequence to the mRNA of the HIV protease.

Explain how this siRNA is able to interfere with HIV replication. [4]

The other strand of the siRNA is complementary to and binds to the mRNA of HIV protease

Interferes with translation of the HIV protease mRNA into the enzyme protease, hence new
HIV protease is not produced.

HIV protease not packaged into new HIV viruses

During subsequent infection, HIV polyprotein is not cleaved into the functional HIV
polypeptides

Interferes with assembly of HIV particles.

One strand of the siRNA is complementary to and binds to the (+) RNA of HIV protease.

Interferes with the reverse transcription of (+)RNA to form single-stranded DNA

No double-stranded DNA is formed and hence no integration of dsDNA of HIV into host
genome

No new (+)RNA genome / HIV mRNA will be formed, hence interfering with assembly.

[Total: 12]

JC2Prelim2013Biology9648/02 11
928

QUESTION 4
Gene expression in eukaryotic cells can be controlled in many ways, one of which is the
regulation of mRNA production. Fig. 4.1 shows how transcription can be controlled.

Fig. 4.1

(a) Using the information in Fig. 4.1 and your knowledge of the control of eukaryotic gene
expression, explain how transcription is initiated. [4]

Histone acetyltransferase neutralizes positive charge on lysine residues on the histone


tail by adding acetyl groups

Loosen the attraction between the negatively-charged DNA and octameric histone

Remodeling protein dissociates/disaggregates/ octameric histone from the DNA

Allows the assembly of transcription complex on the promoter / binding of transcription


factors and RNA polymerase to the promoter to initiate transcription

JC2Prelim2013Biology9648/02 12
929

Ubiquitin-dependent protein degradation in the proteasome is one way by which the amount of
cytosolic proteins in a cell can be controlled at the post-translational level. An amino acid
sequence of proline, glutamic acid, serine and threonine (PEST) in a protein are often recognized
by enzymes and tagged with ubiquitin proteins. Globular proteins in their correct conformation
are not tagged for degradation.

(b) Using the above information, suggest why denatured cytosolic proteins can be tagged with
ubiquitin. [1]

Denaturation exposes the hidden PEST sequence within the hydrophobic core of the protein
to be tagged with ubiquitin by enzymes.

(c) Transmembrane proteins are usually not degraded by proteasome.

Suggest how transmembrane proteins on the cell surface are degraded. [2]

Endocytosis of membrane proteins resulting in formation of vesicles/endosome

Fusion of vesicles with lysosomes degrade the membrane proteins

JC2Prelim2013Biology9648/02 13
930

Ovalbumin
O g
gene from chicken wa as isolated and made single-stra anded, whicch was then mixed
to
ogether witth its maturre mRNA. The
T results were obse erved underr an electroon microsco
ope. Fig.
4.2
4 shows the electron n micrograph and its co orrespondin
ng diagrammatic repreesentation, showing
s
th
he binding oof the mRNA to certain
n regions off the DNA.

F
Fig. 4.2

(d) Explain w
why loops (A
( G) were observed . [3]

The singgle-strandedd DNA is the e template strand from m which the


e mRNA is ttranscribed.
The loop ps are the introns on the DNA th hat are not part
p of the mature
m mRN NA sequenc ce.
This is b
because in mature
m (cytoosolic) mRNNA, the intrrons are exxcised and exons are spliced
togetherr during posst-transcriptional modifiication.
The mRN NA can only bind to / base pair with the ex xons on thee template D
DNA.

e) Explain w
(e why the reg
gion of the mRNA, ind
dicated by th
he arrow in Fig. 4.2, reemains unb
bound to
DNA. [2]

It is the 3 end of the mRNA where the poly(A) ta


ail was add
ded during post-transc
criptional
modificaation.

There is no corresp
ponding stre
etch of thymmine on the template DNA
D where the poly(A)) tail can
form com
mplementarry base pairr with / not ccomplementary to the DNA
D templaate

JC 02
C2Prelim2013Biology9648/0 14
931

Repeating nucleotide sequences are common in the genome of eukaryotes. These repeating
sequences have been commonly referred to as junk DNA.

(f) Suggest why the term junk DNA is misleading. [2]

Junk implies no function / some repeating sequence do have a function


Centromere site of kinetochore assembly for attachment of microtubule / hold two sister
chromatids together
Telomere ref. any one function of telomere
Regions to break and rejoin during crossing over / regions for chiasmata formation
Buffer against mutation on coding regions
AVP

[Total: 14]

JC2Prelim2013Biology9648/02 15
932

QUESTION 5
Hereditary hypophosphatemic rickets is a genetic disorder that results in low level of phosphate
in the blood (hypophosphatemia).

Fig. 5.1 shows the inheritance of this disease over four generations in an extended family.

Fig. 5.1

(a) Explain the mode of inheritance of this disease. [2]

X-linked dominant / sex-linked dominant inheritance

An affected male pasess the disease allele on the X chromosome to all his daughters but
not his son + citing any one example
o I-2 passed the dominant allele to daughters II-1, II-4, II-8 but not to the sons
o III-7 passed the dominant allele to daughters IV-5 and IV-6 but not to the sons
o III-3 passed the dominant allele to daughters IV-3 but not to the sons
o III-11 passed the dominant allele to daughters IV-9 and IV-10 but not to the sons

(b) Using the symbols D/d to represent the alleles, construct a genetic diagram to show how
individual III-7 and III-8 can pass on the disease to their children. [4]

Parental phenotype Affected male Unaffected female


D d d
Parental genotype X Y X X [1]
D d d
Gametes X Y X X [1]

D d D d d d
F1 genotype X X X X X Y X Y

Affected Affected Unaffected Unaffected [1]


F1 phenotype
female female male male

F1 phenotypic ratio 1 affected female : 1 unaffected male [1]

JC2Prelim2013Biology9648/02 16
933

Fur colour of Labrador dogs is controlled by two genes, B/b and E/e. The genes are on different
pairs of chromosomes.

Allele B produces black fur while allele b produces brown fur. However, the colour is expressed
only in the presence of allele E. In the absence of allele E, the dogs are beige in colour, hence
the name Golden Labrador.

Crossing two heterozygotes typically gives the following phenotypic ratio:

9 black : 3 brown : 4 golden

(c) Explain how the phenotypic ratio will differ if genes B/b and E/e are located on the same
homologous pair of chromosomes, as shown in Fig. 5.2. [3]



Fig. 5.2

3 black : 1 golden
Tightly linked, hence no crossing over / chiasma not formed between gene B/b and E/e.
Inherited as one unit
No recombinant gametes

No fixed ratio
Large number of black and golden, small number of brown dogs
Closely linked, hence crossing over between gene B/b and E/e occurs at low frequency.
Small number of recombinant gametes

[Total: 9]

JC2Prelim2013Biology9648/02 17
934

Candidate Name Civics Group Index Number




Meridian Junior College Question 6 Question 7
JC2 Preliminary Examinations 2013
H2 Biology (9648/02) / 13 / 10

Section A (Part II)
Answer all the questions in this section.

QUESTION 6
(a) Describe the role of the following ions in impulse transmission along a nerve cell.
+
(i) Na [2]

Influx of/ movement of Na+ ions into the axoplasm via facilitated diffusion through
voltage-gated Na+ channels and causes depolarization

Trigger action potential and brings membrane potential to +40 mv

+
(ii) K [2]

Efflux/movement of K+ ions out of axoplasm and causes repolarization

Restores the resting potential to -70 mV

2+
(b) Describe the role of Ca in the passage of impulses across a synapse. [3]

Ca2+ influx into presynaptic neuron via facilitated diffusion through voltage-gated Ca2+
channels

The increase in Ca2+ causes vesicles that contain neurotransmitters to move to and fuse
with presynaptic membrane

causes the neurotransmitter to be released into the synaptic cleft by exocytosis

2+
(c) Explain why the concentration of Ca in the cytosol of the synaptic knob remains low despite
frequent opening of channels. [2]

There are carrier protein / protein pump on the pre-synaptic membrane

that actively transport / pump Ca2+ out of the cytosol of synaptic knob at the expense of
ATP.

JC2Prelim2013Biology9648/02 18
935

A water-soluble toxin, muscarine, derived from the mushroom, causes deceleration of the heart
rate, resulting in death of individuals. Fig. 6.1 shows the signalling pathway of muscarine on
postsynaptic membrane.

Fig. 6.1

(d) Describe how muscarine results in inhibition of nerve transmission in heart muscle cells. [4]

1. Muscarine binds to acetylcholine muscarinic receptor to cause the activation of G-


protein by exchanging GDP for GTP

2. The activated G protein activates phospholipase C that convert PIP2 to second


messengers, IP3 and DAG.

3. IP3 binds to the IP3/ligand-gated Ca2+ channel and cause the opening of Ca2+ channels
on the ER, resulting in the release of Ca2+ ions into the cytosol.

4. This increase the membrane potential causing the opening of voltage-gated K+ channel,
and hence the efflux of K+ ions into the synaptic cleft

5. This results in hyperpolarisation / inhibitory postsynaptic potential (IPSP) in heart muscle


cells.

[Total: 13]

JC2Prelim2013Biology9648/02 19
936

QUESTION 7
Fig. 7.1 shows the distribution of malaria and the allele frequency of the sickle cell allele.

Fig. 7.1

(a) Describe and explain the correlation between the occurrence of malaria and the frequency of
the sickle cell allele. [4]

Frequency of sickle cell allele is higher in malaria affected areas

Heterozygotes do not exhibit sickle cell anaemia / phenotypically normal

Heterozygous / sickle cell traits individuals are at selective advantage in malaria affected
areas / more resistant to malaria

Heterozygous individuals survive to reproduce and pass on the sickle cell allele to viable
and fertile offspring

hence increasing the HbS allele frequencies in the population within these malaria
areas over time.

JC2Prelim2013Biology9648/02 20
937

The evolution of haemoglobin molecules has been studied extensively by comparing the amino
acid sequences in both myoglobin and haemoglobin. Myoglobin is used for oxygen storage while
haemoglobin is used for oxygen transport. Ancient prehistoric animals had a single chain of
simple globin for oxygen storage and transport.

About 500 million years ago, a gene duplication event occurred and one copy became the
present day myoglobin and the other evolved into an oxygen-transport protein that gave rise to
the present day haemoglobin (Fig. 7.2).

Fig. 7.2

(b) State how many years ago the haemoglobin molecule diverged into chains and chains. [1]

400 million years ago

(c) Suggest why changes observed in the sequence of amino acids may lead to an under-
estimation of the actual number of mutations. [2]

A mutation may result in the same amino acid being produced due to genetic code being
degenerate

Some mutations may have occurred in the non-coding DNA region / introns

(d) Explain how variations in the haemoglobin gene can be used as a molecular clock. [3]

The mutation in the gene coding for haemoglobin occurred at a constant rate through time

The number of nucleotide differences can be used as a gauge to estimate when two species
diverged from a common ancestor / OWTTE

The lesser the number of nucleotide difference, the more closely related the two species
are [A: reverse argument]

[Total: 10]

JC2Prelim2013Biology9648/02 21
938

Section B
Answer ONE question

Write your answers on the separate answer paper provided.


Your answers should be illustrated by large, clearly labeled diagrams, where appropriate.
Your answers must be in continuous prose, where appropriate.
Your answers must be in set out in questions (a), (b), etc., as indicated in the question.

QUESTION 8

(a) Describe the importance of ATP in cellular processes. [8]

1. Hydrolysis of ATP releases energy

2. Movement of chromosomes to the metaphase plate during nuclear division.

3. Movement of transport vesicles from the rER to Golgi apparatus / from the Golgi apparatus
to the cell surface membrane

4. Proton pumps on the lysosomal membrane, which pumps H+ from the cytosol into the
lysosome, to maintain its acidic pH

5. Sodium and potassium ion pump on the cell surface membrane of axons maintains the
resting potential of neurons. Energy of one ATP molecule is used to actively pump three
sodium ions out to the tissue fluid and two potassium ions in to the axoplasm.

6. Calcium pumps for the active transport of calcium ions out of synaptic knob. This maintains
a low concentration of calcium ions in the synaptic knob, thus regulating synaptic
transmission.

7. Active reabsorption of choline back to the synaptic knob for the reformation of
acetylcholine neurotransmitter. This allows the cell to save cellular resources.

8. In endocytosis / phagocytosis / pinocytosis / receptor-mediated endocytosis, a cell


brings in macromolecules/particulate matter by forming new vesicles from the plasma
membrane by forming pseudopodia / invagination of membrane

9. In light-independent reactions/Calvin cycle of photosynthesis, ATP is required for the


reduction of glycerate-3-phosphate (GP) to glyceraldehydes-3-phosphate (GALP),

10. and for the regeneration of ribulose bisphosphate (RuBP).

11. In the initial stages of glycolysis in respiration, ATP is required to donate phosphate groups
during the phosphorylation of glucose to glucose-6-phosphate

12. fructose-6-phosphate to fructose-1,6-bisphosphate.

13. Form aminoacyl-AMP-enzyme complex, catalysed by the enzyme aminoacyl-tRNA


synthetase. The complex then attaches to a specific tRNA to form aminoacyl-tRNA
complex/activated amino acid.

14. In cell signalling, ATP donates a phosphate group to a target protein, catalysed by protein
kinases.

15. This causes the target protein to undergo a conformational change and convert from the
inactive to active form, which leads to signal transduction.
JC2Prelim2013Biology9648/02 22
939

16. ATP is also used as a substrate for the formation of cAMP by adenylyl cyclase. cAMP
acts as second messenger that leads to signal transduction.

JC2Prelim2013Biology9648/02 23
940

(b) Distinguish between a specialized cell and a cancerous cell. [5]

Property Specialized cell Cancerous cells


Differentiated with specialized Remains undifferentiated with
1 Differentiated?
function. no specialized function

Present. Cells do not divide Absent. Cells continue to


2 Contact inhibition further when in contact with divide even when in contact
other cells. with neighbouring cells.

Cells divide for a limited Cells divide infinitely and


3 No. of cell division
number of times (Hayflick limit) uncontrollably.

4 Apoptosis Undergo apoptosis Does not undergo apoptosis

Angiogenesis
Occurs only when new tissue
(growth of blood Occurs even when growth is
5 is needed to repair damaged
vessels towards not necessary.
tissue.
itself)

Oncogenes code for


Proto-oncogenes code for
Proto-oncogene / hyperactive/excessive proteins
6 proteins that stimulate normal
oncogene that cause excessive cell
cell growth and division.
division.

Mutated tumour suppressor


gene code does not code for a
Functional tumour suppressor
protein / code for non-
Tumour gene code for proteins that
7 functional / less-functional
suppressor gene prevent uncontrolled cell
protein, hence unable to
division.
prevent uncontrolled cell
division.

Telomerase gene Telomerase enzyme absent / Telomerase enzyme present /


8
expression gene not expressed gene expressed

9 AVP AVP AVP

JC2Prelim2013Biology9648/02 24
941

(c) Discuss, with a named example, how mutation to centromeres can result in genetic disorders.
[7]

1. Mutation to the centromere changes the shape/3D conformation of the centromeres due to
changes in nucleotide sequence.

2. This prevents the binding of kinetochore protein complex to the centromere

3. as the kinetochore protein binding site is no longer complementary in shape to the


centromeres.

4. This disallows the attachment of spindle fibres / microtubules.

5. Nondisjunction occurs as the separation of homologous chromosomes during anaphase I


does not occur.

6. Nondisjunction occurs as the separation of sister chromatids during anaphase II does not
occur.

7. Nuclear membrane reforms around the abnormal number of chromosomes.

8. This results in the formation of abnormal gametes / (n+1) or (n-1) gametes during meiosis.

9. When fusion of a normal haploid gamete (n) from one parent and an abnormal gamete
with (n+1) or (n-1) chromosomes from another parent occurs, this results in a genetic
disorder.

10. For example, Downs Syndrome is a condition where there is presence of an extra
chromosome 21 / any valid examples

JC2Prelim2013Biology9648/02 25
942

QUESTION 9

(a) Describe and explain how the structures of chloroplasts and mitochondria are similarly
adapted for their functions. [8]

Structural similarity Function

The presence of electron carriers Passage of electrons via the electron carriers
1 on the thylakoid membrane and release energy to create proton gradient for
mitochondrial inner membrane the synthesis of ATP

Pump protons from the stroma to thylakoid


The presence of proton pump on the
space in chloroplast and matrix to inter-
2 thylakoid membrane and mitochondrial inner
membrane in mitochondria to create a proton
membrane
gradient

The inner membrane of mitochondria and


To increase surface area for attachment of
3 the thylakoid membrane is extensively
e.g. electron carriers / photosynthetic pigments
folded

To allow the facilitated diffusion of protons


Both contain ATP synthase on the thylakoid to generate ATP
4
membrane and mitochondrial inner membrane
To allow phosphorylation of ADP to ATP

To contain genes that code for


5 Both contain circular DNA proteins/enzymes involved in photosynthesis
and respiration

For translation of mRNA into


6 Both contains 70S ribosomes proteins/enzymes involved in photosynthesis
and respiration

For compartmentalization and localization of


enzymes for Krebs Cycle and link reaction in
7 Both are double-membrane bound organelles
mitochondria matrix and Calvin Cycle in
chloroplast stroma

JC2Prelim2013Biology9648/02 26
943

(b) Discuss the factors that contribute to speciation. [7]

1. Speciation is the formation of new species / divergence of existing species into new
species

2. Spontaneous mutations give rise to variation within the population

3. which allow favourable characteristics to be selected for and unfavourable


characteristics to be selected against.

4. Speciation is partly due to disruption to gene flow in a population of a particular species.

Reduced gene flow as a result of:

5. Behavioural isolation, where in a population in a particular area, some members of the


population exhibit variation in behaviour and tended to mate preferentially.

6. Geographical isolation, where in a physical barrier separates members of a population


into 2 sub populations, preventing interbreeding.

7. Physiological isolation, various mechanical, physical, and biochemical differences


between members of a population become barriers to interbreeding between populations,
thus maintaining separate, distinct species

8. Any other isolation, e.g. gametic isolation

9. Genetic drift / Founders effect / Bottleneck effect changes the allele frequencies within
the population by chance.

10. Accumulation of mutations / differences over many generations

11. Over many successive generations, each subpopulation became 2 genetically distinct
species which were unable to interbreed and produce fertile viable offspring.

JC2Prelim2013Biology9648/02 27
944

(c) Explain the concept of negative feedback in controlling insulin levels in the body. [5]

1. Negative feedback is a mechanism where the set point of a parameter is restored by


mechanisms that eliminate the disturbance / by moving the parameter in the direction
opposite of the disturbance.

2. and cells of the islet of Langerhans in the pancreas detect an increase in blood
glucose level above set point of 90mg/100ml.

3. cells of the islet of Langerhans secrete insulin and cells of the islet of Langerhans stop
secretion of glucagon.

4. This decreases the blood glucose level back to the set point.

5. cells of the islet of Langerhans stop secretion of insulin

6. and circulating insulin will be removed from the blood / destroyed in the liver.

7. The level of insulin is reduced.

[Total: 20]

JC2Prelim2013Biology9648/02 28
945

MERIDIAN JUNIOR COLLEGE


JC2 Preliminary Examinations 2013
Higher 2

CANDIDATE
NAME

CIVICS INDEX
GROUP 1 2 S NUMBER

_______________________________________________________________________________

H2 BIOLOGY 9648/03
Applications Paper and Planning Question 23 September 2013
Paper 3 2 hours
Additional Materials: Answer papers
_______________________________________________________________________________

READ THESE INSTRUCTIONS FIRST

Do not open this booklet until you are told to do so.


Write your name, civics group and index number on all the work you hand in.
Write in dark blue or black pen on both sides of the paper.
You may use a soft pencil for any diagrams, graphs or rough workings.
Do not use staples, paper clips, highlighters, glue or correction fluid/tape.

Answer all questions.

At the end of the examination,


1. Fasten all your work securely together.
2. Hand in the following separately:
Structured questions 1 3
Planning question
Essay question

The number of marks is given in brackets [ ] at the end of each


question or part question.
For examiners Use

1 / 15

2 / 16

3 /9

4 / 12
Planning
5 / 20
Essay

Total / 72
________________________________________________________________________________
This paper consists of 11 printed pages.
[Turn over]
JC2Prelim2013Biology9648/03 1
946

Answer all questions. For


Examiners
Use
QUESTION 1
pCMV6-XL5 is a plasmid with a multiple cloning site (MCS) that lies downstream of the CMV
promoter (Fig. 1.1). This plasmid can be inserted into both eukaryotic and prokaryotic host cells.
The arrows denote the direction in which the genes are transcribed.

Fig. 1.1

(a) State the function of the CMV promoter in pCMV6-XL5. [1]

..

..

JC2Prelim2013Biology9648/03 2
947

An artificially-synthesised human growth hormone (hGH) gene with flanking HindIII restriction site For
sequences was created. The restriction sites for the restriction enzymes HindIII and SacI are Examiners
Use
shown in Fig. 1.2.

Fig. 1.2

(b) Using information in Fig. 1.1 and 1.2, explain how the hGH gene can be inserted into
pCMV6-XL5. [3]

..

..

..

..

..

..

(c) State the conditions required for artificial transformation of E. coli cells with the plasmid vector.
[2]
..

..

..

JC2Prelim2013Biology9648/03 3
948

(d) Describe the steps required to select bacterial colonies containing the human growth For
hormone gene after artificial transformation has taken place. [5] Examiners
Use

..

..

..

..

..

..

..

..

..

..

(e) Suggest an explanation for the inclusion of the polyA signal sequence in pCMV6-XL5. [2]

..

..

..

..

(f) Plasmid replication is usually initiated by RNAII, which is a sequence of ribonucleotides that
binds to plasmid DNA prior to replication.

State and explain the role of RNAII in plasmid replication. [2]

..

..

..

..

[Total: 15]

JC2Prelim2013Biology9648/03 4
949

QUESTION 2 For
The location of the gene locus responsible for cystic fibrosis was not discovered until 1985. Examiners
Use
Scientists used restriction fragment length polymorphism to discover the genetic markers
associated with the disease. One such marker was a 950bp-long region known as KM19 that had
been sequenced prior to 1985.

Samples of DNA were obtained from a family known to have the condition. The KM19 locus was
amplified by PCR and mixed with PstI and EcoRI in two separate restriction digests. The results
of gel electrophoresis of both restriction digests are shown in Fig. 2.1.

Fig. 2.1

(a) Using the information in Fig. 2.1,

(i) state and explain which restriction enzyme digest should be used to detect the KM19
genetic marker associated with cystic fibrosis. [3]

...

...

...

...

...

...

(ii) draw a restriction map of the KM19 genetic marker that is associated with cystic fibrosis in
the space below. [1]

JC2Prelim2013Biology9648/03 5
950

(b) Explain why genetic markers like KM19 can be used to detect the presence of disease- For
causing alleles. [2] Examiners
Use

..

..

..

..

(c) Further research in the 1980s identified sequences of genetic markers located both upstream
and downstream of the gene suspected to cause cystic fibrosis.

Using this information, suggest how the gene could be isolated using these two genetic
markers. [2]

..

..

..

..

Clinical trials for the treatment of inherited conditions via gene therapy have been conducted in
the past two decades, but an unsuccessful and lethal trial in 1999 revealed the risks involved in
using adenoviruses in gene therapy. Adeno-associated viruses were then used as alternative
viral vectors for gene therapy trials as they exhibit little or no disease-causing side effects. More
recently, non-viral gene therapies have been explored as safer alternatives to viral vectors.

(d) Describe two other advantages of utilising adeno-associated viruses as compared to


adenoviruses. [2]

1. ...

...

2. ...

...

(e) State two criteria that make cystic fibrosis a condition that can be potentially treated with gene
therapy. [2]

1. ...

2. ...

JC2Prelim2013Biology9648/03 6
951

Immune responses to non-viral gene therapy have been documented. The main cause of the For
immune response was found to be the plasmids carrying the normal allele. Examiners
Use

(f) Suggest a reason why plasmids may elicit an immune response during non-viral gene
therapy. [1]

..

..

(g) State three other differences between non-viral gene therapy and viral gene therapy. [3]

1. ...

...

2. ...

...

3. ...

...

[Total: 16]

JC2Prelim2013Biology9648/03 7
952

QUESTION 3 For
Examiners
Embryonic stem (ES) and induced pluripotent stem (iPS) cells hold great promise for the Use
treatment of a wide variety of acquired or hereditary diseases. One of the major obstacles to
clinical applications is directing them to differentiate into functional cells.

The effect of various growth factors on stem cell differentiation in ectodermic, mesodermic and
endodermic embryonic stem cells was carried out and the process was documented in Fig. 3.1.
Embryonic stem cells were cultured and embryoid bodies allowed to differentiate into ectodermic,
mesodermic and endodermic embryonic stem cells. Growth factors were added to the stem cells
and the expression of cell-specific genes was measured after ten days.

Fig. 3.1

One of the growth factors tested was the epidermal growth factor. 100ng/mL of epidermal growth
factor was given to the three different embryonic stem cells. The results are summarised in
Table 3.1.

Expression of cell-specific genes after ten days


Type of stem cell
Keratin -Globin Insulin
Embryonic Ectoderm +
Embryonic Mesoderm +
Embryonic Endoderm
+ genes are expressed
genes are not expressed

Table 3.1

(a) Using the information given and the data from Table 3.1, describe and explain the effect of
epidermal growth factor on the expression of keratin in stem cells. [3]

..

..

..

..

..

..

JC2Prelim2013Biology9648/03 8
953

(b) Suggest why the insulin gene was not expressed in the three different kinds of embryonic For
stem cells even after treatment with a variety of other growth factors. [1] Examiners
Use

..

..

Researchers have recently used induced pluripotent stem (iPS) cells generated from human skin
cells to create precursor heart cells.

(c) State two advantages of using iPS cells instead of embryonic stem cells for research and
clinical trials. [2]

1. ...

...

2. ...

...

Cord blood stem cell therapy has been used in clinical trials for the treatment of autoimmune
diseases like leukaemia and inherited conditions like Severe Combined Immuno-deficiency
(SCID).

The two main modes of treatment are:

o Allogeneic The patient receives stem cells from a matching donor, either a sibling or
an unrelated donor

o Autologous The patient receives their own stem cells

(d) Describe the properties of cord blood stem cells. [2]

..

..

..

..

(e) Suggest a potential limitation of autologous cord blood stem cell therapy. [1]

..

..

[Total: 9]

JC2Prelim2013Biology9648/03 9
954

Question
Q 4 Planning Question
Q For
You
Y are req quired to pla
an, but not carry out, an investig
gation into the
t effect oof temperatu ure from Examiners
E
Use
10C to 70C C on the peermeability of potato ce
ell membranes. Potato oes are richh in potassiu
um ions.
When
W small disc-shapeed potatoes cut from a core borer (Fig. 4.1) are
a placed iin water, po otassium
io
ons are rele
eased from the cell into
o the water.

T concentration of potassium io
The ons in the w
water can be
e measured
d using an eelectrode (F
Fig. 4.2)
th
hat is selecctive for pota
assium ions
s.


F 4.1
Fig. Fig. 4.2
2

Your
Y plan sh
hould:

Have a clear and helpful


h struc
cture such that the me ethod you use
u is able to be repe eated by
anyone rreading it.
Be illusttrated by re elevant diag
gram(s) to show, for example, the arrangeement of ap pparatus
used.
Include tthe layout of
o result tables and gra phs with cle ear headinggs and labells.
Include ffull details and
a explana ations of the e proceduree that you would
w adoptt to ensure that the
results o
obtained we ere as quanttitative, preccise and reliable as po
ossible.

Your
Y plannin
ng must be based on the
t assump ption that yo
ou have been providedd with the following
f
equipment
e a
and apparattus which yo
ou must use
e:

Potatoess
Potassiuum ion-selecctive electro
ode which m
measures in
n mg/L
A core bborer of 10m
mm in diame eter
Scalpel
Ruler
ubes
Boiling tu
Measurin ng cylinderss
Stopwatch
Thermosstatically-co
ontrolled waater bath
Thermom meter
Isotonic buffer soluttion
De-ionizzed water
Other coommon labo oratory appaaratus

[To
otal: 12]

JC 03
C2Prelim2013Biology9648/0 10
955

Free-response question For


Examiners
Use
Write your answer to this question on the separate answer paper provided.

Your answer:

should be illustrated by large, clearly labeled diagrams, where appropriate;


must be in continuous prose, where appropriate;
must be set out in sections (a), (b) etc., as indicated in the question

QUESTION 5

(a) Explain how genetically modified organisms may provide a solution to global food shortage
issues. [8]

(b) Discuss the social and ethical implications of germ-line gene therapy. [5]

(c) Describe the goals of the Human Genome Project. [7]

End of Paper

JC2Prelim2013Biology9648/03 11
956

MERIDIAN JUNIOR COLLEGE


JC2 Preliminary Examinations 2013
Higher 2

CANDIDATE
NAME

CIVICS INDEX
GROUP 1 2 S NUMBER

_______________________________________________________________________________

H2 BIOLOGY 9648/03
Applications Paper and Planning Question 23 September 2013
Paper 3 2 hours
Additional Materials: Answer papers
_______________________________________________________________________________

READ THESE INSTRUCTIONS FIRST

Do not open this booklet until you are told to do so.


Write your name, civics group and index number on all the work you hand in.
Write in dark blue or black pen on both sides of the paper.
You may use a soft pencil for any diagrams, graphs or rough workings.
Do not use staples, paper clips, highlighters, glue or correction fluid/tape.

Answer all questions.

At the end of the examination,


1. Fasten all your work securely together.
2. Hand in the following separately:
Structured questions 1 3
Planning question
Essay question

The number of marks is given in brackets [ ] at the end of each


question or part question.
For examiners Use

1 / 15

2 / 16

ANSWERSCHEME 3 /9

4 / 12
Planning
5 / 20
Essay

Total / 72
________________________________________________________________________________
This paper consists of __ printed pages.
[Turn over]
JC2Prelim2013Biology9648/03 1
957

Answer all questions.

QUESTION 1
pCMV6-XL5 is a plasmid with a multiple cloning site (MCS) that lies downstream of the CMV
promoter (Fig. 1.1). This plasmid can be inserted into both eukaryotic and prokaryotic host cells.
The arrows denote the direction in which the genes are transcribed.

Fig. 1.1

(a) State the function of the CMV promoter in pCMV6-XL5. [1]

It is where RNA polymerase binds to initiate transcription of the human growth hormone
gene inserted into the multiple cloning site (downstream of the promoter).

JC2Prelim2013Biology9648/03 2
958

An artificially-synthesised human growth hormone (hGH) gene with flanking HindIII restriction site
sequences was created. The restriction sites for the restriction enzymes HindIII and SacI are
shown in Fig. 1.2.

Fig. 1.2

(b) Using information in Fig. 1.1 and 1.2, explain how the hGH gene can be inserted into
pCMV6-XL5. [3]

Cleave gene using HindIII to generate sticky ends.

Cleave plasmid with SacI to generate complementary sticky ends to the HindIII sticky
ends flanking the hGH gene.

Mix the cleaved gene and plasmid together and add DNA ligase to seal the nicks / form
phosphodiester bond between gene and plasmid.

(c) State the conditions required for artificial transformation of E. coli cells with the plasmid vector.
[2]
Addition of calcium chloride solution to the mixture of E. coli and plasmid.

Subject mixture to heat shock in a 42C water bath for 2 minutes (or less).

JC2Prelim2013Biology9648/03 3
959

(d) Describe the steps required to select bacterial colonies containing the human growth
hormone gene after artificial transformation has taken place. [5]

1. Spread transformed bacteria onto agar/nutrient medium plates containing ampicillin.

2. Bacterial colonies on the ampicillin plate are transferred onto a nitrocellulose membrane.

3. Add sodium hydroxide to the filter to lyse bacterial cells and denature dsDNA into
ssDNA.

4. A solution of radioactively-labelled gene probe is added so that it will hybridize with the
complementary DNA sequence of the human growth hormone gene.

5. Wash away excess probe

6. The position of the cell containing the DNA-gene probe hybrid/labelled gene can be located
by autoradiography performed on the filter.

7. The position of dark spots on the x-ray film corresponds to the position of the bacterial
colonies containing the human growth hormone gene.

Accept: Fluorescent-labelled gene probes and subsequent corresponding steps.

(e) Suggest an explanation for the inclusion of the polyA signal sequence in pCMV6-XL5. [2]

If the plasmid is inserted into a eukaryotic host cell, the polyA signal sequence will direct
the addition of a poly A tail to mRNA.

Polyadenylation of mRNA extends the half-life of mRNA / increase mRNA stability /


reduces the rate at which mRNA is hydrolysed by endonucleases, resulting in more proteins
synthesis / translation can occur for a longer period of time.

Facilitate the export of mRNA out of the nucleus into the cytoplasm.

(f) Plasmid replication is usually initiated by RNAII, which is a sequence of ribonucleotides that
binds to plasmid DNA prior to replication.

State and explain the role of RNAII in plasmid replication. [2]

RNAII serves as a primer for DNA replication.

Because DNA polymerase requires a 3OH to bind to in order to start/begin replication.

[Total: 15]

JC2Prelim2013Biology9648/03 4
960

QUESTION 2
The location of the gene locus responsible for cystic fibrosis was not discovered until 1985.
Scientists used restriction fragment length polymorphism to discover the genetic markers
associated with the disease. One such marker was a 950bp-long region known as KM19 that had
been sequenced prior to 1985.

Samples of DNA were obtained from a family known to have the condition. The KM19 locus was
amplified by PCR and mixed with PstI and EcoRI in two separate restriction digests. The results
of gel electrophoresis of both restriction digests are shown in Fig. 2.1.

Fig. 2.1

(a) Using the information in Fig. 2.1,

(i) state and explain which restriction enzyme digest should be used to detect the KM19
genetic marker associated with cystic fibrosis. [3]

PstI

Digestion with PstI produced two bands of 300bp and 650bp in affected individuals but
unaffected individuals showed either a single band of 950bp or three bands of 950bp,
650bp and 300bp.

But after digestion with EcoRI, all individuals had single 950bp fragments and the
affected and unaffected individuals are not differentiated.

(ii) draw a restriction map of the KM19 genetic marker that is associated with cystic fibrosis.
[1]

[Size of fragment + labelling of restriction site]

JC2Prelim2013Biology9648/03 5
961

(b) Explain why genetic markers like KM19 can be used to detect the presence of disease-
causing alleles. [2]

These genetic markers are polymorphic

They are tightly linked to the disease-causing alleles on the same chromosome and are
likely to be inherited together as one unit with the disease-causing allele.

(c) Further research in the 1980s identified sequences of genetic markers located both upstream
and downstream of the gene suspected to cause cystic fibrosis.

Using this information, suggest how the gene could be isolated using these two genetic
markers. [2]

Design primers that are complementary to the genetic markers upstream and downstream
of the suspected gene.

Use the polymerase chain reaction (PCR) to synthesise multiple copies of suspected
gene.

Clinical trials for the treatment of inherited conditions via gene therapy have been conducted in
the past two decades, but an unsuccessful and lethal trial in 1999 revealed the risks involved in
using adenoviruses in gene therapy. Adeno-associated viruses were then used as alternative
viral vectors for gene therapy trials as they exhibit little or no disease-causing side effects. More
recently, non-viral gene therapies have been explored as safer alternatives to viral vectors.

(d) Describe two other advantages of utilising adeno-associated viruses as compared to


adenoviruses. [2]

Adeno-associated viruses are capable of infecting both non-dividing and dividing cells.

Adeno-associated viruses are capable of integrating the normal allele into the patients
genome at a specific site, hence there is a lesser chance of oncogene formation.

Treatment is permanent if stem cells are used as the normal allele is integrated into the
cells genome and the stem cells are capable of self-renewal.

(e) State two criteria that make cystic fibrosis a condition that can be potentially treated with gene
therapy. [2]

CF is a homozygous recessive condition hence insertion/introduction of the normal


dominant allele that can produce / code for a functional protein.

CF is a single-gene disorder hence it can be treated by the introduction of the normal allele
by gene therapy.

JC2Prelim2013Biology9648/03 6
962

Immune responses to non-viral gene therapy have been documented. The main cause of the
immune response was found to be the plasmids carrying the normal allele.

(f) Suggest a reason why plasmids may elicit an immune response during non-viral gene
therapy. [1]

Plasmids used for gene therapy may have sequences that are bacterial/prokaryotic in
origin and are recognised by the human immune system.

(g) State three other differences between non-viral gene therapy and viral gene therapy. [3]

Feature Non-viral Viral

Vector(s) Examples of non-viral vectors: Adenoviral vectors, retroviral vectors,


used liposomes, cationic polymers adeno-associated viral vectors.

Less risk of mutations as most


Greater risk of harmful mutations as a
Potential methods do not involve integrating the
result of viruses inserting genes randomly
mutation normal allele into the genome of the
into the patients genome.
patients cells.

Only a limited length of gene can be


Size of insert Larger size of gene can be inserted.
inserted.

AVP AVP AVP

[Total: 16]

JC2Prelim2013Biology9648/03 7
963

QUESTION 3
Embryonic stem (ES) and induced pluripotent stem (iPS) cells hold great promise for the
treatment of a wide variety of acquired or hereditary diseases. One of the major obstacles to
clinical applications is directing them to differentiate into functional cells.

The effect of various growth factors on stem cell differentiation in ectodermic, mesodermic and
endodermic embryonic stem cells was carried out and the process was documented in Fig. 3.1.
Embryonic stem cells were cultured and embryoid bodies allowed to differentiate into ectodermic,
mesodermic and endodermic embryonic stem cells. Growth factors were added to the stem cells
and the expression of cell-specific genes was measured after ten days.

Fig. 3.1

One of the growth factors tested was the epidermal growth factor. 100ng/mL of epidermal growth
factor was given to the three different embryonic stem cells. The results are summarised in
Table 3.1.

Expression of cell-specific genes after ten days


Type of stem cell
Keratin -Globin Insulin
Embryonic Ectoderm +
Embryonic Mesoderm +
Embryonic Endoderm
+ genes are expressed
genes are not expressed

Table 3.1

(a) Using the information given and the data from Table 3.1, describe and explain the effect of
epidermal growth factor on the expression of keratin in stem cells. [3]

Addition of 100ng/mL of epidermal growth factor results in the expression of keratin in


ectodermic stem cells, but not mesodermic or endodermic stem cells after ten days.

Epidermal growth factor receptor was present only on ectodermal stem cells.

Epidermal growth factor initiates the cell-signalling pathway

leading to the transcription and translation of the keratin gene.

JC2Prelim2013Biology9648/03 8
964

(b) Suggest why the insulin gene was not expressed in the three different kinds of embryonic
stem cells even after treatment with a variety of other growth factors. [1]

The insulin gene will only be transcriptionally active in -cells / will not be synthesised until
the pancreas is fully developed.

Researchers have recently used induced pluripotent stem (iPS) cells generated from human skin
cells to create precursor heart cells.

(c) State two advantages of using iPS cells instead of embryonic stem cells for research and
clinical trials. [2]

iPS cells can be generated from skin cells/adult cells, so there are fewer ethical issues
involved as compared to using embryonic stem cells obtained from an embryo.

iPS cells can be generated from skin cells/adult cells of the patient/sufferer, so there might be
less risk of tissue rejection after gene therapy and transplantation.

Cord blood stem cell therapy has been used in clinical trials for the treatment of autoimmune
diseases like leukaemia and inherited conditions like Severe Combined Immuno-deficiency
(SCID).

The two main modes of treatment are:

o Allogeneic The patient receives stem cells from a matching donor, either a sibling or
an unrelated donor

o Autologous The patient receives their own stem cells

(d) Describe the properties of cord blood stem cells. [2]

Multi-potent and able to undergo differentiation to produce different types of blood cells.
Unspecialised and undifferentiated.
Has telomerase activity.
Has ability to undergo long-term self-renewal.

(e) Suggest a potential limitation of autologous cord blood stem cell therapy. [1]

Cord blood stem cell therapy cannot cure inherited genetic diseases if the donor has the
same inherited condition or the treatment is autologous (as the patient receives his own stem
cells).

The amount of cord blood stem cells that can be collected from a single donor is small and
reduces chances of repeated therapy.

[Total: 9]

JC2Prelim2013Biology9648/03 9
965

Question
Q 4 Planning Question
Q
You
Y are req quired to pla
an, but not carry out, an investig
gation into the
t effect oof temperatu ure from
10C to 70C C on the peermeability of potato ce
ell membranes. Potato oes are richh in potassiu
um ions.
When
W small disc-shapeed potatoes cut from a core borer (Fig. 4.1) are
a placed iin water, po otassium
io
ons are rele
eased from the cell into
o the water.

T concentration of potassium io
The ons in the w
water can be
e measured
d using an eelectrode (F
Fig. 4.2)
th
hat is selecctive for pota
assium ions
s.


F 4.1
Fig. Fig. 4.2
2

Your
Y plan sh
hould:

Have a clear and helpful


h struc
cture such that the me ethod you use
u is able to be repe eated by
anyone rreading it.
Be illusttrated by re elevant diag
gram(s) to show, for example, the arrangeement of ap pparatus
used.
Include tthe layout of
o result tables and gra phs with cle ear headinggs and labells.
Include ffull details and
a explana ations of the e proceduree that you would
w adoptt to ensure that the
results o
obtained we ere as quanttitative, preccise and reliable as po
ossible.

Your
Y plannin
ng must be based on the
t assump ption that yo
ou have been providedd with the following
f
equipment
e a
and apparattus which yo
ou must use
e:

Potatoess
Potassiuum ion-selecctive electro
ode which m
measures in
n mg/L
A core bborer of 10m
mm in diame eter
Scalpel
Ruler
ubes
Boiling tu
Measurin ng cylinderss
Stopwatch
Thermosstatically-co
ontrolled waater bath
Thermom meter
Isotonic buffer soluttion
De-ionizzed water
Other coommon labo oratory appaaratus

[To
otal: 12]

JC 03
C2Prelim2013Biology9648/0 10
966

Objective
To investigate the effect of temperature on the permeability of cell membranes using potatoes

Background and hypothesis


Ions are charged
Ion channels to provide hydrophilic channel
Lateral movement of phospholipids
Fluidity of membrane
Increase temperature, more heat energy, phospholipids gain kinetic energy
Membrane more fluid and leaky
+
At high temperatures, membrane disrupted, K fully released into water
Cell wall is fully permeable to ions
+
Higher temperature, greater membrane permeability, more K diffuse out

Theory on membrane [1]

Independent and dependent variables


Independent variable
Temperature / C
10, 20, 30, 40, 50, 60, 70C
Dependent variable
+ -1
Concentration of K / mgL
Correct variables with units [1]
At least 5 temperatures of sensible range and interval [1]

Other variables to be kept constant


Same size and number of potato discs from the same potato
Volume of water in all tubes
Fresh de-ionized water in all tubes
Time for testing
Same electrode
Any two with an indication of how it is kept constant in the methods [1]

Control
For every temperature examined, no potato disc added to ensure the K+ detected is from
the potato cells.

Description of control with rationale [1]

Methods
1. Cut a potato into cylindrical shapes of diameter 10mm using the core borer.
2. Cut the cylindrical potato into disc shape of height 3mm using scalpels.
3. Place the disc-shaped potatoes into an isotonic buffer. This is to prevent the potato from
drying up and also to maintain its osmotic pressure.
4. Add 10ml of fresh de-ionized water into a boiling tube and equilibrate in a 10C water bath
until the de-ionized water is at 10C. Monitor temperature using thermometer.
5. Add 10 discs of potatoes and start the stopwatch.
6. After 15min (accept range), measure the concentration of K+ with the electrode.
7. Repeat step 4 6 twice to obtain a total of 3 readings to minimize error by calculating
average.
8. Repeat step 4 7 for 20, 30, 40, 50, 60 and 70C.
9. Repeat the whole experiment twice to ensure reproducibility.

Obtaining potato discs (point 1 3) [1]


Measurement of readings (point 4 6, 8) [1]
Performing replicates and repeats (point 7, 9) [1]

JC2Prelim2013Biology9648/03 11
967

Diagram of apparatus set-up [1]

Results
+ -1
Concentration of K / mgL
Temperature / C
1 2 3 Average
10
20
30
40
50
60
70

Table with appropriate headings with units [1]

Labeled axes and correct proposed trend [1]


Risks and precautions
Risk Precaution
Scalpel is sharp and may cause injuries Take extra care when handling with the scalpel

70C water bath is hot enough cause


Use cloth/ for insulation
scalding

Risk with corresponding precaution [1]

JC2Prelim2013Biology9648/03 12
968

QUESTION 5

(a) Explain how genetically modified organisms may provide a solution to global food shortage
issues. [8]

1. Increase yield by genetically modifying food crops that will result in decreased losses caused
by pesticides, fungal infestation, viruses, bacterial infection and pests.

Introduction of genes that code for proteins that confer insecticidal resistance

2. Example: Genetically engineered crop plants that express the Bt-toxin gene from the
bacteria Bacillus thuringiensis produce Bt-toxin protein which kills insect larvae feeding
on the plant.

Introduction of genes that code for proteins that confer herbicidal resistance

3. Example: Genetically engineered crop plants that are resistant to herbicides by introducing
the herbicide/glyphosate resistance gene

4. When herbicide is applied to the field, the weeds are eliminated but not the herbicide
resistance crop plants.

Introduction of genes that code for proteins that confer viral resistance

5. E.g., tobacco plant can be made resistant to the tobacco mosaic virus by expressing the
coat protein gene of a virus.

Introduction of genes that code for proteins that confer fungal resistance

6. Example: Genetically engineered crop plants have an increased resistance to fungal


infestations due to the expression of anti-fungal proteins within the plant.

Or

Inclusion of a gene expressing anti-fungal compounds (e.g. phytoalexin) into transgenic crop
plant (e.g. Alfafa, potato) confer fungal resistance.

Introduction of genes that code for growth hormones

7. Example: The Atlantic salmon has been genetically modified by the addition of a growth
hormone gene from a Pacific Chinook salmon and an active promoter from an ocean pout
placed upstream of the growth hormone gene.

8. The insertion of the growth hormone gene results in faster growth rate and yield of the
salmon, thereby increasing the supply of salmon.

Introduction of genes that delayed ripening

9. Flavr Savr tomatoes are engineered to include a gene for antisense mRNA to
polygalacturonase gene to reduce expression of polygalacturonase.

10. This will result in delayed ripening allowing crops to be stored for longer period of time

JC2Prelim2013Biology9648/03 13
969

(b) Discuss the social and ethical implications of germ-line gene therapy. [7]

Limitation in technology
1. The technology for germ-line gene therapy is not perfect and there is a risk of inducing
mutations, which can be passed down to the offspring, when the insertion of gene
happens to introduce further defects.

Research on embryos
2. Those involved in the research related to germ-line gene therapy regularly create and
destroy embryos as a part of their research.
3. There are objections to killing embryos used for research in gene therapy as human life
may be considered to have begun at conception.

Abuse of technology
4. It may be abused by parents who want to eliminate undesirable alleles from their offspring.

Denial of human rights


5. The individuals that results from germ-line gene therapy did not have a say in whether
their genetic material should have been changed.

The fate of human descendants


6. The genetic makeup of human descendants may be changed hence affecting the human
gene pool.
7. These changes may presumably be for the better. However, any errors in technology and
judgement may have far-reaching consequences.

Ethical and social objections of therapy in general

8. The definition of normal and disability may change. The decision of what is normal and
what is a disability should not rest on just a small group of individuals or medical
professionals. The question of whether disabilities are considered disease and whether these
need to be cured or prevented requires thorough consideration.

9. Demeaning the life of the disabled. There is a concern that the presence of a cure might
demean the lives of individuals presently affected by disabilities. There could be a new
definition of what contributes to a normal being. E.g. the exclusion of individuals who are
merely average in intelligence.

10. Creation of social class distinctions. There can be a formation of class distinctions,
because expensive gene therapy is easily available to individuals with financial means, but
not to those financially less well off. There should also be an establishment of procedural
fairness in selection of patients for research.

11. Religious and cultural belief. Changing the genetic makeup of individuals goes against
many religious and cultural beliefs.

12. Other issues. There may be incidental elimination of genes that are important but with
unknown functions and this may lead to decreased species fitness

(c) Discuss the goals of the Human Genome Project. [5]

1. Identify all the genes in human DNA (approximately 20000 -25000).

2. Determine the DNA sequence of the entire human genome (about 3 billion base pairs).

3. Store this information in databases to be shared publicly by researchers throughout the world

JC2Prelim2013Biology9648/03 14
970

4. Improve tools for data analysis.

5. Transfer related technologies to the private sector and award grants for innovative research,
so as to catalyse the biotechnology industry and the development of new medical
applications.

6. Address the ethical, legal and social issues that may arise from the project.

End of Paper

JC2Prelim2013Biology9648/03 15
971

NATIONAL JUNIOR COLLEGE, SINGAPORE


Senior High 2
Preliminary Examination
Higher 2

CANDIDATE
NAME

BIOLOGY REGISTRATION
CLASS 2BI2___ / 2IPBi2___ NUMBER

BIOLOGY 9648/01
Paper 1 Multiple Choice 19 Sep 2013
1 hour 15 minutes
Additional Materials: Multiple Choice Answer Sheet

READ THESE INSTRUCTIONS FIRST

Write in soft pencil.


Do not use staples, paper clips, highlighters, glue or correction fluid.
Write your name, Biology class, and registration number above and on the Answer Sheet
provided.

There are forty questions on this paper. Answer all questions. For each question there are
four possible answers A, B, C and D.
Choose the one you consider correct and record your choice in soft pencil on the separate
Answer Sheet.

Read the instructions on the Answer Sheet very carefully.

Each correct answer will score one mark. A mark will not be deducted for a wrong answer.
Any rough working should be done in this booklet.
Calculators may be used.

This document consists of 19 printed pages.

[Turn over
972

1 The diagram shows an electron micrograph of a human cell.

Which of the following correctly identifies its structure-function adaptation?

function structural adaptation


A enzyme production P and U
B antibody production Q, S and T
C phagocytosis R
D photosynthesis T

2 Most wild plants contain toxins that deter animals from eating them. A scientist
discovered that a toxin produced by a certain plant was also toxic to the same
plant if it as applied to the roots of the plant. As the first step on finding out why
the plant was not normally killed by its own toxin, he fractionated some plant
cells and found that the toxin was in the fraction that contained the largest cell
organelle. He also found that the toxin was no longer toxic after it was heated.

Which of the following statements are consistent with the scientists


observations?

I. The toxin was stored in the central vacuole.


II. The toxin cannot cross the membrane of the organelle in which it is stored.
III. The toxin was stored in chloroplast.
IV. The toxin is likely to be lipid-soluble.
V. The toxin may be an enzyme.

A I, II and V
B I, IV and V
C II, III and IV
D III, IV and V
973

3 The graph shows the effect of pH on the structure of a protein which consists of
entirely of repeating residues of one amino acid.

-helix

Increasing
symmetry

random coil

0 2 4 6 8 10 12 14
pH

Which statement is true?

A At high acidity the protein loses its secondary structure.


B At high acidity the protein loses its tertiary structure.
C At high acidity the protein loses its tertiary structure.
D At low acidity the protein loses its secondary structure.
974

4 The following graphs show the activities of different enzymes (1-5) under different
conditions:

Rate of reaction

0 20 40 60 80 100
(a)
Temperature
Rate of reaction

(b) pH

Which statement is a correct explanation for the rate of reaction of different


enzymes?

A Rate of reaction of enzyme 3 and its substrate is fastest at 58C.


B At pH 2, most of the R groups at the active site of enzyme 4 are all
negatively charged.
C At pH 8.1, substrate is bonded to the active site of enzyme 5 by hydrogen
bonds only.
D At 60C, several hydrogen bonds between R groups of enzyme 2 are
broken.
975

5
5 The figure below shows some biomolecules numbered 1 to 10.

1 2 3 4 5

6 7

10

Which of the following statements concerning the biomolecules is/are false?

I. Triacylglycerol may be made up of one molecule of 6 and three molecules of 7.


II. Molecules 5 and 10 may be joined together by a glycosidic linkage to form a disaccharide.
III. Molecules 1 and 3 are saturated fatty acids while molecule 4 has an unsaturated chain.
IV. All of the molecules are monomers of biological macromolecules.

A III only
B I and II only
C I, II and IV only
D All of the above
976

6
6 The following events occur during the mitotic cell cycle.

I division of centromeres
II proofreading mechanism in replication
III duplication of chromosomes
IV overlapping of non-kinetochore spindle fibres

Which event(s) in mitosis directly ensure that the daughter cells are genetically identical to
each other and to the parent cell?

A I only
B I and III
C II and III
D II and IV

7 The diagram above shows a bivalent. Which of the following correctly represents the final
products at the end of Meiosis II?
977

8 The mechanism of action of four drugs that inhibit DNA replication is stated below.

Aphidicholine inhibits DNA polymerase


Cytarabine is converted into a molecule that can substitute for a DNA nucleotide
and also inhibits DNA repair mechanisms
Epirubicin inhibits an enzyme involved in the unwinding of DNA and separation of
strands
Hydroxycarbamide inhibits an enzyme involved in the production of
deoxyribonucleotides

Which row correctly matches a drug to an explanation of the mechanism of action?

9 A polypeptide molecule contains the amino acid sequence, glycine leucine


lysine valine. The table shows the DNA codes for these amino acids.

Which anticodons present on tRNA are needed for the synthesis of this
polypeptide?

A CCC GAA TTT CAA


B CCC GAA UUU CAA
C GGG CUU AAA GUU
D GGG CUU UUU GUU
978

10 The diagram shows the results of two types of mutation.

CGATTACCATGCATA
original base sequence

CGATTAACCTGCATA CGATTACCAGGCATA
mutant sequence 1 mutant sequence 2

What types of gene mutation produce mutant sequences 1 and 2?


1 2

A deletion insertion
B insertion deletion
C inversion substitution
D substitution insertion

11 Which of the following mutations will bring about active transcription of the lac operon in E. coli?

A A mutation in the promoter that decreases the affinity of the repressor.


B A mutation in the repressor gene that strengthens the affinity of the repressor
for the inducer.
C A mutation in the repressor gene that weakens the affinity of the repressor for
the operator.
D All of the above

12 E.coli bacteria are grown in a culture of nutrients, which includes glucose and lactose as the
main source of carbon-based nutrient.

R
Q

Which of the following statements account for the growth curve showing the reproduction of
E.coli bacteria?

P Q R

A Lactose is Production of enzymes for lactose Glucose is


metabolised metabolism is repressed metabolised

B Lactose is Enzymes for lactose metabolism Glucose is


metabolised is produced metabolized

C Glucose is Production of enzymes for lactose Lactose is


metabolised metabolism is repressed metabolised

D Glucose is Enzymes for lactose metabolism Lactose is


metabolised is produced metabolised
979

13 Which of the following statement about chromosome structure is true?

I Linker DNA refers to the linear double-stranded DNA between adjacent


nucleosomes.
II The 30 nm chromatin fibre has a solenoid structure which forms due to attraction
between histone tails and linker DNA.
III The 300 nm chromatin fibre is attached to multiple locations on a central protein
scaffold made of histone protein H1.
IV Each nucleosome is made of linear DNA wrapping around four histone proteins.

A I and II
B I and IV
C II and III
D III and IV

14 Four different genes are regulated in different ways.

Gene 1 undergoes tissue-specific patterns of alternative splicing


Gene 2 is part of a group of structural genes controlled by the same regulatory sequences
Gene 3 is in some circumstances subject to methylation
Gene 4 codes for a repressor protein which acts at an operator site close by

Which combination correctly identifies the gene regulatory steps involving prokaryotes and
eukaryotes?

prokaryotic eukaryotic
A 1 and 2 3 and 4
B 1 and 3 2 and 4
C 2 and 3 1 and 4
D 2 and 4 1 and 3

15 Which choice presents items in the proper order (earliest to latest) in which they would be
used in the process of gene expression resulting in the formation of a protein?

A polyribosome, promoter, spliceosome, primary transcript


B spliceosome, promoter, primary transcript, polyribosome
C promoter, primary transcript, spliceosome, polyribosome
D primary transcript, spliceosome, promoter, polyribosome
980

10

16 The hepatitis C virus is a single-stranded, enveloped, positive sense RNA virus, which
attack the liver cells of human beings.
Which of the following correctly describe the reproductive cycle of hepatitis C virus?

A Fusion of the viral envelope with the host cell membrane, uncoating to release
viral genome, followed by synthesis of complementary RNA for translation.
B Fusion of the viral envelope with the host cell membrane, uncoating to release
viral genome, followed by protein synthesis.
C Receptor mediated endocytosis, uncoating to release viral genome, followed
by reverse transcription.
D Receptor mediated endocytosis, uncoating to release viral genome, followed
by transcription.

17 In a hypothetical situation, DNA from T4 phage is mixed with proteins from a lambda
phage, resulting in a hybrid virus. If this hybrid virus were to infect a bacterium and
reproduce, what would the resulting "offspring" viruses be like?

A T4 phage
B Lambda phage
C A hybrid virus with T4 DNA and lambda protein
D A hybrid virus with lambda DNA and T4 protein

18 BRCA1 is expressed in the cells of breast and other tissue, where it helps repair
damaged DNA, or destroy cells if DNA cannot be repaired. BRCA1 gene is on
chromosome 17 of the human genome.

Which individual is most likely to develop breast cancer?

A Has a frameshift mutation of the BRCA1 allele on each chromosome 17.


B Has a BRCA1 allele on each chromosome 17.
C Has a BRCA1 allele on one chromosome 17.
D Has a BRCA1 allele on one chromosome 17 and another, by translocation, on
one chromosome 12.
981

11

19 The diagram shows stalked particles on part of a crista membrane in a mitochondrion.

What occurs in each of the numbered regions?

1 2 3

A ADP synthesis electron transport Krebs cycle


B ATP synthesis glycolysis Krebs cycle
C ATP synthesis Krebs cycle electron transport
D ATP synthesis Krebs cycle glycolysis

20 The diagram represents autoradiograph two-dimensional chromatograms of


different algal extracts. The extracts were obtained after algal suspensions, which
had been exposed to radioactive CO2 for different length of time in conditions
suitable for photosynthesis, were plunged into hot ethanol to prevent further
biochemical reactions from occurring.

Assuming that all techniques and conditions (apart from time) were standardised,
which of the following conclusions is justified from these data?

A compound X must be sucrose


B compound V must be an intermediate in the photosynthetic pathway
C compound T precedes compound Y in the photosynthetic pathway
D compound U precedes compound X in the photosynthetic pathway
982

12

21 In tomato plants, purple pigment and hairy stem are genetically determined and
the genes are tightly linked. Pure breeding, purple, hairy plants crossed with pure
breeding green, hairless plants produced only purple, hairy plants in the F1
generation.

These F1 plants were then crossed to produce the F2 generation and a total of 500
F2 generation plants were produced. Which of the following best describes the
number of offsprings obtained from this cross?

purple, hairy purple, hairless green, hairy green, hairless

A 187 48 52 213
B 204 205 45 46
C 282 218 0 0
D 467 0 0 33

22 In a series of mapping experiments, the recombination frequencies for four different linked
genes of Drosophila were determined as shown below.

What is the order of these genes on a chromosome map?


A b-rb-cn-vg
B cn-rb-b-vg
C rb-cn-vg-b
D vg-cn-b-rb
983

13

23 The feather colors in budgies (a kind of bird) are determined by two pairs of unlinked
genes Y/y and allele B/b. The genotype of green color feather is Y_B_, blue yyB_,
and white _ _bb (where _ indicates the presence of either the dominant or recessive
allele).

If two heterozygous budgies are mated, what proportion of phenotypes can be


expected in the progeny?

A 9 green color feathers : 7 white color feathers


B 9 green color feathers : 3 blue feathers : 4 white feathers
C 15 green color feathers : 1 blue feathers
D 15 green color feathers : 1 white feathers

24 The family tree shows the inheritance of a human defect.

What is the most likely genetic basis for the occurrence of this defect?
A autosomal dominant
B autosomal recessive
C sex-linked dominant
D sex-linked recessive
984

14

25 Test crossing of some tomato plants gave the following results.

plants with round leaves and are tall 157


plants with round leaves and are dwarf 143
plants with oval leaves and are tall 140
plants with oval leaves and are dwarf 160

The expected phenotypic ratio from such test cross is 1 : 1 : 1 : 1. A 2 (chi-


squared) test was performed to test the significance of the difference between
the observed and expected results. The calculated 2 = 8.36.

Which combination correctly describes the result of the 2 test?


985

15

26 The followings are possible events that may lead to speciation.

I Gene mutation
II Increased gene flow
III Natural selection
IV Geographical isolation
V Habitat differentiation within the same geographical location

The correct order of events that may lead to sympatric speciation is

A IV, I, III
B IV, II, III
C V, I, III
D V, II, III

27 Members of two different species possess a similar-looking structure that they use in a
similar fashion to perform the same function. Which information would best help distinguish
between an explanation based on homology versus one based on convergent evolution?

A Both species are well adapted to their particular environments.


B The sizes of the structures in adult members of both species are similar.
C The two species live at great distance from each other.
D The two species share many proteins in common, and the nucleotide
sequences that code for these proteins are almost identical.

28 Which of the following would provide the best information for distinguishing
phylogenetic relationships between several species that are almost identical in
anatomy?

A Comparative embryology.
B Homologous structures.
C Molecular comparisons of DNA and amino acid sequence.
D The fossil records.
986

16

29 The diagram shows the sequence of events occurring as an action potential arrives at a
synapse.
The numbered arrows represent the movement of substances across the membranes.

Presynaptic Postsynaptic
neurone neurone

What are the substances moving across the membranes?


1 2 3 4 5
A K+ Na+ acetylcholine Ca2+ K+
B K+ Na+ K+ Ca2+ acetylcholine
C Na+ K +
Ca 2+
acetylcholine Na+
D Na+ K +
Na +
acetylcholine Ca2+

30 The post-synaptic membrane of a nerve may be stimulated by certain


neurotransmitters to permit the influx of chloride ions into the cell. This process will
result in

A an action potential.
B membrane depolarization.
C the production of an excitatory post-synaptic potential.
D the production of an inhibitory post-synaptic potential.

31 Two organs secrete substances which affect the body.

organ 1 product 1

organ 2 product 2

Negative feedback control of product 2 would be achieved if

A product 1 counteracts product 2.


B product 2 inhibits organ 1 and product 1 inhibit organ 2.
C product 2 inhibits organ 1 and product 1 stimulates organ 2.
D product 2 stimulates organ 1 and product 1 stimulates organ 2.
987

17

32 Which of the following is a similarity between G protein coupled receptors and receptors
tyrosine kinase?

A Conformational change that results from binding of ligand molecule.


B Formation of a dimer following binding of a signal molecule.
C Ligand-binding sites are specific for steroid hormones.
D Seven regions of the receptor span the plasma membrane.

33 Which feature of stem cells obtained from blood in the umbilical cord enables their
use in the treatment of a variety of blood disorders?

A They are totipotent.


B They can differentiate into any type of blood cell.
C They can differentiate into stromal stem cells cells.
D They can renew indefinitely to replace defective blood cells.

34 One therapy to treat -thalassaemia is to transplant bone marrow cells from a


genetically compatible donor into a patient. A potential gene therapy involves
adding the normal, dominant allele for -globin to the patients cells.

Which of the following would ensure that the normal allele is passed on to the next
generation?

A Using a liposome to introduce the normal -globin allele into bone marrow
cells.
B Using a retrovirus to introduce the normal -globin allele into a germ cell.
C Using an adeno-associated virus to introduce the normal -globin allele into
bone marrow cells.
D Using an adenovirus to introduce the normal -globin allele into a germ cell.

35 Each of the statements is concerned with the success or failure of gene therapy as a
permanent cure for genetic disorders.

I Somatic cells cannot pass the modified gene on to any offspring


II The treatment does not last long as the treated cells die and are replaced
III The transfection efficiency of non-viral vectors is rather low.

Which statements explain the limitation of gene therapy as a permanent cure for
genetic disorders in human population?

A I and II
B I and III
C II and III
D All of the above
988

18

36 36 The diagram below shows the results of DNA profiling using gel electrophoresis.

What conclusion can be drawn about the DNA in bands I and II?

A The DNA in the two bands has the same base sequence.
B The DNA in the two bands consists of fragments of the same length.
C The DNA in the two bands has the same ratio of bases.
D The DNA in the two bands came from the same source.

Refer to the following information to answer Q37 and 38.

Erythropoietin (EPO) is a small signaling molecule produced by certain kidney


cells, which stimulates the production and differentiation of red blood cells. A
scientist is interested to clone the cDNA of EPO receptor (EPO-R) from mouse and
express it in suitable human host cells. The first step in library construction is cDNA
synthesis.

37 Which of the following components show the correct combinations that are required
for synthesizing a collection of cDNAs containing copies of EPO-R cDNA?

DNA ligase DNA RNA reverse


polymerase polymerase transcriptase
A
B
C
D
989

19

38 The cDNAs synthesized in this mixture were converted into double-stranded DNA molecules
supplied with Xho restriction endonuclease specific cohesive ends and ligated into the Xho
site of a mammalian expression plasmid.
Which of the following sequence should be present in this plasmid if it is to express inserted
cDNAs in human cells?

A A bacterial promoter downstream of the Xho site


B A bacterial promoter upstream of the Xho site.
C A mammalian promoter downstream of the Xho site.
D A mammalian promoter upstream of the Xho site.

39 Which of the following is not an example of genetically modified organisms?

A Bt corn plants that are insect-resistant.


B Delayed ripening in tomato using anti-sense technology.
C Golden Rice that produce high levels of beta-carotene.
D Inbred Salmon that grow to adult size quickly.

40 Plants are more readily manipulated by genetic engineering than are animal cells because

A a somatic plant cell can often give rise to a complete plant.


B more vectors are available for transferring recombinant DNA into plant cells.
C plant cells have cellulose cell wall which allows substances to pass through
easily.
D plant genes do not contain introns.

--- End of Paper ---


990
991
2

9648/01/NJC SH2 H2 Biology Preliminary Examination 2013 [Turn over


992
National Junior College

SH2 H2 Biology 2013 Prelim Paper 1 Answers

1 B 11 C 21 A 31 C
2 A 12 D 22 A 32 A
3 D 13 A 23 B 33 B
4 D 14 D 24 B 34 B
5 D 15 C 25 B 35 D
6 A 16 B 26 C 36 B
7 D 17 A 27 D 37 D
8 D 18 A 28 C 38 D
9 B 19 C 29 C 39 D
10 C 20 D 30 D 40 A
1 993

NATIONAL JUNIOR COLLEGE, SINGAPORE


Senior High 2
Preliminary Examination
Higher 2

CANDIDATE
NAME

BIOLOGY REGISTRATION
CLASS NUMBER

BIOLOGY 9648/02
Paper 2 30 August 2013
2 hours
Additional Materials: Answer Paper

READ THESE INSTRUCTIONS FIRST

Write your Biology class, registration number and name on all the work you hand in.
You may use a soft pencil for any diagrams, graphs or rough working.
Do not use staples, paper clips, highlighters, glue or correction fluid.

Answer all questions. For Examiners Use

Paper 2
The number of marks is given in brackets [ ] at the end of each
question or part question. Section A
1 / 11
Section A 2 / 15
3 / 10
Write your answers in this booklet using dark blue or black pen.
4 / 12

5 / 10
Section B
6 / 12
Write your answers on the separate Answer Paper using dark
blue or black pen. 7 / 10
Section B
8/9 / 20

TOTAL / 100

This document consists of 20 printed pages.


[Turn over

NJC 2013 9648/02/SH2/H2/Biology/PreliminaryExam


994
2

Section A

Answer all the questions in this section.

1 Fig. 1.1 is a transmission electron micrograph of a mammalian cell.

Fig. 1.1

(a) Complete Table 1.1 to:

name in full, structures A, B, and C.


outline how each structure functions to contribute to its specific role in the
mammalian cell.

NJC 2013 9648/02/SH2/H2/Biology/PreliminaryExam


995
3

Table 1.1
structure name of structure function of structure within mammalian cell

[3]

Table 1.2 below represents the ionic concentrations inside and outside a typical mammalian
cell.

Table 1.2
normal conditions
concentration (mM)
ions intracellular extracellular
+
Na 5-15 145
+ -5
H 7 X 10 4 X 10-5
Cl- 5-15 110

(b) Explain how the intracellular concentration of Na+ is maintained.

[1]

NJC 2013 9648/02/SH2/H2/Biology/PreliminaryExam


996
4

Cholera, a disease caused by a bacterial infection of Vibrio cholerae, results in severe


diarrhoea leading to dehydration. A toxin released by the bacterium causes the release of
chloride ions (Cl-) from cells lining the small intestines into the lumen and inhibits the uptake
of sodium ions (Na+) by these cells. Under normal circumstances, the Cl- intracellular
concentration is maintained due to the natural electrical negativity of the cell.

(c) By comparing normal and diarrhoeal situations, explain how this disruption of cellular
ion concentrations would result in extreme dehydration.

[4]

(d) One of the antibiotics used in the treatment of cholera is tetracycline. Tetracycline
causes the death of the pathogen by binding to ribosomes and inhibiting protein
synthesis.

(i) Suggest two ways in which tetracycline acts at ribosomes to inhibit protein
synthesis.

[2]

(ii) Tetracycline does not harm mammalian cells. Suggest an explanation for this.

[1]

[Total: 11]

NJC 2013 9648/02/SH2/H2/Biology/PreliminaryExam


997
5

2 Meselson and Stahl performed the following experiment with the bacterium Escherichia
coli (E. coli). They grew cells for several generations in the presence of the heavy
isotope of nitrogen, 15N. All the DNA of the progeny that were produced from the initial
small inoculum was labelled with 15N. The 15N-labeled cells were then placed in 14N-
containing medium. The DNA produced in subsequent generations was extracted. The
DNA from the different generations was compared using the technique of differential
centrifugation using a caesium chloride density gradient to separate the DNA
containing 15N from DNA containing 14N, as 15N labelled DNA is more dense than
ordinary 14N DNA. The position of the 15N and 14N DNA absorbance was determined in
ultraviolet light at 260 nm. The results obtained are shown in Fig. 2.1.

Fig. 2.1

NJC 2013 9648/02/SH2/H2/Biology/PreliminaryExam


998
6

(a) (i) With reference to Fig. 2.1, do you consider the replication of DNA in E. coli as
conservative or semi-conservative? Give reasons for your answer.

[3]

(ii) On Fig. 2.1, draw the expected position of the 15


N and 14N DNA absorbance after
three generations in 14N. [1]

(iii) Explain the importance of hydrogen bonding in DNA structure.

[2]

(iv) The enzyme that catalyses the replication of DNA checks for errors in the
process and corrects them. Suggest why checking for errors and correcting them
is necessary.

[1]

NJC 2013 9648/02/SH2/H2/Biology/PreliminaryExam


999
7

A group of scientists has discovered that a change in the chromatin structure of mammalian
cell brought about by Protein A is associated with increased gene expression of several
genes involved in the suppression of tumour growth. Fig. 2.2 shows the overall survival (OS)
percentage of patients in the absence and presence of Protein A.

-------- Protein A absent

________ Protein A present

OS
(%)

Months

Fig. 2.2

(b) (i) With reference to Fig. 2.2, comment on the overall survival percentage of patients
in the presence of Protein A with that in the absence of Protein A, and explain its
differences.

[4]

NJC 2013 9648/02/SH2/H2/Biology/PreliminaryExam


1000
8

(ii) Explain how the presence of transcription factors causes an increase in the
expression of gene involved in suppression of tumour growth.

[3]

(iii) Suggest why the change in chromatin structure, in the presence of Protein A,
does not occur over large area of the chromatin.

[1]

[Total: 15]

NJC 2013 9648/02/SH2/H2/Biology/PreliminaryExam


1001
9

3 Human Immunodeficiency Virus (HIV) is the culprit of Acquired Immunodeiciency


Syndrome (AIDS) which, till now, has no full cure.

Figure 3.1 shows the reproductive cycle of HIV.

Fig.3.1

(a) Name structure A and B.

Structure A:

Structure B: [2]

NJC 2013 9648/02/SH2/H2/Biology/PreliminaryExam


1002
10

(b) Describe the events occurring in stage C leading to formation of structure A on the host
cells plasma membrane.

[3]

(c) After contraction of HIV, the initial symptoms are followed by a stage called clinical
latency when no symptoms are observed for a few years. With reference to the
reproductive features of HIV, explain the cause of clinical latency.

[1]

(d) Flu is caused by influenza viruses.

(i) Distinguish the genome replication between HIV and influenza virus.

[2]

(ii) Every year, new flu vaccines are developed to ensure their efficacy. Explain the
frequent need for new flu vaccines.

[2]

[Total: 10]

NJC 2013 9648/02/SH2/H2/Biology/PreliminaryExam


1003
11

4 The arrow in Fig. 4.1 shows the direction of an impulse in part of a nervous system.

Fig. 4.1

(a) With reference to Fig. 4.1, describe how transmission of impulse is believed to have
taken place.

[4]

NJC 2013 9648/02/SH2/H2/Biology/PreliminaryExam


1004
12

(b) When summation of EPSP and IPSP exceeds threshold, depolarisation occurs. Explain
why the magnitude of the signal initiated at the exon hillock is always at constant
voltage.

[2]

(c) Suggest the purpose of the refractory period during an action potential.

[2]

In a nerve muscle preparation, a novel drug called AWZ with a molecular structure
resembling acetylcholine was applied to the cells. The investigation showed that the drug
significantly increases muscular contractions. However, it was found that the drug, when
applied to either one kind of cells directly, did not affect the axon nor muscle cells.

(d) Suggest, in detail, one possible way in which the drug, AWZ, could cause the
increased muscle contractions.

[4]

[Total: 12]

NJC 2013 9648/02/SH2/H2/Biology/PreliminaryExam


1005
13

Stack 2
Name: ___________________________ Class: __________

5 (a) Pituitary dwarfism is a sex-linked inherited condition in humans in which affected


individuals have very short limbs. The family tree, in Fig. 5.1, shows part of one
affected family.

Fig. 5.1

(i) Explain what is meant by sex-linked inherited condition.

[1]

(ii) With reference to Fig 5.1, explain the mode of inheritance using two pieces of
evidences.

[4]

NJC 2013 9648/02/SH2/H2/Biology/PreliminaryExam


1006
14

A series of breeding experiments were carried out with a certain species of mice. A cross
between pure breeding males with red eyes and bent tails, and females with black eyes and
straight tails produced offspring all with red eyes and bent tails. The same results were
obtained when the cross was carried out again but with the sexes reversed.

(b) Suggest a reason for carrying out the cross with the sexes reversed.

[2]

In another crossing, female offspring from the first experiment were then crossed with black
eyes and straight tails males to obtain the following:

185 red eye, bent tail


153 red eye, straight tail
160 black eye, bent tail
188 purple eye, straight tail

(c) Using the given formula and part of the table of chi-square values as shown in Table
5.1, determine if the null hypothesis for the eye color allele and tail shape allele can be
accepted in the space below.

Part of the table of chi-square values is shown.

Working to be shown in the space:

[3]

[Total: 10]

NJC 2013 9648/02/SH2/H2/Biology/PreliminaryExam


1007
15

6 Fig. 6.1 illustrates what happens on thylakoid membrane during daytime.

Fig.6.1

(a)

(i) Describe how electron transport chain leads to ATP production.

[3]

(ii) Compare the initial electron donor and final electron acceptor between oxidative
phosphorylation and non-cyclic photophosphorylation.

[2]

NJC 2013 9648/02/SH2/H2/Biology/PreliminaryExam


1008
16

(b) A point mutation results in dysfunctional NADP+ reductase. State and explain how the
concentration of Ribulose 1,5 Bisphosphate in the chloroplasts would be affected.

[3]

(c) Glyceraldehyde 3-phosphate exits the Calvin cycle for production of glucose, which will
be used for cell wall formation. Explain how the structure of cellulose allows it to serve
its function.

[4]

[Total: 12]

NJC 2013 9648/02/SH2/H2/Biology/PreliminaryExam


1009
17

7 The greenish warbler, Phylloscopus trochiloides, is a species of small bird that


originated in northern India, on the southern edge of the Himalayan mountain range.

(a) Describe the principles which scientists use to classify organisms into taxonomic
groups.

[2]

Fig. 7.1 shows a greenish warbler.

Fig. 7.1

Thousands of years ago, populations of the greenish warbler spread around the western and
eastern edges of the Himalayan mountain range to establish themselves in north-eastern
Europe and Siberia.

A gradual change in characteristics occurred in these populations, leading to different


forms of the greenish warbler.

One example of gradual change is in the song of the male warbler, which is very
distinctive and is used in mating behavior.

When greenish warblers from north-eastern Europe meet those from Siberia, no mating
takes place.

The greenish warblers from north-eastern Europe and Siberia are now considered to be
two separate species.

NJC 2013 9648/02/SH2/H2/Biology/PreliminaryExam


1010
18

Fig. 7.2 shows the spread of the greenish warbler.

Fig. 7.2

(b) State the likely isolating mechanism taking place in populations of the greenish
warbler.
[1]

(c) Explain how the process of speciation occurred in the greenish warbler populations.

[4]

NJC 2013 9648/02/SH2/H2/Biology/PreliminaryExam


1011
19

An investigation was carried out to study the relationships between the different species of
warbler using DNA analysis. The base sequences of a region of mitochondrial DNA from the
four species were compared and results are shown in Table 7.1.

Table 7.1
P. laetus

P. laetus P. laurae

P. laurae 12.1 P. ruficapilla

P. ruficapilla 16.7 15.0 P. affinis

P. affinis 11.3 8.9 13.2

(d) Researchers have put forward the hypothesis that the three species, P. laetus, P.
laurae and P. ruficapilla, have originated from three separate events in which a few
individuals of P. affinis spread directly from Cuba to three different places.

The smaller the number, the smaller the differences between the base sequences of
the two species.

Explain how the results in Table 7.1 support the researchers hypothesis.

[2]

(e) It was observed that the warblers have remained at approximately the same
frequency for many generations. Suggest an explanation for this phenomenon.

[1]

[Total: 10]

NJC 2013 9648/02/SH2/H2/Biology/PreliminaryExam


1012
20

Section B

Answer one question.

Write your answers on the separate answer paper provided.

Your answers should be illustrated by large, clearly labelled diagrams, where appropriate.

Your answers must be in continuous prose, where appropriate.

Your answers must be set out in sections (a), (b) etc., as indicated in the question.

8 (a) Describe homologous chromosomes and distinguish between their behaviors in


mitosis and the first division of meiosis. [10]

(b) With reference to specific genes, discuss how point mutations and chromosomal
mutations can lead to cancer. [10]

9 (a) Describe how the effects of glucagon are achieved in the liver cell. [10]

(b) Discuss why the eukaryotic genome is considered to be more complex than the
prokaryotic genome. [10]

--- End of Paper ---

NJC 2013 9648/02/SH2/H2/Biology/PreliminaryExam


1013
21

NJC 2013 9648/02/SH2/H2/Biology/PreliminaryExam


1014
1

Answers to NJC H2 Biology Preliminary Paper 2

Section A

1 Fig. 1.1 is a transmission electron micrograph of a mammalian cell.

(a) Complete Table 1.1 to:

name in full, structures A, B, and C.


outline how each structure functions to contribute to its specific role in the
mammalian cell.

Table 1.1
structure name of structure function of structure within mammalian cell

A A: nucleus / Contain gene(s)/ genetic information/ genetic


(eu)chromatin material / DNA that codes for proteins/
polypeptide

Site for transcription (occurring) leading to


R: heterochromatin mRNA synthesis
/chromosome / DNA

B mitochondrion Provides / synthesizes / produces / makes


ATP for protein synthesis / exocytosis
R: mitochondria

C Rough endoplasmic Synthesis / modification / processing /


reticulum transport, of protein / polypeptide

A: translation

[3]

Table 1.2 below represents the ionic concentrations inside and outside a typical mammalian
cell.

(b) Explain how the intracellular concentration of Na+ is maintained. [1]

Sodium-potassium pumps pump out excess Na+ thus requires ATP for the active
transport of this ion.

NJC 2013 9648/02/SH2/H2/Biology/PreliminaryExam (for exchange)


1015
2

Cholera, a disease caused by a bacterial infection of Vibrio cholerae, results in severe


diarrhoea leading to dehydration. A toxin released by the bacterium causes the release of
chloride ions (Cl-) from cells lining the small intestines into the lumen and inhibits the uptake
of sodium ions (Na+) by these cells. Under normal circumstances, the Cl- intracellular
concentration is maintained due to the natural electrical negativity of the cell.

(c) By comparing normal and diarrhoeal situations, explain how this disruption of cellular
ion concentrations would result in extreme dehydration. [4]

Under normal conditions, the chloride ions remain in the cells lining the lumen of the
small intestine creating an overall negative charge within the cells that drives
positively charged sodium ions into the cells
This creates a hyperosmotic condition in the cells/ lower water potential in cells (or
content of the cell would contain more solutes than extracellular fluid) Thus resulting
in water moving into the cells by osmosis.
When suffering from diarrhoea, there is increased concentration of Cl- and Na+ in the
lumen of the small intestine.
Elevated concentrations of these two ions results in the lumen of the small intestines
becoming hyperosmotic/ having a lower water potential compared to the cells lining the
intestines.
This in turn causes large amounts of water through osmosis into the lumen of the
intestine from the cells and blood stream Water passes out of the body as diarrhoea,
leaving the body dehydrated.

(d) One of the antibiotics used in the treatment of cholera is tetracycline. Tetracycline
causes the death of the pathogen by binding to ribosomes and inhibiting protein
synthesis.

(i) Suggest two ways in which tetracycline acts at ribosomes to inhibit protein
synthesis. [2]

Binding of tRNA is prevented no codon-anticodon binding and peptide bond formation


is prevented
mRNA attachment is prevented ribosome movement along the mRNA is hindered /
prevented
Inhibits the association of large and small subunits ribosome movement along the
mRNA is hindered / prevented

NJC 2013 9648/02/SH2/H2/Biology/PreliminaryExam (for exchange)


1016
3

(ii) Tetracycline does not harm mammalian cells. Suggest an explanation for this.
[1]

Cell surface membrane of mammalian cell is impermeable to tetracycline


Possibility of tetracycline to be broken down by enzymes / be degraded before entry into
the cell
Tetracycline targets prokaryotic ribosome and the structure of eukaryotic ribosome is
different difference in ribosomal structure

[Total: 11]

NJC 2013 9648/02/SH2/H2/Biology/PreliminaryExam (for exchange)


1017
4

2 Meselson and Stahl performed the following experiment with the bacterium Escherichia
coli (E. coli). They grew cells for several generations in the presence of the heavy
isotope of nitrogen, 15N. All the DNA of the progeny that were produced from the initial
small inoculum was labelled with 15N. The 15N-labeled cells were then placed in 14N-
containing medium. The DNA produced in subsequent generations was extracted. The
DNA from the different generations was compared using the technique of differential
centrifugation using a caesium chloride density gradient to separate the DNA
containing 15N from DNA containing 14N, as 15N labelled DNA is more dense than
ordinary 14N DNA. The position of the 15N and 14N DNA absorbance was determined in
ultraviolet light at 260 nm. The results obtained are shown in Fig. 2.1.

(a) (i) With reference to Fig. 2.1, do you consider the replication of DNA in E. coli as
conservative or semi-conservative? Give reasons for your answer. [3]
14
It is semi-conservative, because after one generation in N, the DNA had a density
intermediate between that containing 15N and that with 14N.
14
After 2 generations in N, the DNA was of two types: one with an intermediate density
and another with a density corresponding to 14N DNA.
This shows that the DNA molecule unzips and each original strand acts as a template for
the synthesis of new strand. Daughter DNA contains one old and one new strand. If it is
conservative, in each generation there would always present DNA that has density
similar to the parent DNA.

(ii) On Fig. 2.1, draw the expected position of the 15


N and 14N DNA absorbance after
three generations in 14N. [1]

Two peaks: A higher peak at 14N and a smaller peak at 14N-15N sedimentation

(iii) Explain the importance of hydrogen bonding in DNA structure. [2]

Hydrogen bonds hold the two polynucleotides / strands / chains together via
complementary base pairing
Many hydrogen bonds give stability as DNA is a stable molecule
Hydrogen bonds can be broken for the process of DNA replication/ transcription to take
place
Reference made to structure of DNA as double helix

NJC 2013 9648/02/SH2/H2/Biology/PreliminaryExam (for exchange)


1018
5

(iv) The enzyme that catalyzes the replication of DNA checks for errors in the
process and corrects them. Suggest why checking for errors and correcting them
is necessary. [1]

Errors are mutations and the presence of mutations may lead to production of altered
proteins / impaired / loss of function
Altered proteins may lead to different antigens on the surface of cells, so cells will be
rejected by immune system
Impaired coordination of cells i.e. cancerous cells whereby there is uncontrolled cell
division / cell replication / cell growth OR lack of contact inhibition / no apoptosis / proto-
oncogenes

A group of scientists has discovered that a change in the chromatin structure of mammalian
cell brought about by Protein A is associated with increased gene expression of several
genes involved in the suppression of tumour growth. Fig. 2.2 shows the overall survival (OS)
percentage of patients in the absence and presence of Protein A.

(b) (i) With reference to Fig. 2.2, comment on the overall survival percentage of patients
in the presence of Protein A with that in the absence of Protein A, and explain for
its differences. [4]

Quote data for both absence and presence of Protein A


Protein A is involved in histone acetylation where the acetyl groups are attached to
positively charged lysine amino acids in histone tails
When the histone tails of a nucleosome are acetylated, their positive charges are
neutralized
Results in the chromatin having a less compact structure leading to an increase in the
rate of transcription of tumor suppressor gene

(ii) Explain how the presence of transcription factors causes an increase in the
expression of the gene involved in suppression of tumour growth. [3]

Compulsory marking point: Activator proteins (specific transcription factors) bind to distal
control elements grouped as an enhancer in the DNA.
DNA-bending protein binds to the DNA and brings the bound activators closer to the
promoter.
Enables the enhancer to influence a promoter hundreds or even thousands of
nucleotides away. Other transcription factors, mediator proteins and RNA polymerase
are nearby in the cells nucleus.
The activators bind to certain general transcription factors and mediator proteins. These
multiple protein-protein interactions help position and increase the rate of assembly of
the transcription initiation complex on the promoter for RNA synthesis to begin.

NJC 2013 9648/02/SH2/H2/Biology/PreliminaryExam (for exchange)


1019
6

(iii) Suggest why the change in chromatin structure, in the presence of Protein A,
does not occur over large area of the chromatin [1]

Reason 1:
Only a small proportion of genes are to be expressed for tumor suppression
Thus genes that code for proteins that are not required in that particular cell type
will have their transcription repressed instead

Reason 2:
Size of eukaryotic genome is very large with largely non-coding regions
thus histone acetylation (changes to the chromatin structure) do not occur at
these regions as no gene expression is required

[Total: 15]

NJC 2013 9648/02/SH2/H2/Biology/PreliminaryExam (for exchange)


1020
7

3 Human Immunodeficiency Virus (HIV) is the culprit of Acquired Immunodeiciency


Syndrome (AIDS) which, till now, has no full cure.

Figure 3.1 shows the reproductive cycle of HIV.

(a) Name structure A and B. [2]

A: gp120
B: CD4 receptor

(b) Describe the events occurring in stage C leading to formation of structure A on the host
cells plasma membrane. [3]

Polypeptides translated by ribosomes enter the lumen of rough endoplasmic reticulum


for proper folding.
Proteins are packaged in transport vesicles which bud off RER and fuse with cis face of
golgi apparatus in which glycosylation occurs.
Glycoprotein is inserted into the membrane of secretory vesicles when they bud off the
trans face of golgi apparatus and fuse with the plasma membrane. The
glycoprotein/structure A will be incorporated in the plasma membrane.

(c) After contraction of HIV, the initial symptoms are followed by a stage called clinical
latency when no symptoms are observed for a few years. With reference to the
reproductive features of HIV, explain the cause of clinical latency. [1]

The reverse transcribed HIV DNA is integrated into the host chromosome as provirus.
Provirus remains silent/ transcriptionally inactive for years, causing no symptoms.

(d) Flu is caused by influenza viruses.

i. Distinguish the genome replication between HIV and influenza virus. [3]

Feature HIV Influenza virus

Beginning Single stranded Single stranded


material positive/sense RNA negative/anti-sense RNA

Viral Reverse transcriptase RNA-dependent RNA


enzymes synthesize double stranded cDNA polymerase synthesizes the
involved from ss RNA; complementary sense strand
followed by anti-sense strand
Integrase incorporate
the viral cDNA as provirus The viral genome is not
into host chromosome integrated into the host
chromosome

NJC 2013 9648/02/SH2/H2/Biology/PreliminaryExam (for exchange)


1021
8

Time taken to Provirus remains silent for Fast and immediate.


replicate the years. It will be transcribed
genome to form the positive ssRNA
and mRNA when provirus
is activated to produce new
HIV virion.

Examiners comments: Not sufficient information is given in this question for marks
to be awarded. Enzymes should also be stated when mentioning processes.

ii. Every year, new flu vaccines are developed to ensure their efficacy. Explain the
frequent need for new flu vaccines. [2]

Influenza viruses undergo antigenic drift frequently due to high mutation rate during its
genome replication leading to change in conformation of haemagglutinin and
neuraminidase on its viral envelope
The lack of proof reading ability of RNA-dependent RNA polymerase as well as the fast
replication involving two rounds of complementary RNA strand formation account for
the high mutation rate.

[Total: 10]

NJC 2013 9648/02/SH2/H2/Biology/PreliminaryExam (for exchange)


1022
9

4 The arrow in Fig. 4.1 shows the direction of an impulse in part of a nervous system.

(a) With reference to Fig. 4.1, describe how transmission of impulse is believed to take
place. [4]

Arrival of nerve impulse at synaptic knob depolarises presynaptic membrane and


increases permeability to Ca2+, causing influx of Ca2+.
Synaptic vesicles move, fuse with presynaptic membrane to discharge neurotransmitters
by exocytosis.
Neurotransmitter diffuses across cleft to attach to specific receptor sites on post-synaptic
membrane. Ligand gated ion channels opens up to allow influx of Na+.
This bring about local depolarization which when exceeds threshold, an action potential
is triggered in post-synaptic neurone.

(b) When summation of EPSP and IPSP exceeds threshold, depolarisation occurs. Explain
why the magnitude of the signal initiated at the exon hillock is always at constant
voltage. [2]

When threshold potential is reached, all the voltage sensitive sodium ion channels will
open, resulting in the influx of Na+.
When +50mv is reached, all the sodium ion channels will closed
It is an all-or-none event/Axon hillock naming

(c) Suggest the purpose of the refractory period during an action potential. [2]

Cause/explaination of refractory period


Prevent over fatique of the neuron
Prevent back stimulation of the axon.

In a nerve muscle preparation, a novel drug called AWZ with a molecular structure
resembling acetylcholine was applied to the cells. The investigation showed that the drug
significantly increases muscular contractions. However, it was found that the drug, when
applied to either one kind of cells directly, did not affect the axon nor muscle cells.

(d) Suggest, in detail, one possible way in which the drug, AWZ, could cause the
increased muscle contractions. [4]

Shape similar to acetylcholine. Serve as competitive inhibitor to acetylcholinesterase


Competes with acetylcholine for the active site
Acetylcholine unable to be broken down and continuously bind to receptors on post
syanaptic membrane.
Prolonged binding to receptors at post synaptic membrane
leads to increased frequency of depolarization at the post-synaptic neuron.
[Total: 12]

NJC 2013 9648/02/SH2/H2/Biology/PreliminaryExam (for exchange)


1023
10

5 (a) Pituitary dwarfism is a sex-linked inherited condition in humans in which affected


individuals have very short limbs. The family tree, in Fig. 5.1, shows part of one
affected family.

Fig. 5.1

(i) Explain what is meant by sex-linked inherited condition. [1]

A character that is controlled by a gene situated on the X-chromosome or Y


chromosome, although X being more common.

(ii) With reference to Fig 5.1, explain the mode of inheritance using two pieces of
evidences. [4]

1st Piece of evidence:


If the allele for dwarfism is dominant, the affected male, 4, who has only one X
chromosome should pass this allele to all his daughters, who should then be affected.
[Ref to all daughters should be affected]
However, his daughters, 8 and 10 are normal. Hence allele for this condition is not
dominant but recessive.
OR
Male 4 receives its only X chromosome carrying the allele for dwarfism from his mother 2
(since he would have obtaineid Y chromosome from his dad)
Hence his mother 2 is a heterozygote with an allele for normal limb and an allele for
dwarfism
Since the mother of male 4 is normal, the allele for dwarfism must be recessive to the
allele for normal limb.

NJC 2013 9648/02/SH2/H2/Biology/PreliminaryExam (for exchange)


1024
11

A series of breeding experiments were carried out with a certain species of mice. A cross
between pure breeding males with red eyes and bent tails, and females with black eyes and
straight tails produced offspring all with red eyes and bent tails. The same results were
obtained when the cross was carried out again but with the sexes reversed.

(b) Suggest a reason for carrying out the cross with the sexes reversed. [2]

To check whether the genes are sex-linked. [Answers stating linked genes were
rejected]
If sex-linkage involved as the mode of inheritance, then offspring phenotypic ratio will
be different.

In another crossing, female offspring from the first experiment were then crossed with black
eyes and straight tails males to obtain the following:

185 red eye, bent tail


153 red eye, straight tail
160 black eye, bent tail
188 purple eye, straight tail
(c) Using the given formula and part of the table of chi-square values as shown in Table
5.1, determine if the null hypothesis for the eye color allele and tail shape allele can be
accepted in the space below. [3]

Part of the table of chi-square values is shown.

2=1.06+2.00+0.77+1.59 =5.42; Degree of freedom=4-1=3; At P=0.05, 2=7.81

Since 5.42<7.81, there is no significant difference between observed and expected and is
due to chance. Therefore accept null hypothesis.

[1] - Given to correct calculation of 5.42,


[1] - Given to stating Degree of freedom and or P and calculated X2 Value.[ Students are
required to state all 3 items!]
[1] - State the conclusion of the hypothesis. Reject or accept.
[Total: 10]

NJC 2013 9648/02/SH2/H2/Biology/PreliminaryExam (for exchange)


1025
12

6 Fig. 6.1 illustrates what happens on thylakoid membrane during daytime.

(a)
(i) Describe how electron transport chain leads to ATP production. [3]

When electrons are transferred through a series of electron carriers arranged in a


progressively lower energy fashion, energy is released during each transfer
The energy is use to actively transport H+ against its concentration gradient from stroma
to the thylakoid space, creating a electrochemical H+ gradient.
Chemiosmosis occurs when the H+ flows down its concentration gradient via facilitated
diffusion through ATP synthase, driving the synthesis of ATP from ADP and Pi in the
stroma.

(ii) Compare the initial electron donor and final electron acceptor between oxidative
phosphorylation and photophosphorylation. [2]

Oxidative Photophosphorylation
phosphorylation

Initial electron donor NADH2 and FADH2 H2O

Final electron O2 NADP+


acceptor

(b) A point mutation results in dysfunctional NADP+ reductase. State and explain how the
concentration of Ribulose 1,5 Bisphosphate in the chloroplasts would be affected.
[3]

NADP+ cannot be reduced to NADPH which is required as a reducing power for the
conversion of 1,3-bisphosphoglycerate to glyceraldehydes 3-phosphate.
The subsequent regeneration of Ribulose 1,5 Bisphosphate is prevented while carbon
fixation continues to occur,
depleting the supply of Ribulose 1,5 Bisphosphate, causing its concentration to drop.

(c) Glyceraldehyde 3-phosphate exits the Calvin cycle for production of glucose, which will
be used for cell wall formation. Explain how the structure of cellulose allows it to serve
its function. [4]

About 10,000 -glucose units with alternating units rotated 180o in space are linked by
(1,4) glycosidic bonds to form long and linear chains.

Large number of H-bonds / extensive cross-links is formed between the linear cellulose
chains due to the large number of -glucose units with protruding OH groups in each
chain.

NJC 2013 9648/02/SH2/H2/Biology/PreliminaryExam (for exchange)


1026
13

These cross-linked chains are associated to form microfibrils, which are then meshed
into a macrofibril matrix .

The resultant great tensile strength & insolubility in water allows cellulose to be the main
component of plant cell wall, providing structural support to plant cells. The porous matrix
allows free passage of water molecules.

[Total: 12]

NJC 2013 9648/02/SH2/H2/Biology/PreliminaryExam (for exchange)


1027
14

7 The greenish warbler, Phylloscopus trochiloides, is a species of small bird that


originated in northern India, on the southern edge of the Himalayan mountain range.

(a) Describe the principles which scientists use to classify organisms into taxonomic
groups. [2]

Smaller groups within big groups hierarchical (compulsory marking point)


Based upon similarities / features in common
Evolutionary relationships / common ancestry / phylogenetics

Fig. 7.1 shows a greenish warbler.

Thousands of years ago, populations of the greenish warbler spread around the western and
eastern edges of the Himalayan mountain range to establish themselves in north-eastern
Europe and Siberia.

A gradual change in characteristics occurred in these populations, leading to different


forms of the greenish warbler.

One example of gradual change is in the song of the male warbler, which is very
distinctive and is used in mating behavior.

When greenish warblers from north-eastern Europe meet those from Siberia, no mating
takes place.

The greenish warblers from north-eastern Europe and Siberia are now considered to be
two separate species.

Fig. 7.2 shows the spread of the greenish warbler.

(b) State the likely isolating mechanism taking place in populations of the greenish warbler.
[1]

Geographical barrier / isolation

(c) Explain how the process of speciation occurred in the greenish warbler populations.
[4]

Breeding/gene flow between populations is prevented


Gene mutations occurring to make reference to at least 1 type of mutation that leads to
change in protein sequence
Different selection pressures/different (environmental) conditions
Genetic change, whereby different alleles selected for/change in allele frequency/change
in gene pool/advantageous alleles passed on
Presence of genetic drift
Do not recognize the song (ultimately) cannot interbreed/reproductively isolated
Eventually, allopatric speciation

NJC 2013 9648/02/SH2/H2/Biology/PreliminaryExam (for exchange)


1028
15

An investigation was carried out to study the relationships between the different species of
warbler using DNA analysis. The base sequences of a region of mitochondrial DNA from the
four species were compared and results are shown in Table 7.1.

Table 7.1

P. laetus

P. laetus P. laurae

P. laurae 12.1 P. ruficapilla

P. ruficapilla 16.7 15.0 P. affinis

P. affinis 11.3 8.9 13.2

(d) Researchers have put forward the hypothesis that the three species, P. laetus, P.
laurae and P. ruficapilla, have originated from three separate events in which a few
individuals of P. affinis spread directly from Cuba to three different places. The smaller
the number, the smaller the differences between the base sequences of the two
species.

Explain how the results in Table 7.1 support the researchers hypothesis. [2]

P. laetus, P. laurae and P. ruficapilla have smaller differences with P. affinis Thus,
are more closely related to D. affinis than to each other
[Quote figures]: For example the number of base differences between P. laurae and
P. affinis is 8.9, as compared to P. ruficapilla and P. laurae which is 15.0.
Comment about figures for P. laurae with P. laetus with reference to different times of
separation

(e) It was observed that the warblers have remained at approximately the same
frequency for many generations. Suggest an explanation for this phenomenon. [1]

Migration of birds from mainland


Presence of homozygote / heterozygote advantage
Stabilizing selection
Selection pressure remains the same.

[Total: 10]

NJC 2013 9648/02/SH2/H2/Biology/PreliminaryExam (for exchange)


1029
16

Section B

Answer one question.

Write your answers on the separate answer paper provided.

Your answers should be illustrated by large, clearly labelled diagrams, where appropriate.

Your answers must be in continuous prose, where appropriate.

Your answers must be set out in sections (a), (b) etc., as indicated in the question.

6 (a) Describe homologous chromosomes and distinguish between their behaviors in


mitosis and the first division of meiosis. [10]

Similarities:
Same number of genes;
Carry the same genes / carry code for the same characteristics;
Same relative position / loci along the chromosome;
Same length/ size/ shape;
Position of the centromere is the same;

Differences:
Different alleles/ carry genes of an alternative kind;
Each originates from different parents;
Different sequence of bases / nucleotides

STAGE MITOSIS FIRST DIVISION OF MEIOSIS

No synapsis / no pairing of Synapsis occurs / homologous


homologous chromosomes chromosomes pair up to form
bivalents
Prophase
No crossing over / no chiasma
formation Crossing over occurs between non-
sister chromatids of homologous pair
at chiasmata.

Chromosomes arrange lengthwise / Homologous chromosomes lie side-


on the same plane / in a single row by-side / equidistant on either side of
on the equator of spindle. the equator.
Metaphase
No independent assortment since
Independent assortment of alleles
sister chromatids are identical.
occur.

NJC 2013 9648/02/SH2/H2/Biology/PreliminaryExam (for exchange)


1030
17

Centromere of each chromosome Centromeres do not divide


divide
Anaphase Homologous chromosomes separate
Sister chromatids separate
OR
OR
Separating chromosomes are not
Separating chromatids are identical identical.

(b) With reference to specific genes, discuss how point mutations and chromosomal
mutations can lead to cancer. [10]

Proto-oncogenes such as Ras gene codes for Ras protein which is involved in promoting
mitosis in the presence of signaling molecules such as epidermal growth factor.
Gain-of-function mutations change the proto-oncogene to oncogene, causing production
of hyperactive Ras protein or over-production of Ras protein.
Base pair substitutions in the coding sequence can results in missense mutations,
coding different amino acids and resulting in altered three dimensional conformation of
the protein. For instance, the GTPase active site could be altered and is no longer able
to catalyse hydrolysis of GTP bound to Ras protein, making it perpetually active /
hyperactive.
Base pair substitutions in the regulatory sequences such as promoter and enhancer of
the Ras gene can transform the promoter into a more powerful regulator of gene
transcription, resulting in large quantity of Ras protein synthesized.
Chromosomal translocation can cause Ras gene next to a stronger promoter, resulting in
over-production of Ras protein.
Aneuploidy as a result of non-disjunction during mitosis can lead to more than 2 copies
of the Ras gene present in a cell, resulting in over-production of Ras protein also.
Tumor suppressor genes such as p53 gene codes for p53 protein which is a transcription
factor switching on genes whose products are involved in repairing DNA damage or
triggering apoptosis should the damage is beyond repair.
Point mutations and chromosomal mutations can lead to loss-of-function mutation in p53
gene, leading to nonfunctional p53 protein or inadequate amount of p53 protein.
Base pair substitutions in the coding sequence of p53 gene can lead to missense
mutations, coding for a different amino acid. The tertiary structure of p53 protein is
altered, rendering it dysfunctional.

NJC 2013 9648/02/SH2/H2/Biology/PreliminaryExam (for exchange)


1031
18

Base pair substitution in the regulatory sequence such as promoter or enhancer of p53
gene can lead to a non-functional promoter or enhancer not recognizable by transcription
factors and RNA polymerase, hence leading to transcriptional inactivation. No or
insufficient p53 protein is produced.
A chromosomal translocation of p53 gene next to a non-promoter region such as
centromere or other repeated sequence cause the gene to be silent / no expressed.
Aneuploidy as a result of non-disjunction during mitosis can lead to only 1 copy of p53
gene present, increasing the likelihood of losing functional p53 proteins as a mutant
allele will not be masked by the other normal allele.
Chomosomal deletion may also result in loss of heterozyosity. One less copy of p53
gene is present in the genome. Mutation in the only allele will directly cause loss of
functional p53 proteins in the cell.
When both p53 alleles undergo loss-of-function mutation, no functional p53 proteins are
produced to arrest cell cycle when there is severe DNA damage, causing genetic
instability.
When mutations accumulate in the genome, especially mutations in other tumor
suppressor genes and proto-oncogenes occur, cells will undergo mitosis uncontrollably,
leading to cancer.

NJC 2013 9648/02/SH2/H2/Biology/PreliminaryExam (for exchange)


1032
19

7 (a) Describe how the effects of glucagon are achieved in the liver cell. [10]

1. Ligand-receptor interactions:
Hormone glucagon binds to a G protein-coupled receptor (GPCR) present on the cell
surface membrane of liver cells
Ligand-receptor interaction results in the formation of a hormone-receptor complex
The hormone glucagon is the first messenger that triggers the subsequent
intracellular signaling events in the target cell

2. Signal Transduction:
Binding of glucagon triggers a conformation change in the GPCR
providing a signal that activates the G protein (found on the cytoplasmic side of the
membrane) by replacing the GDP to a GTP
The activated G protein diffuses along the plasma membrane to bind to and activate
adenylyl cyclase which in turn catalyses the conversion of many cyclic AMP (cAMP)
molecules from ATP
cAMP acts as a second messenger (within the liver cell) to activate enzymes such as
protein kinase A
Protein kinase A activate other enzymes by phosphorylating them triggering a
phosphorylation cascade that serves to amplify the initial signal
where the binding of a single glucagon molecule results in a strong response inside
the cell/ number of enzyme molecules activated each step exceeds the preceding
step

3. Cellular response:
(Phosphorylation cascade) finally leads to activation of glycogen phosphorylase
which is needed for the breakdown of glycogen stored in liver cells to glucose
molecules
thereby increasing the availability of glucose for cellular respiration / bring blood
glucose levels back to the norm

NJC 2013 9648/02/SH2/H2/Biology/PreliminaryExam (for exchange)


1033
20

(b) Discuss why the eukaryotic genome is considered to be more complex than the
prokaryotic genome. [10]

Prokaryotic Genome Eukaryotic Genome

Smaller number of bases Larger number of bases

Smaller number of genes / coding Larger number of genes / coding


regions regions

Single chromosome Multiple chromosomes

Usually haploid (i.e. only 1 set of Usually in diploid or higher ploidy levels
chromosomes)
Pairing of homologous chromosomes,
No pairing of homologous chromosomes allows for genetic recombination during
possible, hence no genetic meiosis, giving rise to genetic diversity
recombination as a means of giving rise
to genetic diversity

Absence of / very little intergenic regions Presence of intergenic regions

Most of the genome is coding regions, Presence of repetitive DNA, many


hence little regulatory elements possible whose functions are yet unknown and
to be present may play a role in gene regulation

Polycistronic genes / operons present Monocistronic genes

Allows coordinated control of metabolic Individual enzymes are controlled


enzymes involved in the same pathway / individually by separate promoters
all products of operons produced in the
same amount Can be produced in different amounts or
not at all
However this coordinated control results
in fewer possible metabolic pathways for Hence allowing for partitioning of
a given substrate as all substrate will be metabolic intermediates to different
fed into that one single pathway metabolic pathways to suit the cells
needs

Absence of control elements Presence of control elements

Allows only single-step control of gene Fine tuned control of gene expression,
expression, i.e. dependent on promoter allows for different levels of expression
strength and operator state of a particular gene depending on the
activators and repressors present in the
cell at a specific time

Absence of introns Presence of introns

NJC 2013 9648/02/SH2/H2/Biology/PreliminaryExam (for exchange)


1034
21

No alternative splicing possible Allows alternative splicing

Hence only one possible protein product Presence of introns allows for choice of
per gene splice sites, giving rise to varying mature
mRNA molecules and hence multiple
possible protein variants from the same
gene

No association with histones Association with histones

Association instead with proteins similar Allows for increased structural


to (but not the same as) histones complexity / folding to higher degree of
condensation
Does not achieve same level of
condensation complexity as eukaryote Control of rate of gene transcription by
allowing for increased DNA
condensation/conversion between
euchromatin and heterochromatin states

Gene amplification does not occur Gene amplification is possible

No mechanism for increasing copy Increased copy number of genes allows


numbers of genes for increased transcriptional levels

No nuclear membrane Enclosed by nuclear membrane

Simultaneous transcription and No simultaneous transcription and


translation translation

Hence fewer levels of control of gene Allowing for more levels of control of
expression gene expression

--- The End ---

NJC 2013 9648/02/SH2/H2/Biology/PreliminaryExam (for exchange)


1 1035

NATIONAL JUNIOR COLLEGE, SINGAPORE


Senior High 2
Preliminary Examination
Higher 2

CANDIDATE
NAME

BIOLOGY REGISTRATION
CLASS NUMBER

BIOLOGY 9648/03
Paper 3 17 September 2013
2 hours
Additional Materials: Answer Paper

READ THESE INSTRUCTIONS FIRST

Write your Biology class, registration number and name on all the work you hand in.
You may use a soft pencil for any diagrams, graphs or rough working.
Do not use staples, paper clips, highlighters, glue or correction fluid.

Answer all questions.

The number of marks is given in brackets [ ] at the end of each question or part question.

Section A

Write your answers in this booklet using dark blue or black pen.

For Examiners Use


Section B
1 / 14
Write your answers on the separate Answer Paper using dark
blue or black pen. 2 / 15
3 / 11
4 / 12

5 / 20

TOTAL / 72

This document consists of 14 printed pages.


[Turn over

NJC 2013 9648/03/SH2/H2/Biology/PreliminaryExam


1036
2

Section A

Answer all questions.

1. A reporter gene is a gene whose activity is relatively easy to detect. Luciferase is an


enzyme naturally found in fireflies. When its substrate luciferin is present, it catalyses a
chemical reaction which emits light, as shown by the equation below. Therefore,
luciferase gene can be used as a reporter gene in recombinant DNA technology.

luciferin + ATP + oxygen oxyluciferin + AMP + light

Fig. 1.1 shows a recombinant plasmid construct named pMCS-Red Firefly Luc used
to study the regulation of lac operon in bacteria.

Fig. 1.1

Fig 1.2 shows the detailed structure of Red Firefly Luc insert.

Promoter of lac I gene CAP binding Promoter Operator Luciferase


lacI gene site Gene
Fig. 1

NJC 2013 9648/03/SH2/H2/Biology/PreliminaryExam


1037
3

(a) State the factors which you will need to consider when choosing the right restriction
enzyme to construct the pMCS-Red Firefly Luc.

[2]

(b) Both genomic and cDNA libraries of firefly are available. State where you will take the
luciferase gene from for this experiment. Justify your decision.

[2]

(c) Plasmids are introduced into bacteria via electroporation.

Outline how you can select bacterial cells transformed with recombinant plasmid
containing the Red Firefly Luc insert.

[2]

NJC 2013 9648/03/SH2/H2/Biology/PreliminaryExam


1038
4

A few experiments are conducted to study the regulation of lac operon via expression of
luciferase gene in the bacterial colonies.

Complete the table below by putting in + (low light intensity), ++ (high light intensity) and
- (no light emitted) in the last column.

experiment glucose lactose ampicillin luciferin brightness of colonies

1
2

3
4

[2]

(d) Explain the expected brightness of colonies for experiments 1 and 3.

[4]

(e) Instead of Red Firefly Luc, human insulin gene can also be cloned into pMCS plasmid
shown in Fig.1.1. Suggest a suitable method to select bacterial cells transformed with
recombinant plasmid containing the insulin gene insert.

[2]

[Total: 14]

NJC 2013 9648/03/SH2/H2/Biology/PreliminaryExam


1039
5

2. Xeroderma pigmentosum-variant (XP-V) is an autosomal recessive disorder caused by


defective DNA repair. Individuals with this disorder have sun sensitivity and increased
skin cancer risk. One of the genes responsible for this disorder is the POLH gene, which
encodes for DNA polymerase.

The molecular basis of XP-V was determined using DNA samples from a family in North
America. Insertion of a single guanine nucleotide at position 1078 of exon 10 of the
POLH gene creates a new Bsl1 site as shown in Fig. 2.1 and is linked to an RFLP.

353 base pairs

wild-type allele, represented by


symbol H

mutant allele, represented


by symbol h 105 248
base pairs base pairs

position 1078 of exon 10 of the POLH gene

Fig. 2.1

(a) State and explain the possible consequence of such gene mutation.

[2]

(b) Describe two factors that are required for an RFLP marker to be used in disease
detection.

[2]

NJC 2013 9648/03/SH2/H2/Biology/PreliminaryExam


1040
6

(c) PCR amplification of the entire coding region of POLH gene was first performed.

Step B

Step A

Fig. 2.2

With reference to Fig. 2.2, comment on the purpose of:

(i) Step A

[1]

(ii) Step B

[1]

NJC 2013 9648/03/SH2/H2/Biology/PreliminaryExam


1041
7

Digestion of POLH gene from individuals within the family was then carried out using Bsl1.
Fig. 2.3 shows the incomplete pedigree chart and the results obtained from gel
electrophoresis.

1 2
I

I II
3 4 5
III

6 7

Fig. 2.3
Legend:
(-) indicates no Bsl1 treatment; (+) indicates Bsl1 treatment; (*) indicates normal size band;
() indicates bands of digested DNA.

(d) Using the information provided in the question and with reference to Fig. 2.3, state and
explain the genotype of individual III-6.

[2]

NJC 2013 9648/03/SH2/H2/Biology/PreliminaryExam


1042
8

(d) Outline the experimental procedure of obtaining a DNA fingerprint after the action of Bsl1.

[4]

(f) Explain how RFLP analysis facilitates genomic mapping.

[3]

[Total: 15]

NJC 2013 9648/03/SH2/H2/Biology/PreliminaryExam


1043
9

Set 2

Name: _________________________ Class: ________ Registration number: ________

3. (a) Immature anther of a hibiscus flower bud was removed and the procedure shown in
Fig. 3.1 was carried out to make complete plantlets.

obtaining and sterilizing the anther from flower bud

placing a small piece of the anther tissue on sterile medium


(containing sucrose and plant growth regulators)

formation of callus

plant regeneration by varying concentrations of plant growth regulators

addition of colchicine

transfer of plantlets to soil

Fig. 3.1

(i) Explain why it is important to sterilise the anther tissues.

[1]

(ii) Describe briefly the importance of adding sucrose to the sterile medium.

[1]

NJC 2013 9648/03/SH2/H2/Biology/PreliminaryExam


1044
10

(iii) Explain the purpose of adding colchicine in the final stages of the procedure.

[2]

(b) Explain the advantages of tissue culture over more traditional methods of cloning plants,
such as taking cuttings or graftings.

[2]

(c) With a relevant example, explain how genetic engineering has improved the quality of crops.

[3]

(d) Researchers are experimenting with the injection of human brain cells into the brain
tissues of mice. Discuss the ethical concerns that may arise.

[2]

[Total: 11]

NJC 2013 9648/03/SH2/H2/Biology/PreliminaryExam


1045
11

4. Planning question

Write your answers to this question on the separate answer paper provided.

A type of genetically modified sugar cane contains higher amount of sucrose per unit mass. You
are required to measure the sucrose concentration of freshly squeezed sugar cane juice from
this new variant.

Your planning must be based on the assumption that you have also been provided with the
following materials which you must use:

a beaker of sugar cane juice labelled as S


a 10% sucrose stock solution
distilled water
dilute hydrochloric acid
sodium hydrogen carbonate solution
thermostatically controlled water bath
benedicts solution
colorimeter

In addition, you may include in your plan the usage of any equipment that can be found in a
standard school laboratory.

Your plan should have a clear and helpful structure to include:

an explanation of the theory to support your practical procedure.


a description of the method used, including the scientific reasoning behind the method
an explanation of the dependent, independent variables and controlled variables
how you will record your results and ensure that they are as accurate & reliable as possible.
proposed layout of results tables and graph with clear headings and labels
recommended safety measure.
the correct use of technical and scientific terms.

[Total: 12]

NJC 2013 9648/03/SH2/H2/Biology/PreliminaryExam


1046
12

NJC 2013 9648/03/SH2/H2/Biology/PreliminaryExam


1047
13

NJC 2013 9648/03/SH2/H2/Biology/PreliminaryExam


1048
14

Free Response Question

Write your answers to this question on the separate answer paper provided.

Your answer:

should be illustrated by large, clearly labelled diagrams, where appropriate;


must be in continuous prose, where appropriate;
must be set out in sections (a), (b), etc., as indicated in the question.

5 (a) Explain, with examples, the treatment of genetic diseases with reference to viral
and non-viral gene delivery systems. [8]

(b) Discuss the social and ethical concerns that have arisen from gene therapy in the
treatment of genetic diseases. [6]

(c) Discuss on the different types of stem cells. [6]

--- End of Paper ---

NJC 2013 9648/03/SH2/H2/Biology/PreliminaryExam


1049
1

Answers to NJC H2 Biology Preliminary Paper 3

Section A

1. A reporter gene is a gene whose activity is relatively easy to detect. Luciferase is an


enzyme naturally found in fireflies. When its substrate luciferin is present, it catalyses a
chemical reaction which emits light, as shown by the equation below. Therefore,
luciferase gene can be used as a reporter gene in recombinant technology.

luciferin + ATP + oxygen oxyluciferin + AMP + light

Fig. 1.1 shows a recombinant plasmid construct named pMCS-Red Firefly Luc used
to study the regulation of lac operon in bacteria.

(a) State the factors which you will need to consider when choosing the right restriction
enzyme to construct the pMCS-Red Firefly Luc. [2]

The restriction site of the enzyme must be present in both the MCS of the plasmid and
the sequence flanking the Red Firefly Luc insert, so that complementary sticky ends will
be produced for ligation.
The restriction site must only be found once in the whole plasmid / must be a unique
cloning site.

(b) Both genomic and cDNA libraries of firefly are available. State where you will take the
Luciferase gene from for this experiment. Justify your decision. [2]

cDNA libraries
Firefly is a eukaryote whose genes contain introns which need to be removed through
splicing during post-transcriptional modifications of pre-mRNA. Bacteria are prokaryotes
which do not perform post-transcriptional modifications. Hence, cDNA of luciferase gene
which only contain exons is needed for bacteria to produce functional luciferase.

(c) Plasmids are introduced into bacteria via electroporation. [2]

(i) Outline how you are to select bacterial cells transformed with recombinant plasmid
containing the Red Firefly Luc insert.

Grow the bacteria in an agar medium containing ampicillin / puromycin, those


surviving colonies are the ones transformed with plasmids.

Add IPTG/ lactose (served as inducer to switch on the operon) and luciferin
(substrate of luciferase) to the medium. Those colonies which give out
luminescence are cells transformed with recombinant plasmids containing the
luciferase gene.

NJC 2013 9648/03/SH2/H2/Biology/PrelimExam (for exchange)


1050
2

A few experiments are conducted to study the regulation of lac operon via expression of
luciferase gene in the bacterial colonies.

Complete the table below by putting in + (low light intensity), ++ (high light intensity) and -
(no light emitted) in the last column.

Experiment Glucose Lactose Ampicilin Luciferin Brightness of colonies


1 -
2 +
3 ++
4 -
5 -
[2]

(ii) Explain your results in experiment 1 and 3. [4]

In the absence of lactose, constitutively expressed lac I gene produces active


repressor protein which binds to the operator, preventing RNA polymerase from
transcribing the luciferase gene.
No light is emitted when there is no luciferase catalyzing the reaction in
experiment 1.

When lactose is present, allolactose acts as an inducer which binds and changes
the conformation of repressor protein. Thus the repressor protein can no longer
bind the operator, allowing RNA polymerase to transcribe the luciferase gene.

In the absence of glucose, there is high cAMP concentration in bacterial cell,


leading to formation of active cAMP-CAP complex binding to CAP binding site,
increasing the affinity of RNA polymerase for promoter and thus increasing
transcription rate of the luciferase gene.

High amount of luciferase causes more light to be emitted in experiment 3.


(combined marking point with 2 bullet point
nd

(d) Instead of Red Firefly Luc, human insulin gene can also be cloned into pMCS plasmid
shown in Fig.1.1. Suggest a suitable method to select bacterial cells transformed with
recombinant plasmid containing the insulin gene insert. [2]

Grow bacteria on medium containing ampicilin / puromycin to select those transformed


with pMCS plasmid.
To select colonies transformed with recombinant plasmid, add radioactively/fluorescently
labeled single stranded nucleic acid probe complementary to the insulin cDNA sequence
and perform autoradiography / view under UV light.

[Total: 14]

NJC 2013 9648/03/SH2/H2/Biology/PrelimExam (for exchange)


1051
3

2. Xeroderma pigmentosum-variant (XP-V) is a autosomal recessive disorder caused by


defective DNA repair. Individuals with this disorder have sun sensitivity and increased
skin cancer risk. One of gene identified is the POLH gene, which encodes for DNA
polymerase (pol).

The molecular basis of XP-V worldwide was analyzed from a family in North America.
Insertion of a single guanine at position 1078 of exon 10 creates a new Bsl1 site as
shown in Fig. 2.1 and is linked to an RFLP.

(a) State and explain the possible consequence of such gene mutation. [2]

Leads to frame-shift mutation as the bases downstream are re-grouped into different
codons
Extensive missense or nonsense mutations can be resulted Causing different
sequence of amino acid coded or shorter amino acid sequence
Altered three dimensional structure of DNA polymerase / truncated protein leading to a
loss of biological function

(b) Suggest two factors that are required for a RFLP marker to be used in disease
detection. [2]

The marker must be polymorphic, meaning that alternative forms must exist among
individuals for detection among different members in the family.
The marker must be closely linked to the disease locus such that crossing over between
the marker and the gene is unlikely to occur during gamete formation. The greater the
distance between the RFLP and gene locus, the lower the probability of an accurate
diagnosis.

(c) PCR amplification of the entire coding region of POLH was first performed.

With reference to Fig. 2.2, comment on the purpose of:

(i) Step A [1]


Annealing of primers / oligonucleotides to single DNA strands at 3 end via
complementary base pairing

(ii) Step B [1]

DNA strand elongate from the 3 end of the DNA primer using Taq DNA
polymerase where extension of DNA strand occurs from 5 to 3 direction

NJC 2013 9648/03/SH2/H2/Biology/PrelimExam (for exchange)


1052
4

Digestion of POLH gene from individuals within the family was then carried out using Bsl1.
Fig. 2.3 shows the incomplete pedigree chart and the results obtained from gel
electrophoresis.

(d) Using the information provided in the question and with reference to Fig. 2.3, state and
explain the genotype of individual III-6. [2]

Genotype for III-6: Hh

Have the presence of one band that corresponds to the wild-type allele which is 353bp,
while the other two bands correspond to the mutant allele which corresponds to 105bp
and 248bp

(e) Outline the experimental procedure, after the action of Bsl1, of obtaining a DNA fingerprint. [4]

DNA restriction fragments are size-fractionated by gel electrophoresis;

Bands obtained are transferred / blotted from the gel onto a nitrocellulose membrane

The DNA is denatured into single strands by the alkaline buffer

Membrane is then incubated with radioactively / chemically labeled single-stranded DNA


probes which hybridize to complementary target sequences on the membrane

The membrane is washed thoroughly to remove unbound probe;

This is followed by detection of the bound probe by autoradiography / chemical means to


produce a specific pattern of bands.

(f) Apart from disease detection, explain how RFLP analysis facilitates genomic mapping.
[3]

[RFLP as genetic marker]: RFLP markers spaced throughout the chromosomes are used
as genetic markers (such as STRs, and other polymorphic sites)

[RFLP in the construction of linkage map]: Frequency with which two RFLP markers or
an RFLP marker and a certain allele for a gene are inherited together is a measure of the
closeness of the 2 gene loci on a chromosome.

The higher the frequency of 2 RFLP being inherited together, the closer the 2 loci are on
the chromosome / Based on cross-over values / recombination frequency / frequency of
crossing-over between the RFLP marker and the gene loci (or between 2 RFLP markers)
[RFLP in determining the order of markers]: The presence of RFLP on a chromosome
can serve as marker to order the sequence of genes and their relative distances on a
chromosome.
[Total: 15]

NJC 2013 9648/03/SH2/H2/Biology/PrelimExam (for exchange)


1053
5

3. (a) Immature anther of a hibiscus flower bud was removed and the following main
procedures were introduced in the making of a complete plantlet.

(i) Explain why it is important to sterilise the anther tissues. [1]

This is to prevent contamination/ growth of microorganisms such as fungi and bacteria,


which will outgrow the callus / prevent callus formation/compete for nutrients

(ii) Describe briefly the importance of adding sucrose in the sterile medium. [1]

Sucrose acts as a carbon and energy source

as plant cells in the culture medium lack autotrophic ability/ are unable to
photosynthesize. or

to enhance proliferation of cells and regeneration of shoots.

(iii) Explain the purpose of adding the chemical colchicine in the final stages of the procedure. [2]

To double the chromosome number, to restore the haploid cells back to the diploid
condition.
By preventing mitotic spindle formation
Hence sister chromatids cannot be separated during anaphase of mitosis

(b) Explain the advantages of tissue culture over more traditional methods of cloning plants,
such as taking cuttings or graftings. [2]

The rapid and mass multiplication of plants is made possible because a single plant can
be cloned into thousands or millions of genetically identical copies by subdividing
calluses as they grow.

Large numbers of disease-free plants may be produced if explants from meristematic


regions are used. This is called meristem culture. The meristematic regions of a plant are
the shoot tips and root tips. The meristem is free from bacteria and viruses because the
latter either do not easily invade or do not rapidly multiply in the meristematic tissue.

Plant tissue cultures take up relatively little space compared to growing plants in
greenhouses or on farms.

Micropropagation overcomes the barrier posed by seasons in conventional propagation


methods. Therefore, plants can be produced out-of-season to meet consumer demand.

Micropropagated plants can be airfreighted easily and cheaply, because cultures are not
bulky.

Micropropagation makes it possible to standardize growth conditions, and obtain many


batches of identical plants. This ensures product uniformity for customers.

Many genetically engineered plants can be multiplied and mass produced relatively
easily.

NJC 2013 9648/03/SH2/H2/Biology/PrelimExam (for exchange)


1054
6

(c) With relevant example, explain how genetic engineering has improved the quality of crops. [3]

Flavr Savr Tomato antisense technology


Gene inserted which produces mRNA that is complementary to the mRNA of
polygalacturose
Double stranded mRNA formed and ribsome binding prevented, translation cannot occur
Delayed ripening and hence taste, texture improves

Or Golden rice as example.

(d) Researchers are experimenting with the injection of human brain cells into the brain
tissues of mice. Suggest the ethical considerations that may arise. [2]

Whether humans ought to be creating living organisms not envisaged by nature


It is important that animal chimeras not develop uniquely human characteristics such that
some degree of humanity or human dignity has been conferred on them. This can lead to
a sense of moral confusion and could create obligations toward these new entities that
would require rethinking our categories of animals and humans.
Violating the genetic integrity of species, that has been a result of many generations of
evolution.
Others find the alteration of natural physical characteristics the source of their unease or
repugnance.
If the goal of the unnatural process is immediate and life-saving therapy it might be more
morally acceptable than research of some future indeterminate benefit.

[Total: 11]

NJC 2013 9648/03/SH2/H2/Biology/PrelimExam (for exchange)


1055
7

4. Planning question

Write your answers to this question on the separate answer paper provided.

A type of genetically modified sugar cane contains higher amount of sucrose per unit mass. You
are required to measure the sucrose concentration of freshly squeezed sugar cane juice from
this new variant.

Your planning must be based on the assumption that you have also been provided with the
following materials which you must use:

A beaker of sugar cane juice labelled as S


a 10% sucrose stock solution
distilled water
Dilute hydrochloric acid
Sodium hydrogen carbonate solution
Thermostatically controlled water bath
Benedicts solution
Colorimeter

In addition, you may include in your plan the usage of any equipment that can be found in a
standard school laboratory.

Your plan should have a clear and helpful structure to include:

An explanation of the theory to support your practical procedure.


A description of the method used, including the scientific reasoning behind the method
An explanation of the dependent, independent variables and controlled variables
How you will record your results and ensure that they are as accurate & reliable as possible.
Proposed layout of results tables and graph with clear headings and labels
Recommended safety measure.
The correct use of technical and scientific terms.

[Total: 12]

NJC 2013 9648/03/SH2/H2/Biology/PrelimExam (for exchange)


1056
8

Suggested answers for Planning Question

Explanation of theoretical background (2 marks)

Sucrose is a non-reducing disaccharide.


When the glycosidic bond in sucrose is hydrolysed by dil. HCl under heat, glucose and
fructose is produced. Both glucose and fructose are reducing sugar.

When Benedicts solution containing Cu2+ is added, reducing sugar causes Cu2+ to be
reduced to Cu2O, which is a red precipitate.

Method and scientific reasoning (can be mentioned in the procedure also)

Dilute hydrochloric acid is used to hydrolyse sucrose into reducing sugars glucose and
fructose

Benedicts solution is then added to test for the amount of reducing sugars / glucose and
fructose present.

Hypothesis (can be mentioned in prediction and explanation of results also) (2 marks)

The higher the concentration of sucrose, the greater the amount of brick-red precipitate
produced from the Benedicts test, and the less Cu2+ remaining in the Benedicts solution

After filtering the ppt out, use colorimeter to measure the colour intensity of the filtrate, and
lower absorbance value is expected.

Sucrose solutions of known concentration are measured for absorbance values in


colourimeter

Compare the absorbance value of the S with absorbance values of sucrose solutions of
known concentrations by reading off the graph plotted

Variables: (1 mark)

Independent variables: sucrose concentration (%)

Dependent variables: amount of ppt produced in Benedicts test measured as absorbance value
in colorimeter (A.U.)

Controlled variables (can be mentioned in procedure also)


volume of sucrose solution used
volume of HCl used and duration of hydrolysis reaction
volume of Benedicts solution added and duration of boiling
Temperature of hydrolysis reaction and Benedicts Test

Experimental procedure (max 5)

Using simple dilution (include dilution table) to obtain at least five sucrose solutions of
known concentrations, with equal intervals (concentration range: 1% - 10%)

Use syringe to add 2 cm3 of each sucrose solution (including solution S) into a separate test
tube
Use syringe to add 2cm3 dilute hydrochloric acid to test tube and mix well

NJC 2013 9648/03/SH2/H2/Biology/PrelimExam (for exchange)


1057
9

Place the test tubes into a thermostatically controlled water bath set at 100 oC / boiling
water bath for three minutes (accept reasonable time)

Remove the test tubes from water bath. Use syringe to add 5cm 3 (accept reasonable range)
of sodium hydrogen carbonate solution until no effervescence is observed. This is to quench
the acid hydrolysis reaction by neutralising the acid

Use syringe to add 2cm3 Benedicts solution to each test tube

Place test tubes into a thermostatic water bath set at 100oC / boiling water bath; for 2
minutes (accept reasonable time);

Remove the test tubes and filter the ppt out.

Place a small sample of each filtrate into the colorimeter to read the absorbance value

Conduct 3 replicates for each sucrose solution;

Repeat the entire experiment twice to ensure reproducibility;

Data recording (2 marks)

Tabulation of data with correct column headings and units; include average value for
absorbance value;

sucrose absorbance value of filtrate / A.U.


concentration / % Exp 1 Exp 2 Exp 3 Average
2
4
6
8
10
Solution S

Plot a graph of average absorbance value against sucrose concentration;

Correct labelling of x and y-axis;

NJC 2013 9648/03/SH2/H2/Biology/PrelimExam (for exchange)


1058
10

Safety precaution (1 mark)

Reagents such as Benedicts solution, dilute hydrochloric acid, etc are corrosive to skin and
eyes. Wear gloves and goggles. Rinse under running water when in contact occurs.

Boiling water bath can scald. Handle hot glassware such as test tubes with test-tube holders
or oven gloves/rags.

NJC 2013 9648/03/SH2/H2/Biology/PrelimExam (for exchange)


1059
11

Free Response Question

Write your answers to this question on the separate answer paper provided.

Your answer:

Should be illustrated by large, clearly labelled diagrams, where appropriate;


Must be in continuous prose, where appropriate;
Must be set out in sections (a), (b), etc., as indicated in the question.

5 (a) Explain, with examples, the treatment of genetic diseases with reference to viral
and non-viral gene delivery system. [8]

(b) Discuss the social and ethical concerns that have arisen from gene therapy in the
treatment of genetic diseases. [6]

(c) Discuss on the different types of stem cells. [6]

(a) Explain, with examples, the treatment of genetic diseases with reference to viral and
non-viral gene delivery system. [8]

1. Gene therapy is a technique for introducing normal allele for the purpose of generating
functional (correct) protein product to restore the target cell to normal state (correct
function of cell / correct phenotype)

2. Example of viral gene delivery method involves SCID treatment, using ex vivo method

3. Functional ADA allele is cloned into retroviral vector. The viral vector has to be
genetically engineered so that they neither produce disease nor replicate in the host cell.

4. Patients lymphocytes will then be extracted from the blood and be transfected with ADA
recombinants
5. Following infection, the recombinants cells can be cultured in a suitable medium and
suitable cells can then be expanded in culture in vitro before they were re-injected into
the patients blood stream.
(Max: 4)

6. Example of viral gene delivery method involves CF treatment, using in vivo method

7. Functional CFTR allele is cloned into adenoviral vector. The viral vector has to be
genetically engineered so that they neither produce disease nor replicate in the host cell.

8. Recombinant adenovirus must be delivered effectively to introduce a functional CFTR


allele into the nucleus of the epithelial cells lining the bronchial tree within the lungs

9. Directly either through a bronchoscope or through the nasal cavity. the gene is
inserted into cells within body

NJC 2013 9648/03/SH2/H2/Biology/PrelimExam (for exchange)


1060
12

10. Example of using non-viral gene delivery method involves cystic fibrosis treatment, using
the in vivo method

11. In vivo cationic liposomes are used and consists of lipid : DNA complexes that were
developed for the transfection of epithelial cells lining the airway

12. DNA (functional CFTR allele) interacts with the cationic lipids on the inner layer via
electrostatic interactions. The cationic lipids on the outer layer serve to play a role in
enhancing liposome interaction with the target cells

13. The functional CFTR allele enters the target cells (i.e. lung epithelial cells) through the
lipids fusing with the plasma membrane, and the DNA eventually entering the cell
nucleus. direct introduction of DNA into target cells

14. Liposomes containing the functional CFTR allele are administered by the inhalation of
aerosols by the patient.
(Max: 4)

(b) Discuss the social and ethical concerns that have arisen from gene therapy in the
treatment of genetic diseases. [6]

Social considerations
1. Gene therapy is currently very expensive and there is concern about gene therapy
becoming accessible only to the wealthy

2. Possible enhancements in creating an advantage for those who can afford the treatment

3. Large numbers of gene therapy trials are currently directed to diseases with an
enormous social impact, such as cancer, SCID, CF etc.

4. Gene therapy might provide new forms of treatments directed to currently unmet clinical
needs with the potential impact to improve patients quality of life

5. Concern over widespread use of gene therapy making society less accepting of people
who have mild disorders

6. Potential for non-therapeutic enhancement possibilities / eugenic social policies

7. Gene therapy might provide new forms of treatments directed to currently unmet clinical
needs / alternative treatments for patients where conventional treatments have failed.

Ethical considerations
8. Difficulty in following-up with patients in long-term clinical research. Gene therapy
patients have to be under surveillance to monitor for long term effects of the therapy on
future generations.

9. Concerns over whether patients and their family members fully understand the risks
associated with gene therapy trials

10. And whether they make informed decisions before participating in a trial

NJC 2013 9648/03/SH2/H2/Biology/PrelimExam (for exchange)


1061
13

11. Ref. to Jesse Gelsinger (1st death) gene therapy trial he had a relatively mild form of a
disease that was being successfully treated with medication and perhaps should not
have been a gene therapy trial participant

12. Issues pertaining to safety and efficacy of gene therapy as such therapy have not yet
had the success anticipated by many scientists as a few unexpected complications and
side effects have been identified. Therefore, the novelty of risks related to gene transfer
means there will be lots of uncertainties as compared to conventional therapeutics.

13. Raises questions over who should participate in a human trial the terminally ill with no
treatment options or anyone who understands the potential risk but is a willing participant

14. Sponsors of human clinical trials must exercise responsibility in balancing experimental
aims and ensuring that participants are not exposed to known risks / experimental
products are as safe as possible.

15. Issues of justice and allocation where the high cost of gene therapy for the actual
treatment and technical expertise may leads to inequality between individuals with
genetic enhancements and those without.

16. In the event of disastrous outcomes brought about by gene therapy, the challenge lies in
identifying who should be held accountable

17. Germ-line gene therapy will violate the rights of subsequent generations to inherit a
genetic endowment that has not been intentionally modified and permission must be
sought from the individuals in these generations as to whether are they willing to
undergo gene therapy.

18. Concerns about the protection of privacy and confidentiality of medical information of
patients involved in clinical trials

19. Which may have implications on (medical) insurance coverage and/or employability

20. There can be the potential harmful abuse of the technology where individuals are using
the technique for enhancement rather than for therapeutic purpose. Tampering with
human genes / nature and bias handling of trial results

(c) Discuss on the different types of stem cells. [6]

1. Zygotic stem cells [Example : Fertilised zygote]

These are the cells in morula, which are formed during the first few divisions after
fertilisation/fusion of an egg and sperm.

Zygotic stem cells, which include the zygote and early embryo cells are totipotent.

They have the ability to differentiate into any cell type to form whole organisms and
hence are also pluripotent and multipotent.

NJC 2013 9648/03/SH2/H2/Biology/PrelimExam (for exchange)


1062
14

Functions
2. Zygotic stem cell has the potential to differentiate into all the cells that form the complete
organism plus the extra-embryonic tissues which include the placenta and amnion.

3. These cells give rise to form all cells in the body of the organism therefore they have
limited ability to self-renew.

4. Embryonic stem cells

Embryonic stem cells are pluripotent. They have the ability to differentiate into almost
any cell type to form any organ or type of cell and so are not totipotent but are multi-
potent.

They are obtained from the Inner Cell Mass on the fourth day of development where the
embryo forms into two layers, an outer layer (extra-embryonic membranes) and an inner
cell mass (ICM).

Functions
5. Inner cell mass (ICM) forms the three primary germ layers: ectoderm, endoderm and
mesoderm, which later form all the tissues of the human body.

6. Give rise to various organs in organism / multiple specialized cell types that make up the
heart, lung, skin and other tissues in the developing foetus.

7. Adult stem cells [E.g. Blood stem cells, Stromal Stem cells, Gut Epithelium stem cells]

Blood stem cells are multipotent. / found in each tissues or organs which have the ability
to differentiate into a limited range of cell type and so are not pluripotent or totipotent.

Found in bone marrow or blood/skin etc

Functions
8. Help to replenish worn-out or damaged cells / responsible for constant renewal of blood

9. Give rise to all the blood cell types

10. Properties pertaining to stem cells

--- End of Paper ---

NJC 2013 9648/03/SH2/H2/Biology/PrelimExam (for exchange)


1063

NANYANG JUNIOR COLLEGE


JC 2 PRELIMINARY EXAMINATIONS
Higher 2

CANDIDATE
NAME

CLASS

BIOLOGY 9648/01
Paper 1 Multiple Choice September 2013
1 hour 15 minutes
Additional Materials: Multiple Choice Answer Sheet

READ THESE INSTRUCTIONS FIRST

Write in soft pencil.


Do not use staples, paper clips, highlighters, glue or correction fluid.
Write your name and CT on the Answer Sheet in the spaces provided unless this has been done for you.

There are forty questions on this paper. Answer all questions. For each question there are four possible
answers A, B, C and D.
Choose the one you consider correct and record your choice in soft pencil on the separate Answer Sheet.

Read the instructions on the Answer Sheet very carefully.

Each correct answer will score one mark. A mark will not be deducted for a wrong answer.
Any rough working should be done in this booklet.
Calculators may be used.

This document consists of 16 printed pages.

[Turn over
1064
2
1 The diagram shows the ultrastructure of a eukaryotic cell.

Which organelle does not contain nucleic acid?

2 When not involved in protein synthesis, ribosomes exist as separate subunits.

What do these subunits consist of?

A mRNA and lipid


B mRNA and tRNA
C rRNA and lipid
D rRNA and protein

3 Which statement is true for cellulose, but not true for protein?

A It is found in cell surface membranes.


B It is synthesised from identical sub-units.
C It is used as an energy source.
D It may be a structural component.

H2 Biology/9648/01
1065
3
4 Diagrams 1, 2 and 3 show the structural formulae of three polysaccharides.

What are the names of these polysaccharides?

1 2 3
A amylose cellulose Glycogen
B amylose glycogen Amylose
C cellulose glycogen Cellulose
D glycogen amylose Amylose

5 The graph shows the activation energy of an enzyme-catalysed reaction and the same
reaction without a catalyst.

Which arrow shows the activation energy of the uncatalysed reaction?

H2 Biology/9648/01
1066
4
6 Which of the following statement about chromosome structure is true?

1 Linker DNA refers to the linear double-stranded DNA between adjacent


nucleosomes.
2 The 30 nm chromatin fibre has a solenoid structure.
3 The 300 nm chromatin fibre is attached to multiple locations on a central protein
scaffold.
4 Euchromatin is the more diffuse region of the interphase genetic material and is
transcriptionally inactive.

A 1 and 2
B 1 and 4
C 2 and 3
D 3 and 4

7 If there were 30 amino acids and DNA only contained three types of nitrogenous bases,
what would be the minimum number of bases per codon that could code for proteins?

A 3
B 4
C 6
D 8

8 What occurs as a result of base pairing during DNA replication?

A Both strands of the DNA have the same sequence of purines and pyrimidines.
B In each strand, the number of purine bases equals the number of pyrimidine bases.
C Purine bases in one strand become hydrogen-bonded to purine bases in the other
strand.
D The number of purine bases in one strand equals the number of pyrimidine bases in
the other strand.

H2 Biology/9648/01
1067
5
9 Part of the amino acid sequences in normal and sickle cell haemoglobin are shown.

normal haemoglobin sickle cell haemoglobin


thr-pro-glu-glu thr-pro-val-glu

Possible mRNA codons for these amino acids are:

glutamate (glu) GAA GAG proline (pro) CCU CCC


threonine (thr) ACU ACC valine (val) GUA GUG

Which tRNA molecule is not involved in the formation of this part of the sickle cell
haemoglobin?

A B C D

10 Which of the following is characteristic of the lytic cycle?

A Many bacterial cells containing viral DNA are produced.


B Viral DNA is incorporated into the host genome.
C The viral genome replicates without destroying the host.
D A large number of phages are released at a time.

11 A bacterium is infected with an experimentally constructed bacteriophage composed of the


T2 phage protein coat and T4 phage DNA. The new phages produced would have
________.

A T2 protein and T4 DNA


B T2 protein and T2 DNA
C T4 protein and T4 DNA
D a mixture of the DNA and proteins of both phages

H2 Biology/9648/01
1068
6
12 Which statement about the genomes of prokaryotes is correct?

A Prokaryotic chromosomes are sometimes called plasmids.


B Prokaryotic cells have multiple chromosomes, "packed" with a relatively large amount
of protein.
C Prokaryotic genomes are composed of linear DNA (that is, DNA existing in the form of
a line with two ends).
D The prokaryotic chromosome is not contained within a nucleus but, rather, is found at
the nucleoid region.

13 If the experimental population of E. coli lacks an F factor or F plasmid, and if bacteriophage


are excluded from the bacterial cultures, then which of these is a means by which beneficial
mutations might be transmitted horizontally to other E. coli cells?

A via conjugation
B via transduction
C via transformation
D both A and C above

14 The lactose operon is likely to be transcribed when _______.

A there is more glucose in the cell than lactose


B there is glucose but no lactose in the cell
C the cyclic AMP and lactose levels are both high within the cell
D the cAMP level is high and the lactose level is low

15 How is the trp operon different from the lac operon?

trp operon lac operon


A Produces polycistronic mRNA Produces monocistronic mRNA
B Repressor binds to operator Activator binds to operator
C Repressible Inducible
D Transcription occurs in the presence Transcription occurs in the absence
of tryptophan of glucose

H2 Biology/9648/01
1069
7
16 The diagram below depicts a lac operon of the wild type bacterial chromosome in E. coli.
The lacZ gene is regulated via the normal operator sequence (O+). However, the lacZ allele
is a recessive form (lacZ-) that does not code for beta-galactosidase.

Structural gene
for beta-galactosidase

This particular E. coli strain is conjugated with a mutant strain of E. coli carrying the P lac Oc
lacZ+ DNA sequence as shown below. In this sequence, the lacZ allele is normal (lacZ+)
but the operator (O) is mutated. The Oc mutant is unable to bind the lac repressor coded for
by lacI.
sequence W

Structural gene
for beta-galactosidase

How would the expression of the lacZ gene be regulated in the recipient cells after
homologous recombination between the wild type (recipient) and the mutant strain (donor)
involving sequence W?

A There will be normal regulation of lactose metabolism.


B There will be continuous expression of beta-galactosidase regardless of glucose level.
C The inability to synthese the lacZ gene product, beta-galactosidase.
D The lacZ gene is inducible, but unable to be repressed by high glucose levels.

17 Which of the following statements concerning proto-oncogenes is false?

A They can code for proteins associated with cell growth.


B They are produced by somatic mutations induced by carcinogenic substances.
C They can be involved in producing proteins for cell adhesion.
D They can code for proteins involved in cell division.

H2 Biology/9648/01
1070
8
18 Which of the following event(s) is/are necessary for the production of a malignant tumour?

1 activation of an oncogene in the cell


2 inactivation of tumour-suppressor genes within the cell
3 presence of mutagenic substances within the cell's environment
4 presence of a retrovirus within the cell

A 1 and 2
B 1 only
C 3 only
D 3 and 4

19 In Drosophila long wing and broad abdomen are dominant to vestigial wing and narrow
abdomen. A pure-breeding long-winged, broad-abdomened Drosophila is crossed with
pure-breeding vestigial-winged and narrow-abdomened Drosophila. Two of the F1
generation were crossed and the following results were produced in the F2 generation.

long wing, broad abdomen 482


vestigial wing, narrow abdomen 154

Which one of the following is illustrated by these results?

A autosomal linkage
B incomplete dominance
C Mendelian dihybrid inheritance
D Mendelian monohybrid inheritance

20 In cats, the genes controlling coat-colour are co-dominant and carried on the X
chromosomes. When a black female was mated with a ginger male the resulting litter
consisted of black male and tortoise-shell female kittens.

What phenotypic ratio would be expected in the F2 generation?

A 1 black male : 1 ginger male : 2 tortoise-shell females


B 1 black male : 1 ginger male : 1 tortoise-shell female : 1 black female
C 2 black males : 1 tortoise-shell female : 1 ginger female
D 2 black males : 1 tortoise-shell female : 1 black female

H2 Biology/9648/01
1071
9
21 The diagram shows the pedigree of a family carrying sex-linked allele for colour-blindness.

normal male
normal female
colour-blind male
colour-blind female

From which labelled member of his family did F inherit this disorder?

22 In guinea pigs, the allele R for rough coat is dominant over allele r for smooth coat and the
allele B for black fur is dominant over the allele b for white fur. The genes for fur colour and
texture are not linked.

Two guinea pigs with genotype RrBb were mated together and one of the offspring had a
rough and black coat.

What is the probability that this offspring was homozygous for both rough coat and black
fur?

A 1 in 16
B 1 in 9
C 2 in 16
D 2 in 9

23 Which combination would decrease the fluidity of a cell surface membrane?

Saturated fatty acids Non-saturated fatty acids Cholesterol


A decrease increase decrease
B decrease increase increase
C increase decrease decrease
D increase decrease increase

H2 Biology/9648/01
1072
10
24 The flow diagram shows some of the stages in respiration.

During which stages does oxidative decarboxyation occur?

A 1 and 3
B 2 and 3
C 3 and 4
D 4 and 5

H2 Biology/9648/01
1073
11
25 Six tubes containing preparations from animal tissue were set up as shown in the table.

tube Contents
1 glucose + homogenised cells
2 glucose + mitochondria
3 glucose + cytoplasm lacking organelles
4 pyruvate + homogenised cells
5 pyruvate + mitochondria
6 pyruvate + cytoplasm lacking organelles

After incubation, in which three tubes would carbon dioxide by produced?

A 1, 2 and 4
B 1, 4 and 5
C 2, 4 and 5
D 2, 4 and 6

26 Variegated leaves of a plant were supplied with radioactive carbon dioxide ( 14CO2) during
an experiment. Leaf X was kept in the dark and leaf Y was kept in the light.

At the end of the experiment, the radioactivity in the leaves was measured. The results (in
arbitrary units) are shown in the boxes in the diagram

What is the most likely explanation for the level of radioactivity found in the yellow zone of
leaf Y?

A Photosynthesis occurs but no storage of starch occurs in this zone.


B Photosynthesis proceeds slowly in the absence of chlorophylls a and b.
C Products of photosynthesis are transported into the yellow zone.
D Radioactive carbon dioxide diffuses into the leaf and accumulates there.

H2 Biology/9648/01
1074
12
27 The diagram shows an action potential.

time

resting
potential

At which point on the graph is the membrane most permeable to sodium ions?

28 What effect do sexual processes (meiosis and fertilization) have on the allelic frequencies in
a population?

A They tend to reduce the frequencies of deleterious alleles and increase the
frequencies of advantageous ones.
B They tend to increase the frequencies of deleterious alleles and reduce the
frequencies of advantageous ones.
C They tend to selectively combine favourable alleles into the same zygote but do not
change allelic frequencies.
D They have no effect on allelic frequencies.

29 As adults, certain species of whales possess baleen instead of teeth. Baleen is used to filter
the whales diet of planktonic animals from seawater. As embryos, baleen whales possess
teeth, which are later replaced by baleen. The teeth of embryonic baleen whales are
evidence that

A all whales are descendents of terrestrial mammals.


B baleen whales are descendents of toothed whales.
C baleen embryos pass through a stage when they resemble adult toothed whales.
D among ancient whales, baleen evolved before teeth.

H2 Biology/9648/01
1075
13
30 The population of the Malayan Colugo in Singapore shown below are descendants from a
small population of Colugos that were over-hunted and had their habitats destroyed post-
1819 when 90% of primary forests in Singapore were cleared.

Genetic variability in this population would be expected to be _______ due to _______.

A slight, a population bottleneck


B slight, the founder effect
C great, disruptive selection
D great, a population bottleneck

31 In using the average mutation rate of a gene as a molecular clock, which of the following
would act to make the clock underestimate the true amount of time that had elapsed since
two related species diverged from their common ancestor?

A mutation

A to the second base of a nucleotide codon.


B that is synonymous (a.k.a. a silent mutation).
C that returns a mutated nucleotide to its original state.
D substitutes a different amino acid in place of the original amino acid.

32 Which of the following statements about restriction enzymes is false?

A They are protein in nature.


B They are naturally produced by some organisms.
C Different restriction enzymes recognize and cut at different sites.
D Restriction enzymes are not involved in establishing a cDNA library.

H2 Biology/9648/01
1076
14
33 The diagram below shows the general procedure in the cloning of an animal gene to study
gene expression.

Which of the following is true of structures A and B?

Structure A contains Structure B contains


A a genetic code an origin of replication
B a DNA probe selectable markers
C an origin of replication Ribonucleotides
D Ribonucleotides enhancer sequences

34 The diagram below shows a plasmid carrying two selectable markers which code for
resistance to kanamycin and neomycin. The restriction site is also shown.

DraI
kanamycin neomycin
resistance gene resistance gene

A eukaryotic gene is produced by the restriction endonuclease DraI. This gene was inserted
and ligated into a cut plasmid containing compatible ends. The E. coli transformation
mixture was grown in four different media: nutrient medium plus kanamycin, nutrient
medium plus neomycin, nutrient medium plus kanamycin and neomycin, and nutrient
medium containing no antibiotics.

The bacteria which has taken up the religated plasmid would grow in

A nutrient medium containing kanamycin only.


B nutrient medium containing neomycin only.
C nutrient medium containing both kanamycin and neomycin.
D all four types of medium.

H2 Biology/9648/01
1077
15
35 A sample of DNA is cut at the positions indicated (marked with ) below.

When the fragments are loaded and run on an electrophoresis gel, which lane (1 to 5) has
the pattern that you would expect?

A 1
B 2
C 3
D 4

36 Southern blot can be used to detect all of the following except

A whether or not a gene is being expressed.


B the presence/absence of a specific DNA sequence.
C the size of restriction fragment that encodes a particular DNA sequence.
D the differences in DNA sequences from different organisms.

37 The following are advantages of transgenics except:

A A gene for a desirable characteristic can be identified and cloned.


B All the beneficial characteristics of an existing variety can be kept and just the desired
new gene can be added.
C Sexual reproduction is necessary.
D Transgenics is much faster than conventional breeding.

H2 Biology/9648/01
1078
16
38 What are the normal functions of stem cells in a living human?

Functions
A differentiating into producing insulin in cells that can
many kinds of cells in pancreas damaged differentiate into bone
a 3 -5 by type 1 diabetes cells in the skeleton
day old human
embryo
B differentiating into producing red blood cells that can
many kinds of cells in cells worn out by differentiate into
a 3 -5 normal wear and tear cardiac muscles in
day old human the heart
embryo
C differentiating into producing dopamine cells that can
only one kind of cells in brains of people differentiate into
in a 3 -5 with Parkinsons cartilage cells in the
day old human disease joint
embryo
D differentiating into producing cells that can
only one kind of cells differentiated cells differentiate into
in a 3 -5 that can be used to nerve cells in the
day old human screen new drugs brain
embryo

39 Which of the following disorders would gene therapy be least effective?

A Cystic fibrosis
B Haemophilia
C Hungtingtons disease
D Sickle cell anaemia

40 What are some similarities between all cancer cells and all stem cells?

A They replicate indefinitely, are non-differentiated and can be found in various parts of
the body.
B They replicate indefinitely, lack cell-cell adhesion and are able to move from one
location in the body to another.
C They lack contact inhibition, are non-differentiated and are regulated by molecular
signals.
D They are able to move from one location in the body to another, are regulated by
molecular signals and can be found in various parts of the body.

H2 Biology/9648/01
1079

NANYANG JUNIOR COLLEGE


JC 2 PRELIMINARY EXAMINATIONS
Higher 2

CANDIDATE
NAME MARK SCHEME

CLASS

BIOLOGY 9648/01
Paper 1 Multiple Choice September 2013
1 hour 15 minutes
Additional Materials: Multiple Choice Answer Sheet

READ THESE INSTRUCTIONS FIRST

Write in soft pencil.


Do not use staples, paper clips, highlighters, glue or correction fluid.
Write your name and CT on the Answer Sheet in the spaces provided unless this has been done for you.

There are forty questions on this paper. Answer all questions. For each question there are four possible
answers A, B, C and D.
Choose the one you consider correct and record your choice in soft pencil on the separate Answer Sheet.

Read the instructions on the Answer Sheet very carefully.

Each correct answer will score one mark. A mark will not be deducted for a wrong answer.
Any rough working should be done in this booklet.
Calculators may be used.

This document consists of 16 printed pages.

[Turn over
1080
2
1 The diagram shows the ultrastructure of a eukaryotic cell.

Which organelle does not contain nucleic acid? A

2 When not involved in protein synthesis, ribosomes exist as separate subunits.

What do these subunits consist of?

A mRNA and lipid


B mRNA and tRNA
C rRNA and lipid
D rRNA and protein

3 Which statement is true for cellulose, but not true for protein?

A It is found in cell surface membranes.


B It is synthesised from identical sub-units.
C It is used as an energy source.
D It may be a structural component.

H2 Biology/9648/01
1081
3
4 Diagrams 1, 2 and 3 show the structural formulae of three polysaccharides.

What are the names of these polysaccharides?

1 2 3
A amylose cellulose Glycogen
B amylose glycogen Amylose
C cellulose glycogen Cellulose
D glycogen amylose Amylose

5 The graph shows the activation energy of an enzyme-catalysed reaction and the same
reaction without a catalyst.

Which arrow shows the activation energy of the uncatalysed reaction? C

H2 Biology/9648/01
1082
4
6 Which of the following statement about chromosome structure is true?

1 Linker DNA refers to the linear double-stranded DNA between adjacent


nucleosomes.
2 The 30 nm chromatin fibre has a solenoid structure.
3 The 300 nm chromatin fibre is attached to multiple locations on a central protein
scaffold.
4 Euchromatin is the more diffuse region of the interphase genetic material and is
transcriptionally inactive.

A 1 and 2
B 1 and 4
C 2 and 3
D 3 and 4

7 If there were 30 amino acids and DNA only contained three types of nitrogenous bases,
what would be the minimum number of bases per codon that could code for proteins?

A 3
B 4
C 6
D 8

8 What occurs as a result of base pairing during DNA replication?

A Both strands of the DNA have the same sequence of purines and pyrimidines.
B In each strand, the number of purine bases equals the number of pyrimidine bases.
C Purine bases in one strand become hydrogen-bonded to purine bases in the other
strand.
D The number of purine bases in one strand equals the number of pyrimidine bases in
the other strand.

H2 Biology/9648/01
1083
5
9 Part of the amino acid sequences in normal and sickle cell haemoglobin are shown.

normal haemoglobin sickle cell haemoglobin


thr-pro-glu-glu thr-pro-val-glu

Possible mRNA codons for these amino acids are:

glutamate (glu) GAA GAG proline (pro) CCU CCC


threonine (thr) ACU ACC valine (val) GUA GUG

Which tRNA molecule is not involved in the formation of this part of the sickle cell
haemoglobin?

A B C D

10 Which of the following is characteristic of the lytic cycle?

A Many bacterial cells containing viral DNA are produced.


B Viral DNA is incorporated into the host genome.
C The viral genome replicates without destroying the host.
D A large number of phages are released at a time.

11 A bacterium is infected with an experimentally constructed bacteriophage composed of the


T2 phage protein coat and T4 phage DNA. The new phages produced would have
________.

A T2 protein and T4 DNA


B T2 protein and T2 DNA
C T4 protein and T4 DNA
D a mixture of the DNA and proteins of both phages

H2 Biology/9648/01
1084
6
12 Which statement about the genomes of prokaryotes is correct?

A Prokaryotic chromosomes are sometimes called plasmids.


B Prokaryotic cells have multiple chromosomes, "packed" with a relatively large amount
of protein.
C Prokaryotic genomes are composed of linear DNA (that is, DNA existing in the form of
a line with two ends).
D The prokaryotic chromosome is not contained within a nucleus but, rather, is found at
the nucleoid region.

13 If the experimental population of E. coli lacks an F factor or F plasmid, and if bacteriophage


are excluded from the bacterial cultures, then which of these is a means by which beneficial
mutations might be transmitted horizontally to other E. coli cells?

A via conjugation
B via transduction
C via transformation
D both A and C above

14 The lactose operon is likely to be transcribed when _______.

A there is more glucose in the cell than lactose


B there is glucose but no lactose in the cell
C the cyclic AMP and lactose levels are both high within the cell
D the cAMP level is high and the lactose level is low

15 How is the trp operon different from the lac operon?

trp operon lac operon


A Produces polycistronic mRNA Produces monocistronic mRNA
B Repressor binds to operator Activator binds to operator
C Repressible Inducible
D Transcription occurs in the presence Transcription occurs in the absence
of tryptophan of glucose

H2 Biology/9648/01
1085
7
16 The diagram below depicts a lac operon of the wild type bacterial chromosome in E. coli.
The lacZ gene is regulated via the normal operator sequence (O+). However, the lacZ allele
is a recessive form (lacZ-) that does not code for beta-galactosidase.

Structural gene
for beta-galactosidase

This particular E. coli strain is conjugated with a mutant strain of E. coli carrying the P lac Oc
lacZ+ DNA sequence as shown below. In this sequence, the lacZ allele is normal (lacZ+)
but the operator (O) is mutated. The Oc mutant is unable to bind the lac repressor coded for
by lacI.
sequence W

Structural gene
for beta-galactosidase

How would the expression of the lacZ gene be regulated in the recipient cells after
homologous recombination between the wild type (recipient) and the mutant strain (donor)
involving sequence W?

A There will be normal regulation of lactose metabolism.


B There will be continuous expression of beta-galactosidase regardless of glucose level.
C The inability to synthese the lacZ gene product, beta-galactosidase.
D The lacZ gene is inducible, but unable to be repressed by high glucose levels.

17 Which of the following statements concerning proto-oncogenes is false?

A They can code for proteins associated with cell growth.


B They are produced by somatic mutations induced by carcinogenic substances.
C They can be involved in producing proteins for cell adhesion.
D They can code for proteins involved in cell division.

H2 Biology/9648/01
1086
8
18 Which of the following event(s) is/are necessary for the production of a malignant tumour?

1 activation of an oncogene in the cell


2 inactivation of tumour-suppressor genes within the cell
3 presence of mutagenic substances within the cell's environment
4 presence of a retrovirus within the cell

A 1 and 2
B 1 only
C 3 only
D 3 and 4

19 In Drosophila long wing and broad abdomen are dominant to vestigial wing and narrow
abdomen. A pure-breeding long-winged, broad-abdomened Drosophila is crossed with
pure-breeding vestigial-winged and narrow-abdomened Drosophila. Two of the F1
generation were crossed and the following results were produced in the F2 generation.

long wing, broad abdomen 482


vestigial wing, narrow abdomen 154

Which one of the following is illustrated by these results?

A autosomal linkage
B incomplete dominance
C Mendelian dihybrid inheritance
D Mendelian monohybrid inheritance

20 In cats, the genes controlling coat-colour are co-dominant and carried on the X
chromosomes. When a black female was mated with a ginger male the resulting litter
consisted of black male and tortoise-shell female kittens.

What phenotypic ratio would be expected in the F2 generation?

A 1 black male : 1 ginger male : 2 tortoise-shell females


B 1 black male : 1 ginger male : 1 tortoise-shell female : 1 black female
C 2 black males : 1 tortoise-shell female : 1 ginger female
D 2 black males : 1 tortoise-shell female : 1 black female

H2 Biology/9648/01
1087
9
21 The diagram shows the pedigree of a family carrying sex-linked allele for colour-blindness.

normal male
normal female
colour-blind male
colour-blind female

From which labelled member of his family did F inherit this disorder? B

22 In guinea pigs, the allele R for rough coat is dominant over allele r for smooth coat and the
allele B for black fur is dominant over the allele b for white fur. The genes for fur colour and
texture are not linked.

Two guinea pigs with genotype RrBb were mated together and one of the offspring had a
rough and black coat.

What is the probability that this offspring was homozygous for both rough coat and black
fur?

A 1 in 16
B 1 in 9
C 2 in 16
D 2 in 9

23 Which combination would decrease the fluidity of a cell surface membrane?

Saturated fatty acids Non-saturated fatty acids Cholesterol


A decrease increase decrease
B decrease increase increase
C increase decrease decrease
D increase decrease increase

H2 Biology/9648/01
1088
10
24 The flow diagram shows some of the stages in respiration.

During which stages does oxidative decarboxyation occur?

A 1 and 3
B 2 and 3
C 3 and 4
D 4 and 5

H2 Biology/9648/01
1089
11
25 Six tubes containing preparations from animal tissue were set up as shown in the table.

tube Contents
1 glucose + homogenised cells
2 glucose + mitochondria
3 glucose + cytoplasm lacking organelles
4 pyruvate + homogenised cells
5 pyruvate + mitochondria
6 pyruvate + cytoplasm lacking organelles

After incubation, in which three tubes would carbon dioxide by produced?

A 1, 2 and 4
B 1, 4 and 5
C 2, 4 and 5
D 2, 4 and 6

26 Variegated leaves of a plant were supplied with radioactive carbon dioxide ( 14CO2) during
an experiment. Leaf X was kept in the dark and leaf Y was kept in the light.

At the end of the experiment, the radioactivity in the leaves was measured. The results (in
arbitrary units) are shown in the boxes in the diagram

What is the most likely explanation for the level of radioactivity found in the yellow zone of
leaf Y?

A Photosynthesis occurs but no storage of starch occurs in this zone.


B Photosynthesis proceeds slowly in the absence of chlorophylls a and b.
C Products of photosynthesis are transported into the yellow zone.
D Radioactive carbon dioxide diffuses into the leaf and accumulates there.

H2 Biology/9648/01
1090
12
27 The diagram shows an action potential.

time

resting
potential

At which point on the graph is the membrane most permeable to sodium ions? A

28 What effect do sexual processes (meiosis and fertilization) have on the allelic frequencies in
a population?

A They tend to reduce the frequencies of deleterious alleles and increase the
frequencies of advantageous ones.
B They tend to increase the frequencies of deleterious alleles and reduce the
frequencies of advantageous ones.
C They tend to selectively combine favourable alleles into the same zygote but do not
change allelic frequencies.
D They have no effect on allelic frequencies.

29 As adults, certain species of whales possess baleen instead of teeth. Baleen is used to filter
the whales diet of planktonic animals from seawater. As embryos, baleen whales possess
teeth, which are later replaced by baleen. The teeth of embryonic baleen whales are
evidence that

A all whales are descendents of terrestrial mammals.


B baleen whales are descendents of toothed whales.
C baleen embryos pass through a stage when they resemble adult toothed whales.
D among ancient whales, baleen evolved before teeth.

H2 Biology/9648/01
1091
13
30 The population of the Malayan Colugo in Singapore shown below are descendants from a
small population of Colugos that were over-hunted and had their habitats destroyed post-
1819 when 90% of primary forests in Singapore were cleared.

Genetic variability in this population would be expected to be _______ due to _______.

A slight, a population bottleneck


B slight, the founder effect
C great, disruptive selection
D great, a population bottleneck

31 In using the average mutation rate of a gene as a molecular clock, which of the following
would act to make the clock underestimate the true amount of time that had elapsed since
two related species diverged from their common ancestor?

A mutation

A to the second base of a nucleotide codon.


B that is synonymous (a.k.a. a silent mutation).
C that returns a mutated nucleotide to its original state.
D substitutes a different amino acid in place of the original amino acid.

32 Which of the following statements about restriction enzymes is false?

A They are protein in nature.


B They are naturally produced by some organisms.
C Different restriction enzymes recognize and cut at different sites.
D Restriction enzymes are not involved in establishing a cDNA library.

H2 Biology/9648/01
1092
14
33 The diagram below shows the general procedure in the cloning of an animal gene to study
gene expression.

Which of the following is true of structures A and B?

Structure A contains Structure B contains


A a genetic code an origin of replication
B a DNA probe selectable markers
C an origin of replication Ribonucleotides
D Ribonucleotides enhancer sequences

34 The diagram below shows a plasmid carrying two selectable markers which code for
resistance to kanamycin and neomycin. The restriction site is also shown.

DraI
kanamycin neomycin
resistance gene resistance gene

A eukaryotic gene is produced by the restriction endonuclease DraI. This gene was inserted
and ligated into a cut plasmid containing compatible ends. The E. coli transformation
mixture was grown in four different media: nutrient medium plus kanamycin, nutrient
medium plus neomycin, nutrient medium plus kanamycin and neomycin, and nutrient
medium containing no antibiotics.

The bacteria which has taken up the religated plasmid would grow in

A nutrient medium containing kanamycin only.


B nutrient medium containing neomycin only.
C nutrient medium containing both kanamycin and neomycin.
D all four types of medium.

H2 Biology/9648/01
1093
15
35 A sample of DNA is cut at the positions indicated (marked with ) below.

When the fragments are loaded and run on an electrophoresis gel, which lane (1 to 5) has
the pattern that you would expect?

A 1
B 2
C 3
D 4

36 Southern blot can be used to detect all of the following except

A whether or not a gene is being expressed.


B the presence/absence of a specific DNA sequence.
C the size of restriction fragment that encodes a particular DNA sequence.
D the differences in DNA sequences from different organisms.

37 The following are advantages of transgenics except:

A A gene for a desirable characteristic can be identified and cloned.


B All the beneficial characteristics of an existing variety can be kept and just the desired
new gene can be added.
C Sexual reproduction is necessary.
D Transgenics is much faster than conventional breeding.

H2 Biology/9648/01
1094
16
38 What are the normal functions of stem cells in a living human?

Functions
A differentiating into producing insulin in cells that can
many kinds of cells in pancreas damaged differentiate into bone
a 3 -5 by type 1 diabetes cells in the skeleton
day old human
embryo
B differentiating into producing red blood cells that can
many kinds of cells in cells worn out by differentiate into
a 3 -5 normal wear and tear cardiac muscles in
day old human the heart
embryo
C differentiating into producing dopamine cells that can
only one kind of cells in brains of people differentiate into
in a 3 -5 with Parkinsons cartilage cells in the
day old human disease joint
embryo
D differentiating into producing cells that can
only one kind of cells differentiated cells differentiate into
in a 3 -5 that can be used to nerve cells in the
day old human screen new drugs brain
embryo

39 Which of the following disorders would gene therapy be least effective?

A Cystic fibrosis
B Haemophilia
C Hungtingtons disease
D Sickle cell anaemia

40 What are some similarities between all cancer cells and all stem cells?

A They replicate indefinitely, are non-differentiated and can be found in various parts of
the body.
B They replicate indefinitely, lack cell-cell adhesion and are able to move from one
location in the body to another.
C They lack contact inhibition, are non-differentiated and are regulated by molecular
signals.
D They are able to move from one location in the body to another, are regulated by
molecular signals and can be found in various parts of the body.

H2 Biology/9648/01
1095

NANYANG JUNIOR COLLEGE


JC 2 PRELIMINARY EXAMINATIONS
Higher 2

CANDIDATE
NAME

CLASS

BIOLOGY 9648/02
Paper 2 Core Paper September 2013
2 hours
Additional Materials: Answer Paper

READ THESE INSTRUCTIONS FIRST

Write your name and CT on all the work you hand in.
Write in dark blue or black pen on both sides of the paper.
You may use soft pencil for any diagrams, graphs or rough working.
Do no use staples, paper clips, highlighters, glue or correction fluid.

Section A

Answer all questions.

Section B For Examiners Use

Answer any one question. Section A

2
At the end of the examination, fasten all your work securely together.
The number of marks is given in brackets [ ] at the end of each question or part 3
question.
4

Section B

Total

This document consists of 18 printed pages and no blank page.


[Turn over

H2 Biology / 9648 / 02
1096
2
Section A
Answer all the questions in this section.

1 The electron micrograph in Fig. 1.1 shows an acinar cell found in the pancreas.

Fig. 1.1

(a) With reference to the Fig. 1.1, suggest the principal function of the pancreatic acinar
cell. Give two reasons to explain your answer.

[3]

H2 Biology / 9648 / 02
1097
3
Fig. 1.2 shows two different structures, A and B, found in an animal cell.

Fig. 1.2

(b) With reference to the Fig. 1.2, describe three ways in which A is similar to B.

[3]

H2 Biology / 9648 / 02
1098
4
Scientists isolated mitochondria from the liver cells. They broke the cells open in an ice-cold,
isotonic solution. They then used a centrifuge to separate the cell organelles. Fig. 1.3
illustrates the steps that take place in the process of centrifugation.

Fig. 1.3
(c) State and explain which pellet contains mitochondria.

[2]
(d) Suggest why the solution used was ice-cold and isotonic.

[2]

People with mitochondrial disease have mitochondria that do not function properly. These
disorders may be caused by mutations in the mitochondrial DNA or in the nuclear genes
that code for mitochondrial components.

(e) Explain why a person with mitochondrial disease can only exercise for a short time.

[2]
[Total: 12]

H2 Biology / 9648 / 02
1099
5
2 Prokaryotes were first discovered in the late 1600s by Antony van Leeuwenhoek, using the
microscope he invented. With the advancements in science, scientists have been able to
unravel the mysteries between prokaryotes and eukaryotes.
Fig. 2.1 (a) and (b) shows protein synthesis carried out by the two types of cells.

Fig. 2.1

(a) Describe the structure of a RNA molecule that is not shown in Fig. 2.1 in relation to its
function.

[3]
(b) Describe the differences between prokaryotes and eukaryotes with respect to gene
expression at the following levels:
(i) Chromatin level

[2]
(ii) Transcriptional level

[3]

H2 Biology / 9648 / 02
1100
6
During research into the mechanism of DNA replication, bacteria were grown for many
generations in a medium containing only the heavy isotope of nitrogen, 15N. This resulted
in all the DNA molecules containing only 15N. This is illustrated in Fig. 2.2 below.
100
% DNA molecules

75

50

25

0
14 14
DNA containing N only DNA containing N/15N DNA containing 15
N only

Fig. 2.2

Step 1: These bacteria were then grown in a medium containing only light nitrogen, 14
N.
After the time taken for the DNA to replicate once, the DNA was analysed.

Step 2: The bacteria continued to grow in the light nitrogen, 14N, medium until the DNA
had replicated once more. The DNA molecules were then analysed.

Step 3: The bacteria was allowed to grow in the light nitrogen, 14


N, medium for another
round of DNA replication.

(c) Complete the bar chart below to indicate the expected results of the composition of
these DNA molecules after step 3.
[1]
100
% DNA molecules

75

50

25

0
14 14
DNA containing N only DNA containing N/15N DNA containing 15
N only

H2 Biology / 9648 / 02
1101
7

DNA replication is a process which involves DNA polymerase and the deoxyribonucleotides
A, T, C and G, while transcription involves RNA polymerase and the ribonucleotides A, U, C
and G.

(d) Describe two other differences and two similarities between DNA replication and
transcription.

[4]
[Total: 13]

H2 Biology / 9648 / 02
1102
8
3 Fig. 3.1 is an electron micrograph showing T4 bacteriophages invading a bacterium.

Fig. 3.1

(a) Clearly label the base plate of one T4 phage in Fig. 3.1. [1]

(b) Explain how T4 may mediate the transfer of a random piece of this bacterias genetic
material to another bacteria.

[4]
(c) Briefly describe the structure of bacterial chromosomes.

[2]

H2 Biology / 9648 / 02
1103
9
Some genes in bacteria are organized into operons, such as the lac operon. Transcription
of the lac operon in E. coli is dependent on the presence or absence of food molecules such
as glucose and lactose.

(d) Under normal conditions, predict the transcription of the operon in the presence or
absence of these two substrate molecules by writing High or Low in the table below.

Lactose Glucose Transcription


+ +
+ -
- +
- -
[1]

(e) Mutation XX-6 deletes the DNA binding site from the CAP protein, but leaves the rest
of the operon unaffected. Predict the effect on transcription under these four conditions
by writing High or Low in the table below.

Lactose Glucose Transcription


+ +
+ -
- +
- -
[1]
(f) Another mutation on a separate site gives the following results.
Lactose Glucose Transcription
+ + Low
+ - High
- + Low
- - High

Suggest the mutation that could have occurred and explain how it would have given rise
to the above results.

[2]
[Total: 11]

H2 Biology / 9648 / 02
1104
10
4 A woman's risk of developing breast cancer is greatly increased if she inherits BRCA1 gene
mutation. The role of BRCA1 protein is shown in Fig. 4.1.

DNA Repair &


Cell cycle Arrest

Fig. 4.1

(a) With reference to Fig. 4.1,


(i) State the class of genes that BRCA1 belongs to.

[1]

H2 Biology / 9648 / 02
1105
11
(ii) Explain how mutations in BRCA1 gene can contribute to cancer.

[3]
(iii) The expression of BRCA1 gene can be furthered controlled at post-translational
level. Explain how this can occur.

[2]

(b) The HER2 gene codes for a growth factor receptor. When the growth factor binds to
HER2 protein (the growth factor receptor), the signal is given for the cell to start dividing.
Gene amplification of HER2 gene could also result in breast cancer.
(i) Describe a process which results in gene amplification.

[2]
(ii) Suggest how amplification of HER2 could result in breast cancer.

[2]
[Total: 10]

H2 Biology / 9648 / 02
1106
12
5 Rickets is a childhood disorder involving the softening and weakening of bones in humans.
It is usually caused by a lack of vitamin D, calcium ions or phosphate ions. A rare form of
rickets that cannot be successfully treated with vitamin D therapy is caused by an allele on
the X chromosome that arose as a result of a gene mutation.

(a) Explain what is meant by the term gene mutation.

[2]
(b) Fig. 5.1 below illustrates how this rare form of rickets is inherited in an afflicted family.

Fig. 5.1

(i) With reference to Fig. 5.1, state and explain the pattern of inheritance of this form
of rickets.

[3]
(ii) Using appropriate symbols, state the genotypes of the following individuals.
1

10
[2]

H2 Biology / 9648 / 02
1107
13
(iii) Couple 6 and 7 decided to have another son. Draw a genetic diagram to determine
the probability that their second son will suffer from rickets.

[3]

H2 Biology / 9648 / 02
1108
14
Colour blindness and haemophilia are inherited sex-linked recessive conditions in humans. Fig.
5.2 shows two generations of a family in which these conditions are inherited.

Fig. 5.2

Suggest how the genotype of S has arisen.

[2]
[Total: 12]

H2 Biology / 9648 / 02
1109
15
6 Fig. 6.1 below shows the cell signalling pathway in smooth muscle cells in response to the
hormone Epinephrine, resulting in smooth muscle relaxation of blood vessels (vasodilation).

activated
Epinephrine
adenylate cyclase

-adrenergic
receptor

cyclic AMP

inactive protein kinase A (PKA) activated PKA

Responses of target cell:


Smooth muscle relaxation,
vasodilation

Fig. 6.1
(a) State the class of receptors that the -adrenergic receptor belongs to.

[1]
(b) With reference to Fig. 6.1,
(i) describe how adenylate cyclase is activated.

[3]
(ii) identify the stage where signal amplification is observed and explain how the signal
is amplified in that stage.

[2]

H2 Biology / 9648 / 02
1110
16
(c) Explain how the structure of the -adrenergic receptor allows it to carry out its function
as a receptor.

[3]
(d) A drug was designed to ensure that permanent vasodilation occurs. Suggest how this
drug might work within the cell signalling pathway in Fig. 6.1.

[2]
[Total: 11]

H2 Biology / 9648 / 02
1111
17
7 Scientists studying the evolutionary history of the Human Immunodeficiency Virus (HIV)
found that it is closely related to the Simian Immunodeficiency Virus (SIV) found in certain
species of monkeys. The phylogenetic tree in Fig. 7.1 shows part of the evolutionary
relationships between these viruses. A common ancestral virus is believed to have evolved
into strains that infected chimpanzees (SIV). Over time, new strains emerged which infect
humans.

Fig. 7.1

(a) Explain the significance of the branch points in the phylogenetic tree shown in Fig. 7.1.

[2]

(b) Explain how molecular homology could be used to support the claim that HIV and SIV
share an evolutionary relationship.

[2]

H2 Biology / 9648 / 02
1112
18
(c) State two advantages of using molecular techniques in classification.

[2]

SIV infections in their natural hosts appear in many cases to be non-pathogenic (non-
disease causing). Certain individuals within the human population have also shown similar
characteristics when exposed to HIV. These individuals continue to test negative for HIV
despite being repeatedly exposed to HIV.
Investigations have found a mutation present in the CCR5 gene in these individuals. CCR5
is a co-receptor that facilitates the entry of HIV into the T-cell after gp120 (present on the
viral envelope) binds with the CD4 receptor.

(d) Suggest and explain how these individuals continue to resist HIV infection.

[2]

(e) Assuming that humans do not take active steps to prevent HIV infection, explain how
the human population could possibly evolve to not be affected by HIV.

[3]
[Total: 11]

H2 Biology / 9648 / 02
1113
19
Section B
Answer one question.

Write your answers on the separate answer paper provided.


Your answers should be illustrated by large, clearly labelled diagrams, where appropriate.
Your answers must be in continuous prose, where appropriate.
Your answers must be set out in sections (a), (b) etc., as indicated in the question.

8 (a) Outline how the endomembrane system is involved in synaptic transmission. [8]
(b) Describe the main stages of the Calvin cycle [8]
(c) Contrast the structure of cellulose with that of collagen and suggest why plant
cells have cellulose but not collagen. [4]

9 (a) Compare and contrast the structures of influenza and HIV. [8]
(b) Describe how HIV infects host cells and reproduces. [8]
(c) Briefly explain how a person infected only once by one strain of HIV may
eventually have many strains of HIV. [4]

H2 Biology / 9648 / 02
1114

NANYANG JUNIOR COLLEGE


JC 2 PRELIMINARY EXAMINATIONS
Higher 2

CANDIDATE
NAME
Mark Scheme

CLASS

BIOLOGY 9648/02
Paper 2 Core Paper September 2013
2 hours
Additional Materials: Answer Paper

READ THESE INSTRUCTIONS FIRST

Write your name and CT on all the work you hand in.
Write in dark blue or black pen on both sides of the paper.
You may use soft pencil for any diagrams, graphs or rough working.
Do no use staples, paper clips, highlighters, glue or correction fluid.

Section A

Answer all questions.

Section B For Examiners Use

Answer any one question. Section A

2
At the end of the examination, fasten all your work securely together.
The number of marks is given in brackets [ ] at the end of each question or part 3
question.
4

Section B

Total

This document consists of 18 printed pages and no blank page.


[Turn over

H2 Biology / 9648 / 02
1115
2
Section A
Answer all the questions in this section.

1 The electron micrograph in Fig. 1.1 shows an acinar cell found in the pancreas.

Fig. 1.1

(a) With reference to the Fig. 1.1, suggest the principal function of the pancreatic acinar
cell. Give two reasons to explain your answer.
secretion of pancreatic / digestive enzymes;

ANY TWO

presence of large amount of rER, which is the site where pancreatic enzymes are
synthesized;

presence of large number of secretory vesicles for exocytosis to release of pancreatic


enzymes outside the cell;

presence of mitochondria which synthesizes ATP for activation of amino acids /


Formation of peptide bonds (at ribosomes) / movement of vesicles towards cell
surface membrane for exocytosis of pancreatic enzymes ;

[3]

H2 Biology / 9648 / 02
1116
3
Fig. 1.2 shows two different structures, A and B, found in an animal cell.

Fig. 1.2

(b) With reference to the Fig. 1.2, describe three ways in which A is similar to B.
Both are bound by a single membrane / single membrane-bound structures.

Both consist of membranous sacs called cisternae, in which newly synthesized


proteins undergo processing.

Both give rise to membrane-bound vesicles which are involved in transport of newly
synthesized proteins within the cell.

[3]

H2 Biology / 9648 / 02
1117
4
Scientists isolated mitochondria from the liver cells. They broke the cells open in an ice-cold,
isotonic solution. They then used a centrifuge to separate the cell organelles. Fig. 1.3
illustrates the steps that take place in the process of centrifugation.

Fig. 1.3
(c) State and explain which pellet contains mitochondria.
Pellet B ;

Mitochondria is the next biggest organelle after the nucleus ;


[2]
(d) Suggest why the solution used was ice-cold and isotonic.
Ice cold is to reduce enzyme activity / cellular preservation;

Isotonic to prevent net movement of water so organelle will not burst or shrivel /
maintain organelles integrity;
[2]

People with mitochondrial disease have mitochondria that do not function properly. These
disorders may be caused by mutations in the mitochondrial DNA or in the nuclear genes
that code for mitochondrial components.

(e) Explain why a person with mitochondrial disease can only exercise for a short time.
Mitochondria use aerobic respiration to synthesize ATP ;

ATP is needed for muscular contractions ;

[2]
[Total: 12]

H2 Biology / 9648 / 02
1118
5
2 Prokaryotes were first discovered in the late 1600s by Antony van Leeuwenhoek, using the
microscope he invented. With the advancements in science, scientists have been able to
unravel the mysteries between prokaryotes and eukaryotes.
Fig. 2.1 (a) and (b) shows protein synthesis carried out by the two types of cells.

Fig. 2.1

(a) Describe the structure of a RNA molecule that is not shown in Fig. 2.1 in relation to its
function.
Transfer RNA is a single-stranded RNA /, which is folded into a clover leaf structure
then into its three-dimensional structure + to bind to ribosome/ to fit into EPA site of
ribosome;

tRNA possess an anticodon which base pairs complementarily with codon on mRNA;

tRNA has an acceptor stem / 3 end for amino acid attachment, thus allowing tRNA to
transfer the amino acid to the ribosome during translation;

@ rRNA is single stranded and a component of ribosomes;


[3]
(b) Describe the differences between prokaryotes and eukaryotes with respect to gene
expression at the following levels:
(i) Chromatin level
In eukaryotes, histone acetylation neutralizes positive charge and DNA more
accessible to RNA polymerase for transcription
whereas in prokaryotes, packing of prokaryotic DNA do not involve histone proteins /
histone modification does not occur;

In eukaryotes, DNA methylation attracts enzymes which increase condensation and


decreased rate of transcription resulting in gene silencing
whereas in prokaryotes, DNA methylation discriminates between foreign and
endogenous DNA for restriction endonucleases;

[2]

H2 Biology / 9648 / 02
1119
6
(ii) Transcriptional level
In eukaryotic, TATA binding proteins/Transcription factors binds to TATA
box/promoter recruiting RNA polymerase and basal TF for the assembly of
transcription initiation complex
In prokaryotes: Prokaryotic RNA polymerase can bind directly to core promoter
without aid of TF, resulting in basal rate of transcription of operon

In eukaryotes, binding of activators to enhancers loop the DNA and increases rate of
transcription
In prokaryotes, usually lack distal control elements but contains CAP site for binding of
CAP and increases rate of transcription

In eukaryotes, binding of repressors to silencer decreases rate of transcription


In prokaryotes, binding of repressor to operator site within operon prevents
transcription allowing inducibility of operon
[3]

H2 Biology / 9648 / 02
1120
7
During research into the mechanism of DNA replication, bacteria were grown for many
generations in a medium containing only the heavy isotope of nitrogen, 15N. This resulted
in all the DNA molecules containing only 15N. This is illustrated in Fig. 2.2 below.
100
% DNA molecules

75

50

25

0
14 14
DNA containing N only DNA containing N/15N DNA containing 15
N only

Fig. 2.2

Step 1: These bacteria were then grown in a medium containing only light nitrogen, 14
N.
After the time taken for the DNA to replicate once, the DNA was analysed.

Step 2: The bacteria continued to grow in the light nitrogen, 14N, medium until the DNA
had replicated once more. The DNA molecules were then analysed.

Step 3: The bacteria was allowed to grow in the light nitrogen, 14


N, medium for another
round of DNA replication.

(c) Complete the bar chart below to indicate the expected results of the composition of
these DNA molecules after step 3.
[1]
100
% DNA molecules

75

50

25

0
14 14
DNA containing N only DNA containing N/15N DNA containing 15
N only

H2 Biology / 9648 / 02
1121
8

DNA replication is a process which involves DNA polymerase and the deoxyribonucleotides
A, T, C and G, while transcription involves RNA polymerase and the ribonucleotides A, U, C
and G.

(d) Describe two other differences and two similarities between DNA replication and
transcription.

[4]
[Total: 13]

Differences
Features DNA replication Transcription
Template Both strands of the double helix Only one strand of the double
are used as templates. helix is used as template.
Parts of template Entire DNA molecule / Entire Only short segments of DNA /
copied chromosome / Entire or All of 2 gene is transcribed to form RNA
strands is/are replicated
Product Double stranded DNA Single stranded RNA which can
be mRNA, tRNA or rRNA

Similarities
1 Both involve unwinding and unzipping double helix DNA/ separation of 2 DNA strands by
breaking of the weak hydrogen bonds between complementary bases
2 Both involve formation of phosphodiester bonds between neighbouring nucleotides
3 Both involve aligning free nucleoside triphosphates / nucleotides through complementary
base pairing
4 Both products elongate in 5 to 3 direction
5 Both require energy from ATP hydrolysis

H2 Biology / 9648 / 02
1122
9
3 Fig. 3.1 is an electron micrograph showing T4 bacteriophages invading a bacterium.

Fig. 3.1

(a) Clearly label the base plate of one T4 phage in Fig. 3.1. [1]
Answer must show clear line pointing to base plate with appropriate label (base plate
or (a)).

(b) Explain how T4 may mediate the transfer of a random piece of this bacterias genetic
material to another bacteria.
In generalized transduction;

After T4 DNA entered the donor bacterium, phage enzymes hydrolyze bacteria DNA;

During assembly of new phage virus, random piece of hydrolysed bacterial DNA
packaged within phage capsid;

After lysis of bacterial cell, phage infects recipient bacterium, injecting donor DNA
along with its own;

Homologous recombination may result in recipient bacteria acquiring new


characteristics;

Specialised transduction because phage is T4 and genetic material is random


piece of bacterial genetic material.
[4]
(c) Briefly describe the structure of bacterial chromosomes.
1 double-stranded circular DNA;

Form looped domains with small amount of DNA-binding proteins; OR

Further supercoiled to form chromosome;

[2]

H2 Biology / 9648 / 02
1123
10
Some genes in bacteria are organized into operons, such as the lac operon. Transcription
of the lac operon in E. coli is dependent on the presence or absence of food molecules such
as glucose and lactose.

(d) Under normal conditions, predict the transcription of the operon in the presence or
absence of these two substrate molecules by writing High or Low in the table below.

Lactose Glucose Transcription


+ + Low
+ - High
- + Low
- - Low
[1]

(e) Mutation XX-6 deletes the DNA binding site from the CAP protein, but leaves the rest
of the operon unaffected. Predict the effect on transcription under these four conditions
by writing High or Low in the table below.

Lactose Glucose Transcription


+ + Low
+ - Low
- + Low
- - Low
[1]
(f) Another mutation on a separate site gives the following results.
Lactose Glucose Transcription
+ + Low
+ - High
- + Low
- - High

Suggest the mutation that could have occurred and explain how it would have given rise
to the above results.
Mutation of lacI gene, resulting in non-functional repressor / repressor that no longer
binds to operator;
OR
Mutation of operator such that repressor can no longer bind (without affecting binding
of RNA polymerase to promoter);

RNA polymerase free to bind to promoter to initiate transcription regardless of levels


of glucose present.
[2]
[Total: 11]

H2 Biology / 9648 / 02
1124
11
4 A woman's risk of developing breast cancer is greatly increased if she inherits BRCA1 gene
mutation. The role of BRCA1 protein is shown in Fig. 4.1.

DNA Repair &


Cell cycle Arrest

Fig. 4.1

(a) With reference to Fig. 4.1,


(i) State the class of genes that BRCA1 belongs to.
Tumour suppressor genes;
[1]

H2 Biology / 9648 / 02
1125
12
(ii) Explain how mutations in BRCA1 gene can contribute to cancer.
Loss of function mutation in BRCA1 gene which codes for proteins involved in DNA
repair / repair damaged DNA;

DNA repair do not occur resulting in accumulations of mutations in proto-oncogene /


tumour suppressor genes / telomerase gene; OR

Resulting in uncontrolled cell division forming a tumour; OR

Resulting in tumour angiogenesis, invasion of cancer cells into surrounding tissues


and metastasis to distant sites;

[3]
(iii) The expression of BRCA1 gene can be furthered controlled at post-translational
level. Explain how this can occur.
Biochemical modification by phosphorylation / addition of phosphate groups to BRCA1
protein by ATM kinase;

Phosphorylated BRCA1 protein is functional and able to form protein complex /


OWTTE (effect of phosphorylation);
[2]

(b) The HER2 gene codes for a growth factor receptor. When the growth factor binds to
HER2 protein (the growth factor receptor), the signal is given for the cell to start dividing.
Gene amplification of HER2 gene could also result in breast cancer.
(i) Describe a process which results in gene amplification.
Any 1 pair

During DNA replication, DNA polymerase may slip from the DNA template strand;

then reattach and replicate a region that was already replicated;

OR

A copy of the repeat sequence in one chromosome can misalign with a different copy
of the repeat in the homologous chromosome;

Unequal crossing over occurs, resulting in one chromosome being longer than the
other;
[2]
(ii) Suggest how amplification of HER2 could result in breast cancer.
Increases the number of copies of HER2 gene / causes over-expression of HER2
growth receptors;

Therefore more sensitive to growth factors and thus stimulate uncontrolled cell
division;

[2]
[Total: 10]

H2 Biology / 9648 / 02
1126
13
5 Rickets is a childhood disorder involving the softening and weakening of bones in humans.
It is usually caused by a lack of vitamin D, calcium ions or phosphate ions. A rare form of
rickets that cannot be successfully treated with vitamin D therapy is caused by an allele on
the X chromosome that arose as a result of a gene mutation.

(a) Explain what is meant by the term gene mutation.


Refers to a change in DNA / base / nucleotide sequence;

e.g. addition / substitution / deletion of a nucleotide / a base pair;

no / different protein / polypeptide is produced;

produces a different allele;


[2]
(b) Fig. 5.1 below illustrates how this rare form of rickets is inherited in an afflicted family.

Fig. 5.1

(i) With reference to Fig. 5.1, state and explain the pattern of inheritance of this form
of rickets.
(Sex-linked) dominant condition;

Individual 2 has one X chromosome with dominant mutant allele and will pass to all
the daughters 4, 5 and 6;
OR Individual 6 can pass the dominant mutant allele to their daughter 9;

The dominant mutant allele will mask the effect expression of the recessive normal
allele / only one copy of the mutant allele required to manifest rickets;
[3]
(ii) Using appropriate symbols, state the genotypes of the following individuals.
1 Xr Xr

3 XrY

9 X RX r

H2 Biology / 9648 / 02
1127
14

10 X RY

[2]

(iii) Couple 6 and 7 decided to have another son. Draw a genetic diagram to determine
the probability that their second son will suffer from rickets.

Parental phenotypes: Normal male Female with rickets

Parental genotypes: XrY X RX r ;

After meiosis, Xr Y XR Xr
gametes produced

By random fertilisation,

Xr Y

XR X RX r X RY

;
X r XrXr XrY

offspring genotypes: X RX r XrXr X RY XrY

offspring phenotypes: 1 Female : 1 Female : 1 Male : 1 Male


with rickets without rickets with rickets without rickets
1/2 or 50% of the sons will suffer from rickets;

H2 Biology / 9648 / 02
1128
15
(c) Colour blindness and haemophilia are inherited sex-linked recessive conditions in
humans. Fig. 5.2 shows two generations of a family in which these conditions are
inherited.

Fig. 5.2

Suggest how the genotype of S has arisen.


Crossing over of non-sister chromatids of homologous chromosomes during Prophase
I of meiosis in P;

Exchange of alleles between X chromosomes and S inherit a X chromosome carrying


the (dominant) normal allele for haemophilia and (recessive) mutant allele for colour
blind;

Both genes are linked / found on the X chromosome;


[2]
[Total: 12]

H2 Biology / 9648 / 02
1129
16
6 Fig. 6.1 below shows the cell signalling pathway in smooth muscle cells in response to the
hormone Epinephrine, resulting in smooth muscle relaxation of blood vessels (vasodilation).

activated
Epinephrine
adenylate cyclase

-adrenergic
receptor

cyclic AMP

inactive protein kinase A (PKA) activated PKA

Responses of target cell:


Smooth muscle relaxation,
vasodilation

Fig. 6.1
(a) State the class of receptors that the -adrenergic receptor belongs to.
G protein coupled receptors / G protein linked receptors;
[1]
(b) With reference to Fig. 6.1,
(i) describe how adenylate cyclase is activated.
When the epinephrine binds to the -adrenergic receptor, a ligand-receptor complex is
formed causing the receptor undergoes a conformation change;

Bound G-protein (at the cytoplasmic) end of the receptor to exchange its GDP with a
GTP / substitute GTP for GDP;

the G-protein is now activated and translocates / moves along the membrane to bind
to and activate adenylate cyclase

[3]
(ii) identify the stage where signal amplification is observed and explain how the signal
is amplified in that stage.
Stage B / Conversion of ATP to cAMP by adenylate cyclase;

Each enzyme produces many molecules of cAMP;


[2]

H2 Biology / 9648 / 02
1130
17
(c) Explain how the structure of the -adrenergic receptor allows it to carry out its function
as a receptor.
Transmembrane protein embedded on cell surface membrane;

Extracellular binding site is complementary/specific to ligand / ephinephrine;

Intracellular domain able to bind to inactive G-protein / able to undergo conformational


change to cause activation of G-protein;

[3]
(d) A drug was designed to ensure that permanent vasodilation occurs. Suggest how this
drug might work within the cell signalling pathway in Fig. 6.1.

Similar structure to epinephrine and binds permanently to -adrenergic receptor;

G-protein permanently activated, adenylate cyclase permanently activated,


continuously synthesizes cAMP;

[2]
[Total: 11]

H2 Biology / 9648 / 02
1131
18
7 Scientists studying the evolutionary history of the Human Immunodeficiency Virus (HIV)
found that it is closely related to the Simian Immunodeficiency Virus (SIV) found in certain
species of monkeys. The phylogenetic tree in Fig. 7.1 shows part of the evolutionary
relationships between these viruses. A common ancestral virus is believed to have evolved
into strains that infected chimpanzees (SIV). Over time, new strains emerged which infect
humans.

Fig. 7.1

(a) Explain the significance of the branch points in the phylogenetic tree shown in Fig. 7.1.
Each branch point represents the common ancestor shared by descendents /
divergence of two species from common ancestor;

Position of branch point corresponds to the relative time when two species had
diverged;

[2]

(b) Explain how molecular homology could be used to support the claim that HIV and SIV
share an evolutionary relationship.
Direct comparison of DNA / RNA of a homologous sequence
/ amino acid sequence of a common gene;

Two viral species that show high similarity in their sequences probably share a
common ancestor;
The more similar the sequences of two species are, the more closely related they are
in their evolutionary relationship.

[2]

H2 Biology / 9648 / 02
1132
19
(c) State two advantages of using molecular techniques in classification.
Degree of divergence between different species can be quantitatively measured by
comparison of nucleotide / amino acid sequences;

This method is objective compared to comparison of morphology which may be


subjective;

Evolutionary changes between species can be compared even though they may differ
vastly in terms of morphology / Molecular comparisons transcend barriers among
organisms whose relationships cannot be evaluated by traditional experimental
techniques;

[2]

SIV infections in their natural hosts appear in many cases to be non-pathogenic (non-
disease causing). Certain individuals within the human population have also shown similar
characteristics when exposed to HIV. These individuals continue to test negative for HIV
despite being repeatedly exposed to HIV.
Investigations have found a mutation present in the CCR5 gene in these individuals. CCR5
is a co-receptor that facilitates the entry of HIV into the T-cell after gp120 (present on the
viral envelope) binds with the CD4 receptor.

(d) Suggest and explain how these individuals continue to resist HIV infection.
A CCR5 gene mutation may result in lowered expression of CCR5 protein / change in
structure of CCR5 / non-functional CCR5 receptor on cell surface membrane of helper
T-cells;

Virus envelope unable to fuse with cell surface membrane of helper T-cells and thus
unable to replicate in T-cells;

[2]

H2 Biology / 9648 / 02
1133
20
(e) Assuming that humans do not take active steps to prevent HIV infection, explain how
the human population could possibly evolve to not be affected by HIV.
Variation in genotype of humans some with CCR5 mutation, some without;
@ HIV resistant individuals and non HIV resistant individuals
individuals with HIV and individual without HIV

When exposed to HIV, humans with CCR5 mutation that prevent entry of HIV are able
to survive and thus possess a selective advantage over other humans without CCR5
mutation;

Thus this favourable allele trait is inherited by the offspring;

Allele frequency for CCR5 mutation increases in human population, over long period
of time, most humans will be unaffected by HIV;

[3]
[Total: 11]

H2 Biology / 9648 / 02
1134
21
Section B
Answer one question.

Write your answers on the separate answer paper provided.


Your answers should be illustrated by large, clearly labelled diagrams, where appropriate.
Your answers must be in continuous prose, where appropriate.
Your answers must be set out in sections (a), (b) etc., as indicated in the question.

8 (a) Outline how the endomembrane system is involved in synaptic transmission. [8]
(b) Describe the main stages of the Calvin cycle [8]
(c) Contrast the structure of cellulose with that of collagen and suggest why plant
cells have cellulose but not collagen. [4]

9 (a) Compare and contrast the structures of influenza and HIV. [8]
(b) Describe how HIV infects host cells and reproduces. [8]
(c) Briefly explain how a person infected only once by one strain of HIV may
eventually have many strains of HIV. [4]

(a) Outline how the endomembrane system is involved in synaptic transmission. [8]

1 Synaptic transmission involves membrane proteins such as voltage-gated Ca2+ channels


in the pre-synaptic membrane / ligand-gated ion channels in the post-synaptic membrane.
2 Both type of channels are integral membrane protein which is first translated by ribosomes
attached to rER;
3 and modified / folded into right conformation in the rER lumen;
4 The folded proteins are then transported to the cis face of Golgi apparatus via transport
vesicles which bud off the rER;
5 When moving from cis to trans face of the Golgi apparatus, the proteins are further
chemically modified such as glycosylation, phosphorylation, dehydration, and
concentration.
6 The protein is then inserted to the membrane of secretory vesicles which bud off the trans
face of Golgi apparatus;
7 The membrane of secretory vesicles move towards and fuse with the cell surface
membrane, incorporating the channel proteins in the membrane;
8 Some neurotransmitters of peptide nature are also synthesized and packaged in the same
way for exocytosis. (Note: Acetylecholine is NOT a peptide hormone)
9 However, they are enclosed in vesicles instead of inserted into the vesicle membrane.
10 Acetylcholinesterase embedded on post-synaptic membrane also synthesized in similar
manner to channel proteins.

H2 Biology / 9648 / 02
1135
22

(b) Describe the main stages of the Calvin cycle. [8]

1 Calvin cycle occurs in the stroma of chloroplasts;


2 During CO2 fixation, ribulose bisphosphate (RuBP) accepts CO2 to form an unstable 6C
intermediate;
3 catalyzed by ribulose bisphosphate carboxylase-oxygenase (Rubisco);
(Note that rubisco is NOT an acceptable abbrieviation)
4 Unstable 6C intermediate immediately breaks down to 2 molecules of 3C compound known
as glycerate phosphate / phosphoglycerate (GP / PGA);
5 GP can be converted to pyruvate which is used to synthesize fatty acids/amino acids;
6 During reduction, GP is then converted to triose phosphate / glyceraldehyde phosphate (TP
/ GALP)
7 using energy from hydrolysis of ATP and reducing power from reduced NADP, produced by
light dependent reactions;
8 TP contains more chemical energy than PGA / GP, and is the first carbohydrate made in
photosynthesis to leave the Calvin cycle;
9 5/6 of the total amount of TP is used to regenerate the RuBP consumed in the first
reaction which requires energy from the hydrolysis of ATP;
10 1/6 of the total amount of TP is used to make other carbohydrates and glycerol;
11 6 rounds of the cycle is required to produce 1 molecule of glucose;

H2 Biology / 9648 / 02
1136
23

(c) Contrast the structure of cellulose with that of collagen and suggest why plant cells have
cellulose but not collagen. [4]

1 Cellulose is a polysaccharide whereas collagen is a fibrous protein;


2 Monomers which make up cellulose are -glucose residues whereas monomers that make
up collagen are amino acids ( if only mention glycine/proline/hydroxyproline. Other amino
acids are also present albeit in much lower numbers!);
3 -glucose in cellulose are linked by -1,4-glycosidic bonds whereas amino acids in collagen
are linked by peptide bonds;
4 -glucose in cellulose are rotated 180o relative to each other such that they form straight
chains whereas collagen is formed by coiling 3 helical polypeptides around each other to
form a tropocollagen;

5 Genes encoding for polypeptides of collagen is absent / not expressed in plant genome;
6 Lack enzymes necessary for the post-translational processing / hydroxylation of proline /
glycosylation of amino acids in collagen;
7 Plant cells can undergo photosynthesis, so glucose is readily available for the synthesis of
cellulose whereas amino acids are not as readily abundant for formation of collagen;

H2 Biology / 9648 / 02
1137
24

(a) Compare and contrast the structures of influenza and HIV. [8]

1 Both have envelopes with embedded glycoproteins;


2 Envelopes of both are derived from host cell surface membrane;
3 Both genomes are single-stranded RNA;
4 Both are spherical in shapes;

Features Influenza HIV


5. Genome; Single-stranded, segmented Single-stranded positive sense
negative sense RNA RNA

6.; which is complementary in which is identical in sequence to


sequence to mRNA mRNA
7. Number of copies of 1 copy segmented into 8 2 identical copies
genome; pieces
8. Types of Haemagglutinin and gp120 and gp 41
glycoprotein ( neuraminidase
receptor) ;
9. Shape of capsid; Helical nucleoprotein Conical
10. Enzymes present; Viral RNA-dependent RNA Integrase, protease and reverse
polymerase transcriptase

H2 Biology / 9648 / 02
1138
25

(b) Describe how HIV infects host cells and reproduces. [8]

Adsorption / Attachment
1 gp120 on viral envelope recognize and bind to specific receptor molecules (CD4 +) on
cell surface membrane of T helper cells of immune system;
Entry / Penetration
2 Viral envelope fuses with cell surface membrane;
3 Capsid is digested by cellular enzymes ( exonucleases), releasing the viral RNA
molecules, viral proteins and enzymes into the cytoplasm.
Integration
4 Reverse transcriptase catalyses the synthesis of a single DNA strand complementary to
the viral RNA;
5 The viral RNA is degraded and reverse transcriptase catalyses the synthesis of a
second DNA strand complementary to the first;
(OR
4 & 5 may be combined for a single mark as:
Reverse transcriptase catalyses the synthesis of double stranded DNA from RNA
template)
6 The double-stranded viral DNA then enters the cells nucleus and integrates, as a
provirus, into the host cell DNA via the action of integrase;
(The provirus never leaves the hosts genome, remaining permanently in the host cell.)
Synthesis of viral components
7 When host cell is activated, proviral genes are transcribed into RNA;
8 RNA is used as the HIV genome;
9 Or serves as mRNA which is translated into viral proteins (capsid protein and viral
glycoproteins at ribosomes in cytoplasm and surface of endoplasmic reticulum
respectively);
10 Golgi vesicles embedded with viral glycoproteins migrate towards and fuse with the cell
surface membrane such that viral glycoproteins become embedded on the cell surface
membrane;
11 Polyproteins are similarly cleaved by proteases to produce structural proteins.
Viral assembly / maturation
12 Capsid proteins enclose viral genome and viral proteins;
Release
13 Virus buds off from the cell (exocytosis), surrounded by host cell surface membrane
embedded with viral glycoproteins;

H2 Biology / 9648 / 02
1139
26

(c) Although a person may be infected only once by one strain of HIV he may eventually
have many strains of HIV. Briefly explain why. [4]

1 Different strains of HIV are genetic variation that arise via mutations;
2 HIV mutates very rapidly;
3 Reverse transcriptase is inaccurate in its replication / Unlike DNA polymerase, reverse
transcriptase does not proof read the nucleotides added;
4 Reproductive rate of rate of HIV is high increases chances of mutant strains arising;
5 Mutation rate is greater because the genetic material is single stranded (RNA);
6 RNA single stranded less stable because no complementary strand no template
for DNA repair;

H2 Biology / 9648 / 02
1140

NANYANG JUNIOR COLLEGE


JC 2 PRELIMINARY EXAMINATIONS
Higher 2

CANDIDATE
NAME

CLASS

BIOLOGY 9648/03
Paper 3 Applications and Planning September 2013
2 hours
Additional Materials: Answer Paper

READ THESE INSTRUCTIONS FIRST

Write your name and CT on all the work you hand in.
Write in dark blue or black pen on both sides of the paper.
You may use soft pencil for any diagrams, graphs or rough working.
Do no use staples, paper clips, highlighters, glue or correction fluid.

Answer all questions.

At the end of the examination, fasten all your work securely together.
The number of marks is given in brackets [ ] at the end of each question or part question.

Answer questions 1 to 4 in the spaces provided on the question paper.

Answer question 5 on the separate answer paper provided.


For Examiners Use

Total
60
For Examiners Use

4
12

This document consists of 15 printed pages and 1 blank page.


[Turn over

H2 Biology / 9648 / 03
1141
2

Section A
Answer all the questions in this section.

1 Bacteria can be genetically modified to produce insulin for human use. To achieve this,
human insulin genes are transferred into bacteria. Plasmids containing two antibiotic
resistance genes, one coding for resistance to tetracycline and one for resistance to
ampicillin, are used to carry out this transfer.
A restriction enzyme was used to cut up the human DNA and plasmids. Fig. 1.1 shows the
different fragments of human DNA and the type of cut plasmid that was produced.

Fig. 1.1

(a) Describe a plasmid.

[2]
(b) Suggest why the restriction enzyme has cut the human DNA in many places but has
cut the plasmid DNA only once.

[2]

H2 Biology / 9648 / 03
1142
3

The fragments of human DNA and the cut plasmids were mixed together with DNA ligase.
Several types of plasmid were formed. Some contained human DNA in the centre of the
gene coding for resistance to tetracycline. The different types of plasmid are shown in
Fig 1.2.

Fig 1.2

(c) Explain what causes several types of plasmid to be formed.

[2]
(d) Outline the reaction that DNA ligase catalyses.

[1]

H2 Biology / 9648 / 03
1143
4

The plasmids are mixed with the bacteria. Some bacteria take up the plasmids.
Some of the plasmids did not take up the extra human DNA. Replica plating was used to
identify the bacteria with the human DNA. Fig. 1.3 shows the bacterial colonies that grew on
two replica plates.

Fig. 1.3
(e) With reference to Fig. 1.3, explain the results of the replica plate containing:
(i) ampicillin.

[2]
(ii) tetracycline.

[3]

H2 Biology / 9648 / 03
1144
5

(f) Describe how the bacteria containing the insulin gene are used to obtain sufficient
insulin for commercial use.

[3]
[Total: 15]

H2 Biology / 9648 / 03
1145
6

2 Cord blood is the blood that circulates through the umbilical cord from the foetus to the
placenta. The umbilical cord that remains attached to the placenta has been found to be
rich in cord blood stem cells. Cord blood stem cells are the young or immature cells that can
transform into other forms of essential blood cell types, such as red blood cells, white blood
cells and platelets.

Adapted from: http://www.cordlife.com/sg/en/umbilical-cord-tissue/about-umbilical-cord-


tissue

(a) Describe the unique features of all stem cells.

[3]
(b) Suggest an advantage of using cord blood stem cells in place of blood stem cells from
the bone marrow.

[1]
(c) State the potency of cord blood stem cells.

[1]

There exist other groups of stem cells of greater developmental potential.


(d) Describe, with a named example, a group of stem cells with higher developmental
potential than cord blood stem cells.

[2]

H2 Biology / 9648 / 03
1146
7

Besides using stem cell transplant to treat genetic disorders such as SCID, gene therapy
could be another treatment option. Severe Combined Immunodeficiency (SCID) is a
disease of young children in which patients have neither cell-mediated immune responses
nor the ability to make antibodies.
There are two types of SCID: Adenosine Deaminase (ADA) deficient SCID and X-linked
SCID. The delivery system used in SCID gene therapy is usually viral gene delivery
system.

(e) Suggest a type of virus that can be used in SCID gene therapy.

[1]
(f) Explain the factors that keep gene therapy from becoming an effective treatment for
SCID.

[3]
[Total: 11]

H2 Biology / 9648 / 03
1147
8

3 Golden rice is a transgenic variety of rice that has been genetically engineered to be rich in
beta-carotene (a pre-cursor to vitamin A). Fig. 3.1 below shows an artificial DNA sequence
used in the production of golden rice.

Key:
pro promoter sequence
ter terminator sequence
Hyg resist antibiotic resistance from a bacterium

Fig. 3.1

(a) Explain what is meant by transgenic.

[1]
(b) State the organism from which the promoter sequence for Hyg resist were taken from
and explain your choice.

[2]
(c) Briefly outline one way in which this gene construct may be inserted into rice cells.

[2]
(d) State why it is not possible to produce golden rice by breeding.

[1]

H2 Biology / 9648 / 03
1148
9

Hyg resist confers resistance to hygromycin, an antibiotic which kills both bacterial cells and
plant cells.
(e) Explain the role of the Hyg resistance gene in this procedure.

[2]
(f) A concern of having Hyg resistance in the golden rice is the possible horizontal transfer
of Hyg resist to other organisms.
(i) Suggest how Hyg resistance can be acquired by other plants and state a technical
safeguard that can minimize this risk.

[2]
(ii) Hyg resistance may result in a resistant strain of bacteria for which this antibiotic
will no longer be effective. Explain if this claim is biologically valid.

[2]
(g) Suggest 2 possible benefits of golden rice.

[2]
[Total: 14]

H2 Biology / 9648 / 03
1149
10

Planning Question

4 You are required to plan, but not carry out, an investigation into the effect of temperature on
rate of photosynthesis.
Fig. 4.1 shows an oxygen probe and meter that can be used to measure the concentration
of dissolved oxygen in a solution in mg/L.

6.51 mg/L oxygen probe

Design an experiment, using an oxygen sensor to test the hypothesis that:

The rate of photosynthesis is dependent on the temperature.

Your planning can include, but is not limited to the following equipment & materials.

A beaker of suspension of unicellular algae in water


Bench lamp
thermometer
hot water at 80oC
stopwatch
distilled water
glass rod
Stopwatch
a variety of different sized beakers, test tubes, measuring cylinders, syringes,
droppers and pipettes for measuring volumes

Your plan should include:

an explanation of the theory to support your practical procedure,


relevant, clearly labelled diagram(s) to show the arrangement of apparatus used,
an explanation of dependent & independent variables involved,
proposed layout of results tables and graphs with clear headings and labels,
full details and explanations of the procedures you would adopt to ensure that the
results obtained were as quantitative, precise and reliable as possible,
correct use of scientific & technical terms
[Total: 12]

H2 Biology / 9648 / 03
1150
11

Free Response Question

Write your answers on the separate answer paper provided.


Your answers should be illustrated by large, clearly labelled diagrams, where appropriate.
Your answers must be in continuous prose, where appropriate.
Your answers must be set out in sections (a), (b) etc., as indicated in the question.

5 (a) Using your knowledge of the genetic basis of cystic fibrosis, compare liposomal and
viral delivery systems in the treatment of cystic fibrosis using gene therapy. [6]
(b) Describe how RFLP analysis can be used in genome mapping. [8]
(c) Discuss the ethical concerns that have arisen regarding the Human Genome Project.
[6]
[Total: 20]

H2 Biology / 9648 / 03
1151

NANYANG JUNIOR COLLEGE


JC 2 PRELIMINARY EXAMINATIONS
Higher 2

CANDIDATE
NAME
Mark Scheme

CLASS

BIOLOGY 9648/03
Paper 3 Applications and Planning September 2013
2 hours
Additional Materials: Answer Paper

READ THESE INSTRUCTIONS FIRST

Write your name and CT on all the work you hand in.
Write in dark blue or black pen on both sides of the paper.
You may use soft pencil for any diagrams, graphs or rough working.
Do no use staples, paper clips, highlighters, glue or correction fluid.

Answer all questions.

At the end of the examination, fasten all your work securely together.
The number of marks is given in brackets [ ] at the end of each question or part question.

Answer questions 1 to 4 in the spaces provided on the question paper.

Answer question 5 on the separate answer paper provided.


For Examiners Use

Total
60
For Examiners Use

4
12

This document consists of 15 printed pages and 1 blank page.


[Turn over

H2 Biology / 9648 / 03
1152
2

Section A
Answer all the questions in this section.

1 Bacteria can be genetically modified to produce insulin for human use. To achieve this,
human insulin genes are transferred into bacteria. Plasmids containing two antibiotic
resistance genes, one coding for resistance to tetracycline and one for resistance to
ampicillin, are used to carry out this transfer.
A restriction enzyme was used to cut up the human DNA and plasmids. Fig. 1.1 shows the
different fragments of human DNA and the type of cut plasmid that was produced.

Fig. 1.1

(a) Describe a plasmid.


circular double stranded DNA;

separate from main bacterial DNA;


contains only a few genes;
replicates autonomously / independently of bacteria DNA;
[2]
(b) Suggest why the restriction enzyme has cut the human DNA in many places but has
cut the plasmid DNA only once.
enzymes only cut DNA at specific base sequence / recognition site / specific restriction
site;

sequence of bases / recognition site / specific restriction site (on which enzyme acts)
occurs once in plasmid and many times in human DNA;

[2]

H2 Biology / 9648 / 03
1153
3

The fragments of human DNA and the cut plasmids were mixed together with DNA ligase.
Several types of plasmid were formed. Some contained human DNA in the centre of the
gene coding for resistance to tetracycline. The different types of plasmid are shown in
Fig 1.2.

Fig 1.2

(c) Explain what causes several types of plasmid to be formed.


all cut DNA have same / complementary base sequence sticky ends which anneals by
complementary base pairing (between A=T and C=G);

It is a random process by which sticky ends join and subsequently ligated by DNA
ligase;
[2]
(d) Outline the reaction that DNA ligase catalyses.
Formation of phosphodiester bonds between nucleotides (by condensation reaction
with the removal of water);
[1]

H2 Biology / 9648 / 03
1154
4

The plasmids are mixed with the bacteria. Some bacteria take up the plasmids.
Some of the plasmids did not take up the extra human DNA. Replica plating was used to
identify the bacteria with the human DNA. Fig. 1.3 shows the bacterial colonies that grew on
two replica plates.

Fig. 1.3
(e) With reference to Fig. 1.3, explain the results of the replica plate containing:
(i) ampicillin.
Colonies 1,2 3 & 6 have taken up the plasmid;

Because they are resistant to ampicillin / has ampicillin resistance gene;

OR
Colonies 4 & 5 did not take up the plasmid;

Because they are not resistant to ampicillin / lack of ampicillin resistance gene;

[2]
(ii) tetracycline.
Colonies 1 & 3 do not have the recombinant plasmid;

Because they are still resistant to tetracycline / has tetracycline resistance gene;

Colonies 2 and 6 dies because taken up the plasmid with human DNA / recombinant
plasmid;

Tetracycline resistance gene disrupted / insertional inactivation of tetracycline


resistance gene;

[3]

H2 Biology / 9648 / 03
1155
5

(f) Describe how the bacteria containing the insulin gene are used to obtain sufficient
insulin for commercial use.
use of fermenters;

provides nutrients plus suitable conditions for optimum growth / named environmental
factor;

reproduction of bacteria via binary fission increasing the expression of insulin gene /
more insulin proteins are produced;

insulin accumulates and is extracted and purified;


[3]
[Total: 15]

H2 Biology / 9648 / 03
1156
6

2 Cord blood is the blood that circulates through the umbilical cord from the foetus to the
placenta. The umbilical cord that remains attached to the placenta has been found to be
rich in cord blood stem cells. Cord blood stem cells are the young or immature cells that can
transform into other forms of essential blood cell types, such as red blood cells, white blood
cells and platelets.

Adapted from : http://www.cordlife.com/sg/en/umbilical-cord-tissue/about-umbilical-cord-


tissue

(a) Describe the unique features of all stem cells.


Stem cells are unspecialised, there is an absence of tissue-specific structures for
specialised functions;

Stem cells are capable of dividing by mitosis and continually renewing themselves for
long periods;

Stem cells are capable of differentiating into specialised cell types under appropriate
conditions;

[3]
(b) Suggest an advantage of using cord blood stem cells in place of blood stem cells from
the bone marrow.
It is not painful as blood is drawn from umbilical vein after umbilical cord is separated
from the newborn baby / no surgical procedure involved;

Cord blood stem cells are easier to collect as blood is drawn from umbilical vein after
umbilical cord is separated from the newborn baby / no surgical procedure involved;

There is no risk to the baby or mother as blood is drawn from umbilical vein after
umbilical cord is separated from the newborn baby / no surgical procedure involved;
[1]
(c) State the potency of cord blood stem cells.
Multipotent;
[1]
There exist other groups of stem cells of greater developmental potential.
(d) Describe, with a named example, a group of stem cells with higher developmental
potential than cord blood stem cells.
Zygotic stem cells are totipotent;
These cells can differentiate into any cell types to form whole organisms;

Embryonic stem cells are pluripotent;


able to give rise to almost all cell types to form the three primary germ layers;
ectoderm, endoderm and mesoderm / to the multiple specialised cell types / tissues /
organs in the developing foetus;
[2]

H2 Biology / 9648 / 03
1157
7

Besides using stem cell transplant to treat genetic disorders such as SCID, gene therapy
could be another treatment option. Severe Combined Immunodeficiency (SCID) is a
disease of young children in which patients have neither cell-mediated immune responses
nor the ability to make antibodies.
There are two types of SCID: Adenosine Deaminase (ADA) deficient SCID and X-linked
SCID. The delivery system used in SCID gene therapy is usually viral gene delivery
system.

(e) Suggest a type of virus that can be used in SCID gene therapy.
Retrovirus;
[1]
(f) Explain the factors that keep gene therapy from becoming an effective treatment for
SCID.
Gene therapy is short-lived and requires frequent / repeated treatments;

Incorrect insertion of normal gene into tumour-suppressor gene may cause cancer /
insertional mutagenesis;

Immune response may be triggered as a foreign viral vector is introduced;

Problem with controlling the activity of gene expression;


[3]
[Total: 11]

H2 Biology / 9648 / 03
1158
8

3 Golden rice is a transgenic variety of rice that has been genetically engineered to be rich in
beta-carotene (a pre-cursor to vitamin A). Fig. 3.1 below shows an artificial DNA sequence
used in the production of golden rice.

Key:
pro promoter sequence
ter terminator sequence
Hyg resist antibiotic resistance from a bacterium

Fig. 3.1

(a) Explain what is meant by transgenic.


Contains DNA from two different species rice and daffodil
OR
Rice which contains a foreign DNA from another species daffodil;
[1]
(b) State the organism from which the promoter sequence for Hyg resist were taken from
and explain your choice.
Rice / plant;

So that can be recognized by (eukaryotic) RNA polymerase found in rice to be


expressed;
[2]
(c) Briefly outline one way in which this gene construct may be inserted into rice cells.
Inserted into (Ti) plasmid / formation of recombinant plasmid in Agrobacterium;

Agrobacterium infects rice cell to incorporate gene construct into rice cell;

OR
DNA containing gene coated on to gold particles;

Shot with gene gun / biolistics directly into rice cell;


[2]
(d) State why it is not possible to produce golden rice by breeding.
Rice and daffodils belong to different species which will not naturally interbreed to
produce fertile viable offspring;

Even if interbreeding occurred, it involves random fusion of gametes which will not
necessarily contain only favourable traits.

[1]

H2 Biology / 9648 / 03
1159
9

Hyg resist confers resistance to hygromycin, an antibiotic which kills both bacterial cells and
plant cells.
(e) Explain the role of the Hyg resistance gene in this procedure.
It is a selectable marker to enable the selection of the transformed / transgenic cells;

only the cells with the new DNA can grow in the presence of the antibiotic;

[2]
(f) A concern of having Hyg resistance in the golden rice is the possible horizontal transfer
of Hyg resist to other organisms.
(i) Suggest how Hyg resistance can be acquired by other plants and state a technical
safeguard that can minimize this risk.
Transgenic plants may pass their new genes (for hyg-resistance) to close relatives in
nearby wild areas through pollen transfer;

Greater isolation distances between the transgenic crop and nearby areas (fields,
forests etc);
OR
Planting a border of unrelated plants with which transgenic plants would not hybridize
through spread of transgenic pollen;

[2]
(ii) Hyg resistance may result in a resistant strain of bacteria for which this antibiotic
will no longer be effective. Explain if this claim is biologically valid.
Yes; When humans ingest golden rice, the resistance gene may be transferred to gut
bacteria;

No; The resistance gene will probably be broken down by the time it reaches the gut.

No; Direct transfer between plants and bacteria is unlikely since they are highly
unrelated species.
[2]
(g) Suggest 2 possible benefits of golden rice.
May help to prevent night blindness;

Reduce vitamin A deficiency in developing countries;

Enable local farmers to grow a cash crop;

Enable the development of rice breeding to improve local crops;

[2]
[Total: 14]

H2 Biology / 9648 / 03
1160
10

Planning Question

4 You are required to plan, but not carry out, an investigation into the effect of temperature on
rate of photosynthesis.
Fig. 4.1 shows an oxygen probe and meter that can be used to measure the concentration
of dissolved oxygen in a solution in mg/L.

6.51 mg/L oxygen probe

Design an experiment, using an oxygen sensor to test the hypothesis that:

The rate of photosynthesis is dependent on the temperature.

Your planning can include, but is not limited to the following equipment & materials.

A beaker of suspension of unicellular algae in water


Bench lamp
thermometer
hot water at 80oC
stopwatch
distilled water
glass rod
Stopwatch
a variety of different sized beakers, test tubes, measuring cylinders, syringes,
droppers and pipettes for measuring volumes

Your plan should include:

an explanation of the theory to support your practical procedure,


relevant, clearly labelled diagram(s) to show the arrangement of apparatus used,
an explanation of dependent & independent variables involved,
proposed layout of results tables and graphs with clear headings and labels,
full details and explanations of the procedures you would adopt to ensure that the
results obtained were as quantitative, precise and reliable as possible,
correct use of scientific & technical terms
[Total: 12]

H2 Biology / 9648 / 03
1161
11

Theoretical Background (2 m max)


Temperature is a limiting factor of photosynthesis;
Temperature affects rate of enzymatic reactions during light dependent reaction (e,g, NADP
reducatase) and light independent reaction (e.g, Rubisco)
Oxygen is released during photolysis of water and can be used to determine rate of
photosynthesis.

Variables
Independent variable: Temperature
Dependent variable: Concentration of dissolved oxygen
Variable kept constant:
Volume of algae suspension, wavelength of light, light intensity, pH

Hypothesis (same as conclusion): The rate of photosynthesis is maximum at optimum


temperature and decreases away from optimum temperature.

6.51 mg/L
7 cm3 of algal
suspension +
3 cm3 of buffer +
(total volume = 10cm3)

Varying
Temperature

Procedures
1. Using a beaker of hot water and thermometer, prepare a water bath fixed at 300C.
2. Label 5 boiling tubes A E.
3. Using a syringe, add 7.0 cm3 of algae suspension and 3.0 cm3 of pH buffer.
4. Place the boiling tube into the water bath and allow for 5 minutes of equilibration.
5. Using a ruler, place the bench lamp 10 cm away from the water bath.
6. Switch on the bench lamp.
7. After 5 min, insert the oxygen probe into the boiling tube and ensure solution covers probe.
Record the concentration of dissolved oxygen detected.
8. Wash/ rinse the probe with distilled water and put it aside, ready for the next reading.
9. Repeat steps 3 to 8 for the 4 other temperatures of 400C, 500C, 600C and 700C.
10. Repeat steps 3 to 9 twice and calculate the mean concentration of dissolved oxygen.
11. Set up the control by repeat the experiment steps 3 to 8 but carrying out in the dark.
12. Record the data in the table below and calculate rate of reaction as 1/time taken. Calculate
the mean rate of oxygen produced per minute by dividing by 5 minutes.

H2 Biology / 9648 / 03
1162
12

Table 1: Table of results of rate of reaction


Temperature/ 0C Concentration of dissolved oxygen Mean rate of oxygen
in 5 minutes/ mg L-1 production/
1 2 3 Mean mgL-1 min-1

30
40
50
60
70

13. Plot the graph of mean rate of oxygen production / mgL-1 min-1 against temperature / 0C.

Mean rate of
oxygen
production/
mg L-1 min-1

Temperature / 0C

Conclusion
The rate of photosynthesis is maximum at optimum temperature and decreases away from
optimum temperature.

Risk Assessment
1. Scalding by hot water. Use cloth to wrap the mouth of beaker.
2. Electrocution due to thermostat water bath. Avoid touching electrical appliance with water.

H2 Biology / 9648 / 03
1163
13

Mark scheme
Theoretical background [2m max]
T1: Temperature is a limiting factor of photosynthesis;
T2: Temperature affects rate of enzymatic reactions during light dependent reaction (e,g,
NADP reducatase) and light independent reaction (e.g, Rubisco)
T3: Oxygen is released during photolysis of water and can be used to determine rate of
photosynthesis.

Procedures [8m max]


CV1: Fixed volume of algae suspension / volume of buffer / Light intensity / wavelength of light
/ total volume kept constant;
D: Diagram of experimental set-up (with water bath); 80OC
E: Equilibration time
C: Control by replacing conducting experiment in the dark;
R: Repeat experiment three times + calculate mean time;
PD: Calculate mean rate of oxygen production by dividing by time;
TB: tabulation of data with correct headings and units;
G: correct graph with axes labels & units + trend;
CON: optimum temperature;
DV: Dependent variable: concentration of dissolved oxygen + independent variable of at
least 5 temperatures.

Risk assessment [2m max]


RA1: Scalding by hot water. Use cloth to wrap the mouth of beaker. glove
RA2: Electrocution due to thermostat water bath. Avoid touching electrical appliance with
water.
RA3: Putting thermometer back into basket/ put away safely to prevent breakage because the
mercury fumes released when the thermometer is broken is toxic.

H2 Biology / 9648 / 03
1164
14

Free Response Question

Write your answers on the separate answer paper provided.


Your answers should be illustrated by large, clearly labelled diagrams, where appropriate.
Your answers must be in continuous prose, where appropriate.
Your answers must be set out in sections (a), (b) etc., as indicated in the question.

5 (a) Using your knowledge of the genetic basis of cystic fibrosis, compare liposomal and
viral delivery systems in the treatment of cystic fibrosis using gene therapy. [6]
(b) Describe how RFLP analysis can be used in genome mapping. [8]
(c) Discuss the ethical concerns that have arisen regarding the Human Genome Project.
[6]
[Total: 20]
(a) Scientist use either viral (e.g. adenovirus) or non-viral delivery (liposomes) methods to
introduce a normal functional CFTR allele into the human.

Point of Adenovirus-mediated gene Liposome-mediated gene therapy


comparison therapy
Differences
D1 Immune Adenovirus more likely to evoke Less likely to evoke immune
response immune response response

D2 Effectiveness Less likely to be effective after Remains effective as body does not
on evoking the body has developed develop immunity to the vector
immune immunity to the viral vector
response
D3 Disease Adenoviral vector (which does Do not cause disease at all / safer
causing / risk not integrate into the genome (Liposomes have a small degree of
assessment / can cause throat infections but toxicity so many rounds of
Toxicity not human malignancies treatment can be harmful to the cells
(cancer) or tissues being treated)
(Idea that viral delivery methods cause more harm or health implications
than the liposomal method)
D4 Specificity More specific targeting of cells Less specific targeting of cells

D5 Transfection High transfection efficiency. Low transfection efficiency.


efficiency
(Idea that viral delivery methods allow functional gene to be more easily
delivered into the host cell than the liposomal method)
D6 Delivery of Greater chances of delivering No ability to transfer normal
normal normal functional allele into functional allele into nucleus.
functional nucleus via nuclear pore as
CFTR allele adenovirus directly injects DNA
into nucleus / at nuclear pore.
Chances of
integration
into the
genome

H2 Biology / 9648 / 03
1165
15

(Idea that viral delivery methods allow higher success of functional gene
integrating with host cell genome as compared to the liposomal method)
D7 Entry into cell Virus enters cell via endocytosis, Fusion of lipid bilayer of liposome
delivering normal functional with cell membrane leads to delivery
CFTR allele into target cell. of normal functional CFTR allele into
cell.
Similarities
S1 Integration Liposomes and adenoviruses not able to integrate normal functional
CFTR allele into genome (extrachromosomal) / do not cause insertional
mutagenesis;

And hence the treatment is short-lived (due to the transient nature of the
gene that has been inserted into the cell cytoplasm);

S2 Duration of Treatment is transient as epithelial cells are constantly being shed, so


therapeutic repeated therapy is needed;
effects
S3 Target organs Cystic fibrosis affects other organs that are less accessible e.g.,
pancreas and small intestines, and the epithelial cells of the pancreas is
a challenge to target by both liposomal or viral delivery;
(Idea that it is difficult to target all the affected organs)
S4 Target cells Difficult to target the stem cells of the epithelium of every organ
(Idea that the inserted gene is not inheritable because the gene is only
delivered to somatic cells)
S5 In vivo / ex Both systems are carried out using an in vivo approach because it is
vivo approach difficult to extract epithelial cells from respiratory tract.
S6 Level of Fine-tuning the expression of normal functional CFTR allele within target
expression cells is difficult; under-expression of normal functional CFTR allele in
target cells would render gene therapy ineffective.

(b)
1 RFLP is a phenomenon whereby various lengths of restriction fragments are
generated from DNA sequences using restriction enzymes
+ arises due to different nucleotide sequences or / variable number of tandem
repeats (VNTR);
2 RFLP analysis: involves amplification by PCR cutting by restriction enzymes
separation by gel electrophoresis transfer by Southern blotting + nucleic acid
hybridization with radioactive probe;
3 Definition of RFLP marker: a segment of DNA found at a specific site along a
chromosome and can be uniquely recognized by probes;
+ RFLP markers are inherited in a Mendelian fashion;
4 RFLP analysis can be performed on RFLP markers located on two different loci of
the genome
+ 2 individuals, each being homozygous for each RFLP marker, but have different
band patterns for each particular RFLP marker, can be crossed to produce
heterozygous offsprings;
5 Heterozygotes are then crossed with one of the homozygous parent
+ offspring can inherit pairs of RFLP markers like the parents or inherit
recombinant pairs of RFLP markers;

H2 Biology / 9648 / 03
1166
16

6 If the 2 gene loci are not linked, number of offspring showing the four phenotypes will
be of ratio 1:1:1:1;
7 If the 2 gene loci are linked without crossing over, there will only be 2 phenotypes
observed
+ as the two gene loci will assort and segregate together;
8 If the two gene loci are closely linked, the proportion of offspring with recombinant
phenotypes will be less than parental phenotypes;
9 If the two gene loci are linked, frequency at which genetic markers are inherited together
with gene loci, can be used to calculate distance between both gene loci based on
frequency of crossing-over;
10 By performing analysis on many other pairs of RFLP markers and with other
cytogenetic studies, the researcher is able to come up with a detailed map of the
genome;

(c)
1 Fairness in the use of genetic information by insurers, employers, courts, schools,
adoption agencies, and the military, among others / Discrimination by institutions ;
2 Privacy and confidentiality of genetic information;
3 Psychological impact and stigmatization due to an individual's genetic differences /
Discrimination by individuals;
4 Reproductive issues including adequate informed consent for complex and potentially
controversial procedures / use of genetic information in reproductive decision making /
reproductive rights;
5 Clinical issues including the education of doctors and other health service providers,
patients, and the general public in genetic capabilities, scientific limitations, and social
risks / implementation of standards and quality-control measures in testing procedures;
6 Uncertainties associated with gene tests for susceptibilities and complex conditions
(e.g., heart disease) linked to multiple genes and gene-environment interactions;
7 Conceptual and philosophical implications regarding human responsibility, free-will
versus genetic determinism / concepts of health and disease / Religious concerns,
doctrinal issues ;
8 Health and environmental issues concerning genetically modified foods (GM) and
microbes;
9 Commercialization of products including property rights (patents, copyrights, and trade
secrets) and accessibility of data and materials;

H2 Biology / 9648 / 03
1167

PIONEER JUNIOR COLLEGE


JC2 PRELIMS EXAMINATIONS 2013

Candidate
Name

Civics INDEX
Group NUMBER

H2 BIOLOGY 9648 / 01

27 September 2013 1 hour 15 mins

Additional Materials: Multiple Choice Answer Sheet


Soft clean eraser
Soft pencil (type B or HB is recommended)

READ THESE INSTRUCTIONS FIRST

Write in soft pencil.


Do no use staples, paper clips, highlighters, glue or correction fluid.
Write your name, class and index number on the cover page in the spaces provided
unless this has been done for you.

There are forty questions on this paper. Answer all questions. For each question
there are four possible answers A, B, C and D.
Choose the one you consider correct and record your choice in soft pencil on the
OTAS answer sheet provided.

Each correct answer will score one mark. A mark will not be deducted for a wrong
answer. Any rough working should be done in this booklet.

Do not open this booklet until you are told to do so.

This document consists of 23 printed pages (inclusive of this page)

[Turn Over]

1
1168

1. The table gives description of four membranous structures in a cell.

Which structure is correctly matched to its function?

Structure Function
an extensive network of tubes and sacs; each Lipid synthesis
A
tube and sac bounded by a single membrane
a spherical sac bounded by a single Protein synthesis
B
membrane
a sac bounded by two membranes, the inner Packaging of proteins
C
highly folded
a stack of elongated, curved sacs; each sac Photosynthesis
D
bounded by a single membrane

2. The diagram shows a section of a cell surface membrane.

What causes the phospholipid molecules to be arranged as shown?

A The heads are hydrophilic and point towards water molecules.


B The heads are hydrophobic and point away from water molecules.
C The tails are hydrophilic and point away from water molecules.
D The tails are hydrophobic and point towards water molecules.

3. Which carbohydrate has glycosidic bonds between carbon atom 1 and carbon
atom 6 of its component glucose molecules?

A amylose
B glycogen
C maltose
D sucrose

2
1169

4. Compared to globular proteins, fibrous proteins are

I. more regular in structure


II. mostly involved in metabolic activities
III. more readily soluble
IV. more resistant to high temperature

A I and II only
B I and IV only
C II and III only
D I, III and IV only

5. In an investigation to determine the effect of temperature on activity of an


enzyme, the time for all the substrate to disappear from a standard solution was
recorded.

Which graph shows the results of this investigation?

A B

Time taken for


completion of Time taken for
the reaction completion of
the reaction

10 20 30 40 50 60 10 20 30 40 50 60
Temperature / C Temperature / C

C D

Time taken for Time taken for


completion of completion of
the reaction the reaction

10 20 30 40 50 60 10 20 30 40 50 60
Temperature / C Temperature / C

3
1170

6. The amount of DNA per nucleus in a cell during a meiotic cell cycle as shown
below was measured.

Which bar correctly represents the variation in DNA content?

4
1171

7. The packing of DNA in the nucleus is a very precise process. The following
statements describe this process.

I. DNA winds twice around a histone to form nucleosome.


II. Coiling results in formation of condensed chromatin as seen in
metaphase.
III. Looped domains are formed with the aid of chromosome scaffold.
IV. Coiling results in formation of a solenoid fibre.

Which of the following correctly describes the sequence of DNA packing?

A I, III, IV, II
B I, IV, III, II
C III, I, IV, II
D III, IV, I II

8. The diagrams show an investigation into semi-conservative replication of DNA.

Which tube shows the position of the DNA after two more generations of semi-
conservative replication in light nitrogen (14N)?

5
1172

9. Which of the following is not involved in regulating the synthesis of RNA in the
eukaryotic nucleus?

A DNA methylation of DNA regions where genes are active.


B Amplification of some genes such as rRNA genes.
C Spliceosomes that splices introns from pre-mRNA to form mature mRNA.
D Use of different RNA polymerases to transcribe different classes of RNA.

10. Both -actin and D-3-phosphoglycerate dehydrogenase (PHGDH) are proteins


that can be found in the human body. Multiple tissues from the same individual
were taken, followed by the isolation of the respective mRNA from the same
number of cells of each tissue type. The mRNA were then subjected to gel
electrophoresis. The following autoradiograph shows the result of this study.

From the figure above, which conclusion cannot be deduced from these results?

A The gene that codes for PHGDH cannot be found in all types of cells.
B Activators are bound to enhancers of the gene coding for -actin in skeletal
muscle cells.
C The gene coding for -actin is located in euchromatin.
D Inhibitors are bound to silencers of the gene codng for PHGDH in lung cells.

6
1173

11. Which statement defines control elements?

A A segment of DNA where transcription factors bind preferentially.


B A short region of DNA that can bind with proteins to enhance transcription levels.
C DNA sequences that interact with regulator proteins to determine the rate and
timing of gene expression.
D Proteins found only in eukaryotes that regulate transcription by binding to DNA
sequences to control the transfer of genetic information from DNA to mRNA.

12. Which of the following correctly describes the function of components involved in
translation?

Aminoacyl-tRNA
Peptidyl transferase Ribosomes
synthetase
Contains an aminoacyl
Transfers an amino acid
site that is occupied by Attaches an amino acid
A to a growing polypeptide
the initiator tRNA to the 5 end of a tRNA
chain
attached to methionine
Contains a peptidyl site
Forms a peptide bond
that is occupied by the Attaches an amino acid
B between two amino
initiator tRNA attached to the 5 end of a tRNA
acids
to methionine
Contains a peptidyl site
Forms a peptide bond
that is occupied by the Attaches an amino acid
C between two amino
initiator tRNA attached to the 3 end of a tRNA
acids
to methionine
Contains an aminoacyl
Transfers an amino acid
site that is occupied by Attaches an amino acid
D to a growing polypeptide
the initiator tRNA to the 3 end of a tRNA
chain
attached to methionine

13. A point mutation can occur anywhere on a chromosome. Which statements are
true?

I. Point mutation in an exon can alter the codon resulting in a different


amino acid sequence.
II. Point mutation in an exon can produce a shorter protein as a stop codon
is produced.
III. Point mutation in an intron can alter the binding site of a splicing enzyme.

A I and II
B I and III
C II and III
D I, II and III

7
1174

14. The following figure illustrates the reproductive cycle of a virus.

W X Y Z

A viral DNA prophage lytic lysogenic


B viral RNA provirus lysogenic lytic
C bacterial DNA provirus lysogenic lytic
D bacterial DNA prophage lytic lysogenic

8
1175

13

18 15.
TheThe diagram
diagram below below
shows shows the structure
the structure of M13
of an M13 bacteriophage.
bacteriophage. It hasIt ahas single-
single-stranded
stranded circular DNA genome. g3p, g6p, g7p, g8p and g9p are capsid proteins.
circular DNA genome. g3p, g6p, g7p, g8p and g9p are capsid proteins. Its life cycle does
not Its life cycle
cause does
the lysis of not cause
its host the lysis of the host cell.
cell.

Based
Based on on your
your understanding
understanding of
of bacteriophages,
bacteriophages, which
which of
of the
the following
following statements
statements are
are true of the M13 bacteriophage?
true of the M13 bacteriophage?

I I. The Thebasebase composition


composition of its of its genome
genome is that
is such suchthe
that theofrelative
ratio ratio of C:G
A:T is 1:1.
II is 1:1.
At least one of the capsid proteins is responsible for binding to a specific protein on
II. the host
At least
cell.one of the capsid proteins is responsible for binding to a specific
III Its genomeon
protein is the host cell.
injected into the host cell after the phage has attached to the host
III. cell.Its genome is injected into the host cell after the phage has attached to
IV the hostitscell.
It acquires envelope from the cell membrane of its host cell.
IV. It acquires its envelope from the cell membrane of its host cell.
A I and III
A I and III
BB II and
II and III III
CC I, III,and
II andIII III
D II, III and IV
D II, III and IV

9
ACJC 9648/01/Prelim 2012 [Turn Over
1176

16. The photomicrographs below show two different processes of genetic transfer in
bacteria.

A series of statements concerning the two processes is shown below.

I. Physical contact between the donor and recipient bacterium is necessary.


II. The donor bacterium is lysed in the process.
III. Specific genes in the donor bacterium are required for the transfer to take
place.
IV. Any gene on the bacterial chromosome can be transferred.

Which row shows the correct statements for the corresponding process?

Process R Process S

A I and II III and IV


B I and III II and IV
C II and III I and IV
D III and IV I and II

10
1177

17. In prokaryotes, arginine operon is a negative repressible operon. A repressor,


coded by a regulatory gene, controls the operon. Which of the following is
correct?

Absence of arginine Presence of arginine


Arginine binds to the repressor,
Repressor has high affinity for
A resulting in low affinity of the
operator
repressor for the operator
Arginine binds to the repressor and
Repressor has low affinity for
B activates it, causing it to bind to the
operator
operator
Causes bend in the promoter such
Continuous transcription of genes to
C that RNA polymerase has increased
synthesize arginine
affinity for the promoter
No synthesis of genes to break down Transcription of genes to synthesize
D
arginine enzymes to break down arginine

18. Red-green colour blindness is controlled by a gene on the X chromosome. The


allele for colour blindness, g, is recessive to the allele for normal colour vision, G.

Complete colour blindness is controlled by a different gene which is not on the X


chromosome. The blindness
22 Red-green colour allele for the development
is controlled by a gene onofthenormal cones The
X chromosome. (pigment
allele forcells
colourin
the retinal layer
blindness, g, isofrecessive
the eye), B,allele
to the is dominant
for normal to the vision,
colour alleleG.forComplete
no cone development,
colour blindness is
b. controlled by a different gene which is not on the X chromosome. The allele for the development of
normal cones (pigment cells in the retinal layer of the eye), B, is dominant to the allele for no cone
development, b.
The figure below shows the phenotypes of members of a family in which both
typesThe
of figure
colour below shows theoccur.
blindness phenotypes of members of a different family in which both types of
colour blindness occur.

P Q
Red-green colour blind Completely colour blind

R S
Normal vision Red-green colour blind

female
male

Which of the following are possible genotypes for individuals P, Q and S?


Which of the following are possible genotypes for individuals P, Q and S?
P Q S
A BBXGXG bbXGY BBXgY
B P BbXGXg QBbXgY SGY
BbX
C BBXgXg bbXGY BbXgY
A BBXGXG G g bbXGY g BBXGgY
D G gBbX X BbX
g Y BBX GY
B BbX X BbX Y BbX Y
g g G g
C BBX X bbX Y BbX Y
G g g G
D 23 BbX
In the fruit fly, X
recessive mutations in either BbX Y
of 2 independently assorting genes,BBX
brown Y
and purple,
prevents the synthesis of red pigments in the eyes. Thus, homozygotes of either of these mutations
have brownish-purple eyes. However, heterozygotes for both of these mutations have dark red,
that is, wild-type eyes.
11
Crossing two flies which are both homozygous for a different mutation and heterozygous for the
other corresponding mutation produced 144 offspring. How many offspring would be expected to
be mutants?
1178

19. In Drosophila melanogaster, there are two known independently assorting genes
B and P that affect wing shape. Flies homozygous for the recessive b mutant
allele at one gene have bent wings. Flies homozygous for the recessive p mutant
allele have no wings at all, regardless of their genotype at other loci. All other
genotypes give normal wings. If a true breeding bent winged fly is crossed with a
fly with no wings that is homozygous for the normal B allele and the p allele, what
proportion of the F2 would have no wings?

A 1/16
B 1/4
C 3/16
D 9/16

20. When a pure-breeding blue budgerigar is crossed with a pure-breeding yellow


budgerigar, all the budgerigars of the F1 generation are green. When the F1
generation was crossed, there were 40 progeny in the F2 generation. 3 were
white, 22 were green, 8 were yellow and 7 were blue.

What are the genotypes of the parental generation?

A AaBb x AaBb
B aaBb x Aabb
C aaBB x AAbb
D aabb x AABB

21. A maize plant with green seedlings and opaque leaves was crossed with a maize
plant with yellow seedlings and bright leaves. The table below shows the
phenotypes of the offspring.

Green and opaque leaves 240


Green and bright leaves 36
Yellow and opaque leaves 24
Yellow and bright leaves 220

Which statement best explains these results?

A The genes for seedling colour and leaf brightness are linked and crossing over
has taken place in 11.5% of the offspring.
B The genes for seedling colour and leaf brightness are on different chromosomes
and crossing over has taken place in 11.5% of the offspring.
C The genes for seedling colour and leaf brightness are linked and 53.8% of the
offspring contain at least 1 dominant allele.
D The genes for seedling colour and leaf brightness are on different chromosomes
and 6.9% of the offspring express only two recessive alleles.

12
1179

22. The table below shows the results of an early investigation into the genetic
control of phenotypic variation. The dry masses of 5493 bean seeds collected
from many plants were classified into nine categories.

Mass of 51- 151- 251- 351- 451- 551- 651- 751- 851-
bean/mg 150 250 350 450 550 650 750 850 950
Number
5 38 370 1676 2255 928 187 32 2
of beans

Which statement correctly describes these data and could account for the variation
shown?

A The phenotypic variation is continuous and could be the result of two non-linked
genes acting on their own.
B The phenotypic variation is continuous and could be the result of several non-
linked genes acting on their own.
C The phenotypic variation is discontinuous and could be the result of two linked
genes acting on their own.
D The phenotypic variation is discontinuous and could be the result of several
linked genes acting on their own.

23. In the presence of a signal protein, the Ras protein, which is a G-protein,
becomes activated and switches on other proteins. These in turn switch on genes
that lead to cell growth and division. When the Ras gene is mutated, the Ras
protein will prevent GTP hydrolysis, resulting in unintended and overactive
signaling inside the cell. This happens even in the absence of incoming signals,
leading to increased cell growth and division.

Which of the following describes the type of mutation in the Ras gene?
A Gain of function, point mutation
B Loss of function, chromosomal translocation
C Gain of function, chromosomal translocation
D Loss of function, point mutation

13
1180

24. The graph below shows how various factors affect the rate of photosynthesis.

It can be deduced from the data shown in the graph above that the rate of
photosynthesis
A varies directly with carbon dioxide concentrations.
B varies directly with light intensity.
C is dependent on light intensity at high carbon dioxide concentrations.
D is unaffected by carbon dioxide concentration at low light intensity.

25. The light dependent reaction consists of cyclic and non-cyclic


photophosphorylation. Which statement(s) about photophosphorylation is false?

I. It takes place on the thylakoid membrane.


II. NAD+ is the final electron acceptor for non-cyclic photophosphorylation.
III. ATP is synthesized during both cyclic and non-cyclic
photophosphorylation.
IV. Photolysis takes place during both cyclic and non-cyclic
photophosphorylation.

A I only
B II only
C I and III
D II and IV

14
1181

26. The diagram below shows what happens to pyruvate in the root cells of plants in
waterlogged soil.

W Y

X Z

Which of the following correctly explains the reaction above?

W X Y Z
A pyruvate lactate alcohol ethanol
dehydrogenase decarboxylase
B pyruvate ethanal alcohol ethanol
decarboxylase dehydrogenase
C pyruvate ethanol alcohol ethanal
decarboxylase dehydrogenase
D pyruvate ethanal alcohol ethanol
dehydrogenase decarboxylase

27. In aerobic respiration, ATP is synthesized by substrate level phosphorylation


during glycolysis and the Krebs cycle and by oxidative phosphorylation involving
hydrogen carriers.

2.5 molecules of ATP can be made from each reduced NAD and 1.5 molecules
of ATP from each reduced FAD.

What is the maximum number of ATP molecules that can be made during the Krebs
cycle from two molecules of glucose?

A 18
B 26
C 36
D 56

15
1182

28. The diagram below shows a tyrosine kinase receptor on a liver cell.

Extracellular Matrix

Cytosol

What will be the effects when an insulin molecule binds?

Glycogenolysis Glycogenesis Gluconeogenesis


A increase increase increase
B increase decrease increase
C decrease increase decrease
D decrease decrease decrease

29. The figure below shows the relative irritability of a stimulated nerve fibre.

What would happen if a second stimulus is applied during period XY?


A The action potential will take a longer time to be generated.
B The reaction to the stimulus will be greater than normal.
C The nerve fibre will respond repeatedly to a single stimulus.
D A stimulus below the normal threshold will cause an action potential.

16
1183

30. What is the correct sequence of events in the signal transduction pathway when
insulin interacts with receptor tyrosine kinases?

I. Activation of tyrosine kinase in tails


II. Dimerisation of receptor polypeptides
III. Autophosphorylation of tyrosine molecules on the cytoplasmic tail
IV. Activation of series of relay proteins

A I, II, III, IV
B II, III, I ,IV
C II, I, III, IV
D I, III, II IV

31. The chart below shows the hierarchical classification of two species of crocodiles.

To which class do these crocodiles belong?

A Chordata
B Crocodylidae
C Loricata
D Reptilia

17
1184

32. The diagram illustrates variation in the pericarp (fruit wall) for a variety of
methods of seed dispersal.

How do these examples support Darwins theory of natural selection?

A They illustrate how analogous structures have become adapted to perform


different functions.
B They illustrate how groups with separate evolutionary origins differ over time as a
result of divergent evolution.
C They illustrate how biotic factors operating on a variable genotype can bring
about evolutionary change.
D They illustrate the principle of adaptive radiation suggesting the divergent
evolution of homologous structures.

33. The average mutation rate of a gene can be used as a molecular clock. Which
type of mutation would cause an underestimate of the time that has elapsed
since two related species diverged from their common ancestor?

A a mutation to the first base of a nucleotide codon


B a mutation to the third base of a nucleotide codon
C a mutation that returns a mutated nucleotide to its original state
D a mutation that substitutes a different amino acid in place of the original amino
acid

18
1185

34. Which pattern of mutation rate would be most helpful if one desires to use a gene
as a molecular clock to determine evolutionary relatedness of species that are
closely related to each other?

19
1186

35. A gene coding for the production of a human gene product was inserted into a
plasmid with genes coding for resistance to antibiotics ampicillin, streptomycin
and tetracycline. The plasmids were used to transform E.coli and the bacteria
grown on a nutrient medium. The resulting master plate is shown in the diagram.

Transformed cells were selected by replica plating the bacteria colonies onto media
containing various antibiotics.

Which plate contains the colony of bacteria into which the human gene has been
successfully inserted?

36. The construction of a cDNA library involves a number of stages. What is the
correct sequence of stages?

mRNA is isolated inserted into a mRNA is converted single-stranded


from cells vector and cloned into single-stranded cDNA is hybridized
cDNA with a primer
A 1 2 4 3
B 2 1 3 4
C 3 4 2 1
D 1 4 2 3

20
1187

37. Sickle-cell anemia is an autosomal, recessive human disease. A hypothetical


pedigree for parents, each heterozygous for the sickle-cell allele, is shown in the
figure below. The inheritance of the normal and sickle-cell allele in this family can
be followed by RFLP analysis. Each of the different genotypes, homozygous
normal (AA), heterozygous carrier (Aa), and homozygous recessive with sickle-
cell anemia (aa) can be identified by RFLP analysis.

Based on the above analysis, which of the following statements is true?

A The sickle-cell anaemia is caused by a mutation resulting in a gain of restriction


site.
B The sickle-cell anaemia is caused by a mutation resulting in a loss of restriction
site.
C The normal allele is cut using a different enzyme from that of the sickle-cell allele,
hence resulting in different number of fragments produced.
D The normal allele is a result of cutting at more than three restriction sites.

38. A patient suffers from Marfan Syndrome, an autosomal dominant genetic disorder
in the gene coding for fibrillin-1 (FBN1). Almost all cells express FBN1 as its
product is involved in the maintenance of connective tissues, which are abundant
throughout the body. Which of the following treatment would permanently cure
this patient?

A Somatic cell gene therapy to introduce a dominant allele in a non-specific location


B Somatic cell gene therapy to insert the functional allele in a non-specific location
C Germ-line cell therapy to replace the faulty FBN1 gene
D None of the above

21
1188

39. Callus tissue is produced using leaves by the technique shown in the diagram
below. Using this, a student investigated the effect of two different concentrations
of auxin on the volume of callus material produced. The table shows the results
obtained over five days.

volume of callus material (cm3)


Time
(day) low concentration high concentration
of auxin added of auxin added
0 1.0 1.0
1 1.3 1.7
2 1.9 4.1
3 3.0 8.3
4 3.9 10.6
5 5.4 12.9

Which of the following statements about the experiment is false?

I. The leaves are disinfected and sterilized by washing with detergent and
alcohol.
II. The sterilization process can alternatively be done using an autoclave
machine.
III. The only nutrient required to stimulate callus formation is auxin.
IV. Continued subculturing at three- to four-week intervals of small cell
clusters taken from calli can maintain the callus cultures for long periods
of time.

A II only
B I and IV only
C III only
D II and III only

22
1189

40. Salmon has been genetically modified to increase its yield within a shorter period
of time. Which of the following is incorrect?

A Growth hormone gene from another species of salmon is inserted to greatly


enhance growth.
B Gene coding for antifreeze protein is inserted to allow salmon to grow faster
despite the cold waters.
C Transgenic salmon are allowed to mate with wild fish to produce stable
transgenic lines in fish hatcheries.
D The increase in yield of transgenic salmon is due to its ability to grow throughout
the year and increased efficiency in feed conversions.

1 A 21 A
2 A 22 B
3 B 23 A
4 B 24 C
5 C 25 D
6 B 26 B
7 B 27 C
8 D 28 C
9 C 29 D
10 A 30 C
11 C 31 D
12 C 32 D
13 D 33 C
14 D 34 A
15 B 35 A
16 B 36 D
17 B 37 B
18 C 38 D
19 B 39 D
20 C 40 B

23
1190

Section A: Structured Questions [80 marks]



Question 1 [10 marks]

(a) Figure 1.1 shows part of the cell body of a neurone.

Figure 1.1

(i) Identify the organelle labeled A. [1]

..

(ii) Relate the importance of this organelle to the cell. [2]

..
..
..
..
1191

The
e following structure
s in Figure 1.2 iis also foun on of a neurrone.
nd in the axo

Figure 1.2

(iii) Identify the


e structures
s as indicate e diagram. [11]
ed by the arrrows in the






..

(iv) In what way


w is this structure related to the functio
on of orgaanelle A in the
neurone? [1]




..




..

 3
1192

(b) Figure 1.3 shows a cholinergic synapse of the neurone.

Figure 1.3

(i) Identify process B. [1]

..

(ii) With your understanding of the cell membrane, explain how process B occurs.
[1]

..
..

(iii) Suggest the significance of this process with regard to synaptic transmission. [1]

..
..

(iv) On the pre-synaptic membrane, there are also Ca2+ voltage-gated channels
found. Describe how a change in potential difference will result in the diffusion of
Ca2+. [2]

..
..
..
..

 4
1193

Que
estion 2 [13
3 marks]

(a) Figure 2.1 below show


ws the transscription initiation proce
ess.



      Figure 2.1

(i) Identify strructures A and


a B [1]

(ii) With referrence to Figure


F scription iss initiated and
2.1,, describe how trans
regulated. [3]






..






..






..






..






..






..

 5
1194

(b) Figure 2.2 is an elec


ctron microg
graph depic
cting the process of ttranslation in a
prokaryote..


Figure 2.2

(i) Identify strructures C and


a D. [1]

(ii) Describe the importan


nce of struccture D in tra
anslation. [1
1]




..






..

(iii) With referrence to Fig


gure 2.2, ssuggest a difference
d between
b thee translatio
on in
prokaryote
es and eukaaryotes. [1]






..






..

(c) Different genetic muttations on a gene an nd its mRN NA may ressult in diffe
erent
diseases. For
F example ation of -globin mRNA
e, RNA splicce site muta A is responssible
of some cases
c of -thalassem ia and a point mutation on -gglobin gene is
responsible
e for sickle cell
c anaemiia.

(i) Explain the


e significanc
ce of the po int mutation
n to the prop
perties of haaemoglobin
n. [3]






..






..






..






..






..






..

 6
1195

(ii) Explain why a point mutation on a -globin gene does not always produce
mutated globin whereas an RNA splice site mutation will always produce
mutated globin. [3]

..
..
..
..
..
..

 7
1196

Que
estion 3 [13
3 marks]

Figu
ure 3.1 show
ws an electtron microgrraph of H1N
N1.



Figure 3.1

(a) Identify the


e componen
nts labeled A and B. [3]]

(b) RNA-depen ndent RNA A polymera se can be e found in this virion while reverse
transcriptasse can be found
f in rettrovirus. Be ype of viruss, describe one
esides the ty
similarity and
a two diffferences bbetween RN NA-depende ent RNA p olymerase and
reverse transcriptase. [3]






..






..






..






..






..






..

 8
1197

Ose
eltamivir, be
etter knownn under its ttrademark name
n as Taamiflu, is a recommen nded
med
dication for H1N1. Its mechanism
m s illustrated in Figure 3..2 and 3.3.
of action is

Figure 3.2
2

Figure 3.3
3

(c) Explain how


w oseltamiv
vir affects th
he reproduc
ction cycle of
o H1N1. [2]]






..






..






..






..

 9
1198

The effects of oseltamivir were studied on patients infected with influenza virus.
These patients were randomly separated into two groups one group administered
with a placebo (a pseudo-drug with no effect) while the other group was administered
oseltamivir. Symptoms were measured and recorded in Table 3.4.

Study Groups
Symptoms
Placebo (n=129) Oseltamivir, 75mg (n=124)
Cough
Duration, hr 55 (36-73) 31 (24-42)
Severity, arbitrary units 110 67
Nasal Obstruction
Duration, hr 43 (31-64) 33 (25-43)
Severity, arbitrary units 80 60
Sore Throat
Duration, hr 21 (8-29) 10 (4-19)
Severity, arbitrary units 29 20
Fatigue
Duration, hr 41 (26-50) 24 (19-34)
Severity, arbitrary units 89 54
Feverishness
Duration, hr 23 (16-29) 10 (8-14)
Severity, arbitrary units 49 21
Table 3.4

Table abstracted from: http://jama.jamanetwork.com/article.aspx?articleid=192425,


Efficacy and Safety of the Oral Neuraminidase Inhibitor Oseltamivir in Treating
Acute Influenza - A Randomized Controlled Trial

(d) With reference Table 3.4, determine the effectiveness of oseltamivir. [2]

..
..
..
..

(e) In a separate case, a patient was treated for H1N1. During this treatment, he was
infected with H3N3. Upon testing, it was found that his blood now contains H3N3
and a new strain of virus, H1N3. Explain how this new strain could have arisen.
[3]

..
..
..
..
..
..

 10
1199

Question 4 [10 marks]

(a) In maize plants, a recessive mutant allele leads to green leaves as opposed to
bronze leaves. Another recessive mutant allele at a separate locus leads to
yellow cobs as opposed to brown cobs. F1 maize plants from a standard dihybrid
cross was test crossed with a plant with green leaves and yellow cobs. The table
shows the phenotypes of 200 of the offspring.

Bronze leaves and yellow cobs 88

Bronze leaves and brown cobs 12

Green leaves and yellow cobs 14

Green leaves and brown cobs 86

(i) In the space below, draw a genetic diagram to explain the observed results of
the cross. [4]

 11
1200

(ii) Explain the results of the test cross. [3]

..
..
..
..
..
..

(b) A chi-squared test was also carried out for the above cross and the chi-square
value was 109.6. Using the probability table given, does this value for the chi-
squared test support the expected phenotypic ratio? Explain. [3]

..
..
..
..
..
..

 12
1201

Que
estion 5 [13
3 marks]

(a) Figure 5.1 below show


ws the blood
d glucose co
oncentration measuredd.

Figure 5.1
1

(i) Identify wh
hich individu
ual has a no eostatic systtem in contrrolling blood
ormal home d
glucose levvel. Explain
n your answ
wer. [2]






..




..




..






..

(ii) Describe and on of person C in the period


a explain the conditio p betweeen 0 1.0
0
hours. [2]






..






..






..






..

(iii) State the type


t of receeptor that glu
ucagon bind
ds to and ex
xplain how this recepto
or is
the fully acctivated. [2]




..






..




..






..

 13
1202

(b) Ras is a GTTP-binding protein thatt helps to broadcast signals from the cell surrface
arts of the cell to stimu late cell pro
to other pa oliferation. If Ras functtion is inhib
bited,
the cell pro
oliferation responses
r nnormally induced by the activateed receptors s do
not occur. Conversely y, if a hypeeractive mu utant, Ras protein, is introduced into
some cell lines, the efffect on celll proliferatio
on is the same as that induced by y the
binding of ligands to cell surface e receptors. This hype eractive prooduct of mu utant
ras gene prromotes the e developm ent of canc cer.

Figure 5.2 below sho


ows the seq
quence of events
e in a Ras cell cyycle-stimula
ating
pathway.
 

Figure 5.2

(i) Briefly desscribe the sttages A and


d B shown in Figure 5.2
2. [4]






..






..






..






..






..






..






..






..

(ii) Ras protein is a plasm


ma membra ane bound G-protein.
G Suggest
S how w the activa
ated
G-protein can
c relay th he signal fro
om the ligan
nd to activatte cellular reesponses. [2]
[






..






..






..






..

 14
1203

(c) Suggest on
ne function of
o signal tra
ansduction. [1]






..






..

Que
estion 6 [10
0 marks]

Figu
ure 6.1 show
ws an electtromicrograp oroplast
ph of a chlo


B

     Figure 6.1

(a) Identify the


e features: [2]
[

A


. B


.

C


. D


.

 15
1204

(b) The blue dye DCPIP can be converted to colourless reduced DCPIP by gaining
electrons.
A suspension of chloroplasts was made by grinding fresh leaves in buffer solution
and centrifuging the mixture. Tubes were then prepared and treated in the
following way:

Tubes Content Treatment Colour


Start After 20 min
A 1 cm3 chloroplast Illuminated Blue-green Green
suspension strongly
5cm3 DCPIP

B 1 cm3 buffer solution Illuminated Blue blue


5cm3 DCPIP strongly

C 1 cm3 chloroplast Left in the dark Blue-green Blue-green


suspension
5cm3 DCPIP

(i) Explain this observation using your knowledge on light-dependent reactions of


photosynthesis. [4]

..
..
..
..
..
..
..
..

(ii) Explain, using tubes A and C, the effects of illumination on Calvin cycle (dark
reaction). [2]

..
..
..
..

 16
1205

(iii) The chloroplast suspension may be contaminated with mitochondria. Explain the
evidence from the investigation that presence of mitochondria was not
responsible for the reduction of DCPIP. [2]

..
..
..
..

Question 7 [11 marks]

The Galpagos tortoise or Galpagos giant tortoise (Chelonoidis nigra) is the largest
living species of tortoise and 13th-heaviest living reptile, reaching weights of over
400 kg and lengths of over 1.8 metres. With life spans in the wild of over 100 years, it
is one of the longest-lived vertebrates.

The tortoise is native to seven of the Galpagos Islands, a volcanic archipelago


about 1,000 km west of the Ecuadorian mainland.

Shell size and shape vary between populations. On islands with humid highlands, the
tortoises are larger, with domed shells and short necks. Due to the climate, there is
abundant vegetation and vegetation is closer to the ground. On islands with dry
lowlands, the tortoises are smaller, with "saddleback" shells, which are flatter than
domed shells, and long necks. There is less vegetation. Charles Darwin's
observations of these differences on the second voyage of the Beagle in 1835,
contributed to the development of his theory of evolution.

Tortoise numbers declined from over 250,000 in the 16th century to a low of around
3,000 in the 1970s. This decline was caused by exploitation of the species for meat
and oil, habitat clearance for agriculture, and introduction of non-native animals to
the islands, such as rats, goats, and pigs.

(a) (i) Explain how the distinct phenotypic differences between the populations may
have arisen. [4]

..
..
..
..
..
..
..
..

(ii) State the phenomenon that describes the decline in tortoise numbers from the
16th century to the 1970s. [1]

..

 17
1206

(iii) Suggest a disadvantag


d ge to a redu
uction in population siz
ze. [1]




..




..

Figu
ure 7.1 beelow shows the rela ationship beetween number of ssubstitutions
s in
cyto
ochrome c between
b related pairs of species and how lo
ong ago thee two compa
ared
species had divverged from
m a common n ancestor.

Figure 7.1

(b) Describe and explain the


t conclussions that ca n from Figurre 7.1. [3]
an be drawn






..






..




..






..






..






..

(c) Suggest the s that arise when using


e difficulties g the molecular clock. [[2]






..






..






..






..

 18
1207

Section B: [20 marks]

Answer only ONE question.

Write your answers on the separate writing paper provided. Your answers should be
illustrated by large, clear labeled diagrams, where appropriate. Your answers must
be in continuous prose, where appropriate.
Your answers must be set out in sections (a), (b) etc., as indicated in the question.

Question 8

(a) Explain how the structure of triglyceride is related to its function. [7]

(b) Compare and contrast between collagen and cellulose. [7]

(c) Some enzymes function optimally in acidic conditions, while other enzymes
function optimally in alkaline conditions. Explain why different enzymes have
different optimal pH. [6]

Question 9

(a) Explain the role of the F plasmids in bacterial conjugation and suggest the
advantages of conjugation over transduction. [8]

(b) Explain how the lac operon is regulated by both negative and positive control. [8]

(c) Suggest why operons are necessary in bacteria. [4]

 19
1208

Section A: Structured Questions



Question 1 [10 marks]

(a) Figure 1.1 shows part of the cell body of a neurone.

Figure 1.1

(i) Identify the organelle labeled A. [1]

Golgi Apparatus

(ii) Relate the importance of this organelle to the cell. [2]

a. Post-translational modification by chemical/physical modification


b. e.g. modifying proteins that would form the ion channels in the membrane
of the neurone
c. synthesizing and packaging neurotransmitters to be sent to the synaptic
knob
d. ion channels/transmembrane transport proteins are embedded in
membrane of golgi vesicle which also replenishes the membrane at the
synaptic knob
1209

The
e following structure
s in Figure 1.2 iis also foun
nd in the axo
on of a neurrone.

Figure 1.2

(iii) Identify the


e structures
s as indicate e diagram. [11]
ed by the arrrows in the

crotubules
Mic

(iv) In what wa
ay is this strructure relatted to the fu
unction of organelle A iin the
neurone? [1]

ort golgi ves


a. to transpo sicles to th
he respectiive parts off the neuro
one
b. with the us
se of ATP OR
O
c. repair and maintenan nce of the m
membrane e
1210

(b) Figure 1.3 shows a cholinergic synapse of the neurone.

Figure 1.3

(i) Identify process B. [1]

Exocytosis

(ii) With your understanding of the cell membrane, explain how process B occurs.
[1]

a. Phospholipid bilayer is fluid due to weak hydrophobic interactions between


hydrophobic hydrocarbon tails,
b. allowing synaptic vesicles can thus fuse with the pre-synaptic membrane

(iii) Suggest the significance of this process with regard to synaptic transmission. [1]

a. Neurotransmitters must be released into the synaptic cleft in bulk to


transmit information from the impulse quickly to the next neurone;;
OWTTE.

(iv) On the pre-synaptic membrane, there are also Ca2+ voltage-gated channels
found. Describe how a change in potential difference will cause them to open. [2]

a. When the potential difference across the membrane is varied from the
resting potential,
b. conformational change in the protein causes the gate to open
c. This allows charged ions to flow down their electrochemical gradient
d. through the hydrophilic channel provided by this transmembrane protein
1211

Question 2 [13 marks]

(a) Figure 2.1 below shows the transcription initiation process.



A

B

      Figure 2.1

(i) Identify structures A and B [1]

A: Enhancer
B: Promoter

(ii) With reference to Figure 2.1, describe how transcription is initiated and
regulated. [3]

a. Binding of transcription factor to TATA box


b. Recruiting of other TFs and RNA polymerase to bind at the promoter to
form transcription initiation complex
c. Transcription can proceed at a basal rate
d. Activator binds to the enhancer
e. DNA bends to allow activator protein to bring it closer to the TIC with the
help of the DNA bending protein
f. Mediator proteins help in ensuring activator binds to TIC
g. RNA polymerase is stabilized/increase affinity to promoter and it is able to
transcribe at a higher rate.
{max 3}
1212

(b) Figure 2.2 is an electro


on microgra
aph depictin
ng the proce
ess of transslation in a
prokaryote..


Figure 2.2

(i) Identify strructures C and


a D. [1]

C: D
DNA
D: P
Polyribosomes

(ii) Describe the importan


nce of struccture D in tra
anslation. [1
1]

a. It increase
es the rate of
o translatiion as man
ny ribosom
mes are tran
nslating the
e
mRNA at the same time;
b. Increase thhe rate of protein
p pro
oduction;

(iii) With refere


ence to Figu
ure 2.2, sug ggest a diffe
erence betw
ween the traanslation in
prokaryote
es and euka aryotes. [1]

a. Translation in prokarryotes occu ur simultan


neously with transcriiption while
e
eukaryoticc transcripttion is in th
he nucleus and transllation occu
urs separattely
in the ribosomes in the
t cytopla asm.

(c) Different genetic muttations on a gene an nd its mRN NA may ressult in diffe
erent
diseases. For
F example ation of -globin mRNA
e, RNA splicce site muta A is responssible
of some cases
c of -thalassem ia and a point mutation on -gglobin gene is
responsible
e for sickle cell
c anaemiia.

(i) Explain the


e significanc
ce of the po int mutation
n to the prop
perties of haaemoglobin
n. [3]

a. Base A waas changed d to base T


b. Hydrophiliic glutamicc acid was replaced with
w hydrop phobic vali ne;
c. Changing the tertiary structu ure of hae emoglobin as differeent bonds are
formed between the R-groups;
d. low oxygen concentrrations, thiis substituttion creates a hydrop phobic spo
ot on
the outside
e of the prootein struc
cture;
e. that stick
ks to the hydropho obic regio on of an adjacent haemoglo obin
moleculess beta chain
f. This clumpping togethher (polym erization) ofo Hb S mo olecules;
g. forms rigid
d fibers res
sulting in c
crystals thaat distorts the
t cell;
h. unable to transport
t oxygen
o effi ciently;
1213

(ii) Explain why a point mutation on a -globin gene does not always produce
mutated globin whereas an RNA splice site mutation will always produce
mutated globin. [3]

a. Point mutation may result in a silent mutation


b. Where a substitution mutation codes for the same amino acid;
c. Due to the degeneracy of the triplet code;
d. An RNA splice site mutation will not allow the spliceosome to recognize
and bind to the splice site;
e. Therefore an intron is not excised leading to a longer mRNA sequence
f. The introns are translated and leading to a different amino acid sequence
(resulting in a mutated globin chain).

Question 3 [13 marks]

Figure 3.1 shows an electron micrograph of H1N1.





    Figure 3.1

(a) Identify the components labeled A and B. [3]

A
a. Neuraminidase;;
b. Haemagglutinin;;

B
Negative/antisense RNA segments;;

(b) RNA-dependent RNA polymerase can be found in this virion while reverse
transcriptase can be found in retrovirus. Besides the type of virus, describe one
similarity and two differences between RNA-dependent RNA polymerase and
reverse transcriptase. [3]
1214

Similarity:
a. Both are viral proteins, i.e. they are only coded for by viral genes and not
the host genes.
b. Both are dependent on RNA as a template

Differences

Points RdRp Reverse Transcriptase


a. Template RNA (both antisense and RNA template, then using
used sense) the single-stranded cDNA
b. Products Single stranded RNA Double stranded cDNA
c. Catalytic RNA polymerase extension Reverse transcription
activity of RNA using host free extension of cDNA
ribonucleotides
Ribonuclease/RNase
degrade RNA in cDNA-RNA

DNA polymerase formation


of complementary DNA to
form double-stranded cDNA

d. Monomers Ribonucleotides Deoxyribonucleotides


1215

Ose
eltamivir, be
etter known under its trrademark na ame as Tam miflu, is a reecommende ed
dication for H1N1. Its mechanism
med m of action is
s illustrated in Figure 3..2 and 3.3.

Figure 3.2

Figure 3.3

(c) Explain how


w oseltamiv
vir affects th
he reproduc
ction cycle of
o H1N1. [2]]

a. Oseltamivir has similar conforrmation an nd chemica al compati bility as sialic


acid
b. It is thus able
a d to the acttive site of neuraminidase
to bind
c. Prevents neuraminid
n dase from c cleaving sialic acid
d. Hence virio on will not be able to
o be release ed via budd ding to infeect other cells,
(preventing the virus s from spreeading to th he rest of the respirattory tract)
1216

The effects of oseltamivir were studied on patients infected with influenza virus.
These patients were randomly separated into two groups one group administered
with a placebo (a pseudo-drug with no effect) while the other group was administered
oseltamivir. Symptoms were measured and recorded in Table 3.4.

Study Groups
Symptoms
Placebo (n=129) Oseltamivir, 75mg (n=124)
Cough
Duration, hr 55 (36-73) 31 (24-42)
Severity, arbitrary units 110 67
Nasal Obstruction
Duration, hr 43 (31-64) 33 (25-43)
Severity, arbitrary units 80 60
Sore Throat
Duration, hr 21 (8-29) 10 (4-19)
Severity, arbitrary units 29 20
Fatigue
Duration, hr 41 (26-50) 24 (19-34)
Severity, arbitrary units 89 54
Feverishness
Duration, hr 23 (16-29) 10 (8-14)
Severity, arbitrary units 49 21
Table 3.4

Table abstracted from: http://jama.jamanetwork.com/article.aspx?articleid=192425,


Efficacy and Safety of the Oral Neuraminidase Inhibitor Oseltamivir in Treating
Acute Influenza - A Randomized Controlled Trial

(d) With reference Table 3.4, determine the effectiveness of oseltamivir. [2]

a. Oseltamivir is highly effective in treating influenza.


b. Duration of symptoms was reduced in the group administered with
oseltamivir. Quote an example from the table. (For e.g. duration of cough
was reduced from an average of 52 to 31 hours)
c. Severity was significantly reduced in the group administered with
oseltamivir. Quote an example from the table. (For e.g. severity of cough
was reduced from 110 to 67 arbitrary units.)
d. The reduction in symptoms indicates that the spread of influenza virus is
controlled with the drug, allowing the immune system of the body to target
infected cells before the virus can infect even more cells.

(e) In a separate case, a patient was treated for both H1N1 and H3N3. During this
treatment, he was infected with H3N3. Upon testing, it was found that his blood
now contains H3N3 and a new strain of virus, H1N3. Explain how this new strain
could have arisen. [3]

a. The new strain of virus could have arisen due to antigenic shift.
b. As H1N1 and H3N3 infect the same cell simultaneously, their protein
capsids and lipid envelopes are removed, exposing their RNA.
c. Influenza virus can undergo antigenic shift because it contains a
segmented genome composed of 8 different segments.
d. The new strain, H1N3, could have arisen via genetic recombination of the
genome segments.
1217

e. The random assembly of RNA segments from two different viruses


f. resulting in a mixture of surface antigens of both H1N1 and H3N3 strains.

Question 4 [10 marks]

(a) In maize plants, a recessive mutant allele leads to green leaves as opposed to
bronze leaves. Another recessive mutant allele at a separate locus leads to
yellow cobs as opposed to brown cobs. F1 maize plants from a standard dihybrid
cross was test crossed with a plant with green leaves and yellow cobs. The table
shows the phenotypes of 200 of the offspring.

Bronze leaves and yellow cobs 88

Bronze leaves and brown cobs 12

Green leaves and yellow cobs 14

Green leaves and brown cobs 86

(i) In the space below, draw a genetic diagram to explain the observed results of
the cross. [4]

Let B be the dominant allele coding for bronze leaves and let b be the
recessive allele for green leaves.

Let N be the dominant allele for brown cobs and let n be the recessive allele for
yellow cobs. a. legend [1/2]

Parental phenotype Bronze leaves, brown cobs x green leaves, yellow


cobs b. correct phenotypes [1/2]

Genotype Bn x bn
bN bn c. correct genotype [1/2]

Types of Gametes Bn bN bn
BN bn

d. gametes are circled with arrows [1/2]


e. correct gametic genotype [1/2]

Offspring genotypes & phenotypes d. [1 mark]


Bn bN BN bn

bn Bn bN BN bn
bn bn bn bn
(a) Bronze Green leaves, Bronze Green leaves,
leaves, yellow brown cobs leaves, brown yellow cobs
cobs cobs

Expected ratio: 1: 1: 1: 1
Experimental ratio 44% 43% 6% 7%
1218

(parental) (parental) (recombinant) (recombinant)


f. experimental ratio [1/2]

(ii) Explain the results of the test cross. [3]

a. Bulk of experimental plants has parental phenotypes of bronze leaves +


yellow cobs and green leaves and brown cobs for test cross.
b. This means that the two genes (for leaf colour and cob colour) are linked
and found on the same chromosomes.
c. Higher probability that parental phenotypes will predominate. This is
because linked genes do not assort independently.
d. The bronze leaves and brown cobs and green leaves and purple cobs
phenotypes are recombinant phenotypes as a result of crossing over.
e. Since crossing over is a chance event,
f. the recombination phenotype (due to crossing over) to be lower in
frequency than parental phenotype.

(b) A chi-squared test was also carried out for the above cross and the chi-square
value was 109.6. Using the probability table given, does this value for the chi-
squared test support the expected phenotypic ratio? Explain. [3]

a. No;
b. Calculated 2 value of 109.6 is greater than 7.82
c. At 3 degrees of freedom
d. The probability of getting chi-square value is between p<0.001
e. P<0.5 (or less than 5% probability) which is the cut off probability
f. Hence difference between observed and expected results is significant
g. and not due to chance
1219

Que
estion 5 [12
2 marks]

(a) Fig 5.1 belo


ow shows the blood gl ucose conc
centration measured.
m

Fig 5.1

(i) Identify wh
hich individu
ual has a no
ormal home
eostatic systtem in contrrolling blood
d
glucose levvel. Explain
n your answ
wer. [2]

a. Individual D;
b. Increase in
n blood gluucose levell from 90 mg
m to 150 mgm 100 cm-3 -3
of blood
from 0 1..0 hour as glucose is absorbed into the bloodstream m;
c. Beta cells of the pancreas deteects prod duces/trigg
gers insulin
n productio on;
d. increases glucose pe ermeability
y in liver an
nd muscle cells;
e. return gluc
cose level back to sett point 1000 mg per 1000 ml bloodd after 2.5
hours

(ii) Describe and


a explain the conditio
on of person C in the period
p betweeen 0 1.0
0
hours. [2]

a. Hypoglyca aemia;
b. blood gluccose level drop
d from 100 to 35mmg 100 cm-33 of blood; (below sett
point of 90
0 mg / 100 ml
m of bloodd)
c. Person C is
i unable to o absorb th
he glucosee from the intestine
i in
nto his bloo
od
stream/due e to intensive exercis
se/fasting; (AVP)
d. Glucagon not secrete ed/synthessised

(iii) State th
he type of re eceptor thatt glucagon binds to and
d explain hoow this
recepto y activated. [2]
or is the fully

a. G-protein linked rece


eptor;
b. Binding off glucagon to comple or;
ementary binding site on recepto
c. cause chaange in co onformatio
on of the receptor which
w can now bind
d G-
protein

(b) Ras is a GTTP-binding protein thatt helps to broadcast signals from the cell surrface
to other pa oliferation. If Ras functtion is inhib
arts of the cell to stimu late cell pro bited,
1220

the cell pro


oliferation responses
r nnormally induced by the activateed receptors s do
not occur. Conversely y, if a hypeeractive mu utant, Ras protein, is introduced into
some cell lines, the efffect on celll proliferatio
on is the same as that induced byy the
binding of ligands to cell surface e receptors. This hype eractive prooduct of mu
utant
ras gene prromotes the e developm ent of canc cer.

Figure 5.2 below sho


ows the seq
quence of events
e in a Ras cell cyycle-stimula
ating
pathway.
 

Figure 5.2

(i) Briefly desscribe the sttages A and


d B shown in Figure 5.2 2. [4]
A
a. binding off signal mo olecule to tthe 2 recep
ptors cause es 2 recepttor subunitts /
polypeptid de chains / monomerrs / molecu ules to form
m a dimer / dimerise;
b. activationn of the receptor tyros sine kinase
e region lea ads to
c. cross pho osphorylatiion of tyros sine aminoo acids on each
e otherrs tyrosine
e
amino aciid tails
d. and subse equent pho osphorylat ion residue es on theirr respectivee tail of
tyrosine amino
a acid subunits activates s tyrosine amino acid ds makin ng
them reco ognizable asa docking sites for re elay proteins

B
a. relay protein(s), Grb b-2 adaptorr protein reecognizes and
a binds tto specific
phosphorrylated tyro osine(s) whhich acts as s docking protein
b. for Ras GEF which becomes
b a
activated
c. which acttivates G protein,
p Ras s protein by
b removing GDP and d
d. replacing it with GTP P
e. in order to
o activate Ras
R protein n
f. to relay siignal by acctivating otther proteinns or enzym
mes to brin
ng about
various ce ellular resp
ponses (= c cell proliferration).

(ii) Ras protein is a plasm


ma membra ane bound G-protein.
G Suggest
S how w the activa
ated
G-protein can
c relay th he signal fro
om the ligan
nd to activatte cellular reesponses. [2]
[
1221

a. By activating enzymes such as adenylyl cyclase which will catalyse the


production of second messengers, such as cAMP
b. cAMP will activate protein kinase A resulting in a phosphorylation cascade
activate proteins necessary for cellular responses

(c) Suggest one function of signal transduction. [1]

a. may transform the signal into a different form, which is suitable for passing
or stimulating a cell response
OR
b. It may amplify the signal it receives either by producing large amounts of a
small signal intracellular mediators or by activating many copies of a
downstream signaling protein.
1222

Que
estion 6 [11
1 marks]

Figu
ure 6.1 sho
ows an electtromicrogra
aph of a chlo
oroplast


B

     Figure 6.1

(a) Identify the


e features: [2]
[

A grana/thylak
koid memb
branes B stroma

C chloroplast envelope D starch


s grain
n

(b) The blue dyye DCPIP can


c be convverted to colourless red duced DCP IP by gainin ng
electrons.
A suspension of chlorooplasts wass made by grinding
g fres
sh leaves inn buffer solu
ution
and centrifu
uging the mixture.
m Tub es were the
en preparedd and treateed in the
following way:
w

ubes
Tu Contentt Treatment Colo
our
Start
S After 20 min
m
A 1 cm3 chloroplast Illuminated d Blue
e-green Green
suspensio
on strongly
5cm3 DCP PIP

B c 3 buffer so
1 cm olution Illuminated d Blue
B blue
5cm3 DCP PIP strongly

C 1 cm3 chloroplast L
Left in the da
ark Blue
e-green Blue-green
suspensio
on
5cm3 DCP PIP
1223

(i) Explain this observation using your knowledge on light-dependent reactions of


photosynthesis. [4]

a. DCPIP acts like final hydrogen ion and electron acceptor, NADP+ in non-
cyclic photophosphorylation of the light-dependent reaction;

b. Tube A: When light and chloroplast suspension are present, photoexcited


electrons from water/PSII/PSI are passed through the ETC to DCPIP to
reduce it, changing it from blue to colourless.

c. However, when tube C is placed in the dark, DCPIP remained blue,


indicating that it is not being reduced no photoactivation in the dark

d. DCPIP is thus an indicator of whether photoactivation and thus electron


flow through the ETC during light-dependent reaction has taken place.

e. Tube B shows that DCPIP is not reduced and remains blue;

f. Proving that chloroplast suspension is required for reduction of DCPIP;

(iii) Explain, using tubes A and C, the effects of illumination on Calvin cycle (dark
reaction). [2]

a. Light dependent reaction taking place in tube A but not in C means that
ATP & NADPH can be produced in A
b. With ATP and NADPH, Calvin cycle can take place giving rise to sugars
c. No Calvin cycle in tube C
d. ATP and NADPH are required for reduction and regeneration phase in
Calvin cycle.

(iv) The chloroplast suspension may be contaminated with mitochondria. Explain the
evidence from the investigation that presence of mitochondria was not
responsible for the reduction of DCPIP. [2]

a. Respiration in the mitochondria occurs under both dark and light


conditions, producing hydrogen ions and electrons for reduction of NAD+;

b. When left in the dark/ absence of light/ tube C, DCPIP remained blue-green
like at the start of the experiment, indicating that there is no reduction of
DCPIP
1224

Question 7 [11 marks]

The Galpagos tortoise or Galpagos giant tortoise (Chelonoidis nigra) is the largest
living species of tortoise and 13th-heaviest living reptile, reaching weights of over
400 kg and lengths of over 1.8 metres. With life spans in the wild of over 100 years, it
is one of the longest-lived vertebrates.

The tortoise is native to seven of the Galpagos Islands, a volcanic archipelago


about 1,000 km west of the Ecuadorian mainland.

Shell size and shape vary between populations. On islands with humid highlands, the
tortoises are larger, with domed shells and short necks. Due to the climate, there is
abundant vegetation and vegetation is closer to the ground. On islands with dry
lowlands, the tortoises are smaller, with "saddleback" shells, which are flatter than
domed shells, and long necks. There is less vegetation. Charles Darwin's
observations of these differences on the second voyage of the Beagle in 1835,
contributed to the development of his theory of evolution.

Tortoise numbers declined from over 250,000 in the 16th century to a low of around
3,000 in the 1970s. This decline was caused by exploitation of the species for meat
and oil, habitat clearance for agriculture, and introduction of non-native animals to
the islands, such as rats, goats, and pigs.

(a) (i) Explain how the distinct phenotypic differences between the populations may
have arisen. [4]

a. Within the population, there is inherited variation/variations in alleles.


b. Selection pressure is the availability of food source on the different islands.
c. Results in differential survival and reproductive rates in different varieties.
d. Islands that have humid highlands have larger tortoises with short necks as
vegetation is abundant and closer to the ground hence the tortoises with
short necks are more highly adapted to the environment and those with
longer necks do not survive as well.
e. Islands that have dry lowlands have smaller tortoises as the food is less
abundant and unable to grow to larger sizes.
f. Those with longer necks are better adapted to the environment as they are
able to stretch further to reach food that is higher off the ground.
g. Individuals with the advantageous traits will survive better to reproduce
and pass on alleles for these traits to the offspring.
h. Islands - Geographical isolation therefore the tortoises are not able to
interbreed no gene flow between populations
i. Genetic drift might have also occurred through founder effect leading to
different allele frequencies in the populations on different islands.

(ii) State the phenomenon that describes the decline in tortoise numbers from the
16th century to the 1970s. [1]

Bottleneck effect

(iii) Suggest a disadvantage to a reduction in population size. [1]

a. Decreased genetic variation within the population;


b. Leading to the population being susceptible to new diseases or sudden
environmental changes OR
1225

Increase in
n the occurrrence of re
ecessive diseases
d as
s each pareent will most
probably have
h a copyy of the rec
cessive allele.

Figu
ure 7.1 beelow shows the rela ationship beetween number of ssubstitutions
s in
cyto
ochrome c between
b related pairs of species and how lo
ong ago thee two compa
ared
species had divverged from
m a common n ancestor.

Figure 7.1

(b) Describe and explain the


t conclussions that ca n from Figurre 7.1. [3]
an be drawn

a. The graphh suggests a linear re elationshipp between thet numbeer of seque ence
changes and
a the timee of diverg
gence.
b. Such a relationship indicates that the observed
o ra
ate of neu tral mutatiions
remains co onstant over millions s of years.
c. The number of nucle eotide diffe
erences is higher when two speecies share ed a
common ancestor
a in
n the very ddistant pas s in anotheer two species
st than it is
that sharedd a more re ecent comm mon ances stor.
d. Human and cow have a differen nce of 90 nucleotides
n s and they diverged from
f
the commo on ancesto or about 1220 millions of years ag go.
e. Compared d to horse and donke ey which have
h a diffe
erence of 110 nucleotides
and they diverged fromf the c
common an ncestor 8 million yeears ago. (Any
(
other valid
d comparison)
f. The gene sequences s of speciees which had
h diverge ed from eaach other for
f a
longer pe eriod of time wou uld have accumulated more independ dent
mutations,, which ma ake their se
equences more
m different.

(c) Suggest the


e difficulties
s that arise when using
g the molecular clock. [[2]

a. No gene marks
m time with compplete precis
sion. In factt, some poortions of th
he
genome ap ppear to ha
ave evolved
d in irregular fits and
d starts that
at are not att all
clocklike.
1226

b. Even genes that seem to have reliable molecular clocks are accurate only
in the statistical sense of showing a fairly smooth average rate of change.
Over time, there may still be chance deviations above and below that
average rate.

c. The same gene may also evolve at different rates in different groups of
organisms, making it necessary to calibrate and use molecular clock with
care.

d. Even among genes that are clocklike, the rate of the clock may vary greatly
from one gene to another; some genes evolve a million times faster than
others.

Section B: Essay

Question 8

(a) Explain how the structure of triglyceride is related to its function. [7]

Chemical nature Function


Proportionately more H atoms Release more hydrogen ions when
compared to oxygen oxidized during respiration
Allows production of twice the
amount of energy as compared to
same unit mass of carbohydrates
Yields twice the amount of metabolic
water when oxidized during
respiration

More C atoms and H atoms than Lower density/molecular mass


O atoms lower molecular more weight efficient compared to
weight than eg. carbohydrates carbohydrates
Favours locomotion of animals
Less dense Provides buoyancy
especially for aquatic animals
Favours seed dispersal since
lightweight as it can be stored in large
amounts in the seed endosperm.

Hydrophobic / non-polar; thus Long term storage of energy


chemically inert in cells good for hibernating animals
Can surround and protect vital organs
on a long term basis to absorb
mechanical shock in case of
accidents or falls
Good heat insulators
Maintenance of body temperature.

Non-polar so will repel ions or High electrical resistance good


charges electrical insulators.
Ensures rapid transmission of nerve
1227

impulses in myelinated nerve cells

(b) Compare and contrast between collagen and cellulose. [7]

Collagen Cellulose
Monomers Amino acids Only B-glucose molecules
Bonds between Peptide bonds formed B 1,4, glycosidic bonds
monomers between amino acid formed between B-glucose
monomers monomers
Structure Loose helical structure of Linear chain due to rotation
adopted by polypeptide chain due to the of alternate glucose residue,
each chain lack of intra-chain H bonds resulting in bulky side
(because of lack of N-H chains on each side of the
group in proline) chain
Subunit making Tropocollagen 3 Linear unbranched chain
up polypeptide chains wound
macromolecule tightly together
Arrangement of Tropocollagen arranged in Long chains run parallel to
subunit in staggered out of step each other, but not in a
macromolecule manner to increase tensile staggered manner
and mechanical strength
Bonds involved Covalent cross-linkages Cumulative strength of H-
in stabilizing formed between the C- bonds formed between the
macromolecule terminus of one hydroxyl groups projected
tropocollagen and the N- outwards in all directions of
terminus of another each chain
tropocollagen
Organism Produced in animals Produced in plants
Similarities
Both serve as structural support with high tensile and mechanical strength
Both are insoluble in water due to the lack of hydroxyl groups to form H-bonds
with water.

(c) Some enzymes function optimally in acidic conditions, while other enzymes
function optimally in alkaline conditions. Explain why different enzymes have
different optimal pH. [6]

a. Each enzyme is a globular protein


b. with a structurally and chemically specific active site.
c. This is due to the specific sequence of amino acids in the primary structure
d. which determines the types of bonds and location of R groups present on
the amino acids in the protein
e. The conformation of the active site of each enzyme will change when pH
changes
f. due to changes to the catalytic, contact and structural residues.
g. When the pH increases beyond the optimal pH, positively charged R groups
at active sites of enzyme X are neutralised by the abundant OH- in the
solution
h. As pH decreases to low pH for enzyme Y, the negatively charged R groups
at the active sites of enzyme Y are neutralised by the H+ in the solution
i. Both lead to disruption of ionic bonds and hydrogen bonds
j. which in turn disrupt the secondary and 3D conformation (tertiary) &
quaternary structures of the enzyme
1228

k. and thus the conformation of the active site can no longer be maintained.
l. Substrate no longer fits into active site as enzyme is denatured.

Question 9

(a) Explain the role of the F plasmids in bacterial conjugation and suggest the
advantages of conjugation over transduction. [8]

Explain the role of the F plasmids in bacterial conjugation: (max 4 marks)


1 Conjugation - transfer of genetic material (in one direction) through (direct)
cell to cell contact between two bacterial cells;
2 Ref to one with the F plasmid (donor) and the other without (recipient);
3 F plasmid contains genes / codes for synthesis of sex pilus;
4 Used for attachment to recipient cell
5 And for formation of mating bridge/conjugation tube;
6 Ref. to F factor replicates by rolling-circle mechanism (idea of replication is
impt);
7 (single-strand of) F plasmid is transmitted through mating bridge to
recipient;
8 transferred ssDNA is replicated in the recipient cell, producing a ds F
plasmid;
9 Conjugation tube breaks;
10 (if entire F plasmid was transferred) Recipient cell now has F plasmid/ is F+;

Suggest the advantages of conjugation as compared to transduction [max 4]

1 No need for 3rd party / virus / vice versa;


2 Host cell remains alive;
3 Does not reply on chance that wrong DNA fragments are packaged (in
generalised transduction);
4 Does not reply on chance that viral DNA excises imprecisely (in specialised
transduction);
5 Homologous recombination not needed unlike (general) transduction
(though specialised transduction does not require it);
6 Higher chance to transfer genetic material (if cells contact each other) (as
transduction relies heavily on chance);
7 In transduction, genetic material may or may not be degraded / vice versa;
8 Larger portion of DNA/genetic material can be transferred;
9 Ref. defined genes (of F plasmid) transmitted during conjugation vs
random DNA fragments (via general transduction) OR only genes/DNA
fragments beside prophage (via specialised transduction)

(b) Explain how the lac operon is regulated by both negative and positive control. [8]

a. Only when lactose is in high supply AND glucose is in low supply does E
coli use lactose as an energy source.
b. When lactose is present, some of the lactose gets transported into the cells
c. Lactose is converted into the inducer, allolactose.
d. Allolactose is the isomer form of lactose.
e. The inducer binds to the repressor protein and alters its conformation
f. so it cannot bind to the operator site (repressor becomes inactive)
g. In the absence of an operator-bound repressor protein, RNA polymerase
can now transcribe the structural genes into mRNA,
h. which are then translated into enzymes, permease, transacetylase and -
galactosidase, which transport and hydrolyse the disaccharide, lactose.
i. The lac operon has a very inefficient promoter element that is, it has a low
1229

affinity for RNA polymerase, even when the repressor protein is inactivated.
j. cyclic AMP (cAMP), accumulates when glucose is in low concentration in
the cell.
k. Another regulatory protein, catabolite activator protein (CAP) (which is
produced in inactive form) can bind to CAP-binding site.
l. To activate CAP, cAMP must bind to CAP;
m. The resulting cAMP-CAP complex then binds to the CAP-binding site.
n. This binding of active CAP bends DNAs double helix strengthens the
affinity of the promoter region for RNA polymerase
o. the rate of transcriptional initiation increases in the presence of lactose.
p. Lac operon is thus regulated by both negative (due to repressor) and
positive (due to activator) control.

(c) Suggest why operons are necessary in bacteria. [4]

a. bacteria genome is short;


b. having operons saves space;
c. as a few genes share one promoter;
d. producing polycistronic mRNA;
e. allows transcription of a few related genes at the same time (better
regulation of gene expression);
f. being single cellular also means that the bacteria is easily affected by the
fluctuating environment;
g. allows bacteria to adapt to environment quickly as the related genes are
transcribed simultaneously;
h. better use of resources, thus leads to less wastage of energy and
resources
1230

PIONEER JUNIOR COLLEGE


JC2 PRELIMINARY EXAMINATION 2013

Candidate
Name

Civics INDEX
Group NUMBER

H2 BIOLOGY 9648 / 03

20 Sept 2013 2 hours

READ THESE INSTRUCTIONS FIRST

Write your name, class and index number on all the work you hand in.
Write in dark blue or black pen.
You may use a soft pencil for any diagrams, graphs or rough working.
Do not use staples, paper clips, highlighters, glue or correction fluid.

Read the instructions on the answer sheet For Examiners Use


very carefully.
This document consists of 2 sections: Section A

Section A 1 / 12
Answer ALL questions.
2 / 14
Answers are to be written in the spaces provided.
3 / 14
Section B
Answer Question 5. Section B
Write your answer on the lines provided.
5 / 20
Do not open this booklet until you are told to do so.
TOTAL / 60

4 (SPA) /12

This document consists of 22 printed pages (inclusive of this page)

[Turn Over

1231

SectionA:Answerallquestions[40marks]

1. The ability to identify salmon species following food processing such as canning and smoking
is problematic but of great commercial importance. The most commonly used methods of
identifying raw fish are not applicable to thermally treated products. Polymerase chain
reaction-restriction fragment length polymorphism (PCR-RFLP) is thus an alternative method
to authenticate canned or smoked fish products.

(a) Suggest why authentication of processed fish product is important. [1]

..

..

Following extraction of DNA from the processed fish product, PCR was carried out to amplify a
500bp segment from the DNA sample. Figure 1.1 illustrates the thermal cycling characteristic
of PCR.

Figure 1.1

(b) Explain the importance of the temperatures in phases A, B and C.

(i) Phase A [2]

....

 2
1232

(ii) Phase B [1]

(iii) Phase C [1]

Figure 1.2 shows the number of molecules of DNA produced over a number of PCR cycles.

Figure 1.2

(c) Suggest a reason why the graph plateaus. [1]

 3
1233

DNA
D sample
es were takken from 10 0 different ssmoked salmon produc cts to autheenticate the
e salmon
used.
u The a
amplified sa
amples were then subjjected to re estriction digests usingg various reestriction
enzymes.
e Fo
or each reaaction, the DNA
D restricttion fragme
ents were suubjected to gel electrophoresis
and
a visualize ed using a staining kit.

Figure
F 1.3 sshows the RFLP
R profile
es of salmon
n products 1 to 10 follo
owing digesstion with re
estriction
enzymes
e Bssp 1286I, Ecco RII and Sau
S 3AI. 10 00bp DNA laadder lanes s were incluuded as refeerence.

F
Figure 1.3

With
W referen nce to Figurre 1.3,
(d) expla
ain what is meant
m by RF
FLP. [2]

..



..

(e) identify and explain which 2 smoked sa


almon produ
ucts used th
he same sppecies of sa
almon.
[4]

..



..

[Total: 12
2 marks]

 4
1234

2.
2 The first g
gene therap
py trial for an
a inherited
d disorder was
w initiated d on 14 Se ptember 19 990. The
patient, A
Ashanthi De
eSilva, suffeered from a very rare recessively
y inherited ddisorder, ad
denosine
deaminasse (ADA) deeficiency.

An inherited deficien
ncy of ADA has, howevver, particularly severe
e consequeences in the
e case of
T lymph hocytes. As A a resu ult, ADA-deeficient paatients sufffer from severe co ombined
immunoddeficiency (SSCID). 

(a) Explaain the impo


ortance of ADA
A with re
espect the im
mmune sys
stem. [2]

.







..

..



.

(b) Sugggest why a recessivelyy inherited ddisorder succh as ADA--SCID can be treated by gene
thera
apy. [2]

.







..

..



.

(c) The novel ADA gene therapy approacch involved essentially the followinng steps:

Fi gure 2.1

(i) Suggest wh
hy the ex vivo method was used in
n this case. [1]

 5
1235

(ii) Suggest an improvement to the procedure in Figure 2.1 so as to increase the success
rate of the treatment. [3]

...

...

...

...

(d) Name another method, other than gene therapy, which can be used for the treatment of
ADA-deficiency. [1]

...

(e) Gene therapy using the same form of treatment as the above case study has succeeded
for 15 French baby boys suffering from X-linked SCID. However 3 of the little boys later
developed cancer and died. Suggest how this is possible. [3]

....

....

....

....

....

....

 6
1236

(f) State and explain 2 other factors that hinder the effectiveness of current gene therapy
approaches. [2]

....

....

....

....

[Total: 14 marks]

 7
1237

3. AquAdvantage salmon is a transgenic salmon where a promoter for the anti-freeze gene of an
ocean pout and a gene for the Chinook growth hormone was inserted into a plasmid pUC18
(Figure 3.1). The recombinant DNA (Figure 3.2) is then introduced into fertilized egg cells of
the Atlantic salmon by microinjection and stimulated to become triploid.

Figure 3.1

Figure 3.2

(a) With reference to the Figure 3.1 and 3.2,

(i) describe the role of the ampicillin resistance gene. [1]

...

...

 8
1238

(ii) describe how the recombinant DNA is formed. [4]

...

...

......

...

(b) The egg cells were induced to be triploid. Explain the significance of this. [2]

...

......

...

(c) Figure 3.3 below shows the differences between weight of AquAdvantage salmon and
standard salmon.

Figure 3.3

 9
1239

(i) With reference to Figure 3.3, compare the growth curves of standard salmon and
AquAdvantage salmon. [3]

.....

...

.....

...

(ii) Explain the difference in the weight increase of AquAdvantage salmon and the
standard salmon. [2]

........

....

.....

(d) Describe one ethical and one social concern that might arise from the production of
AquAdvantage salmon. [2]

........

....

........

[Total: 14 marks]

 10
1240

4. Planning Question [12 marks]

The Tasmanian Tiger or Thylacine (Thylacinus cynocephalus), the world's largest


carnivorous marsupial, was once common throughout Australia and Papua New Guinea.
The Thylacine resembled a large, short-haired dog with a stiff tail which smoothly
extended from the body in a way similar to that of a kangaroo. e female Thylacine had
a pouch with four teats, but unlike many other marsupials, the pouch opened to the rear
of its body.

An example of convergent evolution, the Thylacine showed many similarities to the
members of the Canidae (dog) family of the Northern Hemisphere: sharp teeth, powerful
jaws, raised heels and the same general body form.
Due to human activities, the Thylacine was hunted to extinction by early 1930. A
Thylacine specimen with soft tissue remaining is found in the Australian Museum in
Sydney.
Imagine that you are a researcher in the Australian Rare Fauna Research Association.
Recently it was reported that locals near Mount Carstensz in Western New Guinea had
sighted creatures that resemble Thylacines. Some members of the Association believe
that the creatures sighted may be descendents of the Thylacine, while other members
believe that the creatures may be a new species. If the former were true, then the
Thylacine is not extinct and conservation effects may revive the species.

Plan an investigation to investigate if these creatures found in Western New Guinea are
descendents of the Thylacine that was thought to be extinct.

Your planning must be based on the assumption that you have been provided with the
following equipment and materials.

Tissue sample from the museum specimen and from the Western New Guinea
creatures under investigation
Pestle and mortar
DNA extraction buffer solution
Microcentrifuge tubes
Centrifuge
Restriction enzymes
Agarose or polyacrylamide gel plate
Suitable source of electric current
Radioactive probe
Nitrocellullose membrane
Autoradiography equipment

Your plan should have a clear and helpful structure to include:

An explanation of the theory to support your practical procedure


A description of the method used including scientific reasoning behind the method
The type of data generated by the experiment
How the results will be analysed including how the origin of the organism can be
determined

 11
1241

 12
1242

 13
1243

 14
1244

 15
1245

 16
1246

Section B: Essay Question [20 marks]

Write your answers to this question on the lines provided.

Your answer:

Should be illustrated by large, clearly labelled diagrams, where appropriate.


Must be in continuous prose, where appropriate.
Must be set out in section (a), (b), etc., as indicated in the question.

(a) Describe how the micropropagation technique is used in production and cloning of
herbicide resistant plants. [9]
(b) Discuss the social and ethical implications of GMO plants using 3 named examples. [6]
(c) Discuss the limitations for the use of gene therapy to treat diseases in humans. [5]


 17
1247

 18
1248

 19
1249

 20
1250

 21
1251

 22
1252

PIONEER JUNIOR COLLEGE


JC2 PRELIMINARY EXAMINATION 2013

Candidate
Name

Civics INDEX
Group NUMBER

H2 BIOLOGY 9648 / 03

20 Sept 2013 2 hours

READ THESE INSTRUCTIONS FIRST

Write your name, class and index number on all the work you hand in.
Write in dark blue or black pen.
You may use a soft pencil for any diagrams, graphs or rough working.
Do not use staples, paper clips, highlighters, glue or correction fluid.

Read the instructions on the answer sheet For Examiners Use


very carefully.
This document consists of 2 sections: Section A

Section A 1 / 12
Answer ALL questions.
2 / 15
Answers are to be written in the spaces provided.
3 / 14
Section B
Answer Question 5. Section B
Write your answer on the lines provided.
5 / 20
Do not open this booklet until you are told to do so.
TOTAL / 60

4 (SPA) /12

This document consists of XX printed pages (inclusive of this page)

[Turn Over
1253

Section A: Answer all questions [40 marks]

1. The ability to identify salmon species following food processing such as canning and
smoking is problematic but of great commercial importance. The most commonly used
methods of identifying raw fish are not applicable to thermally treated products. Polymerase
chain reaction-restriction fragment length polymorphism (PCR-RFLP) is thus an alternative
method to authenticate canned or smoked fish products.

a) Suggest why authentication of processed fish product is important. [1]


to deter or identify mis-labelling so that consumers can make informed choices
about their food.
any other valid reasons

Following extraction of DNA from the processed fish product, PCR was carried out to amplify
a 500bp segment from the DNA sample. Figure 1.1 illustrates the thermal cycling
characteristic of PCR.

Figure 1.1

b) Explain the importance of the temperatures in phases A, B and C.

(i) Phase A [2]


At 95C, hydrogen bonds between DNA strands are broken,
DNA is denatured
DNA double helix unwind and separated into two separate single strands
Each used as a template for replication by DNA polymerase.

(ii) Phase B [1]:

As specific DNA primers are used to anneal to 3 end of target sequence via
complementary base-pairing,
temperature is specific at 55C;
1254

(iii) Phase C [1]

Optimal temperature for heat stable Taq polymerase is 72C


Taq polymerase will extend the primers by adding nucleotides to the 3 OH ends of
the primers

Figure 1.2 shows the number of molecules of DNA produced over a number of PCR cycles.

Figure 1.2

c) Suggest a reason why the graph plateaus. [1]

Taq polymerase gradually becomes denatured; OR


No more primers or deoxyribonucleotides available;

DNA samples were taken from 10 different smoked salmon products to authenticate the
salmon used. The amplified samples were then subjected to restriction digests using various
restriction enzymes. For each reaction, the DNA restriction fragments were subjected to gel
electrophoresis and visualized using a staining kit.

Figure 1.3 shows the RFLP profiles of salmon products 1 to 10 following digestion with
restriction enzymes Bsp 1286I, Eco RII and Sau 3AI. 100bp DNA ladder lanes were included
as reference.
1255

Fig
gure 1.3

With refference to Figure


F 1.3,

d) expla
ain what is meant
m by RFLP. [2]

Diffe
ferent lengtth of fragmments
wheen genome e of 2 indiviiduals dige
ested by the same RE
E
Duee to genetic c mutation
resuulting in loss or gain of restrictiion sites

ducts use the same speecies of salm


almon prod
e) identtify and explain which 2 smoked sa mon. [4]

Salmmon produ ucts 3 and 6;;


6 [1 mark k] or Salmoon products s 8 and 9
For all three REs
R used, the RFLP p profiles for both produucts are thee same.
Bspp 1286I -> both
b produc cts show tw wo bands of 200bp and 300bp. (500bp forr 8 and
9)
EcooRII -> one band of 50 00bp (500b p for 8 and
d 9)
Sauu3AI -> two bands of 3003 and 50 00bp. (300bbp and 500b bo for 8 an
nd 9)
Gennomes from m both prod ducts are tthe same
as tthe restriction sites fo
or each resspective REE are at the
e same locii.

Questio
on 2

(a) Exxplain the im


mportance of
o ADA with respect to the
t immune
e system. [22]

ADA A is importtant for bre


eakdown off deoxyade enosine/ pu
urine metabbolism
Prevvent accummulation off the subst rate deoxy
yadenosine e in cells
T ly
ymphocytes s will rema nal
ain function
Aidss B lympho ocytes in reecognizingg and producing antib
bodies

(b) Suuggest why a recessivvely inheriteed disorderr such as ADA-SCID


A eated by
can be tre
gene therapy. [2]

Ref to single gene


g disord
der;
Losss-of-functiion mutatio
on
Intro
oduction of
o one norm mal / domin
nant allele to
t produce
e sufficientt amounts of
o gene
prodduct / ADA
A
1256

to mask effect of recessive mutation

(c) (i) Suggest why the ex vivo method was used in this case. [1]

Either one:

T-lymphocytes can be easily transfected


Larger amount of T-lymphocytes can be cultured in the laboratory
Allow for selection of successfully transformed cells
Prevent the retrovirus from targeting other rapidly proliferating cells

Reject: viral vector can regain ability to cause disease

(ii) Suggest an improvement to the procedure in Figure 2.1 so as to increase the success
rate of the treatment. [3]

culture blood stem cells / hematopoietic stem cells instead of T-lymphocytes


ref. to origin of stem cells bone marrow / umbilical cord
differentiate to form all types of white blood cells (including B- and T-lymphocytes)
Stem cells are able to self renew by mitotic cell division for a long period of time
thus may provide more long term / permanent cure

also accept:

select for / identify successfully transformed cells


after infection of cells with retrovirus
Increase transformed cells through cell culture before re-implant cells into patient
Ensure that re-introduced cells are all able to express normal ADA gene

(d) Name another method, other than gene therapy, which can be used for the treatment of
ADA-deficiency. [1]

bone marrow transplantation from a perfectly matched sibling donor OR


enzyme replacement therapy / weekly intramuscular injections of ADA

(e) Gene therapy using the same form of treatment as the above case study has
succeeded for 15 French baby boys suffering from X-linked SCID. However 3 of the little
boys later developed cancer and died. Suggest how this is possible. [3]

inability to control the specific site of insertion / random integration of transgene ->
insertional mutagenesis
disrupt gene control regions -> activation of oncogenes
Inactivation of 2 copies of the tumor suppressor gene
Ref function of the TSG
Inactivation of a gene involved in apoptosis
-> uncontrolled cell division/proliferation

Reject: statements that imply insertional inactivation of proto-oncogenes contributes


to cancer.
1257

(f) State and explain 2 other factors that hinder the effectiveness of current gene therapy
approaches. [2]

1. Short-lived nature of gene therapy


a. Problems with integrating therapeutic DNA into the genome and the rapidly
dividing nature of many cells prevent gene therapy from achieving any long-term
benefits.
b. -> multiple rounds of gene therapy required
c. Difficulty in inserting new genes into billions of target cells.
2. Immune response to foreign object
a. Immune system's enhanced response to invaders it has seen before makes it
difficult for repeated gene therapy.
3. Problems with viral vectors
a. Viral vector may recover its ability to cause disease and trigger an immune
response.
4. Multigene disorders
a. Some commonly occurring disorders, such as heart disease, high blood pressure,
Alzheimer's disease, arthritis, and diabetes, are caused by the combined effects
of variations in many genes.
b. Multigene or multifactorial disorders such as these would be especially difficult to
treat effectively using gene therapy as multiple replacements of defective alleles
required.
5. Inability to control gene expression
a. Corrective genes may be over-expressed or a gene may shut off shortly after it
has been introduced.
6. Procedures for introducing DNA into cells typically are inefficient
a. Perhaps only 1 in 1,000 or 100,000 cells may receive gene of interest, thus a
large population of cells is needed to attempt gene therapy.
7. Fate of foreign DNA
a. In some cases the mutant gene is replaced by the normal gene, and in others the
normal gene integrates randomly into the genome elsewhere. -> may result in
insertional mutagenesis
b. Ref TSG and proto-oncogenes and how mutation in these genes may lead to
cancer development
8. Adverse effects not known if DNA is introduced into non-target cells and is expressed
in these cells.
9. Germline may be accidentally targeted resulting in the inheritance of any adverse
conditions.
1258

3. AquAdvantage salmon is a transgenic salmon where a promoter for the anti-freeze gene
of an ocean pout and a gene for the Chinook growth hormone was inserted into a plasmid
pUC18 (Figure 3.1). The recombinant DNA (Figure 3.2) is then introduced into fertilized
egg cells of the Atlantic salmon by microinjection and stimulated to become triploid.

Figure 3.1

Figure 3.2

(a) With reference to Figures 3.1 and 3.2,

(a) describe the role of the ampicillin resistance gene. [1]

Acts as a genetic marker/selection marker


to select for cells that have been transformed/taken up the pUC18 plasmid.

(b) describe how the recombinant DNA is formed. [4]

mRNA from the growth hormone is converted to a single stranded cDNA using
reverse transcriptase;
the RNA from the RNA-cDNA hybrid is hydrolysed by RNAseH;
DNA polymerase is used to form a complementary strand to form double stranded
cDNA;
Promoter and terminator region from ocean pout antifreeze gene is joined to the
growth hormone cDNA using DNA ligase;
Linker DNA is added to ends of the promoter and the terminator region;
1259

Linker DNA and pUC18 plasmid is cut using the same RE (state any RE shown in
the diagram) to produce complementary sticky ends;
Hydrogen bonds are formed/Annealing between the sticky ends;
DNA ligase is used to form phosphodiester bonds between the transgene/gene
construct to form a recombinant DNA.

(b) The egg cells were induced to be triploid. Explain the significance of this. [2]

AquAdvantage salmon are infertile as unable to produce haploid eggs via meiosis;
Any AquAdvantage salmon which might escape into the wild cannot mate with wild
salmon
hence preventing transfer of the transgene to wild species

(c) Figure 3.3 below shows the differences between weight of AquAdvantage salmon and
standard salmon.

Figure 3.3

(i) With reference to Figure 3.3, compare the growth curves of standard salmon and
AquAdvantage salmon. [3]

Both graphs show gradual increase and then exponential increase (necessary)
AquAdvantage salmon is always heavier than standard salmon from the first
feeding.
Standard salmon reached maximum weight of 4000g in 850 days while
AquAdvantage salmon reached more quickly 4000g in 620 days.
The weight gain for AquAdvantage salmon was gradual from 0 to 600g from 0 to
230 days while standard salmon weight increased gradually from 0 to 600g from 0
to 440 days.
AquAdvantage salmon showed an exponential increase in weight from 600g at 230
days to a maximum of 6000 g at 720 days. But standard salmon showed an
1260

exponential increase in weight from 600g at 440 days to a maximum of 4000g at


850 days.

or any other valid comparisons

(ii) Explain the difference in the weight increase of AquAdvantage salmon and the
standard salmon. [2]

AquAdvantage salmon contains the growth hormone gene of the larger Chinook
salmon -> produce the GH to grow larger;
Promoter of the ocean pout antifreeze gene -> produce the hormone all year round;
Enabling the AquAdvantage salmon to grow larger in a shorter period of time.
Total weight gain from AquAdvantage salmon was 6000g over 720 days and
standard salmon was 4000g in 850 days.

(c) Describe one ethical and one social concern that might arise from the production of
AquAdvantage salmon. [2]

Health concern (R: Environmental concern as the fish are triploid, cannot mate with
wild type)
The effects of extra salmon growth hormones and the safety of these salmon for
human consumption

Ethical concern
Increased production of growth hormone has harmful effects on the health of
salmon -> additional stress in salmon -> exploitation of animals for food

or any other valid ethical and social concerns

4. Planning Question [12 marks]

The Tasmanian Tiger or Thylacine (Thylacinus cynocephalus), the world's largest


carnivorous marsupial, was once common throughout Australia and Papua New
Guinea. The Thylacine resembled a large, short-haired dog with a stiff tail which
smoothly extended from the body in a way similar to that of a kangaroo. The female
Thylacine had a pouch with four teats, but unlike many other marsupials, the pouch
opened to the rear of its body.
An example of convergent evolution, the Thylacine showed many similarities to the
members of the Canidae (dog) family of the Northern Hemisphere: sharp teeth,
powerful jaws, raised heels and the same general body form.
Due to human activities, the Thylacine was hunted to extinction by early 1930. A
Thylacine specimen with soft tissue remaining is found in the Australian Museum in
Sydney.
Imagine that you are a researcher in the Australian Rare Fauna Research
Association. Recently it was reported that locals near Mount Carstensz in Western
New Guinea had sighted creatures that resemble Thylacines. Some members of the
Association believe that the creatures sighted may be descendents of the Thylacine,
while other members believe that the creatures may be a new species. If the former
were true, then the Thylacine is not extinct and conservation effects may revive the
species.
1261

Plan an investigation to investigate if these creatures found in Western New Guinea


are descendents of the Thylacine that was thought to be extinct.

Your planning must be based on the assumption that you have been provided with
the following equipment and materials.

Tissue sample from the museum specimen and from the Western New Guinea
creatures under investigation
Pestle and mortar
DNA extraction buffer solution
Microcentrifuge tubes
Centrifuge
Restriction enzymes
Agarose or polyacrylamide gel plate
Suitable source of electric current
Radioactive probe
Nitrocellullose membrane
Autoradiography equipment

Your plan should have a clear and helpful structure to include:

An explanation of the theory to support your practical procedure


A description of the method used including scientific reasoning behind the
method
The type of data generated by the experiment
How the results will be analysed including how the origin of the organism can
be determined

Suggested answer

Introduction: 4 marks
Use of molecular homology: the similarities in DNA nucleotides sequence of
organisms
Especially certain VNTR / DNA sequences that are conserved within a species but
different between species.
Extract the DNA from the specimen from the nucleus
Brief description of RFLP and reference to DNA fingerprinting
Use of a suitable RE to cut the genomic DNA recognise specific RE sites within the
DNA (to ensure that RE used does not cut up the target DNA sequence
Separate fragments using gel electrophoresis via molecular sizes
Perform southern blotting (brief outline of blotting process and denaturation of double
stranded DNA);;
Nucleic Acid hybridisation using a single stranded radioactive DNA probe (DNA if
VNTR is used) that is complementary to the target sequences
Description of autoradiography use of xray film
1262

Hypothesis:
If the RFLP banding patterns obtained are identical or similar, it suggests that the
creatures found in Western New Guinea are closely related to the Thylacine, thus this
suggests that they may share a recent common ancestor.

Procedures: (7 marks)
Proposed method of experiments to achieve results:

(DNA extraction digestion with RE gel electrophoresis Southern blotting)

(i) Method of DNA extraction from tissue samples including use of buffers;;
Mechanical breakage of tissues with pestle and mortar
Use of DNA extraction buffer solution
Centrifuge & DNA in the supernatant

(ii) Digestion with a suitable restriction enzyme for 1 hour at optimum temperature of 370C.
(Restriction enzyme must be able to cleave the DNA sequence at specific sites generating
different sized fragments between species.);;

2l DNA
0.5l - 1 l restriction enzyme

(iii) Preparation of samples for electrophoresis (3 marks)


Preparation of agarose gel dissolve 1g of agarose powder in 1L of
electrophoresis buffer solution (1% gel)
Add loading dye eg: bromophenol blue to DNA samples:
First lane 20 l DNA ladder
Lane 2-4 - Sample 1: 20 l Restriction digest of DNA from museum specimen
Lane 5-7 - Sample 2: 20 l Restriction digest of DNA from creatures under
investigation)
Lane 8- Prepare a control tube with all the reagents except DNA (Blank).

(iv) Load DNA samples in gel as shown: - represents 3 lanes of same sample
DNA Ladder
Sample 2
Sample 1
Control

Agarose gel

(v) Separation of fragments by gel. Run at 110V for 30min. Stop current when loading dye
reach 2/3 gel;

(vi) Transfer of DNA on to nitrocellulose membrane (blotting process);


Gel covered with nitrocellulose membrane, adsorbent papers added on top of
membrane, single stranded DNA drawn up gel by capillary action
1263

(vii) Hybridization with radioactive labelled DNA probe;


Alkali solution to denature DNA fragments into single strands
Radioactive probe binds by complementary base pairing to DNA sequence of
interest
Excess probes are washed off

(viii) Autoradiography method;


A photographic film or an X ray film is laid over the filter. Radioactivity in the bound
probe exposes the film.
DNA that has hybridised to the labeled probe shows up as bands on the
autoradiograph and can be visualized

(ix) Repeat the entire experiment twice using fresh samples;

Control;
Tube with all reagents but no DNA added. This negative control will show that only DNA
fragments with regions complementary to the probe will show up as a dark band after
autoradiography.

More tests can be done on other VNTR / gene sequences to obtain more evidence of
homology.

Sample 2
Control Sample 1

autoradiograph
1264

Safety precautions;
- be careful when working with radioactive DNA probes wear goggles, gloves and lab coat
to prevent touching bare skin

Section C: Essay question

Write your answers to this question on the lines provided.

Your answer:

Should be illustrated by large, clearly labelled diagrams, where appropriate.


Must be in continuous prose, where appropriate.
Must be set out in section (a), (b), etc., as indicated in the question.

(a) Describe, using a named example, how the micropropagation technique is used in [9]
production and cloning of herbicide resistant plants.

a) Obtain mRNA for glyphosate-resistant EPSPS gene and using reverse


transcription method, produce the double stranded cDNA (or any sound
method to isolate this gene)
- Add linker molecules to both ends of the gene of interest
b) Remove the oncogenes and opine producing genes from the Ti plasmids of
Agrobacterium.
c) cut the Ti plasmid using the same restriction enzymes as that for Bt gene,
forming complementary sticky ends
d) Mix the cut plasmid and gene of interest to allow complementary base-
pairing via hydrogen bonds to temporarily hold the fragments together
e) Add DNA ligase to seal the nicks in the sugar-phosphate backbone forms
phosphodiester linkages between fragments
f) Insert recombinant Ti-plasmids back into the Agrobacterium
g) In micropropagation, plant tissues or explants eg: meristematic cells are
removed from plants explants only;
h) Infect the explants with with Agrobacterium containing the recombinant Ti
plasmids. Select explants that were successfully transformed;
i) Surface of explants sterilised with dilute sodium hypochlorite (Clorox) to kill
bacterial and fungal pathogens or organisms;
j) And grown in a containing sterile media containing nutrients and plant
growth regulators needed for plant growth;
k) Containers holding the explants are then sealed and incubated for 1-9
weeks.
l) During this period, the cultured cells divide by mitosis
m) to form a mass of undifferentiated tissue called a callus.
n) As callus increases in size, pieces of callus is sliced off and grown on new
medium composition (subculturing);
o) By adjusting concentration of auxin:cytokinin ratio in growth medium,
p) cells in callus can be induced to differentiate into roots and shoots;
q) Further growth being encouraged by the use of plant growth substances
1265

until plantlets are large enough


r) To be weaned and planted in sterile soil in green house
s) After acclimatization in green house, the plant are transferred to soil for
field planting

b) Discuss the social and ethical implications of GMO plants using 3 named examples. [6]

1) Definition of GMO 1m
Genetically Modified Organism: Organism that has acquired one or more genes by
artificial means. The genes may or may not be from the same species.

Three named examples + description (each 1 mark)

2) Social implications
(a) Threat to Human Safety
(i) Genetic markers
- Vectors used for transforming plant/crop cells contain antibiotic markers/
kanamycin resistance;
- Concerns that the antibiotic resistant gene if it is not properly broken down by the
digestive system may be passed to potentially harmful bacteria in the human gut
making them resistant to the antibiotics;

(b) Threat to Safety of the Environment


(i) Genetically modified crops might establish themselves as weeds (Bt corn,
Roundup Ready soybean, any valid e.g.)
- Seeds from the transgenic crops that have been modified to withstand
unfavourable environmental conditions, may be carried by the wind to other places.
These crops grow quickly and may establish themselves as weeds;

(ii) Spread of resistance from genetically modified crops to weeds (Any herbicide
resistant plant)
- Pollen grains from genetically modified plants can be carried in the wind and fertilise
wild relatives and weeds;
- -> superweeds. These may become more invasive (overrun agricultural areas
rapidly) or compete with the natural species;

(iii) Upsetting the ecological balance (GM salmon, Bt corn etc.)


- the genetically modified organisms themselves may be more aggressive/ out-
compete other plants in the same area;
- -> may upset the balance of the native species / decrease in biodiversity;

3) Ethical implications
(a) Genetic manipulation of plants may be deemed unacceptable as there are no
rights to tampering with nature and affecting the intrinsic value of the plant ;

(b) Is it acceptable to engineer any organism to be used as food for human


consumption. (Give e.g.)
- Plants are not biologically designed to withstand the stress of producing additional
proteins / toxins;
1266

Golden rice/Bt Corn


(c) Genetic engineering to make a staple crop more resistant in marginal conditions
could be a danger of increased dependence on rich "developed" countries
- i.e. richer countries benefiting at the expense of the poorer countries;
- world food production may be dominated by a small number of large companies with the
technical know-how;

Other points:

(d) Non-mandatory labeling of products containing GMO plants -> consumer rights;

(e) Patent rights to GMO plants -> patenting plants is itself unethical as it reduces
them to the level of objects;

(c) Discuss the limitations for the use of gene therapy in treating diseases in humans. [5]

A) Difficulties in ensuring appropriate regulation / activity / uptake of gene


1. ref. cells make appropriate amounts of gene product / low levels of expression
2. worsen by low efficiency of uptake of DNA by target cells
3. inserted genes in heterochromatic region may not be expressed
4. Right time and in the right location of the body not ascertained
5. Technical limitations in producing large numbers of retroviral vectors
6. Non-target cells may take up gene

B) Immune response against the introduced transgenic cell or vector


1. Repeated therapies may thus be ineffective
2. Danger of viral vector recovering its ability to cause disease in the patient
3. ref. difficulty controlling if the transgene is correctly integrated into the genome ->
insertional mutagenesis
4. development of cancer -> ref. tumour suppressor gene or proto-oncogene

C) Difficulty in treating multigene disorders


1. require multiple replacement of defective alleles
2. ref. one named disorder: cancer, heart disease, high blood pressure, Alzheimer's disease,
arthritis, diabetes etc

D) Short-lived nature of gene therapy


1. target cells may be short-lived or may become unstable after introduction of therapeutic
DNA
2. many rounds of gene therapy required for a long period of time
3. rapidly dividing nature of cells prevents efficiency of treatment if gene is
extrachromosomal
1267

RAFFLES INSTITUTION
2013 Year 6 Preliminary Examination
Higher 2

BIOLOGY 9648/01
Paper 1 Multiple Choice 30th September 2013

1 hour 15 min

Additional Materials: Multiple Choice Answer Sheet

READ THESE INSTRUCTIONS FIRST

Write in soft pencil.


Do not use staples, paper clips, highlighters, glue or correction fluid.
Write your name and shade your Index Number on the Answer Sheet in the spaces provided unless this has
been done for you.

There are forty questions in this paper. Answer all questions. For each question there are four possible
answers A, B, C, and D.

Choose the one you consider correct and record your choice in soft pencil on the separate Answer Sheet.

Read the instructions on the Answer Sheet very carefully.

Each correct answer will score one mark. A mark will not be deducted for a wrong answer.
Any rough working should be done in this booklet.
Calculators may be used.

(Erase all mistakes completely. Do not bend or fold the OMR Answer Sheet).

This document consists of 22 printed pages.

Raffles Institution
Internal Examination

RI 2013 Preliminary Examination 9648/01 [Turn over


1268

1. The diagram below shows a lactating mammary secretory cell. In this cell, milk proteins, such as
casein, are transcribed and translated, and eventually secreted into the lumen of the mammary
gland.

Which of the following shows the most likely sequence of locations involved in this process?

start finish

A 6 3 4 7 2 5 1
B 6 4 3 7 5 2 1
C 4 6 7 3 5 2 1
D 4 3 7 6 2 5 1

RI 2013 Preliminary Examination 9648/01


1269

2. Liver tissue produces an enzyme called catalase which breaks down hydrogen peroxide into
water and oxygen.
2H2O2 2H2O + O2
The rate of this reaction can be determined by measuring the volume of oxygen produced in a
given length of time.
Students added small cubes of fresh liver tissue to a range of hydrogen peroxide solutions and
measured the volumes of oxygen produced. Their data were used to produce the graph
showing how changing the concentration of hydrogen peroxide affected the rate of oxygen
production.

Which statements are correct?


1 At P, the rate of reaction is limited by the concentration of enzyme.
2 At Q, all of the enzyme active sites are occupied by substrate molecules.
3 At Q, the rate of reaction is limited by the concentration of the substrate.
4 R represents Km where the reaction rate = Vmax / 2.
5 At S, all of the enzyme active sites are occupied by substrate molecules.

A 1, 3, 4 and 5 only
B 1, 4 and 5 only
C 2 and 3 only
D 2 and 5 only

3. Which two features contribute to the great tensile strength of cellulose?


1 glycosidic bonds linking the long chains of -1,4-glucose molecules
2 -OH groups of the glucose molecules project outwards and form hydrogen bonds with
neighbouring chains
3 strength of the glycosidic bonds between the neighbouring chains of molecules
4 successive glucose molecules are orientated at 180 to each other

A 1 and 3
B 1 and 4
C 2 and 3
D 2 and 4
RI 2013 Preliminary Examination 9648/01 [Turn over
1270

4. The diagrams below show the results of an investigation into the composition of different
mixtures of amino acids. Each mixture of amino acids was separated using chromatography
which separates the amino acids based on their differential solubility in a propanol and aqueous
ammonia mixture. Each chromatogram was then turned through 90 and the amino acids
separated again by electrophoresis. Electrophoresis separates amino acids based on their
charge using an electrical field.
Which diagram shows an amino acid mixture in which the solubility of some of the amino acids
is the same but the charge on those particular amino acids is different?

RI 2013 Preliminary Examination 9648/01


1271

5. Sex determination in some insects such as bees and wasps is not controlled by sex
chromosomes.
male female
P Q

R S

Q P
female male

Using the diagram, which row in the table shows how sex is determined in these insects?

P Q R S
A n n mitosis mitosis
B n 2n mitosis meiosis
C 2n n meiosis meiosis
D 2n 2n meiosis mitosis

6. Compared to a diploid cell, how much genetic material would the nucleus of a haploid cell of the
same organism contain?
A one quarter
B double
C half
D same amount

RI 2013 Preliminary Examination 9648/01 [Turn over


1272

7. The diagram below shows a replication fork.

VI

IV

II V
III

Which of the following options best describes I, II, III, IV, V and VI?

I II III IV V VI
lagging DNA Okazaki
A 3 end 5 end DNA primer
strand polymerase fragments
leading lagging DNA Okazaki
B 5 end RNA primer
strand strand polymerase fragments
lagging Okazaki
C 5 end 3 end DNA ligase RNA primer
strand fragments
template Okazaki
D 5 end 3 end DNA ligase DNA primer
strand fragments

8. Scientists were able to splice out the promoter of eukaryotic gene and attach it to the end of a
gene as shown below. The gene was reintroduced back to the genome of the organism.

gene gene

Which of the following outcomes is true?


A No transcription of the gene takes place as the promoter cannot initiate transcription of the
non-template strand.
B Transcription initiates but may prematurely terminate due to a premature STOP codon.
C Transcription takes place but translation cannot take place because the genetic code is no
longer the original code.
D Transcription takes place but less translation occurs to obtain functional protein because
the mRNA may form a duplex RNA.

RI 2013 Preliminary Examination 9648/01


1273

9. One of the codons for the amino acid phenylalanine is UUC.


Which of the following is the correct anticodon of the tRNA carrying phenylalanine?
A 5 UUC 3
B 5 AAG 3
C 3 AAG 5
D 3 TTC 5

10. Below shows a partial segment of two variant alleles of the gene coding for EcoRI in bacteria.

5 TTATACCTAATAAATGAACCTGAATTCATGCCGG 3
wild type Promoter
3 AATATGGATTATTTACTTGGACTTAAGTACGGCC 5

5 TTATACCTAATAAATGAACCTGAATTCATGACGG 3
mutant Promoter AATATGGATTATTTACTTGGACTTAAGTACTGCC 5
3

Which of the following mutations is seen in this case?


A Nonsense mutation resulting in premature stop codon.
B Silent mutation resulting in no change in amino acid coded.
C Frameshift mutation resulting in change in amino acid sequence downstream of the
mutation.
D Missense mutation resulting in a different amino acid coded.

11. A complete reproductive cycle of the T4 bacteriophage requires all of the following processes to
occur except
A incorporation of viral DNA into host cell DNA.
B translation of viral mRNA.
C replication of viral genome.
D addition of methyl groups to viral genome.

RI 2013 Preliminary Examination 9648/01 [Turn over


1274

12. Wh
hich of the ffollowing sta
atements be
est distinguiishes the type of virus shown
s below
w?

A esises DNA from a RNA


It synthe A template.
B It has an
n envelope tthat contain
ns some hosst-cell memb
brane.
C It is veryy specific in that it can o
only infect a single kind
d of cell.
D It carriess two copiess of RNA.

13. In the presencce of hydrog gen sulphidde and nitratte ions, the bacterium Thiomargarrita namibiensis
neeeds to turn on the ope erons conta aining the geenes for hyydrogen sulpphide oxidaation and ge
enes
forr nitrate ionss reduction.. Both operoons are turn
ned on in thhe presencee of hydrogeen sulphide and
nittrate ions. N
Neither operon is turned d on unless hydrogen ssulphide andd nitrate ions are prese
ent.
Hyydrogen sulp phide and n nitrate ions a
are acting a
as
A inducerss.
B activatorrs.
C co-repre
essors.
D Represssors.

14. Wh onditions is likely to inte


hich of the ffollowing co erfere with conjugation
c in bacteria?
?

A ent of the recipient cellss with DNasse.


Treatme
B Treatme
ent of the do
onor cells with strong viibration.
C Treatme
ent of the ma
ating cell pa
airs with DN
Nase.
D Treatme
ent of the ma
ating cell pa
airs with stro
ong vibratio
on.

RI 2013 Prelim
minary Examinatiion 9648/01
1275

15. Which of the following options is true with regards to gene expression in yeast and E.coli?

yeast E.coli
transcription & transcription &
polycistronic translation polycistronic translation
polyribosome polyribosome
mRNA occur mRNA occur
simultaneously simultaneously

A
B
C
D

16. Which of the following statements about transcription in eukaryotes is/are incorrect?
1 Acetylation of histones leads to decondensation of heterochromatin to euchromatin for
transcription to take place.
2 RNA polymerase, general transcription factors and specific transcription factors make
up the transcription initiation complex.
3 Specific transcription factors such as repressors bind to silencers to prevent assembly
of the transcription initiation complex.
4 The binding of enhancer proteins to activators result in enhanced rate of transcription.

A 1 and 2 only
B 1 and 3 only
C 2 and 4 only
D None of the above

17. The process of cancer involves


A increasing genetic stability within cells.
B Loss-of-function mutation in a proto-ongene such as the Ras gene.
C arrest of cell cycle by tumour suppressor gene products.
D angiogenesis which is important for growth and metastasis of aberrant cells.

RI 2013 Preliminary Examination 9648/01 [Turn over


1276

10

18. Eye colour of Drosophila is determined by the synthesis of two pigments, red and brown. Wild-
type flies have brick-red eyes. There are also flies with scarlet (bright-red), brown and white
eyes. Flies homozygous for allele s have scarlet eyes whereas flies homozygous for allele b
have brown eyes. The double homozygous recessive genotype results in white eyes.
To investigate the inheritance of eye colour in flies, a geneticist carried out 2 crosses as
described below.

CROSS 1
Two strains of wild-type flies that are heterozygous for each of 2 genes, s and b, affecting eye
colour were mated.
Among 240 progeny,
progeny phenotype number of progeny
wild-type eyes 150
scarlet eyes 36
brown eyes 46
white eyes 8
The results are analysed by chi-squared test to determine if the difference between the
observed and expected numbers are significant. The P-value obtained is greater than 0.05.

CROSS 2
A test-cross was performed on wild-type flies that are heterozygous for recessive allele of each
of 2 genes, s and b.
progeny phenotype percentage / %
wild-type eyes 25.1
scarlet eyes 24.9
brown eyes 25.2
white eyes 24.8

Which statement correctly describes the conclusion that can be drawn from the above
crosses?
A Cross 1 results give evidence that we should reject the null hypothesis, indicating that the
inheritance of the two genes follows Mendelian ratio of 9:3:3:1.
B Cross 1 results give evidence that we should not reject the null hypothesis as there is
interaction between the two gene loci.
C Cross 2 results is used to prove that the inheritance of the two genes follows Mendels law
of segregation.
D Cross 2 results are sufficient to prove that the S/s and B/b gene loci are not linked.

RI 2013 Preliminary Examination 9648/01


1277

11

19. Achondroplastic dwarfism is an autosomal dominant disorder and red-green colour blindness is
an X-linked recessive disorder.
An achondroplastic male dwarf with normal vision marries a colour blind woman of normal
height. The mans father is 1.7 meters tall while both the womans parents are of average
height.

What is the probability that their son will be colour blind and of normal height?
A 0.25
B 0.5
C 0.75
D 1.0

20. Feathers in poultry can be white or coloured and this is controlled by two genes, P/p and Q/q.
The phenotypes of offspring that are expected from mating two birds, each of which is
heterozygous at both loci, are shown in the Punnett square.

gametes PQ Pq p'Q p'q


PQ white feathers white feathers white feathers white feathers
Pq white feathers white feathers white feathers white feathers
p'Q white feathers white feathers coloured feathers coloured feathers
pq white feathers white feathers coloured feathers white feathers

Which of the following best explains the proportion of white to coloured feathers in the Punnett
square?
A Dominant epistasis in which a suppressor prevents the expression of epistatic gene.
B Dominant epistasis in which the epistatic allele is P.
C Recessive epistasis in which colour is recessive to no colour at one allelic pair.
D Recessive epistasis in which the epistatic allele is p.

RI 2013 Preliminary Examination 9648/01 [Turn over


1278

12

21. Achillea millefolium, commonly known as yarrow is a flowering plant that was traditionally used
as a herbal medicine to treat wounds and minor bleedings. It is native to temperate regions of
the Northern hemisphere.
Phenotypic variation in the species has been extensively studied along an altitudinal gradient
from sea level to more than 3000m elevation. It is observed that there is variation in height and
biomass. Adaptation to different local environments also results in variation in physiology such
as dormancy period and resistance to cold.
Which is the best method to determine if the observed phenotypic variation in the species is due
to genotypic variation?
A Searching for chromosomal differences between populations at the extremes of altitude.
B Determine if viable offspring can be produced when plants from phenotypically different
populations are crossed.
C Determine if hybrids between phenotypically different populations can grow at altitudes
intermediate between the parent populations.
D Determine if phenotypic differences are maintained when plants from different altitudes are
grown under one environmental condition.

22. A child with Down syndrome has the genotype P1 P2 P3 for a polymorphism on chromosome 21
that has four different alleles allele P1, allele P2, allele P3, allele P4. The childs mother has
the genotype P1 P2 and the father has the genotype P3 P4.
In which parent did non-disjunction occur, and did this event occur in the first or second meiotic
division?
A father; Meiosis I
B father; Meiosis II
C mother; Meiosis I
D mother; Meiosis II

23. Rotene and oligomycin are two metabolic poisons which affect cellular respiration. The effects
of rotene and oligomycin on aerobic respiration are summarised in the table.

ability to use glucose ability to use oxygen ATP yield


rotene yes no decreases
oligomycin yes yes decreases

Which of the following correctly identifies the specific functions of these two metabolic poisons?

rotene oligomycin
A dissipate proton gradient inhibits ATP synthase
B inhibits ATP synthase electron transport inhibitor
C electron transport inhibitor inhibits ATP synthase
D inhibits ATP synthase dissipate proton gradient

RI 2013 Preliminary Examination 9648/01


1279

13

24. The blue dye DCPIP can be converted to colourless DCPIP as shown below:
DCPIP (blue) reduced DCPIP (colourless)

In an experiment, green chloroplast extract was first mixed with DCPIP and the extract turned
blue-green. After exposure to 2 hours of light in the presence of ample carbon dioxide and
water, the extract became green again.

Which of the following correctly shows the products that were formed after the experiment?

O2 ATP reduced NADP glucose


A + +
B + + +
C +
D +

25. Glycophorin, an integral membrane protein, has a single transmembrane helix.

Hydropathy plots are used to estimate the relative hydrophobicity of amino acid residues. Which
of the following hydropathy plot most likely represents the transmembrane nature of
glycophorin?

A B

C D

RI 2013 Preliminary Examination 9648/01 [Turn over


1280

14

26. The d
diagram bellow shows a neuron. Tw
wo stimuli a
are applied, one at X an
nd the otherr at Y.

Whicch of the following corre


ectly show the change
es in membrrane potentials at X, Y and
a Z?

X Y Z

RI 2013 Prelim
minary Examinatiion 9648/01
1281

15

27. It is ffound that th


he absence
e of ATP lim
mits synapticc transmission. Which of
o the follow
wing explain
ns
this oobservation? ?

protein affected e
effect

A Na+-K+ A
ATPase ation of Na+ and K+ gra
no restora adient acrosss
mem
mbrane

B acetylcholinesterase acetylch
holine canno
ot be broken
n down and
hence remains in synaptic cleft

C Ca2+ pump no resstoration of Ca2+ gradie


ent across
memmbrane

D vo d Ca2+ chann
oltage-gated nel ux of Ca2+ a
no influ and fusion o
of synaptic
vesicles w
with membra ane

RI 2013 Prelim
minary Examinatiion 9648/01 [Turn ove
er
1282

16

28. The figure below shows the time course of glucose uptake by the working rat heart in the
absence (I) of insulin and in the presence (II) of insulin. The lower the amount of total blood
glucose, the greater is the uptake of glucose by the rat heart.

600

Total blood glucose ( mol)


I
500

400

300
II

200
10 30 50 70 90 110 130
Time (min)

From the given data, which of the following statements can be concluded to be correct?

1 Cells in the rats heart have receptors for insulin.


2 There is increased uptake of glucose by the rats heart in the presence of insulin.
3 Release of insulin is stimulated by increase in blood glucose level.
4 The rat is given a carbohydrate-rich meal prior to the experiment.

A 1 and 2 only
B 1 and 3 only
C 2 and 3 only
D 3 and 4 only

RI 2013 Preliminary Examination 9648/01


1283

17

29. The diagram below shows the cell signalling pathway involved in cell proliferation and
differentiation in the shoot meristem.

Legend:
inhibition

Which one of the following sequences regarding the activation of the above signalling pathway
is correct?
A binding of CLV3 to subunits CLV2 and CLV1 dimerisation of CLV2 and CLV1
autophosphorylation of serine/threonine residues activation of Rho-like protein
inhibition of gene transcription
B dimerisation of CLV2 and CLV1 binding of CLV3 to dimer autophosphorylation of
serine/threonine residues activation of Rho-like protein activation of gene
transcription
C binding of CLV3 to subunits CLV2 and CLV1 dimerisation of CLV2 and CLV1
dephosphorylation of serine/threonine residues by protein phosphatase activation of
Rho-like protein inhibition of gene transcription.
D dimerisation of CLV2 and CLV1 binding of CLV3 to dimer dephosphorylation of
serine/threonine residues by protein phosphatase activation of Rho-like protein
inhibition of gene transcription

RI 2013 Preliminary Examination 9648/01 [Turn over


1284

18

30. The Galapagos Islands aree a group of volcanic islands in the eastern Pacific O Ocean, abou ut
10000km from So
outh Americca. The dia
agram below w illustratess the finche
es found on the islandss;
they resemble e
each other closely butt differ in th
heir feeding g habits and in the sh
hape of theeir
beakks.

Whicch statement below exp


plains how sso many distinct and ye
et similar spe
ecies arose
e?
A As a resultt of the diffe
erent food ssources ava
ailable, the ancestral
a pecies acquired differen
sp nt
mutations wwhich resulted in the emergence o of a variety o
of beak shaapes.
B The finchees were su ubjected to o similar se election pre
essures wh
hich resulte
ed in simila
ar
phenotypicc characterisstics in the d
distinct populations.
C The differe
ence in food
d sources sserved as a selection p
pressure, alllowing only finches witth
certain bea
ak shapes to
o survive an
nd reproducce.
D The finche es migrated only to envvironments that they arre already a
adapted to; finches witth
different ph
henotypes migrated
m to different isla
ands.

RI 2013 Prelim
minary Examinatiion 9648/01
1285

19

31. Five species, 1, 2, 3, 4 and 5 possess the enzyme cytochrome c oxidise. The primary structure
of the enzyme was determined for each species and the number of amino acid differences
between species is given below.
1 2 3 4 5
1 0 7 8 20 23
2 7 0 3 19 22
3 8 3 0 18 21
4 20 19 18 0 20
5 23 22 21 20 0
From the above evidence, which one of the following represents the probable evolutionary
relationships between the species?
A B C D

32. An experiment is conducted using a small population of 25 stoneflies with the frequency of allele
A at 50% (i.e. 1 in 2 alleles present in the population is allele A).
The graph below shows the changes in the frequency of allele A with subsequent generations.
The experiment is repeated several times with the same condition and each trial is represented
by a line on the graph.
All AA
frequency of allele A

Allele A lost
generation (25 stoneflies at the start of each)
Which of the following statement(s) is/are correct?
1 The populations were undergoing genetic drift.
2 The populations were undergoing selection.
3 Allele A will be lost in approximately 50% of such trials.
4 Over time, 2 separate species will emerge.

A 1 only
B 2 only
C 1 and 3 only
D 2, 3 and 4 only
RI 2013 Preliminary Examination 9648/01 [Turn over
1286

20

33. Which of the following statements is true?

genomic library cDNA library


A bacteria plasmid used as vector bacteriophage used as vector
B fragments contain introns no introns
C used for studying genes responsible for the used for studying alleles of a particular
specialised functions of a particular cell type gene
D represents a sample of genes expressed at represents a random sample of all the DNA
a certain time in the life of an organism sequences in an organism

34. If the first three nucleotides in a six-nucleotide restriction site are CTG, what would the next
three nucleotides most likely be?
A AGG
B GTC
C CTG
D CAG

35. The human X-linked disease haemophilia A is a result of a mutation to a gene that codes for
Factor VIII, a component of the blood coagulation pathway. The haemophilia allele is recessive
to the normal allele.
For the gene locus of haemophilia A, the two alleles exist:

0.9 kbp 0.3 kbp


normal allele
probe
1.2 kbp
hemophilia allele

A man who suffers from the disease married a woman who is a carrier of the disease. The
couple has 2 children. Child R is normal while Child S is affected. Which autoradiogram below
correctly identifies the children?

RI 2013 Preliminary Examination 9648/01


1287

21

36. Which of the following best describes the complete sequence of steps occurring during every
cycle of PCR?
1 The primers hybridise to the target DNA.
2 The mixture is heated to a high temperature to denature the double stranded DNA.
3 Fresh Taq polymerase is added.
4 Taq polymerase extends the primers to make a copy of the target DNA.

A 2, 1, 4
B 3, 2, 4
C 2, 3, 1, 4
D 2, 1, 3, 4

37. Stem cells are found in many tissues that require frequent cell replacement such as the skin, the
intestine or the blood.
However, within their own environments, a bone marrow cell cannot be induced to produce a
skin cell and a skin cell cannot be induced to produce a bone marrow cell. Which statement
explains this?
A Genes not required for a particular cell line are methylated.
B mRNA that is not required for a particular cell line is destroyed.
C Genes not required for a particular cell line are removed using restriction enzymes.
D Different stem cells have only the genes required for their particular cell line.

38. Retrovirus is a more suitable vector than liposome for gene therapy as it
A is non-pathogenic and does not trigger an immune response.
B is more specific and causes mutation to the defective allele.
C has a phospholipid bilayer that can fuse with the target cell.
D results in more stable gene integration.

RI 2013 Preliminary Examination 9648/01 [Turn over


1288

22

39. Which of the following correctly match the plant tissue culture techniques to their main
purposes?

anther culture protoplast culture suspension culture callus culture


to produce to screen for to extract useful
to produce hybrid
A genetically identical recessive desirable secondary
plants
plants traits metabolites
to screen for to extract useful to produce
to produce hybrid
B recessive desirable secondary genetically identical
plants
traits metabolites plants
to produce plantlets
which are to produce to trigger shoot and to produce disease
C
genetically non- transgenic plants root formation free plants
identical
to produce plantlets
to produce disease to trigger shoot and to produce which are
D
free plants root formation transgenic plants genetically non-
identical

40. Bt gene from Bacillus thuringiensis may be inserted into cotton plant cells to produce Bt cotton
plants. Insecticide use and yield in India were compared for Bt cotton hybrid (XBt), the same
hybrid X but without the Bt gene (X_), and another hybrid widely grown in that particular locality
(Y). This process was repeated at more than 150 locations. The table below shows the results:

The following are the conclusions that are drawn from the data:
1 All insect pests are killed when they consume the Bt crop.
2 Bt cotton reduces the amount of pesticide used.
3 Both yield and quality of cotton from XBt crop improved.
4 Bt cotton increases cost effectiveness.
5 Both X_ and Y hybrids contain susceptible genes to the pest.
6 Bt toxin is not found in the plant sap.
Which of the conclusions stated are correct?
A 1, 2 and 3
B 2, 3 and 5
C 4, 5 and 6
D 2, 4 and 6
- End of Paper -
RI 2013 Preliminary Examination 9648/01
1289

23

Answers:

1. C 11. A 21. D 31. B


2. A 12. A 22. C 32. C
3. D 13. A 23. C 33. B
4. B 14. D 24. A/C 34. D
5. B 15. B 25. D 35. C
6. C 16. C 26. D 36. A
7. B 17. D 27. C 37. A
8. D 18. D 28. A 38. D
9. C 19. B 29. A 39. B
10. A 20. B 30. C 40. D

RI 2013 Preliminary Examination 9648/01 [Turn over


1290

RAFFLES INSTITUTION
2013 Year 6 Preliminary Examination
Higher 2

CANDIDATE
NAME

CIVICS
GROUP
1 3 S 0 3 INDEX
NUMBER

BIOLOGY 9648/02
th
Paper 2 Core paper 16 SEPTEMBER 2013
2 hours
Additional materials: Answer Sheet

READ THESE INSTRUCTIONS FIRST For Examiners Use

Write your index number, CT group & name on all the work you hand in. Section A
Write in dark blue or black pen on both sides of the paper.
You may use a soft pencil for any diagrams, graphs or rough working. 1 /12
Do not use staples, paper clips, highlighters, glue or correction fluid.

Section A 2 /13

Answer all questions. 3 /12


Section B
4 /13
Answer either ONE question.
5 /11
At the end of the examination, hand in your essay SEPARATELY.
The number of marks is given in brackets [ ] at the end of each question 6 /10
or part question.
7 /90

Section B

8 or 9 /20

Total /100

This document consists of 19 printed pages.

RI 2013 Preliminary Examination 9648/02


Raffles Institution
Internal Examination
[Turn over
1291
2 For
Examiners
Use

S
Section A
Answ
wer all the q n this sectio
questions in on.

1 Fig. 1..1 shows th


he sequence of bases in a section
n of a gene
e coding forr the first se
even amino
o
acids o
of a polypep
ptide chain.

A A A C T G T T T C T GC T G G AA C A T

Fig. 1.1

(a
a) (i) M
Methionine is coded by
b the initiattion codon. On Fig. 1.1, label thee 3' and 5' end of the
e
D
DNA templaate strand. [1]

(ii) L
List the corrresponding codons in tthe mRNA transcribed
t from the te mplate in Fig.
F 1.1.







.. [1]

T
The polypep ptide was hydrolysed
h tto release the
t first sev
ven amino aacids. It con
ntained fourr
d
different am
mino acids. The
T numbe r of each ty ype obtainedd is shown iin the Table
e 1.1.

Table
e 1.1

(iii) U
Using inform
mation in Ta
able 1.1, wo
ork out the order of am
mino acids inn the polype
eptide.







.. [1]

(b
b) (i) EExplain whyy a single base
b deletio
on in the DN
NA sequenc
ce is more likely to pro
oduce non--
ffunctional proteins
p than
n a single b
base substitution.







.. [3]

RI 2013 Preliminarry Examination 9648/02


9
1292 For
3 Examiners
Use

(ii) Describe the effect of replacing guanine (underlined in Fig.1.1) with adenine.

....

....

....

... [2]

(c) Explain how the structure of the nuclear envelope facilitates translation.

...

...

...

.. [2]

(d) A variety of related proteins can be produced from a single gene. State the process and
explain how the same gene can produce different protein products.

.......

...

...

.. [2]

[Total : 12]

RI 2013 Preliminary Examination 9648/02 [Turn over


1293
4 For
Examiners
Use

2 Chromosome aberrations can have devastating effects on human health.

(a) Down syndrome is a disorder caused by an aberration in chromosomal number. Individuals


with Down syndrome have three copies of chromosome 21, which typically results in some
delay in cognitive ability and physical growth, as well as a characteristic set of facial features.

Fig. 2.1 is a diagrammatic representation of chromosome 21, showing the location of two
different RFLP loci.

RFLP 53

RFLP 99

Fig. 2.1

Fig. 2.2 is a pedigree of a family and the results following restriction digestion, gel
electrophoresis and Southern blotting with specific probes of the genetic material of some
family members.

died

Individual 1 Individual 2

Individual 3 Individual 4

Allele B
Allele C RFLP 53

Allele H
Allele I
Allele J RFLP 99
Allele K

Fig. 2.2

RI 2013 Preliminary Examination 9648/02


1294 For
5 Examiners
Use

(i) Identify which individual (3 or 4) has three copies of chromosome 21. Give a reason for
your answer.

.. [2]

(ii) Non-disjunction of chromosomes during the formation of gametes would have resulted in
gametes with two copies of chromosome 21.
In which parent (individual 1 or 2) did the non-disjunction of chromosomes occur?

.. [1]

(iii) State when the non-disjunction event occurred. Explain your answer.

.. [2]

(iv) Draw chromosome(s) 21 in the gamete that resulted in the individual having Down
syndrome. Clearly label the alleles at the RFLP loci on each homologue of chromosome
21.

gamete

[1]

RI 2013 Preliminary Examination 9648/02 [Turn over


1295
6 For
Examiners
Use

(b) Fig. 2.3 shows the formation of a Philadelphia chromosome which is a chromosomal
abnormality that results from a reciprocal translocation between chromosome 9 and 22. This
chromosomal abnormality is associated with chronic myelogenous leukemia which is
considered a form of cancer.

Philadelphia
Chromosome 22 chromosome

Fused
Bcr gene
Bcr-Abl gene

translocation

Abl gene

Chromosome 9

Changed
Chromosome 9

On chromosome 9
Abl gene On Philadelphia
chromosome

On chromosome 22 translocation
Bcr gene Fused Bcr-Abl gene

Fig. 2.3

The Abl gene normally codes for a receptor involved in signal transduction that results in the
cell proliferation.
Using the information in Fig. 2.3, suggest how the fused gene product can result in cancer.

...

...

...

...

...

.. [3]

RI 2013 Preliminary Examination 9648/02


1296 For
7 Examiners
Use

(c) Cancer cells require tetrahydrofolate to make purines and pyrimidines for cell proliferation.
Conversion of precursors to tetrahydrofolate is carried out by the enzyme dihydrofolate
reductase (DHFR). Competitive inhibition of DHFR by administering the drug Methotrexate
(MTX) is a key treatment for certain cancers such as acute lymphoblastic leukemia.

(i) Using the information provided, comment on the structure of Methotrexate.

.. [1]

(ii) However, researchers have since found that one of the main causes of failure in the
treatment of acute lymphoblastic leukemia is MTX resistance due to the amplification of
the DHFR gene.
During the early stages of the research, scientists suggested a mechanism for the
amplification of the DHFR gene as shown in Fig. 2.4.

Direction of rolling

Original DNA strand


x Newly synthesised
DNA strand

3- OH 5- P

5- P

5- P

Fig. 2.4

With reference to Fig. 2.4, describe the process.

.. [3]

[Total : 13]

RI 2013 Preliminary Examination 9648/02 [Turn over


1297
8 For
Examiners
Use

3(a) A study was conducted on the lac operon of E. coli. Fig. 3.1 below shows the activity of -
galactosidase as a function of time for a culture of E. coli. The bacterial cells were initially
grown in glycerol as the only carbon source. At time A, lactose was added to the culture. When
the lactose supply is exhausted at time B, glucose was added.

Activity of Time B
-galactosidase
(a. u.)

Time A

Time (sec)

Fig. 3.1

(i) Why is the lac operon considered inducible?

.. [2]

(ii) Explain the change in -galactosidase activity from time A to B. [4]

.. [4]

(iii) Draw in Fig. 3.1 what a continuation of the graph would look like from time B.
[1]

RI 2013 Preliminary Examination 9648/02


1298 For
9 Examiners
Use

(b) Fig. 3.2 shows the arrangement of the lac operon and its regulatory gene.

promoter promoter operator

lac I lac Z lac Y lac A

Fig. 3.2

A series of mutations was introduced to the lac operon.

(i) The operator is deleted. Assuming lactose is present, what is the effect of this on the
transcription of lac Z, lac Y and lac A genes?

.. [2]

(ii) A point mutation was introduced into lac Z, resulting in the production of a truncated non-
functional protein. Assuming lactose is present, explain if the proteins encoded by lac Y
and lac A genes are produced?

.. [2]

(c) An E. coli strain was isolated from the intestines of a mammal. The bacterium was found
to have two different copies of the lac operon. Describe a genetic transfer process that
resulted in this observation.

.. [1]

[Total : 12]

RI 2013 Preliminary Examination 9648/02 [Turn over


1299
10 For
Examiners
Use

4 An absorption spectrum
s is a graph o f the absorrption of diffferent wavvelengths off light by a
photossynthetic pig
gment.
An acttion spectrum is a graaph of the rrate of phottosynthesis at differentt wavelengtths of light.
Fig. 4..1 shows thhe absorptio
on spectra of chlorophhyll a and chlorophyll
c b as well as
s an action
spectruum.

percentage
light
absorption

Fig. 4.1
1

(a
a) With re
eference to Fig. 4.1,
(i) ccompare the
e absorptioon spectra of chloroph
hyll a and ch
hlorophyll b..







.. [2]

(ii) e
explain the shape of th
he action sp
pectrum.







.. [2]

RI 2013 Preliminarry Examination 9648/02


9
1300 For
11 Examiners
Use

(b) The light-dependent stage of photosynthesis takes place on the thylakoids of the chloroplast.
Fig. 4.2 represents a section through a thylakoid to show some of the components involved in
the light-dependent stage.

Fig. 4.2

(i) Identify the structures labelled R and S.

R : S : [2]

(ii) Atrazine is a widely used herbicide (weed killer). It binds to a chloroplast protein involved
in electron transfer between photosystems.
Explain how atrazine works as an effective herbicide.

.. [3]

RI 2013 Preliminary Examination 9648/02 [Turn over


1301
12 For
Examiners
Use

(c) In an investigation, mammalian liver cells were homogenised (broken up) and the resulting
homogenate centrifuged. Samples of the complete homogenate and samples with only
mitochondria were incubated with:

1 glucose
2 pyruvate
3 glucose and cyanide
4 pyruvate and cyanide

Cyanide inhibits oxidative phosphorylation.

After incubation, the presence or absence of carbon dioxide and lactate in each sample was
determined. The results are summarised in Table 4.1.

Sample of homogenate

Complete Only mitochondria


Carbon dioxide Lactate Carbon dioxide Lactate
1 glucose
X X
2 pyruvate
X
3 glucose and
X X X
cyanide
4 pyruvate and
X X X
cyanide

X = absent = present

Table 4.1

(i) Explain why carbon dioxide is produced when mitochondria are incubated with pyruvate
but not when they are incubated with glucose.

.. [2]

(ii) With reference to Table 4.1, explain the difference in the results when the complete cell
homogenate is incubated with 2 and 4.

.. [2]

RI 2013 Preliminary Examination 9648/02


1302 For
13 Examiners
Use

[Total : 13]]

5 Grove snails are known to have


h brightlyy coloured shells, with the alleless CP and CY coding forr
pink an
nd yellow coloured
c she
ells respecttively. Shellls of the gro
ove snails hhave differe
ent banding
B O
pattern
ns as well, with allele
e B confe rring a she ell with ban nds and al lele B conferring an
unbandded shell.

Banded
d snail Unb
banded snail

A grovve snail with


h a banded d pink shell is crossed with another grove snnail with ann unbanded
yellow shell, and the resultaant F1 gene ration all haad unbandeed pink sheells. The F1 were test--
ed and gave
crosse e rise to 250
0 offspring w
with the follo
owing phen
notypes:

ban
nded pink sh
hell 121
unb
banded pinkk shell 14
ban
nded yellow
w shell 11
unb
banded yello
ow shell 104
4

(a
a) State tthe expecte
ed offspring phenotypicc ratio of the
e test cross..








.. [1]

(b
b) (i) S our working clearly, calcculate the 2 value in the space beelow.
Showing yo

RI 2013 Preliminaryy Examination 9648/02


9 [Turn over
1303
14 For
Examiners
Use

[2]
(ii) Using the table, explain the conclusion drawn from the calculated value in b(i).
2 2

.. [2]

(c) With the use of a genetic diagram, explain the observed results of the test cross.

[5]

RI 2013 Preliminary Examination 9648/02


1304 For
15 Examiners
Use

The saame test crross was ca arried out a


another time and the resulting
r nuumber of baanded pinkk
snails and unbanded yellow snails were e equal. Thhe resulting number off unbanded pink snailss
o that of the banded ye
was also equal to ellow snails.. Through th
his test crosss, the 2 ge
ene loci are
e
determ
mined to be 28 map uniits apart.

(d
d) State tthe observe
ed phenotyp
pic ratio of th
he offspring
g from this test cross.








.. [1]

[Total : 11]]

6 Homeo ostatic mecchanisms maintain a co


onstant environment in the body.
Fig. 6..1 shows chhanges in plasma
p at occurred in a person
gluccose concentration tha n who wentt
withou
ut food for so
ome time.

Fig. 6.1
1

(a
a) Use evvidence from
m Fig. 6.1 to explain th
he change in plasma glucose conccentration.







...

..






..




..





..






..






...



RI 2013 Preliminaryy Examination 9648/02


9 [Turn over
1305
16 For
Examiners
Use

.. [4]

Adrenaline is a signal molecule that stimulates similar cellular responses from liver cells and
adipose cells in a way similar to that of glucagon.

Step 1

Step 2

Fig. 6.2
(b) With reference to Fig. 6.2,
(i) describe the events occurring in step 1 after the binding of adrenaline molecule to the
adrenergic receptor.

.. [2]

(ii) explain the significance of step 2 that results in effective signal transduction.

RI 2013 Preliminary Examination 9648/02


1306 For
17 Examiners
Use

.. [2]
(iii) describe the series of events that occur after the activation of protein kinase A that
results in a cellular response.

.. [2]

[Total : 10]

7 For a long time, the prevailing idea among evolutionary biologists is that geographical barriers
are a prerequisite for speciation. However, recently, more and more examples of an alternative
speciation process which does not require a geographical barrier were discovered but this
remains the exception rather than a norm.

(a) (i) What alternative speciation process are we referring to?

.. [1]

One case study involved studying the palm genus Howea from Lord Howe Island, a tiny
volcanic island in the Tasman Sea. The two related palm species found on the island have a
different soil preference. The different types of soil are interspersed throughout the island and
pollen from the palms is wind-dispersed.

RI 2013 Preliminary Examination 9648/02 [Turn over


1307
18 For
Examiners
Use

Fig. 7.1

(ii) Explain how speciation could have taken place to give rise to the two sister species from
a common ancestor.

.. [5]

(iii) Give an evidence why scientists can rule out geographical isolation mechanisms?

.. [1]

RI 2013 Preliminary Examination 9648/02


1308 For
19 Examiners
Use

(b) Howea belmoreana goes by the names, curly palm, kentia palm and Belmore sentry palm
while Howea forsteriana goes by the names thatch palm and kentia palm.

Explain why the scientific name is better than the common names.

...

...

...

.. [2]

[Total: 9]

Section B
Answer EITHER 8 OR 9.

Write your answers on the separate answer paper provided.


Your answers should be illustrated by large, clearly labeled diagrams, where appropriate.
Your answers must be in continuous prose, where appropriate.
Your answers must be set out in sections (a), (b) etc., as indicated in the question.

8 (a) Explain how meiosis can contribute to genetic variation in offspring. [6]

(b) Discuss how bacteria evolve through natural selection and the implication in [8]
antibiotic resistance.

(c) Describe the link between the frequency of sickle cell anaemia allele and the [6]
number of cases of malaria.

[Total: 20]

9 (a) Explain how gene expression can be controlled at the translational level and [8]
describe the mechanisms present in prokaryotes and eukaryotes.

(b) Explain how the structure of specific transcription factors is linked to their roles [7]
in control of eukaryotic gene expression.

(c) With reference to changes in protein structure, explain how the level of protein [5]
activity can be regulated post-translationally.

RI 2013 Preliminary Examination 9648/02 [Turn over


1309
20 For
Examiners
Use

[Total: 20]

---- End of Paper -----

RI 2013 Preliminary Examination 9648/02


1310

S
Section A
Answ
wer all the q n this sectio
questions in on.

1 Fig. 1..1 shows th


he sequence of bases in a section
n of a gene
e coding forr the first se
even amino
o
acids o
of a polypep
ptide chain.

5 A A A C T G T T T C T G C T G G AA C A T 3

Fig. 1.1

a)
(a (i) M
Methionine is coded by
b the initiattion codon. On Fig. 1.1, label thee 3' and 5' end of the
e
D
DNA templaate strand. [1]

(ii) L
List the corrresponding codons in tthe mRNA transcribed
t from the te mplate in Fig.
F 1.1.

AUG
A UUC C
CAG CAG AAA
A CAG UUU
U
A: 3 UUU GAC
C AAA GAC
C GAC CUU
U GUA 5
[
[1]

T
The polypep ptide was hydrolysed
h tto release the
t first sev
ven amino aacids. It con
ntained fourr
d
different am
mino acids. The
T numbe r of each ty ype obtainedd is shown iin the Table
e 1.1.

Table
e 1.1

(iii) U
Using inform
mation in Ta
able 1.1, wo
ork out the order of am
mino acids inn the polype
eptide.

mett phe
e gln
n glln ly
ys g
gln phe
p

R
R: reverse sequence [1]

(b
b) (i) EExplain whyy a single base
b deletio
on in the DN
NA sequenc
ce is more likely to pro
oduce non--
ffunctional proteins
p than
n a single b
base substitution.

1
1. deletionn results in
n frameshiift
2
2. Change e in sequen nce of codo
ons thus /ccausing all amino acidids downsttream of
mutatioon to chang ge
3
3. substituution usually cause cchange in 1 aa/1 codo on;
4. or may result in silent mutattion that re
4 esults in no
o change inn aa (if it oc
ccurs at
3rd base
e as a resuult of degen
neracy of genetic
g codde) Or aa haas similar property
p
5
5. Substittution may not lead too conforma ational change of pro tein but de eletion
mutatioon will. WTTE

RI 2013 Preliminaryy Examination 9648/02


9

[Turn
n over
1311
2 For
Examiners
Use

[3]
(ii) Describe the effect of replacing guanine (underlined in Fig.1.1) with adenine.

AAG AAA / UUC UUU


Due to degenerate code (as more than one codon may code for the same
aa);
still code for phe / no change in aa (silent mutation);
[2]

(c) Explain how the structure of the nuclear envelope facilitates translation.

1. The presence of nuclear pores* ;


2. Which provide the passage for mature mRNA to move out of nucleus to ribosome in
cytoplasm for translation;
3. Ribosomal subunits to move out of nucleus to cytoplasm to form ribosome /tRNA
move out of nucleus to cytoplasm to undergo amino acid activation;
4. Ribosomal proteins to move into nucleus from cytoplasm to form ribosomal subunit;

Pt 1 must be in
Mark for 2 to 4 is awarded for direction of movement and molecule [2]

(d) A variety of related proteins can be produced from a single gene. State the process and
explain how the same gene can produce different protein products.

1. Alternative splicing*;
(Spliceosome binds to intron-exon boundaries in the pre-mRNA)
2. Resulting in different exons being spliced together in different ways leading to
formation of many different mature mRNA resulting in different proteins translated
[2]

[Total : 12]

RI 2013 Preliminary Examination 9648/02


1312 For
3 Examiners
Use

2 Chrom
mosome abe
errations can have devvastating efffects on hum
man health..

(a
a) Down syndrome is a disorder caused by an aberrration in chromosomaal number. Individualss
with D
Down syndro ome have three copie es of chrom
mosome 21, which typpically resullts in some e
delay iin cognitive ability and physical grrowth, as we
ell as a characteristic sset of facial features.

Fig. 2.1 is a diagrammatic representa


ation of chrromosome 21, showinng the location of two
o
different RFLP locci.

RFLP 53
3

RFLP 99
9

Fig. 2.1

Fig. 22.2 is a pedigree


p off a family and the results following resttriction digestion, gel
electro
ophoresis and
a Southern blotting with speciffic probes of the geneetic materia
al of some
e
family members.

died

Individu
ual 1 Ind
dividual 2

Individual 3 Individu
ual 4

Allele B
Allele C RFLP 53

Allele H
Allele I
Allele J RFLP 99
Allele K

Fig. 2.2

RI 2013 Preliminaryy Examination 9648/02


9 [Turn over
1313
4 For
Examiners
Use

(i) Identify which individual (3 or 4) has three copies of chromosome 21. Give a reason for
your answer.

1. Individual 3
Reasons (any one):
2. 3 bands/alleles of RFLP 99
Or
3. Has 1 copy of allele H, 1 copy of allele I and 1 copy of allele J of RFLP 99
[2]

(ii) Non-disjunction of chromosomes during the formation of gametes would have resulted in
gametes with two copies of chromosome 21.
In which parent (individual 1 or 2) did the non-disjunction of chromosomes occur?

1. Individual 1 [1]

(iii) State when the non-disjunction event occurred. Explain your answer.

(Assuming that there is no crossing-over. This is a valid assumption as there is no


mention of crossing over occurring in the question. This was the assumption that you
were expected to make.)

1. Anaphase I or Meiosis I
2. Homologous chromosomes with different combinations of RFLP alleles (BH on one
and BJ on another) move to the same daughter cell [2]

(However if crossing over occurred in Prophase I between RFLP 53 and RFLP 99.)

3. Anaphase II or Meiosis II
4. Crossing over occurred between RFLP 53 and RFLP 99 (in prophase I) and the non-
identical sister chromatids moved to the same daughter cell in Meiosis II

(iv) Draw chromosome(s) 21 in the gamete that resulted in the individual having Down
syndrome. Clearly label the alleles at the RFLP loci on each homologue of chromosome
21.

B B

J H

gamete

RI 2013 Preliminary Examination 9648/02


1314 For
5 Examiners
Use

[1]
(b) Fig. 2.3 shows the formation of a Philadelphia chromosome which is a chromosomal
abnormality that results from a reciprocal translocation between chromosome 9 and 22. This
chromosomal abnormality is associated with chronic myelogenous leukemia which is
considered a form of cancer.

Philadelphia
Chromosome 22 chromosome

Fused
Bcr gene
Bcr-Abl gene

translocation

Abl gene

Chromosome 9

Changed
Chromosome 9

On chromosome 9
Abl gene On Philadelphia
chromosome

On chromosome 22 translocation
Bcr gene Fused Bcr-Abl gene

Fig. 2.3

The Abl gene normally codes for a receptor involved in signal transduction that results
in the cell proliferation.
Using the information in Fig. 2.3, suggest how the fused gene product can result in cancer.

1. Fused gene is an oncogene*


2. As a result of gain-in-function* mutation of the gene
3. Which results in the formation of abnormal/different receptor that is able to activate relay
proteins without signal molecules (wtte)
or
Constitutively active (even without signal)
4. Which causes uncontrolled cell division/proliferation [3]

RI 2013 Preliminary Examination 9648/02 [Turn over


1315
6 For
Examiners
Use

(c) Cancer cells require tetrahydrofolate to make purines and pyrimidines for cell proliferation.
Conversion of precursors to tetrahydrofolate is carried out by the enzyme dihydrofolate
reductase (DHFR). Competitive inhibition of DHFR by administering the drug Methotrexate
(MTX) is a key treatment for certain cancers such as acute lymphoblastic leukemia.

(i) Using the information provided, comment on the structure of Methotrexate.


1. Methotrexate is a competitive inhibitor and is complementary in shape and charge to
the active site of DHFR.
Or
Similar in shape and charge to the precursors of tetrahydrofolate
[1]

(ii) However, researchers have since found that one of the main causes of failure in the
treatment of acute lymphoblastic leukemia is MTX resistance due to the amplification of
the DHFR gene.
During the early stages of the research, scientists suggested a mechanism for the
amplification of the DHFR gene as shown in Fig. 2.4.

Direction of rolling

Original DNA strand


x Newly synthesised
DNA strand

3- OH 5- P

5- P

5- P

Fig. 2.4

With reference to Fig. 2.4, describe the process.


1. DHFR gene is excised and circularised;
2. (1) One strand of a double-stranded DNA is nicked/cut at a site;
3. (2) Using the un-nicked/intact strand as a template;
4. (DNA polymerase) adds nucleotides to the free 3-OH end to synthesise a
complementary DNA strand;
5. (3) 5-P end of the nicked strand is displaced as synthesis/strand elongation
proceeds;
6. (3) + (4) Displaced strand can be used as a template and replicated discontinuously;
[3]

[Total : 13]

RI 2013 Preliminary Examination 9648/02


1316 For
7 Examiners
Use

3(a) A study was conducted on the lac operon of E. coli. Fig. 3.1 below shows the activity of -
galactosidase as a function of time for a culture of E. coli. The bacterial cells were initially
grown in glycerol as the only carbon source. At time A, lactose was added to the culture. When
the lactose supply is exhausted at time B, glucose was added.

Activity of Time B
-galactosidase
(a. u.)

Time A

Time (sec)

Fig. 3.1
(i) Why is the lac operon considered inducible?

1. Expression of the 3 genes (lac Z, lac Y and lac A*) is usually off/not transcribe ;
2. but can be turned on/trascribed in the presence of inducer allolactose/lactose ;
[2]

(ii) Explain the change in -galactosidase activity from time A to B. [4]

High lactose,
1. Time is needed for the expression /activation of lac operon to produce enzyme before
lactose can be metabolised
2. Lactose converted to allolactose
3. Lactose/ allolactose binds to and inactivates/alters tertiary structure of repressor such
that it fails to bind to operator
4. RNA polymerase is free to bind to promoter and actively transcribe structural genes of
the operon.
Low glucose
5. High cAMP* will result in active catabolite activator protein (CAP) that strengthen
affinity of promoter for RNA polymerase. Thus, rate of transcription increases [4]

RI 2013 Preliminary Examination 9648/02 [Turn over


1317
8 For
Examiners
Use

(iii) Draw in Fig. 3.1 what a continuation of the graph would look like from time B. [1]
Time B
Activity of
-galactosidase
(a. u.)

Time A

Time (sec)
Figure 2.1
(b) Fig. 3.2 shows the arrangement of the lac operon and its regulatory gene.

promoter promoter operator

lac I lac Z lac Y lac A

Fig. 3.2

A series of mutations was introduced to the lac operon.

(i) The operator is deleted. Assuming lactose is present, what is the effect of this on the
transcription of lac Z, lac Y and lac A genes?

1. There is no operator for the repressor to bind to thus;


2. RNA polymerase can bind to promoter to start transcription of the genes; [2]

(ii) A point mutation was introduced into lac Z, resulting in the production of a truncated non-
functional protein. Assuming lactose is present, explain if the proteins encoded by lac Y
and lac A genes are produced?

1. Yes they will be produced;


2. The enzymes are translated separately because each has its own start and stop
codon on the polycistronic mRNA, so an unexpected stop codon in one does not
affect the others; [2]

(c) An E. coli strain was isolated from the intestines of a mammal. The bacterium was found
to have two different copies of the lac operon. Describe a genetic transfer process that
resulted in this observation.

1. Conjugation* - F plasmid carrying a second copy of lac operon is transferred to this


bacterial cell;
2. Transduction* - bacteriophage added a second copy of lac operon from a previous
host;
3. Transformation* - DNA fragment containing a second copy of lac operon taken up by
cell; [1]

RI 2013 Preliminary Examination 9648/02


1318 For
9 Examiners
Use

[Total : 12]]

4 An absorption spectrum
s is a graph o f the absorrption of diffferent wavvelengths off light by a
photossynthetic pig
gment.
An acttion spectrum is a graaph of the rrate of phottosynthesis at differentt wavelengtths of light.
Fig. 4..1 shows thhe absorptio
on spectra of chlorophhyll a and chlorophyll
c b as well as
s an action
spectruum.

percentage
light
absorption

Fig. 4.1
1

(a
a) With re
eference to Fig. 4.1,
(i) ccompare the
e absorptioon spectra of chloroph
hyll a and ch
hlorophyll b..

D
Differences
s
1. For percentage of light a yll a peaks at 430nm with
absorption,, chlorophy w 80%
of light absorp
ption while chlorophy
yll b peaks at 450nm wwith a high
her 90%
light absorptio
on;
2. Chloorophyll a peaks at 6660nm with much high her light ab
bsorption of
o 70%
whille chloroph
hyll b peak
ks at 63564 40nm with 45% light absorptionn/ peak of
light absorptio
on for chlorrophyll a at
a 660nm.
S
Similarities
s
3. Botth shows higher light absorption
n of blue light (4005500nm) and
d red light
(600
0700nm) than
t other wavelengtths;
4. Botth peaks in blue light are higherr than red light;
5. Botth have littlle/least abs
sorption, between
b 500
0600nm / in green light;

RI 2013 Preliminaryy Examination 9648/02


9 [Turn over
1319
10 For
Examiners
Use

Or
Little light absorption for chlorophyll a between 450600nm and for
chlorophyll b between 500600nm ;

Mark awarded as follows:


Ref. wavelength
1 chlorophyll a peaks at 430nm and chlorophyll b peaks at 450nm ;
Or
2 chlorophyll a peaks at 660nm and chlorophyll b peaks at 635640nm ;
3 ref. linking 400500nm with blue light and ref. linking 600700nm with red light ;
4 both have little absorption, between 500590/600nm / in green light ;
A: little absorption, chlorophyll a 450600 and chlorophyll b 500600 ;

Ref. light absorption


5 both peaks in blue light are higher than peaks in red light ;
6 chlorophyll b higher than chlorophyll a in the blue light / chlorophyll a higher
than chlorophyll b in the red light / AW ; A converse
7 comparative figures on % of light absorption for pt 1(90% vs 80%) or 2(70% vs
45%) and 5(eg. for b 90% in blue light vs 45% in red light) and 6(e.g. peak of b is
10% higher than a for blue light)
Nb: only awarded if specific wavelength is quoted ; [2]

(ii) explain the shape of the action spectrum.

1. Peak(s) of action spectrum/ highest rate of photosynthesis, correspond to


absorption peak(s);
2. Light absorbed by different photosynthetic pigments is used for
photoactivation/photosynthesis thus the higher percentage of light
absorption leads to higher rate of photosynthesis in red and blue light ;
3. The region between 500 to 600nm is not an exact match between
absorption and action spectra
4. due to the role of carotenoids and other accessory pigments in light
absorption that pass the energy via resonance to the reaction centre
A: chlorophyll a (link to pt 3) [2]

(b) The light-dependent stage of photosynthesis takes place on the thylakoids of the chloroplast.
Fig. 4.2 represents a section through a thylakoid to show some of the components involved in
the light-dependent stage.

Fig. 4.2

RI 2013 Preliminary Examination 9648/02


1320 For
11 Examiners
Use

(i) Identify the structures labelled R and S.

R : Photosystem I/Photosystem 700 S : ATP synthase/ stalked particle [2]

(ii) Atrazine is a widely used herbicide (weed killer). It binds to a chloroplast protein involved
in electron transfer between photosystems.
Explain how atrazine works as an effective herbicide.

1. Electron cannot be passed down the electron transport chain* A: electron


carriers (ETC);
2. No reduced NADP/ NADPH + H+ produced (linked to 1);
3. Resulting in no / reduced, pumping of protons across the thylakoid
membrane from stroma to thylakoid lumen and thus no proton gradient /
proton motive force
4. Chemiosmosis does not occur thus no ATP is produced (linked to 3)
5. no carbon fixation / Calvin cycle / light independent stage thus no
glucose/triose phosphate(as glycerate phosphate cannot be converted to
glyceraldehyde-3-phosphate/triose phosphate) formed for respiration and
plant dies ; [3]

(c) In an investigation, mammalian liver cells were homogenised (broken up) and the resulting
homogenate centrifuged. Samples of the complete homogenate and samples with only
mitochondria were incubated with:

1 glucose
2 pyruvate
3 glucose and cyanide
4 pyruvate and cyanide

Cyanide inhibits oxidative phosphorylation.

After incubation, the presence or absence of carbon dioxide and lactate in each sample was
determined. The results are summarised in Table 4.1.

Sample of homogenate

Complete Only mitochondria


Carbon dioxide Lactate Carbon dioxide Lactate
1 glucose
X X
2 pyruvate
X
3 glucose and
X X X
cyanide
4 pyruvate and
X X X
cyanide

X = absent = present

Table 4.1

RI 2013 Preliminary Examination 9648/02 [Turn over


1321
12 For
Examiners
Use

(i) Explain why carbon dioxide is produced when mitochondria are incubated with pyruvate
but not when they are incubated with glucose.

1. Pyruvate can enter the mitochondrion but glucose cannot


2. Glucose needs to be converted to pyruvate via glycolysis that takes place
only in the cytoplasm/ cytosol;
3. Carbon dioxide is produced by decarboxylation* of pyruvate via link
reaction / of acetyl CoA in Krebs cycle in mitochondrion; [2]

(ii) With reference to Table 4.1, explain the difference in the results when the complete cell
homogenate is incubated with 2 and 4.

1. Carbon dioxide is produced only when incubated in 2;


2. oxidative phosphorylation cannot take place and reduced NAD is not
oxidised/ NAD is not regenerated AW ;
3. Krebs cycle /link reaction stops so no decarboxylation takes place ; [2]

[Total : 13]

5 Grove snails are known to have brightly coloured shells, with the alleles CP and CY coding for
pink and yellow coloured shells respectively. Shells of the grove snails have different banding
patterns as well, with allele BB conferring a shell with bands and allele BO conferring an
unbanded shell.

Banded snail Unbanded snail

A grove snail with a banded pink shell is crossed with another grove snail with an unbanded
yellow shell, and the resultant F1 generation all had unbanded pink shells. The F1 were test-
crossed and gave rise to 250 offspring with the following phenotypes:

banded pink shell 121


unbanded pink shell 14
banded yellow shell 11
unbanded yellow shell 104

(a) State the expected offspring phenotypic ratio of the test cross.
1 banded pink shell : 1 unbanded pink shell : 1 banded yellow shell : 1 unbanded yellow shell
.. [1]

RI 2013 Preliminary Examination 9648/02


1322 For
13 Examiners
Use

(b
b) (i) S our working clearly, calcculate the 2 value in the space beelow.
Showing yo

(121 62.5)2 5)2


(104 62.5 (14 62.5)2 (11 62.55)2
=
2
62.5 + 62.5 + 62.5 + 62.5

6 + 27.556 + 37.636 + 4
= 54.756 42.436

= 162.38
84

1 Mark for working


w 1 Mark for re
esult

[2]
(ii) Using the 2 table, explain the con
U nclusion dra e calculatedd 2 value in
awn from the n b(i).

1
1. df = 3 an
nd p < 0.001
2
2. At 5% significant
s level (if p< 0.001 men ntioned)/Since p < 0.055, differenc
ce between n
expecteed and obseerved resultts is signific
cant and is due to som
me other fac ctor and nott
by chance alone.
3
3. May be due to autoosomal linkaage [2]

c)
(c With th
he use of a genetic diagram, expla
ain the obse
erved resultts of the tesst cross.

RI 2013 Preliminaryy Examination 9648/02


9 [Turn over
1323
14 For
Examiners
Use

Parental unbanded banded yellow [1]


phenotype pink shell shell

P Y Y Y
C C C C [1]
Parental X
genotype
B O
B B B
B
B
B

P Y P Y Y
C C C C C
gametes [1]
B O O B B
B B B B B

Y Y P Y Y
Offspring C
P Y C C C C C
Y
C
C
genotype
[1]
O B O B B
B
B B
B B B B B B
B B

Offspring banded pink unbanded unbanded pink banded


phenotypes shell yellow shell shell yellow shell
52 48 14 11

[1]
parental recombinant

[5]

The same test cross was carried out another time and the resulting number of banded pink
snails and unbanded yellow snails were equal. The resulting number of unbanded pink snails
was also equal to that of the banded yellow snails. Through this test cross, the 2 gene loci are
determined to be 28 map units apart.

(d) State the observed phenotypic ratio of the offspring from this test cross.

18 banded pink shell : 18 unbanded yellow shell : 7 unbanded pink shell : 7 banded yellow
shell [1]

[Total : 11]

6 Homeostatic mechanisms maintain a constant environment in the body.


Fig. 6.1 shows changes in plasma glucose concentration that occurred in a person who went
without food for some time.

RI 2013 Preliminary Examination 9648/02


1324 For
15 Examiners
Use

Fig. 6.1
1

(a
a) Use evvidence from he change in plasma glucose conccentration.
m Fig. 6.1 to explain th

1. No food giiven thus de ecrease in pplasma/bloo od glucose concentrati on below th he norm


level/set point
p of 90m
mg/100ml is ;
2. detected byb the cells of islet off Langerhanns in pancre eas which sttimulates th he
(increased d) secretion of glucago on into the blood
b stream
m;
3. Glucagon binds to ce ell surface reeceptors of the effectorr e.g. liver ccell that stim
mulates the
conversion n of glycoge
en to glucosse/gluconeo ogenesis/co onversion off amino acid ds to
glucose inncrease in blood
b glucosse via negative feedback; R: if m muscle cell mentioned
m
4. detected byb cells in pancreas tthat stimula ate (increaseed) secretioon of insulin n into the
blood;
5. insulin is transported to the effecctor e.g. muuscle/liver ce
ell that resuults in increaase uptake
of glucose e/membrane e permeabillity to glucose that redu uces the bloood glucose e level via
negative feedback*
f (can be in ppt 3);
6. Antagonistic action off both horm mones resultt in fluctuations of bloodd glucose
concentrattion about the norm wh hich diminis
shed over timme, eventuaally reachin ng norm;
[4]

RI 2013 Preliminaryy Examination 9648/02


9 [Turn over
1325
16 For
Examiners
Use

Adrenaline is a signal molecule that stimulates similar cellular responses from liver cells and
adipose cells in a way similar to that of glucagon.

Step 1

Step 2

Fig. 6.2
(b) With reference to Fig. 6.2,
(i) describe the events occurring in step 1 after the binding of adrenaline molecule to the
adrenergic receptor.

1. Binding results in a conformational change of the receptor at the cytoplasmic


region/intracellular domain of the receptor ;
2. That allows binding to G protein that activates it by replacing GDP with GTP ;
3. subunit dissociates from the and subunits, moves along the plasma membrane
and binds with and activates the adenylyl cyclase ; [2]

(ii) explain the significance of step 2 that results in effective signal transduction.

1. An activated adenylyl cyclase catalyses the formation of many cAMP from many
ATP
2. Thus amplifying the signal as
3. Idea of a single ligand/ligand can produce a large cellular response (Any 2 of 1-3)
Or
4. cAMP is a second messenger* that is small and diffusible and

RI 2013 Preliminary Examination 9648/02


1326 For
17 Examiners
Use

5. can diffuse easily in the cell to activate the next relay protein (2M for 4&5)

[2]
(iii) describe the series of events that occur after the activation of protein kinase A that
results in a cellular response.

1. Activated protein kinase A passes through the nuclear pore into the nucleus ;
2. Phosphorylate transcription factor resulting in a conformation change/activation thus
allowing it to
3. bind to a specific regulatory DNA sequence e.g. enhancer
4. recruiting RNA polymerase or promote assembly of transcription initiation complex/
result in the transcription of the target gene ;

[2]

[Total : 10]

7 For a long time, the prevailing idea among evolutionary biologists is that geographical barriers
are a prerequisite for speciation. However, recently, more and more examples of an alternative
speciation process which does not require a geographical barrier were discovered but this
remains the exception rather than a norm.

(a) (i) What alternative speciation process are we referring to?

Sympatric speciation* [1]

One case study involved studying the palm genus Howea from Lord Howe Island, a tiny
volcanic island in the Tasman Sea. The two related palm species found on the island have a
different soil preference. The different types of soil are interspersed throughout the island and
pollen from the palms is wind-dispersed.

RI 2013 Preliminary Examination 9648/02 [Turn over


1327
18 For
Examiners
Use

Fig. 7.1

(ii) Explain how speciation could have taken place to give rise to the two sister species from
a common ancestor.

1. Ancestral palm growing on acidic volcanic soil developed a later/different


flowering time compared to palms growing on basic calcareous soil that flowered
earlier;
2. resulting in reproductive/physiological isolation*;
3. This prevented interbreeding and disrupted the gene flow* between the two
populations of palm;
4. As the different populations evolve independently from each other, they
accumulate different genetic mutations* over time;
5. With variation among the palms and exposure to different selection pressures,
natural selection will select for those best adapted to their environment,
6. who are more likely to survive, reproduce and passed on their alleles to the next
generation;
7. Over time, allele frequencies change can also be due to genetic drift [5]

(iii) Give an evidence why scientists can rule out geographical isolation mechanisms?

It is a tiny island;
R: no major barrier only
Pollen grains can be easily dispersed by wind;
Different soil where the 2 species grow on are interspersed throughout the island;
[1]

RI 2013 Preliminary Examination 9648/02


1328 For
19 Examiners
Use

(b) Howea belmoreana goes by the names, curly palm, kentia palm and Belmore sentry palm
while Howea forsteriana goes by the names thatch palm and kentia palm.

Explain why the scientific name is better than the common names.

1. It is unambiguous and precise with one unique name for one species;
2. unlike the common name Kentia palm which is used to refer to both species or
multiple names referring to one species (evidence);
Or
3. The binomial nomenclature is a systematic naming system that allows one to
categorise/group similar organisms together involving both the species name and
genus name (evidence);
4. while the common names do not reveal their position in the plant kingdom nor attempts
to categorise them; ORA

1 and 2 (link) or 3 and 4 (link). If 1 and 3 or 4 allow only 1 mark (unlink) [2]

[Total: 9]

Section B
Answer EITHER 8 OR 9.

Write your answers on the separate answer paper provided.


Your answers should be illustrated by large, clearly labeled diagrams, where appropriate.
Your answers must be in continuous prose, where appropriate.
Your answers must be set out in sections (a), (b) etc., as indicated in the question.

8 (a) Explain how meiosis can contribute to genetic variation in offspring. [6]

1. Meiosis produces haploid daughter cells/gametes that are genetically


different from the parent, contributing to genetic variation;
2. During prophase I*, crossing over* of corresponding segments of non-
sister* chromatids of homologous chromosomes/bivalent at chiasmata;
3. leads to exchange of genetic materials / linkage groups are broken to form
non-identical sister chromatids with new combinations of alleles on
chromosomes;
4. During metaphase I*, independent assortment of homologous
chromosomes* occurs when arrangement of one pair of homologues at
the metaphase plate is independent of the arrangement of the other pairs of
homologues;
5. New combination of maternal and paternal chromosomes;
6. Subsequently chromosomes of each homologous pair separate
independently during anaphase I*;
7. Random/independent arrangement of non-identical sister chromatids at the
metaphase plate during metaphase II and subsequent separation of non-
identical sister chromatids during anaphase II;
8. This results in gametes with new combinations of alleles that differ from
parental combination of alleles with 2n * possible combinations/types of
gametes where n is number of homologous pairs of chromosomes; (1m as
long as new combination of allele has been written)
RI 2013 Preliminary Examination 9648/02 [Turn over
1329
20 For
Examiners
Use

9. During fertilization*, random fusion* of gametes;


10. results in greater number of genotypic combinations of a zygote;

(b) Discuss how bacteria evolve through natural selection and the implication in [8]
antibiotic resistance.

how antibiotic resistance in bacteria arises:


1. Due to spontaneous mutation, there is existing variation* in the population
of bacteria with antibiotic resistant strains and non-resistant strains;
2. (idea of horizontal gene transfer) The allele for antibiotic resistance can
also be acquired through conjugation* / transduction* / transformation*
(1m = need to state the process + elaborate process) [2m max]
3. When exposed to antibiotic, antibiotic act as selection pressure* to select
against/kill those non-resistant bacteria;
4. while those bacteria with antibiotic resistance allele are selected for/best
adapted and survive;
5. and pass on the allele for antibiotic resistance to subsequent generations of
bacteria cells during binary fission thus most of the bacteria are resistant to
antibiotic (idea of vertical gene transfer);
6. allele frequency changes as a result of natural selection;
implication in antibiotic resistance:
7. Over many generations, bacteria cells evolve by natural selection and the
bacteria become resistant to the antibiotic rendering that antibiotic
ineffective;
8. Thus, other types / different combinations of antibiotics have to be used;
9. possibility of developing multiple antibiotic resistance;

(c) Describe the link between the frequency of sickle cell anaemia allele and the [6]
number of cases of malaria.

1. Frequency of sickle cell anaemia allele is highest in regions of endemic


malaria;
2. Individuals with two recessive HbS alleles/HbSHbS develop sickle cell
disease as sickle red blood cells exhibit poor oxygen transport;
3. Homozygous individuals with two normal HbA alleles/HbAHbA have normal
red blood cells;
4. but have a greater risk of dying from malaria in regions of endemic malaria;
5. Heterozygous individuals with one recessive HbS allele/HbAHbS produce
both normal and abnormal haemoglobin; (idea of carrier with sickle cell
trait)
Heterozygote protection:
6. Idea of heterozygote protection plus type of selection pressure:
Heterozygous condition hide recessive HbS allele that is less favourable
from natural selection which only acts on sickle cell anaemia phenotypes
7. thus maintaining the recessive allele in the population;
Heterozygote advantage:
8. Heterozygotes are less likely to suffer from malaria thus they have selective
advantage over homozygotes/heterozygote advantage;
9. Sickle shape of red blood cell results in lower oxygen carrying capacity thus
malaria parasite cannot survive;
Or idea that malaria parasite cannot survive well in cell with HbS
10. Heterozygotes are more likely to survive and reproduce to pass on the HbS
allele;

[Total: 20]

RI 2013 Preliminary Examination 9648/02


1330 For
21 Examiners
Use

9 (a) Explain how gene expression can be controlled at the translational level and [8]
describe the mechanisms present in prokaryotes and eukaryotes.

Prokaryotes Eukaryotes
mRNA half-life 1. Both involve regulation of mRNA half-life longer half-life
allows for translation of more proteins from mRNA;
2. mRNA rapidly degraded / 4. Half-life of mRNA
has short half life to allow for influenced by length of 3 poly-
fast changes in gene A tail;
expression; Or
3. Binding of anti-sense when the polyA tail shorten to
mRNA can target mRNA for a critical length, 5 cap is
degradation, further removed and mRNA is
decreasing its half-life; degraded
5. longer 3 poly-A tail = more
stable / longer half life;
6. siRNA bind to target mRNA
for degradation

Binding to small ribosomal 7. Translation can be blocked by preventing the binding of the
subunit small ribosomal subunit to the mRNA for assembly of the
translation initiation complex;

8. Small ribosomal subunit 10. Interaction between 5 cap


recognises and binds to Shine and 3 poly-A tail is required
Dalgarno region on mRNA for for translation initiation;
proper positioning of start 11. Translational repressor
codon; protein binds to 5 cap or 3
9. Translational repressor poly-A tail to prevent
protein or anti-sense mRNA interaction;
binds to / near Shine Dalgarno
region on mRNA to prevent
binding of small ribosomal
subunit;
Availability of initiation factors 12. Initiation factors required for proper positioning of the small
ribosomal subunit together with initiator tRNA on the mRNA,
and recruitment of the large ribosomal subunit / assembly of
translation initiation complex;

(b) Explain how the structure of specific transcription factors is linked to their roles [7]
in control of eukaryotic gene expression.

1. Specific transcription factors: activators & repressors ;


2. 2 regions/domains: DNA binding site & protein binding site;
3. DNA binding site complementary in shape and charge for specific DNA
sequences;
4. Activators binding to enhancers + Repressors binding to silencers;
5. Protein binding site complementary in shape and charge proteins that
they interact with;
a. Activator: recruit RNA polymerase / general transcription factors
help assembly of transcription initiation complex;
b. Repressors: sequester RNA polymerase / transcription factors /

RI 2013 Preliminary Examination 9648/02 [Turn over


1331
22 For
Examiners
Use

block binding of transcription factors prevent assembly of


transcription initiation complex at the promoter;
c. Activator: chromatin remodeling complexes & histone acetylase
to tighten/loosen coiling of DNA around histones or increase
accessibility of DNA;
d. Repressor: chromatin remodeling complexes & histone
deacetylase to tighten/loosen coiling of DNA around histones or
decrease accessibility of DNA;
6. Coordinated gene expression achieved combinations of activators &
repressors which bind to enhances & silencers which are associated
with groups of genes with related functions;

(c) With reference to changes in protein structure, explain how the level of protein [5]
activity can be regulated post-translationally.

1. Cleavage and/or covalent modification change in conformation


2. Give 1 suitable example - glycosylation, disulfide bond formation,
attachment of prosthetic groups etc. is required;
3. Form functional protein - newly synthesized proteins need to be
modified for proper assembly / functioning;

4. Eg: phosphorylation/dephosphorylation affect charge/conformational


change on protein may activate/inactivate (ORA) proteins
5. Eg. change folding to expose active sites of enzymes / allow for binding
to another protein;
6. Regulate - control cellular activity / influence biological activity;

7. Addition of recognition tags for targeting E.g. Proteins are linked to


ubiquitin;
8. addition of recognition site for targeting to proteasome;
9. Degrade proteins allows control of protein activities OR prevent
aberrant activities so that proteins will not stay too long in cytoplasm
and still be active;

[Total: 20]

---- End of Paper -----

RI 2013 Preliminary Examination 9648/02


1332

RAFFLES INSTITUTION
2013 Year 6 Preliminary Examination
Higher 2

CANDIDATE
NAME

CIVICS INDEX
GROUP 1 3 S0 3 NUMBER 1 3

BIOLOGY 9648/03
Paper 3 23 September 2013
2 hours
Additional materials: Answer Booklet/Paper

READ THESE INSTRUCTIONS FIRST


Write your index number, CT group and name on all the work you hand in.
Write in dark blue or black pen on both sides of the paper.
You may use a soft pencil for any diagrams, graphs or rough working.
Do not use staples, paper clips, highlighters, glue or correction fluid.

Answer all questions.

At the end of the examination, hand in your essay SEPARATELY.


The number of marks is given in brackets [ ] at the end of each question For Examiners Use
or part question.
1 /13

2 /12

3 /15

4 /12

5 /20

Total /72


This document consists of 16 printed pages.

Raffles Institution
Internal Examination
1333

For
2 Examiners
Use

Answer all questions.

1 (a) Restriction mapping is a technique used to identify restriction enzyme cleavage sites. In an
experiment, restriction mapping was used to investigate an unknown piece of DNA. This
piece of DNA was 3 000 base pairs (bp) long.

The scientists first took plasmids that had one restriction site for the enzyme KpnI and one
restriction site for the enzyme BamHI. The unknown piece of DNA was ligated into the
multiple cloning site of the plasmid.

Fig. 1.1 shows the recombinant plasmid.

Fig. 1.1

When the scientists digested one of the recombinant plasmids with KpnI, they obtained two
fragments. One fragment was measured as 1000 bp. The other fragment was described as
"very large".

i. What does this show about the unknown piece of DNA?

... [2]

ii. One of the fragments that the scientists obtained was described as "very large". What is
represented by this very large fragment?

... [1]

RI 2013 Preliminary Examination 9648/03 [Turn over



1334

For
3 Examiners
Use

iii. When the scientists digested another of the recombinant plasmids with BamHI, they
obtained three fragments. How many BamHI restriction sites are there in the unknown
piece of DNA?

... [1]

iv. When carrying out restriction mapping of the unknown DNA, scientists need to make sure
that the restriction enzyme they have used has completely digested the DNA.

Suggest how they could use results from agarose gel electrophoresis to show that this
DNA has not been completely digested.

... [1]


RI 2013 Preliminary Examination 9648/03 [Turn over

1335

For
4 Examiners
Use

(b) The urate transporter is a crucial protein found primarily in the kidney. Fig. 1.2 below
shows how genetic engineering can be used to clone the human urate transporter gene in
bacteria.

Human genomic DNA

Fig. 1.2

i. The human genomic DNA sample was digested with restriction enzyme. What is an
important assumption made in this procedure to clone the urate transporter gene?

... [1]


RI 2013 Preliminary Examination 9648/03 [Turn over

1336

For
5 Examiners
Use

ii. Comment on the choice of using the kidney for the extraction of the urate transporter gene
in the procedure shown in Fig. 1.2.

... [2]

iii. The human DNA fragments are inserted into the gene coding for resistance to tetracycline.
What effect will this have?

... [2]

iv. During replica plating, why is the velvet surface used?

... [1]

v. Use an arrow () to point to the bacterial colony containing the human DNA fragment. [1]

vi. State one disadvantage of using the bacterium as a host cell for expressing the urate
transporter gene.

... [1]

[Total: 13]

RI 2013 Preliminary Examination 9648/03 [Turn over



1337

For
6 Examiners
Use

2 Primordial dwarfism (PD) is a rare autosomal recessive disorder in which the lack of normal
growth is not due to a deficiency of growth hormone. In linkage analysis using RFLPs, scientists
can examine segments of DNA for a haplotype that is associated with PD. A haplotype refers to
a group of gene loci or genetic markers that is inherited together on one chromosome. An
individual inherits one haplotype from each parent.

Fig. 2.1 shows a pedigree of a family affected by PD followed by haplotype analysis with
polymorphic microsatellite markers on a segment of chromosome 3. Each microsatellite marker
has 7 different known alleles.

Fig. 2.1

(a) By examining the haplotypes in Fig. 2.1,

i. identify all the microsatellite marker alleles that are associated with primordial dwarfism.

... [2]

ii. identify which descendant is a recombinant.

... [1]

RI 2013 Preliminary Examination 9648/03 [Turn over



1338

For
7 Examiners
Use

(b)i. One of the conditions for a microsatellite marker to be useful for disease detection is that it
must be polymorphic.

Explain why the following 2 conditions are also necessary:

Condition 1: locus of the microsatellite marker allele must be tightly linked to the gene
locus of the disease allele;

...

..... [1]

Condition 2: restriction sites must flank the microsatellite marker allele.

...

..... [1]

ii. Suggest why the disease allele causing PD can only be mapped using linkage analysis.

...

..... [1]


RI 2013 Preliminary Examination 9648/03 [Turn over

1339

For
8 Examiners
Use

After more research, scientists discovered another RFLP marker that is also linked to PD.
Fig. 2.2 shows the two alleles of the RFLP marker with their restriction sites.

They carried out PCR to amplify the target DNA segment containing the RFLP allele. The
amplified DNA can then be subjected to PCR-RFLP analysis where restriction digestion
using BtgI is done, followed by gel electrophoresis. The products of restriction digestion
using BtgI can also be subjected to Southern blot analysis.
Fig. 2.3 shows the results of the PCR-RFLP analysis and the Southern blot analysis.
I I I I I

Fig. 2.2

Fig. 2.3

(c) Outline the principle of gel electrophoresis.

... [3]

(d) Using information from Fig. 2.2 and Fig. 2.3,

i. indicate on the RFLP allele 1 in Fig. 2.2 the position where the probe should hybridise; [1]

ii. explain the banding pattern of individual II4 as seen in the PCR-RFLP analysis in Fig. 2.3.

... [2]

[Total: 12]
RI 2013 Preliminary Examination 9648/03 [Turn over

1340

For
9 Examiners
Use

3 The Syngonanthus mucugensis species has significant economic value for the Brazilian
ornamental dry flower business. However, extreme exploitation has rendered this species
endangered.

Attempts at conventional propagation of this species by seeding have generated little success
and studies are ongoing to develop alternative techniques that can assure propagation on a
commercial scale. In a particular experiment, the potential of three different explants (stem, leaf
and root) to form callus was studied using two plant growth regulators (BAP and NAA). The data
from the experiment is shown in Table 3.1.

Table 3.1

(a) With reference to Table 3.1,

i. identify which explant seemed most suitable for use in micropropagation.

...... [1]

ii. explain your answer to (a) i.

...

...

... [4]

RI 2013 Preliminary Examination 9648/03 [Turn over



1341

For
10 Examiners
Use

(b) In a separate experiment, the ability of the explant to undergo organogenesis was studied
using the same plant growth regulators. Data obtained is shown in Table 3.2.

Table 3.2

With reference to Table 3.2, explain which plant growth regulator has an inhibitory effect
on shoot regeneration.

... [3]

To create transgenic maize which can produce the Bacillus thuringiensis toxin, the recombinant
plasmid shown in Fig. 3.1 was constructed. The recombinant plasmid also contains the gene for
phosphomannose isomerase (PMI), an enzyme which allows plants cells to utilise mannose as a
carbon source.

Fig. 3.1
RI 2013 Preliminary Examination 9648/03 [Turn over

1342

For
11 Examiners
Use

To produce transgenic maize using the plasmid shown in Fig. 3.1, the procedure shown in
Fig. 3.2 was carried out.

Fig. 3.2

(c) With reference to Fig. 3.2,

i. explain why the sterilised explant was treated with cellulase and pectinase before
electroporation.

... [2]

RI 2013 Preliminary Examination 9648/03 [Turn over



1343

For
12 Examiners
Use

ii. In the preparation of protoplasts, mannitol must be present. In the absence of mannitol,
no protoplast is formed even if the sterilised explant was treated with cellulase and
pectinase.

State what happens to the protoplasts if mannitol was not used.

..... [1]

(d) Antibiotic or herbicide resistance genes are commonly included in the plasmid used for
creating transgenic plants.

Explain why such genes were not needed in this case.

..

..

..

.[2]

(e) Explain the benefit of incorporating a Bt gene into a crop plant.

..

..

..

.[2]

[Total: 15]

RI 2013 Preliminary Examination 9648/03 [Turn over



1344

For
13 Examiners
Use

Planning question

4 The purple colour of beet root (Beta vulgaris) is due to the water soluble betacyanin pigment
molecules that are found in the vacuole of intact cells. Beet root pigment is used
commercially as a food dye and heat is typically used as a method to extract the pigment
from intact beet root cells.

A colorimeter can be used to measure the absorbance of light of 550nm by betacyanin


solution. The concentration of betacyanin is proportional to its absorbance. The colorimeter
is shown in Fig. 4.1 below.

Cuvette holding
betacyanin solution
Cover
Filter that allows light of
Light sensor 550nm to pass through

Bulb

Colorimeter

% absorbance
shown by meter
Fig. 4.1

Using the information and your own knowledge, design an experiment to investigate the
effect of temperature on membrane permeability in beetroot.

Your plan must be based on the assumption that you have been provided with the following
which you must use.

Beet root
Cork borer
Scalpel
Thermometer
Source of boiling water and tap water
Distilled water
Large 500 cm3 beaker
Colorimeter
2cm3 cuvettes

You may also suggest other appropriate equipment to use.

RI 2013 Preliminary Examination 9648/03 [Turn over



1345

For
14 Examiners
Use

Your plan should:


have a clear and helpful structure such that the method you use is able to be
repeated by anyone reading it,
be illustrated by labeled diagrams, if necessary,
identify independent and dependent variables,
describe the method with scientific reasoning used to decide the method so that the
results are as accurate and reliable as possible,
include layout of results table(s) and graph(s) with clear headings and labels,
use the correct technical and scientific terms,
include reference to safety measures to minimize any risks associated with the
proposed experiment.

[Total: 12]

....

....

....

....

....

....

....

....

....

....

....

....

....

....

....

....

....

....

....

RI 2013 Preliminary Examination 9648/03 [Turn over



1346

For
15 Examiners
Use

....

....

....

....

....

....

....

....

....

....

....

....

....

....

....

....

....

....

....

....

....

....

....

....

....

....

RI 2013 Preliminary Examination 9648/03 [Turn over



1347

For
16 Examiners
Use

Free-response question

Write your answer to this question on the separate answer paper provided.

Your answer:
should be illustrated by large, clearly labeled diagrams, where appropriate;
must be in continuous prose, where appropriate;
must be set out in sections (a), (b) etc, as indicated in the question.

5 (a) Explain the normal functions of stem cells in a living organism. [6]

(b) Discuss the social and ethical concerns for the use of gene therapy. [6]

(c) Describe cystic fibrosis and how it can be treated using a non-viral gene delivery system. [8]

[Total: 20]

--- End of Paper ---

RI 2013 Preliminary Examination 9648/03 [Turn over



1348

Answe
er all questiions.

1 (a) Resstriction mappping is a te


echnique ussed to identtify restrictio
on enzyme cleavage ssites. In an
expe eriment, resstriction ma
apping was used to innvestigate a an unknownn piece of D
DNA. This
piecce of DNA wwas 3 000 ba ase pairs (b
bp) long.

The scientists ffirst took pla


asmids thatt had one re
estriction site for the enzyme
e Kpn
nI and one
restriction site for the enzzyme BamH HI. The unknown piecce of DNA was ligated into the
multtiple cloning
g site of the plasmid.

Fig. 1.1 shows the recomb


binant plasm
mid.

Fig. 1.1

Whe en the scien


ntists digestted one of tthe recombiinant plasm
mids with Kp
pnI, they obttained two
fragments. One e fragment wwas measured as 1 00 00 bp. The o other fragment was desscribed as
"verry large".

i. Wha
at does this show abou
ut the unkno
own piece off DNA? [2]

1. H
Has KpnI resstriction site;
2. K nique/ cuts unknown piece only on
KpnI restriction site is un nce;
3. 1000 bp fromm KpnI site ofo plasmid; (A: restrictio
on site is fo
ound at the 1/3 position
n along the
le
ength of unkknown DNA)

ii. Onee of the frag


gments tha at the scienttists obtaine
ed was desscribed as "very large". What is
reprresented by this very la
arge fragment? [1]

mid and rest of unknown
(Most of) plasm n DNA;


1349

For
2 Examiners
Use

iii. Whe en the scien


ntists digested anotherr of the reccombinant p plasmids witth BamHI, they
t obtaine
ed
three
e fragmentss. How man ny BamHI re
estriction sitees are there
e in the unkn
nown piece of DNA? [1
1]

2;

iv. Wheen carrying out restriction mappingg of the unkknown DNA A, scientistss need to m
make sure th
hat
the restriction
r e
enzyme theyy have used
d has complletely digestted the DNA A.

Suggest how thhey could usse results frrom agarosee gel electro o show that this DNA has
ophoresis to
not been comp pletely digessted? [1]

1. R
Restriction ffragments a
add up to mo ore than tota
al length of original DN
NA;
2. P
Presence off undigested d 3000bp fraagment (+ some
s other smaller frag
gments)

(b) The urate transsporter is a crucial pro n the kidneyy. Fig. 1.2 below show
otein found primarily in ws
how
w genetic engineering ca an be used to clone the
e human ura ate transporter gene inn bacteria.

Human geno
omic DNA

Fig. 1.2

RI 2013 Preliminary Examination 96


648/03 [Turn over

1350

For
3 Examiners
Use

i. The human genomic DNA sample was digested with restriction enzyme. What is an important
assumption made in this procedure to clone the urate transporter gene? [1]

assumes restriction enzyme doesnt cut within the urate transporter gene rendering it inactive;

ii. Comment on making kidney the choice organ to extract the urate transporter gene in the
procedure above. [2]

1. not necessary; (A: suggested use of other tissue)

2. urate transporter gene is present in all cells so can use any human tissue as starting material;
Or

3. procedure involves creating a genomic library so do not need to use a tissue with high levels
of urate transporter mRNA;

iii. The human DNA fragments are inserted into the gene coding for resistance to tetracycline.
What effect will this have? [2]

1. gene coding for resistance to tetracycline no longer functional / bacteria not resistant to
tetracycline/wtte;

2. insertional inactivation* has disrupted reading frame

iv. During replica plating, why is the velvet surface used? [1]

So that bacteria stick to it / transfer of bacteria without damage

v. Use an arrow () to point to the bacterial colony containing the human DNA fragment. [1]

Colony present on ampicillin plate but not on tetracycline plate

vi. State one disadvantage in using the bacterium as a host cell for expressing the urate
transporter gene. [1]

1. lack post-transcriptional modifications (source DNA / Gene of interest used cannot contain
introns)

2. limited post-translational modifications (eukaryotic proteins may not be processed correctly)

[Total: 13]

RI 2013 Preliminary Examination 9648/03 [Turn over



1351

For
4 Examiners
Use

2 Primordiaal dwarfism
m (PD) is a rrare autosommal recessivve disorder in which the lack of no
ormal growth is
not due to a deficiency of grow wth hormone e. In linkage
e analysis u
using RFLPPs, scientistss can exam
mine
segmentts of DNA fo or a haplotyype that is aassociated with PD. A haplotype refers to a group of ge ene
loci or ge
enetic marke
ers that is in gether from one parent on one chro
nherited tog omosome.

Fig. 2.1 shows a pe edigree of a family affeccted by PD followed byy haplotype analysis with polymorp
phic
microsattellite marke
ers on a se egment of chromosom me 3 for individuals III1 to II5 aand III1. Ea
ach
microsattellite marke
er has 7 diffe
erent knownn alleles.

Fig. 2.1

(a) By examining the haplotypes in Fig. 2.1,

i. ide
entify all the microsatellite marker alleles
a that a
are associatted with prim
mordial dwa
arfism. [2]

1. e 2, 3D6 alle
3D5 allele ele 4, 3D7 allele 2, 3D8
8 allele 2;

(1 mark
m for 2 correctly
c ide
entified allele
es)

ii. ide
entify which descendantt is a recom
mbinant. [1]

II3

RI 2013 Preliminary Examination 96


648/03 [Turn over

1352

For
5 Examiners
Use

(b)i. One of the con


nditions for a microsate
ellite markerr to be usefu
ul for diseasse detection
n is that it m
must
be polymorphic.

Exp
plain why th
he following 2 condition
ns are also n
necessary:

Condition 1: lo
oci of the miicrosatellite marker alle ed to the gene locus of the
ele must be tightly linke
disease allele;; [1]

1. So that the
e microsateellite markerr allele and tthe disease allele always(higher ch
hance) of co
o-
ng/(being inherited toge
segregatin ether);
ORR
2. To minimise the chance of cros ssing overr* at the reg gion betwee en the dise
ease allele and
a
marker allele so thaat the linkag ge of micro osatellite marker allele
e to disease allele is not
disrupted;

Condition 2: re
estriction sittes must flan
nk the micro
osatellite ma
arker allele.. [1]

1. Restrictionn enzyme shhould not cu


ut within the microsatelllite marker allele
a will result in the
as it w
generation n of the sam
me sized RFFLP fragme ents disallow
wing us to distinguish
d between the 7
different alleles;

ORR
2. Ensures th hat excised RFLP fragm ments will re ariable lengtths of each different allele
eflect the va
(due to the
eir difference
e in numberr of tandem repeats);

ii. Sug
ggest why tthe disease allele causing PD can only be ma
apped using linkage ana
alysis. [1]

1. Nucleotidee sequence of the disea ase allele iss not known;


2. Mutation in the diseasse allele in tthe genome e is not knowwn;
3. The mutattion in the disease allele did not inttroduce or rremove a re
estriction site
e;
R: There are n
no restriction
n sites in the
e disease aallele unlikkely.

Afteer more ressearch, scie


entists disco
overed another RFLP m marker that is also linkeed to PD. F Fig.
2.2
2 shows the two alleles of the RFLP P marker with their resttriction sitess.
The ey carried out PCR to o amplify thhe target D
DNA segment containin ng the RFL
LP allele. A
After
which, restricttion digestio
on using B BtgI is done
e followed b by gel elecctrophoresiss to obtain the
ressults shown in the PCRR-RFLP anallysis in Fig. 2.3.

Fig. 2.2

Fig. 2.3

RI 2013 Preliminary Examination 96


648/03 [Turn over

1353

For
6 Examiners
Use

(c) Outline the principle of gel electrophoresis. [3]

1. Negatively-charged DNA migrate out of well towards the positive electrode/anode when
subjected to an electric field / current;
2. Meshwork of polymer fibers that makes up agarose gel impedes movement of DNA
fragments;
3. thus longer DNA fragments migrate slower towards the positive electrode than shorter DNA
fragments (or vice versa)

(d) Using information from Fig. 2.2 and Fig. 2.3,

i. indicate on the RFLP allele 1 in Fig. 2.2 the position where the probe should hybridised to; [1]

ii. explain the banding pattern of individual II4 as seen in the PCR-RFLP analysis in Fig. 2.3. [2]

1. Using restriction enzyme BtgI for restriction digestion, RFLP allele 1 will be cut into two
fragments of sizes 200bp and 175bp;
2. No restriction digestion within RFLP allele 2 thus resulting in (one fragment) 375bp band;
3. 3 bands shown for Individual II4 as he is heterozygous;

[Total: 12]

RI 2013 Preliminary Examination 9648/03 [Turn over



1354

For
7 Examiners
Use

3 The Syngonanthus mucugensis species has significant economic value for the Brazilian
ornamental dry flower business. However, extreme exploitation has rendered this species
endangered.

Attempts at conventional propagation of this species by seeding have generated little success
and studies are ongoing to develop alternative techniques that can assure propagation on a
commercial scale. In a particular experiment, the potential of three different explants (stem, leaf
and root) to form callus was studied using two plant growth regulators (BAP and NAA). The
data from the experiment are shown in Table 3.1 below.

Table 3.1

With reference to Table 3.1,

(a) i. identify which explant seemed most suitable for use in micropropagation. [1]

leaf

ii. explain your answer to (a) i. [4]

1. Root does not give rise to callus in the whole experiment.


2. Stem gives rise to 8.62% while leaves give rise to 19.73% (or 12.35%) callus (no
need to mention concentration and type of growth regulator)
3. Leaves give rise to more callus compared to stem with data (either difference i.e.
11.11% or a factor of e.g. about 2.3x that of)
4. More callus giving rise to more plantlets

iii. In a separate experiment, the ability of the explant to undergo organogenesis was studied
using the same plant growth regulators. Data obtained is shown in Table 3.2.

RI 2013 Preliminary Examination 9648/03 [Turn over



1355

For
8 Examiners
Use

Table 3.2

With reference to Table 3.2, explain which plant growth regulator has an inhibitory effect
on shoot regeneration. [3]

1. NAA has inhibitory effect on shoot regeneration.


2. In the presence of BAP and absence of NAA, there is shoot regeneration.
3. however in the presence of both BAP and NAA, there is no shoot regeneration.
4. correct and relevant use of data (mention concentration of BAP and NAA and
% shoot for both points 1 and 2)

To create transgenic maize which can produce the Bacillus thuringiensis toxin, the recombinant
plasmid shown in Fig. 3.1 was constructed. The recombinant plasmid also contains the gene
for phosphomannose isomerase (PMI), an enzyme which allows plants cells to utilize mannose
as a carbon source.

Fig. 3.1
To produce transgenic maize using the plasmid shown in Fig. 3.1, the procedure shown in Fig.
3.2 was carried out.

RI 2013 Preliminary Examination 9648/03 [Turn over



1356

For
9 Examiners
Use

Fig. 3.2

With reference to Fig. 3.2,

(b)i. explain why the sterilized explant was treated with cellulase and pectinase before
electroporation. [2]

1. To remove cell wall to form protoplasts;


2. Protoplasts which are lacking in cell wall can then take up plasmid containing Bt
gene during electroporation / cell wall is a barrier to uptake of Bt gene during
electroporation;

ii. In the preparation of protoplasts, mannitol must be present. In the absence of mannitol,
no protoplasts are formed even if the sterilised explant was treated with cellulase and
pectinase.

State what happens to the protoplasts if mannitol was not used. [1]

The protoplasts lysed;

RI 2013 Preliminary Examination 9648/03 [Turn over



1357

For
10 Examiners
Use

(c) Anntibiotic or herbicide


h re
esistance ge
enes are co
ommonly included in th
he plasmid used for
cre
eating transgenic plantss.

Exxplain why ssuch genes w


were not ne
eeded in thiss case. [2]

a. The PM
MI gene serrves as the selectable marker* instead;
b. In med
dium with only manno ose as sole
e carbon ssource, tran
nsformed pllant cells
would have the PMI gene and survive O OR non-transformed plaant cells do not have
the PM
MI gene and
d will not surrvive;

(d) Exxplain the be


enefit of inco d as shown in Fig.3.1. [2]
orporating a Bt gene in the plasmid

a. Increase crop yielld;


b. Bt toxiin is synthe
esized, making crops re esistant to iinsects thatt consume it/plant is
pest re
esistant or kkill the pest tthat consum
me it;

[T
Total: 15]

Pllanning question

4 Thhe purple ccolour of b Beta vulgarris) is due to the watter soluble betacyanin
beet root (B n
pigment mole ecules that are found in
i the vacuo ole of intacct cells. Bee
et root pigm
ment is used
d
coommerciallyy as a food dye and heeat is typica
ally used ass a method to extract tthe pigmentt
fro
om intact be
eet root cells.

A colorimeter can be ussed to mea asure the absorbance of light of 550 5 nm by betacyanin n
soolution. The
e concentrattion of betaccyanin is prroportional tto its absorb
bance. The colorimeterr
is shown in Fig.
F 4.1 belo ow.

Cuveette holding
betacyan
nin solution
Cover
Filter that allows light of 550nm
m
L
Light sensorr to passs through thhrough

Bulb

Note:
Collorimeter 550nm is the
t waveleng gth of light
that the be
etacyanin abssorbs the
most. The 550nm light will pass
the betacyyanin solution
n in the
% abso
orbance cuvette annd then deteccted by a
shown by meter sensor. Thhe reading fro
om the
Fig
g. 4.1 meter is % absorbance e.

RI 2013 Preliminary Examination 96


648/03 [Turn over

1358

For
11 Examiners
Use

Using the information and your own knowledge, design an experiment to investigate the
effect of tempera sampleture on membrane permeability in beetroot.

Your plan must be based on the assumption that you have been provided with the following
which you must use.

Beet root
Cork borer gives a cylinder (e.g 1 cm diameter) of beet root tissue
Scalpel for cutting.
Thermometer for checking the temperature of water bath
Source of boiling water and tap water to create water bath of a certain temperature and
to add more for maintenance of temperature.
Distilled water for rinsing and immersing the beet root
Large 500 cm3 beaker used as a water bath
Colorimeter measures 550nm light absorbed by betacyanin pigment (absorbance) which
is a reflection of the concentration of betacyanin that has leaked out of beet root
2 cm3 cuvettes a small tube that holds solution whose absorbance is to be measured.
Cuvette will be placed into colorimeter. 2cm3 is the maximum volume the cuvette can hold.

You may also suggest other appropriate equipment to use.

Your plan should:


have a clear and helpful structure such that the method you use is able to be
repeated by anyone reading it,
be illustrated by labeled diagrams, if necessary,
identify independent and dependent variables,
describe the method with scientific reasoning used to decide the method so that the
results are as accurate and reliable as possible,
include layout of results table(s) and graph(s) with clear headings and labels,
use the correct technical and scientific terms,
include reference to safety measures to minimize any risks associated with the
proposed experiment.

[Total: 12]
1. Aim : To investigate the effect of temperature on membrane permeability in beetroot

2. Theory [T1 to T5 - max 3]:

(Main theory)
[T1] Betacyanin is found in beetroot cells is prevented from leaking out of cells by
membranes (vacuole & plasma membrane) which are made up of phospholipid
bilayer embedded with proteins
[T2] increasing temperature increases kinetic energy of phospholipids and membrane
becomes more fluid or
increasing temperature increases more intramolecular vibrations and membrane
proteins are denatured

(Measurable quantity)
[T3, T4 and T5 - max 2]
[T3] the degree of permeability of the membrane will be reflected by the amount of
betacyanin pigment leaking out of cells
RI 2013 Preliminary Examination 9648/03 [Turn over

1359

For
12 Examiners
Use

[T4] The concentration of betacyanin is proportional to its absorbance measured by


colorimeter.

(Predicted trend)
[T5] Increasing temperature increases the absorbance, which means permeability of
the membrane is increased.

[DV] Dependent variable: % absorbance which indicates permeability of membrane


[1]
[IV] Independent variable: temperature 30, 40, 50, 60, 70C [1]

3. Procedure (PAN CR)


a) [P] Pilot test*
Conduct a pilot experiment to determine suitability of apparatus, suitability of range
of independent variable (e.g. temperature range), optimum conditions, amount of
materials used (number of pieces of beetroot, volume of distilled water) [1]

b) Annotated diagram

Test tube with 5 discs (1cm


diameter) of 2 mm thick beet
roots and 10 cm3 of distilled
water
Thermometer to monitor
temperature of water bath and
add boiling or tap water to
maintain temperature

500 cm3 beaker for


water bath at 20C
c) Numbered steps

Constant variables (What, How and Why) [max 3]

a. (what) Surface area of beetroot (how) Use similar sized beetroot sizes using
cork borer and scalpel (2mm height, 1cm diameter discs) as (why) increasing
surface area increases surface for pigment to leak out, thus affecting rate of
diffusion of pigment.
b. (what) Volume of distilled water in test tube (how) Use 10 cm3 of distilled water
as (why) different volume of water will affect the concentration of leaked
pigment.
c. (what) Duration of experiment (how) 20 min as (why) the greater the duration,
the longer the time where pigment may leak out.
d. (what) Wavelengths of light used in the colorimeter when measuring absorbance
(how) Use 550 nm as (why) different wavelengths may be absorbed differently
by the pigment, affecting accuracy of results.
[CV1] what and how variable 1;
[CV2] what and how variable 2;
[CVW] why for variables 1 and 2

RI 2013 Preliminary Examination 9648/03 [Turn over



1360

For
13 Examiners
Use

1. Using a cork borer* (diameter 1cm), cut out cylinders of beetroot*. Cut 75 pieces
of 2 mm thick discs from the cylinder using scalpel*.

2. [B] Rinse the discs in running water until the water is colourless. (or idea of baseline
for calibration of colorimeter) [1]

3. Place 5 pieces of washed beetroot discs in test tube. Prepare 2 more test tubes with
beetroot discs to serve as replicates, (why) to check for anomalous results.

4. Add 10 cm3 of distilled water* into 3 separate test tubes and place them in the 500
cm3 beaker* with 30C water. Place the 3 test tubes with beetroot into the same
water bath. Add boiling or tap water*, where necessary, to attain and maintain the
temperature. Use thermometer * to monitor temperature.
5. [E] Allow 2 min of equilibration time for the (why) temperature in the test tube to
reach that of the water bath at 30C. [1]
6. Add the 5 beetroot discs to the 10 cm3 distilled water.
7. Incubate the 3 tubes in the 30C water bath for 20 min.
8. Decant the extract from tube and pour into liquid into a cuvette*.
9. Fill a second cuvette with distilled water.
10. Place a filter that results in light of 550nm into the colorimeter* and use the distilled
water to adjust the reading to zero absorbance. (do not mark for this)
11. Measure the % absorbance of solution using colorimeter.
12. Repeat steps 1-10 for 40oC, 50oC, 60oC, 70oC.
13. [RR] Repeat the entire experiment twice to check for reproducibility. [1]

d) [C] Control [1]


Place 5 washed beetroot disc into 10 cm3 of distilled water into a test tube place on
test-tube rack at room temperature. All other conditions remain the same. This
shows that raising temperature will result in increased permeability of membrane.

4. Data Recording and Processing

Record and process data as follows:

[T] Table showing intensity of colour of extract [1]


Temperature / Absorbance/%
O
C Replicate 1 Replicate 2 Replicate 3 Average
30
40
50
60
70

RI 2013 Preliminary Examination 9648/03 [Turn over



1361

For
14 Examiners
Use

[G] Graph showing intensity of colour of extract obtained from incubation at different
temperature [1]

Absorbance/ %

temperature / oC

Note:
trend accepted as long as there is increasing absorbance with increasing temp

5. [RP] Risks & Precautions [1]


a. When using scalpel, cut the beetroot away from the hand to avoid getting cut.
b. Use insulating gloves when handling hot water to prevent scalding
c. Dry your hands before switching the colorimeter on or off to avoid being
electrocuted.

Free-response question

Write your answer to this question on the separate answer paper provided.

Your answer:
should be illustrated by large, clearly labeled diagrams, where appropriate;
must be in continuous prose, where appropriate;
must be set out in sections (a), (b) etc, as indicated in the question.

5 (a) Explain the normal functions of stem cells in a living organism. [6]

1. Stem cells are a group of undifferentiated* and unspecialized* cells ;


2. Ability to differentiate* into specialized cells when appropriate molecular signals are
present
3. Zygotic stem cells are totipotent* and can differentiate into all cell types (including
extra-embryonic tissue) / make up the whole organism ;
4. Embryonic stem cells and pluripotent* which can differentiate into all cell types
except the extra-embryonic tissue/placenta ;
5. adult stem cells are multipotent* which can differentiate into limited range of related
cell types ;
6. any example of adult stem cells i.e. haematopoietic stem cells which differentiate
into red and white blood cells or neural stem cells from our central nervous system
can proliferate and differentiate into both neurons and supporting cells to replace
those that died etc ;
7. Stem cells are capable of self renewal* to ensure a constant pool of stem cells with
the same development potential ;
8. Stem cells divide by mitosis* to produce genetically identical daughter cells ;
9. function: for growth and development ;
10. function: to regenerate or replace cells lost due to cell death and injury ;

RI 2013 Preliminary Examination 9648/03 [Turn over



1362

For
15 Examiners
Use

(b) Discuss the social and ethical concerns for the use of gene therapy. [6]
1. Gene therapy is a method of treating genetic diseases which involves the
introduction of a normal functional allele into cells of an affected individual where it
will express the normal protein ;
2. Gene therapy which involves an alteration to the genes of somatic cells affects only
that individual, but an alteration to the genes of germline cells affect offspring and
successive generations ;
3. Social issue: injustice and unequal resource allocation as gene therapy is very
expensive. There are concerns that this treatment is being made available only to
those who can afford it ;
4. Ethical issue: There is concern that many gene therapy candidates are children who
are too young to understand the ramifications of gene therapy treatment and can
become unwitting participants ;
5. Ethical issue: Germ-line gene therapy involves research on early embryos which
may be considered by some as living and death of embryos in the process of gene
therapy (e.g. extraction of inner cell mass) is unethical ;
6. Ethical/social issue: Abuse of gene therapy for genetic enhancement could
exacerbate problems of social discrimination, social inequality and a lowered
tolerance for human diversity ;

(c) Describe cystic fibrosis and how it can be treated using non-viral gene delivery system. [8]
Cystic fibrosis description (max 4)
1. Cystic fibrosis (CF) is a autosomal recessive disease;
2. caused by mutation of cystic fibrosis transmembrane conductance regulator*
(CFTR/ CFTCR) gene;
3. The mutation of CFTR gene in chromosome 7 is a deletion of 3 bases resulting in
the loss of phenylalanine in the polypeptide;
4. A normal CFTR gene will code for a protein that function as a gated channel protein
to facilitate the transport of chloride ions;
5. as a result of this loss of function* mutation, channels no longer able to transport
chloride ions out of the epithelial cell (as in the lung);
6. causing thick mucus in lungs, resulting in blockage/congestion of airway in the lung
that reduces gaseous exchange leading to breathing difficulties
Or
pancreatic duct is choked by thick mucus preventing release of enzymes resulting in
indigestion ;
7. thick mucus provides suitable environment for bacteria growth which makes patient
susceptible to chronic lung infection resulting in death (lowering of life expectancy,
not living beyond 30years old);
Treatment (max 4)
8. to treat CF, a normal functioning allele* of CFTR is isolated and inserted into a
plasmid vector through recombinant techniques;
9. this DNA construct is then packaged in the aqueous core of the microscopic lipid
envelopes/phospholipid bilayer sphere forming liposomes* ;
10. delivered to epithelial cell in the lung by aerosol spray (nasal spray);
11. the liposome fuses with the plasma membrane of the epithelial cell lining the lung
and releases the DNA into the cell (this process is called lipofection);
12. the plasmid vector enters nucleus and the normal CFTR allele is transcribed and
translated;
13. to produce normal CFTR protein which can be incorporated into plasma membrane
to facilitate the transport of chloride;
[Total: 20]

RI 2013 Preliminary Examination 9648/03 [Turn over

1363

RIVER VALLEY HIGH SCHOOL


YEAR 6 PRELIMINARY EXAMINATION

CANDIDATE
NAME

CENTRE INDEX CLASS


NUMBER NUMBER

H2 BIOLOGY 9648/01
Paper 1 Multiple Choice 27 September 2013
1 hour 15 minutes
Additional Materials: Multiple Choice Answer Sheet

READ THESE INSTRUCTIONS FIRST

Write in soft pencil.


Do not use staples, paper clips, highlighters, glue or correction fluid.
Write your name, centre number and index number on the Answer Sheet in the spaces
provided unless this has been done for you.

There are forty questions in this paper. Answer all questions. For each question, there
are four possible answers, A, B, C and D.
Choose the one you consider correct and record your choice in soft pencil on the
separate Answer Sheet.

Read the instructions on the Answer Sheet carefully.

Each correct answer will score one mark. A mark will not be deducted for a wrong
answer.
Any rough working should be done on the Question Paper.
Calculators may be used.

_____________________________________________________________________
This Question Paper consists of 24 printed pages.

River Valley High School Pg 1 of 24 Year 6 H2 Biology 9648 Paper 1


Preliminary Examination 2013
1364

For each question, there are four possible answers, A, B, C and D.


Choose the one you consider correct and record your choice in soft pencil on the separate
Answer Sheet.

1. A ribosome consists of a large and a small subunit, each subunit containing ribosomal
RNA (rRNA) complexed with protein.
Which sequence of events concerning ribosomes is correct?

A Within the nucleolus rRNA and protein are synthesised and subunits formed. They
become membrane bound as they are exported through the nuclear envelope to
the cytoplasm and RER.

B rRNA and protein are synthesised in the Golgi body and are transported to the
nucleolus for subunit formation.

C rRNA is synthesised in the SER and proteins are synthesised by the RER. Subunit
formation occurs within the cytoplasm for free ribosomes and within RER for
attached ribosomes.

D rRNA synthesised within the nucleus is complexed with protein that has been
imported from cytoplasm. The subunits formed are exported to the cytoplasm via
the nuclear pores.

2. The diagram below shows the structure of an amino acid.

Which of the following are true about this amino acid?


1. Non polar R group
2. Hydrophobic R group
3. Insoluble in water
4. Soluble in water

A 1 and 2 only

B 1, 2 and 3

C 1, 2 and 4

D All of the above

River Valley High School Pg 2 of 24 Year 6 H2 Biology 9648 Paper 1


Preliminary Examination 2013
1365

3. The diagrams show short sections of some common polysaccharides and modified
polysaccharides.

The polysaccharides can be described as below.


Polysaccharide F is composed of -glucose monomers with 1,4 glycosidic bonds
Polysaccharide G is composed of -glucose monomers with 1,4 and 1,6 glycosidic
bonds
Polysaccharide H is composed of N-acetylglucosamine and N-acetylmuramic acid
monomers with -1,4 glycosidic bonds
Polysaccharide J is composed of -glucose monomers with 1,4 glycosidic bonds
Polysaccharide K is composed of N-acetylglucosamine monomers with -1,4 glycosidic
bonds
Which shows the correct pairings of polysaccharide descriptions and diagrams?

Polysaccharide Polysaccharide Polysaccharide Polysaccharide Polysaccharide


F G H J K

A 2 4 5 3 1

B 2 5 4 1 3

C 3 4 1 2 5

D 3 5 4 1 2

River Valley High School Pg 3 of 24 Year 6 H2 Biology 9648 Paper 1


Preliminary Examination 2013
1366

4. Ca
atechol is oxxidised to benzoquinon
b ne, as show
wn in the eq
quation, res ulting in darkening of
pe
eeled fruits.

Ca
atechol oxid dase is an enzyme wh hich is inhibited by pa
arahydroxybbenzoic acid (PHBA)
wh
hich is struccturally similar to catecchol.
It is also inhib
bited by phe
enylthiourea a (PTU) wh hich binds to
o a copper atom in the
e enzyme.
Thhe copper atom
a is esse
ential for the
e oxidative activity.
a
Th
he graph sh
hows the ratte of the rea
action with and
a without inhibitors.

Which is the correct


c key to the curve
es?

w
with PHBA
A with
h PTU uninhibitted

River Valle
ey High Schoool Pg 4 of 24 Year 6 H2 Biology 9648 Paper 1
Preliminaryy Examination 2013
1367

5. Meesselson and Stahl fo ound that inn dividing cells,


c DNA is copied bby semi-connservative
replication. At
A the time of
o their discoovery it was hat DNA miight be copied in one
s thought th
of three wayss: semi-cons
servative, cconservativee and dispersive.
Thhe bacteria were grown ogen, 15N, uuntil all the DNA was
n in culture containing heavy nitro
labbelled. Thesse bacteria were then transferred to a medium containinng only lightt nitrogen,
14
N and allow cate for two generations.
wed to replic
Which row shows the correct
c prop
portion of each
e type of
o DNA if D
DNA replication was
co
onservative?
?

All 14N Half 14N,


N half 15N All 15N

A 50% 50%
5 0%

B 0% 10
00% 0%

C 50% 0%
0 50%

D 75% 0%
0 25%

6. Thhe diagram
m shows the synthesiss of a polynucleotide
e. Moleculee W is a nucleotide
n
trip
phosphate.

Which statem
ments are co
orrect?
1. The base in W could be the puriine, adenine
e
2. The base in Y is the purine, gua
anine
3. The base in X is the pyrimidine, cytosine
midine, urac
4. The base in X could be the pyrim cil

A 1 and 3 only

B 2 and 3 only

C 2 and 4 only

D 1, 2, 3 and
a 4
River Valle
ey High Schoool Pg 5 of 24 Year 6 H2 Biology 9648 Paper 1
Preliminaryy Examination 2013
1368

7. Which statements about the genetic code are correct?


1. There is only one codon for the amino acid methionine.
2. Codons act as stop and start signals during transcription and translation.
3. Prokaryotes generally use the same genetic code as eukaryotes.
4. Stop codons are UAA, UGG, UGA.
5. mRNA codons have the same nucleotide sequence as DNA triplet codes.

A 1 and 2 only

B 1 and 3 only

C 1, 3 and 4

D 2, 4 and 5

8. The petals of the morning glory flower are purple when the flower is in bud but become
blue as the flower opens. The colour change is caused by an increase in pH of the
vacuoles of the petal cells.
A mutation in a gene coding for a type of hydrogen ion (H+) pump in the vacuole
membranes of the petal cells results in flowers that are purple when open.
The difference in DNA sequence of the gene coding for the ion pump in plants with blue
or purple flowers is shown below.

plant with blue flowers -T-T-A-A-T-C-C-T-G-A-G-A-T-T-T-

plant with purple flowers -T-T-A-A-T-C-C-T-G-C-T-G-A-G-A-T-T-T-

Which statements explain the purple colour of the flowers of the mutant plant?
1. A frameshift mutation alters the sequence of amino acids in the ion pump.
2. The primary structure and the shape of the ion pump are altered.
3. The activity of the ion pump is altered by changes in its shape.
4. The mutant ion pump cannot pump H+ ions into the vacuoles of the petal cells

A 1 and 2

B 1 and 4

C 2 and 3

D 3 and 4

River Valley High School Pg 6 of 24 Year 6 H2 Biology 9648 Paper 1


Preliminary Examination 2013
1369

9. Ce
ell division is the means of almostt all growth and reprodu
uction.
Which graph correctly re
epresents a form of cell division th
hat maintainns genetic stability
s at
exxpense of va
ariation?

10. A certain (hyypothetical) organism is diploid, has eitherr blue or oorange winggs as the
co
onsequence e of one of its genes, a
and has eith
her long or short antennnae as a result
r of a
se
econd gene,, as shown below.

If an organism m is hetero
ozygous forr both genes, what will be the poossible chro
omosomes
an ound in one of her eggss?
nd alleles fo

A Two chrromosomes
s 12 with blu
ue and oran
nge wings alleles.
a

B Chromo gs allele and chromoosome 19 with long


osome 12 with an orrange wing
antenna
ae allele.

C Chromo osome 19 with


w both o
orange wing
gs allele an
nd short an tennae alle
ele due to
crossing
g over.

D Chromoosome 12 with
w blue an d orange wings
w alleles
s and chrom
mosome 19
9 with long
and sho
ort antennae
e alleles.

River Valle
ey High Schoool Pg 7 of 24 Year 6 H2 Biology 9648 Paper 1
Preliminaryy Examination 2013
1370

11. When a bacterial cell infected by a lambda phage gets co-infected with another lambda
phage at a later point in time, this may not result in the death of the bacterial cell. Which
of the following statement(s) is a possible mechanism(s) for the phenomenon?
1. One of the viruses makes a larger number of phages.
2. The transcription and translation of viral DNA is not initiated.
3. A repressor protein is synthesised to shut down replication of the phage.
4. Mutation of host genome results in the inability of lambda phage to enter host cell.

A 2 only

B 2 and 3 only

C 1 and 4 only

D 1 and 3 only

12. Genetic diversity in bacteria is maintained by all of the following except

A mutation

B transformation

C transduction

D binary fission

13. Which of the following is / are true about trp and lac operons?

trp operon lac operon

1. produces polycistronic mRNA produces monocistronic mRNA

2. repressor binds to operator repressor binds to promoter

3. inducible repressible

4. transcription occurs in the presence of transcription occurs in the absence of


tryptophan glucose

A 2 only

B 1 and 4 only

C 1, 2 and 4

D None of the above

River Valley High School Pg 8 of 24 Year 6 H2 Biology 9648 Paper 1


Preliminary Examination 2013
1371

14. Use your knowledge and the information provided to work out which description of
telomeres and telomerase reverse transcriptase (TERT) is correct.

telomeres telomerase reverse transcriptase (TERT)

A Present in eukaryotes Use RNA as a template to make single stranded


DNA

B Present in eukaryotes Inhibits the loss of telomeres from DNA during semi-
conservative replication

C Present in prokaryotes Inhibits the loss of telomeres from DNA during semi-
conservative replication

D Present in prokaryotes Uses RNA as a template to make single stranded


DNA

15. Which of the following statements about eukaryotic transcription are correct?
1. Attachment of RNA polymerase to promoter is achieved by interaction of transcription
factors.
2. Enhancers and silencers are DNA sequences found downstream of structural genes
and can regulate gene expression.
3. Attachment of activator proteins to enhancers increases the basal activity of the
promoter.
4. Attachment of repressor proteins to silencers suppresses the basal activity of
promoter.

A 1 and 2 only

B 3 and 4 only

C 1, 2 and 3

D 1, 3 and 4

River Valley High School Pg 9 of 24 Year 6 H2 Biology 9648 Paper 1


Preliminary Examination 2013
1372

16. The table shows a comparison of some aspects of the genomes and protein-coding
genes of the prokaryote bacterium Escherichia coli and the eukaryote fungus
Saccharomyces cerevisiae.

E. coli S. cerevisiae
Genome length / base pairs 4 640 000 12 068 000
Number of protein-coding genes 4300 5800
Proteins with roles in:
metabolism 650 650
energy release / storage 240 175
membrane transport 280 250
transcription 240 400
translation 180 350
cell structure 180 250
Which of the following statement(s) accounts for the differences in the number of protein-
coding genes?
1. Many catabolic pathways for using carbon compounds in prokaryotes.
2. The presence of introns.
3. The presence of membrane-bound organelles in eukaryotes.
4. The use of histones to package DNA in eukaryotes.

A 1 only

B 1 and 2 only

C 2, 3 and 4

D 1, 3 and 4

17. Which statement(s) about tumour suppressor genes is not correct?


1. Tumour suppressor genes act in a genetically dominant manner.
2. There are fewer tumour suppressor genes known than oncogenes.
3. Tumour suppressor genes normally become oncogenes by mutations that eliminate
their normal activity.
4. Tumour suppressor genes can be responsible for some familial cancers.

A 1 only

B 3 only

C 1 and 3 only

D 2 and 4 only

River Valley High School Pg 10 of 24 Year 6 H2 Biology 9648 Paper 1


Preliminary Examination 2013
1373

18. The table shows the results of a series of crosses in a species of small mammal.

coat colour phenotype


male parent female parent offspring
dark grey light grey dark grey, light grey, albino
light grey albino light grey, white with black patches
dark grey white with black patches dark grey, light grey
light grey dark grey dark grey, light grey, white with
black patches

What explains the inheritance of the range of phenotypes shown by these crosses?

A one gene with a pair of co-dominant alleles

B one gene with multiple alleles

C sex linkage of the allele for grey coat colour

D two genes, each with a dominant and recessive allele

19. In mice, the gene for dappled coat (D) and its recessive allele for plain coat (d), are
located on the X chromosome. The gene for straight whiskers (W) and its recessive
allele for bent whiskers (w) are autosomal.
A male mouse with plain coat and bent whiskers was mated on several occasions to the
same female and the large number of offspring consisted of males and females in equal
numbers in all combinations of phenotypes, as shown in the table.

Offspring
Dappled, straight whiskers
Dappled, bent whiskers
Plain, straight whiskers
Plain, bent whiskers

If this female mouse is mated to a male with dappled coat and bent whiskers.
What proportion of the offspring would have at least one dominant allele at each gene
locus?

A 1/8

B 3/8

C 1/4

D 3/4

River Valley High School Pg 11 of 24 Year 6 H2 Biology 9648 Paper 1


Preliminary Examination 2013
1374

20. The family tree shows the inheritance of a human defect.

What is the most likely genetic basis for the occurrence of this defect?

A Autosomal recessive

B Autosomal dominant

C Sex-linked recessive

D Sex-linked dominant

21. The foxglove plant has three different coloured flowers - purple, pink and white.
Coloured pigment is synthesised enzymatically from a white precursor as shown below:
white precursor pigment X pigment Y
E1 E2

Two genetically identical plants with white flowers were crossed. This produced an F1
generation with 2170 white-flowered, 530 purple-flowered and 180 pink-flowered plants.
The alleles are represented by the following symbols:
R: no production of pigment r: production of pigment
B: purple pigment b: pink pigment
Which statement correctly describes the relationship between the two genes?

A Dominant epistasis in which the epistatic allele is B.

B Recessive epistasis in which the epistatic allele is b.

C Dominant epistasis in which the epistatic allele is R.

D Recessive epistasis in which the epistatic allele is r.

River Valley High School Pg 12 of 24 Year 6 H2 Biology 9648 Paper 1


Preliminary Examination 2013
1375

22. Which statement concerning chrysanthemum plants, of the genus Dendranthema, is a


valid example of how the environment may affect the phenotype?

A Anthocyanins and anthoxanthins are vacuolar pigments, whereas xanthophylls and


carotenes are pigments found in membrane-bound organelles known as plastids.
These, together with molecules known as co-pigments, are responsible for the
variation observed in petal colour in Dendranthema.

B Identical genetic crosses performed between varieties of Dendranthema result in a


greater proportion of offspring plants with plastids exhibiting a yellow colour when
grown in a field and a greater proportion of offspring plants with colourless plastids
when grown in a glass house.

C The seeds of a cross between Dendranthema weyrichii and Dendranthema


grandiflora produce plants that are far more frost-tolerant and exhibit an extended
flowering season compared with both parent plants.

D The seeds of a cross between Dendranthema weyrichii (height varying between


12.5 15.0 cm) and Dendranthema grandiflora (height varying between 8.0 25.0
cm) produce plants, when grown in natural day length, of a height varying between
55.0 71.0 cm.

River Valley High School Pg 13 of 24 Year 6 H2 Biology 9648 Paper 1


Preliminary Examination 2013
1376

23. Rubisco is the carbon dioxide-fixing enzyme. One rubisco molecule has eight active sites
where carbon dioxide fixation occurs, with each active site catalysing only three reduction
reactions per second.
The enzyme also catalyses, at the same active sites, the addition of oxygen to rubisco
bisphosphate (RuBP). This reaction is favoured when oxygen concentrations in the leaf
are high and carbon dioxide concentrations are low.

Which of the facts is paired with a correct explanation?

Fact Explanation

A On very hot, dry days stomata close to This reduces the availability of oxygen,
prevent water loss. increasing the production of 3-
phosphoglycerate.

B Plants synthesise large volumes of This may be an adaptive response to


rubisco. compensate for low concentrations of
oxygen.

C Processing 2-phosphoglycolate will This will increase the rate of reduction


eventually release carbon dioxide. and increase the rate of RuBP
regeneration, increasing the rate of
photosynthesis.

D Rubisco is an inefficient photosynthetic This is because the rate of carbon


enzyme. dioxide reduction can be decreased by
the competitive binding of oxygen
molecules to the active site.

River Valley High School Pg 14 of 24 Year 6 H2 Biology 9648 Paper 1


Preliminary Examination 2013
1377

24. The table below shows reactions occurring in a plant cell, and their respective locations.

Reaction Location in a cell

1. ribulose bisphosphate + CO2 glycerate-3-phosphate Stroma

2. glucose + ATP glucose-6-phosphate + ADP Matrix

3. oxygen + 4H+ + 4e- 2 H2O Stroma

4. oxaloacetate + acetyl-CoA citrate Matrix

Which of the following is / are incorrectly matched?

A 2 only

B 4 only

C 2 and 3 only

D 1, 2 and 3

25. Which of the following statement(s) about membranes is correct?


1. All intracellular membranes in a eukaryotic cell have the same type of lipids and
proteins.
2. The outer and inner membranes of mitochondria have the same type of transport
proteins.
3. Carbohydrates from part of glycoproteins or glycolipids in the membranes.
4. All plant cell membranes have cholesterol.

A 1 only

B 3 only

C 3 and 4 only

D 1, 3 and 4

River Valley High School Pg 15 of 24 Year 6 H2 Biology 9648 Paper 1


Preliminary Examination 2013
1378

26. In an experim
ment on osm mosis in plaant cells, th
he solute potential (ss), pressure
e potential
p) and wate
( er potential (w) of the cell were recorded in the table beelow.

Con
ndition of plant
p cell Pote
ential / kPa

s p w

Plasmoly
ysed -100
00 P Q

Fully turgid -600 R S

Which of the following co


orrectly reprresents the value of P, Q, R and S
S?

P Q R S

A +
+1000 -2
2000 +6
600 0

B -1000 -2
2000 0 -600

C 0 -1
1000 -6
600 -12
200

D 0 -1
1000 +6
600 0

27. Th
he diagram below illusttrates the m
main features of homeo
ostasis.

What are P, Q,
Q R and S?
?

P Q R S

A Fee
edback Recceptor / Controller / Effe
ector
De tector Integ
grator

B Recceptor / Con troller / Feed


dback Effe
ector
De
etector egrator
Inte

C Fee
edback Recceptor / Effe
ector Contrroller /
De tector Integ
grator

D Efffector Recceptor / Feed


dback Contrroller /
De tector Integ
grator

River Valle
ey High Schoool Pg 16 of 24 Year 6 H2 Biology 9648 Paper 1
Preliminaryy Examination 2013
1379

28. Thhe diagram shows how


w a nerve ce
ell and an en
ndocrine ce
ell interact w
with cell X and cell Y
resspectively.

Wh
hich of the following
f is true of both
h interaction
ns?

A Diffusion
n is involve
ed in the m
movement of
o chemicals M and N to their respective
r
target ce
ells.

B Chemica
al M is secrreted faster than chemical N.

C When bothb cells X and Y a ated, phosp


are stimula phorylation cascades must be
initiated.

D Both ch
hemicals M and N m ust bind to
o cell-surface receptoors on cell X and Y
respectively.

River Valle
ey High Schoool Pg 17 of 24 Year 6 H2 Biology 9648 Paper 1
Preliminaryy Examination 2013
1380

29. The figure below shows the time course of glucose uptake by the working rat heart in the
absence and presence of insulin. The lower the amount of total perfusate glucose, the
greater is the uptake of glucose by the rat heart.

From the given data, which of the following statements can be concluded to be correct?
1. Cells in the rats heart have receptors for insulin.
2. There is increased uptake of glucose by the rats heart in the presence of insulin.
3. Release of insulin is stimulated by increase in blood glucose level.
4. The rat is given a carbohydrate-rich meal prior to the experiment.

A 1 and 2 only

B 1 and 4 only

C 2 and 3 only

D 3 and 4 only

30. Which process triggers the uptake of Ca2+ ions in synaptic transmission?

A The influx of Na+ ions

B The release of neurotransmitters

C The depolarisation of the post-synaptic membrane

D The arrival of nerve impulse in the pre-synaptic neuron

River Valley High School Pg 18 of 24 Year 6 H2 Biology 9648 Paper 1


Preliminary Examination 2013
1381

31. Organisms are classified using taxons. The scientific names of three species of Hawaiian
honeycreeper are below.
Short-billed Honeycreeper, Cyanerpes nitidus
Shining Honeycreeper, Cyanerpes lucidus
Purple Honeycreeper, Cyanerpes caeruleus
Red-legged Honeycreeper, Cyanerpes cyaneus
What is the highest taxon that these honeycreepers have in common?

A genus

B order

C family

D class

32. The graphs represent the frequency of alleles in species, X, Y and Z during the after
selection.

In which species does evolution take place?

A X only

B Y only

C Y and Z

D None of X, Y nor Z
River Valley High School Pg 19 of 24 Year 6 H2 Biology 9648 Paper 1
Preliminary Examination 2013
1382

33. The following statements relate to molecular phylogenetics.


1. Lines of descent from a common ancestor to present-day organisms have undergone
similar, fixed rates of DNA mutation.
2. Organisms with similar base sequences in their DNA are closely related to each
other.
3. The number of differences in the base sequences of DNA of different organisms can
be used to construct evolutionary trees.
4. The proportional rate of fixation of mutations in one gene relative to the rate of fixation
of mutations in other genes stays the same in any given line of descent.
Which statements, when taken together, suggest the existence of a molecular clock that
enables scientists to estimate the time at which one species might have diverged from
another?

A 1 and 2

B 1 and 4

C 2 and 3

D 3 and 4

River Valley High School Pg 20 of 24 Year 6 H2 Biology 9648 Paper 1


Preliminary Examination 2013
1383

34. The diagram shows four plasmids.

Which plasmids would be suitable to use for genetic engineering using restriction
enzyme S?

A 1 and 4 only

B 2 and 3 only

C 1, 2 and 4

D 1, 3 and 4

River Valley High School Pg 21 of 24 Year 6 H2 Biology 9648 Paper 1


Preliminary Examination 2013
1384

35. In people witth a mutan


nt allele, a protein conntains just one differeent amino acid
a in its
primary struccture. To identify the
e presence e of this mutant
m alleele, DNA nucleotide
n
se
equences were
w comparred using ellectrophore
esis.
Th
he electroph
horesis resu
ults from the
e DNA of a normal alle
ele of a genee are shown
n below.

Which diagram represen


nts the DNA
A sequence for the mutant allele off this gene?
?

River Valle
ey High Schoool Pg 22 of 24 Year 6 H2 Biology 9648 Paper 1
Preliminaryy Examination 2013
1385

36. Which of the following is not necessary for Polymerase Chain Reaction to occur?
1. dATP
2. RNA primers
3. DNA fragment
4. Ribonucleotides

A 1 only

B 2 and 4 only

C 1, 3 and 4

D All of the above

37. In parallel with the Human Genome Project, the DNA of a set of model organisms (such
as bacteria, roundworms, fruitflies, mice and chimpanzees) are being studied.
Which technique might be most useful in using the information from model organisms to
provide critical clues about the structure and function of human genes?

A Sequencing

B DNA fingerprinting

C Comparing molecular homology

D Finding the regions of repetitive sequences

38. A key feature of most multicellular organisms is the ability to differentiate and produce
specialised cells.
Which row best describes the ability of zygotic cells to differentiate?

totipotent pluripotent multipotent

A Key

B = ability

C = no ability

River Valley High School Pg 23 of 24 Year 6 H2 Biology 9648 Paper 1


Preliminary Examination 2013
1386

39. It is possible to introduce an allele for a functioning CFTR protein into lung epithelial cells
of patients suffering from the genetically inherited condition cystic fibrosis.
Why this strategy can never provide a permanent cure for the patient?

A epithelial cells are continually dying and being replaced

B the DNA molecule that makes up the functioning allele is very unstable

C the methods of inserting the allele have low success rate

D this is only somatic and not germ line therapy

40. Marker genes are often inserted into genetically engineered crop plant cells, along with
desired genes. Bacterial antibiotic resistance genes are sometimes used as marker
genes. These may include short DNA repeats to make them unstable so that they are
quite quickly eliminated by the genetically engineered crop plant cells.
Which is not a reason why elimination of such marker genes is favoured?

A It is theoretically possible for the antibiotic resistance marker gene in human food to
pass to bacteria in the human gut.

B It is difficult to carry out repeated transformations using the same antibiotic.

C The antibiotics may affect the growth and differentiation of the fields of crop plants.

D There are a few such antibiotic resistance marker genes available.

River Valley High School Pg 24 of 24 Year 6 H2 Biology 9648 Paper 1


Preliminary Examination 2013
1387

RVHS H2 Biology Prelim 2013 Paper 1 Solution

1. D 11. B 21. C 31. D

2. C 12. D 22. B 32. B

3. A 13. D 23. D 33. B

4. C 14. A 24. C 34. A

5. D 15. D 25. B 35. C

6. A 16. D 26. D 36. B

7. B 17. B 27. A 37. C

8. C 18. B 28. A 38. A

9. A 19. B 29. A 39. A

10. B 20. A 30. D 40. C

River Valley High School Pg 1 of 1 Year 6 H2 Biology 9648 Paper 1


Preliminary Examination 2013
1388

RIVER VALLEY HIGH SCHOOL


YEAR 6 PRELIMINARY EXAMINATION
CANDIDATE
NAME

CENTRE INDEX CLASS


NUMBER NUMBER

H2 BIOLOGY 9648/02
Paper 2 Core Paper 20 September 2013
2 hours
Additional Materials: Answer Paper

READ THESE INSTRUCTIONS FIRST

Do not open this booklet until you are told to do so.


Write your Centre number, index number and name on all the work you hand in.
Write in dark blue or black pen on both sides of the paper.
You may use a soft pencil for any diagrams, graphs or rough working.
Do not use staples, paper clips, highlighters, glue or correction fluid.

Section A For Examiners Use


Answer all questions.
Section A
Section B 1 /10
Answer any one question.
2 / 12
At the end of the examination, fasten all ruled paper 3 / 12
securely.
Hand in the Question Paper and answers to Section B 4 / 12
separately. 5 / 12
6 / 10
The number of marks is given in brackets [ ] at the end of
each question or part question. 7 / 12
Section B
8 or 9* / 20
Total

_____________________________________________________________________
This Question Paper consists of 25 printed pages.

River Valley High School Pg 1 of 25 Year 6 H2 Biology 9648 Paper 2


Preliminary Examination 2013
1389

Section A (80 marks)

Answer all the questions in this section.

1. Fig. 1.1 is a diagram of an electron micrograph of part of a plant cell.

Fig. 1.1

(a) Using lines, indicated precisely in Fig. 1.1, a possible location where each of
the following molecules can be found:
A: chlorophyll
B: cellulose [2]

(b) (i) Identify the organelle labelled C and state a visible feature that
enabled your identification. [2]

River Valley High School Pg 2 of 25 Year 6 H2 Biology 9648 Paper 2


Preliminary Examination 2013
1390

1. (b) (ii) Explain how the functions of the two organelles, C and D, are linked. [2]

Fig. 1.2 shows a magnified imaged of organelle D.

Fig. 1.2

(c) (i) Describe the function of the protein that forms the membrane channel. [3]

River Valley High School Pg 3 of 25 Year 6 H2 Biology 9648 Paper 2


Preliminary Examination 2013
1391

1. (c) (ii) Suggest how prokaryotes, which have no membrane-bound


organelles, are able to secrete proteins. [1]

[Total: 10]

River Valley High School Pg 4 of 25 Year 6 H2 Biology 9648 Paper 2


Preliminary Examination 2013
1392

2. Yeast haploid cells secrete pheromones to signal mating, and respond by growing a
mating projection towards a potential mate. Upon contact of the two partner cells, these
fuse to form a diploid zygote.
Fig. 2.1 shows the pheromone-signaling pathway mediated by yeast G-protein coupled
receptor, Ste2. The activation of the pathway induces the expression of FUS1 gene which
is required for yeast mating.

Fig. 2.1

(a) With reference to Fig. 2.1, describe how factor triggers the activation of
Cdc42. [4]

River Valley High School Pg 5 of 25 Year 6 H2 Biology 9648 Paper 2


Preliminary Examination 2013
1393

2. (b) Suggest why factor cannot enter the yeast cell. [2]

Fig. 2.2 shows the Ste2 receptor on another yeast cell membrane.

Fig. 2.2

(c) Describe the structure of Ste2 receptor. [3]

River Valley High School Pg 6 of 25 Year 6 H2 Biology 9648 Paper 2


Preliminary Examination 2013
1394

2. (d) Explain how Ste2 receptor remains embedded in the yeast cell membrane. [3]

[Total: 12]

River Valley High School Pg 7 of 25 Year 6 H2 Biology 9648 Paper 2


Preliminary Examination 2013
1395

3. PART I

Lamivudine is an antiviral medication that prevents Human Immunodeficiency Virus (HIV)


from multiplying in the body. The structure of lamivudine, an analogue of viral nucleotide,
is shown in Fig. 3.1.

Fig. 3.1

(a) Explain how lamivudine prevents HIV from multiplying in the body. [2]

River Valley High School Pg 8 of 25 Year 6 H2 Biology 9648 Paper 2


Preliminary Examination 2013
1396

3. Many drugs have been designed to make it possible for AIDS patients to improve their
overall health and extend their lives. However, most of these drugs have low efficacy due
to high variance in the HIV genome. Fig. 3.2 shows the partial cDNA sequence of
glycoprotein 120 (gp120) on two different strains of HIV found in a host cell. The difference
in the two DNA sequences is indicated via a box.

HIV Strain 1
61 ctgaaaatct cgaaaacaat gccaaaacca taatagtgca ccttaataaa tctgtagaaa
121 tcaattgtac cagaccctcc aacaatacaa gaacaagtat aactatagga ccaggacaag
181 tatggtatag aacaggagac ataataggaa atataagaaa agcatattgt gagattaatg
241 gaacaaaatg gaataaagtt ttaaaacagg taactgaaaa attgaaagag cactttaata
301 ataagacaat aatctttcaa ccac

HIV Strain 2
61 ctgaaaatct cgaaaacaat gccaaaacca taatagtgca ccttaataaa tctgtagaaa
121 tcaattgtac cagaccctcc aacactacaa gaacaagtat aactatagga ccaggacaag
181 tatggtatag aacaggagac ataataggaa atataagaaa agcatattgt gagattaatg
241 gaacaaaatg gaataaagtt ttaaaacagg taactgaaaa attgaaagag cactttaata
301 ataagacaat aatctttcaa ccac

Fig. 3.2

(b) Explain how the genomic variation shown in Fig. 3.2 affects the structure of
gp120 on the two strains of HIV. [3]

River Valley High School Pg 9 of 25 Year 6 H2 Biology 9648 Paper 2


Preliminary Examination 2013
1397

3. (c) Describe how HIV enters host cell. [3]

PART II

Fig. 3.3 shows the structure of an isolated virus.

Fig. 3.3

(d) State two visible structural similarities between influenza virus and the virus
shown in Fig. 3.3. [2]

River Valley High School Pg 10 of 25 Year 6 H2 Biology 9648 Paper 2


Preliminary Examination 2013
1398

3. (e) Describe how the genome of influenza virus differs from that of the virus
shown in Fig. 3.3. [2]

[Total: 12]

River Valley High School Pg 11 of 25 Year 6 H2 Biology 9648 Paper 2


Preliminary Examination 2013
1399

4. PART I

Fig. 4.1 is an electron micrograph of chromatin maintained at the 30 nm higher-order fiber.

Fig. 4.1

(a) Most of the chromatin is seen to be in the form of a fiber with a diameter of about
30 nm.

(i) Describe how DNA is packed into the 30 nm chromatin fibre. [2]

(ii) Explain why the genome is packaged this way. [2]

River Valley High School Pg 12 of 25 Year 6 H2 Biology 9648 Paper 2


Preliminary Examination 2013
1400

4. Processes such as transcription and replication require the two strands of DNA to come
apart temporarily, thus allowing access of polymerases to the DNA template. However, the
folding of chromatin into 30 nm fibers poses barriers to the enzymes that unwind and copy
DNA.

(b) Explain how the histones are removed transiently to permit replication or
transcription to proceed. [3]

PART II

Fig. 4.2 and Fig. 4.3 are diagrams showing DNA replication and transcription,
respectively.

Fig. 4.2

Fig. 4.3
River Valley High School Pg 13 of 25 Year 6 H2 Biology 9648 Paper 2
Preliminary Examination 2013
1401

4. (c) Explain why RNA primer is required for the synthesis of new strand in DNA [1]
replication.

(d) Describe two ways in which the functions of molecules P and Q are

(i) similar to each other; [2]

(ii) different from each other. [2]

[Total: 12]

River Valley High School Pg 14 of 25 Year 6 H2 Biology 9648 Paper 2


Preliminary Examination 2013
1402

5. In a particular variety of tomato plant, the allele for red fruit colour (A) is dominant over the
allele for orange fruit (a) and the allele for green base when ripe (B) is dominant over the
allele for no green base when ripe (b).
Two students, Eugene and Cornelius crossed plant with red fruit and green base when
ripe with pure bred plant with orange fruit and no green bases when ripe. The phenotypes
of 50 offspring of each of Eugenes and Corneliuss crosses were recorded and are shown
in Table 5.1.

Table 5.1
Phenotypes of offspring of test crosses
Red fruit with Red fruit with no Orange fruit with Orange fruit with
green base green base green base no green base
Eugenes cross 23 4 3 20
Corneliuss cross 3 21 23 3

(a) Describe briefly how a plant breeder ensures that the offspring produced are
only from the desired cross. [2]

(b) Using the results from Eugenes cross,

(i) Carry out a 2 test using the formula below. [1]

River Valley High School Pg 15 of 25 Year 6 H2 Biology 9648 Paper 2


Preliminary Examination 2013
1403

5. (b) (ii) With reference to the table of probabilities provided, explain how the
value for the 2 test supports the hypothesis that the two genes are
independently segregating. [2]

degrees of probability, p
freedom 0.10 0.05 0.02 0.01 0.001
1 2.71 3.84 5.41 6.64 10.83
2 4.61 5.99 7.82 9.21 13.82
3 6.25 7.82 9.84 11.35 16.27
4 7.78 9.49 11.67 13.28 18.47

(c) Using the symbols given, state the genotypes of the tomato plants in
Eugenes and Corneliuss crosses. [2]

Eugenes cross:

Corneliuss cross:

River Valley High School Pg 16 of 25 Year 6 H2 Biology 9648 Paper 2


Preliminary Examination 2013
1404

5. (d) Draw a genetic diagram to explain the results of Corneliuss cross. [5]

[Total: 12]

River Valley High School Pg 17 of 25 Year 6 H2 Biology 9648 Paper 2


Preliminary Examination 2013
1405

6. PART I

Fig. 6.1 is a diagram of an electron micrograph of part of a chloroplast showing thylakoid


membranes.

Fig. 6.1

(a) Describe the role of thylakoid membrane in photosynthesis. [4]

River Valley High School Pg 18 of 25 Year 6 H2 Biology 9648 Paper 2


Preliminary Examination 2013
1406

6. PART II

Arabidopsis thaliana is a small flowering plant native to Asia. A mutation in the gene
coding for NADP oxidase results in plants with short root hairs. NADP oxidase is an
enzyme that converts NADPH to NADP+.
Fig. 6.2 shows the root hairs in the two variant of A. thaliana.

Fig. 6.2

(b) Explain the role of NADPH in photosynthesis. [2]

River Valley High School Pg 19 of 25 Year 6 H2 Biology 9648 Paper 2


Preliminary Examination 2013
1407

6. In a separate experiment, the length of root hairs in wild type and mutant A. thaliana were
measured at intervals, together with the activity of NADP oxidase in the tips of the root
hairs. The results are shown in Fig. 6.3.

Fig. 6.3

(c) With reference to Fig. 6.3, and your knowledge of photosynthesis, describe
and explain the difference in the growth of root hairs in the two types of
A. thaliana. [4]

[Total: 10]

River Valley High School Pg 20 of 25 Year 6 H2 Biology 9648 Paper 2


Preliminary Examination 2013
1408

7. Fig. 7.1 shows the arrangement of bones in the pentadactyl forelimbs of four vertebrates.

Fig. 7.1

(a) Explain how the relationship between the structures in Fig. 7.1 provides
evidence to support the theory of evolution. [2]

(b) Explain how fossil evidence can also be used as evidence to support the
idea that evolution has taken place? [2]

River Valley High School Pg 21 of 25 Year 6 H2 Biology 9648 Paper 2


Preliminary Examination 2013
1409

7. (c) Suggest how natural selection could have brought about the evolution of the
skeleton of a bats wing. [3]

The pipistrelle is the most common species of bat in Europe. Fig. 7.2 shows a map of
Canary Islands, a Spanish archipelago located just off the northwest coast of mainland
Africa. El Hierro is the most recent Canary Island, having appeared approximately
1.2 million years ago.

Fig. 7.2

River Valley High School Pg 22 of 25 Year 6 H2 Biology 9648 Paper 2


Preliminary Examination 2013
1410

7. It was originally thought that all pipistrelles belonged to the same species. However,
mtDNA reveals that there were two species: Pipistrellus maderensis and Pipistrellus kuhlii.
The data for both species are provided in Table 7.1.

Table 7.1

Species Mean body Mean Colour


mass (g) wingspan (cm)
P. maderensis 5.7 30 dark brown fur with light wing
markings
P. kuhlii 5.5 30 light brown to reddish brown with
light wing markings

(d) Using the data in Table 7.1, suggest why pipistrelles were originally
classified as one species. [1]

Evolution of the two bat species, Pipistrellus maderensis and Pipistrellus kuhlii, was
studied by comparing ~1 kbp of mtDNA (from cytochrome b and 16S rRNA genes) from
the bats in different islands.
Sequence divergence of P. maderensis is the lowest between La Gomera and El Hierro as
compared to between La Gomera and the remaining islands (i.e. Tenerife and La Palma).

(e) Using the above information and Fig. 7.2,

(i) explain why the sequence divergence of P. maderensis is the lowest


between La Gomera and El Hierro. [3]

River Valley High School Pg 23 of 25 Year 6 H2 Biology 9648 Paper 2


Preliminary Examination 2013
1411

7. (e) (ii) suggest why no gene flow occurs between the P. maderensis on
La Gomera and El Hierro. [1]

[Total: 12]

River Valley High School Pg 24 of 25 Year 6 H2 Biology 9648 Paper 2


Preliminary Examination 2013
1412

Section B (20 marks)

Answer one question.

Write your answers on the separate answer paper provided.


Your answers should be illustrated by large, clearly labelled diagrams, where appropriate.
Your answers must be in continuous prose, where appropriate.
Your answers must be set out in sections (a), (b) etc., as indicated in the question.

A NIL return is necessary if you have not attempted this section.

8. (a) Describe the structure of the cell membrane and relate it to its properties. [7]

(b) Explain the significance of mitosis in growth, repair and asexual reproduction. [4]

(c) Outline the behaviour of chromosomes during meiosis I and explain the
significance of each phase in leading to genetic variation. [9]

[Total: 20]

9. (a) Discuss, using named examples, how dual control of gene expression is
achieved in Escherichia coli. [8]

(b) Explain the significance of operons in bacteria. [4]

(c) Explain how the structure of neurone facilitates impulse transmission. [8]

[Total: 20]

River Valley High School Pg 25 of 25 Year 6 H2 Biology 9648 Paper 2


Preliminary Examination 2013
1413

RIVER VALLEY HIGH SCHOOL


YEAR 6 PRELIMINARY EXAMINATION
CANDIDATE
NAME

CENTRE INDEX CLASS


NUMBER NUMBER

H2 BIOLOGY 9648/02
Paper 2 Core Paper 20 September 2013
2 hours
Additional Materials: Answer Paper

READ THESE INSTRUCTIONS FIRST

Do not open this booklet until you are told to do so.


Write your Centre number, index number and name on all the work you hand in.
Write in dark blue or black pen on both sides of the paper.
You may use a soft pencil for any diagrams, graphs or rough working.
Do not use staples, paper clips, highlighters, glue or correction fluid.

Section A For Examiners Use


Answer all questions.
Section A
Section B 1 /10
Answer any one question.
2 / 12
At the end of the examination, fasten all ruled paper 3 / 12
securely.
Hand in the Question Paper and answers to Section B 4 / 12
separately. 5 / 12
6 / 10
The number of marks is given in brackets [ ] at the end of
each question or part question. 7 / 12
Section B
8 or 9* / 20
Total

_____________________________________________________________________
This Question Paper consists of 25 printed pages.

River Valley High School Pg 1 of 30 Year 6 H2 Biology 9648 Paper 2


Preliminary Examination 2013
1414

Section A (80 marks)

Answer all the questions in this section.

1. Fig. 1.1 is a diagram of an electron micrograph of part of a plant cell.

Fig. 1.1

(a) Using lines, indicated precisely in Fig. 1.1, a possible location where each of
the following molecules can be found:
A: chlorophyll
B: cellulose [2]

(b) (i) Identify the organelle labelled C and state a visible feature that
enabled your identification. [2]

1. Golgi apparatus
2. Made up of stacks of flattened membrane bound sacs
3. Vesicles fusing with / budding off from / associated with structure

River Valley High School Pg 2 of 30 Year 6 H2 Biology 9648 Paper 2


Preliminary Examination 2013
1415

1. (b) (ii) Explain how the functions of the two organelles, C and D, are linked. [2]

1. Vesicles containing proteins buds off from D


2. Vesicle fuses with (cis face of) C
3. releasing proteins into lumen of C
4. for further modification/ packaging/ sorting/ targeting to specific
sites (any 2)

Fig. 1.2 shows a magnified imaged of organelle D.

Fig. 1.2

(c) (i) Describe the function of the protein that forms the membrane channel. [3]

1. Allow large molecules / polypeptide chain to move across the


membrane
2. Function as an anchor for ribosomes during protein synthesis
3. Function as a receptor to bind signal proteins to anchor ribosomes

River Valley High School Pg 3 of 30 Year 6 H2 Biology 9648 Paper 2


Preliminary Examination 2013
1416

1. (c) (ii) Suggest how prokaryotes, which have no membrane-bound


organelles, are able to secrete proteins. [1]

1. Pores / protein channels present in cell surface membrane of


prokaryotes

[Total: 10]

River Valley High School Pg 4 of 30 Year 6 H2 Biology 9648 Paper 2


Preliminary Examination 2013
1417

2. Yeast haploid cells secrete pheromones to signal mating, and respond by growing a
mating projection towards a potential mate. Upon contact of the two partner cells, these
fuse to form a diploid zygote.
Fig. 2.1 shows the pheromone-signaling pathway mediated by yeast G-protein coupled
receptor, Ste2. The activation of the pathway induces the expression of FUS1 gene which
is required for yeast mating.

Fig. 2.1

(a) With reference to Fig. 2.1, describe how factor triggers the activation of
Cdc42. [4]

1. factor binds to Ste2


2. via complementary shape
3. triggering a conformational change in Ste2
4. allowing Ste2 to bind Gpa1(-Ste4-Ste18 complex)
5. This causes GTP to displace GDP
6. thus activation the (Gpa1-)Ste4-Ste18 complex
7. In turn, allowing for Ste4-Ste18 dimer to dissociate from the complex
8. to activate Cdc42

River Valley High School Pg 5 of 30 Year 6 H2 Biology 9648 Paper 2


Preliminary Examination 2013
1418

2. (b) Suggest why factor cannot enter the yeast cell. [2]

1. factor is polar / charged


2. thus is blocked by the phospholipid bilayer of yeast cell membrane that
forms a hydrophobic barrier
Accept: Too large to pass through transient gaps between phospholipid
molecules forming the membrane.

Fig. 2.2 shows the Ste2 receptor on another yeast cell membrane.

Fig. 2.2

(c) Describe the structure of Ste2 receptor. [3]

1. A single polypeptide chain


2. consisting of (seven) helix segments
3. Folds upon itself
4. Such that the helices are gathered together
5. forming a cylindrical / globular structure

River Valley High School Pg 6 of 30 Year 6 H2 Biology 9648 Paper 2


Preliminary Examination 2013
1419

2. (d) Explain how Ste2 receptor remains embedded in the yeast cell membrane. [3]

1. Polar R groups of amino acid residues on Ste2


2. form hydrogen bonds / hydrophilic interactions
3. with polar phosphate heads of membrane phospholipid molecules
Accept: ionic bonds between oppositely charged R groups & phosphate
heads
4. Non-polar R groups of amino acid residues on Ste2
5. form hydrophobic interactions
6. with non-polar hydrocarbon tails of membrane phospholipid molecules
Reject: Lack of reference to Ste2 and yeast cell membrane

[Total: 12]

River Valley High School Pg 7 of 30 Year 6 H2 Biology 9648 Paper 2


Preliminary Examination 2013
1420

3. PART I

Lamivudine is an antiviral medication that prevents Human Immunodeficiency Virus (HIV)


from multiplying in the body. The structure of lamivudine, an analogue of viral nucleotide,
is shown in Fig. 3.1.

Fig. 3.1

(a) Explain how lamivudine prevents HIV from multiplying in the body. [2]

1. Prevents viral cDNA synthesis


2. Reverse transcriptase incorporates
3. lamivudine instead of viral nucleotide
4. Terminating DNA chain elongation
5. due to absence of 3 OH grp

River Valley High School Pg 8 of 30 Year 6 H2 Biology 9648 Paper 2


Preliminary Examination 2013
1421

3. Many drugs have been designed to make it possible for AIDS patients to improve their
overall health and extend their lives. However, most of these drugs have low efficacy due
to high variance in the HIV genome. Fig. 3.2 shows the partial cDNA sequence of
glycoprotein 120 (gp120) on two different strains of HIV found in a host cell. The difference
in the two DNA sequences is indicated via a box.

HIV Strain 1
61 ctgaaaatct cgaaaacaat gccaaaacca taatagtgca ccttaataaa tctgtagaaa
121 tcaattgtac cagaccctcc aacaatacaa gaacaagtat aactatagga ccaggacaag
181 tatggtatag aacaggagac ataataggaa atataagaaa agcatattgt gagattaatg
241 gaacaaaatg gaataaagtt ttaaaacagg taactgaaaa attgaaagag cactttaata
301 ataagacaat aatctttcaa ccac

HIV Strain 2
61 ctgaaaatct cgaaaacaat gccaaaacca taatagtgca ccttaataaa tctgtagaaa
121 tcaattgtac cagaccctcc aacactacaa gaacaagtat aactatagga ccaggacaag
181 tatggtatag aacaggagac ataataggaa atataagaaa agcatattgt gagattaatg
241 gaacaaaatg gaataaagtt ttaaaacagg taactgaaaa attgaaagag cactttaata
301 ataagacaat aatctttcaa ccac

Fig. 3.2

(b) Explain how the genomic variation shown in Fig. 3.2 affects the structure of
gp120 on the two strains of HIV. [3]

1. Base pair substitution mutation


2. at 145th base pair
3. changes adenine to cytosine
4. results in change in corresponding codon on mRNA
5. thus changes the amino acid encoded in the resultant polypeptide chain
6. This changes the property of amino acid residue / folding of polypeptide
Reject: protein
7. resulting each HIV having gp120 with different three-dimensional
conformation

River Valley High School Pg 9 of 30 Year 6 H2 Biology 9648 Paper 2


Preliminary Examination 2013
1422

3. (c) Describe how HIV enters host cell. [3]

1. gp120 on viral envelope binds to CD4 receptor


2. on host cell membrane
3. This induces a change in gp120s three-dimensional conformation
4. allowing for gp120 to bind to chemokine receptor on host cell membrane
5. causing gp41 to change its three-dimensional conformation
6. triggering the fusion of viral envelope with host cell membrane

PART II

Fig. 3.3 shows the structure of an isolated virus.

Fig. 3.3

(d) State two visible structural similarities between influenza virus and the virus
shown in Fig. 3.3. [2]

1. Both viruses are surrounded by a membrane envelope


2. Both viruses have capsid enclosing their genome
3. Both viruses have additional components embedded in their membrane.

River Valley High School Pg 10 of 30 Year 6 H2 Biology 9648 Paper 2


Preliminary Examination 2013
1423

3. (e) Describe how the genome of influenza virus differs from that of the virus
shown in Fig. 3.3. [2]

Factor Influenza virus Virus on Fig.3.3

Nature of genome DNA RNA

Number of strands Double stranded Single stranded


Reject: short form: ss

Number of molecules 1 8

AVP

[Total: 12]

River Valley High School Pg 11 of 30 Year 6 H2 Biology 9648 Paper 2


Preliminary Examination 2013
1424

4. PART I
Fig. 4.1 is an electron micrograph of chromatin maintained at the 30 nm higher-order fiber.

Fig. 4.1

(a) Most of the chromatin is seen to be in the form of a fiber with a diameter of
about 30 nm.

(i) Describe how DNA is packed into the 30 nm chromatin fibre. [2]

1. DNA is wound around a histone octamer


2. to form nucleosome
3. together with linker DNA made up the 10 nm chromatin fibre /
nucleosome fibre
4. further coiling involved Histone H1 and
5. linker DNA
to form 30-nm chromatin fibre

(ii) Explain why the genome is packaged this way. [2]

1. to make it compact/ condensed; thus DNA can be packed within


the cells nucleus
2. to regulate gene expression/ transcription
3. to prevent the long DNA molecules; from breaking or damage or
getting tangled

River Valley High School Pg 12 of 30 Year 6 H2 Biology 9648 Paper 2


Preliminary Examination 2013
1425

4. (b) Processes such as transcription and replication require the two strands of
DNA to come apart temporarily, thus allowing access of polymerases to the
DNA template. However, the folding of chromatin into 30 nm fibers pose
barriers to the enzymes that unwind and copy DNA.

Explain how the histones are removed transiently to permit replication or


transcription to proceed. [3]

1. histone acetylation
2. acetyl groups are attached to lysines in histone tails
3. the positive charges are neutralized
4. histone tails no longer interact with the neighbouring nucleosomes
5. chromatin has a more diffused structure
6. allows access of DNA polymerase and RNA polymerase for
replication and transcription, respectively

PART II
Fig. 4.2 and Fig. 4.3 are diagrams showing DNA replication and transcription respectively.

Fig. 4.2

Fig. 4.3

River Valley High School Pg 13 of 30 Year 6 H2 Biology 9648 Paper 2


Preliminary Examination 2013
1426

4. (c) Explain why RNA primer is required for the synthesis of new strand in DNA [1]
replication.

DNA polymerase can only join nucleotides; onto existing chain with free 3
OH group at one end

(d) Describe two ways in which the functions of molecules P and Q are

(i) similar to each other; [2]

Type of bond formed: phosphoester bond between nucleotides


Template used: DNA as template
Direction of reading the template: from 3 to 5 direction

(ii) different from each other. [2]


Feature Replication Transcription
Start site Begins at regions of Begins at regions of
DNA known as origins DNA known as
of replication promoter
Nucleotides base Free Free ribonucleotides/
pair with template deoxyribonucleotides/ ribonucleoside
deoxyribonucleoside triphosphate
triphosphate
Base pairing rule A=T and CG A=U and CG

[Total: 12]

River Valley High School Pg 14 of 30 Year 6 H2 Biology 9648 Paper 2


Preliminary Examination 2013
1427

5. In a particular variety of tomato plant, the allele for red fruit colour (A) is dominant over the
allele for orange fruit (a) and the allele for green base when ripe (B) is dominant over the
allele for no green base when ripe (b).
Two students, Eugene and Cornelius crossed plant with red fruit and green base when
ripe with pure bred plant with orange fruit and no green bases when ripe. The phenotypes
of 50 offspring of each of Eugenes and Corneliuss crosses were recorded and are shown
in Table 5.1.
Table 5.1

Phenotypes of offspring of test crosses

Red fruit with Red fruit with Orange fruit Orange fruit
green base no green base with green with no green
base base

Eugenes cross 23 4 3 20

Corneliuss 3 21 23 3
cross

(a) Describe briefly how a plant breeder ensures that the offspring produced are
only from the desired cross. [2]

1. anthers removed (before maturity) (to produce male sterility)


2. pollen transferred by hand
3. plants isolated / flowers bagged (before and after pollination)
any two

(b) Using the results from Eugenes cross,

(i) Carry out a 2 test using the formula below.

2 = + + + ;
= 8.82+ 5.78 + 7.22 + 4.50 = 26.32;
[1]

River Valley High School Pg 15 of 30 Year 6 H2 Biology 9648 Paper 2


Preliminary Examination 2013
1428

5. (b) (ii) With reference to the table of probabilities provided, explain how the
value for the 2 test supports the hypothesis that the two genes are
independently segregating.

degrees of probability, p
freedom 0.10 0.05 0.02 0.01 0.001
1 2.71 3.84 5.41 6.64 10.83
2 4.61 5.99 7.82 9.21 13.82
3 6.25 7.82 9.84 11.35 16.27
4 7.78 9.49 11.67 13.28 18.47
[2]

1. Given 2= 26.32, =3, the probability of the deviation is due to


chance is < 0.001 which is less than 0.05
2. Hence, reject Ho in favour of HA
3. There is significant difference between the observed and the
expected results
4. The two genes are not independently segregating

(c) Using the symbols given, state the genotypes of the tomato plants in
Eugenes and Corneliuss crosses. [2]
AB x ab Reject: different symbols used
Eugenes cross: ;;
a b ab
Ab x ab
Corneliuss cross: ;;
a B ab

River Valley High School Pg 16 of 30 Year 6 H2 Biology 9648 Paper 2


Preliminary Examination 2013
1429

(d) Draw a genetic diagram to explain the results of Corneliuss cross. [5]
Parental Red fruit with green x Orange fruit with no
phenotype: base; green base;
Parental Ab ab
genotype: aB ab
Gametes:
A b a B a b;

A B a b ;
Punnet
Parental gametes Recombinant gametes
square
A b a B A B a b
A b a B A B a b Correct
a b
a b a b a b a b genotype;;
Red fruit Orange Red fruit Orange Correct
with no fruit with with fruit with phenotype;;
green green green no green
base base base base
21 23 3 3 ;
Parental Recombinants ;
[Total: 12]

River Valley High School Pg 17 of 30 Year 6 H2 Biology 9648 Paper 2


Preliminary Examination 2013
1430

6. PART I

Fig. 6.1 is a diagram of an electron micrograph of part of a chloroplast showing thylakoid


membranes.

Fig. 6.1

(a) Describe the role of thylakoid membrane in photosynthesis. [4]

1. Large surface area for embedment of


2. photosynthetic pigments for harvesting light energy
3. ATP synthase for phosphorylation of ADP to ATP
4. photosystems to generate ATP & NADPH (for Calvin cycle)
5. electron transport chain for photophosphorylation
6. Impermeable to protons
7. allow for build-up of proton gradient for chemiosmosis

River Valley High School Pg 18 of 30 Year 6 H2 Biology 9648 Paper 2


Preliminary Examination 2013
1431

6. PART II

Arabidopsis thaliana is a small flowering plant native to Asia. A mutation in the gene
coding for NADP oxidase results in plants with short root hairs. NADP oxidase is an
enzyme that converts NADPH to NADP+.
Fig. 6.2 shows the root hairs in the two variant of A. thaliana.

Fig. 6.2

(b) Explain the role of NADPH in photosynthesis. [2]

1. Provides reducing power in Calvin cycle


2. to convert 1,3-bisphosphoglycerate to glyceraldehyde-3-phosphate

River Valley High School Pg 19 of 30 Year 6 H2 Biology 9648 Paper 2


Preliminary Examination 2013
1432

6. In a separate experiment, the length of root hairs in wild type and mutant A. thaliana were
measured at intervals, together with the activity of NADP oxidase in the tips of the root
hairs. The results are shown in Fig. 6.3.

Fig. 6.3

(c) With reference to Fig. 6.3, and your knowledge of photosynthesis, describe
and explain the difference in the growth of root hairs in the two types of A.
thaliana. [4]

Describe
45 to 95 min,
1. Mean length of root hair for wild type A. thaliana increases from 10um to
38um
2. while that for mutant A. thaliana increases from 10um to 14um
3. Mean activity of NADP in wild type A. thaliana increases from 13au to
42au
4. while that in mutant A. thaliana decreases from 9au to 6au
Explain
5. NADP oxidase catalyses the conversion of NADPH to NADP+ for light
reaction of photosynthesis
Accept: Calvin cycle of photosynthesis
6. where ATP is synthesised; for cell growth / growth of root hair
Accept: synthesis of glucose (if Calvin cycle mentioned in point 5)

[Total: 10]

River Valley High School Pg 20 of 30 Year 6 H2 Biology 9648 Paper 2


Preliminary Examination 2013
1433

7. Fig. 7.1 shows the arrangement of bones in the pentadactyl forelimbs of four vertebrates.

Fig. 7.1

(a) Explain how the relationship between the structures in Fig. 7.1 provides
evidence to support the theory of evolution. [2]

1. Same arrangement of bones in the pentadactyl forelimbs of four


vertebrates
2. Though they have different functions
3. Therefore they were likely to have originated from a common ancestor
4. Had been subject to different selection pressures

(b) Explain how fossil evidence can also be used as evidence to support the
idea that evolution has taken place? [2]

1. fossils show changes over time


Accept: many fossil organisms dissimilar from modern organisms /
general trend from, small / simple, to, large / complex
2. fossils or rocks can be dated
Accept: idea of fossils in chronological order
3. fossils showing intermediate forms
4. many fossils no longer exists

River Valley High School Pg 21 of 30 Year 6 H2 Biology 9648 Paper 2


Preliminary Examination 2013
1434

7. (c) Suggest how natural selection could have brought about the evolution of the
skeleton of a bats wing. [3]

1. Variation in the skeletal structure


2. due to mutations/ presence of different alleles
3. Those with favourable phenotype (have a selective advantage) survive to
reproduce
4. Pass on the allele to the offspring
5. Those at selective disadvantage will not survive
6. An increase in frequency of the allele for the favourable phenotype

7. The pipistrelle is the most common species of bat in Europe. Fig. 7.2 shows a map of
Canary Islands, a Spanish archipelago located just off the northwest coast of mainland
Africa. El Hierro is the most recent Canary Island, having appeared approximately 1.2
million years ago.

Fig. 7.2

River Valley High School Pg 22 of 30 Year 6 H2 Biology 9648 Paper 2


Preliminary Examination 2013
1435

7.
It was originally thought that all pipistrelles belonged to the same species. However,
mtDNA reveals that there were two species: Pipistrellus maderensis and Pipistrellus kuhlii.
The data for both species are provided in Table 7.1.
Table 7.1
Species Mean body mass Mean wingspan Colour
(g) (cm)
P. maderensis 5.7 3.0 dark brown fur with light
wing markings
P. kuhlii 5.5 3.0 light brown to reddish
brown with light wing
markings

(d) Using the data in Table 7.1, suggest why pipistrelle were originally classified
as one species. [1]

1. similar, (body) mass, i.e. 5.5 g


2. same wingspan, i.e. 3.0 cm
3. same wing markings, i.e. light wing marking
4. previously unable to measure sequence similarity

Evolution of the two bat species, Pipistrellus maderensis, Pipistrellus kuhlii, was studied by
comparing ~1 kbp of mtDNA (from cytochrome b and 16S rRNA genes) from the bats in
different islands.
Sequence divergence of P. maderensis is the lowest between La Gomera and El Hierro as
compared to between La Gomera and the remaining islands (i.e. Tenerife and La Palma).

(e) Using the above information and Fig. 7.2,

(i) explain why the sequence divergence of P. maderensis is the lowest


between La Gomera and El Hierro. [3]

1. a recent colonization of El Hierro by P. maderensis from La


Gomera
2. as the island was formed only 1.2 mya, insufficient time for
evolution
3. similar selection pressure

River Valley High School Pg 23 of 30 Year 6 H2 Biology 9648 Paper 2


Preliminary Examination 2013
1436

(ii) suggest why no gene flow occurs between the P. maderensis on La


Gomera and El Hierro. [1]

The distance between La Gomera and El Hierro is greater than


between other islands

[Total: 12]

River Valley High School Pg 24 of 30 Year 6 H2 Biology 9648 Paper 2


Preliminary Examination 2013
1437

Section B (20 marks)

Answer one question.

Write your answers on the separate answer paper provided.


Your answers should be illustrated by large, clearly labelled diagrams, where appropriate.
Your answers must be in continuous prose, where appropriate.
Your answers must be set out in sections (a), (b) etc., as indicated in the question.

A NIL return is necessary if you have not attempted this section.

8. (a) Describe the structure of the cell membrane and relate it to its properties [7]

Structure Property Selective permeability


1. comprises of phospholipid P1. permeable to non-polar / hydrophobic
bilayers substances
2. hydrophobic tails facing
inwards and hydrophilic
heads facing the aqueous
medium
3. hydrophobic barrie
S2a. interspersed with proteins P2. transmembrane proteins allow
S2b. integral proteins are transport of ions/ polar/ hydrophilic
embedded in the membrane substrances across the membrane
and
S2c. peripheral proteins are not
embedded in the membrane
Structure Property Fluidity
S3. phospholipids can move P3a. fluidity
laterally P3b. can reseal itself after it is disrupted
P3c. can invaginate and form vesicles
P3d. transient gap for movement of small,
uncharged &/ non-polar molecules
S4. cholesterol wedged between P5. regulates the fluidity
phospholipid molecules
Structure Property Cellular identity
S5a. presence of glycolipids and P5a. membrane is asymmetrical
glycoproteins; P5b. carbohydrate chain serves as cell
S5b. carbohydrates chains face surface marker / cellular recognition
the external matrix outside
the cell

River Valley High School Pg 25 of 30 Year 6 H2 Biology 9648 Paper 2


Preliminary Examination 2013
1438

[4]
(b) Explain the significance of mitosis in growth, repair and asexual reproduction.

1. Daughter cells are genetically identical to parent cell


2. contain the same number of chromosomes as the parent cell
3. Due to semi-conservative replication of DNA during interphase
4. results in genetic stability within populations of cells derived from the same
parental cells
5. daughter cells can then function harmoniously as part of the same tissue,
organ and organism

Growth
6. allows the growth and development of a zygote into a multicellular organism
7. forming new cells that are genetically identical to the existing cell(s)

Repair
8. produces new cells required to repair worn-out or damaged tissues
9. to return a tissue to its former condition

Asexual reproduction
10. to produce genetically identical daughter cells that can form new offspring
11. to reproduce rapidly in an environment that the parent plant is well adapted
to
12. colonises the habitat much more quickly than via sexual reproduction
13. AVP

River Valley High School Pg 26 of 30 Year 6 H2 Biology 9648 Paper 2


Preliminary Examination 2013
1439

(c) Outline the behaviour of chromosomes during meiosis I and explain the [9]
significance of each phase in leading to genetic variation.

Phase Event Significance


Prophase 1. Chromatin condenses to form chromosome S1. New
I 2. Chromatin forms looped domains around combination of
nonhistone protein alleles
3. further coiling to form chromosomes
4. each chromosome consists of 2 sister
chromatids joined at centromere
5. DNA replicated to form 2 daughter DNA
during interphase
6. Homologous chromosomes pair up to form
bivalent - synapsis
7. Chiasma formed
8. Crossing over between non-sister
chromatids of homologous chromosomes
9. At the end of prophase, each member of
homologous pair of chromosome is
attached to spindle fibres at the centromere
Metapha 10. Homologous chromosomes align at S2.Random
se I metaphase plate separation of
11. Orientation of each bivalent is independent homologous
of other bivalents chromosomes
Anaphas 12. Homologous chromosomes separate S3. Halving of
eI 13. pulled to opposite poles of cell chromosome
14. centromere leading number
S4. Results in
variation in
combination of
paternal and
maternal
chromosomes in
gametes

Telophas 15. Chromosome arrive at opposite poles of S5. Haploid set of


eI the cell chromosomes /
16. Chromosomes decondense to form forming 2 haploid
chromatin cells for fertilisation

[Total: 20]

River Valley High School Pg 27 of 30 Year 6 H2 Biology 9648 Paper 2


Preliminary Examination 2013
1440

9. (a) Discuss, using named examples, how dual control of gene expression is
achieved in Escherichia coli. [8]

Negative gene regulation: Scenario 1 or 2 [max 4 marks]


Scenario 1
1. Negative gene regulation
2. by lac repressor protein
3. Lactose present
4. Lactose enter cell due to a few molecules of permease
5. Lactose is converted to allolactose by -galactosidase
6. Allolactose binds to allosteric site on lac repressor protein
7. causing lac repressor protein to adopt its inactive conformation
8. Lac operon switched on

Scenario 2
9. Negative gene regulation
10. by lac repressor protein
11. Lactose absent
12. thus no allolactose bound to lac repressor protein
13. allowing for synthesis of small amounts of lac repressor protein
14. Lac repressor protein is in its active conformation
15. Lac repressor protein binds to the operator
16. Lac operon switched off

Positive gene regulation: Scenario 3 or 4 [max 4 marks]


Scenario 3
17. Positive gene regulation
18. by catabolite activator protein
19. Glucose absent, lactose present
20. Level of cAMP is high
21. Level of CAP-cAMP complex is high / CAP assumes active conformation
22. CAP-cAMP complex binds to promoter
23. increases affinity of RNA polymerase for promoter
24. stimulate / increases rate of transcription of structural genes

River Valley High School Pg 28 of 30 Year 6 H2 Biology 9648 Paper 2


Preliminary Examination 2013
1441

Scenario 4
25. Positive gene regulation
26. by catabolite activator protein
27. Glucose present, lactose present
28. Level of cAMP is low
29. Level of CAP-cAMP complex is low / most CAP assumes inactive conformation
30. CAP-cAMP complex does not bind to / detaches from promoter
31. Low affinity of RNA polymerase for promoter
32. Low rate of transcription of structural genes

(b) Explain the significance of operons in bacteria. [4]

1. Operons in bacteria group together genes


2. that are involved in a common metabolic pathway
3. for coordinated control of gene expression
4. by a single promoter and operator in that operon
5. This allows for the structural genes in an operon to be transcribed together
6. resulting in the concurrent synthesis of proteins involved in the same pathway
7. Operons enable bacteria to only synthesize the required proteins
8. This preventing wastage of energy and resources / allocating resources and energy
for other use
9. Operons also allow for bacteria to use a variety of sugars
10. and preferentially use resources to gain selective advantage

River Valley High School Pg 29 of 30 Year 6 H2 Biology 9648 Paper 2


Preliminary Examination 2013
1442

(c) Explain how the structure of neurone facilitates impulse transmission. [8]

Structure Function

1. Dendrites Extensively branched so that many synapses can


be made with many other neurons

2. Numerous mitochondria Produce ATP for sodium-potassium pumps /


in cell body and axon transport of vesicles / synthesis of proteins

3. Axon Conducts impulses away from cell body

4. Nissl's granules Synthesis of proteins such as Na+-K+ pump /


acetylcholinesterase

5. Myelin sheath Acts as an electrical insulator, preventing the


formation of action potentials

6. Nodes of Ranvier Enable action potential to jump from node to node,


increasing the speed of transmission of a nerve
impulse

7. Presence of synaptic Synaptic vesicles contain neurotransmitters for


vesicles in synaptic knob synaptic transmission

8. Na+-K+ pump Pumps 3 Na+ out of in exchange for 2 K+ into


axoplasm for maintenance of resting potential

9. Voltage-gated Na+ / K+ / Allows for influx of Na+ / efflux of K+ / efflux of Ca2+


Ca2+ channel upon trigger by change in membrane voltage

10. Chemically-gated Na+ Allows for diffusion of Na+ upon binding of


channel neurotransmitter

11. AVP

[Total: 20]

River Valley High School Pg 30 of 30 Year 6 H2 Biology 9648 Paper 2


Preliminary Examination 2013
1443

RIVER VALLEY HIGH SCHOOL


YEAR 6 PRELIMINARY EXAMINATION

CANDIDATE
NAME

CENTRE INDEX CLASS


NUMBER NUMBER

H2 BIOLOGY 9648/03
Applications Paper and Planning Question 26 September 2013
Paper 3 2 hours

Additional Materials: Answer Paper

READ THESE INSTRUCTIONS FIRST

Do not open this booklet until you are told to do so.


Write your centre number, index number and name on all the work you hand in.
Write in dark blue or black pen on both sides of the paper.
You may use a soft pencil for any diagrams, graphs or rough working.
Do not use staples, paper clips, highlighters, glue or correction fluid.

Answer all questions. For Examiners Use

At the end of the examination, fasten ruled paper for 1 / 15


Question 5 separately. 2 / 12

Hand in the Question Paper, answers to Question 5 3 / 13


separately. 4 (SPA) / 12

The number of marks is given in brackets [ ] at the end of 5 / 20


each question or part question. Total

_____________________________________________________________________
This Question Paper consists of 17 printed pages.

River Valley High School Pg 1 of 17 H2 Biology 9648 Paper 3


Preliminary Examination 2013
1444

Answer all questions.

1. DNA fingerprinting is a method of identification based on an individuals DNA. In traditional


DNA fingerprinting, DNA is first isolated from cells, and then amplified by Polymerase
Chain Reaction (PCR). Fig. 1.1 illustrates the process of PCR.

Fig. 1.1

(a) (i) Taq polymerase is derived from Thermus aquaticus, a species of


bacteria that thrives in hot springs. Explain why Taq polymerase is
able to increase the amount of DNA from most species of organisms. [2]

River Valley High School Pg 2 of 17 Year 6 H2 Biology 9648 Paper 3


Preliminary Examination 2013
1445

1. (a) (ii) Explain how the specific nucleotide sequence of the primer is
important for its role in PCR. [2]

(b) Distinguish between PCR and DNA replication in human skin cells. [2]

River Valley High School Pg 3 of 17 Year 6 H2 Biology 9648 Paper 3


Preliminary Examination 2013
1446

1. DNA amplified using PCR is then is cut by a specific restriction enzyme and separated
using gel electrophoresis. The DNA is then transferred to a nitrocellulose membrane,
before the addition of radioactive DNA probes for nucleic acid hybridisation. Fig. 1.2
shows the nucleic acid hybridisation process.

Fig. 1.2

1. (c) With reference to Fig. 1.2, explain the significance of the temperatures used
during nucleic acid hybridisation. [3]

River Valley High School Pg 4 of 17 Year 6 H2 Biology 9648 Paper 3


Preliminary Examination 2013
1447

1. (d) Suggest why several probes have to be used in order for DNA fingerprinting
to be effective. [2]

DNA fingerprinting is often used in paternity testing. This involves the investigation of
various Variable Number of Tandem Repeat (VNTR) loci in the individuals genome. The
table below shows the genotype of five individuals at each of the five loci examined.

Locus on chromosome

D19S433 D3S1358 SE33 D8S1179 D21S11

Mother 16,16 15,16 20,22 13,14 28,30

Child 16,18 16,16 22,24 13,14 28,29

Male 1 16,18 16,16 22,23 14,15 28,31

Male 2 17,18 15,16 24,25 11,12 28.30

Male 3 15,18 16,16 19,24 12,13 29,31


-

(e) Define Variable Number of Tandem Repeat. [2]

(f) (i) State the male(s) who may possibly be the father of the child. [1]

River Valley High School Pg 5 of 17 Year 6 H2 Biology 9648 Paper 3


Preliminary Examination 2013
1448

1. (f) (ii) Explain why the test above cannot conclude with absolute certainty
that a given male is the father of the child. [1]

[Total: 15]

River Valley High School Pg 6 of 17 Year 6 H2 Biology 9648 Paper 3


Preliminary Examination 2013
1449

2. In humans, the gene RPE65 encodes a protein responsible for regenerating visual
pigment in rod and cone cells after they have been exposed to light. A recessive allele of
this gene causes impaired vision from birth, progressing to complete blindness in early
adulthood. This condition is called LCA.
In 2008, trials were carried out into the possibility and safety of treating LCA using gene
therapy.

(a) Suggest and explain why LCA is suitable for treatment using gene therapy. [3]

Six adults with this condition were used in the study. Genetically modified adenoviruses (a
type of virus that can cause respiratory infections) were used as vectors. The vectors were
injected beneath the retina of one eye of each of the participants.

(b) Suggest two ways in which the genome of the adenoviruses used as vectors
would differ from that of normal adenoviruses. [2]

River Valley High School Pg 7 of 17 Year 6 H2 Biology 9648 Paper 3


Preliminary Examination 2013
1450

2. Improvements were found in the vision of all the participants, but the small number in the
trials made most of these improvements not statistically significant.

(c) Suggest why these trials were designed to include such a small number of
participants. [2]

Scientists have developed stem cell therapy for retinal disease in mouse. The adult stem
cells from donors form functional photoreceptors after transplantation in the adult diseased
mouse retina. The immune system of the mouse was suppressed to prevent them from
rejecting the foreign stem cells.

(d) Adult stem cells such as these are described as multipotent.


Explain what is meant by the term multipotent. [1]

(e) Suggest how it is possible that adult stem cells could differentiate into the
range of cell types needed for repairing retina. [2]

River Valley High School Pg 8 of 17 Year 6 H2 Biology 9648 Paper 3


Preliminary Examination 2013
1451

2. (f) Suggest the advantages of using stem cells from donors as compared to the
use of adenoviruses in (b). [2]

[Total: 12]

River Valley High School Pg 9 of 17 Year 6 H2 Biology 9648 Paper 3


Preliminary Examination 2013
1452

3. Rice is the staple diet in many parts of the world. Rice plants produce the precursor of
vitamin A, -carotene, in their green tissues, but not in the edible endosperms of their
seeds. Adequate concentrations of vitamin A give protection from night blindness. Higher
concentrations act as an antioxidant that may give some protection from cancer and heart
disease.
Golden rice, which contains -carotene, was developed in Switzerland by genetically
modifying rice using genes from a daffodil (a flowering plant) and a bacterium. The rice
produces seeds containing -carotene.
Fig. 3.1 shows the DNA sequence used.

pro psy ter pro crt 1 ter pro Hyg resist ter

Key
pro - start site for polymerase enzymes
ter - end signal for polymerase enzymes
psy - gene from daffodil (a flowering plant)
crt 1 - gene from bacterium
Hyg resist - antibiotic resistance gene from a bacterium
Fig. 3.1

The following steps were taken to produce golden rice:


Step 1: a length of DNA (shown in Fig. 3.1) was made including a rice endosperm-
specific promoter, psy gene, crt 1 gene and Hyg resist gene,
Step 2: copies of this length of DNA were inserted into plasmids from the bacterium
Agrobacterium tumefaciens,
Step 3: A. tumefaciens containing the plasmids were mixed with rice embryos and
cultured in a medium containing the antibiotic Hyg,
Step 4: the embryos growing successfully on Hyg were grown into plantlets and then
plants.

(a) Describe the role of the rice endosperm-specific promoter that was added to
psy and crt 1 in Step 1. [2]

River Valley High School Pg 10 of 17 Year 6 H2 Biology 9648 Paper 3


Preliminary Examination 2013
1453

3. (b) Explain how a length of DNA can be inserted into a plasmid in Step 2. [3]

(c) Explain the role of the Hyg resistance gene in this procedure. [1]

Field trials of genetically engineered crops, such as golden rice, hope to identify
any risks to the environment or to human health of growing and eating the crops.

(d) Suggest:

(i) one possible risk to the environment of growing a genetically


engineered crop; [1]

(ii) one possible risk to human health of eating a genetically engineered


crop. [1]

River Valley High School Pg 11 of 17 Year 6 H2 Biology 9648 Paper 3


Preliminary Examination 2013
1454

3. A group of adult volunteers were given a 500 g of labelled golden rice dose for three days
together with 10 g butter. Each gram of golden rice contains 1.6 ug of -carotene. Serum
samples from the volunteers were analysed for golden rice -carotene. The result of the
analysis was shown in Fig. 3.2.

Fig. 3.2

(e) (i) With reference to Fig. 3.2, explain the change in labelled -carotene. [2]

(ii) Suggest why the volunteers need to consume 10 g of butter together


with the labelled golden rice. [1]

(iii) Suggest why children are not included in the study. [1]

River Valley High School Pg 12 of 17 Year 6 H2 Biology 9648 Paper 3


Preliminary Examination 2013
1455

3. The daily recommended intake of -carotene is 2 mg.

(f) Suggest if the daily intake of golden rice by children will overcome the
problem of vitamin A deficiency. Explain your answer. [1]

[Total: 13]

River Valley High School Pg 13 of 17 Year 6 H2 Biology 9648 Paper 3


Preliminary Examination 2013
1456

4. Planning Question

You are required to plan, but not carry out, an investigation into the effect of the
independent variable, copper sulphate concentration, on amylase.
Amylase catalyses the hydrolysis of starch to glucose.
Copper sulfate can inhibit amylase.
The extent of inhibition depends on the concentration of the copper sulfate solution. A
student investigated the inhibition of amylase at concentrations of copper sulfate solution
greater than 3.0% and found that the enzyme was completely inhibited.
The student suggested the hypothesis:
Concentrations of copper sulfate solution below 3.0% will continue to inhibit the
enzyme.
You are required to investigate this hypothesis by carrying out a serial dilution of copper
sulfate solution which reduces the concentration by ten-fold between each successive
dilution.
Using this information and your own knowledge, design an experiment to test the students
hypothesis.
Your plan should:
have a clear and helpful structure such that the method you use is able to be repeated
by anyone reading it,
be illustrated by relevant diagrams, if necessary,
identify the independent and dependent variables,
describe the method with the scientific reasoning used to decide the method so that
the results are as accurate and reliable as possible,
include layout of results tables and graphs with clear headings and labels,
use the correct technical and scientific terms,
include reference to safety measures to minimise any risks associated with the
proposed experiment.

Your planning must be based on the assumption that you have been provided with the
following equipment and materials, which you must use:
1% amylase solution,
1% starch suspension,
copper sulfate (powder),
iodine in potassium iodide solution,
any normal laboratory glassware, e.g. test-tubes, beakers, measuring cylinders, glass
rod, etc.,
syringes,
timer e.g. stopwatch,
colourimeter,
cuvette.

[Total: 12]

River Valley High School Pg 14 of 17 Year 6 H2 Biology 9648 Paper 3


Preliminary Examination 2013
1457

River Valley High School Pg 15 of 17 Year 6 H2 Biology 9648 Paper 3


Preliminary Examination 2013
1458

River Valley High School Pg 16 of 17 Year 6 H2 Biology 9648 Paper 3


Preliminary Examination 2013
1459

Free Response Question

Write your answers on the separate answer paper provided.

Your answers:
should be illustrated by large, clearly labelled diagrams, where appropriate.
must be in continuous prose, where appropriate.
must be set out in sections (a), (b) etc., as indicated in the question.

A NIL return is necessary if you have not attempted this section.

5. (a) Distinguish between a genomic DNA library and cDNA library, and outline
their formation. [8]

(b) Gene therapy clinical trials have mostly shown to be unsuccessful in terms of
therapeutic interest. Explain why. [7]

(c) The genetic manipulation of plants and animals, and their use in agriculture,
is one the most controversial scientific development of recent times. What are
some moral issues associated with the rise of such technology? [5]

[Total: 20]

River Valley High School Pg 17 of 17 Year 6 H2 Biology 9648 Paper 3


Preliminary Examination 2013
1460

RIVER VALLEY HIGH SCHOOL


YEAR 6 PRELIMINARY EXAMINATION

CANDIDATE
NAME

CENTRE INDEX CLASS


NUMBER NUMBER

H2 BIOLOGY 9648/03
Applications Paper and Planning Question 26 September 2013
Paper 3 2 hours

Additional Materials: Answer Paper

READ THESE INSTRUCTIONS FIRST

Do not open this booklet until you are told to do so.


Write your centre number, index number and name on all the work you hand in.
Write in dark blue or black pen on both sides of the paper.
You may use a soft pencil for any diagrams, graphs or rough working.
Do not use staples, paper clips, highlighters, glue or correction fluid.

Answer all questions. For Examiners Use

At the end of the examination, fasten ruled paper for 1 / 15


Question 5 separately. 2 / 12

Hand in the Question Paper, answers to Question 5 3 / 13


separately. 4 (SPA) / 12

The number of marks is given in brackets [ ] at the end of 5 / 20


each question or part question. Total

_____________________________________________________________________
This Question Paper consists of 17 printed pages.

River Valley High School Pg 1 of 18 H2 Biology 9648 Paper 3


Preliminary Examination 2013
1461

Answer all questions.

1. DNA fingerprinting is a method of identification based on an individuals DNA. In traditional


DNA fingerprinting, DNA is first isolated from cells, and then amplified by Polymerase
Chain Reaction (PCR). Fig. 1.1 illustrates the process of PCR.

Fig. 1.1

(a) (i) Taq polymerase is derived from Thermus aquaticus, a species of


bacteria that thrives in hot springs. Explain why Taq polymerase is
able to increase the amount of DNA from most species of organisms. [2]

1. Most species of organisms share the same genetic code


2. containing adenine, thymine, guanine and cytosine
3. Thus their DNA nucleotide can fit into the active site of Taq
polymerase
4. for formation of phosphoester bonds between adjacent nucleotides

River Valley High School Pg 2 of 18 Year 6 H2 Biology 9648 Paper 3


Preliminary Examination 2013
1462

1. (a) (ii) Explain how the specific nucleotide sequence of the primer is
important for its role in PCR. [2]

1. Allows for selective amplification


2. of a segment of DNA
3. via complementary base pairing
4. to 3 ends of the segment of interest

(b) Distinguish between PCR and DNA replication in human skin cells. [2]

Factor PCR DNA

Nature of primer DNA primer RNA primer

Enzyme for Taq polymerase Human DNA


polymerisation polymerase

Temperature High temperatures between Body temperature


60C to 96C of 37C

Separation of Heat Helicase


template strand

River Valley High School Pg 3 of 18 Year 6 H2 Biology 9648 Paper 3


Preliminary Examination 2013
1463

1. DNA amplified using PCR is then is cut by a specific restriction enzyme and separated
using gel electrophoresis. The DNA is then transferred to a nitrocellulose membrane,
before the addition of radioactive DNA probes for nucleic acid hybridisation. Fig. 1.2
shows the nucleic acid hybridisation process.

Fig. 1.2

1. (c) With reference to Fig. 1.2, explain the significance of the temperatures used
during nucleic acid hybridisation. [3]

82C
1. Separation of DNA strands
2. by breaking hydrogen bonds
3. between complementary base pairs
65C
4. Annealing of radioactive DNA probe
5. to DNA fragment of interest
6. via complementary base pairing
7. between GCAT and CGTA

River Valley High School Pg 4 of 18 Year 6 H2 Biology 9648 Paper 3


Preliminary Examination 2013
1464

1. (d) Suggest why several probes have to be used in order for DNA fingerprinting
to be effective. [2]

1. To examine more than one VNTR locus


2. To obtain a combination of band patterns for comparison

DNA fingerprinting is often used in paternity testing. This involves the investigation
of various Variable Number of Tandem Repeat (VNTR) loci in the individuals
genome. The table below shows the genotype of five individuals at each of the five
loci examined.

Locus on chromosome

D19S433 D3S1358 SE33 D8S1179 D21S11

Mother 16,16 15,16 20,22 13,14 28,30

Child 16,18 16,16 22,24 13,14 28,29

Male 1 16,18 16,16 22,23 14,15 28,31

Male 2 17,18 15,16 24,25 11,12 28.30

Male 3 15,18 16,16 19,24 12,13 29,31

(e) Define Variable Number of Tandem Repeat. [2]

1. DNA sequences
2. repeated in tandem
3. a variable number of times
4. at certain loci

(f) (i) State the male(s) who may possibly be the father of the child. [1]

Male 3
Reject multiple answers

River Valley High School Pg 5 of 18 Year 6 H2 Biology 9648 Paper 3


Preliminary Examination 2013
1465

1. (f) (ii) Explain why the test above cannot conclude with absolute certainty
that a given male is the father of the child. [1]

Not all VNTRs in the genome are verified as a match


Reject reference to coincidence without elaboration

[Total: 15]

River Valley High School Pg 6 of 18 Year 6 H2 Biology 9648 Paper 3


Preliminary Examination 2013
1466

2. In humans, the gene RPE65 encodes a protein responsible for regenerating visual
pigment in rod and cone cells after they have been exposed to light. A recessive allele of
this gene causes impaired vision from birth, progressing to complete blindness in early
adulthood. This condition is called LCA.
In 2008, trials were carried out into the possibility and safety of treating LCA using gene
therapy.

(a) Suggest and explain why LCA is suitable for treatment using gene therapy. [3]

1. caused by a single gene


2. caused by a recessive allele
3. delivery of, correct / dominant / normal, allele (could correct the condition)
4. ease of access to affected area
5. serious so worth the risk
6. only targets eye / no surgery needed
7. AVP

Six adults with this condition were used in the study. Genetically modified
adenoviruses (a type of virus that can cause respiratory infections) were used as
vectors. The vectors were injected beneath the retina of one eye of each of the
participants.

(b) Suggest two ways in which the genome of the adenoviruses used as vectors
would differ from that of normal adenoviruses. [2]

1. virus no longer able to cause infections


2. correct / dominant / normal, allele (of RPE65) added
3. promoter added

Improvements were found in the vision of all the participants, but the small number
in the trials made most of these improvements not statistically significant.

(c) Suggest why these trials were designed to include such a small number of
participants. [2]

1. ref. to safety / not known if the technique might have side effects
2. rare condition
3. expense
4. AVP e.g. trial to see if delivery method works

River Valley High School Pg 7 of 18 Year 6 H2 Biology 9648 Paper 3


Preliminary Examination 2013
1467

2. Scientists have developed stem cell therapy for retinal disease in mouse. The adult stem
cells from donors form functional photoreceptors after transplantation in the adult diseased
mouse retina. The immune system of the mouse was suppressed to prevent them from
rejecting the foreign stem cells.

(d) Adult stem cells such as these are described as multipotent.


Explain what is meant by the term multipotent. [1]

Multipotent cells can differentiate into a limited number of cell types (but not
into whole organism)

(e) Suggest how it is possible that adult stem cells could differentiate into the
range of cell types needed for repairing retina. [2]

1. each stem cell contains a complete genome


2. cells will be in slightly different conditions / differentiate under appropriate
conditions
3. through selective changes in genetic activity / different genes will be
expressed, in different conditions

(f) Suggest the advantages of using stem cells from donors as compared to the
use of adenoviruses in (b). [2]

1. not all of the cells in the retina has taken up the normal allele
2. while all the stem cells from a donor will contain the normal allele
3. viral vector used to introduce the normal allele elicit host immune
response
4. resulting in destruction of the vector

[Total: 12]

River Valley High School Pg 8 of 18 Year 6 H2 Biology 9648 Paper 3


Preliminary Examination 2013
1468

3. Rice is the staple diet in many parts of the world. Rice plants produce the precursor of
vitamin A, -carotene, in their green tissues, but not in the edible endosperms of their
seeds. Adequate concentrations of vitamin A give protection from night blindness. Higher
concentrations act as an antioxidant that may give some protection from cancer and heart
disease.
Golden rice, which contains -carotene, was developed in Switzerland by genetically
modifying rice using genes from a daffodil (a flowering plant) and a bacterium. The rice
produces seeds containing -carotene.
Fig. 3.1 shows the DNA sequence used.

pro psy ter pro crt 1 ter pro Hyg resist ter
Key
pro start site for polymerase enzymes
ter end signal for polymerase enzymes
psy gene from daffodil (a flowering plant)
crt 1 gene from bacterium
Hyg resist antibiotic resistance gene from a bacterium
Fig. 3.1
The following steps were taken to produce golden rice:
Step 1: a length of DNA (shown in Fig. 3.1) was made including a rice endosperm-
specific promoter, psy gene, crt 1 gene and Hyg resist gene,
Step 2: copies of this length of DNA were inserted into plasmids from the bacterium
Agrobacterium tumefaciens,
Step 3: A. tumefaciens containing the plasmids were mixed with rice embryos and
cultured in a medium containing the antibiotic Hyg,
Step 4: the embryos growing successfully on Hyg were grown into plantlets and then
plants.

(a) Describe the role of the rice endosperm-specific promoter that was added to
psy and crt 1 in Step 1. [2]

1. role of promoter is to switch on genes


2. ref to, transcription
3. only in the rice endosperm
4. not expressed anywhere else (would be a waste of energy / resources)
5. in edible part of seed

River Valley High School Pg 9 of 18 Year 6 H2 Biology 9648 Paper 3


Preliminary Examination 2013
1469

3. (b) Explain how a length of DNA can be inserted into a plasmid in Step 2. [3]

1. use restriction, enzyme / endonuclease


2. cut plasmid / open plasmid and the length of DNA
3. sticky ends / complementary bases
4. ref to anneal via hydrogen bonding / A - T / C - G
5. role of ligase, in sealing sugar-phosphate backbone / catalyses formation
of phosphoester bond / AW

(c) Explain the role of the Hyg resistance gene in this procedure. [1]

1. To enable selection of the transgenic cells / cells that have taken up the
plasmid
2. only the cells with the new DNA can grow in the presence of the antibiotic

Field trials of genetically engineered crops, such as golden rice, hope to identify
any risks to the environment or to human health of growing and eating the crops.

(d) Suggest:

(i) one possible risk to the environment of growing a genetically


engineered crop; [1]

gene transfer to other rice


contamination of other crop (e.g. ref to organic crop)
gene transfer to wild relative
recipient plant outcompetes others / ref to superweed
toxic to non pests
AVP must be something specific not a vague answer
any one

River Valley High School Pg 10 of 18 Year 6 H2 Biology 9648 Paper 3


Preliminary Examination 2013
1470

3. (d) (ii) one possible risk to human health of eating a genetically engineered
crop. [1]

allergy
long term toxicity
antibiotic resistance of gut bacteria if antibiotic marker used
gene transfer to, virus to human cells / gut bacteria, with unknown
effect
AVP
any one

A group of adult volunteers were given a 500 g of labelled golden rice dose for
three days together with 10 g butter. Each gram of golden rice contains 1.6 ug of -
carotene. Serum samples from the volunteers were analysed for golden rice -
carotene. The result of the analysis was shown in Fig. 3.2.

Fig. 3.2

(e) With reference to Fig. 3.2, explain the change in labelled -carotene. [2]

(i) Increase from day 0 to 3; due to intake


Decrease from day 3 to 25; due to use of -carotene in vitamin A
synthesis

(ii) Suggest why the volunteers need to consume 10 g of butter together


with the labelled golden rice. [1]

-carotene is fat soluble; facilitate intestinal uptake

River Valley High School Pg 11 of 18 Year 6 H2 Biology 9648 Paper 3


Preliminary Examination 2013
1471

3. (e) (iii) Suggest why children are not included in the study. [1]

Frequent blood draws are not suitable for children

(f) The daily recommended intake of -carotene is 2 mg.


Suggest if the daily intake of golden rice by children will overcome the
problem of vitamin A deficiency. Explain your answer. [1]

No
Children consume less than 150 grams of rice a day/ less than adult

[Total: 13]

River Valley High School Pg 12 of 18 Year 6 H2 Biology 9648 Paper 3


Preliminary Examination 2013
1472

4. Planning Question

You are required to plan, but not carry out, an investigation into the effect of the
independent variable, copper sulphate concentration, on amylase.
Amylase catalyses the hydrolysis of starch to glucose.
Copper sulfate can inhibit amylase.
The extent of inhibition depends on the concentration of the copper sulfate solution. A
student investigated the inhibition of amylase at concentrations of copper sulfate solution
greater than 3.0% and found that the enzyme was completely inhibited.
The student suggested the hypothesis:
Concentrations of copper sulfate solution below 3.0% will continue to inhibit the
enzyme.
You are required to investigate this hypothesis by carrying out a serial dilution of copper
sulfate solution which reduces the concentration by ten-fold between each successive
dilution.
Using this information and your own knowledge, design an experiment to test the students
hypothesis.
Your plan should:
have a clear and helpful structure such that the method you use is able to be repeated
by anyone reading it,
be illustrated by relevant diagrams, if necessary,
identify the independent and dependent variables,
describe the method with the scientific reasoning used to decide the method so that
the results are as accurate and reliable as possible,
include layout of results tables and graphs with clear headings and labels,
use the correct technical and scientific terms,
include reference to safety measures to minimise any risks associated with the
proposed experiment.

Your planning must be based on the assumption that you have been provided with the
following equipment and materials, which you must use:
1% amylase solution
1% starch suspension
copper sulfate (powder)
iodine in potassium iodide solution
any normal laboratory glassware, e.g. test-tubes, beakers, measuring cylinders, glass
rod, etc.,
syringes,
timer e.g. stopwatch,
colourimeter,
cuvette.
[Total: 12]

River Valley High School Pg 13 of 18 Year 6 H2 Biology 9648 Paper 3


Preliminary Examination 2013
1473

Introduction i.e. explanation of the dependent and independent variables involved [I=max 2]:
1. Amylase catalyses the hydrolysis of starch into glucose
2. Amylase has a specific active site
3. Starch fits into the active site to form an E-S complex
4. Resulting in the breakage of the glycosidic bonds
5. Copper sulfate inhibit amylase, prevent the binding of starch to the active site
6. The concentration of starch left can be measured by addition of iodine reagent
7. Measuring the colour intensity using a colourimeter
8. The intensity is proportional to the concentration of starch left and
9. inversely proportional to the rate of hydrolysis
Note: No mark for wrong concept

Expected trend [T=max 2]


10. As concentration of copper sulfate increases from 0% to 3.0%, the rate decreases
11. More enzymes are inactivated
12. Less enzyme-substrate complexes formed
13. Increased colour intensity
14. Beyond 3.0% copper sulfate
15. All the enzymes are inhibited
16. No reaction occurs

Experimental variables [V=max 2]


17. Independent : Concentration of copper sulfate
variable Range: 0.0003% to 3.0%
18. Dependent variable : Colour intensity in term of absorbance
19. Controlled variables :

Controlled Variable Appropriate Quantity


a volume of starch suspension used 5 cm3
b volume of amylase used 1 cm3
c volume of copper sulfate used 1 cm3
d temperature 37oC / optimum temperature
i.e. equilibrate amylase & starch to 37 C
separately using water bath for 1 minute
e volume of iodine reagent added 0.5 cm3

River Valley High School Pg 14 of 18 Year 6 H2 Biology 9648 Paper 3


Preliminary Examination 2013
1474

Method i.e. description of the method used including the scientific reasoning behind the method
/ relevant, clearly labelled diagrams: [M=max 3]
20. Preparation of 3.0% copper sulfate solution: dissolve 3.0 g of copper sulfate in 100 cm3
of distilled water.
21. Dilute the copper sulphate stock solution to give 10 cm3 each of the following
concentrations: 0.0003%, 0.003%, 0.03% and 0.3%.
22. Add 1 cm3 of 3.0% copper sulfate into the test tube labelled as 3.0%
23. Put 5 cm3 of starch suspension into the same test-tube.
24. Put 1 cm3 of amylase into the same test-tube.
25. Place the test tube in a water bath maintained at 37 C.
26. Mix and start the stopwatch.
27. At the end of 10 minutes, add 0.5 cm3 of iodine reagent and mix well.
28. Transfer 1 cm3 of the mixture to the cuvette and measure the colour intensity using the
colourimeter.
29. Record the reading in an appropriate table.
30. Repeat step 23 30 for the rest of copper sulfate concentration 0.0003%, 0.003%,
0.03%, 0.3% and control.

25. Water bath maintained at 37oC


Reject: direct heating

22. 1 cm3 copper sulfate


23. 5 cm3 starch suspension
24. 1 cm3 amylase

31. ensure that there are 3 replicates for each concentration of copper sulfate
32. Repeat the entire experiment twice times

Control setup [C=max 1]


3
33. A negative control is subjected to the same factors (i.e. 5 cm starch suspension and 1
cm3 amylase) as that for the experiment, except the copper sulphate solution is replaced
with equivolume of distilled water.
34. It is expected that yellow colour is observed upon addition of iodine reagent at the end of
incubation time. This proves that it is indeed copper sulfate that inhibits the amylase

River Valley High School Pg 15 of 18 Year 6 H2 Biology 9648 Paper 3


Preliminary Examination 2013
1475

Data manipulation i.e. Include layout of results tables and graphs with clear headings and
labels [D=max 2]

35. Concentration of 36. Colour intensity/ Absorbance


copper sulfate, C / %
A1 A2 A3
0.0003
0.003
0.03
0.3
3.0
control

37. Absorbance

38. Concentration of copper sulfate, C / %

Safety precautions i.e. relevant risks and precautions taken: [S=max 1]


39. Amylase and iodine reagent are skin irritant. Wear glove and goggles
40. Copper sulfate is a harmful irritant. Wear gloves and goggles
41. AVP

River Valley High School Pg 16 of 18 Year 6 H2 Biology 9648 Paper 3


Preliminary Examination 2013
1476

Free Response Question

Write your answers on the separate answer paper provided.

Your answers:
should be illustrated by large, clearly labelled diagrams, where appropriate.
must be in continuous prose, where appropriate.
must be set out in sections (a), (b) etc., as indicated in the question.

A NIL return is necessary if you have not attempted this section.

5. (a) Distinguish between a genomic DNA library and cDNA library, and outline
their formation. [8]

Formation
gDNA
1. Entire genome is cleaved
2. with a specific restriction endonuclease
3. Resulting fragments are inserted into plasmids before introducing into bacteria
cDNA
4. Total RNA
5. converted into single-stranded cDNA
6. Using reverse transcriptase
7. Resulting cDNA molecules are inserted into plasmids before introducing into bacteria

Differences

Feature Genomic DNA library cDNA library

8. DNA Contains DNA fragments Contains DNA fragments


fragments in representing the entire representing only the coding region
clones genome of a genome

9. Introns Introns are included in the No introns present in the clones


clones (intron sequences removed by RNA
splicing)

8. Frequency All genes are represented Genes that are transcribed


equally in the library abundantly will be represented
more frequently in the library

9. Number of Larger number of clones Smaller no. of clones to screen (as


clones to screen it contains only the coding regions)

10. Regulatory Presence of regulatory Absence of regulatory sequences


sequences sequences

12. AVP

River Valley High School Pg 17 of 18 Year 6 H2 Biology 9648 Paper 3


Preliminary Examination 2013
1477

(b) Gene therapy clinical trials have mostly shown to be unsuccessful in terms of
therapeutic interest. Explain why. [7]

1. Difficult to get DNA to integrate into the target cell genome


2. Results can be short-lived / patients need to be treated on a frequent basis
3. Problem with controlling the activity of gene expression
4. Risk of stimulating immune response
5. Virus vector may regain / develop virulence
6. Incorrect insertion of gene may cause named / described problem to patient
7. Possible problem with finding vector for large gene
8. Many genetic diseases are a result of presence of many defective genes / impossible
to introduce many genes at same time
9. Problem dealing with, dominant condition / non dividing cells
10. Toxicity / low efficiency of liposomes
11. AVP

(c) The genetic manipulation of plants and animals, and their use in agriculture,
is one the most controversial scientific development of recent times. What are
some moral issues associated with the rise of such technology? [5]

1. Violation of natural organisms intrinsic values with the mixing of genes among species
/ tampering of genes
2. Animal welfare could be adversely affected due to little concern about whether animals
could withstand the additional burden of increased milk / meat production
3. Controversies on patent of GMO as patent of transgenic plants / animals reduces them
to the level of objects
4. Religious / dietary concerns with consuming animal genes in plants / relevant
examples
5. Labelling of GM food is not mandatory in some countries thus lack of awareness for
those who feel strongly against consumption of foreign DNA incorporated into their
food
6. AVP

[Total: 20]

River Valley High School Pg 18 of 18 Year 6 H2 Biology 9648 Paper 3


Preliminary Examination 2013
1478

SERANGOON JUNIOR COLLEGE


JC2 PRELIMINARY EXAMINATION 2013
Paper 1
BIOLOGY
Higher 2
Wednesday
28 August 2013

1 hour 15 minutes
Additional materials:
Optical Mark Sheet

INSTRUCTIONS TO CANDIDATES
Write you name, CG and index number in the spaces at the top of this page.
On the Optimal Mark Sheet, enter your name, subject title, test name, class. For your
index number, enter your full NRIC number. Shade the corresponding lozenges on
the OMS according to the instructions given by the invigilators.
There are forty questions in this paper. Answer all questions. For each question,
there are four possible answers, A, B, C, D. Choose the one you consider correct
and record your choice in soft pencil on the OMS.

AT THE END OF THE EXAMINATION, HAND IN BOTH THE OMS AND


QUESTION PAPER.

INFORMATION FOR CANDIDATES


Each correct answer will score one mark. A mark will not be deducted for a wrong
answer.
Any rough working should be done on the question paper.

___________________________________________________________________
This question paper consists of 24 printed pages.

1
1479

1. A population of cells that produces secretory proteins is transferred into a


medium containing only radioactively-labelled amino acids. In which of the
following cell structures will radioactivity first be significantly detected?

A endoplasmic reticulum

B nucleus

C phagocytic vesicle

D secretory vesicle

2. Fig. 2 below shows two lipid molecules, Lipid A and Lipid B.

Lipid A

Lipid B

Fig. 2

Which of the following correctly indicates a similarity and difference


between Lipids A and B?

Similarity Difference
Lipid A Lipid B
A Presence of strong ester Presence of Absence of
bonds. phosphodiester phosphodiester
bond bond
B Hydrogen bonding between Presence of two Presence of three
hydrocarbon chains ester bonds ester bonds
C Presence of straight Amphipathic Hydrophobic
hydrocarbon chains
D Complete hydrolysis Contains Does not contain
releases three water phosphorus atom phosphorus atom
molecules

2
1480

3. Which of the following correctly indicates a difference between collagen


and haemoglobin?

Collagen Haemoglobin
A Contains mostly hydrophobic Contains mostly hydrophilic
amino acids amino acids
B Stabilised largely by numerous
Stabilised largely by numerous
hydrogen bonds ionic and hydrogen bonds, and
hydrophobic interactions
C Has primary and secondary Has primary, secondary,
structures only tertiary and quaternary
structures
D Three subunits wind together Each subunit has an active site
to form triple helix which binds haem

4. Which of the following accounts for why starch and glycogen are good
energy stores in living organisms?

A A large amount of energy is released when (14) glycosidic


bonds in starch and glycogen are hydrolysed.

B The occurrence of branch points every 26 to 30 residues in


starch and glycogen provides many branch ends from which
digestion can begin.

C The projection of hydroxyl groups into the helical structures of


starch and glycogen causes both to be insoluble in water.

D Starch and glycogen are highly reactive macromolecules that


can be rapidly broken down when required.

5. Which of the following does not describe the active site of an enzyme?

A The active site is usually a pocket comprising several amino


acids.

B The active site binds with an inhibitor which has a


complementary shape.

C In anabolic reactions, the active site brings substrates close


together and catalyses bond formation.

D Beyond the optimum temperature, the active site is first


denatured before the rest of the enzyme structure is denatured.

3
1481

6. Which of the following regarding homologous chromosomes is correct?

A The X and Y chromosomes in human males are not homologous


chromosomes.

B Homologous chromosomes are not present in a cell undergoing


mitosis.

C Centromeres of homologous chromosomes occupy the same


position that is approximately equidistant from both ends of the
linear chromosomes.

D Homologous chromosomes usually carry the same alleles at


corresponding gene loci.

7. Fig. 7 below shows measurements during one mitotic cell cycle.

Fig. 7

Which stage of mitosis begins at X and which measurements are


illustrated by curves 1 and 2?

Stage beginning Distance between Distance between


at X centromeres of the centromeres of
chromosomes and sister chromatids
poles of spindle
A Metaphase 1 2
B Metaphase 2 1
C Anaphase 1 2
D Anaphase 2 1

4
1482

8. Which of the following does not explain how meiosis leads to genetic
variation?

A At metaphase II, the orientation of a particular chromosome to


the poles is not affected by the orientation of the other
chromosomes along the equator.

B By the end of meiosis I, each daughter nucleus contains either


the paternal or maternal copy of a chromosome.

C Homologous regions of DNA are exchanged within a bivalent


after synapsis has occurred.

D The chromatids that are separating during anaphase II may be


genetically different.

9. Several aspects of DNA replication are listed below:

1. Synthesis of complementary RNA primer


2. Base-pairing between DNA nucleotides
3. Removal of mismatched DNA nucleotides
4. Initiation only at origin of replication

Which of the above contributes directly to the accuracy of DNA


replication?

A 1 and 3 only.

B 2 and 3 only.

C 3 and 4 only.

D 2, 3 and 4 only.

5
1483

10. A simplified representation of a replication bubble is shown in Fig. 10


below. Parental strands 1 and 2 and the growing daughter strands X and
Y are indicated.

Parent
strand 1
Daughter Daughter
strand X strand Y

Parent
strand 2

Fig. 10

Which of the following statements about the syntheses of Daughter


strands X and Y is correct?

A Strands X and Y are synthesised away from their respective


replication forks.

B Strand X is synthesised continuously while Strand X is


synthesised in the form of Okazaki fragments.

C Strand X is synthesised in the 5 3 direction while Strand Y is


synthesised in the 3 5 direction.

D DNA ligase will eventually catalyse the fusion of Strand X with


Strand Y.

11. Translation of the information on an mRNA into the amino acid sequence
of a polypeptide chain is an energy-requiring process, involving further
processes such as:

1. Amino acid activation


2. Assembly of translation initiation complex
3. Attachment of amino acid to growing polypeptide chain
4. Translocation of ribosome along mRNA

In which of the above processes is energy in the form of ATP required?

A 1 only.

B 1 and 3 only.

C 1, 3 and 4 only.

D All of the above.

6
1484

12. The first five DNA triplets that code for a particular protein is shown below:

3 CAC GGA AGC CCA GAA 5

The genetic information in the sequence above is eventually converted


into a specific amino acid sequence according to Fig. 12 below.

Fig. 12

With the aid of Fig. 12, what is the sequence of the protein encoded by
the DNA sequence above?

A His Gly Ser Pro Glu

B Val Pro Ser Gly Leu

C Lys Thr Arg Arg His

D Phe Try Ala Ser Val

7
1485

13. Which of the following does not occur after lambda phage infection of an
E. coli host cell?

A Lambda phage DNA is preferentially replicated compared to


host DNA after phage DNA is incorporated into host genome.

B The expression of most lambda phage genes is repressed


immediately after prophage formation.

C Lambda phage DNA temporarily circularises immediately after


host cell entry.

D The viral head and tail structures remain on the surface of the
host cell, and may be eventually degraded.

14. Fig. 14 below is a simple representation of the structure of HIV (not drawn
to scale). Structures W to Z are labelled as shown.

W
X

Fig. 14

Which of the following does not correctly describe the function of the
respective structures?

A Structure W binds with glycoprotein receptors on the host cell.

B Structure X synthesises double-stranded DNA.

C Structure Y causes the insertion of viral RNA into host genome.

D Structure Z promotes fusion with host cell surface membrane.

8
1486

15. The process of bacterial conjugation is illustrated in Fig. 15 below.

Fig. 15

The following properties of the F plasmid are demonstrated in Fig. 15


above except

A the F plasmid is heritable.

B the F plasmid replicates semi-conservatively.

C the F plasmid has an origin of replication and transfer.

D the F plasmid carries the genes encoding the pilus proteins.

9
1487

16. Several possible properties of operons are listed below:

1. Inducible
2. Contains more than one structural gene
3. Can be negatively controlled by repressor
4. Can be positively controlled by activator

Which of the above describe(s) both the lac and trp operons?

A 2 only.

B 2 and 3 only.

C 1, 2 and 3 only.

D All of the above.

17. Which of the following is/are mostly likely to contain DNA sequences that
are tandemly repeating?

1. Centromere
2. Origin of replication
3. Promoter
4. Restriction Site

A 1 only.

B 1 and 3 only.

C 1, 2 and 3 only.

D All of the above.

10
1488

18. In transcription initiation, several steps are involved:

1. RNA polymerase II binds.


2. Transcription factor II B binds.
3. Histone proteins are acetylated at specific sites.
4. RNA polymerase II is phosphorylated on multiple sites.
5. TATA binding protein of transcription factor II D recognises TATA
box.

Which of the following indicates the correct sequence of the steps above?

A 2, 3, 5, 4, 1

B 2, 3, 5, 1, 4

C 3, 2, 5, 4, 1

D 3, 5, 2, 1, 4

19. The following properties are common to both eukaryotic and prokaryotic
genomes, except

A genetic material being in the form of double-stranded DNA.

B the association of genetic material with histone proteins.

C the presence of one or more origins of replication.

D the control of gene expression by activators and repressors.

20. Which of the following is true of cancers?

A A cancer cell will always eventually develop into a tumour.

B Cancer cells are likely to have longer-than-usual telomeres


despite having inactivated telomerases.

C A cell that has a copy of the p53 tumour suppressor gene


inactivated can be considered to be cancerous.

D When a copy of the ras proto-oncogene is activated into an


oncogene in a normal cell, cancer immediately develops.

11
1489

21. Two types of plumage (feathers) exist in chickens: hen-feathering and


cock-feathering. The type of plumage results from the expression of an
autosomal gene locus that carries two alleles:

Allele H (hen-feathering) is dominant in both female and male


chickens.
Allele h (cock-feathering) is recessive, and the phenotype is
expressed only in male chickens.

Which of the following would be the result of a mating between a cock-


feathering male chicken, and a hen-feathering female chicken which is
heterozygous at the gene locus?

A The probability of a male offspring being cock-feathering is 50%


and that of a female offspring being hen-feathering is 50%.

B The probability of a male offspring being cock-feathering is 50%


and that of a female offspring being hen-feathering is 100%.

C The probability of a male offspring being cock-feathering is


100% and that of a female offspring being hen-feathering is 50%.

D The probability of a male offspring being cock-feathering is


100% and that of a female offspring being hen-feathering is
100%.

12
1490

22. The pedigree chart below shows the inheritance of a genetic disease in a
family.

Key:

What is the nature of the allele that causes this disease?

A autosomal dominant

B sex-linked dominant

C autosomal recessive

D sex-linked recessive

13
1491

23. Wheat kernel colour is dependent upon a biochemical reaction that


converts a white precursor into a coloured pigment, when the products of
both gene A and gene B are both present.

product of gene A
white precursor coloured kernel
product of gene B

When 2 breeds of pure-breeding white wheat kernel are crossed, all the
F1 offspring show coloured kernel. When the F1 offspring are selfed, what
is the probability of the F2 offspring having the same phenotype as the F1
phenotype?

A 1 in 16

B 3 in 16

C 4 in 16

D 9 in 16

14
1492

24. Fig. 24 below shows a human karyotype.

Fig. 24

Which of the following is evident from Fig. 24?

A The karyotype indicates a female patient with a single


chromosomal aberration.

B The karyotype indicates a male patient with a single


chromosomal duplication.

C The karyotype indicates a female patient with two chromosomal


duplications.

D The karyotype indicates a male patient with two chromosomal


aberrations.

15
1493

25. In an experiment, chloroplast extracts are first treated with a chemical that
snatches away the electron that is accepted by the electron acceptor in
photosystem I. The extracts are then treated with 2 hours of light and are
provided with ample carbon dioxide and water.

Which of the following correctly shows the products that are formed after
the experiment?

O2 Reduced
ATP Glucose
NADP
A + +
B + + +
C +
D +
Key: (+) = present, () = absent

16
1494

26. Fig. 26 below illustrates several metabolic processes that occur within the
body. Table 26 contains three statements that describe some of these
processes.

Glycogen Glucose

P Q
Fructose 1,6-bisphosphate

Pyruvate

S T

Carbon dioxide Lactic acid


and water

Fig. 26

Statement 1 NAD+ is regenerated without the use of the electron


transport system.
Statement 2 ATP is synthesised via substrate-level phosphorylation.
Statement 3 It can occur under anaerobic conditions.
Table 26

Which of the following correctly relates processes P, Q, R, S, T to


statements 1, 2 and 3?

Statement 1 Statement 2 Statement 3


A R S T
B T R P
C S Q S
D Q S R

17
1495

27. Which of the following is necessary for an organism to respond to


changes in its internal environment?

A Positive feedback from effector to receptor.

B Negative feedback from receptor to effector.

C A communication system involving nerve cells.

D A communication system between receptors and effectors.

28. Certain drugs act at the synapses and affect the action of neurotransmitter
substances. The table below shows the effects of four different drugs on a
synapse.

Drug Effect
1 Uncontrolled release of acetylcholine
2 Competes with acetylcholine at the receptor sites
3 Inhibits the enzyme acetylcholinesterase
4 Inhibits the opening of voltage-gated sodium ion channels

Which combination of drugs will reduce the possibility of the formation of


an excitatory post-synaptic potential?

A 2 and 4 only.

B 3 and 4 only.

C 1, 2 and 4 only.

D 2, 3 and 4 only.

18
1496

29. The Guinea baboon and King colobus are two of the many Old World
monkeys that exist.

Guinea baboon King colobus

The classification of these monkeys is shown below.

Animalia
Chordata
Mammalia
Primates
Cercopithecidae (sub: Cercopithecinae)
Papio
Papio papio (Guinea baboon)

Animalia
Chordata
Mammalia
Primates
Cercopithecidae (sub: Colobinae)
Colobus
Colobus polykomos (King colobus)

Which of the following conclusions is consistent with the information


above?

A Both belong to the same class but different families.

B Both belong to the same genus but different classes.

C Both belong to the same family but different genera.

D Both belong to the same species but different orders.

19
1497

30. The table shows the number of estimated nucleotide substitutions that
have occurred since the divergence of seven species a to g.

b c d e f g
a 39 72 128 126 159 269
b 81 130 128 158 268
c 129 127 157 267
d 56 154 271
e 151 268
f 273

Which of the following phylogenetic trees best shows the relationship


among these seven species?

A B

C D

20
1498

31. Which of the following describes the process of natural selection?

A Different rates of reproductive success of different genotypes.

B Spontaneous occurrence of advantageous mutations.

C Change from simple to more complex organisms over time.

D Change in the size of population over time.

32. Which of the following statements is not consistent with the neutral theory
of molecular evolution?

A Genetic drift is more significant in explaining the high degree of


genetic variation in a population than natural selection.

B The frequency of a particular allele in the gene pool of a


population may decrease over time even if there is no selection
pressure acting on it.

C The frequency of a particular allele in the gene pools of two


populations is likely to be identical over time even if these
populations are geographically isolated.

D If all mutations are selectively neutral, differences in DNA


sequences across different species can serve as a molecular
clock to chart evolution.

33. Which of the following statements regarding restriction enzymes is


incorrect?

A Host DNA is prevented from digestion by restriction enzymes


due to specific methylation of DNA.

B A restriction enzyme can create either sticky or blunt ends


depending on the chemical conditions present.

C Cleavage by a restriction enzyme involves the breaking of two


covalent phosphodiester bonds.

D Restriction enzymes protect bacteria against invading viruses by


preventing the replication of viral DNA.

21
1499

34. Which of the following correctly indicates a difference between a genomic


and a cDNA library?

Genomic library cDNA library


A Created from primary transcripts Created from mature mRNAs
B Varies with stage of organism Varies with cell type
development
C Creation involves use of Creation does not involve use of
restriction enzymes restriction enzymes
D Contains promoter regions of Does not contain promoter
genes regions of genes

35. Which of the following is an outcome of the Human Genome Project that
has ethical implications?

A Screening of the genetic make-up of newborn infants for


susceptibility to certain key diseases.

B Creation of customised medicines that are potentially more


expensive to produce than traditional drugs.

C Consideration of a suspects genetic pre-disposition to violent


behaviour in criminal trials.

D The free availability and accessibility of the complete sequence


of the human genome on the Internet.

36. Totipotency is demonstrated when

A an embryonic stem cell divides and differentiates.

B cancer cells give rise to heterogeneous cell types.

C an isolated plant cell develops into a normal adult plant.

D a hematopoietic stem cell differentiates into a lymphocyte.

22
1500

37. The ras oncogene is involved in a variety of human cancers. DNA isolated
from a number of normal and cancerous human tissues were digested
with EcoRI. A Southern blot was performed and the membrane was
probed with radioactive-labelled cloned ras DNA. After exposure to x-ray
film, the resulting autoradiograph is shown below.

1 2 3 4

10 kb

5 kb

more than 2 copies / genome


2 copies / genome
1 copy / genome

1 kb

Lane Sample
1 white blood cells from a healthy human
2 lymphoma cells (cancerous)
3 bladder carcinoma cells (cancerous)
4 sarcoma cells (cancerous)

Based on the experimental results above, which of the following


conclusions can be deduced?

A The result for tissue 2 can be explained by point mutations that


caused the removal of an EcoRI site and the creation of another
within the probe DNA.

B In tissue 3, there was no mutation within the ras coding region.

C In tissue 3, the mutation did not affect existing EcoRI sites within
the ras gene.

D Some of the cancer cells in tissue 4 are likely to be polyploid.

23
1501

38. Which of the following is not an advantage of liposome-mediated gene


therapy over retrovirus-mediated gene therapy?

A No insertional mutagenesis.

B General targeting of various cells.

C Lower stimulation of immune system.

D Possible delivery of all types of nucleic acids.

39. Which of the following methods can be used to introduce a foreign gene
into a plant cell?

1. By Agrobacterium-mediated transfection
2. By microinjection of naked DNA
3. Using a phage delivery vector
4. Using a gene gun

A 1 and 3 only.

B 1, 2 and 4 only.

C 1, 3 and 4 only.

D All of the above.

40. There is concern regarding the escape of GM salmon into the wild
because they

A may replace wild salmon populations.

B may not survive as well in the wild.

C are prone to further mutations.

D will produce sterile offspring with wild salmon.

24
1502

SERANGOON JUNIOR COLLEGE


JC2 PRELIMINARY EXAMINATION 2013
Paper 1
BIOLOGY
Higher 2
Wednesday
28 August 2013

1 hour 15 minutes
Additional materials:
Optical Mark Sheet

INSTRUCTIONS TO CANDIDATES
Write you name, CG and index number in the spaces at the top of this page.
On the Optimal Mark Sheet, enter your name, subject title, test name, class. For your
index number, enter your full NRIC number. Shade the corresponding lozenges on
the OMS according to the instructions given by the invigilators.
There are forty questions in this paper. Answer all questions. For each question,
there are four possible answers, A, B, C, D. Choose the one you consider correct
and record your choice in soft pencil on the OMS.

AT THE END OF THE EXAMINATION, HAND IN BOTH THE OMS AND


QUESTION PAPER.

INFORMATION FOR CANDIDATES


Each correct answer will score one mark. A mark will not be deducted for a wrong
answer.
Any rough working should be done on the question paper.

___________________________________________________________________
This question paper consists of 24 printed pages.

1
1503

1. A population of cells that produces secretory proteins is transferred into a


medium containing only radioactively-labelled amino acids. In which of the
following cell structures will radioactivity first be significantly detected?

A endoplasmic reticulum

B nucleus

C phagocytic vesicle

D secretory vesicle

2. Fig. 2 below shows two lipid molecules, Lipid A and Lipid B.

Lipid A

Lipid B

Fig. 2

Which of the following correctly indicates a similarity and difference


between Lipids A and B?

Similarity Difference
Lipid A Lipid B
A Presence of strong ester Presence of Absence of
bonds. phosphodiester phosphodiester
bond bond
B Hydrogen bonding between Presence of two Presence of three
hydrocarbon chains ester bonds ester bonds
C Presence of straight Amphipathic Hydrophobic
hydrocarbon chains
D Complete hydrolysis Contains Does not contain
releases three water phosphorus atom phosphorus atom
molecules

2
1504

3. Which of the following correctly indicates a difference between collagen


and haemoglobin?

Collagen Haemoglobin
A Contains mostly hydrophobic Contains mostly hydrophilic
amino acids amino acids
B Stabilised largely by numerous
Stabilised largely by numerous
hydrogen bonds ionic and hydrogen bonds, and
hydrophobic interactions
C Has primary and secondary Has primary, secondary,
structures only tertiary and quaternary
structures
D Three subunits wind together Each subunit has an active site
to form triple helix which binds haem

4. Which of the following accounts for why starch and glycogen are good
energy stores in living organisms?

A A large amount of energy is released when (14) glycosidic


bonds in starch and glycogen are hydrolysed.

B The occurrence of branch points every 26 to 30 residues in


starch and glycogen provides many branch ends from which
digestion can begin.

C The projection of hydroxyl groups into the helical structures of


starch and glycogen causes both to be insoluble in water.

D Starch and glycogen are highly reactive macromolecules that


can be rapidly broken down when required.

5. Which of the following does not describe the active site of an enzyme?

A The active site is usually a pocket comprising several amino


acids.

B The active site binds with an inhibitor which has a


complementary shape.

C In anabolic reactions, the active site brings substrates close


together and catalyses bond formation.

D Beyond the optimum temperature, the active site is first


denatured before the rest of the enzyme structure is denatured.

3
1505

6. Which of the following regarding homologous chromosomes is correct?

A The X and Y chromosomes in human males are not homologous


chromosomes.

B Homologous chromosomes are not present in a cell undergoing


mitosis.

C Centromeres of homologous chromosomes occupy the same


position that is approximately equidistant from both ends of the
linear chromosomes.

D Homologous chromosomes usually carry the same alleles at


corresponding gene loci.

7. Fig. 7 below shows measurements during one mitotic cell cycle.

Fig. 7

Which stage of mitosis begins at X and which measurements are


illustrated by curves 1 and 2?

Stage beginning Distance between Distance between


at X centromeres of the centromeres of
chromosomes and sister chromatids
poles of spindle
A Metaphase 1 2
B Metaphase 2 1
C Anaphase 1 2
D Anaphase 2 1

4
1506

8. Which of the following does not explain how meiosis leads to genetic
variation?

A At metaphase II, the orientation of a particular chromosome to


the poles is not affected by the orientation of the other
chromosomes along the equator.

B By the end of meiosis I, each daughter nucleus contains either


the paternal or maternal copy of a chromosome.

C Homologous regions of DNA are exchanged within a bivalent


after synapsis has occurred.

D The chromatids that are separating during anaphase II may be


genetically different.

9. Several aspects of DNA replication are listed below:

1. Synthesis of complementary RNA primer


2. Base-pairing between DNA nucleotides
3. Removal of mismatched DNA nucleotides
4. Initiation only at origin of replication

Which of the above contributes directly to the accuracy of DNA


replication?

A 1 and 3 only.

B 2 and 3 only.

C 3 and 4 only.

D 2, 3 and 4 only.

5
1507

10. A simplified representation of a replication bubble is shown in Fig. 10


below. Parental strands 1 and 2 and the growing daughter strands X and
Y are indicated.

Parent
strand 1
Daughter Daughter
strand X strand Y

Parent
strand 2

Fig. 10

Which of the following statements about the syntheses of Daughter


strands X and Y is correct?

A Strands X and Y are synthesised away from their respective


replication forks.

B Strand X is synthesised continuously while Strand X is


synthesised in the form of Okazaki fragments.

C Strand X is synthesised in the 5 3 direction while Strand Y is


synthesised in the 3 5 direction.

D DNA ligase will eventually catalyse the fusion of Strand X with


Strand Y.

11. Translation of the information on an mRNA into the amino acid sequence
of a polypeptide chain is an energy-requiring process, involving further
processes such as:

1. Amino acid activation


2. Assembly of translation initiation complex
3. Attachment of amino acid to growing polypeptide chain
4. Translocation of ribosome along mRNA

In which of the above processes is energy in the form of ATP required?

A 1 only.

B 1 and 3 only.

C 1, 3 and 4 only.

D All of the above.

6
1508

12. The first five DNA triplets that code for a particular protein is shown below:

3 CAC GGA AGC CCA GAA 5

The genetic information in the sequence above is eventually converted


into a specific amino acid sequence according to Fig. 12 below.

Fig. 12

With the aid of Fig. 12, what is the sequence of the protein encoded by
the DNA sequence above?

A His Gly Ser Pro Glu

B Val Pro Ser Gly Leu

C Lys Thr Arg Arg His

D Phe Try Ala Ser Val

7
1509

13. Which of the following does not occur after lambda phage infection of an
E. coli host cell?

A Lambda phage DNA is preferentially replicated compared to


host DNA after phage DNA is incorporated into host genome.

B The expression of most lambda phage genes is repressed


immediately after prophage formation.

C Lambda phage DNA temporarily circularises immediately after


host cell entry.

D The viral head and tail structures remain on the surface of the
host cell, and may be eventually degraded.

14. Fig. 14 below is a simple representation of the structure of HIV (not drawn
to scale). Structures W to Z are labelled as shown.

W
X

Fig. 14

Which of the following does not correctly describe the function of the
respective structures?

A Structure W binds with glycoprotein receptors on the host cell.

B Structure X synthesises double-stranded DNA.

C Structure Y causes the insertion of viral RNA into host genome.

D Structure Z promotes fusion with host cell surface membrane.

8
1510

15. The process of bacterial conjugation is illustrated in Fig. 15 below.

Fig. 15

The following properties of the F plasmid are demonstrated in Fig. 15


above except

A the F plasmid is heritable.

B the F plasmid replicates semi-conservatively.

C the F plasmid has an origin of replication and transfer.

D the F plasmid carries the genes encoding the pilus proteins.

9
1511

16. Several possible properties of operons are listed below:

1. Inducible
2. Contains more than one structural gene
3. Can be negatively controlled by repressor
4. Can be positively controlled by activator

Which of the above describe(s) both the lac and trp operons?

A 2 only.

B 2 and 3 only.

C 1, 2 and 3 only.

D All of the above.

17. Which of the following is/are mostly likely to contain DNA sequences that
are tandemly repeating?

1. Centromere
2. Origin of replication
3. Promoter
4. Restriction Site

A 1 only.

B 1 and 3 only.

C 1, 2 and 3 only.

D All of the above.

10
1512

18. In transcription initiation, several steps are involved:

1. RNA polymerase II binds.


2. Transcription factor II B binds.
3. Histone proteins are acetylated at specific sites.
4. RNA polymerase II is phosphorylated on multiple sites.
5. TATA binding protein of transcription factor II D recognises TATA
box.

Which of the following indicates the correct sequence of the steps above?

A 2, 3, 5, 4, 1

B 2, 3, 5, 1, 4

C 3, 2, 5, 4, 1

D 3, 5, 2, 1, 4

19. The following properties are common to both eukaryotic and prokaryotic
genomes, except

A genetic material being in the form of double-stranded DNA.

B the association of genetic material with histone proteins.

C the presence of one or more origins of replication.

D the control of gene expression by activators and repressors.

20. Which of the following is true of cancers?

A A cancer cell will always eventually develop into a tumour.

B Cancer cells are likely to have longer-than-usual telomeres


despite having inactivated telomerases.

C A cell that has a copy of the p53 tumour suppressor gene


inactivated can be considered to be cancerous.

D When a copy of the ras proto-oncogene is activated into an


oncogene in a normal cell, cancer immediately develops.

11
1513

21. Two types of plumage (feathers) exist in chickens: hen-feathering and


cock-feathering. The type of plumage results from the expression of an
autosomal gene locus that carries two alleles:

Allele H (hen-feathering) is dominant in both female and male


chickens.
Allele h (cock-feathering) is recessive, and the phenotype is
expressed only in male chickens.

Which of the following would be the result of a mating between a cock-


feathering male chicken, and a hen-feathering female chicken which is
heterozygous at the gene locus?

A The probability of a male offspring being cock-feathering is 50%


and that of a female offspring being hen-feathering is 50%.

B The probability of a male offspring being cock-feathering is 50%


and that of a female offspring being hen-feathering is 100%.

C The probability of a male offspring being cock-feathering is


100% and that of a female offspring being hen-feathering is 50%.

D The probability of a male offspring being cock-feathering is


100% and that of a female offspring being hen-feathering is
100%.

12
1514

22. The pedigree chart below shows the inheritance of a genetic disease in a
family.

Key:

What is the nature of the allele that causes this disease?

A autosomal dominant

B sex-linked dominant

C autosomal recessive

D sex-linked recessive

13
1515

23. Wheat kernel colour is dependent upon a biochemical reaction that


converts a white precursor into a coloured pigment, when the products of
both gene A and gene B are both present.

product of gene A
white precursor coloured kernel
product of gene B

When 2 breeds of pure-breeding white wheat kernel are crossed, all the
F1 offspring show coloured kernel. When the F1 offspring are selfed, what
is the probability of the F2 offspring having the same phenotype as the F1
phenotype?

A 1 in 16

B 3 in 16

C 4 in 16

D 9 in 16

14
1516

24. Fig. 24 below shows a human karyotype.

Fig. 24

Which of the following is evident from Fig. 24?

A The karyotype indicates a female patient with a single


chromosomal aberration.

B The karyotype indicates a male patient with a single


chromosomal duplication.

C The karyotype indicates a female patient with two chromosomal


duplications.

D The karyotype indicates a male patient with two chromosomal


aberrations.

15
1517

25. In an experiment, chloroplast extracts are first treated with a chemical that
snatches away the electron that is accepted by the electron acceptor in
photosystem I. The extracts are then treated with 2 hours of light and are
provided with ample carbon dioxide and water.

Which of the following correctly shows the products that are formed after
the experiment?

O2 Reduced
ATP Glucose
NADP
A + +
B + + +
C +
D +
Key: (+) = present, () = absent

16
1518

26. Fig. 26 below illustrates several metabolic processes that occur within the
body. Table 26 contains three statements that describe some of these
processes.

Glycogen Glucose

P Q
Fructose 1,6-bisphosphate

Pyruvate

S T

Carbon dioxide Lactic acid


and water

Fig. 26

Statement 1 NAD+ is regenerated without the use of the electron


transport system.
Statement 2 ATP is synthesised via substrate-level phosphorylation.
Statement 3 It can occur under anaerobic conditions.
Table 26

Which of the following correctly relates processes P, Q, R, S, T to


statements 1, 2 and 3?

Statement 1 Statement 2 Statement 3


A R S T
B T R P
C S Q S
D Q S R

17
1519

27. Which of the following is necessary for an organism to respond to


changes in its internal environment?

A Positive feedback from effector to receptor.

B Negative feedback from receptor to effector.

C A communication system involving nerve cells.

D A communication system between receptors and effectors.

28. Certain drugs act at the synapses and affect the action of neurotransmitter
substances. The table below shows the effects of four different drugs on a
synapse.

Drug Effect
1 Uncontrolled release of acetylcholine
2 Competes with acetylcholine at the receptor sites
3 Inhibits the enzyme acetylcholinesterase
4 Inhibits the opening of voltage-gated sodium ion channels

Which combination of drugs will reduce the possibility of the formation of


an excitatory post-synaptic potential?

A 2 and 4 only.

B 3 and 4 only.

C 1, 2 and 4 only.

D 2, 3 and 4 only.

18
1520

29. The Guinea baboon and King colobus are two of the many Old World
monkeys that exist.

Guinea baboon King colobus

The classification of these monkeys is shown below.

Animalia
Chordata
Mammalia
Primates
Cercopithecidae (sub: Cercopithecinae)
Papio
Papio papio (Guinea baboon)

Animalia
Chordata
Mammalia
Primates
Cercopithecidae (sub: Colobinae)
Colobus
Colobus polykomos (King colobus)

Which of the following conclusions is consistent with the information


above?

A Both belong to the same class but different families.

B Both belong to the same genus but different classes.

C Both belong to the same family but different genera.

D Both belong to the same species but different orders.

19
1521

30. The table shows the number of estimated nucleotide substitutions that
have occurred since the divergence of seven species a to g.

b c d e f g
a 39 72 128 126 159 269
b 81 130 128 158 268
c 129 127 157 267
d 56 154 271
e 151 268
f 273

Which of the following phylogenetic trees best shows the relationship


among these seven species?

A B

C D

20
1522

31. Which of the following describes the process of natural selection?

A Different rates of reproductive success of different genotypes.

B Spontaneous occurrence of advantageous mutations.

C Change from simple to more complex organisms over time.

D Change in the size of population over time.

32. Which of the following statements is not consistent with the neutral theory
of molecular evolution?

A Genetic drift is more significant in explaining the high degree of


genetic variation in a population than natural selection.

B The frequency of a particular allele in the gene pool of a


population may decrease over time even if there is no selection
pressure acting on it.

C The frequency of a particular allele in the gene pools of two


populations is likely to be identical over time even if these
populations are geographically isolated.

D If all mutations are selectively neutral, differences in DNA


sequences across different species can serve as a molecular
clock to chart evolution.

33. Which of the following statements regarding restriction enzymes is


incorrect?

A Host DNA is prevented from digestion by restriction enzymes


due to specific methylation of DNA.

B A restriction enzyme can create either sticky or blunt ends


depending on the chemical conditions present.

C Cleavage by a restriction enzyme involves the breaking of two


covalent phosphodiester bonds.

D Restriction enzymes protect bacteria against invading viruses by


preventing the replication of viral DNA.

21
1523

34. Which of the following correctly indicates a difference between a genomic


and a cDNA library?

Genomic library cDNA library


A Created from primary transcripts Created from mature mRNAs
B Varies with stage of organism Varies with cell type
development
C Creation involves use of Creation does not involve use of
restriction enzymes restriction enzymes
D Contains promoter regions of Does not contain promoter
genes regions of genes

35. Which of the following is an outcome of the Human Genome Project that
has ethical implications?

A Screening of the genetic make-up of newborn infants for


susceptibility to certain key diseases.

B Creation of customised medicines that are potentially more


expensive to produce than traditional drugs.

C Consideration of a suspects genetic pre-disposition to violent


behaviour in criminal trials.

D The free availability and accessibility of the complete sequence


of the human genome on the Internet.

36. Totipotency is demonstrated when

A an embryonic stem cell divides and differentiates.

B cancer cells give rise to heterogeneous cell types.

C an isolated plant cell develops into a normal adult plant.

D a hematopoietic stem cell differentiates into a lymphocyte.

22
1524

37. The ras oncogene is involved in a variety of human cancers. DNA isolated
from a number of normal and cancerous human tissues were digested
with EcoRI. A Southern blot was performed and the membrane was
probed with radioactive-labelled cloned ras DNA. After exposure to x-ray
film, the resulting autoradiograph is shown below.

1 2 3 4

10 kb

5 kb

more than 2 copies / genome


2 copies / genome
1 copy / genome

1 kb

Lane Sample
1 white blood cells from a healthy human
2 lymphoma cells (cancerous)
3 bladder carcinoma cells (cancerous)
4 sarcoma cells (cancerous)

Based on the experimental results above, which of the following


conclusions can be deduced?

A The result for tissue 2 can be explained by point mutations that


caused the removal of an EcoRI site and the creation of another
within the probe DNA.

B In tissue 3, there was no mutation within the ras coding region.

C In tissue 3, the mutation did not affect existing EcoRI sites within
the ras gene.

D Some of the cancer cells in tissue 4 are likely to be polyploid.

23
1525

38. Which of the following is not an advantage of liposome-mediated gene


therapy over retrovirus-mediated gene therapy?

A No insertional mutagenesis.

B General targeting of various cells.

C Lower stimulation of immune system.

D Possible delivery of all types of nucleic acids.

39. Which of the following methods can be used to introduce a foreign gene
into a plant cell?

1. By Agrobacterium-mediated transfection
2. By microinjection of naked DNA
3. Using a phage delivery vector
4. Using a gene gun

A 1 and 3 only.

B 1, 2 and 4 only.

C 1, 3 and 4 only.

D All of the above.

40. There is concern regarding the escape of GM salmon into the wild
because they

A may replace wild salmon populations.

B may not survive as well in the wild.

C are prone to further mutations.

D will produce sterile offspring with wild salmon.

END OF PAPER

24
1526

CANDIDATE NAME __________________________ INDEX NUMBER _________

CG _________

SERANGOON JUNIOR COLLEGE

JC2 Preliminary Examination 2013 Paper 2

H2 BIOLOGY 9648
19 August 2013/ Monday

2 Hours

Additional materials:

Writing papers / OMS

INSTRUCTIONS TO CANDIDATES

Write your name, CG and index number in the spaces at the top of FOR EXAMINERS USE
this page and on all separate writing papers used.
Section A
Write in dark blue or black pen.
1 /10
You may use a soft pencil for any diagrams, graphs or rough
working. 2 /10

3 /10

Section A 4 /10
Answer all questions.
Write your answers in spaces provided on the question paper. 5 /13

6 /13

Section B 7 /10
Answer only one question out of two.
Write your answers on the separate answer paper provided. 8 /4

Section B

9 /20

INFORMATION FOR CANDIDATES 10 /20

The intended number of marks is given in brackets [ ] at the end of TOTAL /100
each question or part question.
______________________________________________________________________

This question paper consists of 21 printed pages and 1 blank page

1
1527

Question 1

Gene expression refers to genes being turned on and producing a product. The product
could be an enzyme, a structural protein, or a control molecule.

Eukaryotes can regulate their gene expression at various levels whereas prokaryotes
regulate gene expression predominantly at the transcriptional level.

a) Explain how the process of transcription is regulated in eukaryotes. [2]

b) Explain why prokaryotes regulate gene expression predominantly at the transcriptional


level. [2]

c) Post transcriptional modification in eukaryotes involves the addition of the 5 guanosine


cap, splicing as well as the addition of the 3 polyA tail. Explain what will happen to the
resulting protein structure if a mutation occurs at the splice site of an intron. [3]

2
1528

Figure 1.1

d) Name and describe the process that is shown in Figure 1.1. [2]

e) Suggest why there is a need for this process. [1]

[Total: 10 marks]

3
1529

Question 2

a) Sex hormone estrogen promotes mammary epithelial cell proliferation by stimulating the
expression of genes encoding cell cycle regulatory proteins. Estrogen receptors (ER) are
intracellular receptors of estrogen. Estrogen-receptor complexes bind to estrogen response
elements (ERE) in enhancers. Estrogenic control of the cell cycle occurs by increasing the
expression of cyclin and myc C which promote cell cycle progression.

Figure 2.1

With reference to Figure 2.1

i) State the level at which gene expression is regulated by estrogen. [1]

ii) Describe and explain how estrogen regulates gene expression. [4]

4
1530

b) There are two isoforms of estrogen receptors: ER and ER encoded by different genes.
Genomic DNA samples taken from women suffering from breast cancer were analysed for
the gene copy number present for the ER gene.

Figure 2.2

i) With reference to Figure 2.2, describe the effect of different ER gene copy number on
the women. [2]

c) Suggest an explanation as to why patient 2 developed cancer despite a low copy number.
[1]

5
1531

d) Suggest how amplification of the ER gene contributes to breast cancer. [2]

[Total: 10 marks]

6
1532

Question 3

a) A homozygous dominant black mouse was crossed with a homozygous recessive white
mouse. All the F1 offspring were black. When the F1 offspring were selfed, the ratio of black
to brown to white mice was 9:3:4.

Represent the above information in the form of a genetic cross. [4]

b) Explain the molecular mechanism of this cross. [3]

7
1533

c) Table 3.1 shows the inheritance of kernel colour in wheat when 2 intermediate parents
were crossed. (AaBbCc x AaBbCc). The offspring contain seven different shades of kernel
color based on the number of capital letters in each genotype. The higher the number of
capital letters, the darker the shade of the wheat kernel.

Gametes ABC ABc AbC Abc aBC aBc abC abc

ABC 6 5 5 5 4 4 3

ABc 5 4 4 3 4 3 3 2

AbC 5 4 4 3 4 3 3 2

Abc 4 3 3 2 3 2 2 1

aBC 5 4 4 3 4 3 3 2

aBc 4 3 3 2 3 2 2 1

abC 4 3 3 2 3 2 2 1

abc 3 2 2 1 2 1 1 0

Table 3.1

i) Identify and state the characteristics of such an inheritance. [3]

[Total: 10 marks]

8
1534

Question 4

a) Antibiotic medications are used to kill bacteria, which can cause illness and disease. They
have made a major contribution to human health. Many diseases that once killed people can
now be treated effectively with antibiotics. However, some bacteria have become resistant to
commonly used antibiotics.

Antibiotic resistant bacteria are bacteria that are not controlled or killed by antibiotics. Due to
mutation, they are able to survive and even multiply in the presence of an antibiotic. Most
infection-causing bacteria can become resistant to at least some antibiotics. Bacteria that
are resistant to many antibiotics are known as multi-drug resistant organisms (MROs).

Antibiotic resistance can cause serious disease and is an important public health problem. It
can be prevented by minimising unnecessary prescribing and overprescribing of antibiotics,
the correct use of prescribed antibiotics, and good hygiene and infection control.

i) Explain why a mutation acquired by a bacteria will most certainly affect its pheonotype. [2]

ii) Explain how natural selection has led to the emergence of such antibiotic resistant
bacteria. [4]

9
1535

b) According to the Red Queen hypothesis, sexual reproduction persists because it enables
many species to rapidly evolve new genetic defenses against parasites that attempt to live
off them.

Scientists have tested this idea by observing different groups of small fish Poecilopsis
species (Gila Topminnow) in Mexico.

Some populations of the topminnow reproduce sexually, while others practice


parthenogenesis. Parthenogenesis occurs when females produce offspring without any male
contribution and the female's gametes develop directly into female offspring.

Topminnows are constantly parasitized by black spot disease caused by black spot worms
that encyst in the skin. Parasitized topminnows rarely survive. The researchers found that
identical populations of the asexually reproducing topminnows harbored many more black-
spot worms than those reproducing sexually, a finding that fit the Red Queen hypothesis: the
sexual topminnows could devise new defenses faster by recombination than the asexually
reproducing clones.

However, it was observed that after a drought, the sexually reproducing topminnows were
more heavily parasitized than the cloned topminnows. This process of whereby chance
events cause the allele frequency to change unpredictably is known as genetic drift.

i)With reference to the example of topminnow, explain the concept of genetic drift and how it
can alter the populations gene pool significantly. [2]

10
1536

ii) Topminnows are brought by man to a new place with totally different environmental
conditions. Over time, speciation occurs and a new species of fish related to the topminnows
is discovered.

Explain how speciation occurred to give rise to the new species of fish. [2]

[Total: 10 marks]

11
1537

Question 5

Figure 5.1 shows a root tip of the onion plant. The meristematic region contains cells
undergoing rapid nuclear division.

Figure 5.1

(a) Explain the function of the type of nuclear division that is occurring in the cells found in
the meristematic region in Figure 5.1. [3]

12
1538

(b) Describe how two distinct daughter cells are formed at the end of nuclear division. [2]

A researcher randomly extracted two meristematic cells. He proceeded to analyse them and
found them to be genetically non-identical. He concluded and wrote down in his research log
book that this was due to independent assortment of chromosomes before non-identical
sister chromatids are pulled apart to opposite poles of the dividing cell.

(c) Do you agree with the researchers conclusion? Explain why. [3]

The graph in Figure 5.2 shows the amount of DNA in the nuclei of cells taken from the
meristematic region of the onion plant.

Figure 5.2

13
1539

(d) With reference to Figure 5.2, state precisely the molecular process that accounts for the
change in DNA amount in the nuclei of cells from 4au to 8 au. [1]

(e) Compare the above process in (d) with transcription. [4]

[Total: 13 marks]

14
1540

Question 6

(a) Figure 6.1 shows a generalized graph of the relationship between HIV copies and CD4
counts over the average course of untreated HIV infection.

+
CD4 Lymphocyte count (cells per L)
HIV RNA copies per mL of plasma

Figure 6.1

i) Describe the relationship between the graph of CD4+ Lymphocyte count and the
graph of HIV RNA copies. [1]

ii) Explain the relationship stated in (a). [4]

15
1541

iii) As indicated in the graph above, clinical latency refers to the period where HIV
infection remains undetected and patients are mostly symptom free.

Explain how the HIV virus enables itself to remain undetected during clinical latency.
[2]

(b) A population of E.Coli was grown on a culture medium containing a mixture of glucose
and lactose for over 4 hours. The growth curve was plotted onto the graph in Figure 6.2 as
shown.

Sugar B
metabolized
Sugar A
metabolized X

Figure 6.2

i) Identify sugars A and B. [1]

16
1542

ii) Outline the events that are taking place along the lac operon of E coli during the
metabolism of sugar after 3 hours. [4]

iii) Suggest an explanation for the plateau marked X in Figure 6.2. [1]

[Total: 13 marks]

17
1543

Question 7

a) Green plants undergo photosynthesis and respiration simultaneously in the presence of


light and oxygen. In an experiment, only the photosynthetic rate of two species of plants was
examined and represented in Figure 7.1. Both plants were distinct in their leaf growth
pattern. Plant A had vertical leaves while Plant B had horizontal leaves.

Figure 7.1

i) Explain the difference in the shape of the graphs for Plant A and Plant B in Figure
7.1. [4]

18
1544

b) Plant B was extracted and placed into another environment. Conditions of the previous
and new environments are tabled below in Figure 7.2.

Condition Quantity
Previous New
Environment Environment
Amount of light 80% 0%
Amount of CO2 50% 0%
Amount of O2 21% 21%
Water availability Yes Yes

Figure 7.2

i) Identify the process that is occurring within Plant B. [1]

ii) Explain the importance of oxygen in the identified process. [3]

iii) Predict and explain briefly how the level of NADP+ and NAD+ in Plant B in this new
environment compares to that in the previous environment in part a) of the question.
[2]

[Total: 10 marks]

19
1545

Question 8

Radioactively-labelled amino acids were introduced into Cell A and Cell B. The muddle-
headed researcher could not recall the identities of the cells, which was either the gastric
epithelial cell or the cardiac muscle cell. The amount of radioactivity within three organelles
(golgi apparatus, ribosomes, rough endoplasmic reticulum) in both cells were monitored over
45 min and tabled as shown in Figure 8.1.

Radioactive levels / au
Cell A Cell B
0 min 15 min 30 min 45 min 0 min 15 min 30 min 45 min
Organelle 0 80 50 10 0 80 10 0
X
Organelle 0 0 20 15 0 60 25 5
Y
Organelle 0 0 0 5 0 0 60 20
Z

Figure 8.1

(a) Identify the organelles X, Y and Z. [1]

(b) Identify the cell types of cells A and B and explain your answer. [3]

[Total: 4 marks]

20
1546

Section B (20 Marks)

Answer one question

Write your answers on the separate answer paper provided.

Your answers should be illustrated by large, clearly labeled diagrams, where appropriate.

Your answers must be in continuous prose where appropriate.

Your answers must be set out in sections (a), (b), etc as indicated in the question.

Question 9

a) Outline the differences between prokaryotic control of gene expression with the
eukaryotic model. [7]

b) Outline the significance of post-translational modifications of eukaryotic proteins. [6]

c) Explain how the molecular structure of the axon membrane facilitates nervous
transmission along the axon. [8]

Question 10

a) Compare the Calvin cycle and the Krebs cycle. [6]

b) Explain the importance of operons in bacteria. [4]

c) Describe the processes through which genetic diversity can be achieved in bacteria.
[10]

END OF PAPER

21
1547

BLANK PAGE

22
1548

Question 1

Gene expression refers to genes being turned on and producing a product. The product
could be an enzyme, a structural protein, or a control molecule.

Eukaryotes can regulate their gene expression at various levels whereas prokaryotes
regulate gene expression predominantly at the transcriptional level.

a) Explain how the process of transcription is regulated in eukaryotes. [2]

1. Acetylation of histone increase transcription / deacetylation decrease transcription


2.Methylation of DNA decrease transcription/ Demethylation increase transcription

b) Explain why prokaryotes regulate gene expression predominantly at the transcriptional


level. [2]

1. due to absence of a membrane-bound nucleus;


2. which means that transcription and translation occurs simultaneously in prokaryotes;
hence by regulating transcription, the bacteria is also regulating translation;

OR

3. genes are arranged in the form of operons in prokaryotes;


4. transcriptional control allows many genes involved in the same metabolic pathway to
be regulated at the same time;

Post transcriptional modification in eukaryotes involve the addition of the 5 guanosine cap,
splicing as well as the addition of the 3 polyA tail. Explain what will happen to resulting
protein structure if a mutation occurs at the splice site of an intron. [3]

1. Small nuclear ribonucleoproteins (snRNPs) will not be able to recognize and bind to
splice site of the intron on the mRNA;
1. spliceosome (pre-mRNA-snRNP complex) is not formed;
2. Thus the intron is not excised out;
3. and the mature mRNA formed is longer than normal;to be translated to a much
longer polypeptide than usual;
4. which will not be folded to the same 3D conformation as the normal protein; on
functional protein
1549

d) Name and describe the process that is shown in Figure 1.1. [3]
Alternative splicing
Spliceosomes recognize the intron- exon junctions and remove all the introns;
Different pre-mRNA can be spliced into different mature mRNA by the inclusion of
different sets of exons
Resulting in different polypeptides

e) Suggest why there is a need for this process. [1]


Eukaryote has a larger of non coding region compared to coding region,
alternative splicing allows for a single transcript to potentially generate a large
number of different protein variants
1550

Question 2

a) Sex hormone estrogen promotes mammary epithelial cell proliferation by stimulating the
expression of genes encoding cell cycle regulatory proteins. Estrogen receptors (ER) are
intracellular receptors of estrogen. Estrogen-receptor complexes bind to estrogen response
elements (ERE) in enhancers. Estrogenic control of the cell cycle occurs by increasing the
expression of cyclin and myc C which promote cell cycle progression.

Figure 2.1

With reference to Figure 2.1

i) State the level at which gene expression is regulated by estrogen. [1]

transcriptional level;
1551

ii) describe and explain how estrogen regulates gene expression. [4]

Estrogen lipid soluble, diffuse across phosplipid bilayer to bind to intracellular


receptor form hormone receptor complex;

ER in cytoplasm, estrogen binds to receptors and causes dimerisation;

Conformation change when estrogen bind to receptor => activated

enters nucleus and binds to ERE on the enhancer sequence,


Identify hormone receptor complex as an activator

attracting other general transcription factors and RNA polymerase to bind; (


reject nuclear proteins just what exactly are they?

increases rate of transcription of cyclin and myc Cwhich causes cell division.

b) There are two isoforms of estrogen receptors: ER and ER encoded by different genes.
Genomic DNA samples taken from women suffering from breast cancer were analysed for
the gene copy number present for the ER gene.

Figure 2.2
1552

i) With reference to the Figure 2.2, describe the effect different ER gene copy number on
the women. [2]

high gene copy number ratio is correlated to occurrence of breast cancer

Women 1,3, 4 who had 14 copies, 6 copies and 7 copies of ER gene


suffered from breast cancer;

Woman 2 and 5 had 1 copy of the gene but woman 2 had cancer while
woman 5 did not suffer from cancer

c) Suggest an explanation as to why patient 2 developed cancer despite a low copy number.
[1]

Gain of function mutations on other proto-oncogenes. Just require 1 copy of proto-


oncogene to be converted to oncogene

Loss of function of tumour suppressor genes both copies or alleles NOT both genes

This results in permanent activation of hormone receptor complex

d) Suggest how amplification of the ER gene contributes to breast cancer. [2]

more intracellular receptors are synthesized due to increased number of


copies;

more activators bind to enhancers to increase rate of transcription of


genes encoding proteins which trigger cell division/ proto-oncogenes;

[Total: 10 marks]
1553

Question 3

a) A homozygous black mouse was crossed with a homozygous recessive white mouse. All
the F1 offspring were black. When the F1 offspring were selfed, the ratio of black to brown to
white was 9:3:4.

Represent the above information in the form of a genetic cross. [4]

Parental Homo black x Homo white


phenotype mouse mouse
Parental AABB x aabb
genotype
Gametes AB x ab

F1 genotype AaBb
F1 phenotype AaBb x AaBb
Gametes AB AB
Ab Ab
aB aB
ab ab

Punett Square

AB Ab aB Ab
AB AABB AABb AaBB AaBb
black black black black
Ab AABb AAbb AaBb Aabb
black brown black brown
aB AaBB AaBb aaBB aaBb
black black white white
Ab AaBb Aabb aaBb Aabb
black brown white white

Phenotypic ratio: black: brown: white


9:3:4
1554

b) Explain the molecular mechanism of this cross. [3]

-Recessive epistasis
-2 recessive alleles for gene A will mask the effect of Gene B
-aa_ _ =white
-A_B_=black
-A_bb =brown

c) Table 3.1 shows the inheritance of kernel colour in wheat when 2 intermediate parents
were crossed. (AaBbCc x AaBbCc). The offspring contain seven different shades of kernel
color based on the number of capital letters in each genotype. The higher the number of
capital letters, the darker the shade of the wheat kernel.

Gametes ABC ABc AbC Abc aBC aBc abC abc

ABC 6 5 5 4 5 4 4 3

ABc 5 4 4 3 4 3 3 2

AbC 5 4 4 3 4 3 3 2

Abc 4 3 3 2 3 2 2 1

aBC 5 4 4 3 4 3 3 2

aBc 4 3 3 2 3 2 2 1

abC 4 3 3 2 3 2 2 1

abc 3 2 2 1 2 1 1 0

Table 3.1

i) Identify and state the characteristics of such an inheritance. [3]


-Polygenic inheritance
-Additive effect
-Explain additive:The colour is the combined effect of many different genes on multiple loci.
-Continuous variation/ range of phenotype in F2
1555

Question 4

a) Antibiotic medications are used to kill bacteria, which can cause illness and disease. They
have made a major contribution to human health. Many diseases that once killed people can
now be treated effectively with antibiotics. However, some bacteria have become resistant to
commonly used antibiotics.

Antibiotic resistant bacteria are bacteria that are not controlled or killed by antibiotics. Due to
mutation, they are able to survive and even multiply in the presence of an antibiotic. Most
infection-causing bacteria can become resistant to at least some antibiotics. Bacteria that
are resistant to many antibiotics are known as multi-resistant organisms (MROs).

Antibiotic resistance can cause serious disease and is an important public health problem. It
can be prevented by minimising unnecessary prescribing and overprescribing of antibiotics,
the correct use of prescribed antibiotics, and good hygiene and infection control.

i) Explain why a mutation acquired by a bacteria will most certainly affect its pheonotype. [2]

-monoploid=1 allele (reject haploid)


-No presence of dominant allele to mask the effect

ii) Explain how natural selection has led to the emergence of such antibiotic resistant
bacteria. [4]

-Variation: different variation of bacteria some resistant to antibiotic some not


-Selection pressure: Antibiotic
-Selective advantage: Bacteria that is resistant to antiobiotic
-Bacteria survived to produce offspring by binary fission and pass on their antibiotic
resistance gene to the next generation=> Frequency of antibiotic resistance allele increases
in population.
1556

b) According to the Red Queen hypothesis, sexual reproduction persists because it enables
many species to rapidly evolve new genetic defenses against parasites that attempt to live
off them.

Scientists have tested this idea by observing different groups of small fish Poecilopsis
species (Gila Topminnow) in Mexico.

Some populations of the topminnow reproduce sexually, while others practice


parthenogenesis. Parthenogenesis occurs when females produce offspring without any male
contribution and the female's gametes develop directly into female offspring.

Topminnows are constantly parasitized by black spot disease caused by black spot worms
that encyst in the skin. Parasitized topminnows rarely survive. The researchers found that
identical populations of the asexually reproducing topminnows harbored many more black-
spot worms than did those producing sexually, a finding that fit the Red Queen hypothesis:
the sexual topminnows could devise new defenses faster by recombination than the
asexually producing clones.

However, it was observed that after a drought, the sexually reproducing topminnows were
more heavily parasitized than the cloned topminnows. This process of whereby chance
events cause the allele frequency to change unpredictably is known as genetic drift.

i) With reference to the example of topminnow Explain the concept of genetic drift and how it
can alter the populations gene pool significantly. [2]

-Genetic drift is the random change in allele frequencies due to chance factors/not due to
selection pressure
-Chance factor: Drought
-allele that make them resistant to parasite frequency decrease significantly => eventually
the entire population may be homozygous
1557

ii) Topminnows are brought by man to a new place with totally different environmental
conditions. Over time, speciation occurs and a new species of fish related to the topminnows
is discovered.

Explain how speciation occurred to give rise to the new species of fish. [2]

-Geographical isolation prevent the sub-group from interbreeding with the rest of the
population -> leading to genetic isolation -> no gene flow
-Different selection pressure

[Total: 10 marks]
1558

Question 5

Figure 5.1 shows a root tip of the onion plant. The meristematic region contains cells
undergoing rapid nuclear division.

Figure 5.1

(a) Explain the function of the type of nuclear division that is occurring in the cells found in
the meristematic region in Figure 5.1. [3]
Mitosis
Main function is to maintain genetic stability by maintaining the same number
and type of chromosomes during nuclear division
Thus enabling growth and cell regeneration in the meristamitic region of the
root tip.
(max 3)
1559

(b) Describe how two distinct daughter cells are formed at the end of nuclear division. [2]
Cytokinesis occurs
Golgi vesicles containing cellulose move along microtubules to the middle of
the cell;
Membranes of golgi vesicles fuse, cellulose deposited forms cell plate that
extends across the equator, forming two distinct daughter cells.

(max 2)

A researcher randomly extracted two meristematic cells. He proceeded to analyse them and
found them to be genetically non-identical. He concluded and wrote down in his research log
book that this was due to independent assortment of chromosomes before non-identical
sister chromatids are pulled apart to opposite poles of the dividing cell.

(c) Do you agree with the researchers conclusion? Explain why. [3]
Do not agree/ conclusion was Invalid, because mitosis is occurring in
meristematic cells instead of meiosis.
Independent assortment occurs during metaphase I of meiosis, where
bivalents/ pairs of homologous chromosomes line up randomly in two rows at
the equator.
However, during metaphase of mitosis, chromosomes with identical sister
chromatids align in a single row at the equator thus no independent
assortment occurs.
1560

The graph in Figure 5.2 shows the amount of DNA in the nuclei of cells taken from the
meristematic region of the onion plant.

Figure 5.2

(d) With reference to Figure 5.2, state precisely the molecular process that accounts for the
change in DNA amount in the nuclei of cells from 4au to 8 au. [1]

Semi-consvervative DNA replication occurring during S phase of Interphase

.
1561

(e) Compare the above process in (d) with transcription. [4]


(similarity) Both DNA replication and transcription has elongation of daughter
strands in the 5 3 direction.
(similarity) Both DNA replication and transcription uses DNA as a template to
synthesise their respective products.
(similarity) Both DNA replication and transcription involves complementary
base-pairing (A- T/U; C-G with template strand to sequence/ synthesise their
respective products.
(difference) Product of DNA replication is double stranded DNA molecule
consisting of one old and one new strand, while product of transcription is a
single stranded RNA molecule.
(difference) DNA replication uses both strands of DNA as a template while
transcription only uses antisense strand as template.
(difference) DNA replication involves the use of thymine to complementary
base-pair with adenine, while transcription uses uracil to complementary
base-pair with adenine.
(2 similarities + 2 differences, max 4)

[Total: 13 marks]
1562

Question 6

(a) Figure 6.1 shows a generalized graph of the relationship between HIV copies and CD4
counts over the average course of untreated HIV infection.

+
CD4 Lymphocyte count (cells per L)
HIV RNA copies per mL of plasma

Figure 6.1

i) Describe the relationship between the graph of CD4+ Lymphocyte count and the
graph of HIV RNA copies. [1]
Inverse relationship/ as the graph of CD4+ lymphocyte count decreases, the graph of
HIV RNA copies increases and vice versa.

ii) Explain the relationship stated in (a). [4]


CD4+ lymphocytes are the specific host cells for HIV virus,
The specific glycoproteins (gp120/gp41) on the surface of the HIV viral envelope
binds to the specific CD4 receptors on the lymphocytes, facilitating entry of HIV virus
Allows replication of its RNA genome, and reassembly to form large no of new HIV
virions which bud off from host cell,pinching off host cell membrane
Which can lead to host cell death, thus when RNA copies increases, CD4+
lymphocyte count decreases (and vice versa).
1563

iii) As indicated in the graph above, clinical latency refers to the period where HIV
infection remains undetected and patients are mostly symptom free.

Explain how the HIV virus enables itself to remain undetected during clinical latency.
[2]
Single stranded RNA genome of the HIV virus is able to be reverse transcribed to
form double stranded cDNA using reverse transcriptase enzyme from HIV virus;
Intergrase enzyme allows double stranded cDNA to be integrated with host cell
genome and replicate as host genome replicates; (not recognised as foreign/ not
transcribed to form viral proteins)
Therefore no clinical symptoms are caused, HIV infection is undetected.

(b) A population of E.Coli was grown on a culture medium containing a mixture of glucose
and lactose for over 4 hours. The growth curve was plotted onto the graph in Figure 6.2 as
shown.

Sugar B
metabolized
Sugar A
metabolized X

Figure 6.2

i) Identify sugars A and B. [1]


A: glucose, B: lactose

ii) Outline the events that are taking place along the lac operon of E coli during the
metabolism of sugar after 3 hours. [4]
After 3 hours, concentration of glucose is very low while concentration of lactose is
high, thus cAMP levels increases.
cAMP binds to the allosteric site on CAP, alters CAP to an active conformation.
1564

Active CAP binds to CAP binding site on DNA, bends the DNA to facilitate binding of
RNA polymerase to the promoter of the lac operon for transcription of structural
genes in lac operon.
At the same time, lactose is converted to allolactose inducer by b-galactosidase,
inactivating the lac repressor. Lac repressor does not bind to operator of lac operon.

iii) Suggest an explanation for the plateau marked X in Figure 6.2. [1]
Plateau X indicates a time delay as time is required for the lac operon to be
completely induced and the production of enzymes like beta-galactosidase.

Any possible answer accepted.

[Total: 13 marks]
1565

Question 7

a) Green plants undergo photosynthesis and respiration simultaneously in the presence of


light and oxygen. In an experiment, only the photosynthetic rate of two species of plants was
examined and represented in Figure 7.1. Both plants were distinct in their leaf growth
pattern. Plant A had vertical leaves while Plant B had horizontal leaves.

Figure 7.1

i) Explain the difference in the shape of the graphs for Plant A and Plant B in Figure
7.1. [4]
The maximum rate of photosynthesis in Plant A is higher than that in Plant B.
The horizontal leaves found on the top region of plant B block light from reaching the
leaves at the bottom region of plant B, while the lower leaves in plant A are not
blocked from light.
Hence only the chlorophyll of the leaves in the top region of plant B reach light
saturation point as compared to plant A where chlorophyll in all the leaves can reach
light saturation point eventually.
Less electrons emitted in plant B as compared to plant A, less ATP and NADPH
produced, lower rate of photosynthesis as compared to plant A.

OR

The rate of photosynthesis in plant B reaches a maximum at lower light intensity than
plant A.
The horizontal leaves found on the top region of plant B block light from reaching the
leaves at the bottom region of plant B, while the lower leaves in plant A are not
blocked from light.
At lower light intensity, all the chlorophylls in plant B that are exposed to light reach
light saturation point, while not all the chlorophyll in plant A have reached light
saturation point.
1566

Thus at lower light intensity, chlorophyll in plant B emits maximum number of


electrons, and produces ATP and NADPH maximally, thus reaches maximum
photosynthesis rate but plant A does not.

b) Plant B was extracted and placed into another environment. Conditions of the previous
and new environments are tabled below in Figure 7.2.

Condition Quantity
Previous New
Environment Environment
Amount of light 80% 0%
Amount of CO2 50% 0%
Amount of O2 21% 21%
Water availability Yes Yes

Figure 7.2

i) Identify the process that is occurring within Plant B. [1]


Aerobic respiration

ii) Explain the importance of oxygen in the identified process. [3]


Oxygen serves as final electron acceptor at the end of the electron transport chain,
accepting electrons and combining with H+ to form water via cytochrome oxidase.
Electronegative oxygen helps to maintain the pull of electrons down the electron
transport chain.
As electrons are pulled down the ETC comprising electron carriers of progressively
lower energy levels, energy released is used to maintain a proton gradient across
inner mitochondrial membrane for ATP synthesis (chemiosmosis)

iii) Predict and explain briefly how the level of NADP+ and NAD+ in Plant B in this new
environment compares to that in the previous environment in part a) of the question.
[2]
In the new environment, aerobic respiration is occurring, NADP+ level increases,
NADP+ is only used up and converted to NADPH during photosynthesis and not
aerobic respiration.
In the new environment, NAD+ level remains similar to that in the previous
environment, aerobic respiration occurs at a constant rate in both environments.

[Total: 10 marks]
1567

Question 8

Radioactively-labelled amino acids were introduced into Cell A and Cell B. The muddle-
headed researcher could not recall the identities of the cells, which was either the gastric
epithelial cell or the cardiac muscle cell. The amount of radioactivity within three organelles
(golgi apparatus, ribosomes, rough endoplasmic reticulum) in both cells were monitored over
45 min and tabled as shown in Figure 8.1.

Radioactive levels / au
Cell A Cell B
0 min 15 min 30 min 45 min 0 min 15 min 30 min 45 min
Organelle 0 80 50 10 0 80 10 0
X
Organelle 0 0 20 15 0 60 25 5
Y
Organelle 0 0 0 5 0 0 60 20
Z

(a) Identify the organelles X, Y and Z. [1]


X: ribosome, Y: RER. Z: GA
(Award 1mark if all are correct)

(b) Identify the cell types of cells A and B and explain your answer. [3]
Cell A is the cardiac muscle cell while Cell B is the gastric epithelial cell.
Radioactivity from radioactive amino acids in cell A is mainly found in free ribosomes
(organelle X) and less found in bound ribosomes on RER (organelle Y), thus cell is
not a secretory cell and proteins made remain in the cell.
Radioactivity from radioactive amino acids in cell B is passed from ribosomes in RER
to the RER lumen to the Golgi apparatus, which lies on the secretory pathway of
protein synthesis, thus Cell B is a secretory cell gastric epithelial cell.

Essay
1568

Outline the differences between prokaryotic control of gene expression with the eukaryotic
model. [7]

Point of Prokaryotes Eukaryote


comparison
a) Control -At transcriptional level -Mainly at transciritonal
level
-No Genomic -Genomic
-No Post transcirptional -Post transcirptional
-Translational
-No Translational
-Post-translational
-No Post-translational

b) Type of General transcription factors General transcription


transcription bind to promoter factors bind to promoter
factors Specific transcription
factors bind to enhancers
and silencers

c) Operon Allows bacteria to coordinately Not present/ not significant


regulate a group of genes that
encode gene products with
related functions.

d) Promoter One promoter controlling a One promoter for each


group of structural genes coding gene
for enzymes involved in the
same metabolic pathway

e) mRNA Polycistronic mRNA Monocistronic mRNA


one mRNA code for more than 1 one mRNA code for 1
protein protein

Presence of several start and 1 start codon 1 stop codon


stop codons
f) Enhancer Absent/ not significant Major control; increase
efficiency of transcription

g) Silencer Absent/ not significant Major control; decrease


efficiency of transcription

h) Accessibility of Absent/ not significant Eg:


RNA polymerase to Histone Acetylation / DNA
promoter methylation /

i Presence of introns No introns no need for post Intorns => need for need
and therefore transcriptional modifications so for post transcriptional
spliceosome no spliceosome modifications
=>spliceosome
1569

Explain how the molecular structure of the axon membrane facilitates nervous
transmission along the axon. [8]

Phospholipid bilayer;
with a hydrophobic core;
Prevents direct entry of charged ions Na+, K+ into the axon;
Allows accumulation of ions on one side of the membrane;

Proteins distributed on the membrane;


Leak channels for Na+ and K+
Always open and allows ions to move down concentration gradient by facilitated
diffusion;
Helps maintain resting potential (with Na+/K+ pump);

Voltage-gated channels for Na+ and K+ ions;


Acts as channels for facilitated diffusion of ions down the respective concentration
gradients of ions;
Opening of Na+ channels allows Na+ influx and depolarization of membrane resulting
in an action potential;
Opening of K+ channels allows K+ efflux and repolarization of membrane;

Na+/K+ pump;
Actively transport ions against concentration gradient/ 3 Na+ out and 2 K+ in;
Maintains the resting membrane potential / maintain unequal concentration gradients
of Na+ and K+ across the membrane;
Along with the selective permeability of membrane to K+ over Na+ ions; OR
supported by more leak channels for K+ present than Na+ leak channels on the
membrane;

(c) Outline the significance of post-translational modifications of eukaryotic proteins. [6]

a) Cleavage and/or covalent modification


Give 1 suitable example - glycosylation, disulfide bond formation, attachment of
prosthetic groups etc. is required.

b) Form functional protein - newly synthesized proteins need to be modified for proper
assembly / functioning

c) Regulate - control cellular activity / influence biological activity

d) Eg: phosphorylation/dephosphorylation may activate/inactivate (ORA) proteins

e) Degrade proteins allows control of protein activities OR prevent aberrant activities so


that proteins will not stay too long in cytoplasm and still be active

f) E.g. Proteins are linked to ubiquitin that will target a protein for degradation.

g) (Save/recycle resources) - proteins not needed can be hydrolysed to amino acids, to


be used for synthesis of new proteins
1570

h) (Heterogeneity) - many different proteins modified from one polypeptide serve


different function so smaller no of proteins/ small genome needed

i) (Localisation) direct proteins to particular locations inside and outside cell

j) Eg: modifications at terminus of amino acid chain help target proteins for
transporting to final destination in the cell / move across membranes/ tag proteins to
be incorporated in various cellular and organelle membranes

Question 10
1571

a) Compare the Calvin cycle and the Krebs cycle. [6]

Similarities

I. Involves regeneration of the initial compound, Krebs cycle - oxaloacetate & Calvin
cycle - ribulose bisphosphate (RuBP);

II. Involve oxidation-reduction / redox reactions;

III. Involve co-enzymes as electron and proton carriers, Krebs cycle NAD+ & FAD, In
Calvin cycle reduced NADP / NADPH.

Differences

Features Calvin cycle Krebs cycle


Site stroma of chloroplast matrix of mitochondria
Nature of process Anabolic - formation of triose Catabolic - breakdown of
phosphate or starch acetyl coA
Role of carbon 1 molecule of CO2 is 2 molecules of CO2 is
dioxide accepted released
by RuBP catalyzed by RuBP by oxidative decarboxylation
carboxylase - oxygenase
Coenzymes involved Reduced NADP / NADPH +
NAD+ & FAD as electron and
H+
as electron and proton donor proton acceptors
Role of ATP Expenditure of ATP - in the Synthesis of ATP by substrate
reduction of PGA / GP to TP
/ level phosphorylation
GALP and in regeneration of
RuBP

b) Explain the importance of operons in bacteria. [4]


1572

1. Small genome
2. need to pack all essential genes;
3. save space;
4. Operons are a linear arrangement of genes and its regulatory sites;
5. arrangement allow genes that encode proteins that execute similar functions to
cluster together;
6. allows for proteins performing similar functions to be turn on together;
7. Allow for common mode of regulation;
Able to share common promoter;

c) Describe the processes through which genetic diversity can be achieved in bacteria.
[10]
mutation

Transformation (max 3)
1. Uptake of naked foreign DNA from surrounding external environment into
recipient bacteria cell
2. DNA binds to cell surface, facilitated by DNA binding proteins
3. Nuclease degrades one strand of double stranded foreign donor DNA, single
stranded DNA enters
4. Single stranded DNA is then integrated into the bacterial chromosome by
means of RecA proteins
5. Recombination between homologous regions
6. Can be either artificial or natural process
7. Cells must be competent.

Transduction (max 4)
8. Process by which DNA is transferred from donor bacterial cell to another
recipient by bacteriophages
9. Generalised and Specialised transduction both result from aberrations in the
phage reproductive cycle
10. Generalised transduction: random segment of host bacterial chromosome is
transferred to recipient bacterial cell
11. During assembly stage, a small piece of host cells degraded DNA is
packaged within the phage capsid in place of the phage genome.
12. Transducing phage infects recipient cell, injects the DNA acquired from donor
bacterium into recipient bacterium
13. Donor DNA is integrated into recipient bacterial chromosome by homologous
recombination.
14. Specialised transduction: occurs in temperate bacteriophages, involves
transfer of a specific set of bacterial genes from donor to recipient bacterium.
15. In the lysogenic cycle, the genome of a temperate phage integrates as a
prophage into the host bacteriums chromosome at a specific site.
16. When the phage genome is excised from the host bacteriums chromosomes,
sometimes a small region of the host bacterial DNA adjacent to the prophage
is excised together.
17. The phage DNA that incorporated some bacterial genes replicates and is
packaged to form a specialized transducing phage.
1573

18. Specialized transducing phage released during host cell lysis to infect a
recipient bacterial cell.
19. Foreign bacterial DNA subsequently integrated into the recipient bacterial
chromosome by homologous recombination.

Conjugation (max 3)
20. Transfer of F plasmid from F+ donor to F- recipient cell
21. Via sex pilus / conjugation tube encoded by genes in F plasmid
22. Sex pilus attaches to F- cell and retracts, forming cytoplasmic bridge
23. Nuclease cleaves one strand of double stranded F plasmid at Ori, single
stranded F DNA of F plasmid transferred
24. Rolling circle replication
25. Both cells end up with F plasmid and are thus both F+.
1574

CANDIDATE NAME (CG) _________________________ INDEX NUMBER _________

SERANGOON JUNIOR COLLEGE


JC2 Preliminary Examination 2013
Paper 3
BIOLOGY
Higher 2

9648
26 August 2013 / Monday
2 hours
Additional materials:
Answer paper

READ THESE INSTRUCTIONS FIRST

Write your name and index number in the spaces at the top of this page and on all
the work you hand in.

Write in dark blue or black pen. FOR EXAMINERS USE


1 /11
2 /4
You may use a soft pencil for any diagrams, graphs or rough
3 /15
working. 4 /10
Planning /12
Do not use staples, paper clips, highlighters, glue or correction Essay /20
fluid. TOTAL /72

Answer all questions in all the sections.

At the end of examination,


1. Fasten all your work securely together;

INFORMATION FOR CANDIDATES


The intended number of marks is given in brackets [ ] at the end of each question or
part question.

___________________________________________________________________
This question paper consists of 17 printed pages and 1 blank page

1
1575

Question 1

a) Restriction fragment length polymorphism, RFLP, refers to differences in fragment


length generated in different individuals by restriction enzymes.

Figure 1.1 below shows an autosomal dominant disease segregating in a family.


Genomic DNA from each family member was assayed for RFLP for EcoRI sites, on
Southern blots using probes derived from the gene.

The results for all individuals, except those for the foetus, are shown below the
pedigree chart. The sizes of the DNA fragments obtained are as indicated.

Figure 1.1

i) The diagram below shows the 2 possible alleles of the gene. Indicate with
thick straight lines on the diagram the possible location recognized by the
probe that was added. [1]
RS RS

RS RS RS

RS= restriction site

ii) Which RFLP fragment did the mother (I-1) contribute to the diseased children?
[1]

2
1576

iii) What is the probability of the foetus having the disease? Explain your answer.
[2]

b) A different study was carried out on another family. Figure 1.2 below shows a
Southern blot with an RFLP detected by using a probe for a sequence located on
a sex chromosome. The variant exhibits two forms, one at 8 kb and one at 6 kb.
The mother, father and daughter are normal.

Mother Father Daughter Son 1 Son2

Figure 1.2

i) From Figure 1.2, deduce which son is abnormal and explain how you arrive
at your answer. [3]

3
1577

c) RFLP is widely used as a tool for disease detection. In disease detection, genetic
markers are often used to predict if a person is likely to suffer from a particular
disease.

i) Explain the features of genetic markers that allowed them to be used for the
indirect analysis of the inheritance of disease. [2]

..

..

..

..

..

..

d) DNA fingerprinting through the use of RFLP is a technique employed by forensic


scientists to assist in the identification of individuals by their respective DNA profiles.

i) Explain why genes, especially protein-encoding genes, are seldom used for genetic
fingerprinting. [2]

..

..

..

[Total: 11 marks]

4
1578

Question 2

Flavr Savr is a genetically modified tomato. It is made more rot-resistant by adding


an antisense gene to interfere with the production of the enzyme polygalacturonase
(PG). This enzyme causes the softening of the fruit by degrading pectin.

PG production during fruit ripening was measured in plants with one and with two
antisense genes, and in normal plants. The results are shown in Figure. 2.2.

Figure 2.2

i) With reference to Figure 2.2, describe and explain the difference between normal
tomato plants and transformed plant with one anti-sense gene. [4]

..

..

..

..

..

..

[Total: 4 marks]

5
1579

Question 3

a) Cystic Fibrosis (CF) is an autosomal recessive genetic disorder that affects


predominantly the lungs and other mucosal organs in the body. A technique that
measures the nasal potential difference has been investigated recently as a possible
diagnostic tool. When chloride ion channels open, a potential difference will be
recorded as a positive spike in the potential difference against time graph.

In the lab, laboratory researchers performed experiments on two groups of nasal


epithelial cells, X and Y. To observe changes in nasal potential difference, the
researchers perfused a solution of amiloride (Am) into the nasal activity, followed by
the subsequent perfusion of the nasal cavity with a low chloride solution (LC).

Amiloride serves to block sodium absorption, resulting in a reduction of the potential


difference. This provides an electrical gradient for the secretion of chloride, which is
stimulated by the biochemical gradient provided by a low chloride solution.

The potential difference against time graph was plotted as shown in Figure 3.1 below.

Figure 3.1

i) Explain what is meant by autosomal recessive genetic disorder. [2]

..

..

..

..

..

6
1580

ii) With reference to Figure 3.1, identify the cell with cystic fibrosis. Explain your
answer. [3]

..

..

..

..

..

..

iii) Describe how cystic fibrosis can be cured by gene therapy using a non-viral
vector. [4]

..

..

..

..

..

..

..

iv) Suggest two technical challenges preventing the effectiveness of gene therapy
for cystic fibrosis. [2]

..

..

..

..

7
1581

b) The procedure of current gene therapy for severe combined immunodeficiency


disease (SCID) is illustrated in Figure 3.2 below.

Figure 3.2

i) State two advantages of using the ex vivo method for gene therapy that is
illustrated in Figure 3.2. [2]

..

..

..

ii) Suggest and explain how you can improve the above gene therapy procedure in
order to sustain long-term success of the treatment. [2]

..

..

..

[Total: 15 marks]

8
1582

Question 4

a) There has been rampant speculation that the consumption of genetically modified
(GM) crop and livestock has impacted birth rates in the USA negatively. A study was
carried out in the USA to investigate the claim. Figure 4.1 shows a graphical plot of
the data obtained.

Figure 4.1

i) With reference to Figure 4.1, comment whether the speculation can be


justified by the data. [3]

ii) Describe two threats that GM food can pose to human health. [2]

9
1583

b) The effectiveness of CpTi against insect pests was tested using two groups of
tobacco plants.

Plants genetically engineered to express the allele coding for CpTi.


Control plants which had been genetically engineered by inserting the allele
coding for CpTi in the wrong orientation. These plants did not produce CpTi.

Equal numbers of each type of plant were infested with equal numbers of newly
hatched larvae of an insect pest and left for seven days.

The mean percentage of leaf area eaten and the mean mass of surviving insects
were then found. The results are shown in Figure. 4.2.

Figure 4.2

With reference to Figure 4.2,

i) Describe the effect of CpTi on the yield of tobacco leaves. [2]

10
1584

ii) CpTi is a trypsin inhibitor. Suggest an explanation of the effect of the trypsin
inhibitor, CpTi, on insects.[3]

[Total: 10 marks]

11
1585

Planning Question (12m)

A millionaire has recently passed on and news of his death has become the talk of
the town. The deceased old man was known to be childless and thus his massive
fortune amounting $100 million was willed and donated to help the less fortunate
orphans at Chen Su Lan Childrens Home.

However, three days after his death, a man named Bernard Soon has stepped
forward and claimed to be his son. According to Bernard Soons allegations, the
deceased millionaire had an affair with his mother during a year-long work trip in the
1930s. However, both had lost contact over the years and had not met since. Mr
Bernard Soon has claimed that he has a picture of the millionaire and his mother, as
well as his mothers ring with the millionaires initials engraved onto it. Due to the
large amount of money involved, this case has been covered at length by the media.

Bernard Soon has hired you as the lead researcher to prove his identity. Plan an
investigation using RFLP analyses to confirm that Bernard Soon is indeed the son of
the millionaire.

Your planning must be based on the assumption that you have been provided the
following equipment and materials:

Tissue sample from the millionaire and Bernard Soon


Pestle and mortar
DNA extraction buffer solution
Microcentrifuge tubes
Centrifuge
Restriction enzymes
Agarose or polyacrylamide gel plate
Suitable source of electric current
Radioactive probe
Nitrocellullose membrane
Autoradiography equipment

Your plan should have a clear and helpful structure to include:

An explanation of the theory to support your practical procedure


A description of the method used including scientific reasoning behind the
method
The type of data generated by the experiment
How the results will be analysed including how the origin of the organism can
be determined

12
1586

13
1587

14
1588

15
1589

16
1590

Free-response question (20 marks)

Write your answer on the separate answer paper provided.

Your answers should be illustrated by large, clearly labeled diagrams, where


appropriate.

a) Outline the large scale production of a named commercially important protein by


genetic engineering. [10]

b) Using specific example(s), compare viral and non-viral delivery systems in the
delivery of a functional CFTR allele.[6]

c) Describe the advantages and limitations of the Polymerase Chain Reaction (PCR).
[4]

END OF PAPER

17
1591

BLANK PAGE

18
1592

Question 1

a) Restriction fragment length polymorphism, RFLP, refers to differences in fragment


length generated in different individuals by restriction enzymes.

Figure 1.1 below shows an autosomal dominant disease segregating in a family.


Genomic DNA from each family member was assayed for RFLP for EcoRI sites, on
Southern blots using probes derived from the gene.

The results for all individuals, except those for the foetus, are shown below the
pedigree chart. The sizes of the DNA fragments obtained are as indicated.

Figure 1.1

i) The diagram below shows the 2 possible alleles of the gene. Indicate on the
diagram the possible location recognized by the probe that was added. [2]

RS RS

RS RS RS

RS= restriction site

Which RFLP fragment did the mother (I-1) contribute to the diseased children? [1]

10 kb fragment
1593

ii) What is the probability of the foetus having the disease? Explain your answer. [2]

50% or probability
Mothers genotype is heterozygote; fathers genotype is homozygous recessive
so chance of receiving dominant allele

b) A different study was carried out on another family. Figure 1.2 below shows a
Southern blot with an RFLP detected by using a probe for a sequence located on a
sex chromosome. The variant exhibits two forms, one at 8 kb and one at 6 kb. The
mother, father and daughter are normal.

Figure 1.2

From Figure 1.2, deduce which son is abnormal and explain how you arrive at our
answer. [3]

1. Son 1

2. Sex linked disease. Y chromosome does not carry any allele FOR SEX LINKED
diseases. Hence male should only exhibit one band.

3. Son 1 is a case of XXY/ Klinefelters syndrome

c) RFLP is widely used as a tool for disease detection. In disease detection, genetic
markers are often used to predict if a person is likely to suffer from a particular
disease.

i) Explain the features of genetic markers that allowed them to be used for the
indirect analysing of the inheritance of disease. [3]

1 known sequence (DNA/gene); with known location on a chromosome;

2 genetic marker found close to the gene of interest;;

3 3 little probability of crossing over


hence presence of genetic marker = presence of disease
1594

d) DNA fingerprinting through the use of RFLP is a technique employed by forensic


scientists to assist in the identification of individuals by their respective DNA profiles.

i) Explain why genes especially protein-encoding genes are seldom used for genetic
fingerprinting. [2]

1. sequence of protein-encoding genes are conserved; due to its important biological


function;

2. not able to differentiate between individuals if you use protein encoding region

[Total: 11 marks]
1595

Question 2

Flavr Savr is a genetically modified tomato. It is made more rot-resistant by adding


an antisense gene to interfere with the production of the enzyme polygalacturonase
(PG). This enzyme causes the softening of the fruit by degrading pectin.

PG production during fruit ripening was measured in plants with one and with two
antisense genes, and in normal plants. The results are shown in Figure. 2.2.

Figure 2.2

With reference to Figure 2.2, describe and explain the difference between
normal tomato plants and transformed plant with one anti-sense gene. [4]

PG production in normal plants higher than plants with one antisense


gene, min ~4 a.u./max~9 a.u. as compared to about 2 a.u.;

antisense gene is being expressed in transformed plant cells, resulting in


formation of duplex RNA;

less translation initiation complex formation / decreased binding of


ribosome to mRNA;

resulting less translation of mRNA into PG proteins;


1596

Question 3

a) Cystic Fibrosis (CF) is an autosomal recessive genetic disorder that affects


predominantly the lungs and other mucosal organs in the body. A technique that
measures the nasal potential difference has been investigated recently as a possible
diagnostic tool. When chloride ion channels open, a potential difference will be
recorded as a positive spike in the potential difference against time graph.

In the lab, researchers performed experiments on two groups of nasal epithelial cells,
X and Y. To observe changes in nasal potential difference, the researchers perfused
a solution of amiloride (Am) into the nasal activity, followed by the subsequent
perfusion of the nasal cavity with a low chloride solution (LC).

Amiloride serves to block sodium absorption, resulting in a reduction of the potential


difference. This provides an electrical gradient for the secretion of chloride, which is
stimulated by the biochemical gradient provided by a low chloride solution.

The potential difference against time graph was plotted as shown in Figure 3.1 below.

Figure 3.1

i) Explain what is meant by autosomal recessive genetic disorder. [2]


Condition arises only when both alleles of the gene are mutated/ Only
expressed in homozygotes
Gene is carried on chromosomes other than sex chromosomes.
1597

ii) With reference to Figure 3.1, identify the cell with cystic fibrosis. Explain your
answer. [3]
Cell Y
The graph for cell Y did not show a positive spike/ change in potential
difference unlike the graph for cell X when LC (low chloride solution) was
perfused into nasal cavity, hence showing that chloride channels failed to
open properly and chloride ions could not exit cell.
Due to mutation of the CFTR gene that encodes a mutant CFTR channel
protein that does not allow movement of chloride ions thus no change in
potential difference.

iii) Describe how cystic fibrosis can be cured by gene therapy using a non-viral
vector. [4]
In vivo gene therapy method
Functional allele coding for CFTR on chromosome 7 is isolated from a
normal individual
Aerosol spray containing liposomes with the normal allele can be sprayed
into the nose and moith of CF patients
Membrane of liposomes fuse with the cell membrane of target lung
epithelial cells, releasing the normal CFTR allele into cytoplasm via
endocytosis
Normal CFTR protein is expressed and incorporated into target cells
membrane.

iv) Suggest two technical challenges preventing the effectiveness of gene therapy
for cystic fibrosis. [2]
Gene therapy is short-lived as target cells for CF treatment are epithelial
cells that are constantly being shed, thus repeat therapy is likely to be
required.
Use of viral vectors (adenovirus) can trigger host immune response.
Other less accessible organs like the liver and pancreas are also affected
by CF and is difficult for therapy to reach effectively.
Liposomes are not specific in their action and thus have a low
transfection efficiency.
(max 2)
1598

b) The procedure of current gene therapy for severe combined immunodeficiency


(SCID) is illustrated in Figure 3.2 below.

Figure 3.2

i) State two advantages of using the ex vivo method for gene therapy that is
illustrated in Figure 3.2. [2]
Allows for selection of successfully transfected cells;
Ease of transfection and culture of T lymphocytes in the lab;
Specific approach ensures that retrovirus targets the T lymphocytes
instead of other cells in the body.

ii) Suggest and explain how you can improve the above gene therapy procedure in
order to sustain long-term success of the treatment. [2]
Use blood stem cells extracted from bone marrow apart instead of T
lymphocytes.
Blood stem cells are able to differentiate into T lymphocytes and show
long-term self renewal so as to provide a sustained cure
1599

Question 4

a) There has been rampant speculation that the consumption of genetically modified
(GM) crop and livestock has impacted birth rates in the USA negatively. A study was
carried out in the USA to investigate the claim. Figure 4.1 shows a graphical plot of
the data obtained.

Figure 4.1

i) With reference to Figure 4.1, comment whether the speculation can be


justified by the data. [3]
Speculation cannot be justified
From 1910 to 2009, the US crude birth rate has been on a decline from
30.1 to 13.5 with only two exceptions between 1940 to 1950 and 1980 to
1990.
Also, commercialisation of GMOs in 1996 did not lead to a significant
decline in birth rate ten years later in 2006 (approximately 15.0 to 13.0),
as compared to the sharpest decline in birth rate from 1920 (27.7) to
1930 (21.3), thus shows that other factors could have been responsible
for the declining birth rate.

ii) Describe two threats that GM food can pose to human health. [2]
Vectors used in the process of engineering GM food might contain
antibiotic resistance that may be passed on to bacteria like E Coli in the
human gut.
GM food might contain allergens and can cause unwanted allergic
reactions in the body.
1600

b) The effectiveness of CpTi against insect pests was tested using two groups of
tobacco plants.

Plants genetically engineered to express the allele coding for CpTi.


Control plants which had been genetically engineered by inserting the allele
coding for CpTi in the wrong orientation. These plants did not produce CpTi.

Equal numbers of each type of plant were infested with equal numbers of newly
hatched larvae of an insect pest and left for seven days.

The mean percentage of leaf area eaten and the mean mass of surviving insects
were then found. The results are shown in Figure. 4.2.

Figure 4.2

With reference to Figure 4.2,

(i) Describe the effect of CpTi on yield of tobacco leaves. [2]


Has a smailler mean percentage of leaf area eaten
52% in control, 21% in plants producing CpTi

(ii) Suggest an explanation of the effect of the trypsin inhibitor, CpTi, on insects.
Refer to the lack of digestion of protein.
Enzyme inhibition, hence active sites not able to bind to substrate.
Insects would dies of malnutrition or starvation

[Total: 10 marks]
1601

Section B (20 marks)

Free-response question

Write your answer on the separate answer paper provided.

Your answers should be illustrated by large, clearly labeled diagrams, where


appropriate.

a) Outline the large scale production of a named important protein by genetic


engineering. [10]

1. Construction of recombinant DNA molecule (foreign + vector DNA)


Source of gene - mRNA for insulin [2 marks max]
Isolate mRNA from beta cells of pancreas;
RT is used to reverse transcribed mRNA to form complementary DNA
(cDNA);
DNA pol then synthesizes the second DNA strand;
Mentioned use of adaptors/ linkers to create sticky ends (since cDNA will be
blunt);

b. Creation of recombinant DNA molecule [2 marks max]


Vector and insulin gene must be cleaved with same restriction enzyme at
specific recognition sequence/site;
So that complementary base pairing can form via hydrogen bonds between
insulin gene fragment and vector;
Example (of vector/ restriction enzyme);
DNA ligase is added to seal phosphodiester bond;

2. Introduction of recombinant DNA molecule into living host (e.g. bacteria) [2 marks
max]
Via transformation;
Method of transformation;
Reason of method (to create permeability/ transient enlargement of pores);
Purpose of living host (provide enzyme machinery);

3. Multiplication of recombinant DNA molecule and cell growth/ division; [1 mark]

4. Screening/ Selection & identification of clones with recombinant DNA molecules [2


marks max]
Insertional inactivation;

Lac Z/ Blue-white selection or antibiotic selection;


Description of either lac Z or antibiotic selection;
1602

5. Purification of insulin product [2 marks max]


Upscale/ production/ fermentor/ bioreactor (max 1 mark);

Post-translational modification/ cleave fusion polypeptide with CNBr / combining


the 2 chains/polypeptide via disulphide bonds; [max 1 mark]

b) Using specific example(s), compare viral and non-viral delivery systems in the
delivery of a functional CFTR allele.[6]

Introductory statement:

Viral delivery systems for treatment of cystic fibrosis involves the use of
adenoviruses and adeno-associated viruses; Non-viral delivery system for
treatment of cystic fibrosis involves use of liposomes OR gene gun;

Similiarities:

Both adeno-associated viruses and liposomes do not produce immune


response in the body. [1]
Both adenoviruses and non-viral do not result in long term expression of
the gene and these methods does not result in integration of normal gene
into host cell genome. [1]

Differences:

Basis of comparison Viral methods non-viral methods

effectiveness of adenoviruses naturally both liposome [1]


gene transfer infect lungs, and hence are and gene gun are
effective for delivering the less effective
gene
long term use of adeno-associated low [1]
expression of gene virus can result in
integration of normal gene
into host cell genome, and
may result in long term
expression of gene.
immune response use of adenoviruses can minimal host [1]
cause body to produce immune
immune response response is
generated
side effects not known minimal [1]
1603

c) Describe the advantages and limitations of the Polymerase Chain Reaction (PCR).
[4]

Advantage:
sensitive;
amplification possible from trace amount of template DNA;
fast/efficient method of testing as 35 40 PCR cycles can be completed in several
hours/(ref to short times required for each PCR segment); [R: fast/efficient without any
elaboration];
use of thermostable DNA polymerase allows for automation of procedure without need
to add polymerase for every cycle;
allows amplification/replication of selected target region of DNA flanked
by/usingspecific PCR primers;
Programmable thermal cycler allows automation of PCR;
faster method of amplifying/cloning DNA sequence than conventional cell-based
cloning;

Reject: create many copies of DNA using small sample, sensitive allowing amplification of
small DNA

Disadvantage:
sensitive to contamination of template DNA by foreign/non-target nucleic acids;
may give false positives;
requires careful handling to prevent contamination;
by non-target nucleic acids;
Use of Taq DNA polymerase; may result in misincorporation errors in PCR products; as
(Taq DNA polymerase) lacks proof-reading activity / 35 exonuclease activity (to correct
misincorporated bases);
Needs preliminary knowledge of part of/nature of target sequence; before specific primers
can be designed;
Mutations in regions complementary to primer(s) may cause reduction in
annealing/amplification efficiency;
PCR products labile/unstable as they are not associated with histones;
1604

H2 P3 Planning Question - Mark Scheme Summary

Theoretical basis of experiment [max 3]:

1. Differences in DNA sequences among different individuals differences in


the number of restriction site locations
2. Basis of RFLP: SAME Restriction enzyme digest different number and
length of restriction fragments
3. If Bernard Soon is indeed the child of the billionaire, then his RFLP bands
should match either the billionaires or his mothers/his bands should eb
accounted for by the billionaire or his mother..

Procedure:

Extraction of DNA [max1]

1. Break the tissues: pestle and mortar.


2. Lyse cells: DNA extraction buffer.
3. Centrifuge, add RNase and protein precipitation buffer obtain pure DNA

Polymerase Chain Reaction [max 1]

4. Taq polymerase.
5. Heat DNA mixture (95 oC)
separate double-stranded DNA into 2 complementary single-stranded DNA.
break hydrogen .
6. Cool (55 oC)
primer hybridization to DNA template, primer provide 3OH end
7. (72 oC) Replication and extension of the DNA in 5 to 3 direction
8. Each cycle is repeated. exponential increase of target DNA sequences.

Restriction Enzyme Digestion [max 1]

9. Use same restriction enzyme yield restriction fragments.

Gel Electrophoresis [max 1]

10. Negatively charged DNA molecules will migrate toward the positive end of the
field (anode).
11. Separated according to their molecular size.
12. The smaller molecules move faster than the larger one

Southern Blot [max 1]

13. Transfer DNA bands to a piece of nitrocellulose membrane by capillary action


14. The nitrocellulose membrane: replica of banding pattern
1605

Probing [max 1]

15. Denature DNA into single-strands using NaOH


break hydrogen bonds between the complementary base pairs of double
helix
16. Add single stranded radioactively labelled DNA probe complementary to
gene of interest..

Visualization [max1]

17. The hybridized probe is detected by autoradiography.


18. Exposure to X-ray image of radioactively labelled DNA bands.

Comparing DNA bands [max 1]

19. Compare DNA banding patterns

Conclusion [max1]

20. If Bernard Soon is indeed the child of the billionaire, then his RFLP bands
should match either the billionaires or his mothers/his bands should eb
accounted for by the billionaire or his mother.
1606

TAMPINES JUNIOR COLLEGE

H JC2 PRELIMINARY EXAMINATION

2
CANDIDATE
NAME

CIVICS GROUP 1 2
TUTOR
NAME

Biology 9648/ 01
Paper 1 Multiple Choice Tuesday, 24 September 2013
1 hour 15 minutes
Additional Materials: Optical Answer Sheet (OAS)

READ THESE INSTRUCTIONS FIRST

Write your name and Civics Group in the spaces at the top of this page and on any separate writing
paper used.

There are 40 questions in this paper.


Answer all questions. For each question there are four possible answers A, B, C and D.
Choose the one you consider correct and record your choice in soft pencil on the separate OAS.

Read the instructions on the OAS very carefully.

Each correct answer will score one mark. A mark will not be deducted for a wrong answer.
Any rough working should be done in this booklet.

This document consists of 19 printed pages and 1 blank page


1607
2
MULTIPLE CHOICE QUESTIONS (40 MARKS)
1. An electron micrograph of a cell is shown below.

J E

Match the organelles E, F, G, H and J associated with the cellular processes listed.

E F G H J

A DNA replication Digestion of Organizes the Oxidative Packaging of


material spindle phosphorylation secretory products

B Oxidative Organizes the Digestion of DNA replication Packaging of


phosphorylation spindle material secretory products

C Organizes the Digestion of Oxidative Packaging of DNA replication


spindle material phosphorylation secretory products

D DNA replication Organizes the Packaging of Oxidative Digestion of


spindle secretory products phosphorylation material

TPJC 9648/01/JC2 Prelim/2013 [Turn Over


1608
3
2. Which row correctly links a feature of a phospholipid molecule with its function in
membranes?
Both polar and
non-polar Phosphate group Unsaturated fatty Saturated fatty
(amphipatic) acids acids

A Allowing diffusion Aiding Decreasing Binding proteins


of small membrane fluidity
molecules stability

B Forming a bilayer Repelling Increasing fluidity Decreasing


charged fluidity
molecules
C Forming Binding proteins Increasing Increasing fluidity
glycolipids permeability
D Forming vesicles Forming Binding proteins Decreasing
glycolipids permeability

3. The diagram below shows the structure of a general transcription factor.

Which correctly describes the structure of the part enclosed in box Z?

Number of Amino Acids Present Types of bonds Present

A Cannot be determined Hydrogen Bonds only


B Cannot be determined Hydrogen Bonds and hydrophilic bonds
C Approximately 11 amino acids Hydrogen bonds only
D Approximately 11 amino acids Hydrogen bonds and hydrophobic bonds

TPJC 9648/01/JC2 Prelim/2013 [Turn Over


1609
4
4. Each polypeptide chain of haemoglobin contains a number of regions in the form of an
alpha helix.
Which feature of the haemoglobin molecule is responsible for this?
A bonding between the four polypeptide chains
B hydrophobic interactions at the centre of the molecule
C the amino acid sequence of the globin
D the presence of iron in the haem groups

Which of the following statement about enzymes is inaccurate?


5.

A Enzyme function is not influenced by factors such as lead nitrate.

B Enzyme function is reduced if the three-dimensional structure or conformation of an


enzyme is altered.

C Enzymes increase the rate of chemical reaction by lowering activation energy


barriers.

D Enzymes may require a non-protein cofactor or ion for catalysis to take place.

6. Which of the following does not describe the active site of an enzyme?

A Its specificity is defined by the arrangement of the monomers.


B It is usually a crevice or cleft.
C It initially binds substrates by weak attractions.
D It is two-dimensional in structure.

TPJC 9648/01/JC2 Prelim/2013 [Turn Over


1610
5
7.

Which of the following statements is true?

A The bond in 1 is phosphodiester bond.


B The bond in 5 is peptide bond.
C 7 refers to the 5 end of the mRNA.
D 8 is showing unwinding of the DNA molecule.

TPJC 9648/01/JC2 Prelim/2013 [Turn Over


1611
6
8. The following coding sequence of DNA is taken randomly from a bacterial genome.

3 TTACGCTTCGAAATAGGAATATCATAGGCT 5

This sequence is cloned into a plasmid and transformed into a suitable host. What would
be the first four amino acids of a polypeptide generated from this sequence as expressed
by the host?

The mRNA codons for some amino acids are shown in the table below:

Arg CGA, CGG, AGA, AGG Leu CUU, CUC, CUA, CUG
Asp GAU, GAC Lys AAA, AAG
Ile AUU, AUC, AUA Phe UUU, UUC
Start AUG Ser UCA, UCG, AGU, AGC
Stop UAG, UGA, UAA Tyr UAU, UAC

A Met-Arg-Ser-Phe
B Met-Arg-Ser-Lys
C Met-Ile-Phe-Leu
D Met-Tyr-Lys-Asp.

9. Which of the following is unique to mitosis and not a part of meiosis?

A Homologous chromosomes pair forming bivalent.

B Homologous chromosomes cross over.

C Chromatids are separated during anaphase.

D Homologous chromosomes behave independently.

10. Cells treated with colchicine often mutate. They fail to produce spindles during cell
division. What would be the result of this?
A chromosome deletion
B chromosome translocation
C gene deletion
D polyploidy

TPJC 9648/01/JC2 Prelim/2013 [Turn Over


1612
7
11. Four bacteriophage genes were mapped by recombination. The recombination
frequencies between the four genes are shown in the table below.

L M N P
L -- 0.18 0.10 0.20
M -- 0.15 0.37
N -- 0.24
P --

What is the order of the genes along the chromosome?

A L, M, N, P
B N, M, P, L
C M, L, P, N
D P, L, N, M

12. During a viral infection, attachment is usually specific to a particular cell type because

A the virus is attracted to the appropriate host cells by proteins secreted into the
extracellular fluid.
B the virus recognises and binds to specific molecules in the cytoplasm of the host
cell.
C the virus recognises and binds to specific molecules on the surface of the host
cell.
D the host cell produces channel proteins that provide passageways for the viruses
to enter the cytoplasm.

13. When glucose is absent and lactose is present, lactose binds to the operon's repressor.
Which statement best describes what happens subsequently?

A Repressor binds to promoter, transcription is initiated.


B Repressor binds to operator, transcription is not initiated.
C Repressor fails to bind to structural genes, transcription is not initiated.
D Repressor fails to bind to the operator, transcription is initiated

TPJC 9648/01/JC2 Prelim/2013 [Turn Over


1613
8
14. The diagram below shows 2 species of bacteria. The bacteria that are shaded are
resistant to the antibiotic penicillin.

Species 1 Species 2

A D

B C E F

Which one of the following statements is likely to be true?

A Bacterium D is resistant to penicillin due the transfer of the complete F plasmid and
penicillin resistant gene through transduction involving Bacterium A.
B Bacterium D is resistant to penicillin due to horizontal gene transfer involving a
bacteriophage and Bacterium A.
C Bacterium E acquired resistance to penicillin as a result of random mutation and
genetic transformation.
D Bacteria E and F are resistant to penicillin as a result of conjugation from Bacterium
D.

15. The statements are descriptions of aspects of genetics.

1. The phenotype is affected by both alleles at the same locus of a heterozygous


individual.
2. The combined effects of alleles at two or more gene loci equal the sum of their
individual effects.
3. Many different alleles present in a gene pool can occupy the same gene locus.
4. Alleles of one gene mask the effects of the alleles of another gene at a
different locus.

Match the description to each of the following terms

codominant alleles epistatic alleles multiple alleles additive genes

A 1 4 2 3

B 1 4 3 2

C 2 1 4 3

D 3 1 4 2

TPJC 9648/01/JC2 Prelim/2013 [Turn Over


1614
9

16. Skin colour in a variety of pumpkins is controlled by two pairs of alleles, Pp and Rr, which
segregate independently.

The allele P is dominant and must be present for the development of pigmentation in the
skin. Pumpkins without pigment are white. Allele R is dominant and produces a red
pigment in the skin. The recessive allele r gives a yellow colour.

In a testcross with a parent of unknown genotype, a phenotypic ratio of 1 red : 1 white


was obtained.

What was the genotype of the unknown parent?

A PPRR

B PpRR

C PPRr

D Pprr

17. Studies on the giant axon of the squid showed that bacteria toxin BAXIN results in an
increase in negativity of the postsynaptic membrane. What is the possible mode of action
of BAXIN?

A The release of acetylcholine

B Analog of acetylcholine

C Release of potassium ions from postsynaptic membrane

D Mimicking of cholinesterase

18. The diagram shows the sequence of events occurring as an action potential arrives at the
synapse. The numbered arrows represent the movement of substances across the
membranes.

What are the substances moving across the membranes?

1 2 3 4 5
A K+ Na+ acetylcholine Ca+ K+
B K+ Na+ Ca+ acetylcholinesterase acetylcholine
C Na+ K+ Ca+ acetylcholine Na+
D Na+ K+ acetylcholinesterase acetylcholine Ca+

TPJC 9648/01/JC2 Prelim/2013 [Turn Over


1615
10

19. The diagram shows how a nerve cell and an endocrine cell interact with cell X and cell Y
respectively.

Which of the following is true of both interactions?

A It involves the movement of chemicals M and N to their respective target cells.


B Chemical M is secreted faster than chemical N.
C When both cells X and Y are stimulated, phosphorylation cascades must be
initiated.
D Both chemicals M and N must bind to cell-surface receptors on cell X and Y
respectively.

20. In the experiment shown in the diagram, the oxygen given off at X and Y will be labelled
with the isotopes _____ and _____ respectively.

X Y
16 16
A O O
16 18
B O O
18 16
C O O
18 18
D O O

TPJC 9648/01/JC2 Prelim/2013 [Turn Over


1616
11
21. The weed killer DCMU blocks the flow of electrons from the electron transport chains in
photophosphorylation.
Why does this kill the plant?
A Active transport of mineral ions is prevented.
B ATP and reduced NADP are not produced.
C Photoactivation of the chlorophyll cannot occur.
D Photolysis of water does not occur.

22. To what extent can glucose be oxidised by the soluble portion of cytoplasm of muscle
cells, from which all organelles have been removed?
A Acetyl-CoA but no further
B Carbon dioxide and water
C Lactate but no further
D Pyruvate but no further

23. In which of the following changes is most energy released during the metabolism of
carbohydrates?
A Maltose to glucose

B Glucose to phosphoglyceraldehyde

C Phosphoglyceraldehyde to pyruvic acid


D Pyruvic acid to carbon dioxide and water

24. Which of the following is true of gene regulation in both prokaryotes and eukaryotes?

A Gene regulation of eukaryotes involves bending of DNA while in prokaryotes,


chemical modifications to DNA is required.
B In eukaryotes, increase in efficiency of RNA polymerase requires specific
transcription factors to activate the initiation complex while in prokaryotes,
activators are required to bind directly to promoter site to increase efficiency of
RNA polymerase.
C In eukaryotes, RNA polymerase binds to the TATA box while in prokaryotes, RNA
polymerase binds to the regulatory gene.
D Repressors and activators regulate expression in eukaryotes while prokaryotes are
regulated by repressors only.

25. The following events occur in the extension of telomeres.

TPJC 9648/01/JC2 Prelim/2013 [Turn Over


1617
12
1 Further extension of 3 end of telomere.
2 Translocation of telomerase to end of telomere.
3 Reverse transcription to extend the 3 end of the telomere.
4 Complementary base-pairing of RNA template with single-stranded end of
telomere.
5 DNA polymerase catalyses formation of complementary strand to form double-
stranded DNA.
In which order do these events take place?

A 3 2 4 5 1
B 3 4 2 1 5
C 4 2 3 5 1
D 4 3 2 1 5

26. As shown in the diagram below, acetylation of histones promotes loose chromatin
structure. Recent evidence has shown that chemical modification of histones play a direct
role in regulation of gene expression. Suggest how this works.

A Helicase action is enhanced by acetylation.


B Acetylation of histones neutralizes their negative charges and encourages binding to
DNA polymerase.
C When nucleosomes are highly acetylated, chromatin becomes less compact and
DNA is more accessible for transcription.
D RNA polymerase works better by binding with acetyl groups.

27. The hormone insulin binds to the tyrosine kinase receptors and initiates various signal
transduction pathway to generate cellular responses. Which of the following shows the
TPJC 9648/01/JC2 Prelim/2013 [Turn Over
1618
13
correct sequence of events, following the binding of insulin to the receptor?

a. phosphorylation of tyrosine kinases


b. dimerization of tyrosine kinase receptor
c. activation of enzyme

A a,b,c
B c,b,a
C b,a,c
D c,a,b

28. Many signal transduction pathways use second messengers to

A amplify the message by directly phosphorylating proteins.


B induce a signal from the outside to the inside of the cell.
C relay the message from the inside of the membrane throughout the cytoplasm.
D transport a signal through the lipid bilayer portion of the plasma membrane.

29. Which of the following is a similarity between G-protein-linked receptors and tyrosine-
kinase receptors?

A activation that results from binding of GTP


B formation of a dimer following binding of a signal molecule
C regions of the receptor that span the plasma membrane
D signal-binding sites specific for steroid hormones

30. Which of the following increases the number of different alleles in a population?

A crossing over
B gene mutation
C random fusion of gametes
D random assortment of chromosomes in meiosis

31. The graph below shows data on a population of a species of moth which shows

TPJC 9648/01/JC2 Prelim/2013 [Turn Over


1619
14
considerable variation in colour intensity. Which conclusion can be made from this
graph?

A Colour variation is environmentally induced.


B Colour variation is genetically determined.
C Extreme forms are favoured by natural selection.
D The species shows discontinuous variation with respect to colour.

32. Approximately 1 in 20 Europeans are heterozygous for a recessive allele responsible for
the genetic condition, cystic fibrosis (CF). People who are homozygous for CF have a
reduced life expectancy. Heterozygotes are more resistant to some bacterial infections of
the gut, such as typhoid fever, than homozygotes for the normal, dominant allele.

What could explain the high incidence of the recessive CF allele in the European
population?

A natural selection favouring heterozygotes

B natural selection favouring homozygotes for the recessive CF allele

C lack of genetic drift in the European population

D Mutation

33. What is one similarity and one difference between classification and phylogeny?

TPJC 9648/01/JC2 Prelim/2013 [Turn Over


1620
15

A Both organise organisms into groups based on their characteristics but phylogeny
takes into consideration the evolutionary relationships between groups.

B Both put organisms together according to increasingly inclusive groups but


classification does not show a branching pattern of evolutionary relationships.

C Both rely on physical characteristics of organisms but phylogeny does not require
scientific names.

D Both take into account the common ancestors of organisms but classification is
hierachical in nature.

34. Which uses of the information from the human genome project are generally considered
to be unethical?

1. an insurance company only giving cheap rates to people with genetic


predispositions to fewer diseases
2. genetic archaeologists identifying the earliest forms of genes to show evolutionary
relationships
3. cytologists developing tests for only some defective genes
4. doctors only giving specific drugs to block the actions of faulty genes to carriers of
those genes
5. genetic counselors giving specific lifestyle information only to people genetically
predisposed to risks
6. parents choosing embryos for implantation only after ante-natal tests for
acceptable genes

A 1 and 3
B 1 and 6
C 2 and 5
D 3 and 4

35. The insulin gene and a plasmid with ampicilin and tetracycline resistant genes were cut

TPJC 9648/01/JC2 Prelim/2013 [Turn Over


1621
16
using the same restriction enzyme, EcoRI. The cut plasmid and insulin gene was mixed
together and DNA ligase was allowed to react. Subsequently the mixture was heat shocked
at 42 degrees with E. coli. The mixture was then plated on nutrient agar to obtain a master
plate with colonies shown below.

Master plate (nutrient agar)

Three replica plates containing different antibiotics were obtained from the master plate.
The diagram below shows the results of the replica plates.

Nutrient agar with Nutrient agar with Nutrient agar with


ampicillin tetracycline tetracycline and
ampicillin

Which of the following can be concluded from the results obtained?

A The ampicilin resistant gene is inactivated by insertion of the insulin gene


B The tetracycline resistant gene is inactivated by insertion of the insulin gene.
C The tetracycline resistant gene does not act as a selectable marker
D The ampicillin resistant gene does not act as a selectable marker.

36. A student performed an experiment to determine if a particular gene has been inserted in

TPJC 9648/01/JC2 Prelim/2013 [Turn Over


1622
17
a genetically modified organism.
i. Transfer of DNA to nitrocellulose membrane
ii. Restriction digestion of genomic DNA
iii. Cleaved DNA separated using gel electrophoresis
iv. Create radioactive probe
v. Incubate probe and membrane

Which is the correct sequence to the above steps?

A ii iii i v iv.
B ii iii i iv v.
C iv v i ii iii.
D iii i ii iv v.

37. The genetically engineered super salmon was created from Atlantic salmon stocks and
are capable of growing to a large size in 14 months. Which of the following is not a
benefit intended from the crop?

A Higher yield for farmers

B Minimising pollution

C Decreasing the food consumption of the crops in their lifetime

D Increase in supply to meet worlds demand

TPJC 9648/01/JC2 Prelim/2013 [Turn Over


1623
18
38.

Which of the following options is not a characteristic of the form of treatment shown
above?

1 Easy to extract in laboratory


2 Readily available
3 Difficult to control
4 Limited longevity

A 1 only B 1 and 2 C 1 and 3 D 1, 2, 3 and 4

39. Germ-line gene therapy has not been prevalent in medical treatment because of many
reasons. Which of the following is/are implications of germ-line therapy?

1 unforeseeable clinical risks


2 opens problems for social discrimination
3 cost
4 issue of consent

A 1 only B 2, 3 and 4 C 1, 2 and 4 D 1, 2, 3 and 4

TPJC 9648/01/JC2 Prelim/2013 [Turn Over


1624
19

40. A possible route of escape of an inserted gene from a genetically engineered crop of oil-
seed rape, Brassica napus, is into populations of the wild turnip, Brassica rapa.
Two populations of wild turnip growing next to large fields of a cultivar of oil-seed rape
were studied. Seeds were collected from these wild turnips plants and their DNA
analysed using restriction enzymes and electrophoresis.

Using the key provided below, determine


(i) whether plants S and/or T are interspecific hybrids and
(ii) the parental plants they are derived from

Interspecific hybrid Parent plants

A Both S and T S: P x R ii iii i

T: P x Q

B Both S and T S: P x Q ii iii i

T: P x R

C T only QxR iv v i

D S only PxR iii i ii

TPJC 9648/01/JC2 Prelim/2013 [Turn Over


1625

TAMPINES JUNIOR COLLEGE

H JC2 PRELIMINARY EXAMINATION


[mark scheme]

2
CANDIDATE
NAME

CIVICS GROUP 1 2
TUTOR
NAME

Biology 9648/ 01
Paper 1 Multiple Choice Tuesday, 24 September 2013
1 hour 15 minutes
Additional Materials: Optical Answer Sheet (OAS)

READ THESE INSTRUCTIONS FIRST

Write your name and Civics Group in the spaces at the top of this page and on any separate writing
paper used.

There are 40 questions in this paper.


Answer all questions. For each question there are four possible answers A, B, C and D.
Choose the one you consider correct and record your choice in soft pencil on the separate OAS.

Read the instructions on the OAS very carefully.

Each correct answer will score one mark. A mark will not be deducted for a wrong answer.
Any rough working should be done in this booklet.

This document consists of 17 printed pages and 1 blank page


1626
2
MULTIPLE CHOICE QUESTIONS (40 MARKS)
1. An electron micrograph of a cell is shown below.

J E

Match the organelles E, F, G, H and J associated with the cellular processes listed.

E F G H J

A DNA replication Digestion of Organizes the Oxidative Packaging of


material spindle phosphorylation secretory products

B Oxidative Organizes the Digestion of DNA replication Packaging of


phosphorylation spindle material secretory products

C Organizes the Digestion of Oxidative Packaging of DNA replication


spindle material phosphorylation secretory products

D DNA replication Organizes the Packaging of Oxidative Digestion of


spindle secretory products phosphorylation material

TPJC 9648/01/JC2 Prelim/2013 [Turn Over


1627
3
2. Which row correctly links a feature of a phospholipid molecule with its function in membranes?
Both polar and
non-polar Phosphate group Unsaturated fatty Saturated fatty
(amphipatic) acids acids

A Allowing diffusion Aiding Decreasing Binding proteins


of small membrane fluidity
molecules stability

B Forming a bilayer Repelling Increasing fluidity Decreasing


charged fluidity
molecules
C Forming Binding proteins Increasing Increasing fluidity
glycolipids permeability
D Forming vesicles Forming Binding proteins Decreasing
glycolipids permeability

3. The diagram below shows the structure of a general transcription factor.

Which correctly describes the structure of the part enclosed in box Z?

Number of Amino Acids Present Types of bonds Present

A Cannot be determined Hydrogen Bonds only


B Cannot be determined Hydrogen Bonds and hydrophilic bonds
C Approximately 11 amino acids Hydrogen bonds only
D Approximately 11 amino acids Hydrogen bonds and hydrophobic bonds

TPJC 9648/01/JC2 Prelim/2013 [Turn Over


1628
4
4. Each polypeptide chain of haemoglobin contains a number of regions in the form of an
alpha helix.
Which feature of the haemoglobin molecule is responsible for this?
A bonding between the four polypeptide chains
B hydrophobic interactions at the centre of the molecule
C the amino acid sequence of the globin
D the presence of iron in the haem groups

5. Which of the following statement about enzymes is inaccurate?

A Enzyme function is not influenced by factors such as lead nitrate.

B Enzyme function is reduced if the three-dimensional structure or conformation of an


enzyme is altered.

C Enzymes increase the rate of chemical reaction by lowering activation energy


barriers.

D Enzymes may require a non-protein cofactor or ion for catalysis to take place.

6. Which of the following does not describe the active site of an enzyme?

A Its specificity is defined by the arrangement of the monomers.


B It is usually a crevice or cleft.
C It initially binds substrates by weak attractions.
D It is two-dimensional in structure.

TPJC 9648/01/JC2 Prelim/2013 [Turn Over


1629
5
7.

Which of the following statements is true?

A The bond in 1 is phosphodiester bond.


B The bond in 5 is peptide bond.
C 7 refers to the 5 end of the mRNA.
D 8 is showing unwinding of the DNA molecule.

TPJC 9648/01/JC2 Prelim/2013 [Turn Over


1630
6
8. The following coding sequence of DNA is taken randomly from a bacterial genome.

3 TTACGCTTCGAAATAGGAATATCATAGGCT 5

This sequence is cloned into a plasmid and transformed into a suitable host. What would
be the first four amino acids of a polypeptide generated from this sequence as expressed
by the host?

The mRNA codons for some amino acids are shown in the table below:

Arg CGA, CGG, AGA, AGG Leu CUU, CUC, CUA, CUG
Asp GAU, GAC Lys AAA, AAG
Ile AUU, AUC, AUA Phe UUU, UUC
Start AUG Ser UCA, UCG, AGU, AGC
Stop UAG, UGA, UAA Tyr UAU, UAC

A Met-Arg-Ser-Phe
B Met-Arg-Ser-Lys
C Met-Ile-Phe-Leu
D Met-Tyr-Lys-Asp.

9. Which of the following is unique to mitosis and not a part of meiosis?

A Homologous chromosomes pair forming bivalent.

B Homologous chromosomes cross over.

C Chromatids are separated during anaphase.

D Homologous chromosomes behave independently.

10. Cells treated with colchicine often mutate. They fail to produce spindles during cell
division. What would be the result of this?
A chromosome deletion
B chromosome translocation
C gene deletion
D polyploidy

TPJC 9648/01/JC2 Prelim/2013 [Turn Over


1631
7
11. Four bacteriophage genes were mapped by recombination. The recombination
frequencies between the four genes are shown in the table below.

L M N P
L -- 0.18 0.10 0.20
M -- 0.15 0.37
N -- 0.24
P --

What is the order of the genes along the chromosome?

A L, M, N, P
B N, M, P, L
C M, L, P, N
D P, L, N, M

12. During a viral infection, attachment is usually specific to a particular cell type because

A the virus is attracted to the appropriate host cells by proteins secreted into the
extracellular fluid.
B the virus recognises and binds to specific molecules in the cytoplasm of the host
cell.
C the virus recognises and binds to specific molecules on the surface of the host
cell.
D the host cell produces channel proteins that provide passageways for the viruses
to enter the cytoplasm.

13. When glucose is absent and lactose is present, lactose binds to the operon's repressor.
Which statement best describes what happens subsequently?

A Repressor binds to promoter, transcription is initiated.


B Repressor binds to operator, transcription is not initiated.
C Repressor fails to bind to structural genes, transcription is not initiated.
D Repressor fails to bind to the operator, transcription is initiated

TPJC 9648/01/JC2 Prelim/2013 [Turn Over


1632
8
14. The diagram below shows 2 species of bacteria. The bacteria that are shaded are
resistant to the antibiotic penicillin.

Species 1 Species 2

A D

B C E F

Which one of the following statements is likely to be true?

A Bacterium D is resistant to penicillin due the transfer of the complete F plasmid and
penicillin resistant gene through transduction involving Bacterium A.
B Bacterium D is resistant to penicillin due to horizontal gene transfer involving a
bacteriophage and Bacterium A.
C Bacterium E acquired resistance to penicillin as a result of random mutation and
genetic transformation.
D Bacteria E and F are resistant to penicillin as a result of conjugation from Bacterium
D.

15. The statements are descriptions of aspects of genetics.

1. The phenotype is affected by both alleles at the same locus of a heterozygous


individual.
2. The combined effects of alleles at two or more gene loci equal the sum of their
individual effects.
3. Many different alleles present in a gene pool can occupy the same gene locus.
4. Alleles of one gene mask the effects of the alleles of another gene at a
different locus.

Match the description to each of the following terms

codominant alleles epistatic alleles multiple alleles additive genes

A 1 4 2 3

B 1 4 3 2

C 2 1 4 3

D 3 1 4 2

TPJC 9648/01/JC2 Prelim/2013 [Turn Over


1633
9

16. Skin colour in a variety of pumpkins is controlled by two pairs of alleles, Pp and Rr, which
segregate independently.

The allele P is dominant and must be present for the development of pigmentation in the
skin. Pumpkins without pigment are white. Allele R is dominant and produces a red
pigment in the skin. The recessive allele r gives a yellow colour.

In a testcross with a parent of unknown genotype, a phenotypic ratio of 1 red : 1 white


was obtained.

What was the genotype of the unknown parent?

A PPRR

B PpRR

C PPRr

D Pprr

17. Studies on the giant axon of the squid showed that bacteria toxin BAXIN results in an
increase in negativity of the postsynaptic membrane. What is the possible mode of action
of BAXIN?

A The release of acetylcholine

B Analog of acetylcholine

C Release of potassium ions from postsynaptic membrane

D Mimicking of cholinesterase

18. The diagram shows the sequence of events occurring as an action potential arrives at the
synapse. The numbered arrows represent the movement of substances across the
membranes.

What are the substances moving across the membranes?

1 2 3 4 5
A K+ Na+ acetylcholine Ca+ K+
B K+ Na+ Ca+ acetylcholinesterase acetylcholine
C Na+ K+ Ca+ acetylcholine Na+
D Na+ K+ acetylcholinesterase acetylcholine Ca+

TPJC 9648/01/JC2 Prelim/2013 [Turn Over


1634
10

19. The diagram shows how a nerve cell and an endocrine cell interact with cell X and cell Y
respectively.

Which of the following is true of both interactions?

A It involves the movement of chemicals M and N to their respective target cells.


B Chemical M is secreted faster than chemical N.
C When both cells X and Y are stimulated, phosphorylation cascades must be
initiated.
D Both chemicals M and N must bind to cell-surface receptors on cell X and Y
respectively.

20. In the experiment shown in the diagram, the oxygen given off at X and Y will be labelled
with the isotopes _____ and _____ respectively.

X Y
16 16
A O O
16 18
B O O
18 16
C O O
18 18
D O O

TPJC 9648/01/JC2 Prelim/2013 [Turn Over


1635
11
21. The weed killer DCMU blocks the flow of electrons from the electron transport chains in
photophosphorylation.
Why does this kill the plant?
A Active transport of mineral ions is prevented.
B ATP and reduced NADP are not produced.
C Photoactivation of the chlorophyll cannot occur.
D Photolysis of water does not occur.

22. To what extent can glucose be oxidised by the soluble portion of cytoplasm of muscle
cells, from which all organelles have been removed?
A Acetyl-CoA but no further
B Carbon dioxide and water
C Lactate but no further
D Pyruvate but no further

23. In which of the following changes is most energy released during the metabolism of
carbohydrates?
A Maltose to glucose

B Glucose to phosphoglyceraldehyde

C Phosphoglyceraldehyde to pyruvic acid


D Pyruvic acid to carbon dioxide and water

24. Which of the following is true of gene regulation in both prokaryotes and eukaryotes?

A Gene regulation of eukaryotes involves bending of DNA while in prokaryotes,


chemical modifications to DNA is required.
B In eukaryotes, increase in efficiency of RNA polymerase requires specific
transcription factors to activate the initiation complex while in prokaryotes,
activators are required to bind directly to promoter site to increase efficiency of
RNA polymerase.
C In eukaryotes, RNA polymerase binds to the TATA box while in prokaryotes, RNA
polymerase binds to the regulatory gene.
D Repressors and activators regulate expression in eukaryotes while prokaryotes are
regulated by repressors only.

25. The following events occur in the extension of telomeres.

TPJC 9648/01/JC2 Prelim/2013 [Turn Over


1636
12
1 Further extension of 3 end of telomere.
2 Translocation of telomerase to end of telomere.
3 Reverse transcription to extend the 3 end of the telomere.
4 Complementary base-pairing of RNA template with single-stranded end of
telomere.
5 DNA polymerase catalyses formation of complementary strand to form double-
stranded DNA.
In which order do these events take place?

A 3 2 4 5 1
B 3 4 2 1 5
C 4 2 3 5 1
D 4 3 2 1 5

26. As shown in the diagram below, acetylation of histones promotes loose chromatin
structure. Recent evidence has shown that chemical modification of histones play a direct
role in regulation of gene expression. Suggest how this works.

A Helicase action is enhanced by acetylation.


B Acetylation of histones neutralizes their negative charges and encourages binding to
DNA polymerase.
C When nucleosomes are highly acetylated, chromatin becomes less compact and
DNA is more accessible for transcription.
D RNA polymerase works better by binding with acetyl groups.

27. The hormone insulin binds to the tyrosine kinase receptors and initiates various signal
transduction pathway to generate cellular responses. Which of the following shows the
TPJC 9648/01/JC2 Prelim/2013 [Turn Over
1637
13
correct sequence of events, following the binding of insulin to the receptor?

a. phosphorylation of tyrosine kinases


b. dimerization of tyrosine kinase receptor
c. activation of enzyme

A a,b,c
B c,b,a
C b,a,c
D c,a,b

28. Many signal transduction pathways use second messengers to

A amplify the message by directly phosphorylating proteins.


B induce a signal from the outside to the inside of the cell.
C relay the message from the inside of the membrane throughout the cytoplasm.
D transport a signal through the lipid bilayer portion of the plasma membrane.

29. Which of the following is a similarity between G-protein-linked receptors and tyrosine-
kinase receptors?

A activation that results from binding of GTP


B formation of a dimer following binding of a signal molecule
C regions of the receptor that span the plasma membrane
D signal-binding sites specific for steroid hormones

30. Which of the following increases the number of different alleles in a population?

A crossing over
B gene mutation
C random fusion of gametes
D random assortment of chromosomes in meiosis

31. The graph below shows data on a population of a species of moth which shows

TPJC 9648/01/JC2 Prelim/2013 [Turn Over


1638
14
considerable variation in colour intensity. Which conclusion can be made from this
graph?

A Colour variation is environmentally induced.


B Colour variation is genetically determined.
C Extreme forms are favoured by natural selection.
D The species shows discontinuous variation with respect to colour.

32. Approximately 1 in 20 Europeans are heterozygous for a recessive allele responsible for
the genetic condition, cystic fibrosis (CF). People who are homozygous for CF have a
reduced life expectancy. Heterozygotes are more resistant to some bacterial infections of
the gut, such as typhoid fever, than homozygotes for the normal, dominant allele.

What could explain the high incidence of the recessive CF allele in the European
population?

A natural selection favouring heterozygotes

B natural selection favouring homozygotes for the recessive CF allele

C lack of genetic drift in the European population

D Mutation

33. What is one similarity and one difference between classification and phylogeny?

TPJC 9648/01/JC2 Prelim/2013 [Turn Over


1639
15

A Both organise organisms into groups based on their characteristics but phylogeny
takes into consideration the evolutionary relationships between groups.

B Both put organisms together according to increasingly inclusive groups but


classification does not show a branching pattern of evolutionary relationships.

C Both rely on physical characteristics of organisms but phylogeny does not require
scientific names.

D Both take into account the common ancestors of organisms but classification is
hierachical in nature.

34. Which uses of the information from the human genome project are generally considered
to be unethical?

1. an insurance company only giving cheap rates to people with genetic


predispositions to fewer diseases
2. genetic archaeologists identifying the earliest forms of genes to show evolutionary
relationships
3. cytologists developing tests for only some defective genes
4. doctors only giving specific drugs to block the actions of faulty genes to carriers of
those genes
5. genetic counselors giving specific lifestyle information only to people genetically
predisposed to risks
6. parents choosing embryos for implantation only after ante-natal tests for
acceptable genes

A 1 and 3
B 1 and 6
C 2 and 5
D 3 and 4

35. The insulin gene and a plasmid with ampicilin and tetracycline resistant genes were cut

TPJC 9648/01/JC2 Prelim/2013 [Turn Over


1640
16
using the same restriction enzyme, EcoRI. The cut plasmid and insulin gene was mixed
together and DNA ligase was allowed to react. Subsequently the mixture was heat shocked
at 42 degrees with E. coli. The mixture was then plated on nutrient agar to obtain a master
plate with colonies shown below.

Master plate (nutrient agar)

Three replica plates containing different antibiotics were obtained from the master plate.
The diagram below shows the results of the replica plates.

Nutrient agar with Nutrient agar with Nutrient agar with


ampicillin tetracycline tetracycline and
ampicillin

Which of the following can be concluded from the results obtained?

A The ampicilin resistant gene is inactivated by insertion of the insulin gene


B The tetracycline resistant gene is inactivated by insertion of the insulin gene.
C The tetracycline resistant gene does not act as a selectable marker
D The ampicillin resistant gene does not act as a selectable marker.

36. A student performed an experiment to determine if a particular gene has been inserted in

TPJC 9648/01/JC2 Prelim/2013 [Turn Over


1641
17
a genetically modified organism.
i. Transfer of DNA to nitrocellulose membrane
ii. Restriction digestion of genomic DNA
iii. Cleaved DNA separated using gel electrophoresis
iv. Create radioactive probe
v. Incubate probe and membrane

Which is the correct sequence to the above steps?

A ii iii i v iv.
B ii iii i iv v.
C iv v i ii iii.
D iii i ii iv v.

37. The genetically engineered super salmon was created from Atlantic salmon stocks and
are capable of growing to a large size in 14 months. Which of the following is not a
benefit intended from the crop?

A Higher yield for farmers

B Minimising pollution

C Decreasing the food consumption of the crops in their lifetime

D Increase in supply to meet worlds demand

TPJC 9648/01/JC2 Prelim/2013 [Turn Over


1642
18
38.

Which of the following options is not a characteristic of the form of treatment shown
above?

1 Easy to extract in laboratory


2 Readily available
3 Difficult to control
4 Limited longevity

A 1 only B 1 and 2 C 1 and 3 D 1, 2, 3 and 4

39. Germ-line gene therapy has not been prevalent in medical treatment because of many
reasons. Which of the following is/are implications of germ-line therapy?

1 unforeseeable clinical risks


2 opens problems for social discrimination
3 cost
4 issue of consent

A 1 only B 2, 3 and 4 C 1, 2 and 4 D 1, 2, 3 and 4

TPJC 9648/01/JC2 Prelim/2013 [Turn Over


1643
19

40. A possible route of escape of an inserted gene from a genetically engineered crop of oil-
seed rape, Brassica napus, is into populations of the wild turnip, Brassica rapa.
Two populations of wild turnip growing next to large fields of a cultivar of oil-seed rape
were studied. Seeds were collected from these wild turnips plants and their DNA
analysed using restriction enzymes and electrophoresis.

Using the key provided below, determine


(i) whether plants S and/or T are interspecific hybrids and
(ii) the parental plants they are derived from

Interspecific hybrid Parent plants

A Both S and T S: P x R ii iii i

T: P x Q

B Both S and T S: P x Q ii iii i

T: P x R

C T only QxR iv v i

D S only PxR iii i ii

TPJC 9648/01/JC2 Prelim/2013 [Turn Over


1644

TAMPINES JUNIOR COLLEGE

H JC2 PRELIMINARY EXAMINATION

2
CANDIDATE
NAME

CIVICS
GROUP
1 2
TUTOR
NAME

Biology 9648/ 02
Paper 2 Monday, 2 September 2013
2 hours
Candidates answer on the Question Paper
Additional Materials: Writing Paper

READ THESE INSTRUCTIONS FIRST


For Examiners Use
Write your name and class on all the work you hand in.
Section A
Write in dark blue or black pen.
You may use a soft pencil for any diagrams, graphs or rough 1 10
working.
Do not use paper clips, highlighters, glue or correction fluid. 2 10

3 17
Section A
4 11
Answer all questions.
Section B 5 10

Answer one question. 6 9

7 13

Section B
The number of marks is given in brackets [ ] at the end of each 20
8 OR
question or part question.
9 20

Total 100

This document consists of 23 printed pages and 1 blank page


1645
2
SECTION A - STRUCTURED QUESTIONS (80 MARKS)
Answer all the questions in this section in the space provided.
QUESTION 1

Fig. 1.1 shows a simplistic outline of the processes of mitosis and meiosis.

(a) State four ways in which the behavior of chromosomes in meiosis differs from their behavior in mitosis.
[4]

TPJC 9648/02/JC2 PRELIM/2013 [Turn Over]


1646
3
Fig. 1.2 shows the karyotype of a person who suffers from a double genetic disorder due to the inheritance
of extra copies of chromosomes.

Fig. 1.2

(a) With reference to Fig. 1.2, suggest the genetic disorder(s) the person whose karyotype is shown is
suffering from. [1]

(b) Describe the aberration that has occurred during cell and nuclear division that has resulted in the
inheritance of the extra chromosomes. [3]

TPJC 9648/02/JC2 PRELIM/2013 [Turn Over]


1647
4
(c) Explain the effects of having extra copies of chromosomes on the person. [2]

[Total: 10]

TPJC 9648/02/JC2 PRELIM/2013 [Turn Over]


1648
5
QUESTION 2

Fig. 2.1 is a diagram showing part of a bacterial chromosome with a single origin of replication and two
replication forks which proceed in opposite directions until replication of the entire bacterial chromosome is
complete.

Fig. 2.1

(a) With reference to Fig. 2.1, explain why this replication is said to be semi-conservative. [2]

(b) With reference to Fig. 2.1, replication only occurs in a 5 to 3 direction, explain why this is so. [2]

TPJC 9648/02/JC2 PRELIM/2013 [Turn Over]


1649
6
The Hayflick limit is the number of times a normal human cell population will divide until cell division stops.
Empirical evidence shows that the telomeres associated with each cell's DNA will get slightly shorter with
each new cell division until they shorten to a critical length. The Hayflick limit has been found to correlate
with the length of the telomere region at the end of a strand of DNA. Fig. 2.2 shows the position of telomeres
in a DNA double helix.

Fig. 2.2

(c) Outline the role of telomeres. [3]

(d) Explain how telomeres shorten with each cell division. [3]

[Total: 10]

TPJC 9648/02/JC2 PRELIM/2013 [Turn Over]


1650
7
QUESTION 3

Fig. 3.1 shows a signaling pathway involving Ras protein, a proto-oncogene. Ras protein is involved in the
activation of PI3K a kinase which is a relay molecule for the downstream activation of subsequent molecules
within the phosphorylation cascade. The effect of this signaling pathway is the triggering of cell growth,
proliferation and survival.

Fig. 3.1

(a) With reference to Fig 3.1, describe how binding of Growth Factors could result in the activation of Ras
protein. [2]

TPJC 9648/02/JC2 PRELIM/2013 [Turn Over]


1651
8
Ras proto-oncogene could be mutated such that it becomes oncogenic. This mutation of Ras proto-
oncogene is frequently seen in many cases of cancer. However, in order for oncogenic Ras protein to induce
cancer formation, another protein, PTEN, must be mutated. In the signaling pathway shown in Fig. 3.1,
PTEN is involved in the inhibition of PI3K and thereby signals cells to stop dividing and can cause cells to
undergo programmed cell death (apoptosis) when necessary.

(b) State what you understand by the terms,

(i) Proto-oncogene [1]

(ii) Onco-gene [1]

(c) Explain how mutation could make Ras oncogenic. [4]

TPJC 9648/02/JC2 PRELIM/2013 [Turn Over]


1652
9
PTEN is an enzyme that acts as a tumour suppressor by regulating cell cycle and induces apoptosis. PTEN
inhibits PI3K, a kinase, by dephosphorylating PI3K (refer to Fig. 3.1). Dephosphorylated PI3K is inactive and
will not activate downstream relay molecules of the phosphorylation cascade to induce cell division. Fig. 3.2
shows the structure of PTEN.

Fig. 3.2

(d) Explain how a mutation in the PTEN gene could affect the structure of its product. [3]

(e) Suggest why some mutations of the PTEN gene have no effect on the activity of the PTEN protein. [3]

TPJC 9648/02/JC2 PRELIM/2013 [Turn Over]


1653
10
Fig. 3.3 shows another protein, P53, which has tumour suppressor activities, binding to DNA. p53 works as
a transcription factor by binding to DNA to trigger the transcription of a series of genes which work in
suppressing tumour formation.

Fig. 3.3

(f) Suggest why the p53 protein only binds to certain parts of the DNA molecule. [3]

[Total: 17]

TPJC 9648/02/JC2 PRELIM/2013 [Turn Over]


1654
11

QUESTION 4

Fig. 4.1 shows some of the main structural features of the H7N9 influenza virus.

Fig. 4.1

The viral RNA genome contains genes (HA and NA) that codes for Haemagglutinin and Neuraminidase,
which are widely researched for the development of antiviral drugs.

In H7N9, the H refers to the type of haemagglutinin and the N refers to the type of neuraminidase present
as part of the viral envelope. The avian influenza A (H7N9) virus designation of H7N9 identifies it as having
HA of the H7 subtype and NA of the N9 subtype

After the influenza virus enters the host cells, the genes in the RNA segments are used as templates for
mRNA production

(a) Suggest the functions of haemagglutinin and neuraminidase in Influenza virus. [2]

TPJC 9648/02/JC2 PRELIM/2013 [Turn Over]


1655
12
(b) Describe how an enveloped virus such as H7N9 virus enters a host cell. [4]

(c) The viral RNA also contains PB2, PB1 and PA which code for the viral RNA polymerases. Explain why
such RNA segments are needed by the virus. [2]

(d) The influenza virus does not contain ribosomes. Despite this, the influenza virus is able to trigger
synthesis of viral proteins. Explain how does the influenza virus do so? [1]

TPJC 9648/02/JC2 PRELIM/2013 [Turn Over]


1656
13
H7N9 is an Avian influenza virus which normally circulates amongst avian populations with some variants
known to occasionally infect humans. An H7N9 virus was first reported to have infected humans in 2013 in
China and now this virus has spread among humans. Fig. 4.2 show the evolution of H7N9 virus.

Fig. 4.2

TPJC 9648/02/JC2 PRELIM/2013 [Turn Over]


1657
14
(e) Within reference to Fig. 4.2, explain how the new combination of RNA segments in H7N9 could have
arisen. [2]

[Total: 11]

TPJC 9648/02/JC2 PRELIM/2013 [Turn Over]


1658
15

QUESTION 5

Carbon is essential to life. All of our molecular machines are built around a central scaffolding of organic
carbon. Unfortunately, carbon in the earth and atmosphere is locked in highly oxidized forms, such as
carbonate minerals and carbon dioxide gas. In order to be useful, this oxidized carbon must be "fixed" into
more organic forms, rich in carbon-carbon bonds and decorated with hydrogen atoms. Powered by the
energy of sunlight, plants perform this central task of carbon fixation.

Inside plant cells, the enzyme ribulose bisphosphate carboxylase (rubisco) forms the bridge between life and
the lifeless, creating organic carbon from the inorganic carbon dioxide in the air. Fig. 5.1 shows the structure
of ribulose bisphosphate carboxylase (rubisco).

Fig. 5.1

(a) Rubisco is an enzyme made of sixteen polypeptide chains.

Describe how the structure of rubisco is maintained. [2]

TPJC 9648/02/JC2 PRELIM/2013 [Turn Over]


1659
16
In spite of its central role, rubisco is remarkably inefficient. Typical enzymes can process a thousand
molecules per second, but rubisco fixes only about three carbon dioxide molecules per second. Fig. 5.2
shows the relationship between CO2 assimilation rate and increasing light intensity in a plant, when carbon
dioxide is present in excess.

Fig. 5.2

(b) Explain how light energy assists in the process of carbon fixation in plants. [3]

(c) Explain why the CO2 assimilation rate levels off at higher light intensity. [3]

TPJC 9648/02/JC2 PRELIM/2013 [Turn Over]


1660
17
Rubisco also shows an embarrassing lack of specificity. Unfortunately, oxygen molecules and carbon
dioxide molecules are similar in shape and chemical properties. In rubisco, an oxygen molecule can bind
comfortably in the active site designed to bind carbon dioxide. Rubisco then attaches the oxygen to the
sugar chain, forming a faulty oxygenated product.

(c) With reference to Fig 5.2, suggest why the CO2 assimilation rate is declining as the light intensity
increases higher. [2]

[Total: 10]

TPJC 9648/02/JC2 PRELIM/2013 [Turn Over]


1661
18
QUESTION 6

Hair color is the pigmentation of hair follicles due to two types of melanin: eumelanin and pheomelanin.
Pheomelanin colors hair reddish-orange. Eumelanin, which has two subtypes of black or brown,
determines the darkness of the hair color. Generally, if more eumelanin is present, the color of the hair is
darker. If less eumelanin is present, the hair is lighter. Particular hair colors are associated with ethnic
groups.

The genetics of hair colors are not yet firmly established. According to one theory, at least two unlinked
gene pairs that control human hair color.

In Americans, gene B and R are involved in controlling the shades of hair colour. Gene B controls the
production of brown eumelanin. This gene has a dominant brown allele and a recessive blond allele.
Gene R controls the production of pheomelanin. This gene has a non-red allele (which suppresses
production of pheomelanin) which is dominant and the allele for red hair is recessive. Gene B is epistatic
over Gene R.

(a) State and explain the genotypes for a blonde hair phenotype. [3]

A couple, both with blonde hair, was filing for a divorce as the husband felt that her wife has cheated on
him. His rationale of saying so was based on the recent birth of their boy who has red-coloured hair.

(b) With the use of a genetic diagram, explain whether or not it is possible for two parents with blonde
hair to have an offspring with red-coloured hair. [4]

TPJC 9648/02/JC2 PRELIM/2013 [Turn Over]


1662
19

(c) The husband felt bad for doubting his wife. He wants to find our whether there is a significant
difference between what they observed in their baby boy and what is expected.

Explain whether it is possible to carry out a chi-square test in this case. [2]

[Total: 9]

TPJC 9648/02/JC2 PRELIM/2013 [Turn Over]


1663
20
QUESTION 7

Fig. 7.1 shows Honeycreepers of the Family Drepanididae from the Hawaiian Islands. These
Honeycreepers reside on the various Hawaiian Islands. Through a series of studies, these
Honeycreepers are found to be evolved from an ancestral Honeycreeper which resided originally on,
Kauai, one of the Hawaiian Islands.

Fig. 7.1

(a) Explain how these various species of Honeycreepers could have arisen. [4]

TPJC 9648/02/JC2 PRELIM/2013 [Turn Over]


1664
21
Our understanding of the phylogeny of species has been greatly improved through the use of molecular
techniques.

Many phylogenetic studies use mitochondrial DNA rather than nuclear DNA.

(b) Explain what is meant by the term phylogeny. [2]

A new Honeycreeper (Fig. 7.2) has been found on one of the Hawaiian Islands. Based on its structural
features, Scientists are speculating that this new found Honeycreeper is also a descendant of the
ancestral Honeycreeper found on Kauai.

Fig. 7.2

(d) Suggest why it is not advisable to determine the phylogenetic relationship of this new found
Honeycreeper to the rest of them from the morphological features. [2]

TPJC 9648/02/JC2 PRELIM/2013 [Turn Over]


1665
22
(e) Outline how DNA can be studied to determine phylogeny of organisms. [5]

[Total: 13]

TPJC 9648/02/JC2 PRELIM/2013 [Turn Over]


1666
23
SECTION B (20 MARKS)

Answer one question.


Write your answers on the separate writing paper provided.
Your answers should be illustrated by large, clearly labeled diagrams, where appropriate.
Your answers must be in continuous prose.
Your answers must be set out in sections (a), (b) etc., as indicated in the question.

QUESTION 8

(a) Compare anaerobic respiration and aerobic respiration. [6]

(b) Describe the main features of oxidative phosphorylation. [8]

(c) Discuss how the mitochondrion is adapted for carrying out respiration. [6]

QUESTION 9

(a) Compare the mode of infection of Influenza Virus and Human Immunodeficiency Virus. [6]

(b) Describe the mechanisms of gene transfer in prokaryotes. [8]

(c) Discuss how the prokaryote, E. coli, is able to respond to varying concentrations of lactose. [6]

End of Paper

TPJC 9648/02/JC2 PRELIM/2013 [Turn Over]


1667
24
BLANK PAGE

TPJC 9648/02/JC2 PRELIM/2013 [Turn Over]


1668

TAMPINES JUNIOR COLLEGE

H JC2 PRELIMINARY EXAMINATION


MARK SCHEME

2
CANDIDATE
NAME

CIVICS
GROUP
1 2
TUTOR
NAME

Biology 9648/ 02
Paper 2 Monday, 2 September 2013
2 hours
Candidates answer on the Question Paper
Additional Materials: Writing Paper

READ THESE INSTRUCTIONS FIRST


For Examiners Use
Write your name and class on all the work you hand in.
Section A
Write in dark blue or black pen.
You may use a soft pencil for any diagrams, graphs or rough 1 10
working.
Do not use paper clips, highlighters, glue or correction fluid. 2 10

3 17
Section A
4 11
Answer all questions.
Section B 5 10

Answer one question. 6 9

7 13

Section B
The number of marks is given in brackets [ ] at the end of each 20
8 OR
question or part question.
9 20

Total 100

This document consists of 26 printed pages and 0 blank pages


1669
2
SECTION A - STRUCTURED QUESTIONS (80 MARKS)
Answer all the questions in this section in the space provided.
QUESTION 1

Fig. 1.1 shows a simplistic outline of the processes of mitosis and meiosis.

(a) State four ways in which the behavior of chromosomes in meiosis differs from their behavior in mitosis.
[4]

Synapsis of homologous chromosomes forming a bivalent during prophase of meiosis I and not
during prophase in mitosis.

Chiasmata/ crossing over between non-sister chromatids during prophase of meiosis I and not
during prophase in mitosis.

Homologous pairs line up at the equator in metaphase I but in metaphase of mitosis,


chromosomes line up singly;

Separation of homologous chromosomes at anaphase I in meiosis whereas there is the Separation


of sister chromatids at anaphase II of meiosis and anaphase of mitosis.

[1 mark each]

TPJC 9648/02/JC2 PRELIM/2013 [Turn Over]


1670
3
Fig. 1.2 shows the karyotype of a person who suffers from a double genetic disorder due to the inheritance
of extra copies of chromosomes.

Fig. 1.2

(a) With reference to Fig. 1.2, suggest the genetic disorder(s) the person whose karyotype is shown is
suffering from. [1]

Down syndrome and Klinefelter. [1]

(b) Describe the aberration that has occurred during cell and nuclear division that has resulted in the
inheritance of the extra chromosomes. [3]

Non-disjunction has occurred during meiosis


During meiosis I (Metaphase I), there could be a failure of attachment of spindle fibre to
kinetochore of the chromosome
And failure of the M checkpoint to detect the aberration
Homologous chromosomes are not separated during Anaphase I; hence, inherited.
Sister chromatids could fail to separate during meiosis II (Anaphase II)
Due to failure of attachment of spindle fibre to kinetochore
Or due to failure of separation of the centromere.

[any 6]

TPJC 9648/02/JC2 PRELIM/2013 [Turn Over]


1671
4
(c) Explain the effects of having extra copies of chromosomes on the person. [2]

Having extra copies of chromosomes would mean that there are extra set of genes within the cell
These extra genes could also be expressed through transcription and translation
Protein (gene product) formed would now be more than the normal level
Exert extra physiological effects on cell -> phenotypes that are aberrant or more pronounced.

[1/2 mark each]

[Total: 10]

TPJC 9648/02/JC2 PRELIM/2013 [Turn Over]


1672
5
QUESTION 2

Fig. 2.1 is a diagram showing part of a bacterial chromosome with a single origin of replication and two
replication forks which proceed in opposite directions until replication of the entire bacterial chromosome is
complete.

Fig. 2.1

(a) With reference to Fig. 2.1, explain why this replication is said to be semi-conservative. [2]
DNA unzips / DNA strands separate / ref. to H bonds break ;
Both strands / each strand acts as a template ;Each new molecule of DNA contains one new and
one old / original strand of DNA
[1 mark each]

(b) With reference to Fig. 2.1, replication only occurs in a 5 to 3 direction, explain why this is so. [2]
structure of active site of DNA polymerase is specific [1]
allows recognition of a 3 OH from a nucleotide to initiate the replication process ; [1/2]
active site fits nucleotide in a particular orientation (5 forms phosphodiester bond with 3) [1/2]

TPJC 9648/02/JC2 PRELIM/2013 [Turn Over]


1673
6
The Hayflick limit is the number of times a normal human cell population will divide until cell division stops.
Empirical evidence shows that the telomeres associated with each cell's DNA will get slightly shorter with
each new cell division until they shorten to a critical length. The Hayflick limit has been found to correlate
with the length of the telomere region at the end of a strand of DNA. Fig. 2.2 shows the position of telomeres
in a DNA double helix.

Fig. 2.2

(c) Outline the role of telomeres. [3]


To provide stability to the chromosome by rendering chromosome ends generally inert in
interactions with other chromosome ends. (i.e. they keep the ends of the various chromosomes in
the cell from accidentally becoming attached to each other.)
The telomere regions acts as a buffer of genes near the ends of chromosomes by allowing for the
shortening of chromosome ends, which necessarily occurs during replication.
Telomere shortening in humans can induce senescence, which blocks cell division. This
mechanism appears to prevent genomic instability and development of cancer in human aged cells
by limiting the number of cell divisions.

[1 mark each]

(d) Explain how telomeres shorten with each cell division. [3]

Telomeres shorten in part because of the end replication problem that is exhibited during DNA
replication
Because DNA replication does not begin at either end of the DNA strand, but starts in the center
considering that all known DNA polymerases move in the 5' to 3' direction, one finds a leading and
a lagging strand on the DNA molecule being replicated.
Eventually when the last primer in the lagging strand is removed by DNA polymerase I and
replacing it with the correct DNA nucleotides
In the case at the position of the last primer, the 3OH group is absent.
RNA primer is removed but the gap is not replaced by DNA nucleotides.

[1/2 mark each]

[Total: 10]

TPJC 9648/02/JC2 PRELIM/2013 [Turn Over]


1674
7
QUESTION 3

Fig. 3.1 shows a signaling pathway involving Ras protein, a proto-oncogene. Ras protein is involved in the
activation of PI3K a kinase which is a relay molecule for the downstream activation of subsequent molecules
within the phosphorylation cascade. The effect of this signaling pathway is the triggering of cell growth,
proliferation and survival.

Fig. 3.1

(a) With reference to Fig 3.1, describe how binding of Growth Factors could result in the activation of Ras
protein. [2]

Binding of growth factors on the ligand binding site of the RTK causes the RTK to dimerise and
undergo cross phosphorylation -> RTK activated
Activated RTK will activate Ras protein by causing a displacement of the GDP with a GTP.

[1 mark each]

TPJC 9648/02/JC2 PRELIM/2013 [Turn Over]


1675
8
Ras proto-oncogene could be mutated such that it becomes oncogenic. This mutation of Ras proto-
oncogene is frequently seen in many cases of cancer. However, in order for oncogenic Ras protein to induce
cancer formation, another protein, PTEN, must be mutated. In the signaling pathway shown in Fig. 3.1,
PTEN is involved in the inhibition of PI3K and thereby signals cells to stop dividing and can cause cells to
undergo programmed cell death (apoptosis) when necessary.

(b) State what you understand by the terms,

(i) Proto-oncogene [1]

Proto-oncogenes code for proteins that stimulate normal cell growth and division.

(ii) Onco-gene [1]

An oncogene arises from a genetic change/mutated proto-oncogene that leads to an increase either
(1) in the amount of the proto-oncogenes protein product, or (2) in the activity of each protein
molecule.

(c) Explain how mutation could make Ras oncogenic. [4]


Ras gene mutation to produce a hyperactive ras protein
Point mutation within a control element of Ras gene protein produced in excess
Gene duplication - > gene amplification protein produced in excess
Translocation of Ras gene to a new locus with a hyperactive promoter protein produced in excess
Translocation of hyperactive promoter to the locus next to Ras gene protein produced in excess
Translocation of Ras gene into a coding sequence production of fusion protein with hyperactive
Ras function.
[ Any 4]

TPJC 9648/02/JC2 PRELIM/2013 [Turn Over]


1676
9

PTEN is an enzyme that acts as a tumour suppressor by regulating cell cycle and induces apoptosis. PTEN
inhibits PI3K, a kinase, by dephosphorylating PI3K (refer to Fig. 3.1). Dephosphorylated PI3K is inactive and
will not activate downstream relay molecules of the phosphorylation cascade to induce cell division. Fig. 3.2
shows the structure of PTEN.

Fig. 3.2
(d) Explain how a mutation in the PTEN gene could affect the structure of its product. [3]
A point mutation which involves either an addition or deletion of a nucleotide leads to a
frameshift ;
This could lead to an early stop codon being transcribed leading to the formation of a truncated
protein.
Or the resulting protein coded could have a series of very different A.As and thereby folded
differently to result in a non-functional protein.
OR
If a substitution mutation occurs such that a nucleotide is changed this could result in a
missense mutation or nonsense mutation which would affect the structure of its product.
In a missense mutation the change in nucleotide codes for a different a.a which could have a
different R group nature and thereby interacts with the other a.as differently -> different folding
A nonsense mutation, the change in nucleotide codes for an early stop codon -> truncated
protein
In both cases, the product is non-functional due to the change in the 3D structure of PTEN.

(e) Suggest why some mutations of the PTEN gene have no effect on the activity of the PTEN protein. [3]
Change in a.a in region away from binding site ;
May not affect shape / binding properties ;
Ref. idea some a.a coded for by more than 1 codon / triplet of bases / ref. genetic code / some
codons degenerate ;
So (change in base / codon) may not change a.a / neutral ;

TPJC 9648/02/JC2 PRELIM/2013 [Turn Over]


1677
10
Fig. 3.3 shows another protein, p53, which has tumour suppressor activities, binding to DNA. p53 works as a
transcription factor by binding to DNA to trigger the transcription of a series of genes which work in
suppressing tumour formation.

Fig. 3.3

(f) Suggest why the p53 protein only binds to certain parts of the DNA molecule. [3]
Specific shape of DNA binding domain of p53 protein ;
Corresponds to / complementary to shape of DNA molecule ;
Ref. to idea different sequences of bases (have different shapes) ;
Ref. to major and minor grooves of DNA ;

[Total: 17]

TPJC 9648/02/JC2 PRELIM/2013 [Turn Over]


1678
11
QUESTION 4

Fig. 4.1 shows some of the main structural features of the H7N9 influenza virus.

Fig. 4.1

The viral RNA genome contains genes (HA and NA) that codes for Haemagglutinin and Neuraminidase,
which are widely researched for the development of antiviral drugs.

In H7N9, the H refers to the type of haemagglutinin and the N refers to the type of neuraminidase present
as part of the viral envelope. The avian influenza A (H7N9) virus designation of H7N9 identifies it as having
HA of the H7 subtype and NA of the N9 subtype

After the influenza virus enters the host cells, the genes in the RNA segments are used as templates for
mRNA production

(a) Suggest the functions of haemagglutinin and neuraminidase in Influenza virus. [2]
recognises and bind to neuraminic acid residues on host cell membrane;
enable specificity in the type of host cells ( enable influenza to only attach to specific host cells
like respiratory cells);
Promote the release of progeny viruses from the host cell surface to infect other host cells
By cleaving the attachment/bond between the HA on influenza and neuraminic acid residues on
the host cell membrane;

TPJC 9648/02/JC2 PRELIM/2013 [Turn Over]


1679
12
(b) Describe how an enveloped virus such as H7N9 virus enters a host cell. [4]

Via receptor-mediated endocytosis


Haemagglutinin (HA) protein (on viral envelope) binds to neuraminic acid residues (on
cellular glycoproteins on the host cell surface membranes)
Host cell surface membrane invaginates and pinches off, placing the virus in an endocytic
vesicle known as an endosome.
Lowered pH within the endosome triggers the fusion of endosome membrane with the viral
envelope membrane thus releasing the nucleocapsid to the cytoplasm.

(c) The viral RNA also contains PB2, PB1 and PA which code for the viral RNA polymerases. Explain why
such RNA segments are needed by the virus. [2]
Viral RNA polymerases required by the virus would not be present in the host cell;
Viral RNA polymerases able to recognise viral negative sense RNA
Synthesis of (+) sense RNA to serve as mRNA to make viral proteins;
RNA polymerases are required to replicate (-) Sense viral RNA;

(d) The influenza virus does not contain ribosomes. Despite this, the influenza virus is able to trigger
synthesis of viral proteins. Explain how does the influenza virus do so? [1]

Use of the host cells ribosomes;


Translation of (+) mRNA made from the action of RNA polymerase on (-) sense RNA.

TPJC 9648/02/JC2 PRELIM/2013 [Turn Over]


1680
13
H7N9 is an Avian influenza virus which normally circulates amongst avian populations with some variants
known to occasionally infect humans. An H7N9 virus was first reported to have infected humans in 2013 in
China and now this virus has spread among humans. Fig. 4. 2 show the evolution of H7N9 virus.

Fig. 4.2

TPJC 9648/02/JC2 PRELIM/2013 [Turn Over]


1681
14
(e) Within reference to Fig. 4.2, explain how the new combination of RNA segments in H7N9 could have
arisen. [2]
Antigenic shift;
Where two influenza strains (one carrying the H7 and one carrying the N9) infect the same
intermediate host cell at the same time;
The cell is replicating the viral genomes of the two strains at the same time;
Random assembly of RNA segments from the three different influenza viruses - Ressortment
of genes occur resulting in the new combination of RNA segments in influenza virus H7N9
when it exits by budding;

[Total: 11]

TPJC 9648/02/JC2 PRELIM/2013 [Turn Over]


1682
15

QUESTION 5

Carbon is essential to life. All of our molecular machines are built around a central scaffolding of organic
carbon. Unfortunately, carbon in the earth and atmosphere is locked in highly oxidized forms, such as
carbonate minerals and carbon dioxide gas. In order to be useful, this oxidized carbon must be "fixed" into
more organic forms, rich in carbon-carbon bonds and decorated with hydrogen atoms. Powered by the
energy of sunlight, plants perform this central task of carbon fixation.

Inside plant cells, the enzyme ribulose bisphosphate carboxylase (rubisco) forms the bridge between life and
the lifeless, creating organic carbon from the inorganic carbon dioxide in the air. Fig. 5.1 shows the structure
of ribulose bisphosphate carboxylase (rubisco).

Fig. 5.1

(a) Rubisco is an enzyme made of sixteen polypeptide chains.

Describe how the structure of rubisco is maintained. [2]

The primary sequence of the 16 polypeptide chains formed by having peptides held between the
amino acids
The folding of the individual polypeptides to form secondary structures (Alpha helices and Beta-
pleated sheets) and maintained by H bonds between the imino and carbonyl group.
These secondary structures will form tertiary structures with further compacting and folding and
these are held by hydrogen bonds, hydrophobic interaction, ionic bonds and disulphide bonds.
These tertiary structures will associate with each other to form rubisco with each tertiary structure
held by the same interactions as well.

TPJC 9648/02/JC2 PRELIM/2013 [Turn Over]


1683
16
In spite of its central role, rubisco is remarkably inefficient. Typical enzymes can process a thousand
molecules per second, but rubisco fixes only about three carbon dioxide molecules per second. Fig. 5.2
shows the relationship between CO2 assimilation rate and increasing light intensity in a plant, when carbon
dioxide is present in excess.

Fig. 5.2

(b) Explain how light energy assists in the process of carbon fixation in plants. [3]

Light energy is harvested by the antenna complex and causes excitation of electrons in the
photosystem I and II
Put through the process of photophosphorylation
The excitation of electron of the PS I and II is captured by the primary electron acceptors of
each PS
This electron from PSII is passed down an electron transport chain to generate ATP through
chemiosmosis
The ATP formed will be used during the Calvin cycle during regeneration of RuBP
In a second ETC, NADP is reduced by NADP reductase to form reduced NAD -> needed for
Calvin cycle as a reducing power.

(c) Explain why the CO2 assimilation rate levels off at higher light intensity. [3]

CO2 assimilation rate levels off at high light intensity maximum rate as all photosystems are
saturated with light energy (limited amount of chlorophyll in the photosystems)

Maximum rate of photosynthesis occurs at high LI hence amount of CO2 assimilation now remained
constant due to the constant (maximum) amount of NADPH produced;

Light is no longer limiting factor, but other factor e.g. rate of carbon fixation is.

TPJC 9648/02/JC2 PRELIM/2013 [Turn Over]


1684
17
Rubisco also shows an embarrassing lack of specificity. Unfortunately, oxygen molecules and carbon
dioxide molecules are similar in shape and chemical properties. In rubisco, an oxygen molecule can bind
comfortably in the active site designed to bind carbon dioxide. Rubisco then attaches the oxygen to the
sugar chain, forming a faulty oxygenated product.

(c) With reference to Fig 5.2, suggest why the CO2 assimilation rate is declining as the light intensity
increases higher. [2]

As light intensity gets higher, the rate of production of oxygen from photolysis of water to
replace the lost of electrons in PS II may increase.
Oxygen produced within the plant cell will be greater than amount of carbon dioxide present and
Oxygen will bind rubisco preferentially and thereby leading to a decline in CO2 assimilation

[Total: 10]

TPJC 9648/02/JC2 PRELIM/2013 [Turn Over]


1685
18
QUESTION 6

Hair color is the pigmentation of hair follicles due to two types of melanin: eumelanin and pheomelanin.
Pheomelanin colors hair reddish-orange. Eumelanin, which has two subtypes of black or brown,
determines the darkness of the hair color. Generally, if more eumelanin is present, the color of the hair is
darker. If less eumelanin is present, the hair is lighter. Particular hair colors are associated with ethnic
groups.

The genetics of hair colors are not yet firmly established. According to one theory, at least two unlinked
gene pairs that control human hair color.

In Americans, gene B and R are involved in controlling the shades of hair colour. Gene B controls the
production of brown eumelanin. This gene has a dominant brown allele and a recessive blond allele.
Gene R controls the production of pheomelanin. This gene has a non-red allele (which suppresses
production of pheomelanin) which is dominant and the allele for red hair is recessive. Gene B is epistatic
over Gene R.

(a) State and explain the genotypes for a blonde hair phenotype. [3]

bbRr and bbRR [1]


to have blonde hair the person must be homozygous recessive for gene B (bb) such that the
will be no production of brown eumelanin.
At the loci for Gene R should contain at least a copy of the dominant allele R to suppress
production of pheomelanin to tint the hair red.

A couple, both with blonde hair, was filing for a divorce as the husband felt that her wife has cheated on
him. His rationale of saying so was based on the recent birth of their boy who has red-coloured hair.

(b) With the use of a genetic diagram, explain whether or not it is possible for two parents with blonde
hair to have an offspring with red-coloured hair. [4]

Woman Man
Parental phenotype Blonde Hair x Blonde Hair
1m
Parental genotype bbRr bbRr

Gametes
bR br bR br 1m

Offspring phenotype
(fertilization lines or bR br
punnett square needs
to be shown) bbRr
[1] for correct bbRR
bR Blonde Hair 1m
genotype and Blonde hair
phenotype
bbRr bbrr
br Blonde hair Red hair

Offspring genotypic bbRR : 2 bbRr : bbrr :


ratio 1m
Offspring phenotypic : : :
Blonde hair Blonde hair Red hair
ratio

TPJC 9648/02/JC2 PRELIM/2013 [Turn Over]


1686
19

Possible as the both parents might be heterozygous for the loci of Gene R.

(c) The husband felt bad for doubting his wife. He wants to find our whether there is a significant
difference between what they observed in their baby boy and what is expected.

Explain whether it is possible to carry out a chi-square test in this case. [2]

Not possible [1]


No large sample size of offsprings to conduct a chi-squared test.

[Total: 9]

TPJC 9648/02/JC2 PRELIM/2013 [Turn Over]


1687
20

QUESTION 7

Fig. 7.1 shows Honeycreepers of the Family Drepanididae from the Hawaiian Islands. These
Honeycreepers reside on the various Hawaiian Islands. Through a series of studies, these
Honeycreepers are found to be evolved from an ancestral Honeycreeper which resided originally on,
Kauai, one of the Hawaiian Islands.

Fig. 7.1

(a) Explain how these various species of Honeycreepers could have arisen. [4]

Mutation
Spontaneous mutation or inherited variation in the genotype of these organisms provided
genetic variation of each organism

Natural Selection
Difference in selection pressure (e.g. food type) due to the different habitat that these forms
are found.
There would be one phenotype that would be at a selective advantage in the environment
as it is better adapted to the living conditions. They will survive, reproduce and pass on their
genes to offsprings.

Isolation
As these different forms are found on the different islands/ habitats, they are geographically
isolated. Interbreeding is prevented and hence, prevention of gene flow.

TPJC 9648/02/JC2 PRELIM/2013 [Turn Over]


1688
21
Genetic drift
Also, the common ancestor might be genetically different in the natural population in the
area of origin. When they migrate to a new island, there will be Founder effect. As a result of
the loss of genetic variation, the new population may be distinctively different, both
genetically and phenotypically, from the parent population from which it is derived.

Our understanding of the phylogeny of species has been greatly improved through the use of molecular
techniques.

Many phylogenetic studies use mitochondrial DNA rather than nuclear DNA.

(b) Explain what is meant by the term phylogeny. [2]

Phylogeny is the organisation of species according to particular characteristics


which takes into consideration the evolutionary relationship between the species.

A new Honeycreeper (Fig. 7.2) has been found on one of the Hawaiian Islands. Based on its structural
features, Scientists are speculating that this new found Honeycreeper is also a descendant of the
ancestral Honeycreeper found on Kauai.

Fig. 7.2

(c) Suggest why it is not advisable to determine the phylogenetic relationship of this new found
Honeycreeper to the rest of them from the morphological features. [2]

Basing on morphological features is subjective and non-quantifiable.


Similarities in morphological structures could be due to convergent evolution

(d) Outline how DNA can be studied to determine phylogeny of organisms. [5]

Studying DNA from different organisms is working on the principle of molecular homology
DNA samples can be extracted from target organisms or from the bone marrow of fossils. With
techniques of DNA sequencing technology, the DNA can be sequenced.
For basis of valid comparison the sequence of similar genes (for example, cytochrome c
oxidase gene) are compared amongst the organisms.
A data base of DNA sequences of genes could also be used to retrieve information of particular
organism of interest.
Degree of similarity in sequences/ degree which sequences of gene are conserved between
species are interpreted as DNA evidence for evolution.
Generally, a higher degree which sequences are being conserved a higher degree of
relatedness different species are to one another and vice versa.
TPJC 9648/02/JC2 PRELIM/2013 [Turn Over]
1689
22

[Total: 13]

SECTION B (20 MARKS)

Answer one question.


Write your answers on the separate writing paper provided.
Your answers should be illustrated by large, clearly labeled diagrams, where appropriate.
Your answers must be in continuous prose.
Your answers must be set out in sections (a), (b) etc., as indicated in the question.

QUESTION 8

(a) Compare anaerobic respiration and aerobic respiration. [6]


Max 3 for differences Anaerobic Respiration Aerobic Respiration
Definition/ Components Glycolysis + Glycolysis + Link Reaction + Krebs Cycle
Fermentation + Oxidative phosphorylation
Oxygen requirement Absence of Oxygen Presence of Oxygen
ATP production 2 ATP 38 ATP
Electron carriers involved NAD only NAD and FAD
Location Cytoplasm only Cytoplasm and Mitochondria
Similarities [3 marks]
Starting materials is glucose
Both has glycolysis as a step in the process of oxidation of glucose
Both involves the use of electron carriers (NAD or FAD)
Carbon dioxide gas is produced in both processes

(b) Describe the main features of oxidative phosphorylation. [8]

1) Occurs in the inner mitochondrial membrane


2) Synthesis of ATP occurs as electrons are transferred from reduced NAD and reduced FAD
generated from the Krebs cycle, link reaction or glycolysis
3) Down the Electron Transport Chain (ETC) found on the inner mitochondrial membrane
4) The energy released from the electron transport chain is used to actively pump protons (H+)
from the matrix to the intermembrane space of the mitochondria.
5) This thus creates a proton gradient across the inner membrane. The proton gradient serves
as a source of potential energy for the synthesis of ATP.
6) The stalked particles on the inner mitochondrial membrane consist of hydrophilic protein
channels called F0 proteins and the enzyme ATP synthase.
7) As H+ diffuses through the F0 proteins into the matrix of the mitochondria, the energy
released by the protons is used by ATP synthase to attach an inorganic phosphate to ADP,
thus forming ATP. The electrochemical gradient is said to be coupled to ATP synthesis.
8) Oxidation of one molecule of NADH yields 3 ATP while oxidation of one molecule of FADH2
yields 2 ATP by oxidative phosphorylation.
TPJC 9648/02/JC2 PRELIM/2013 [Turn Over]
1690
23
9) The electrons are passed down the electron transport chain. O2 the final electron acceptor is
reduced to H2O.
10) OP allows regeneration of NAD and FAD for the use the other components of respiration.

TPJC 9648/02/JC2 PRELIM/2013 [Turn Over]


1691
24

(c) Discuss how the mitochondrion is adapted for carrying out respiration. [6]

Fig. 9 Adaptation of mitochondrion to aerobic respiration 1. The cristae of the inner


8. Mitochondria can membrane effectively increase
its surface area to provide
move to areas in the
cell where a lot of abundant space for the layout
activity is taking place. of the multi-enzyme systems.
i.e. the sequence of carriers for
the respiratory chain.

2. The immediate
bathing of the
cristae by the
matrix and the
great surface
6. The liquid matrix area provided by
contains most of the the cristae allow
7. Mitochondria enzymes controlling the the carriers
in active cells Krebs cycle and thus greater access
are larger and allows the enzymes to the substrates
contain more and the substrates to found in the
cristae. interact freely. matrix.

3. Elementary
particles
associated with
ATP synthesis are
found on the
matrix side of the
inner membrane
5. The selectively
permeable membrane 4. The inner membrane is impermeable to protons moving in
contains translocase
from the matrix. A proton pump in the inner membrane pumps
enzymes for the active protons from matrix into intermembrane space.
transport of ATP and
ADP through it.
The outer membrane is freely permeable to protons and
hence allows protons to move out of the mitochondria. Protons
accumulate outside the mitochondria and create a pH gradient.
The pH gradient allows protons to diffuse back into the
mitochondrion in which ATP is synthesized during the process.
TPJC 9648/02/JC2 PRELIM/2013 [Turn Over]
1692
25
QUESTION 9

(a) Compare the mode of infection of Influenza Virus and Human Immunodeficiency Virus. [6]

3 Differences Influenza Virus HIV


Adsorption With the use of HA With the use of GP 120
Penetration By Receptor mediated endocytosis Fusion of Viral envelope with host cell
membrane
Reverse No reverse transcription of Viral RNA Reverse transcription of viral RNA
transcription genome genome to for Viral DNA
Integration No integration of Viral genome into host Integration of Viral DNA to host cell
cell genome genome by integrase
Similarities (3 Similarities)
Nucleocapsid are both removed enzymatically
Both viruses involve embedding viral glycoproteins on the host cell membrane for the formation of
viral envelope
Host cell ribosomes are used for the translation of viral polypeptide for the formation of viral
particles.

(b) Describe the mechanisms of gene transfer in prokaryotes. [8]

There are three mechanisms of gene transfer in prokaryotes, Conjugation, Transformation and
Transduction.

Conjugation [max 2 marks]


Ref. to mating signal
F+ cell (cell containing the F Plasmid) trigger the formation of sex pillus act as a mating bridge
between donor and recipient.
Plasmid from the donor cell replicates via the Rolling Circle Replication a single strand of DNA
enters the recipient cell through the sex pillus
Plasmid in recipient reforms its double stranded structure -> recipient cell acquire genes from donor
cell.

Transformation [max 3 marks]


Bacteria cells must be competent (possesses competence-specific proteins)
DNA from the environment binds to these competence-specific proteins
Autolysin are present to degrade the cell wall to allow DNA to penetrate through cell wall of bacterium
Nucleases present then cut the bound DNA into fragments -> single strand of DNA
Single- stranded DNA will remain bound by competence-specific protein and be brought near
bacterial chromosome
RecA protein will take over to promote genetic exchange between the DNA fragment with the
recipient bacterial chromosome.

TPJC 9648/02/JC2 PRELIM/2013 [Turn Over]


1693
26

Transduction [max 3 marks]


Transduction is a genetics exchange method between bacterial cells facilitated by bacteriophages
Two mode of operation: specialised transduction and generalised transduction
By T4 phage and Lambda phage respectively
During the reproductive cycle of these phages, there is incorporation of bacterial genes into the
phage capsid during formation of mature phages
These bacteriophages with the incorporated bacterial DNA will subsequently infect a new bacterial
host cell
Genetic recombination will occur between the bacterial genes from the previous host cell and the
genes in the new bacterial cell.

(c) Discuss how the prokaryote, E. coli, is able to respond to varying concentrations of lactose. [6]

The prokaryotes possess lac operon which contains the structural genes lac Z, lac Y and
lac A genes
Which codes for beta-galactosidase, lactose permease and lactose transacetylase
respectively
These structural genes are organised under the control of a single promoter and
operator
Expression of these genes are regulated depending on the level of lactose in the
environment.
When [lactose] low, some lactose is transported by the low levels of permease into
bacterial cells
Lactose converts to allolactose by beta-galactosidase
Allolactose binds to lac repressor to induce a change in conformation in repressor
Repressor unable to bind to operator
RNA polymerase can bind promoter to initiate transcription
When [lactose] high, more repressors are repressed by allolactose
These are occurring in absence of glucose, when glucose present -> glucose prevent
synthesis of cAMP
cAMP needed for activation of CAP
CAP needed to bind to CAP binding site for the recruitment of RNA polymerase to
initiate transcription.

End of Paper

TPJC 9648/02/JC2 PRELIM/2013 [Turn Over]


1694

TAMPINES JUNIOR COLLEGE

H JC2 PRELIMINARY EXAMINATION

2
CANDIDATE
NAME

CIVICS GROUP 1 2
TUTOR
NAME

Biology 9648/ 03
Applications Paper and Planning Question Friday, 20 September 2013
Paper 3
2 hours
Candidates answer on the Question Paper
Additional Materials are required: Writing Paper

READ THESE INSTRUCTIONS FIRST

Write in dark blue or black pen.


You may use a soft pencil for any diagrams, graphs or rough
working.
Do not use paper clips, highlighters, glue or correction fluid.

For Examiners Use

1 15
Answer all questions.
2 15

3 10
At the end of the examination:
4 12
1. Fasten all answer scripts for Question 5 securely;
2. Hand in Paper 3 and the answer scripts for 5 20
Question 5 separately.
Total 72

The number of marks is given in brackets [ ] at the end of each


question or part question.

This document consists of 15 printed pages and 1 blank page


1695
2
STRUCTURED QUESTIONS (40 MARKS)
Answer all the questions in this section in the space provided.

QUESTION ONE ( 15 MARKS)

One of the applications of RFLP analysis is for restriction mapping. BamH1, EcoRI and HindIII
restriction enzymes are used to map a 15kb bacterial DNA. Three reactions were set up to cut
the 15kb DNA sample, each with one of the three enzymes. After digestion with restriction
enzymes, the DNA fragments were separated into bands. Results of the reactions are shown
in Fig 1.1.
Describe briefly how, in such DNA analysis,
(i) DNA is separated into bands; [4]

(ii) the bands are made visible. [1]

(iii) Based on the information given and the results in Fig 1.1, which of the following map
(W, X, Y or Z) in Fig 1.2 is not a possible map? [1]

Map __________________________

E = EcoRI

B = BamHI

H = HindIII
W

Scale

TPJC 9648/02/JC2 Prelim/2013 [Turn Over


Fig 1.1 Fig 1.2 15kb
1696
3

Further procedures were performed by setting up additional digestions using pairs of


restriction enzymes: BamH1/EcoR1, BamH1/HindIII, and EcoRI/HindIII in three
separate reactions.

(iv) With reference to the results shown in Fig 1.3, explain which of the following W, X Y or
Z is the correct map. [3]

Fig 1.3

Another application of RFLP analysis is for forensic scientist to identify criminals. Two
men (Suspect 1 and 2) were accused of kidnapping and raping a woman. RFLP
analysis was used to examine the genetic characteristics of the two suspects, the
victim, and semen from the vagina of victim. Two genetic loci, D4S139 (a) and D10S28
(b), were examined. These two loci are variable number of tandem repeats (VNTR).

TPJC 9648/02/JC2 Prelim/2013 [Turn Over


1697
4
Fig 1.4 shows two autoradiograms of the DNA banding patterns. Three lanes of the
autoradiograms (on the far left, far right, and middle) display multiple bands called size
markers.
D4S139 gene loci D10S28 gene loci

(i) State the function of the size markers. [2]

(ii) Explain the term VNTR. [1]

(iii) Identify and explain the criminal with evidences shown in Fig 1.4. [3]

[Total: /15 marks]

TPJC 9648/02/JC2 Prelim/2013 [Turn Over


1698
5
QUESTION TWO (15 MARKS)

Children with severe combined immunodeficiency disorder (SCID) cannot produce the many
types of white blood cells that fight infections. This is because they do not have the functional
gene to make the enzyme, adenosine deaminase (ADA).
Researchers are currently exploring the possibility of using stem cells in therapy for SCID.
Some children with SCID have been treated with stem cells. Stem cells can divide and develop
into any type of blood cell, including white blood cells.
The treatment used with the children is described in Fig. 2.1.

Fig. 2.1

(a) Using the information given, state two reasons why stem cells were used in this
treatment. [2]

(b) Suggest the type and the potency of stem cells that were used in this treatment. [2]

Type: ___________________________________________________________________

Potency: _________________________________________________________________

TPJC 9648/02/JC2 Prelim/2013 [Turn Over


1699
6
A child was treated with genetically engineered stem cells. Fig. 2.2 shows the number of
functioning white blood cells in the child during the year following treatment. Children who do
not suffer from SCID have between 5000 and 8000 white blood cells per mm3 of blood.

Fig. 2.2

(c) Describe the results shown in Fig. 2.2. [3]

(d) Explain why a person treated by gene therapy may still have children who suffer from
SCID. [2]

TPJC 9648/02/JC2 Prelim/2013 [Turn Over


1700
7

SCID is an inherited autosomal recessive disorder. Therefore it is easier to treat SCID by


gene therapy.
(e) Suggest why it is easier to perform gene therapy when a mutant allele is recessive
instead of dominant. [1]

Although the use of gene therapy for treatment of genetic disorders in humans seems
promising, there are still many concerns to be addressed.
(f) Explain the factors that may keep gene therapy from becoming an effective treatment
for genetic disorders. [3]

(g) Discuss one ethical and one social concern regarding the use of gene therapy. [2]

[Total: /15 marks]

TPJC 9648/02/JC2 Prelim/2013 [Turn Over


1701
8
QUESTION THREE ( 10 MARKS)

Read the following passage.

Genetic engineering is a technique for improving crop plants. It is a process by which a single
new gene can be added to the genes already present in the plants. One way of doing this is to
use a natural genetic engineer, a soil microorganism called Agrobacterium. This bacterium
possesses a plasmid which can be modified in the laboratory so that it becomes the carrier of
new genetic information.

For example, a gene coding for the ability to break down the herbicide glyphosate can be
introduced with the assistance of the enzymes restriction endonuclease and ligase. Once inside
the crop plant, the gene for glyphosate breakdown makes the plant resistant to the effects of
this herbicide. All surrounding weeds are destroyed when sprayed with glyphosate.

The flow-chart shows how the plasmid carrying this gene can be used to produce glyphosate-
resistant plants.

TPJC 9648/02/JC2 Prelim/2013 [Turn Over


1702
9
Use the information from the passage and your own knowledge to answer the
following questions.

(a) One mature plant grows from each glyphosate-resistant plant cell. Explain why all
the cells of the mature plant contain the gene for glyphosate-resistance. [1]

(b) Outline the technique of tissue culture needed to obtain matured plants from the
transformed cells which are resistant to the herbicide. [3]

(c) Suggest and explain one possible problem associated with the use of herbicides
together with genetically modified, herbicide-resistant crop plants. [1]

TPJC 9648/02/JC2 Prelim/2013 [Turn Over


1703
10
Another technique required in plant biotechnology is plant tissue culture. Various
organs or tissues of a plant may be used in tissue culture to produce a large number of
genetically identical plants.

In the process of anther culture, haploid plantlets are produced from immature anthers,
from which mature fertile homozygous plants are obtained. The process is shown
diagrammatically in the figure below.

Unopened flower bud



Anthers removed and placed on culture medium 1

Mitosis of immature pollen cells to give haploid callus

Pieces of callus removed and placed on culture medium 2

Haploid plantlets

Treated with colchicines to double the chromosome number

Mature fertile homozygous plants

(d) With reference to the figure,


(i) Explain what is meant by haploid callus. [2]

(ii) State one precaution that would be taken when transferring material onto
culture medium 1 or 2. [1]

(iii) Suggest why variation may be generated in plantlets derived from the same
callus. [1]

(iv) Suggest how colchicine could bring about the doubling of chromosome
number in haploid cells. [1]

[Total: /10 marks]

TPJC 9648/02/JC2 Prelim/2013 [Turn Over


1704
11

PLANNING QUESTION

QUESTION FOUR (12 MARKS)

You are required to plan, but not carry out, an investigation into the transformation of
Escherichia coli bacteria using the pGLO plasmid as shown below.

The GFP gene is regulated by a promoter of the arabinose operon. The operon can be
induced by the presence of arabinose. The successful uptake of the pGLO can be
detected by bacteria that florescence green under UV light.

(Modified from http://www.mikeblaber.org/oldwine/BCH4053l/Lecture07/Lecture07.htm)

Your planning must be based on the assumption that you have been provided with the
following equipment and materials which you must use:
microfuge tubes
different types of agar plates
foam microfuge tube holder
parafilm sealing tapes
stopwatch
thermometer
micropipettes and tips
incubator oven
transformation solution
plasmid DNA solution (pGLO)
competent Escherichia coli cells in microfuge tube
sterile water
tripod stand and bunsen burner
hot water and crushed ice
UV visualiser

Your plan should have a clear and helpful structure to include


an explanation of theory to support your practical procedure
a description of the method used including the scientific reasoning behind the
method
how you will record your results and ensure that they are as accurate and reliable
as possible
proposed layout of results tables and graphs with clear headings and labels
the correct use of technical and scientific terms.
TPJC 9648/02/JC2 Prelim/2013 [Turn Over
1705
12

TPJC 9648/02/JC2 Prelim/2013 [Turn Over


1706
13

TPJC 9648/02/JC2 Prelim/2013 [Turn Over


1707
14

TPJC 9648/02/JC2 Prelim/2013 [Turn Over


1708
15

FREE-RESPONSE QUESTION (20 MARKS)

Write your answer to this question on the separate answer paper provided.
Your answer:
should be illustrated by large, clearly labeled diagrams, where appropriate;
must be in continuous prose, where appropriate;
must be set out in sections (a), (b) etc., as indicated in the question.

QUESTION FIVE

(a) Describe PCR and compare and contrast with replication process. [8]

(b) Outline the process of gel electrophoresis and explain how it is able to separate
fragments of DNA. [9]

(c) State the uses of RFLP analysis. [3]

End Of Paper

TPJC 9648/02/JC2 Prelim/2013 [Turn Over


1709

TAMPINES JUNIOR COLLEGE

H JC2 PRELIMINARY EXAMINATION


[mark scheme]

2
CANDIDATE
NAME

CIVICS GROUP 1 2
TUTOR
NAME

Biology 9648/ 03
Applications Paper and Planning Question Friday, 20 September 2013
Paper 3
2 hours
Candidates answer on the Question Paper
Additional Materials are required: Writing Paper

READ THESE INSTRUCTIONS FIRST

Write in dark blue or black pen.


You may use a soft pencil for any diagrams, graphs or rough
working.
Do not use paper clips, highlighters, glue or correction fluid.

For Examiners Use

1 15
Answer all questions.
2 15

3 10
At the end of the examination:
4 12
1. Fasten all answer scripts for Question 5 securely;
2. Hand in Paper 3 and the answer scripts for 5 20
Question 5 separately.
Total 72

The number of marks is given in brackets [ ] at the end of each


question or part question.

This document consists of 16 printed pages and 0 blank page


1710
2
STRUCTURED QUESTIONS (40 MARKS)
Answer all the questions in this section in the space provided.

QUESTION ONE ( 15 MARKS)

One of the applications of RFLP analysis is for restriction mapping. BamH1, EcoRI and HindIII
restriction enzymes are used to map a 15kb bacterial DNA. Three reactions were set up to cut
the 15kb DNA sample, each with one of the three enzymes. After digestion with restriction
enzymes, the DNA fragments were separated into bands. Results of the reactions are shown in
Fig 1.1.

(b) Describe briefly how, in such DNA analysis,


(i) DNA is separated into bands; [4]
via gel electrophoresis;
negatively charged DNA migrate towards positive electrode / anode;
DNA fragments thus separated by their molecular size / length ;
smaller / shorter restriction fragments migrate faster / move a longer
distance in a fixed time;

(ii) the bands are made visible. [1]


DNA fragments are stained with Et Br
which intercalate between DNA
visualize under UV light
Or
stain with methylene blue solution
view with naked eyes

(iii) Based on the information given and the results in Fig 1.1, which of the following
map (W, X, Y or Z) in Fig 1.2 is not a possible map? [1] A

(iv) With reference to the results shown in Fig 1.3, explain which of the following W,
X Y or Z is the correct map. [3]

Fig 1.3
Y;;
TPJC 9648/02/JC2 Prelim/2013 [Turn Over
1711
3
When the 15kb DNA is cut with the two enzyme BamHI and EcoRI, there
are three bands, 5kb, 3kb and 2kb
the 5kb band is thicker- two copies of the 5kb
when cut with the BamH1 and HindIII, there are 3 bands, 7kb, 3kb and
2kb,
where 3kb is the doublet
When cut with enzyme EcoR1 and HindIII, there are 3 bands, 8kb, 5kb
and 2kb

(c) Another application of RFLP analysis is for forensic scientist to identify criminals. Two
men (Suspect 1 and 2) were accused of kidnapping and raping a woman. RFLP analysis
was used to examine the genetic characteristics of the two suspects, the victim, and a
vaginal aspirate. Two genetic loci, D4S139 (a) and D10S28 (b), were examined. These
two loci are variable number of tandem repeats (VNTR).

Fig 1.4 shows two autoradiograms of the DNA banding patterns. Three lanes of the
autoradiograms (on the far left, far right, and middle) display multiple bands called size
markers.

(i) State the function of the size markers. [2]


Size markers are DNA fragments of known sizes;
They are compared to the bands in the other lanes;
in order to allow the size of these fragments to be estimated;
1max

(ii) Explain the term VNTR. [1]


non coding DNA portions containing repeated sequences of between 20
and 100 base pairs;
sequence repeat in series/one after another;

(iii) Identify the criminal with evidences shown in Fig 1.4. [3]
Suspect 2;
when genetic locus D4S139 were used, banding pattern of suspect two
(lane 4) is exactly the same as the semen from vagina (lane 7) in (a);
when genetic locus D10S28 were used, one DNA band (lowest band) in
suspect 1s matches the lowest band of the semen from the vagina,
however, there are two DNA bands from the suspect one which
correspond to the two out of three bands of the semen from the vagina,
which provides additional evidence against him;
(students could label the bands and explain using the labeled terms)
[Total: /15 marks]

QUESTION TWO (15 MARKS)


Children with severe combined immunodeficiency disorder (SCID) cannot produce the
many types of white blood cells that fight infections. This is because they do not have the
functional gene to make the enzyme, adenosine deaminase (ADA).
Researchers are currently exploring the possibility of using stem cells in therapy for SCID.
Some children with SCID have been treated with stem cells. Stem cells can divide and
develop into any type of blood cell, including white blood cells.
The treatment used with the children is described in Fig. 2.1.

TPJC 9648/02/JC2 Prelim/2013 [Turn Over


1712
4

Fig. 2.1

(a) Using the information given, state two reasons why stem cells were used in this
treatment.
Capable of dividing for long periods to generate replacements for cells that are unable to
produce ADA;
Unspecialised and can differentiate to specialized cell types such as white blood cells to
fight infection;
The stem cells used belongs to the child and there will be no / little risk of rejection /
triggering immune response; [any 2]
[2]
(b) Suggest the type and the potency of stem cells that were used in this treatment.
Type Adult / hematopoietic / blood stem cells;

Potency Multipotent;

[2]

TPJC 9648/02/JC2 Prelim/2013 [Turn Over


1713
5
A child was treated with genetically engineered stem cells. Fig. 2.2 shows the number of
functioning white blood cells in the child during the year following treatment. Children who
do not suffer from SCID have between 5000 and 8000 white blood cells per mm3 of blood.

Fig. 2.2

(c) Describe the results shown in Fig. 2.2.


From 0 to 4 months of stem cells injection, the number of functioning white blood cells
per mm3 increases slowly from 100 to 1000;
From 4 to 8 months of stem cells injection, the number of functioning white blood cells
per mm3 increases exponentially from 1000 to 5500;
From 8 to 12 months of stem cells injection, the number of functioning white blood
cells per mm3 plateau / level off at 5800;
[3]
(d) Explain why a person treated by gene therapy may still have children who suffer from
SCID.
Therapeutic gene inserted into somatic cells /
Therapeutic gene not inserted into gametes / germ cells;
Defective gene is inherited;
[2]

TPJC 9648/02/JC2 Prelim/2013 [Turn Over


1714
6
SCID is an inherited autosomal recessive disorder. Therefore it is easier to treat SCID by
gene therapy.

(e) Suggest why it is easier to perform gene therapy when a mutant allele is recessive
instead of dominant.
When mutant allele is recessive, addition of functional / normal dominant allele by gene
therapy will mask the effect of recessive allele /
If mutant allele is dominant, it has to be removed / repaired / inactivated to block
production of defective gene product;
[1]
Although the use of gene therapy for treatment of genetic disorders in humans seems
promising, there are still many concerns to be addressed.
(f) Explain the factors that may keep gene therapy from becoming an effective treatment
for genetic disorders.
Difficult to ensure that therapeutic gene is integrated into genome of host cells;
Incorrect insertion of therapeutic gene into genome of host cell results in cancer, as the
insertion is random;
Immune response may be triggered resulting in rejection because a foreign virus vector
is introduced;
Difficult to control expression of normal functional gene to give a fully functional protein;
[3]
(g) Discuss one ethical and one social concern regarding the use of gene therapy.
Ethical: Gene therapy may be viewed as tampering with nature and going against the
natural way of life;
Social: There may be genetic discrimination in the society as the issue of defining what
is a disability/disorder/disease is unclear;
The poor will be discriminated against because they cannot afford the expensive
treatment of gene therapy;
[2]

[Total: /15 marks]

TPJC 9648/02/JC2 Prelim/2013 [Turn Over


1715
7
QUESTION THREE( 10 MARKS)

Read the following passage.

Genetic engineering is a technique for improving crop plants. It is a process by which a single
new gene can be added to the genes already present in the plants. One way of doing this is
to use a natural genetic engineer, a soil microorganism called Agrobacterium. This bacterium
possesses a plasmid which can be modified in the laboratory so that it becomes the carrier of
new genetic information.

For example, a gene coding for the ability to break down the herbicide glyphosate can be
introduced with the assistance of the enzymes restriction endonuclease and ligase. Once
inside the crop plant, the gene for glyphosate breakdown makes the plant resistant to the
effects of this herbicide. All surrounding weeds are destroyed when sprayed with glyphosate.

The flow-chart shows how the plasmid carrying this gene can be used to produce glyphosate-
resistant plants.

Use the information from the passage and your own knowledge to answer the following
questions.

(a) One mature plant grows from each glyphosate-resistant plant cell. Explain why all the
cells of the mature plant contain the gene for glyphosate-resistance. [1]

The cells are genetically identical / clone of the glyphosate cells due to mitotic
division of the cell.

TPJC 9648/02/JC2 Prelim/2013 [Turn Over


1716
8
(b) Outline the technique of tissue culture needed after obtaining matured plants from the
transformed cells which are resistant to the herbicide. [3]

The explant is first sterilized using sodium hypochlorite

The explant is then transferred to an aseptic culture medium that contains


nutrients, growth factors/hormones.

The cells of the explant will divide by mitosis to form a callus

The callus can be subcultured into many calli in different culture

The ratio of plant hormones will result in the plant tissues developing into
specialized tissues.

e.g. of a ratio leading to either root inducing or shoot inducing

When a plantlet is formed, it can be transferred to soil to grow into a whole


plant.

Suggest and explain one possible benefit and problem associated with the use of
herbicides together with genetically modified, herbicide-resistant crop plants. [1]

The creation of superweeds with horizontal gene transfer

The production of toxins by the gene that can cause allergy in human beings
[Any 1 point]

Another technique required in plant biotechnology is plant tissue culture. Various organs or
tissues of a plant may be used in tissue culture to produce a large number of genetically
identical plants.

In the process of anther culture, haploid plantlets are produced from immature anthers, from
which mature fertile homozygous plants are obtained. The process is shown diagrammatically
in the figure below.

Unopened flower bud



Anthers removed and placed on culture medium 1

Mitosis of immature pollen cells to give haploid callus

Pieces of callus removed and placed on culture medium 2

Haploid plantlets

Treated with colchicines to double the chromosome number

Mature fertile homozygous plants

(d) With reference to the figure,

(i) explain what is meant by haploid callus. [2]

The cells are genetically identical / undifferentiated glyphosate cells

With 1 set of chromosomes

TPJC 9648/02/JC2 Prelim/2013 [Turn Over


1717
9

(ii) State one precaution that would be taken when transferring material onto culture
medium 1 or 2. [1]

Ensure that the environment for the transfer is aseptic to prevent


contamination.

(iii) Suggest why variation may be generated in plantlets derived from the same callus.
[1]
It can be due to chromosomal mutations

(iv) Suggest how colchicine could bring about the doubling of chromosome number in
haploid cells. [1]

It may prevent the sister chromatids from separating during anaphase/

Cause non-disjunction

[Total: /10 marks]

PLANNING QUESTION

QUESTION FOUR (12 MARKS)

You are required to plan, but not carry out, an investigation into the effect of XXX

Your planning must be based on the assumption that you have been provided with the following
equipment and material which you must use:

A large piece of beetroot tissue with the skin removed


Sharp knife
White tile
Ruler
Thermostatically controlled water bath
Test-tubes
Stopwatch
Distilled water
Colorimeter
A variety of different-sized beakers, measuring cylinders and syringes for measuring
volume

Your plan should have a clear and helpful structure to include:

a description of the method used including the scientific reasoning behind the method,
an explanation of the dependent and independent variables involved,
relevant, clearly labelled diagrams,
how you will record your results and ensure they are as accurate and reliable as
possible,
proposed layout of results tables and graphs with clear headings and labels,
the correct use of technical and scientific terms,
relevant risks and precautions taken.

TPJC 9648/02/JC2 Prelim/2013 [Turn Over


1718
10

Mark scheme

INTRODUCTION/Theoretical Background (2 marks)


1. Explanation of theory to support practical procedure
Define transformation as uptake of foreign DNA (pGLO plasmid) from the external
environment by E. coli which expresses the genetic information encoded by the
plasmid.
antibiotic resistance gene on plasmid will confer ampicillin resistance to the bacterial
cells able to survive when exposed to ampicillin to form colonies
arabinose in the agar medium will allow RNA polymerase to bind to promoter
allows for the transcription and translation of GFP gene green colonies under UV
light

2. Scientific reasoning behind the method


Ca2+ ions in transformation solution neutralize the negative charges of the plasmid DNA
to facilitate its entry into the bacteria
Heat shock methods create temporary pores in bacterial cell membrane to facilitate the
entry of plasmid

PROCEDURE/METHOD (Total number of points = 11 [3 9 & 11]; max is 6 marks)


3. Preparation of different nutrient agar plates
(table/ any equivalent to show labels and composition of each plate)

4. Chemical-heat shock method for transformation


a) appropriate volume of competent cells + plasmid solution / sterile water + transformation
solution in a microfuge tube
b) ice 42oC, 50s heat shock ice again
c) Method of achieving and maintaining desired temperature

Note that it is a brief heat shock at 420C for 50s and there is ice before and after the step.
Many students specify an incubation period of 10 30 minutes at room temperature to 40 0C
instead which is for transduction.

5. a) Method of plating
Removal of suitable amount of bacteria appropriate plate spreader/ inoculating loop/
beads (spreading)

b) Incubation of bacteria for growth


(incubator oven, 37oC, overnight)

6. Use of control plate (any one)


(What Grow the bacterial cells treated with sterile water instead of pGLO plasmid solution
on LB agar plates
Why to show that bacterial cells are still alive after going through the transformation
procedure/ to compare with those cells grown on LB/Amp agar plates to show that ampicillin
in the agar plate is effective is killing bacteria cells with no resistance
What grow bacterial cells treated with sterile water on LB/Amp agar plates
Why To compare with pGLO treated cells grown on LB/Amp agar plates to show that
ampicillin resistance is indeed conferred by the pGLO plasmid
What Grow pGLO treated bacterial cells on LB/Amp agar plate
Why To compare with LB/Amp/Arabinose agar plate to show that Arabinose is causing
fluorescence in pGLO treated bacterial cells)

TPJC 9648/02/JC2 Prelim/2013 [Turn Over


1719
11

7. Use of sterile equipment to prevent cross contamination between plates


(Use different sterile glass beads/ spreader/ inoculating loop for spreading the cells evenly
across plate and new micropipette tips for every transfer of volume)

8. Repeat twice with replicates


(Idea of consistency 6 sets of data)

9. Method of determining successful transformation


(White colonies / Green colonies and count the number of colonies that grow on nutrient
plates)

USE OF DATA TO REACH A CONCLUSION


10. Labeled table for data collection (1 mark)
(Independent variable in leftmost column, headings: Name and type of agar plate,
Observations: description of bacterial growth & (average) number of colonies; Experiment
and replicates)

11. Processing of data


(Calculate average number of bacterial colonies)

12. Graph to represent processed data (1 mark)


(Plot average number of colonies against Name and type of agar plate labels for x-axis, y-
axis)

SAFETY ISSUE (1 mark)


13. Safety in handling of bacteria
(e.g. heat sterilization of bacterial plates at 121oC/ autoclaving before disposal / wear gloves
when handling bacteria)

14. The use of correct technical and scientific terms (1 mark)


Scientific terms such as restriction enzymes, recombinant and re-annealed vector, replica
plating are not correct terms to be used here as there is no need to cut the pGLO plasmid
provided since it already has the GFP gene (GOI) inserted into it.

TPJC 9648/02/JC2 Prelim/2013 [Turn Over


1720
12

FREE-RESPONSE QUESTION (20 MARKS)

Write your answer to this question on the separate answer paper provided.
Your answer:
should be illustrated by large, clearly labeled diagrams, where appropriate;
must be in continuous prose, where appropriate;
must be set out in sections (a), (b) etc., as indicated in the question.

QUESTION FIVE

(a) Describe PCR and compare and contrast with replication process. [6]

(b) Outline the process of gel electrophoresis and explain hot is is able to separate
fragments of DNA.
[9]

(c) State the uses of RFLP analysis. [5]

Answers

(a) Describe PCR and compare and contrast with replication process. [6]

1st step: One to several minutes at 94-96 degrees C, during which the DNA is denatured
into single strands by breaking of hydrogen bonds.

2nd step: One to several minutes at 50-65 degrees C, during which the primers hybridize
or "anneal" (by way of hydrogen bonds) to their complementary sequences on either
side of the target sequence.

3rd step: One to several minutes at 72 degrees C, during which the Taq polymerase binds
and extends a complementary DNA strand from each primer, using the single stranded
DNA as a template.

Replication PCR
primer RNA primers DNA primers
enzyme DNA polymerase III & I Taq DNA polymerase
proofreading yes no
Temperature dependent no yes

TPJC 9648/02/JC2 Prelim/2013 [Turn Over


1721
13

TPJC 9648/02/JC2 Prelim/2013 [Turn Over


1722
14
(b)
Outline the process of gel electrophoresis and explain how it is able to separate
fragments of DNA. [9]

Agarose gel electrophoresis is a method for separating and visualizing DNA


fragments produced by restriction digestion of DNA.
The fragments are separated by charge and size by forcing them to move through a
agarose gel matrix which is subjected to an electric field.

Principles of agarose gel electrophoresis

Biological molecules such as DNA, RNA and proteins are electrically charged
particles at a given pH.

DNA is negatively charged due to the phosphate groups of the sugar-


phosphate backbone.
If the sample contains DNA fragments of different sizes, they can be
separated based on their sizes when they are subjected to an electric field.
Smaller fragments will move faster then the larger fragments.
Step 1: Casting of the agarose gel

Agarose is a polysaccharide derived from seaweed that forms a gel when


dissolved in an aqueous solution.
Agarose solution will be poured into casting tray and allowed to solidify.
With the help of the comb, small indentations called wells are formed at one end of
the gel, where DNA samples can be loaded.
Each well corresponds to one lane.

Step 2: Loading of sample

DNA samples are first mixed with a loading dye which increases its density and
helps the DNA to sink to the bottom of the well.
Loading dye also helps to monitor the progress of the electrophoresis as DNA is
invisible.
Markers are usually run in the 1st lane. Markers are mixtures of DNA fragments of
known sizes. By comparing the markers fragment against the unknown fragments,
we can estimate the sizes of the unknown fragments.
Samples are loaded into wells with the help of a micropipette.

Step 3: Running of gel

Current is switched on.


Negatively charged DNA fragments will migrate out of the well and are attracted to
the positive electrode.
Separation of the fragments will be based on their sizes and can be seen as
discrete bands on the gel.

Step 4: Visualization
Before the band reaches the end of the gel, current will be switched off.
Gel is then stained with DNA binding dye (ethidium bromide which is
carcinogenic).
Separated DNA fragments can been clearly under the UV light.
TPJC 9648/02/JC2 Prelim/2013 [Turn Over
1723
15

(c) State the uses of RFLP analysis. [3]

Genomic mapping in terms of linkage mapping

Disease detection, e.g sickle cell anaemia

DNA fingerprinting

TPJC 9648/02/JC2 Prelim/2013 [Turn Over


1724
2

1 The electron micrograph shows a chloroplast.

The length of the chloroplast along the line shown is 80 mm. The actual length of the
chloroplast is 10 m.

What is the magnification of the chloroplast?

A 8X102 B 8X103 C 8 X104 D 8X106

9648/ 01 / Preliminary examination / YJC 2013


1725
3

2 The flow chart shows processes which take place inside animal cells.

phagocytosis

Which processes require the activity of lysosomes?

A W, X, Y and Z

B W and X only

C X and Y only

D Y and Z only

9648/ 01 / Preliminary examination / YJC 2013


1726
4

3 Lipid membranes can be formed in the laboratory by painting phospholipids over a PTFE
sheet with a hole in it.

Such a lipid membrane is impermeable to water-soluble materials including charged ions such
as Na+ or K+. In one experiment with Na+ ions, no current flowed across the membrane until a
substance called gramicidin was added, at which time current flowed.

Which statement is consistent with this information and your knowledge of membrane
structure?

Gramicidin becomes incorporated into the membrane and is

A a carbohydrate molecule found only on the outside of the membrane.

B a non-polar lipid which passes all the way through the membrane.

C a protein molecule with both hydrophilic and hydrophobic regions.

D a protein molecule which has only hydrophobic regions.

4 Iodine test gives a blue-black colour in the presence of starch. The time taken for the blue-
black colour of a fixed amount of starch to disappear was recorded over a range of
temperature.

Which graph shows the results of this investigation?

9648/ 01 / Preliminary examination / YJC 2013


1727
5

5 The diagram shows four different amino acids, each with a different R group (side chain).

Which amino acid could form a hydrophobic interaction between their R groups?

A 1 and 2 only

B 1 and 3 only

C 2 and 4 only

D 3 and 4 only

9648/ 01 / Preliminary examination / YJC 2013


1728
6

6 A living cell from the lung epithelium of a newt is shown at different stages of mitosis in the
micrographs (A to F) below. Identify the correct sequence of these micrographs.

A DECAFB

B EDCAFB

C ECDAFB

D EDCBAF

7 Human immunodeficiency virus (HIV) is a retrovirus. After infecting a host cell, viral DNA is
produced which is incorporated into the DNA of the host cell. The modified host genome now
codes for the production of new HIV particles.

Which could be used as a potential treatment to slow down the spread of HIV?

1. inhibitors of restriction endonucleases

2. inhibitors of reverse transcriptase

3. restriction endonucleases

4. reverse transcriptase

A 1 only B 2 only C 1 and 4 only D 2 and 3 only

9648/ 01 / Preliminary examination / YJC 2013


1729
7

8 Which of the following shows the possible effects of a single nucleotide substitution in each of
the following locations in a gene on the production of the protein it codes for?

Transcription Middle of an
Promoter Start codon Stop codon
terminator intron
Protein product Too much Protein product Protein product No protein
A is longer than protein product is normal is shorter than product is
normal is produced normal produced
No protein Protein product Protein product Protein product Too much
B product is is shorter than is longer than is normal protein product is
produced normal normal produced
Protein product Protein product Protein product Too much Protein product
C is normal is longer than is shorter than protein product is longer than
normal normal is produce normal
Too much Protein product No protein Protein product Protein product
D protein product is normal product is is longer than is normal
is produced produced normal

9 Sex determination in some insects such as bees and wasps is not controlled by sex
chromosomes.

Using the diagram, which row in the table shows how sex is determined in these insects?

P Q R S
A n 2n mitosis meiosis

B n n mitosis mitosis

C 2n n meiosis meiosis

D 2n 2n meiosis mitosis

9648/ 01 / Preliminary examination / YJC 2013


1730
8

10 Meselson and Stahl found that in dividing cells, DNA is copied by semi-conservative
replication. At the time of their discovery it was thought that DNA might be copied in one of
three ways. In the diagram, the original DNA strands are shown by solid lines and the copy
strands by dotted lines.

Which set of results would have proved that the DNA replication was conservative?

A Key

0 = original culture in 15N

1 = first generation in 14N

2 = second generation in 14N

3 = third generation in 14N


B

9648/ 01 / Preliminary examination / YJC 2013


1731
9

11 Multiple proteins are involved in the process of blood clotting. The diagram below shows how
factor V functions as a co-factor in blood clotting.

Thrombin goes on to cleave fibrinogen to form fibrin, which helps in blood clot formation.
Another protein that is involved in blood clotting is activated protein C (aPC), a natural
anticoagulant that acts to limit the extent of clotting by cleaving and degrading factor V. A
mutation in factor V protein results in a variant protein called factor V Leiden. The Leiden
variant of the protein is resistant to degradation by aPC.

Which of the following option shows the mode of inheritance of the Leiden allele, the correct
type of mutation this is and the result this mutation will have on blood clotting?

Mode of
Type of mutation Result on blood clotting
inheritance
A Dominant Gain-of-function Blood clots more easily
B Dominant Loss-of-function Blood clots less easily
C Recessive Gain-of-function Blood clots less easily
D Recessive Loss-of-function Blood clots more easily

12 Mature mRNA and DNA are extracted from the same cells and mixed in conditions that favour
the mRNA and complementary parts of the DNA becoming linked. Some parts of the DNA
remain as double-stranded DNA. Other parts of the DNA form equal numbers of single-
stranded DNA loops and double-stranded DNA-mRNA loops.

What describes these loops?

A exons form single-stranded loops, introns form double-stranded loops

B exons form double-stranded loops, introns form single-stranded loops

template exons form double-stranded loops, non-template exons form single-stranded


C
loops

template introns form double-stranded loops, non-template introns form single-


D
stranded loops

9648/ 01 / Preliminary examination / YJC 2013


1732
10

13 Some studies suggest that smoking increases the risk of developing lung cancer. The two
graphs show the percentage of smokers and the deaths from lung cancer in men of two age
groups between 1950 and 1998.

Which statement is not supported by the data in the graphs?

A Deaths from lung cancer in men aged 35-59 decreased by 50% over the period of the
study.
B Deaths from lung cancer in men aged 60-74 increased up to 1970.

C The data for men aged 60-74 between 1950 to 1970 suggests that lung cancer takes
up to 20 years to develop.

D The number of men aged 35-59 who were smokers decreased by approximately 60%
over the period of the study.

14 Which statement correctly describes a role of histone proteins?

A All eukaryotic genes are transcribed continuously because they are not packaged by
histones.

B DNA must be selectively released from its histone packaging before transcription can
occur in bacteria.

C Histones package prokaryote chromatin into the nucleosomes that form the bulk of the
chromosome.

D The organisation of DNA by histones in eukaryotes allows some gene control


sequences to be thousands of base pairs away from the gene concerned.

9648/ 01 / Preliminary examination / YJC 2013


1733
11

15 Which of the following statements about RNA splicing of a single pre-mRNA are correct?

1. Alternative splicing controls the amount of gene products formed by having


different promoters.

2. Alternative splicing controls the type of gene products formed by having


different exons.

3. Different gene products can be formed at the same time within the same cells.

4. Different gene products can be formed at different stages of an organisms life


cycle.

5. Different mRNA transcripts can be produced at the same time.

A 2 and 4 only

B 3 and 5 only

C 1, 2 and 4 only

D 1, 3 and 5 only

16 Staphylococcus aureus is a common bacterium, found on human skin. There are many strains
of S. aureus. The antibiotic methicillin was used to treat infection by S. aureus. Now there are
at least 15 different strains of MRSA (methicillin resistant Staphylococcus aureus).

Which of the following are valid reasons for the emergence of 15 different strains of MRSA?

1. The bacteria mutated when it was exposed to methicillin, thus becoming resistant.

2. The bacteria underwent spontaneous mutation and some strains happened to be


resistant to methicillin.

3. The bacteria may have been exposed to a virus that carried a methicillin-resistant
gene and underwent specialised transduction.

4. The antibiotic caused the bacteria to produce methicillin-resistant proteins.

A 1 and 2 only

B 2 and 3 only

C 1, 2 and 4 only

D 2, 3 and 4 only

9648/ 01 / Preliminary examination / YJC 2013


1734
12

17

With reference to the lambda phage shown in the diagram above, which of the following
statements are false?
1. The genetic material of the virus can be either DNA or RNA.
2. The adsorption of viral particles to host cell is a chemical interaction in which
strong covalent bonds are formed between the tail fibres of the virus and
complementary receptor sites on the host cell wall.
3. Once the viral genetic material has reached the cytoplasm of the host cell, viral
proteins will be degraded.
4. The host genes can be transferred by the virus to another host cell through the
process of generalized transduction.
5. Mature infective virions have spontaneously assembled and are ready to leave
host cell to infect other cells.

6. The virus is able to multiply via both the lytic and lysogenic cycles.

A 1 and 5 only

B 2 and 4 only

C 1, 2 and 5 only

D 1, 5 and 6 only

9648/ 01 / Preliminary examination / YJC 2013


1735
13

18 Ribonuclease is an enzyme that digests RNA. The first five amino acids of the functioning
molecule of ribonuclease are:
lys glu thr ala ala .............

The mRNA of the gene coding for ribonuclease, for the first 15 nucleotides, has the following
sequence.
AUGAAGGAAACUGCU

A genetic code, showing mRNA codons, is shown below.

Which event occurs to explain the information given above?

A The first amino acid on the polypeptide chain is removed in post-translational


modification.

B The first codon is removed from the mRNA transcript in post-transcriptional


modification.

C The mRNA binds to the rRNA in the second codon position.

D There is no tRNA with an anticodon complementary to the first codon.

9648/ 01 / Preliminary examination / YJC 2013


1736
14

19 Researchers studied the regulation of a hormone-responsive gene isolated 750 base pairs of
DNA immediately preceding the start site of transcription (+1). They demonstrated that if
these sequences are cloned upstream of the bacterial chloramphenicol acetyltransferase
(CAT) gene and the DNA then introduced into mammalian cells, CAT enzyme activity
increases in response to hormone treatment.

To define the sequences involved in the regulation of this gene, they made a series of
deletions containing various lengths of the 5 regulatory sequences. They cloned these
truncated DNA fragments upstream of the CAT gene as shown in the figure below,
introduced the constructs into mammalian cells, and assayed for CAT enzyme activity in the
absence (-) and presence (+) of hormone. The figure below gives the results of a
representative experiment.

5 regulatory sequences Units of CAT


activity

Hormone: - +
+1

-742
CAT gene 21 212
-638
CAT gene 27 228
-424
CAT gene 5 54
-315
CAT gene 6 69
-116
CAT gene 5 7
-27
CAT gene 0.2 0.2

CAT gene 0.2 0.1

Assuming that there is a single responsive regulatory element in the gene, that element is
located between

A -742 and -638

B -638 and -424

C -424 and -315

D -315 and -116

9648/ 01 / Preliminary examination / YJC 2013


1737
15

20 Downs syndrome can be caused by a trisomy of chromosome 21, but can also result from
translocation of chromosome 21 onto chromosome 13, forming a single chromosome 13-21.
The diagram shows chromosomes 13 and 21 in the nucleus of a diploid (2n) testis cell from a
phenotypically normal male carrier of a 13-21 translocation. This cell has a chromosome
number of 45.

Which is not a likely outcome of fertilisation of normal oocytes by sperm from this male?

chromosomes in sperm embryo


A 13 and 21 2n = 46; normal phenotype

B 13-21 2n = 45; normal phenotype

C 13-21 and 21 2n = 46; Downs syndrome

D 13-21 and 21 2n = 47; Downs syndrome

21 The diagram shows a family tree.

What is the probability that Individual I will be a girl with blood group B and Rh positive
blood?

A 1 in 16

B 1 in 8

C 1 in 4

D 1 in 2

9648/ 01 / Preliminary examination / YJC 2013


1738
16

22 In a species of plant, the allele for hairy stems is dominant to the allele for smooth stems. The
allele for purple stems is dominant to the allele for green stems. A plant was self-fertilised and
the offspring appeared in the ratio 9 hairy, purple stem: 3 hairy, green stem: 3 smooth, purple
stem: 1 smooth, green stem.

The offspring with smooth, green stems were crossed to all the other offspring.

What proportion of these crosses would be expected to produce offspring in the ratio 1 hairy,
purple stem: 1 hairy, green stem: 1 smooth, purple stem: 1 smooth, green stem?

A 4/15

B 5/15

C 6/15

D 9/15

23 Albino horses have recessive alleles which cannot be transcribed to produce pigmentation.
Geneticists studying horses have found a different allele of the same gene which predisposes
carriers to premature aging of the hair follicles. This allele is dominant, so, even when born
with pigmented hair, the coats of horses with this allele will gradually turn white over the first 6
to 8 years of life.

Which white female horse would be best to use in crosses to determine the genotype of a
ten-year-old, white male horse?

A an unrelated, ten-year-old female

B an unrelated, three-year-old-female

C a sister of the male

D the males mother

9648/ 01 / Preliminary examination / YJC 2013


1739
17

24 A plant of genotype SSrr with hairy leaves and white flowers was crossed with a plant of
genotype ssRR with smooth leaves and red flowers. The F1 plants were allowed to self-
fertilise. A 2 test was carried out on the results obtained for the F2 generation.

Part of the table of values for 2 is shown.

The value of 2 in this investigation was 9.75.

Which combination correctly describes the results of the 2 test?

Difference between
Probability expected and observed Null hypothesis
results
A between 0.001 and 0.01 Significant accept
B between 0.001 and 0.01 Not significant reject
C between 0.01 to 0.05 Significant reject
D Between 0.01 to 0.05 Not significant reject

25 Two pairs of allele govern the colour of onion bulbs. A pure-breed red strain crossed with a
pure-breed white strain produces all red coloured bulbs in F1 progeny. Self-crossing of F1
generation produced 47 white, 38 yellow and 109 red bulbs. What does this experiment
demonstrate?

A recessive epistasis

B dominant epistasis

C codominance

D there is not enough evidence to come to a conclusion

9648/ 01 / Preliminary examination / YJC 2013


1740
18

26 De Vivo disease is an autosomal dominant developmental disorder associated with deficiency


in a type of glucose transporter. In an investigation to determine the chromosomal locus of the
disease, linkage analysis of the glucose transporter gene was carried out on members of one
family. Three separate VNTR loci, P, Q and R, located on separate chromosomes, were used.

The results of the linkage analysis are shown in the figure below. Different alleles present at
various individuals VNTR loci are denoted using different numbers.

Alleles present at various individuals VNTR loci


I-1 I-2 II-1 II-2 II-3 II-4 II-5 II-6 II-7
VNTR locus P 2,6 1,6 1,2 2,6 1,6 6,6 1,2 6,6 1,6
VNTR locus Q 1,5 2,7 7,5 2,5 7,1 2,1 7,5 2,1 7,1
VNTR locus R 4,8 5,8 5,8 8,8 5,4 8,4 5,8 8,4 5,4

Which of the following can be concluded from the figure?

A Individual I-1 is unaffected as he does not possess two copies of allele 5 for VNTR
locus Q.

B Individual II-2 is affected as he inherited two copies of allele 8 for VNTR locus R.

C Individual II-5 is unaffected as she did not inherit allele 2 for VNTR locus Q.

D Individual II-6 is affected as he inherited two copies of allele 6 for VNTR locus P.

9648/ 01 / Preliminary examination / YJC 2013


1741
19

27 Two pure-bred lines of two varieties of maize were crossed. The length of each of the cobs by
the two parental varieties and their offspring were measured. The number of cobs in each
length category was counted.

The graph shows the results.

What is the cause of the phenotypic variation in cob length within the two parental varieties
and their offspring?

A additive effect of different genes

B various environmental factors

C linkage and crossing-over at meiosis

D segregation and independent assortment of alleles

9648/ 01 / Preliminary examination / YJC 2013


1742
20

28 The graph shows the absorption spectra of some pigments found in chloroplasts.

Which statements are correct?

1. Having several pigments, rather than one, increases the efficiency of photosynthesis.

2. Most leaves are green as chlorophyll absorbs light in the blue and red regions of the
spectrum.

3. Photosynthesis will be fastest in light at the red end of the spectrum, as red light has
higher energy than blue light.

4. Prior to leaf fall, chlorophyll is broken down, leaving carotenoids which makes leaves
look yellow or red.

A 1 and 2 only

B 1 and 3 only

C 1, 2 and 4 only

D 2 and 4 only

9648/ 01 / Preliminary examination / YJC 2013


1743
21

29 The diagram shows the electron transport chain embedded in thylakoid membranes of
chloroplasts.

DCMU and paraquat are inhibitors of electron transport chain at the positions indicated in the
diagram. These two inhibitors are added separately to isolated chloroplasts.

Which of the following correctly represents the results in the presence of DCMU and
paraquat?

DCMU only Paraquat only


NADPH Oxygen ATP NADPH oxygen ATP
A Present Absent Present Absent Absent Present

B Absent Present Absent Absent Present Present

C Absent Present Present Present Present Absent

D Absent Absent Absent Absent Absent Present

9648/ 01 / Preliminary examination / YJC 2013


1744
22

30 The diagram below shows the Krebs Cycle.

At which stages are NAD reduced?

A 1, 4, 5 and 7

B 1, 4, 5 and 9

C 2, 4, 5 and 7

D 4, 5, 7 and 9

31 A person has malfunctioning -cells in the islets of Langerhans. What will be the levels of
glucose, insulin and glucagon in the blood after that person has eaten a meal rich in
carbohydrates?

glucose insulin glucagon


A low high low
B high low low
C low high high
D high low high

9648/ 01 / Preliminary examination / YJC 2013


1745
23

32 The ability of organisms to respond rapidly to stimuli is limited by the speed of the impulses in
their neurones.

The axons of invertebrate neurones lack a myelin sheath. The axons of most vertebrate
neurons are myelinated.

The graphs show the speed of impulses in these two types of axon.

Which statements about these data are correct?

1. The action potential in myelinated axons is greater than the action potential in non-
myelinated axons.

2. The speed of impulses is changed by the diameter of the axon.

3. Increasing the diameter of a myelinated axon causes a greater change to the


conduction speed than increasing the diameter of a non-myelinated axon.

4. The presence of myelin increases the speed at which impulses are conducted.

A 1, 2 and 3 only

B 1, 2 and 4 only

C 2, 3 and 4 only

D 3 and 4 only

9648/ 01 / Preliminary examination / YJC 2013


1746
24

33 The diagram shows part of a phylogenetic classification of four species into taxonomic groups.

Which row correctly describes the order M in comparison with genera I and J?

number of similarities time since members of


relative size of
between members of the taxonomic groups shared a
taxonomic groups
taxonomic groups common ancestor
fewer similarities between members of M shared a
A M is larger than I or J members of M than common ancestor a longer time
between members of I or J ago than those of I or J
more similarities between members of M shared a
B M is larger than I or J members of M than common ancestor a shorter
between members of I or J time ago than those of I or J
fewer similarities between members of M shared a
C M is smaller than I or J members of M than common ancestor a shorter
between members of I or J time ago than those of I or J
more similarities between members of M shared a
D M is smaller than I or J members of M than common ancestor a longer time
between members of I or J ago than those of I or J

9648/ 01 / Preliminary examination / YJC 2013


1747
25

34 The herbaceous plant Achillea lanulosa is widespread in the Northern Hemisphere.


Phenotypic variation in the species has been extensively studied along an altitudinal gradient
from sea level to over 3,000 metres. In California, there is conspicuous variation in height;
alpine plants are only several centimetres tall while those in the San Joaquin Valley may
reach a height of 1.8 metres. Adaptation to different local environments also results in
variation in physiological processes such as photosynthetic rate, resistance to cold, and timing
of dormancy.

Which of the following methods would be best to determine whether the phenotypic variation
in Achillea lanulosa is due to genotypic variation?

A Assessing the amount of phenotypic variation within populations along the entire
altitudinal range

B Determining whether phenotypic differences are maintained when plants from different
altitudes are grown under the same environmental conditions

C Determining whether viable hybrids between phenotypically different populations can


be produced

D Determining whether hybrids between phenotypically different populations grow at


altitudes intermediate between the parent populations

35 The following statements relate to molecular phylogenetics.

1. Lines of descent from a common ancestor to present-day organisms have


undergone similar, fixed rates of DNA mutation.

2. Organisms with similar base sequences in their DNA are closely related to
each other.

3. The number of differences in the base sequence of DNA of different organisms


can be used to construct evolutionary trees.

4. The proportional rate of fixation of mutations in one gene relative to the rate of
fixation of mutations in other genes stays the same in any given line of
descent.

Which statements, when taken together, suggest the existence of a molecular clock
that enables scientists to estimate the time at which one species might have diverged
from another?

A 1 and 2 B 1 and 4 C 2 and 3 D 3 and 4

9648/ 01 / Preliminary examination / YJC 2013


1748
26

36 The table shows the number of estimated nucleotide substitutions that have occurred since
the divergence of seven species a to g.

b c d e f g
a 39 72 128 126 159 269
b 81 130 128 158 268
c 129 127 157 267
d 56 154 271
e 151 268
f 273

Which of the following phylogenetic trees best shows the relationship among these seven
species?

37 In obtaining the gene that codes for human actin, reverse transcriptase was used.

Why is reverse transcriptase not used for obtaining the gene that produces a human sodium
ion channel protein?

A Reverse transcriptase is a viral enzyme, and in the current environment of suspicion of


viruses, it is not possible to use such enzymes.

B The amino acid sequence for this protein is not known, so this is a gene that has not
yet been located by the human genome project.

C The antibiotic resistance genes transferred by reverse transcriptase with the insulin
gene may be transferred to pathogens.

D The sodium ion channel protein gene is expressed at low levels in human cells, so the
mRNA produced is swamped by other mRNA.

9648/ 01 / Preliminary examination / YJC 2013


1749
27

38 A gene sequence to be cloned is shown below. Nx denotes an unspecified sequence.

5 AATCGCCGAGCTCGGTTACG(Nx)TAGGCATCTCGAGCCTAGG 3

Which pair of primers will be effective for PCR amplification of the gene?

P: 5 AATCGCC 3

Q: 5 ATCCGTA 3

R: 5 TAGGCAT 3

S: 5 TTAGCGG 3

T: 5 CCTAGGC 3

A P and Q

B P and R

C P and T

D Q and S

9648/ 01 / Preliminary examination / YJC 2013


1750
28

39 pGEM-T is a 7.25kb plasmid cloning vector often used in creating recombinant DNA. It
contains two selectable marker genes, TetR and lacZ. The figure below shows a portion of the
multiple cloning site (MCS) of the cloning vector. It is located within the lacZ gene.

Using SacII as the restriction enzyme, a 2.31kb insert (gene of interest) is cloned into the
MCS shown above, and the recombinant plasmid is transformed into E. coli cells. The
transformants are then plated on medium containing tetracycline and X-Gal. The following
result is obtained.

Colonies X and Y are isolated and the plasmid DNA from the colonies is extracted. The
extracted plasmids are then digested with SacII. Which of the following correctly shows the
fragments obtained?

Colony X Colony Y
A A 9.56kb fragment A 7.25kb fragment

B A 7.25kb fragment A 9.56kb fragment

C 7.21kb and 0.34kb fragments 7.21kb and 2.31kb fragments

D 7.21kb and 0.34kb fragments 7.25kb and 0.34kb fragments

9648/ 01 / Preliminary examination / YJC 2013


1751
29

40 When a fertilized egg begins development, the outcome of the first three cell cycles is shown
as follows:

The localization of a specific protein during each of the above four early developmental
stages is examined. This protein is required for the formation of ectoderm. The location of the
protein is indicated by grey shading.

At the four-cell stage, are the four cells equivalent in terms of the different fates their daughter
cells can take on, and why?

Yes or No to the
four-cell
Justification
equivalence in
terms of their fates
The protein required for ectoderm formation is found only in the
upper two cells; therefore, only these two cells can become
A No ectoderm. The lower two cells can continue to differentiate to
become mesoderm or endoderm at this point, but cannot become
ectoderm.
The protein required for ectoderm formation is found only in the
upper two cells. These proteins can be transported to the other
B No
two lower cells such that the mesoderm or endoderm can be
formed.
The protein required for ectoderm is needed for differentiation of
C Yes the other germ layers since the ectodermal cells still remain
totipotent and can take on any developmental pathway.
The protein required for ectoderm is needed only by the upper
D Yes two cells since the lower two cells can differentiate to become
mesoderm or endoderm at this point.

9648/ 01 / Preliminary examination / YJC 2013


1752
30

2013 H2 Biology Preliminary Examination Paper 1 ANSWER KEY

1 B 2 A 3 C 4 C 5 D

6 B 7 B 8 D 9 A 10 D

11 A 12 B 13 D 14 D 15 A

16 B 17 C 18 A 19 D 20 D

21 B 22 A 23 A 24 C 25 A

26 C 27 B 28 C 29 D 30 B

31 B 32 C 33 A 34 A 35 B

36 D 37 D 38 C 39 A 40 A

9648/ 01 / Preliminary examination / YJC 2013


1753
2

Section A

Answer all the questions in this section.

1 Enzyme X digests a sugar. Fig. 1.1 shows the rate of reaction of enzyme X and a mixture
of enzyme X and substance Y on the sugar. Both reactions were carried out in a conical
flask.

Fig. 1.1

(a) With reference to Fig. 1.1, describe and explain the effect of substance Y on
the enzymatic reaction. [3]

9648/ 02 / Preliminary Examination / YJC 2013


1754
3

(b) Substance Y was found in one of the products of the reaction catalysed by
enzyme X after the substrate concentration, A, in Fig.1.1.

Predict how the trend of the graph for enzyme X only would continue after
sugar concentrations greater than A. Sketch the part of the graph in Fig. 1.1. [1]

Enzyme X may be immobilised in alginate beads. Fig. 1.2 shows a comparison between
the activity of free enzyme X and immobilised enzyme X over a range of temperatures.
Equal concentrations of free enzyme X and immobilised enzyme X were used.

Fig. 1.2

9648/ 02 / Preliminary Examination / YJC 2013


1755
4

(c) With reference to the Fig. 1.2,

(i) describe and explain the effect of temperature on the activity of free
enzyme X. [3]

(ii) describe and explain the differences between the activity of free
enzyme X and immobilised enzyme X up to 40 C. [2]

9648/ 02 / Preliminary Examination / YJC 2013


1756
5

The oil stored in the seeds of the castor oil plant Ricinus communis has been used in the
production of pharmaceutical products such as zinc and castor oil cream, and as a
lubricant. However, Ricinus seeds also produce a poisonous protein ricin, which is
classified as an agent of bioterrorism.

Fig. 1.3 shows the structure of ricin which consists of two polypeptides the A chain of
ricin (RTA, ricin toxin A) joined to a B chain (RTB, ricin toxin B).

RTA

RTB

Fig. 1.3

(d) With reference to Fig. 1.3,

(i) identify the level of organisation of the structure of ricin. [1]

9648/ 02 / Preliminary Examination / YJC 2013


1757
6

(ii) describe how the conformation of RTA (boxed section) is obtained


from a linear polypeptide chain. [3]

(e) The RTB component of ricin has two binding sites specific for galactose. This
enables RTB to bind to cell surface components that contain galactose. This
is the first step of cellular intoxication. Identify a structure found on the
mammalian cell surface that makes it a good target for ricin. [1]

(f) The RTA component of ricin is an enzyme that damages an essential


component of the protein synthesis machinery the ribosome. RTA catalyses
the removal of one particular nitrogenous base (adenine) in a crucially
important section of ribosomal RNA of the large subunit of eukaryotic
ribosomes. This results in eventual cell death.

The two polypeptides RTA and RTB are covalently linked by a disulfide
bond between two cysteine residues. Reduction of ricin into its subunits, RTA
and RTB, is catalysed by protein disulfide isomerase (PDI), an enzyme that
resides in the lumen of the endoplasmic reticulum. PDI breaks the interchain
disulfide bond.

Fig. 1.4 illustrates the reduction process catalysed by PDI.

Reduction of interchain disulfide bond

Fig. 1.4
9648/ 02 / Preliminary Examination / YJC 2013


1758
7

(i) Explain why the reduction process shown in Fig. 1.4 is necessary
before RTA can exhibit its catalytic activity. [2]

(ii) Suggest one way in which plant ribosomes in the seed cells are
protected from the effect of ricin. [1]

[Total: 17]

9648/ 02 / Preliminary Examination / YJC 2013


1759
8

2 (a) Fig. 2 is an electron micrograph of an organelle found in eukaryotic cells.

outer membrane

inner membrane
cristae
matrix

intermembrane space

Fig. 2

State two similarities between the organelle above and a typical prokaryotic
cell like E. coli. [2]

9648/ 02 / Preliminary Examination / YJC 2013


1760
9

-
(b) (i) Mutants that are lac Y , meaning that the lac Y gene is inactivated,
retain the capacity to synthesise -galactosidase. However, even
though the lac I gene encoding for the repressor protein is still intact,
-galactosidase can no longer be induced by adding lactose to the
medium. Explain why this is so. [4]

(ii) Positive control of the lac operon results from breakdown of glucose,
which alters the levels of cyclic adenosine monophosphate (cAMP) in
the cell. cAMP binds to a catabolite activator protein (CAP), forming a
CAP-cAMP complex which increases the affinity of RNA polymerase
for the lac promoter. However, this mechanism will not cause
upregulation (genetic regulation that leads to an increase in gene
expression) of the lac operon in the absence of lactose. Explain why
this is so. [2]

(iii) Suggest a reason how a mutation in the operon can result in a


repressor that continually binds to the operator. [1]

9648/ 02 / Preliminary Examination / YJC 2013


1761
10

(c) Why is any bacterial gene capable of being transferred during generalised
transduction? [3]

[Total: 12]

9648/ 02 / Preliminary Examination / YJC 2013


1762
11

3 The human papillomavirus (HPV) is a small non-enveloped virus that has a double-
stranded circular DNA genome. Some HPVs have been well established as the main risk
factor of cervical cancers.

The HPV genome is replicated in the nucleus of infected cells in the form of a circular
DNA (episome). The HPV E2 gene encodes a transcription factor E2 that suppresses
transcription of HPV E6 and E7, which are viral oncogenes. In certain instances during
the replication cycle of HPV, the viral DNA can be linearised and integrated into
chromosomal DNA of host cells. A breakpoint that disrupts the HPV E2 gene will prevent
the synthesis of E2 proteins that normally regulate the transcription of E6 and E7.

Fig. 3.1 illustrates the integration of HPV DNA into the host chromosome.

Fig. 3.1

Part of the HPV genome consisting of the long control region (LCR), E6 gene and E7
gene is integrated into the host DNA (Fig 3.2). The LCR contains a variety of regulatory
sequences that will regulate viral replication and gene expression. The HPV E6 and E7
genes are consistently expressed, whereas the remaining HPV genes are often deleted
or not transcribed after integration.

Fig. 3.2
9648/ 02 / Preliminary Examination / YJC 2013


1763
12

(a) With reference to the information provided and Fig. 3.2, suggest how a dual-
cistronic E6-E7 mRNA transcript is produced. [2]

An increase in the synthesis of E6 and E7 oncoproteins is associated with the risk of


development of carcinoma. Fig. 3.3 illustrates the HPV inactivation of two cellular tumour
suppressor proteins, p53 and pRb.

Fig. 3.3

(b) With reference to Fig. 3.3, describe how the absence of E2 and hence an
increase in E6 and E7 protein synthesis, can result in the degradation of p53
and pRb. [2]

9648/ 02 / Preliminary Examination / YJC 2013


1764
13

(c) Explain why the degradation of p53 may lead to tumour formation. [3]

(d) The ends of eukaryotic chromosomes are protected by identical repeating


lengths of DNA called telomeres, which prevent chromosomes from
unravelling during cell division. Each time a cell divides, the length of the
telomeres shortens and the ability of the cell to divide decreases.

Another function of the HPV E6 proteins is their ability to activate the


expression of the catalytic subunit of telomerase (hTERT). Explain the
relationship between high levels of telomerase expression and cancer. [4]

[Total: 11]

9648/ 02 / Preliminary Examination / YJC 2013


1765
14

4 Wing colour in a butterfly species is determined by two alleles, one coding for red
pigment and another coding for yellow pigment.

When true breeding yellow butterflies are mated with true breeding red butterflies, the
wings of male progeny are always orange. Female progeny always have the same wing
colour as their fathers.

(a) Using suitable symbols, draw two different genetic diagrams in the space
below to fully explain the results obtained in the progeny. [5]

9648/ 02 / Preliminary Examination / YJC 2013


1766
15

(b) Explain the mode of inheritance of wing colour in the butterfly species. [3]

(c) Observed results of the above genetic cross differ from the expected results.

Suggest two reasons why such a discrepancy occurs, referring only to events
that occur after meiosis. [2]

[Total: 10]

9648/ 02 / Preliminary Examination / YJC 2013


1767
16

5 Fig. 5 shows the steps involved during glycolysis.

Fig. 5
9648/ 02 / Preliminary Examination / YJC 2013


1768
17

(a) With reference to Fig. 5,

(i) explain the role of hexokinase in the first step of glycolysis. [2]

(ii) explain the role of glycolysis in aerobic respiration. [2]

(iii) describe how a named organism obtains energy for survival in the
absence of oxygen from glycolysis. [2]

9648/ 02 / Preliminary Examination / YJC 2013


1769
18

(b) Glyceraldehyde-3-phosphate is an intermediate in glycolysis which is also


found in the Calvin cycle in plants.

Describe how glyceraldehyde-3-phosphate is formed in the Calvin cycle in


plants. [4]

[Total: 10]

9648/ 02 / Preliminary Examination / YJC 2013


1770
19

6 Cry toxins are produced by the soil bacterium Bacillus thuringiensis (Bt). By genetic
engineering, Cry toxin gene can be incorporated into commercially important crops to
specifically kill insect pests without affecting other organisms.

Cry toxins are believed to cause death in target insects by oligomerising into pore
complexes in cell surface membrane of midgut epithelial cells.

(a) Explain how formation of pores in the cell surface membrane of midgut
epithelial cells causes death in target insects. [1]

Another mechanism of action of Cry toxin in midgut epithelial cells of target insects has
been proposed. Fig. 6 shows the proposed magnesium ion-dependent signalling
cascade. Single-pass membrane receptors that cross the membrane only once, such as
aminopeptidase N (APN) receptors and cadherin-like receptors, are involved. Cry toxins
eventually lead to destabilisation of the cytoskeleton and ion channels in the cell
membrane causing cell death.

Fig. 6

9648/ 02 / Preliminary Examination / YJC 2013


1771
20

(b) (i) With reference to Fig. 6, describe the proposed cell signalling cascade
of Cry toxin. [4]

(ii) State one advantage and one disadvantage of a signalling cascade. [2]

9648/ 02 / Preliminary Examination / YJC 2013


1772
21

(iii) NF449 is an antagonist of G protein that prevents binding of GTP.


Suggest how NF449 could affect the signalling pathway of Cry toxins. [2]

(c) Suggest why Cry toxins only kill target insects and not other organisms. [1]

[Total: 10]

9648/ 02 / Preliminary Examination / YJC 2013


1773
22

7 Antifreezes are specialised proteins that prevent the organisms that have them from
freezing in the polar oceans of the Arctic and the Antarctic. One of these proteins is the
antifreeze glycoprotein (AFGP), which is produced in some fish species. The AFGP
proteins protect the fish by lowering their bodies freezing point temperature so that it is
lower than that of the surrounding seawater. This prevents ice crystals from forming
within the fishes body tissues when fish in icy waters drink and feed, which would be
lethal.

Fig. 7 shows three species of fish with AFGPs.

Arctic cod (Suborder: Gadoilei).

Lives at the waters surface and at depths


below 900 metres in the Arctic.

Bald notothen (Suborder: Notothenioidei).

Lives on the underside of the surface ice layer in


the Antarctic.

Threadfin pithead (Suborder: Notothenioidei).

Lives in the deep-sea habitats of the Antarctic


(600 metres).

Fig. 7
9648/ 02 / Preliminary Examination / YJC 2013


1774
23

A circumpolar current contributes to the thermal isolation and low temperatures of


present-day Antarctica. This current formed about 30 million years ago when Antarctica
separated from South America. Up until this time, the waters around Antarctica had been
relatively warm and a wide diversity of fish species flourished.

With the separation of Antarctica from South America, water temperatures began to drop,
reaching freezing levels about 1015 million years ago. At this time, it is thought that the
modern day notothenioids were represented by a single species in which the gene for
AFGP may have arisen.

Today the seawater temperature around Antarctica seldom varies more than a few
degrees above freezing. These waters are now dominated by the notothenioids, of which
there are more than 100 species.

AFGP is also found in a species of arctic fish, the Arctic cod. This species is unrelated to
the notothenioids, and its AFGP probably evolved about 45 million years ago during the
glaciation of the Arctic seas. The Arctic cod and the notothenioids separated long before
the antifreeze glycoproteins developed.

Researchers have found that the gene for AFGP in the Antarctic notothenioids is quite
different in its origin and in its location within the genome from the gene expressing
AFGP in the Arctic cod. However, their AFGPs are virtually identical and they both
contain the same repeating sequence of three amino acids (threonine-alanine-alanine).

(a) Explain how natural selection could have brought about the evolution of
antifreeze glycoprotein (AFGP) in the notothenioids in the seawater around
Antarctica. [5]

9648/ 02 / Preliminary Examination / YJC 2013


1775
24

(b) (i) What is the term used to describe the evolutionary pattern observed in
the Antarctic notothenioids and the Arctic cod? [1]

(ii) Explain how the evolutionary pattern you have identified in (b)(i)
resulted in the evolution of virtually identical AFGPs in the Antarctic
notothenioids and the Arctic cod. [4]

[Total: 10]

9648/ 02 / Preliminary Examination / YJC 2013


1776
25

Section B

Answer one question.

Write your answers on the separate answer paper provided.


Your answers should be illustrated by large, clearly labelled diagrams, where appropriate.
Your answers must be in continuous prose, where applicable.
Your answers must be set out in sections (a), (b) etc., as indicated in the question.

8 (a) Outline the roles of phospholipids, cholesterol and glycolipids in the cell [6]
surface membrane.

(b) Explain how the molecular structure of haemoglobin is related to its function. [7]

(c) Describe the structural differences between collagen and cellulose. [7]

9 (a) Outline the ways in which mutations can occur. [6]

(b) Explain how gene mutation leads to cystic fibrosis. [7]

(c) Describe the role of mitosis in maintaining genetic stability. [7]

9648/ 02 / Preliminary Examination / YJC 2013


1777
2

Section A

Answer all the questions in this section.

1 Enzyme X digests a sugar. Fig. 1.1 shows the rate of reaction of enzyme X and a mixture
of enzyme X and substance Y on the sugar. Both reactions were carried out in a conical
flask.

Fig. 1.1

(a) With reference to Fig. 1.1, describe and explain the effect of substance Y on
the enzymatic reaction. [3]

Even at very high substrate concentration, the maximum rate of reaction in


the presence of substance Y is lower than that in absence of substance Y/
Increasing substrate concentration cannot overcome inhibition by
substance Y/ Maximum rate of reaction Vmax is lower and affinity for the
sugar is not affected by substance Y/ Km remains unchanged;
Substance Y is a non-competitive inhibitor which has no close structural
resemblance to the substrate;
binds irreversibly to a region other than active site of enzyme X;
[1mark each]

9648/ 02 / Preliminary Examination / YJC 2013


1778
3

(b) Substance Y was found in one of the products of the reaction catalysed by
enzyme X after the substrate concentration, A, in Fig.1.1.

Predict how the trend of the graph for enzyme X only would continue after
sugar concentrations greater than A. Sketch the part of the graph in Fig. 1.1. [1]

Decreasing trend due to end-product inhibition (refer to Fig. 1).

Enzyme X may be immobilised in alginate beads. Fig. 1.2 shows a comparison between
the activity of free enzyme X and immobilised enzyme X over a range of temperatures.
Equal concentrations of free enzyme X and immobilised enzyme X were used.

Fig. 1.2

9648/ 02 / Preliminary Examination / YJC 2013


1779
4

(c) With reference to the Fig. 1.2,

(i) describe and explain the effect of temperature on the activity of free
enzyme X. [3]

As temperature increases from 20 to 35 C, the rate of reaction of free


enzyme X increases from 0.60 to 0.95 A.U. due to the increased kinetic
energy of the sugar substrate and enzyme molecules;
These molecules move faster colliding with one another in correct
orientation to form more enzyme-substrate complexes;
The increased rate of reaction increases until it reaches its maximum rate
of 0.95 A.U. at the optimum temperature of 35 C when most enzyme-
substrate complexes formed;
Beyond the optimum temperature of 35 C to 60 C, rate of reaction
decreases from 0.95 to 0.1 A.U. as intramolecular bonds (reference to
hydrogen bonds, hydrophobic interactions) are broken and three-
dimensional conformation of active sites in enzymes is altered such that
substrate can no longer fit
[1 mark each, max 3 marks]

(ii) describe and explain the differences between the activity of free
enzyme X and immobilised enzyme X up to 40 C. [2]

Description
From 20 to 40 C, activity of immobilised enzyme is lower than free
enzymes (reference to any suitable data range) and the maximum activity
of immobilised enzyme (0.80 A.U at 35 C) is lower than that of free
enzyme (0.95 A.U. at 40C/ between 40 to 45 C);
[1 mark each, max 1 mark]
Explanation
Enzymes immobilised in alginate beads diffuse out from beads less easily
to come into contact with sugar substrate/ less able to move and collide
successfully with substrates leading to fewer enzyme-substrate complexes
formed;
Products formed within the beads diffuse out less easily and accumulate
leading to end-product inhibition.
[1 mark each, max 1 mark]

9648/ 02 / Preliminary Examination / YJC 2013


1780
5

The oil stored in the seeds of the castor oil plant Ricinus communis has been used in the
production of pharmaceutical products such as zinc and castor oil cream, and as a
lubricant. However, Ricinus seeds also produce a poisonous protein ricin, which is
classified as an agent of bioterrorism.

Fig. 1.3 shows the structure of ricin which consists of two polypeptides the A chain of
ricin (RTA, ricin toxin A) joined to a B chain (RTB, ricin toxin B).

RTA

RTB

Fig. 1.3

(d) With reference to Fig. 1.3,

(i) identify the level of organisation of the structure of ricin. [1]


Quaternary structure

9648/ 02 / Preliminary Examination / YJC 2013


1781
6

(ii) describe how the conformation of RTA (boxed section) is obtained


from a linear polypeptide chain. [3]

The linear polypeptide chain undergo folding to form the secondary


structure;
which is made up of the -helix and the -pleated sheet;
The secondary structure of RTA is maintained by hydrogen bonds formed
between the CO and NH groups of the main amino acid chain;
Note: R groups are not involved in the formation of hydrogen bonds in the
secondary structure of a polypeptide;
The secondary structure undergo further folding and bendings to form the
tertiary structure / globular structure of RTA,
The tertiary structure of RTA is maintained by 4 types of bonds (disulfide
bond, hydrogen bond, ionic bond and hydrophobic interaction) that are
formed between R groups of RTA.

(e) The RTB component of ricin has two binding sites specific for galactose. This
enables RTB to bind to cell surface components that contain galactose. This
is the first step of cellular intoxication. Identify a structure found on the
mammalian cell surface that makes it a good target for ricin. [1]
Glycoprotein

(f) The RTA component of ricin is an enzyme that damages an essential


component of the protein synthesis machinery the ribosome. RTA catalyses
the removal of one particular nitrogenous base (adenine) in a crucially
important section of ribosomal RNA of the large subunit of eukaryotic
ribosomes. This results in eventual cell death.

The two polypeptides RTA and RTB are covalently linked by a disulfide
bond between two cysteine residues. Reduction of ricin into its subunits, RTA
and RTB, is catalysed by protein disulfide isomerase (PDI), an enzyme that
resides in the lumen of the endoplasmic reticulum. PDI breaks the interchain
disulfide bond.

Fig. 1.4 illustrates the reduction process catalysed by PDI.

Reduction of interchain disulfide bond

Fig. 1.4
9648/ 02 / Preliminary Examination / YJC 2013


1782
7

(i) Explain why the reduction process shown in Fig. 1.4 is necessary
before RTA can exhibit its catalytic activity. [2]

In intact ricin, the RTB polypeptide hides the catalytic site of RTA;
Only after RTA is separated from RTB (after the reduction process)
does RTA exhibit its catalytic activity.

(ii) Suggest one way in which plant ribosomes in the seed cells are
protected from the effect of ricin. [1]

Either one:

Ricin is made as an inactive protein;

It is stored in the plant vacuoles, away from its substrates;

[Total: 17]

9648/ 02 / Preliminary Examination / YJC 2013


1783
8

2 (a) Fig. 2 is an electron micrograph of an organelle found in eukaryotic cells.

outer membrane

inner membrane
cristae
matrix

intermembrane space

Fig. 2

State two similarities between the organelle above and a typical prokaryotic
cell like E. coli. [2]

Both do not have a true nucleus


Both contain circular DNA
Both have 70S ribosomes
[Max 2 marks

9648/ 02 / Preliminary Examination / YJC 2013


1784
9

-
(b) (i) Mutants that are lac Y , meaning that the lac Y gene is inactivated,
retain the capacity to synthesise -galactosidase. However, even
though the lac I gene encoding for the repressor protein is still intact,
-galactosidase can no longer be induced by adding lactose to the
medium. Explain why this is so. [4]

The lac Y gene codes the enzyme permease which enables cells to take
up lactose efficiently. [1]

Mutants that are lac Y- do not have the enzyme permease cells cannot
take up lactose which will thus not be converted to allolactose; [1]
Which is required to bind to the allosteric site of the lac repressor protein
to inactivate it; [1]
[max 2m]
Hence the repressor protein continues to occupy the operator site on the
lac operon; [1]

RNA polymerase is blocked from accessing the promoter site, thus the
structural gene which encodes -galactosidase cannot be transcribed.
[1]

(ii) Positive control of the lac operon results from breakdown of glucose,
which alters the levels of cyclic adenosine monophosphate (cAMP) in
the cell. cAMP binds to a catabolite activator protein (CAP), forming a
CAP-cAMP complex which increases the affinity of RNA polymerase
for the lac promoter. However, this mechanism will not cause
upregulation (genetic regulation that leads to an increase in gene
expression) of the lac operon in the absence of lactose. Explain why
this is so. [2]

This is because in the absence of lactose, the repressor protein will be


bound to the operator; [1]

and will prevent the binding of RNA polymerase to the promoter.


Transcription cannot proceed even though glucose is being actively
metabolised. [1]

(iii) Suggest a reason how a mutation in the operon can result in a


repressor that continually binds to the operator. [1]

There is a mutation in the lac Z, lac Y or lac A gene leading to a


dysfunctional enzyme (mention the enzyme). Thus allolactose is not
formed and the repressor protein remains bound to the operator. (Any
one of the gene is accepted).
Reject: mutation to Lac I gene (lies outside the operon)
9648/ 02 / Preliminary Examination / YJC 2013


1785
10

(c) Why is any bacterial gene capable of being transferred during generalised
transduction? [3]

During generalised transduction, the phage infects the cell and the
bacterial chromosome is fragmented. [1]
While viral particles are being assembled, any one of the bacterial
chromosome gene fragments can possibly be incorporated within the viral
protein coat. [1]
This is because it is the amount of DNA and not the information content of
the DNA that governs viral particle formation. [1]
Max 2 marks
When the new phage carrying the bacterial gene infects another cell, it
brings with it the new bacterial gene. [1]

[Total: 12]

9648/ 02 / Preliminary Examination / YJC 2013


1786
11

3 The human papillomavirus (HPV) is a small non-enveloped virus that has a double-
stranded circular DNA genome. Some HPVs have been well established as the main risk
factor of cervical cancers.

The HPV genome is replicated in the nucleus of infected cells in the form of a circular
DNA (episome). The HPV E2 gene encodes a transcription factor E2 that suppresses
transcription of HPV E6 and E7, which are viral oncogenes. In certain instances during
the replication cycle of HPV, the viral DNA can be linearised and integrated into
chromosomal DNA of host cells. A breakpoint that disrupts the HPV E2 gene will prevent
the synthesis of E2 proteins that normally regulate the transcription of E6 and E7.

Fig. 3.1 illustrates the integration of HPV DNA into the host chromosome.

Fig. 3.1

Part of the HPV genome consisting of the long control region (LCR), E6 gene and E7
gene is integrated into the host DNA (Fig 3.2). The LCR contains a variety of regulatory
sequences that will regulate viral replication and gene expression. The HPV E6 and E7
genes are consistently expressed, whereas the remaining HPV genes are often deleted
or not transcribed after integration.

Fig. 3.2
9648/ 02 / Preliminary Examination / YJC 2013


1787
12

(a) With reference to the information provided and Fig. 3.2, suggest how a dual-
cistronic E6-E7 mRNA transcript is produced. [2]

The LCR region contains a promoter sequence;


this allows general transcription factors and RNA polymerase to bind to
the promoter to form the transcription-initiation complex to initiate
transcription;
The E6-E7 genes are located adjacent to each other, and will thus be
under the control of the same promoter; this results in the formation of a
dual-cistronic mRNA following transcription;
[max 2]

An increase in the synthesis of E6 and E7 oncoproteins is associated with the risk of


development of carcinoma. Fig. 3.3 illustrates the HPV inactivation of two cellular tumour
suppressor proteins, p53 and pRb.

Fig. 3.3

(b) With reference to Fig. 3.3, describe how the absence of E2 and hence an
increase in E6 and E7 protein synthesis, can result in the degradation of p53
and pRb. [2]

E6 binds to p53 in the cytosol and results in the binding of E6AP ubiquitin
ligase, which ubiquitinates p53 which promotes its degradation in the
proteasome;
E7 binds to pRb in the cytosol and recruits cullin 2 ubiquitin ligase, which
ubiquitinates pRb which promotes its degradation in the proteasome;
9648/ 02 / Preliminary Examination / YJC 2013


1788
13

(c) Explain why the degradation of p53 may lead to tumour formation. [3]

p53 is a transcription factor and its production is induced by DNA damage


and abnormalities in chromosomal attachment;
the role of p53 includes the following (mention either 1):
o activates genes that promote DNA repair;
o activates genes that arrest cell division;
o activates genes that promote apoptosis;
degradation of p53 will allow a cell that has DNA damage (which in this
case is the result of the integration of the HPV genome into host
chromosome) to continue to divide and hence increases the risk of
tumour formation;

(d) The ends of eukaryotic chromosomes are protected by identical repeating


lengths of DNA called telomeres, which prevent chromosomes from
unravelling during cell division. Each time a cell divides, the length of the
telomeres shortens and the ability of the cell to divide decreases.

Another function of the HPV E6 proteins is their ability to activate the


expression of the catalytic subunit of telomerase (hTERT). Explain the
relationship between high levels of telomerase expression and cancer. [4]

Cells with high levels of telomerase, which adds specific DNA sequence
repeats / more specific repeating DNA units OR telomeric repeats;
to telomeres / DNA strands in telomere regions at the ends of
chromosomes;
thus elongating telomeres OR preventing telomeres from shortening OR
preventing shortening of chromosomes to critical length beyond which cell
division can no longer occur;

do not undergo replicative cell senescence;


as they proliferate, and their division is not checked by the Hayflick limit;
therefore they continue to divide uncontrollably;
and do not undergo apoptosis; contributing to development of cancer.

Accept: telomerase expression is a step in multi-step progression of cancer /


more mutations accumulated
Reject: higher chance of cancer

[1m each, max 4]

[Total: 11]

9648/ 02 / Preliminary Examination / YJC 2013


1789
14

4 Wing colour in a butterfly species is determined by two alleles, one coding for red
pigment and another coding for yellow pigment.

When true breeding yellow butterflies are mated with true breeding red butterflies, the
wings of male progeny are always orange. Female progeny always have the same wing
colour as their fathers.

(a) Using suitable symbols, draw two different genetic diagrams in the space
below to fully explain the results obtained in the progeny. [5]

ZY represents Z chromosome with the allele that codes for yellow pigment in wings
ZR represents Z chromosome with the allele that codes for red pigment in wings
W represents W chromosome
[1 mark for correct key]
Scenario 1
Parental phenotypes Female, yellow X Male, red
Y
Parental genotypes Z W X Z RZ R

Gametes ZY W ZR

1 mark for correct parental phenotype, parental genotype and circled


gametes
F1 genotypes Z YZ R Z RW
F1 phenotypes Male, orange Female, red
(same as father)
F1 phenotypic ratio 1 : 1
1 mark for correct F1 genotype, F1 phenotype and F1 phenotypic ratio

Scenario 2
Parental phenotypes Female, red X Male, yellow
Parental genotypes Z RW X Z YZ Y
Gametes
ZR W ZY

1 mark for correct parental phenotype, parental genotype and circled


gametes
F1 genotypes Z YZ R Z YW
F1 phenotypes Male, orange Female, yellow
(same as father)
F1 phenotypic ratio 1 : 1
1 mark for correct F1 genotype, F1 phenotype and F1 phenotypic ratio

9648/ 02 / Preliminary Examination / YJC 2013


1790
15

(b) Explain the mode of inheritance of wing colour in the butterfly species. [3]

In the butterfly species, the female is the heterogametic sex. Females with
the ZY allele (ZYW) would have yellow wings whereas those with the ZR
allele (ZRW) would have red wings. (Hint: Female progeny always have
the same wing colour as their fathers.)
The male is the homogametic sex. Males with genotype ZYZY would have
yellow wings and those with genotype ZRZR would have red wings.
The alleles ZY and ZR exhibit incomplete dominance relative to each other.
Hence, males with the genotype ZYZR have an intermediate phenotype of
orange wings.
[1 mark each]

(c) Observed results of the above genetic cross differ from the expected results.

Suggest two reasons why such a discrepancy occurs, referring only to events
that occur after meiosis. [2]

Sample size is too small


Variation is due to chance/ insignificant
Differential survival of gametes/ non-random mating;
Differential survival of fertilised zygotes/ some individuals die before being
sampled
[1 mark each, max 2 marks]

[Total: 10]

9648/ 02 / Preliminary Examination / YJC 2013


1791
16

5 Fig. 5 shows the steps involved during glycolysis.

Fig. 5
9648/ 02 / Preliminary Examination / YJC 2013


1792
17

(a) With reference to Fig. 5,

(i) explain the role of hexokinase in the first step of glycolysis. [2]

Hexokinase phosphorylates glucose by transferring a phosphate group


from ATP to glucose, converting it to glucose-6-phosphate;
This activates/ raises the energy level of glucose for ATP production in
subsequent steps of glycolysis / respiration.
[1 mark each]

(ii) explain the role of glycolysis in aerobic respiration. [2]

Glycolysis produces a net gain of two ATPs by substrate-level


phosphorylation and reduces NAD for oxidative phosphorylation in
aerobic respiration (to produce ATP);
Glycolysis also breaks down one glucose molecule into two molecules of
pyruvate that enter into the mitochondrion / link reaction, followed by
Krebs cycle where pyruvate is further broken down / completely oxidized
to produce more ATP.
[1 mark each]

(iii) describe how a named organism obtains energy for survival in the
absence of oxygen from glycolysis. [2]

Alcoholic fermentation in yeast: Pyruvate broken down to acetaldehyde/


ethanal then to ethanol;
OR
Lactate fermentation in any named mammal: Pyruvate broken down to
lactate;
NAD+ regenerated in process to allow glycolysis to continue.
[1 mark each]

9648/ 02 / Preliminary Examination / YJC 2013


1793
18

(b) Glyceraldehyde-3-phosphate is an intermediate in glycolysis which is also


found in the Calvin cycle in plants.

Describe how glyceraldehyde-3-phosphate is formed in the Calvin cycle in


plants. [4]

RuBP carboxylase (Rubisco) catalyses the fixation of a carbon dioxide


molecule with a molecule of ribulose bisphosphate (RuBP);
forming two molecules of glycerate-3-phosphate / 3-phosphoglycerate
(GP);
glycerate-3-phosphate/ 3-phosphoglycerate (GP) utilises ATP from light-
dependent reaction to give 1,3-biphosphoglyerate;
1,3-biphosphoglyerate is then reduced to glyceraldehyde-3-phosphate
(GALP/ PGAL/ G3P/ TP) using NADPH from light-dependent reaction.
(Reject use of acronyms without spelling full names.)
[1 mark each]

[Total: 10]

9648/ 02 / Preliminary Examination / YJC 2013


1794
19

6 Cry toxins are produced by the soil bacterium Bacillus thuringiensis (Bt). By genetic
engineering, Cry toxin gene can be incorporated into commercially important crops to
specifically kill insect pests without affecting other organisms.

Cry toxins are believed to cause death in target insects by oligomerising into pore
complexes in cell surface membrane of midgut epithelial cells.

(a) Explain how formation of pores in the cell surface membrane of midgut
epithelial cells causes death in target insects. [1]

The pores allow ions and water to diffuse freely into the cells causing
changes in membrane permeability leading to cell swelling/ cell lysis and
death of midgut epithelial cells in target insects.

Another mechanism of action of Cry toxin in midgut epithelial cells of target insects has
been proposed. Fig. 6 shows the proposed magnesium ion-dependent signalling
cascade. Single-pass membrane receptors that cross the membrane only once, such as
aminopeptidase N (APN) receptors and cadherin-like receptors, are involved. Cry toxins
eventually lead to destabilisation of the cytoskeleton and ion channels in the cell
membrane causing cell death.

Fig. 6

9648/ 02 / Preliminary Examination / YJC 2013


1795
20

(b) (i) With reference to Fig. 6, describe the proposed cell signalling cascade
of Cry toxin. [4]

When Cry toxin (ligand) binds to the receptor, the receptor is activated and
cause the membrane-bound G-protein (at the cytoplasmic end) to
exchange its GDP with a GTP in the presence of Mg2+;
(Reject Reference to G protein-coupled receptors (GPCR). Receptors of
Cry toxins are not GPCR as they only pass the membrane once unlike
GPCR that are multi-pass membrane receptors which pass membrane
seven times.)
-subunit of G protein, G , dissociates from its - and - subunits and

moves along the membrane to adenylyl cyclase;


Upon binding of G subunit, adenylyl cyclase undergoes conformational

change and catalyses conversion of ATP to cyclic adenosine


monophosphate (cAMP) in the presence of Mg2+;
cAMP acts as a second messenger to activate relay protein, protein
kinase A (PKA) which phosphorylates many downstream effector proteins
and enzymes in a phosphorylation cascade to induce incorporation of
more receptors of Cry toxin at cell surface membrane and to promote cell
death.
[1 mark each]
(ii) State one advantage and one disadvantage of a signalling cascade. [2]

Advantage
amplification of signalling cascade: One signal gives rise to multiple
cellular responses;
Disadvantage
Prone to error: error along any part of the cascade would result in
undesirable consequences;
OR
Non-specificity of response causes wastage of energy/ resources.
[1mark each]

(iii) NF449 is an antagonist of G protein that prevents binding of GTP.


Suggest how NF449 could affect the signalling pathway of Cry toxins. [2]

GDP of G protein is not exchanged for GTP inactivating G protein and


adenylyl cyclase is not activated leading to a low level of cAMP;
PKA is inactivated causing reduced cytotoxicity of Cry toxins/ fewer cell
death.
[1 mark each]

9648/ 02 / Preliminary Examination / YJC 2013


1796
21

(c) Suggest why Cry toxins only kill target insects and not other organisms. [1]

Membrane receptors that bind to Cry toxins are only expressed in the
midgut epithelium cells of target insects and not in that of other organisms;
OR
Conditions of gastrointestinal tract such as pH and ion balance that
facilitate activation of Cry signalling cascade in insects are different in
other organisms.
[1 mark]

[Total: 10]

9648/ 02 / Preliminary Examination / YJC 2013


1797
22

7 Antifreezes are specialised proteins that prevent the organisms that have them from
freezing in the polar oceans of the Arctic and the Antarctic. One of these proteins is the
antifreeze glycoprotein (AFGP), which is produced in some fish species. The AFGP
proteins protect the fish by lowering their bodies freezing point temperature so that it is
lower than that of the surrounding seawater. This prevents ice crystals from forming
within the fishes body tissues when fish in icy waters drink and feed, which would be
lethal.

Fig. 7 shows three species of fish with AFGPs.

Arctic cod (Suborder: Gadoilei).

Lives at the waters surface and at depths


below 900 metres in the Arctic.

Bald notothen (Suborder: Notothenioidei).

Lives on the underside of the surface ice layer in


the Antarctic.

Threadfin pithead (Suborder: Notothenioidei).

Lives in the deep-sea habitats of the Antarctic


(600 metres).

Fig. 7
9648/ 02 / Preliminary Examination / YJC 2013


1798
23

A circumpolar current contributes to the thermal isolation and low temperatures of


present-day Antarctica. This current formed about 30 million years ago when Antarctica
separated from South America. Up until this time, the waters around Antarctica had been
relatively warm and a wide diversity of fish species flourished.

With the separation of Antarctica from South America, water temperatures began to drop,
reaching freezing levels about 1015 million years ago. At this time, it is thought that the
modern day notothenioids were represented by a single species in which the gene for
AFGP may have arisen.

Today the seawater temperature around Antarctica seldom varies more than a few
degrees above freezing. These waters are now dominated by the notothenioids, of which
there are more than 100 species.

AFGP is also found in a species of arctic fish, the Arctic cod. This species is unrelated to
the notothenioids, and its AFGP probably evolved about 45 million years ago during the
glaciation of the Arctic seas. The Arctic cod and the notothenioids separated long before
the antifreeze glycoproteins developed.

Researchers have found that the gene for AFGP in the Antarctic notothenioids is quite
different in its origin and in its location within the genome from the gene expressing
AFGP in the Arctic cod. However, their AFGPs are virtually identical and they both
contain the same repeating sequence of three amino acids (threonine-alanine-alanine).

(a) Explain how natural selection could have brought about the evolution of
antifreeze glycoprotein (AFGP) in the notothenioids in the seawater around
Antarctica. [5]

There is variation in the original / ancestral population; some notothenioids


having AFGPs and others without;
A mutation producing the AFGP allele present in the original / ancestral
species occurred some time prior to / around the time of the cooling of the
waters;
The colder water / decreasing water temperature acted as a selection
pressure on the fish;
Individuals with the AFGP protein were selected for / were at a selective
advantage and are fitter / more likely to survive to sexual maturity and
have greater reproductive success;
to pass on the AFGP allele to their offspring;
The frequency of the AFGP allele increases in the population;
Because of mass extinction of fish which lack the AFGP allele / AFGP
protein, many new / empty niches became available;
Speciation of notothenioids occurs, all possessing the AFGP allele;

[1m each, max 5]

9648/ 02 / Preliminary Examination / YJC 2013


1799
24

(b) (i) What is the term used to describe the evolutionary pattern observed in
the Antarctic notothenioids and the Arctic cod? [1]

Convergent evolution

(ii) Explain how the evolutionary pattern you have identified in (b)(i)
resulted in the evolution of virtually identical AFGPs in the Antarctic
notothenioids and the Arctic cod. [4]

The antifreeze gene evolved independently as each species arose


from different ancestor / unrelated species;
Analogous features / the same short repeating amino acid sequences
found in both Arctic Cod and Antarctic notothenioids results in the
expression of the same protein that carry out the same function;
AFGP genes located in different locations in the genomes of the Arctic
Cod and the Antarctic notothenioids indicate that they evolve
independently;
Similar selection pressures on both Arctic Code and in the Antarctic
notothenioids resulted in the evolution of the same AFGP molecule;

[Total: 10]

9648/ 02 / Preliminary Examination / YJC 2013


1800
25

Section B

Answer one question.

Write your answers on the separate answer paper provided.


Your answers should be illustrated by large, clearly labelled diagrams, where appropriate.
Your answers must be in continuous prose, where applicable.
Your answers must be set out in sections (a), (b) etc., as indicated in the question.

8 (a) Outline the roles of phospholipids, cholesterol and glycolipids in the cell [6]
surface membrane.

Phospholipids

1. Phospholipids are arranged in two parallel layers/a phospholipid bilayer ;


2. Hydrophobic core restricts/prevents movement of ions/large polar
molecules;
3. Conferring selectivity to membrane;

Cholesterol

4. Cholesterol wedged between phospholipid molecules helps to stabilise


the membrane ;
5. Acts like a plug, reducing the escape or entry of small polar molecules
across the membrane;
6. Increases flexibility and stability of membranes;
7. At warm temperatures, cholesterol restrains the movement of
phospholipids / makes the membrane less fluid;
8. At low temperatures, cholesterol hinders the close packing of
phospholipids / makes the membrane more fluid;
9. prevents solidification lowers the temperature required for the membrane
to solidify;
10. prevent cell membranes from breaking up;
[Points 1-10, max 4]

Glycolipids

11. Cell-cell recognition;


12. the carbohydrate chain (oligosaccharides) function as a marker that
distinguish one cell from another;
13. Cell-cell adhesion;
14. For binding cells together into tissues;
15. Act as receptor sites for chemical signals (e.g. hormones);
[max 2]

9648/ 02 / Preliminary Examination / YJC 2013


1801
26

(b) Explain how the molecular structure of haemoglobin is related to its function. [7]

Structure Function

Each subunit is made up of a protein The ability of the haem group to bind
component called globin and a non-protein to oxygen allows oxygen to be
component called haem group. transported in the blood to parts of
The haem group contains an iron ion (Fe2+) the body via the circulatory system.
and a porphyrin ring that binds O2.

Each haem group is able to bind 1


oxygen molecule, there are a total
of 4 haem groups in one
haemoglobin molecule, so 4 oxygen
molecules can be carried at one
time efficient oxygen carrier.

Haemoglobin binds reversibly to


oxygen so that it will dissociate to
release oxygen to the cells that
require oxygen.
The subunits are arranged so that most of its Soluble in water and can be
hydrophilic / polar amino acid residues are transported about in the blood
on the external surface of the globular stream.
structure while its hydrophobic amino acid
residues are buried in the interior of the
globular structure, away from the aqueous How?
surroundings.
Water molecules cluster around their
outward-pointing hydrophilic /
polar side chains and can form
hydrogen bonds with water
molecules.

The haemoglobin molecule has a quaternary The proportion of globin chains are
structure with 4 polypeptide subunits (2 - necessary for the normal function of
chain and 2 -chain subunits) that are held haemoglobin.
together by bonds such as hydrogen bonds,
hydrophobic interactions and ionic bonds.
Polypeptide subunits up of amino acids that Amphoteric - Haemoglobin can act
have amino (-NH3+) and carboxyl (-COO-) as buffer molecules by accepting
groups which allow them to act as both an acid hydrogen ions, enabling large
and a base. quantities of carbonic acid to be
carried to the lungs without any
major alteration in the blood pH
(buffering blood pH).
[1 mark per bulleted point, max 7]

9648/ 02 / Preliminary Examination / YJC 2013


1802
27

(c) Describe the structural differences between collagen and cellulose. [7]

Collagen Cellulose
Collagen is a polymer made up of amino Cellulose is a polymer made up of
acids. -glucose monomers.
Compose of three repeat units of amino Compose of one repeat unit of glucose.
acids (Gly-Pro-Hydroxyproline). (only -glucose)
Monomers of collagen (amino acids) are Monomers of cellulose (-glucose) are
linked via peptide bonds. linked via glycosidic bonds.
Collagen is composed of three helical Cellulose chain is made up of a single
polypeptide chains twisted together to straight and unbranched chain of -
form a triple helix / tropocollagen. glucose polymer.
Inter-chain H-bonding maintains the Inter-chain H-bonding maintains the
stability of within a single triple helix / stability between parallel chains of
tropocollagen. cellulose molecules this cross-
linking by hydrogen bonds form
microfibrils.
H-bonding is between -C=O groups of H-bonding is between OH / hydroxyl
proline residues on one chain and NH groups attached to carbon atom 3 on
groups of glycine residues on an one chain and carbon atom 6 on a -
adjacent polypeptide chain. glucose on an adjacent chain.
Each tropocollagen crosslinks with a No covalent cross-links exist between
neighbouring tropocollagen running parallel the cellulose microfibrils.
to it via covalent bonds formed
between R groups of lysine residues on
adjacent tropocollagen forming collagen
fibrils
[1 mark per point of comparison]

9648/ 02 / Preliminary Examination / YJC 2013


1803
28

9 (a) Outline the ways in which mutations can occur. [6]

1. Factors that promotes rate of mutation exposure to carcinogen,


ultraviolet radiation, X-ray, AVP (any 2);
2. Error occurring during replication/repair;
[max 1]
Gene mutation:

3. Gene mutation is defined as the changes in the nucleotide base


sequence of the genes;
4. Substitution replacement of one nucleotide base-pair with another
base-pair in a gene;
5. results in missense mutation/nonsence mutation/significant change in
the encoded protein/silent mutation/no/little effect on the encoded
protein;
6. Insertion addition of loss of one or more nucleotide base pairs in a
gene;
7. Deletion loss of one or more nucleotide base pair in a gene;
8. results in frameshift mutation, whereby codons are grouped differently;
9. Inversion a sequence of nucleotides becomes separated from the
allele, it rejoins the original position but inverted;

Chromosomal mutation:

10. Chromosomal mutation is defined as the changes in the nucleotide


base sequence of the genes or the amount of chromosome;
11. Structural alteration to chromosomes;
12. Translocation a section of chromosome breaks off and become
attached to another chromosome (leading to a new combination of
alleles) ;
13. Duplication (addition of genes) a section of a chromosome replicates
(so that a set of gene loci is repeated) ;
14. Inversion a chromosome breaks at two locations and (the middle
portion) flips through 180 before rejoining;
15. Deletion (loss of genes) a chromosome breaks at two points, the
middle portion is displaced with the two ends joining together; (A shorter
chromosome containing fewer genes will form).

16. Change in chromosome number;


17. e.g. Aneuploidy addition or loss of one or more chromosomes (in the
nucleus)
or
18. due to non-disjunction during meiosis/ fusion of a normal haploid
gamete with a gamete carrying n-2, n-1, n+1 or n+2 chromosomes;
19. Polyploidy - three or more times the haploid number of chromosomes;
20. due to non-disjunction / the fusion of a diploid gamete with a normal
haploid gamete giving a triploid nucleus;
[Points 3 20: max 5]

9648/ 02 / Preliminary Examination / YJC 2013


1804
29

(b) Explain how gene mutation leads to cystic fibrosis. [7]

1. Cystic fibrosis (CF) is an inherited disease where the mucus glands are
affected.
2. inherited in an autosomal recessive pattern;

3. caused by a mutation of the cystic fibrosis transmembrane conductance


regulator (CFTR) gene;
4. found on chromosome 7;
5. a mutation results in deletion of 3 nucleotides CTT;
6. CFTR protein loses an amino acid phenylalanine;
7. at position 508;

8. CFTR protein is a transmembrane chloride channel that allows efflux of


chloride ions out of the epithelial cells;
9. mutation of the CFTR gene results in a change in the three-dimensional
conformation of the chloride channel;
10. channel is no longer able to transport chloride ions;
11. prevent normal efflux of chloride ions out of epithelial cells;
12. chloride ions accumulate in the epithelial cells, attracting water into the
epithelial cells;
13. causing thick and sticky mucus;
14. cells that line the passageways of the lungs, pancreas, and other organs
produce thick and sticky mucus.
15. mucus obstruct the airways
16. leading to severe problems with breathing/ resulting in bacterial infections
in the lungs/ permanent lung damage/ formation of scar tissue (fibrosis)
and cysts in the lungs / causes progressive damage to the respiratory
system.
17. mucus blocks the duct of the pancreas/ interferes with the function of the
pancreas
18. preventing enzymes and hormones from reaching the intestines to aid
digestion and assimilation, leading to digestive problems;
19. reduces life expectancy;
[max 7]

9648/ 02 / Preliminary Examination / YJC 2013


1805
30

(c) Describe the role of mitosis in maintaining genetic stability. [7]

1. Mitosis maintains genetic stability by ensuring that the two daughter cells
contain exactly the same number of chromosomes as the parent cell.

2. Genetically identical
3. The chromatin of the parental nucleus replicate during S phase of
interphase before mitosis begins.
4. All chromosomes align at the equator during metaphase...
5. ...to ensure that the sister chromatids are shared equally between the two
daughter nuclei after anaphase occurs.
6. The division of the centromere ensures that one sister chromatid of each
chromosome will move to opposite poles.
7. There is no association between homologous pairs of chromosomes /
bivalents do not form, therefore no chiasmata are formed at prophase I.
8. There is thus no crossing over which would have resulted in genetic
variation/recombination of alleles.

[max 7]

9648/ 02 / Preliminary Examination / YJC 2013


1806
2

Answer all questions.

1 The Sunda slow loris (Nycticebus coucang) is a primate native to forests of Indonesia,
western Malaysia, southern Thailand and Singapore. Listed as Vulnerable" on the
International Union for Conservation of Nature (IUCN) Red List, its population is fast
declining due to extensive habitat loss and illegal poaching for pet trade.

In order to understand more about the species before it is extinct, molecular research
work is conducted by a wildlife conservation research facility. A gene of interest from the
species and a bacterial plasmid vector are cleaved using the same restriction enzyme
and ligated before being cloned in a bacterial host model organism for further analysis.
The restriction enzyme used in the experiment generates sticky ends while the bacterial
plasmid used contains marker genes.

(a) Explain how restriction enzymes cut DNA. [1]

(b) Explain what is meant by marker gene. [1]

(c) Suggest why it is possible to obtain two different types of recombinant


plasmids in the experiment. [2]

9648/ 03 / Preliminary Examination / YJC 2013


1807
3

The Sunda slow loris is protected by law under the Convention on International Trade in
Endangered Species of Wild Fauna and Flora (CITES) signed by many countries. It is an
offence to be in possession or to trade in any illegally imported or acquired CITES
species.

Before the Sunda slow loris is sold as pet, its teeth are often removed to prevent injury to
potential buyers. However, this frequently results in infection and death of the animal. In
a recent search of the home of an alleged wildlife trader, no live animal was recovered
except three removed teeth of suspected slow loris species.

DNA from dental pulp extracted from these teeth were analysed based on several
Variable Number Tandem Repeat (VNTR) loci. Fig. 1.1 shows the genetic fingerprints of
the three teeth found, labelled as Teeth 1, Teeth 2 and Teeth 3, and teeth known to be
taken from a Sunda slow loris.

Fig. 1.1

(d) (i) Briefly explain the basis of genetic fingerprinting. [2]

9648/ 03 / Preliminary Examination / YJC 2013


1808
4

(ii) With reference to Fig. 1.1, which tooth is most likely removed from the
Sunda slow loris? Explain your answer. [2]

(iii) Rescued captive Sunda slow lorises that have their teeth removed
could not be reintroduced into the wild, as they are unlikely to survive.
Attempts at breeding Sunda slow lorises in captivity in zoos were
made. However, animals continuously bred in captivity might
experience inbreeding depression.

Using your biological knowledge on restriction fragment length


polymorphism, explain how the banding patterns of a genetic
fingerprint can show whether or not an animal is inbred. [3]

[Total: 11]

9648/ 03 / Preliminary Examination / YJC 2013


1809
5

2 Cystic fibrosis (CF) is one of the most common genetic disorders in the United States. It
is estimated that 30,000 Americans are living with CF and approximately 1,000 new
cases are diagnosed every year. CF is caused by mutations in a gene located on
chromosome 7.

(a) Explain how CF is inherited. [2]

In the early 1980s, attempts were made to map the CF gene on chromosome 7 using
DNA markers.

The relationship between the CF gene and two DNA markers DM1 and DM2 were
studied in a family with many affected children. DNA were extracted from these
individuals and cut with two different restriction enzymes, BamHI and EcoRI, separately.
The products of digestion were separated using gel electrophoresis and the RFLP
banding patterns is shown in Fig. 2.1 below.


Figure 3.1
(Note: BamHI was used on DM1 and EcoRI on DM2)

Fig. 2.1

9648/ 03 / Preliminary Examination / YJC 2013


1810
6

(b) (i) Using the information provided in Fig. 2.1, deduce which marker is
more suitable in detecting the presence of CF. [3]

(ii) Using your understanding of linked genes, provide an explanation for


your answer to (b)(i). [2]

9648/ 03 / Preliminary Examination / YJC 2013


1811
7

(c) An experiment using a non-viral vector to deliver the cystic fibrosis transmembrane
conductance regulator (CFTR) gene was carried out on five individuals with CF.
Three doses of plasmids bearing the CFTR gene were administered to the patients
at four-week intervals. After each dose, tests were carried out on the patients to
detect the following: plasmid-derived CFTR DNA, plasmid-derived CFTR mRNA,
plasmid-derived CFTR protein and CFTR protein function. The results are shown in
Table 2.2, with positive samples marked +.

Table 2.2

Dose 1 Dose 2 Dose 3


Patient
protein protein protein
DNA mRNA protein DNA mRNA protein DNA mRNA protein
function function function

1 + + + + + +

2 + + + +

4 + + + + + +

5 + + + + + +

DNA : detection of plasmid-derived CFTR DNA


mRNA : detection of plasmid-derived mRNA
protein : detection of plasmid-derived CFTR protein
protein function : detection of CFTR protein function

For (c) (i)-(iv), use the data from Table 2.2.

(i) Calculate the percentage success rate of CFTR gene delivery by


liposomes for all 15 doses. [1]

(ii) Calculate the percentage success rate of transcription of plasmid-


derived CFTR DNA for cases where gene delivery was successful. [1]

9648/ 03 / Preliminary Examination / YJC 2013


1812
8

(iii) From your answer in (c)(i) and (c)(ii), determine whether gene delivery
or transcription of the delivered gene was the more significant obstacle
to successful gene therapy. [1]

(iv) Suggest an alternative delivery method that can help address the
obstacle discussed in (c)(iii). [1]

(d) Currently, gene therapy involves the introduction of the normal allele into somatic
cells of individuals suffering from CF.

(i) Explain why there seems to be fewer objections to this method


compared to one where gene therapy is applied to gametic cells. [2]

(ii) Explain why gene therapy provides a possible cure for CF but not stem
cell therapy. [2]

[Total: 15]

9648/ 03 / Preliminary Examination / YJC 2013


1813
9

3 A number of diseases, such as dengue fever, are spread by mosquitoes. The incidence
of this disease has increased dramatically in recent years and this has been linked
with the spread of the mosquito, Aedes aegypti.

In an attempt to reduce the numbers of A. aegypti, genetically modified (GM) male


mosquitoes were produced. One of the genes added to these mosquitoes, when
switched on, results in the production of a protein that is toxic to mosquitoes.

In 2010, in the Cayman Islands and in Malaysia, GM male mosquitoes were released
into the wild to mate with females. All the resulting offspring died in the larval stage.

In Malaysia, both GM male and non-GM male mosquitoes were released in order to
compare their dispersal and life span in the wild. The GM mosquitoes could be identified
because they also carried a gene for green fluorescent protein (GFP).

(a) Explain why, in many examples of gene technology, fluorescent markers are
used in preference to antibiotic resistance genes. [1]

(b) In addition to the gene for GFP, the DNA that has been added to the GM
mosquitoes consists of
a promoter
a gene coding for a toxic protein, tTA
a binding site for tTA.

When a GM mosquito larva hatches from an egg, the promoter induces the
production of only a small amount of tTA, so that the larva does not die
immediately. In a process of positive feedback, the tTA produced binds to the
DNA as shown in Fig. 3.1. This increases the expression of the gene until the
increased concentration of tTA kills the larva.

Fig. 3.1

9648/ 03 / Preliminary Examination / YJC 2013


1814
10

(i) Explain why, in gene technology, a promoter needs to be transferred


along with the desired gene. [2]

(ii) Switching on the gene coding for tTA in the mosquito larvae, rather
than in the eggs, increases the effectiveness of this method of
controlling mosquito numbers.

Explain why this is so. [2]

(c) GM mosquitoes carrying the tTA gene can live and reproduce normally when fed
on a diet containing an added chemical, A.

With reference to Fig. 3.1,

(i) suggest how A could prevent death of the GM mosquitoes. [1]

(ii) suggest why there is little danger of the gene carried by these GM
mosquitoes being passed to other organisms from GM mosquitoes
which escape or are released into the wild. [1]

9648/ 03 / Preliminary Examination / YJC 2013


1815
11

In 1907, scientists found that a common plant tumour was caused by the invasion of a
bacterium called Agrobacterium tumefaciens. They discovered that the tumour was
caused when plasmids from the bacteria were taken into the DNA of the host plants cells
and expressed. By 1983, biotechnologists were able to introduce genes into plant cells
using modified plasmids that do not result in tumour formation. Fig. 3.2 shows the
procedure of transforming Brassica using this method.

Fig. 3.2

9648/ 03 / Preliminary Examination / YJC 2013


1816
12

(d) With reference to Fig. 3.2, explain why carbenicillin and kanamycin were
added in the various media. [3]

(e) Explain why plant tissue culture technique is used to regenerate the
transformed plants. [2]

(f) State the difference in composition between the root initiation medium and the
shoot outgrowth medium. [1]

(g) Suggest why seeds were used instead of explants from adult Brassica plants,
as starting plant material for this procedure. [1]

[Total: 14]

9648/ 03 / Preliminary Examination / YJC 2013


1817
13

4 Planning question

The yield of many cultivated varieties of crop plants such as the onion, Allium cepa, is
reduced by the presence of relatively high concentrations of salts, particularly sodium
chloride, in the soil. There are, however, salt-tolerant varieties of onion that can be grown
successfully in such places. One possible explanation for this is that the fluid in the cell
vacuoles of plants of the tolerant varieties differs in solute potential from that of non-
tolerant varieties.

Plan an investigation to find out whether or not the solute potentials of the cell vacuoles
of epidermal cells of the bulbs of salt-tolerant onions differ from those of their non-
tolerant counterparts.

You are provided with the following equipment. Choose your equipment from this list.
You may not use any additional equipment.
an unlimited supply of 1.0 mol dm3 sodium chloride solution
an unlimited supply of bulbs of two varieties of onion, one salt- tolerant and the
other non-tolerant
unlimited supply of distilled or deionised water
beakers of various sizes
glass specimen tubes
microscope slides and coverslips
volumetric flasks and measuring cylinders of various sizes
graduated pipettes of various sizes, and pipette fillers
dropping pipettes
light microscope with low (100), medium (200) and high (400)
magnification and built-in illumination
wash bottle
marker pen
stopwatch
scalpel
forceps
glass rods for stirring
mounted needles
iodine in potassium iodide solution

Your plan should:


include a clear statement of the hypothesis or prediction
identify the key variables
give full details and explanations of the procedures that you would adopt to
ensure that the results are as precise and reliable as possible
show how you would present and analyse your results
include a brief risk assessment
be written in clear scientific language, using the correct technical and scientific
terms.

[Total: 12]

9648/ 03 / Preliminary Examination / YJC 2013


1818
14

9648/ 03 / Preliminary Examination / YJC 2013


1819
15

9648/ 03 / Preliminary Examination / YJC 2013


1820
16

9648/ 03 / Preliminary Examination / YJC 2013


1821
17

Free-response question

Write your answers on the separate answer paper provided.

Your answers should be illustrated by large, clearly labelled diagrams, where appropriate.
Your answers must be in continuous prose, where applicable.
Your answers must be set out in sections (a), (b) etc., as indicated in the question.

5 (a) Outline how a named protein can be produced by genetic engineering for the [6]
treatment of diabetes mellitus.

(b) Discuss the benefits and associated ethical implications of the human [8]
genome project.

(c) Describe and give examples of unique features of zygotic stem cells, [6]
embryonic stem cells and adult stem cells.

9648/ 03 / Preliminary Examination / YJC 2013


1822
2

Answer all questions.

1 The Sunda slow loris (Nycticebus coucang) is a primate native to forests of Indonesia,
western Malaysia, southern Thailand and Singapore. Listed as Vulnerable" on the
International Union for Conservation of Nature (IUCN) Red List, Its population is fast
declining due to extensive habitat loss and illegal poaching for pet trade.

In order to understand more about the species before it is extinct, molecular research
work is conducted by a wildlife conservation research facility. A gene of interest from the
species and a bacterial plasmid vector are cleaved using the same restriction enzyme
and ligated before being cloned in a bacterial host model organism for further analysis.
The restriction enzyme used in the experiment generates sticky ends while the bacterial
plasmid used contains marker genes.

(a) Explain how restriction enzymes cut DNA. [1]

Each restriction enzyme recognizes and binds to a specific sequence of 4


to 8 nucleotide bases on the DNA molecule called a restriction site to
hydrolyse the phosphodiester bond between two specific nucleotides in
the restriction site.

(b) Explain what is meant by marker gene. [1]

Gene which will give a visible phenotype / traits /characteristic to the


bacteria that is easily detected for identification/selection of bacteria with
vector/plasmid (R. recombinant vector/ plasmid);

(c) Suggest why it is possible to obtain two different types of recombinant


plasmids in the experiment. [2]

The restriction sites recognised by most restriction enzymes are


palindromic/ each strand has the same sequence of bases when read
from 5 to 3 end;
Sticky ends of both ends of the gene of interest are the same if the
fragment is rotated in either orientation/ gene of interest could be inserted
in two different/ either orientation in the plasmid.

9648/ 03 / Preliminary Examination / YJC 2013


1823
3

The Sunda slow loris is protected by law under the Convention on International Trade in
Endangered Species of Wild Fauna and Flora (CITES) signed by many countries. It is an
offence to be in possession or to trade in any illegally imported or acquired CITES
species.

Before the Sunda slow loris is sold as pet, its teeth are often removed to prevent injury to
potential buyers. However, this frequently results in infection and death of the animal. In
a recent search of the home of an alleged wildlife trader, no live animal was recovered
except three removed teeth of suspected slow loris species.

DNA from dental pulp extracted from these teeth were analysed based on several
Variable Number Tandem Repeat (VNTR) loci. Fig. 1.1 shows the genetic fingerprints of
the three teeth found, labelled as Teeth 1, Teeth 2 and Teeth 3, and teeth known to be
taken from a Sunda slow loris.

Fig. 1.1

(d) (i) Briefly explain the basis of genetic fingerprinting. [2]

Different species/ individuals have different VNTR, hence differ in distance


between restriction sites of a particular RE and there are different number
of restriction sites due to restriction fragment length polymorphism (RFLP)
;
The length and number of restriction fragments produced differ when the
two DNA are digested with the same restriction enzyme .

9648/ 03 / Preliminary Examination / YJC 2013


1824
4

(ii) With reference to Fig. 1.1, which tooth is most likely removed from the
Sunda slow loris? Explain your answer. [2]

Teeth 2;

its genetic fingerprint has three restriction fragments of length 2 kb, 3


kb and 6 kb that match restriction fragments from tooth sample of a
known Sunda slow loris;
OR
genetic fingerprints of Teeth 1 and 3 have restriction fragments of 1 kb
and 4 kb that do not match restriction fragments from tooth sample of
a known Sunda slow loris.

Note: Difference at 5 kb restriction fragment between genetic fingerprints of


Teeth 2 and known Sunda slow loris could be attributed to RFLP explained in
(d)(i).

(iii) Rescued captive Sunda slow lorises that have their teeth removed
could not be reintroduced into the wild, as they are unlikely to survive.
Attempts at breeding Sunda slow lorises in captivity in zoos were
made. However, animals continuously bred in captivity might
experience inbreeding depression.

Using your biological knowledge on restriction fragment length


polymorphism, explain how the banding patterns of a genetic
fingerprint can show whether or not an animal is inbred. [3]

Compare banding pattern with other individuals of the same species.


Differences in RFLP markers give bands in different places due to
differences in length of restriction fragment produced;
Common bands indicate related individuals;
If the Sunda slow lorises are inbred, there would be a reduced number
of bands between the Sunda slow lorises compared. More inbred, the
greater the similarities in banding patterns.

[Total: 11]

9648/ 03 / Preliminary Examination / YJC 2013


1825
5

2 Cystic fibrosis (CF) is one of the most common genetic disorders in the United States. It
is estimated that 30,000 Americans are living with CF and approximately 1,000 new
cases are diagnosed every year. CF is caused by mutations in a gene located on
chromosome 7.

(a) Explain how CF is inherited. [2]

autosomal recessive condition;


autosomal males and females are equally affected/ expression of
characteristic not dependent on the sex of the individual;
recessive requires both alleles to be defective before the child is
phenotypically CF; (Reject: individual must be homozygous recessive
because recessive is not explained)
carrier x carrier chance of producing a CF child;
[max 2]

In the early 1980s, attempts were made to map the CF gene on chromosome 7 using
DNA markers.

The relationship between the CF gene and two DNA markers DM1 and DM2 were
studied in a family with many affected children. DNA were extracted from these
individuals and cut with two different restriction enzymes, BamHI and EcoRI, separately.
The products of digestion were separated using gel electrophoresis and the RFLP
banding patterns is shown in Fig. 2.1 below.


Figure 3.1
(Note: BamHI was used on DM1 and EcoRI on DM2)

Fig. 2.1

9648/ 03 / Preliminary Examination / YJC 2013


1826
6

(b) (i) Using the information provided in Fig. 2.1, deduce which marker is
more suitable in detecting the presence of cystic fibrosis? [3]

DM1 more suitable /DM2 less suitable;


(mark awarded only if the student is able to provide some support for the
answer.)

Either
Becos for DM1, 7 out of 8 (88%) of the CF individuals have the only one
5kb band/ homozygous for the 5 kb fragments;
5 out of 8 (62.5%) of the CF individuals have only one 4kb band/
homozygous for the 4 kb fragments for DM2;

Or
For DM1 - All CF individuals have only one 5kb band/ homozygous for
the 5 kb fragments except II-3 who has 2 bands 5kb and 7 kb;
For DM2 - 5 CF individuals have only one 4kb band/ homozygous for
the 4 kb fragments and 4 CF individuals have 2 bands 3kb and 4kb /
about equal number of CF individuals and normal individuals have a
single 4kb band as well as 2 bands 3kb and 4kb;
(So the inheritance of the DM1/5kb fragment is more consistent
compared to DM2)

(ii) Using your understanding of linked genes, provide an explanation for


your answer to (b)(i). [2]

DM1 closer to CF gene compared to DM2;


(note: some form of comparison required between DM1 and DM2 before a
full mark is given.)

Higher tendency for the 5kb fragment to be inherited together with the CF
gene;
Hence lower number of recombinants (individuals II-3 and II-10 compared
to DM2 (II-1,2,,6,7 and 10);
[max 2]

9648/ 03 / Preliminary Examination / YJC 2013


1827
7

(c) An experiment using a non-viral vector to deliver the cystic fibrosis transmembrane
conductance regulator (CFTR) gene was carried out on five individuals with cystic
fibrosis (CF). Three doses of plasmids bearing the CFTR gene were administered
to the patients at four-week intervals. After each dose, tests were carried out on the
patients to detect the following: plasmid-derived CFTR DNA, plasmid-derived CFTR
mRNA, plasmid-derived CFTR protein and CFTR protein function. The results are
shown in Table 2.2, with positive samples marked +.

Table 2.2

Dose 1 Dose 2 Dose 3


Patient
protein protein protein
DNA mRNA protein DNA mRNA protein DNA mRNA protein
function function function

1 + + + + + +

2 + + + +

4 + + + + + +

5 + + + + + +

DNA : detection of plasmid-derived CFTR DNA


mRNA : detection of plasmid-derived mRNA
protein : detection of plasmid-derived CFTR protein
protein function : detection of CFTR protein function

For (c) (i)-(iv), use the data from Table 2.2.

(i) Calculate the percentage success rate of CFTR gene delivery by


liposomes for all 15 doses. [1]

8/15= 53.3%

(ii) Calculate the percentage success rate of transcription of plasmid-


derived CFTR DNA for cases where gene delivery was successful. [1]

7/8= 87.5%

(iii) From your answer in (c)(i) and (c)(ii), determine whether gene delivery
or transcription of the delivered gene was the more significant obstacle
to successful gene therapy. [1]

gene delivery

9648/ 03 / Preliminary Examination / YJC 2013


1828
8

(iv) Suggest an alternative delivery method that can help address the
obstacle discussed in (c)(iii). [1]

Use adenovirus as vector for in vivo inhalation of aerosol spray

(d) Currently, gene therapy involves the introduction of the normal allele into somatic
cells of individuals suffering from CF.

(i) Explain why there seems to be fewer objections to this method


compared to one where gene therapy is applied to gametic cells. [2]

Somatic cell gene therapy: (1 max)


Only the individual receiving the treatment is affected;
The allele that is introduced is not inherited by his offspring;
Individual has consented to the therapy;

Gametic cell gene therapy: (1 max)


Inheritable by the offspring/ future generations;
Such treatment produces generations of unconsenting research
subjects;
Long term effects of gene therapy still unknown;
May tamper with human evolution eg. genes which cause harm in one
environment may be of evolutionary significance in another;

(ii) Explain why gene therapy provides a possible cure for CF but not stem
cell therapy. [2]

Multiple organs affected lungs, pancreas etc. / many different cell types
are involved;
not all organs involved have stem cells that can be used for stem cell
therapy/ difficulty in extracting the stem cells for use; Difficulty in obtaining
stem cells that are able to differentiate into many different types of cells;

[Total: 15]

9648/ 03 / Preliminary Examination / YJC 2013


1829
9

3 A number of diseases, such as dengue fever, are spread by mosquitoes. The incidence
of this disease has increased dramatically in recent years and this has been linked
with the spread of the mosquito, Aedes aegypti.

In an attempt to reduce the numbers of A. aegypti, genetically modified (GM) male


mosquitoes were produced. One of the genes added to these mosquitoes, when
switched on, results in the production of a protein that is toxic to mosquitoes.

In 2010, in the Cayman Islands and in Malaysia, GM male mosquitoes were released
into the wild to mate with females. All the resulting offspring died in the larval stage.

In Malaysia, both GM male and non-GM male mosquitoes were released in order to
compare their dispersal and life span in the wild. The GM mosquitoes could be identified
because they also carried a gene for green fluorescent protein (GFP).

(a) Explain why, in many examples of gene technology, fluorescent markers are
used in preference to antibiotic resistance genes. [1]

easier to, identify/screen for transformed cells;


more economical/time saving/labour saving;
resistance gene(s) can be passed to other bacteria;
idea of antibiotics no longer effective
OR
requiring development of new, antibiotics/treatments;
[max 1]

(b) In addition to the gene for GFP, the DNA that has been added to the GM
mosquitoes consists of
a promoter
a gene coding for a toxic protein, tTA
a binding site for tTA.

When a GM mosquito larva hatches from an egg, the promoter induces the
production of only a small amount of tTA, so that the larva does not die
immediately. In a process of positive feedback, the tTA produced binds to the
DNA as shown in Fig. 3.1. This increases the expression of the gene until the
increased concentration of tTA kills the larva.

Fig. 3.1

9648/ 03 / Preliminary Examination / YJC 2013


1830
10

(i) Explain why, in gene technology, a promoter needs to be transferred


along with the desired gene. [2]

RNA polymerase and general transcription factors bind at the promoter to


initiate transcription / switch on gene expression;
If the promoter is not transferred along with the desired gene, the gene
has to be inserted near an existing promoter, this may disrupt expression
of existing gene;
in eukaryotes precise position of promoter is important for gene
expression;
[max 2]

(ii) Switching on the gene coding for tTA in the mosquito larvae, rather
than in the eggs, increases the effectiveness of this method of
controlling mosquito numbers.

Explain why this is so. [2]

GM larvae do not die immediately thus this gives longer time for tTA
production / tTA to build up;
so tTA can gets into environment (when GM larvae die) and kills non-
GM larvae;

(c) GM mosquitoes carrying the tTA gene can live and reproduce normally when fed
on a diet containing an added chemical, A.

With reference to Fig. 3.1,

(i) suggest how A could prevent death of the GM mosquitoes. [1]

chemical A has, similar shape to tTA/ complementary shape to


DNA binding site, thus chemical A can binds to the binding site,
and prevents tTA from binding;
OR
chemical A, binds to/changes shape of tTA, thus prevents tTA
from binding to DNA binding site, this stops the positive feedback/
small quantity of tTA does not kill;
OR
chemical A, binds to/changes shape of/breaks down of tTA, thus
tTA is no longer toxic;
[Any 1, max 1]

(ii) suggest why there is little danger of the gene carried by these GM
mosquitoes being passed to other organisms from GM mosquitoes
which escape or are released into the wild. [1]

GM males die if they cannot get chemical A;


(if males mate), their offspring die;
GM males only mate with other A. aegypti / their own species;
[Any 1, max 1]
9648/ 03 / Preliminary Examination / YJC 2013


1831
11

In 1907, scientists found that a common plant tumour was caused by the invasion of a
bacterium called Agrobacterium tumefaciens. They discovered that the tumour was
caused when plasmids from the bacteria were taken into the DNA of the host plants cells
and expressed. By 1983, biotechnologists were able to introduce genes into plant cells
using modified plasmids that do not result in tumour formation. Fig. 3.2 shows the
procedure of transforming Brassica using this method.

Fig. 3.2

9648/ 03 / Preliminary Examination / YJC 2013


1832
12

(d) With reference to Fig. 3.2, explain why carbenicillin and kanamycin were
added in the various media. [3]

Carbenicillin and kanamycin are antibiotics, which were added to select for
the growth of transformed Brassica;
OR
Carbenicillin and kanamycin are antibiotics; carbenicillin is added to inhibit
the growth of A. tumefaciens attached to the explants and kanamycin
added subsequently to select for the growth of transformed Brassica;
The genetic markers on the plasmid are therefore genes resistant to
carbenicillin and kanamycin;
Only successfully transformed cells/ plantlets will survive in culture;

(e) Explain why plant tissue culture technique is used to regenerate the
transformed plants. [2]

[Note: Question focuses only on the final stage of plant regeneration]

(Any 2:)
They have the advantage of being free from fungal or bacterial infections;
Quick production of mature plants;
Regeneration of whole plants from genetically modified explants;
Amount of space required for field trials to determine which plants have
desirable traits is reduced;

(f) State the difference in composition between the root initiation medium and the
shoot outgrowth medium. [1]

Root initiation medium: high auxin:cytokinin whereas;


Shoot outgrowth medium: high cytokinin:auxin

(g) Suggest why seeds were used instead of explants from adult Brassica plants,
as starting plant material for this procedure. [1]

Seeds are easier to surface sterilise than adult plant parts/ more effective
surface sterilisation

OR

Adult plants may be diseased, thus lowering the chances of successful


cultures

OR

Any other valid point;

[Total: 14]
9648/ 03 / Preliminary Examination / YJC 2013


1833
13

4 Planning question

Content Expected answer Marks


P P: Planning [max 2]
Defining 1. Hypothesis or prediction
the e.g. tolerant variety has a lower (more negative) solute
problem potential than non-tolerant / incipient plasmolysis brought
about in tolerant variety by higher concentration of external
solution than in case of cells of the non-tolerant variety;

2. Theory to support hypothesis or prediction


water moves by osmosis from, higher to lower / towards the
more negative solute potential therefore when external solute
potential is low (as in a concentrated salt solution) the internal
solute potential must be even lower (as in more concentrated
internal solution)

3. Outline of strategy and justification


e.g. determining the external concentration / solute potential
which results in incipient plasmolysis / 50% plasmolysis in cells
of each variety

Key variables: [max 1]

4. independent variable: concentration of / solute potential


of, external solution;
5. dependent variable: number / percentage, of cells
plasmolysed;
6. controlled variable: immersion time and volume of
immersion solution;
[max 1]
7. Risk Assessment:

must refer to hazard and precaution


e.g. care in attaching pipette filler to pipette / care in use of
scalpel to remove epidermal tissue from onion / dispose of
broken cover slips with care

Acceptable: Some points may be taken from a diagram or


a flow sequence diagram.

9648/ 03 / Preliminary Examination / YJC 2013


1834
14

Content Expected answer Marks


M S: Preparation of solutions [max 2]
Methods
8. prepare range of sodium chloride concentrations using
the1 mol dm3 solution provided ;
9. suggested range of 0 and at least 5 concentrations at
equal intervals, e.g. 0.2, 0.4, 0.6, 0.8, 1.0 mol dm3;
10. how dilution made: e.g. to make 100 cm3 of 0.2 mol dm3
solution, pipette 10 cm3 1 mol dm3 solution into 100 cm3
volumetric flask or measuring cylinder and make up to 100
cm3 with deionised water;
11. evidence of awareness that it is easier to make accurate
dilution using an appropriate sized pipette and reasonably
large total volume;
M O: Obtaining results [max 2]
Methods
12. remove epidermis from inner surface of onion scale with
scalpel;
13. and immerse in constant volume of solution;
14. in labelled specimen tubes;
15. leave for minimum of 15 minutes;
16. ref. to staggering to allow for sampling;
17. (after time period) transfer specimen to microscope slide;
18. in drop of external solution and add coverslip;
19. may add iodine as a coloured liquid make plasmolysed
cells easier to see;
20. (for each specimen) count cells and record % plasmolysed;
21. repeats at each concentration for both varieties of onion;
Analysis of results [max 2]
G: Graph
22. plot graph concentration of external solution (x-axis);
23. against mean % plasmolysis (y-axis);
24. draw line of best fit (probably sigmoid);
25. plot both varieties of onion separately (could be on same
axes);
26. read concentration causing 50% plasmolysis for each
variety;
Analysis of results [max 2]
Biostatistics: Mathematical processing
27. then the difference could be tested for significance using a
biostatistical test / t-test;
28. in making the conclusion the solute potential of the cells of
each variety of onion is equivalent to that of the solution
which causes 50% plasmolysis;
29. because at this point, wall / turgor, pressure is zero
therefore external and internal solute potential are equal;
30. the solute potential of a given concentration of sodium
chloride solution can be found by consulting an appropriate
table;
31. solute potential can be expressed in kPa and is normally a
negative value;
[Total: 12]
9648/ 03 / Preliminary Examination / YJC 2013


1835
15

Free-response question

Write your answers on the separate answer paper provided.

Your answers should be illustrated by large, clearly labelled diagrams, where appropriate.
Your answers must be in continuous prose, where applicable.
Your answers must be set out in sections (a), (b) etc., as indicated in the question.

5 (a) Outline how a named protein can be produced by genetic engineering for the [6]
treatment of diabetes mellitus.

1. Insulin can be produced for the treatment of diabetes mellitus;


2. Based on known amino acid sequences;
3. DNA constructs of A chain and B chain of human insulin were
synthesised;
Reject: Gene of A chain
4. Each gene was placed under the control of a lac promoter;
5. And part of the -galactosidase structural gene;
6. Both recombinant plasmids;
7. Were transformed separately into E.coli;
8. Transcription of lac promoter switched on when E. coli were grown in the
presence of lactose;
9. The 2 artificial genes were expressed independently as fusion proteins;
10. The correct signal for initiation of translation is provided by the region
immediately upstream of the -galactosidase gene;
11. Cyanogen bromide is used to cleave the methionine residue;
12. Separating insulin polypeptides from the -galactosidase fragments;
13. Purified A and B chains are attached to each other via disulphide bonds
to form functional insulin;
[max 6]

9648/ 03 / Preliminary Examination / YJC 2013


1836
16

(b) Discuss the benefits and associated ethical implications of the human [8]
genome project.

Benefits Ethical implications


Pharmacogenomics / knowing about Most effective drug may not be
which genes affect a persons response to a affordable for patient. Such testing
drug / genetic differences affect the way we may not be affordable &
react to a drug. psychological impact of knowing
Possibility of tailoring drugs prescribed to treatment is available but not affordable.
fit patients pharmacogenomic profile for
greater efficacy / avoiding dangerous
side effects.

Risk assessment for certain diseases. Risk profiles for diseases may be
Knowledge of genetic variations can help accessible to insurers, employers,
prevent / treat these diseases. schools etc. / individuals privacy
infringed. May be used in
discriminatory manner/ stigmatization/
social risks.
Psychological impact due to an
individuals genetic differences.
Doctors, other health service providers,
patients and the public must be
educated to make informed choices,
and be aware of scientific capabilities
and limitations.
Standards and quality-control
measures in testing procedures must
be enforced to ensure reliability of
tests. Proper counselling must be done
in conjunction with risk profiling and
where there is no treatment for certain
diseases.
Reproductive issues / decision
making / rights and potentially
controversial procedures e.g.
eugenics, foetal genetic testing and
abortion.

Insights via comparative genomics / Infringement of animal rights. Also,


comparing human genome sequence and animal disease models may not
the genome sequence of model organisms / accurately reflect disease in humans.
identification of genetic similarities and
differences to model organisms.

Leads to gene discovery / follow disease


development / study disease treatment /
elucidating evolutionary history and
relationships between genes, genome and
species.

9648/ 03 / Preliminary Examination / YJC 2013


1837
17

Benefits Ethical implications


Large scale studies on genetically Uncertainties associated with gene
homogeneous populations to identify tests.
alleles linked to certain disease Genetic information of individuals
conditions. becoming private intellectual property
of biotechnology companies.
Commercialization of related
technologies derived from such
research, but possibly no reciprocal
direct benefit for individuals
contributing their genetic information to
the research.

[1mark per bulleted point, max 8]

(c) Describe and give examples of unique features of zygotic stem cells, [6]
embryonic stem cells and adult stem cells.

1. Zygotic stem cells [Example: Fertilised zygote]


o These are the cells in morula, which are formed during the first
few divisions after fertilisation/fusion of an egg and sperm.
o Zygotic stem cells, which include the zygote and early embryo
cells , are totipotent.
o They have the ability to differentiate into any cell type to form
whole organisms and hence are also pluripotent and
multipotent.

Functions :
o Zygotic stem cell has the potential to differentiate into all the
cells that form the complete organism plus the extra-embryonic
tissues which include the placenta and amnion.
o These cells give rise to form all cells in the body of the
organism therefore they have limited ability to self-renew .

2. Embryonic stem cells


o Embryonic stem cells are pluripotent.
o They have the ability to differentiate into almost any cell type to
form any organ or type of cell and so are not totipotent but are
multi-potent.
o They are obtained from the Inner Cell Mass on the fourth day
of development where the embryo forms into two layers, an
outer layer (extra-embryonic membranes) and an inner cell
mass (ICM).

o Functions :
Outer layer forms the placenta .
Inner cell mass (ICM) forms the three primary germ
layers: ectoderm, endoderm and mesoderm, which
later form all the tissues of the human body.
give rise to various organs in organism / multiple

9648/ 03 / Preliminary Examination / YJC 2013


1838
18

specialized cell types that make up the heart, lung, skin


and other tissues in the developing foetus.

3. Adult stem cells [E.g. Blood stem cells, Stromal Stem cells, Gut
Epithelium stem cells]
o Blood stem cells are multipotent. / Found in each tissues or
organs which help to replenish worn-out or damaged cells.
o They have the ability to differentiate into a limited range of cell
type and so are not pluripotent or totipotent .
o Found in bone marrow or blood/skin etc
o Functions:
Responsible for constant renewal of blood.
Give rise to all the blood cell types

[max 2 marks per type of stem cells, total 6 max ]

9648/ 03 / Preliminary Examination / YJC 2013

Vous aimerez peut-être aussi